You are on page 1of 796

,-,

'
'\

"Q~~~lQping Skills for the TOEFL iBT


. . . . . .

Second Edition

I intermediate\

',.

.. '

'

-<' ,

. TOEFL" is a registered trademark of Educational Testing Service (ETS).


This publication is not endorsed or approved by ETS.

'
(

Developing Skills for the TOEFL iBT Second Edition


Paul Edmunds Nancie McKinnon Jeff Zeter
2009 Compass Publishing

All rights reserved. No part of this book may be


reproduced, stored in a retrieval system, or transmitted
in any form or by any means, electronic, mechanical,
photocopying, recording,_ or otherwise, without prior
permission in writing from the publisher.
Authors: Paul Edmunds, Nancie Mckinnon, Jeff Zeter
Acquisitions Editor: Tanya Shawlinski
Content Editor: J.K. Runner
Copy Editor: Moraig Macgillivray
Cover/Interior Design: Design Plus
Email: info@compasspub.com
http://www.compasspub.com

ISBN: 978-1-59966-518-4
10 9 8 7 6 5 4 3 2 1
12 11 10 09

Photo Credits
pp. 191, 193, 195, 604, 620, 624 Shutterstock, Inc.
pp. 604, 636, 652 istock International Inc.
pp. 600, 602, 606, 608, 610, 617, 622, 631, 634, 638, 644 Jupiterlmages Corporation

ll
lj

Paul Edmunds Nancie McKinnon Jeff Zeter


~Compass
" \ Publishing

TOEFL" is a registered trademark of Educational Testing Service (ETS).


This publication is not endorsed or approved by ETS.

Introduction to the TOEFL iBT ---------------------

WR--~---

----------

11

The TOEFL" iBT Reading Section ----------------------------

14

Preview Test -----------

------------------------ 18

Chapter 1 Fact Questions ---------

23

Chapter 2 Negative Fact Questions

37

Chapter 3 Inference Questions ----

51

Chapter 4 Rhetorical Purpose Questions ------------------- 65


Vocabulary Review 1 ------------------------------- 78
Mini Test 1 ---- --------------------------------------------- 80

Chapter 5 Vocabulary Questions


Chapter 6

---------,------------~---

85

Reference Questions ---------~---------------- 99

Chapter 7 Sentence Simplification Questions --------------- --------- 113


Chapter 8 Text Insertion Questions _____ _:__ ________________ 127
l\Jlini Test :;,2

--------- --- ---------

Chapter 9

Prose Summary Questions -- -- -- ---- - - - -- - -

------- --------------------------------

Chapter 10 Table/Chart Questions --- ----------------

142
147

- 161
176

....... ------ --------- --------- - 185


The TOEFL iBT Listening Section Preview Test

188
..... 190

Chapter 1 Main Idea Questions

197

Chapter 2 Detail Questions

209
220

' -

.- '~. - .
j

'

222

Chapter 3

Function Questions ------------------------ 225

Chapter 4

Stance Questions-------

-- 237

Vocabulary Review 2 ------------------------- 248


Mini Test 2 ------------------------------------------ 250
Chapter 5

Organization Questions

------ 253

Chapter 6

Content Questions - - -

Chapter 7

Inference Questions - - - - - - - - - - - - - - - - - - 277

265

Vocabulary Review 3 ---------.-------------------------- 288


Mini Test 3 - - - - - - - - - - - - - - - - - - - - - - - - 290

~ ------------------------------ 293
' The TOEFL iBT Speaking Section---------------------- 296
_, Preview

------------------------------...----------:..---- 299

Part 1 Thinking and Speaking --------------------- 309


Part 2 Making Speech Coherent --------------------------- ----- 363
Part 3 Speaking Naturally-------------------------------- 419

mRI ------------------------------------------------------'-!__ ___________ .429The TOEFL" iBT Writing


Preview

Section------------------------~.:. __

432

-------------------------------------------- 437

Part 1 Integrated Writing

---~------------------

----------- 445

Part 2 Independent Writing -------------------------- - --- ---- ------- 501


Part 3 Writing and Grammar --------- -

--------- ------- ----------- 557

Practice Test Reading, Listening, Speaking, Writing --

----- ------ ---- 575

Transcripts -------- ----- ------ ----------------------- -- - ----- --


Answer Key

657
727

What to Expect on the TOEFL


The TOEFL (Test of English as a Foreign Language) is an Internet-based test designed to assess
English proficiency in non-native speakers who want to achieve academic success and communicate
effectively in English. Most people take the TOEFL iBT to gain admission into universities and
colleges where instruction is in English. Additionally, many employers and government agencies
use the scores to determine a person's English ability. It is not meant to test academic knowledge or
computer ability, and as such, questions are always based on information found in the tesi (computer
tutorials are available for those not familiar with personal computers). We have designed this practice
book to be as similar as possible to the actual TOEFL iBT in format and appearance in order to better
prepare you for the test.
The TOEFL iBT is divided into four sections: reading, listening, speaking, and writing.

Major Changes to the Internet-Based TOEFL (i!ST)


General
o The test measures all four language skills equally; a speaking section is included.
o The Test of Spoken English (TSE) will now be part of the TOEFL. Test takers will no longer
take the TSE as a separate test.
o Order of sections on the test:
Reading
Listening
(10-minute break)
Speaking
Writing
o The test is approximately four hours long and is taken in one day.
o Tests are administered through the Internet in Educational Testing Service (ETS) test centers
around the world.
o There is no structure section, as there was in previous tests.
c> Note-taking is allowed in every section, and is not marked.
o The test is a linear exam, not computer adaptive; each test taker receives the same range of
questions.
o The scores will be viewed online.
Reading I Listening
co Passages for the reading and listening sections are longer than those in the computer-based
test (CBT). Refer to the introduction of individual sections for further details.

6 Introduction to the

TOEFL~ iBT

Speaking I Writing
o Tasks for the speaking and writing sections include integrated questions that require more
than one skill to complete, i.e., reading and/or listening, then speaking or writing.
o For the speaking section, test takers speak into a microphone, and their responses are
digitized and sent to the ETS Online Scoring Network.
o For the writing section, test takers must type their responses.

The TOEFL iBT Format


Section

Number of Questions

Time (minutes)

Score

Reading

3-5 passages
12-14 questions each
700 words per passage

60-100

30 points

Listening

4-6

2-3

60-90

30 points

':'

lectures
6 questions each
500-800 words (4-6 min.)
conversations
5 questions each
400-500 words (2-3 min.)

BREAK
Speaking

10
2independenttasks
1 personal experience
1 preference/choice
2 integrated tasks (Read-Listen-Speak)
Reading 100 words
Conversation 200 words (1-2 min.) '
Lecture 200-300 words (1-2 min.)
2 integrated tasks (Listen-Speak)
Conversation 200 words (1-2 min.)
Lecture 200-300 words (1-2 min.)

20

30 points

50

30 points

Writing

1 independent task
1 integrated task (Read-Listen-Write)
- Reading 250-300 words
- Lecture 250-300 words (2 min.)

Introduction to the TOEFL" iBT

1 -

i'

11

Study Tips

11

1i

i1,,!

The only way to be certain of an excellent TOEFL score is to be able to read, write, understand,
and speak English like an educated native speaker. You have no doubt been developing your
ability in these areas for many years now. Unfortunately, this is not something one can accomplish
by studying in the traditional way. However, research conducted over the years by applied linguists,
psychologists, and educators has yielded a considerable amount of information on the best
methods for refining these skills for the purposes of standardized tests. By keeping the following
study tips in mind, you can optimize your study habits and achieve the highest possible scores with
the level of language proficiency you have obtained.
Prepare a study area tor yourself. This should include the following:
c> A comfortable chair and spacious table or desk
'"' Suitable lighting
c> Good ventilation and air quality - an open window or a house plant are good ideas
c> An area free of distractions such as outside noise, television, or radio (unless you are using
the television or radio to study)
co Proper space to keep all the materials you will need
when studying, such as books, paper,
,
pens, pencils, a tape recorder or other recording device, and if possible, a computer with
Internet access
Study regularly over a long period of time. Do not study to the point of exhaustion, as this has been
shown to be ineffective in retaining information.
"Cramming," i.e., studying intensely for a day or two before an exam, is not effective, as it strains
your general health and well-being and does not lead to good long-term retention of information or
skills.
Psychologists have discovered a principle called "state-specific memory." This means you
remember things better in the same conditions that you learned them. For example, it you always
study math at night, you will do better on a math exam taken at night. Use this concept to your
advantage. If you know when and under what conditions you will take the TOEFL, simulate these
in your study environment and habits. For instance, if you plan to take the TOEFL on a Saturday
afternoon, then make a point to study in the afternoons.

Introduction to the TOEFL" iBT

Be well rested on the day of the exam. Do not stay up all night studying. Also, eat healthy foods
including fruits and vegetables.
Be relaxed and confident. Do the best that you can and do not worry excessively about any
mistakes or uncertainties.

Registering For the TOEFL


Students must get registration information for the TOEFL. Registration information can be obtained
online at the ETS website. The Internet address is www.ets.org/toefl. The website provides information
such as testing locations, costs, and identification requirements. The website also provides other
test-preparation material.
The registration information, such as the test center location, identification requirements, and costs,
will vary depending on the country in which you take the test. Be sure to follow these requirements
carefully. If you do not have the proper requirements in order, you may not be able to take the test.
Remember that if you register online, you will need to have your credit card information ready.

_\

What TOEFL Score Are Used For


The primary use of TOEFL scores is for acceptance into universities, colleges, and other institutions
where English is the main language of instruction. It is estimated that about 4,400 such institutions
require TOEFL scores for admission.
Th.e highest possible score on the iBT is 120 points. Different institutions will have their own specific
score requirements for admission. For that reason, it is very important to check with each institution
individually to find out what its admission requirements are. For example, a passing score at one
university may not be a passing score at another university. It is the responsibility of the student to
find out what the requirements are for each institution.
Although TOEFL scores are used primarily to satisfy the admissions requirements of universities, they
are also necessary when applying for certain kinds of jobs. Many government agencies as well as
multinational corporations require applicants to submit TOEFL'" scores. Even English-teaching institutes
may request TOEFL scores in order to place students at the appropriate level of instruction .
. Certainly, doing well on the TOEFL can be very helpful for students in both their academic and
professional careers. However, success requires consistent and dedicated practice. We hope that
you will take full advantage of this practice book and study hard. Your hard work and dedication will
provide you with the best opportunity to do well on the TOEFL and rneet your goals for the future.

Introduction to the TOEFL~ iBT

9 --

Academic Subjects in the TOEFL


The following is a list of academic subject areas typically seen in the TOEFL:
Humanities

Social Sciences

Biological Sciences

Physical Sciences

Archaeology
Architecture
Art History
Fine Arts
Linguistics .
Literature
Music
Philosophy

Anthropology
Business
Economics
Education
Geography
History
Political Science
Psychology
Public Health
Sociology
Urban Studies

Agriculture
Anatomy
Biology
Botany
Entomology
Environmental Science
Medicine
Zoology

Astronomy
Chemistry
Computer Science
Engineering
Geology
Mathematics
Oceanography
Physics

L.

;.
i

111--- 10 Introduction to the TOEFL" iBT

-,

The TOEFL" iBT Reading Section-------------------------- 14


Preview Test --------------------------------------------------- 18
Chapter 1 Fact Questions

---------- 23

Chapter 2 Negative Fact Questions ---------------------------- 37


Chapter 3 Inference Questions ------------------------ 51
Chapter 4 Rhetorical Purpose Questions ----------------- 65
Vocabulc1ry Review 1 -----------------------------

78

IVlini Ticst 1 ------------------------------------------------- 80

Chapter 5 Vocabulary Questions ---------------------------------- 85


Chapter 6 Reference Questions ------------ ---------------------- 99
Chapter 7 Sentence Simplification Questions

--------------- 113

Chapter 8 Text Insertion Questions------------------------- 127


Vocabulary Review 2 ---------------- -------------------------------- 140
iVlini

rest ;2

------------------------------ 142

----------- --- - - - ---------

Chapter 9 Prose Summary Questions


Chapter 10 Table/Chart Questions

147

161

174
-- 176

Answer Key Reading

-- - ---- . 728

-..:

In the reading section o\the TOEFL iBT, you will be required to read three to five passages on varying
topics. After each passage, you will answer twelve to fourteen questions that test your ability to
understand vocabulary, sentence structure, and factual information, as well as implied information
and the writer's intention. You will not see the questions until after you have read the passage. While
answering the questions, you will be permitted to look back at the reading. You do not need any
previous knowledge on the topic in order to answer the questions correctly.

Passage Types
1. Exposition - Material that provides information about or an explanation of a topic
2. Argumentation - Material that presents a point of view about a topic and provides supporting
evidence in favor of a position
3. Narrative - An account of a person's life or a historical event
Question Types
Questions in the reading section of the TOEFL iBT are multiple choice and much like those
found in older versions of the TOEFL. The following list explains the question types and number
of each type on the test. Questions will not necessarily appear in this order.
Question Type

Number

Task

Factual Information

3-6

Selectdetails or facts provided in the passage.

Negative Factual
Information

0-2

Identify something that is not in the passage or not true


according to the passage.

Inference

0-2

Select an answer based on information not actually stated


in thG passage, but that is implied or can be inferred.

Rhetorical Purpose

0-2

Identify the author's method in explaining a point, or why


the author has mentioned something.

Vocabulary

3-5

Choose the best synonym.

Pronoun Reference

0-2

Identify the noun to which a pronoun refers.

Sentence
Simplification

0-1

Choose the best paraphrase of part of the passage or a


sentence and analyze its meaning.

The other three question types found in the TOEFL iBT are not multiple-choice and are types not _
found on older versions of the TOEFL..One is a sentence insertion activity, and the last question
can be one of two types of activities: either a summary question or a table/chart question.

I
c

Text Insertion Question


This question shows you a sentence that could be added to the passage. You must decide where
the sentence would best fit in the passage. While y0u are reading, you will notice several icons that
look like this on the actual Internet-based test. You will be required to click on the square []
where you feel the new sentence should be added. For the purposes of this book, simply choose the
letter beside the appropriate square. This question tests how well you understand the organization
of the passage, as well as grammatical connections between sentences.

i!!!
z

G'l

'

VI
"'O

~z

G'l

Table/Chart Question
For passages on topics that explain groups or categories of information, you may be asked to
demonstrate your understanding of the groups or categories mentioned by completing a chart.
There will be two or three categories and either five or seven correct choices. Two of the answer
choices will not be used. This question requires the ability to organize important information and to
understand relationships between major ideas presented in the passage.
'Example:
'

Frogs

Toads

Correct answers:
Frogs

Toads

eggs in a bunch

bumpy, dry skin

longer legs

eggs in a chain

smooth, wet skin

shorter legs

(A) bumpy, dry skin


(B) eggs in a chain
(C) build nests
(D) shorter legs
(E) eggs in a bunch
(F) have live babies
(G) longer legs
(H) smooth, wet skin
(I) bulging eyes

bulging eyes

Not used: build nests, have live babies


The chart questions are worth up to three points if there are five correct answers and four points if there
are seven correct answers. Partial credit is awarded for having some but not all correct answers.

The TOEFL 0 iBT Reading Section

15 - -

ii~

I,
il-1;

Ii';

'I

Summary Question
In this type of question, you will be presented with an introductory sentence for a brief summary of
the passage. You will then find six additional sentences. Three of the sentences express major
ideas in the passage, and the others do not. Incorrect choices will be either minor ideas or ideas
that are not presented in the passage. This question measures your ability to recognize important
ideas from the passage and distinguish them from minor ideas or ideas that are not in the passage.
Example: .
First sentence of introduction:
Animals in the desert have different ways to live with little water.
. o Camels can live for a long time without water.
o Desert plants do not need much water.
o Desert reptiles and birds do not sweat.
o Larger animals get the water they need from things they eat.
o At night, desert temperatures can drop below ten degrees Celsius.
w Some animals stay underground to keep water in their skin.
Correct answers:
First sentence of introduction:
Animals in the desert have different ways to live with little water.
o Desert reptiles and birds do not sweat.
o Larger animals get the water they need from things they eat.
o Some animals stay underground to keep water in their skin.
Not used:
c.> Camels can live for a long time without water. (minor detail)
o Desert plants do not need much water. (incorrect information)
c_, At night, desert temperatures can drop below ten degrees Celsius. (minor detail)

The summary question is worth up to 2 points. Partial credit is awarded for having some but not all
correct answers.

Practice reading passages of academic English regularly (the Internet can be a great source of
practice materials).
Become a master of vocabulary and constructions.:
"' Make it your goal to understand all the words you come across when studying.

16

The TOEFL~ iBT Heading Section

Keep a vocabulary notebook listing new terms and their definitions. Write out the definitions in
English. Only refer to bilingual dictionaries if you cannot understand a word used in context or
its definition from an English-only dictionary. Set aside a period of time every week to review your
new vocabulary. Practice it by writing out your own sentences using the words.
Master any and all grammatical and rhetorical constructions you encounter. Discover their
meanings and uses by asking a teacher or doing an Internet search and viewing multiple
examples of their use. You can keep a notebook of constructions as well.

Learn how to take notes. You are permitted to take notes during the reading section of the
TOEFL. Note-taking is not writing down every word of the reading. A good idea is to note the
main idea and then note the information that supports it. Note-taking must be learned, and it
takes time. The better your note-taking skills, the easier you should find the TOEFL'" reading
section, as well as other sections of the TOEFL iBT.
Do not use a pencil or your finger when you are reading. Your eyes move faster than your finger,
so you slow yourself down if you trace lines with a pencil or finger while reading.

Test Management
Questions cannot be viewed until after the passage has been read.
You will be allowed to study the reading as you answer the questions.
Use the Review icon at the top of the screen to return to previous questions.
There is a glossary available. Simply select the particular word with the cursor to find ifs meaning.
When reading passages, ask yourself the following questions:
=.) What is the main idea of the passage?
=.) How is the main idea developed/supported in the passage?
For each paragraph or new point in the passage, ask yourself why the author mentions it and
how it relates to the main idea.
Keep in mind that you have 60 to 100 minutes to read the passages and answer all of the questions
in the reading section. This means that you can spend roughly 20 minutes on each set. Try to
pace yourself accordingly. For each set, first answer the questions that you can answer easily.
Then go back and answer more difficult questions. If you find that you have exceeded 20 minutes
for a particular section, it is best to guess an answer and move on to the next section rather than
remain on a particularly difficult question for several minutes

The TOEFL" iBT Reading Se.ction

,.

17 - -

You will read one passage and then answer reading comprehension questions about it. Most
questions are worth one point, but the last question is worth more than one point. The directions
indicate how many points you may receive.
You will have 20 minutes to read the passage and answer the questions.
You may see a word or phrase in the passage that is underlined in blue. Definitions or
explanations for these words or phrases are provided at the end of the passage.
You can skip questions and return to them later as long as there is time remaining.
When you are ready, press Continue.

-~~
:'i

18

PreviewTest

I
The ocean is an ecosystem that offers a bounty that humans have found invaluable for
years. Thriving on the ocean floor is a form of seaweed, or alga, known as kelp. Kelp is a
cold-water species, living primarily in the temperate and Arctic waters of the Northern
Hemisphere. Kelp requires clear, shallow waters, since it relies on light for photosynthesis. In
cold, nutrient-rich waters, the alga grows in underwater masses known as kelp forests, which offer
a habitat for some ocean creatures and a food source for others. In the marine environment,
they rank among the most biologically productive territories. This particular seaweed has a
variety of purposes and has been utilized for centuries in the industrial and household realms.
It plays a vital role in the balance of the ecosystem, affecting ocean life and human existence
as well.
The appearance of kelp resembles that of a plant, but it is not classified as such. Kelp
belongs to the Protista kingdom. Members of the Protista kingdom resemble fungi, plants, or
animals, but share a relatively simple organization, being either unicellular or multi-cellular with
unspecialized tissues. Kelp is similar to a plant, but has a simple, multi-cellular development.
The structure of the alga consists of the holdfast, the stalk or stipe, and the fronds made up
oflong, leaf-like blades. The holdfast roots the seaweed to the ocean floor. Unlike plant roots
that grow into the g'round and gain nutrients from the soil, the holdfast attaches to the top of
a rocky surface, anchoring the seaweed in place. Between the stipe and each blade is a gas
bladder that supports the frond and keeps it afloat.
These unusual characteristics of kelp make it a valuable commodity to harvest. Humans
gather the long, flat, broad leaves of the seaweed to use for production. In the past, harvesters
would encircle a stand of kelp with a cable and pull on the cable to dislodge the alga from the
rocky surface of the ocean floor. This method removed the whole plant, including the holdfast.
Modern procedures only cut off the fronds of the plant, leaving the long-living holdfast intact
and able to continue growing. The process involves barges equipped with blades that float
on the ocean's surface into kelp forests. Their purpose is to gather the top of the alga without
damaging the alga itself.
Harvested kelp, once it is processed, has multiple roles in the manufacturing field. Burning
kelp is a common technique to process the alga once it is removed from its environment. and
the product offers manifold functions. This kelp ash, or soda ash, is primarily sodium carbonate,
and it is rich in iodine and alkali. Businesses worldwide reap the benefits of utilizing seaweed.
Uses for iodine span the industrial spectrum, including pharmaceuticals, antiseptics, catalysts,
food supplements, water purifiers, halogen lights, and photography. An alkali is a basic, ionic ,
salt that dissolves in water and comes from alkaline earth metals. Components of gunpowder
and lye soap are two of the numerous purposes for the alkali found in kelp.
I

l-----------.. . .-.. . . . . . . . _.... --------"'"""''""_. . . . . . . . . . -------.. . . . ------------....1

Preview Test

19 - -

'

''.ii
'

''

Kelp's qualities extend beyond the inherent iodine and alkali elements. A kelp derivative
called alginate is a carbohydrate used as a thickening substance in ice cream, jelly, toothpaste,
and salad dressing. Another derivative of kelp is applied as an emulsifier to create a smooth
texture in products such as processed foods, pharmaceuticals, paints, and cosmetics. In
Japanese cuisine, an essential ingredient is kombu, one of several Pacific species of kelp. It
is used as a garnish, a vegetable, and a flavoring for broth or porridge.
A) Scotland, Norway, China, and the US are now the p'rimary producers of alga products,
with smaller amounts contributed by Japan, Chile, France, and Spain. B) Centuries ago,
Scotland was the leading source of kelp ash production until the introduction of newer
processes, beginning around the 1800s. C) Production continues today. D) However,
despite kelp's natural abundance, a great deal of responsibility is required. Studies have
shown that the number of kelp forests has decreased over the past several decades. After
gathering kelp for centuries, harvesters must notice the effects of their labors on ocean life.
Vital to human manufacturing as kelp has become, its original role in the environment is to
provide essential food and habitat for ocean organi.sms as well as to keep the balance of the
marine ecosystem it inhabits.
~stand

a group of several plants growing together in one place


~barge

a long, flat boat


~

emulsifier

. a substance added to food to prevent liquid and solid parts from separating

1. According to paragraph 1, which of the

following characteristics is unique to a


kelp forest?
(A) It grows in all types of ocean water.
(B) It requires deep water to reach its
.growth potential.
(C) It needs water with nutrients for food.
(D) It grows in clear waters within the
reach of light.

\: : - - 20

Preview Test

2. Which of the following can be inferred from


paragraph 1 about kelp?
(A) Kelp forests do not support much
marine life.
(B) Kelp does not grow in warm
equatorial waters.
(C) Kelp requires a lot of prey to survive.
(D) The waters of Antarctica provide a
suitable habitat for kelp.

3. According to paragraph 2, what is true


about the individual structure of kelp?
(A) The holdfast makes up the leafy,
larger portion of the organism.
(B) The gas bladders are essential in
keeping kelp upright.
(C) The root is secured on the ocean
floor, allowing the fronds to float
freely.
(D) The stipe serves as the main support
for kelp's structure.
4. The word encircle in the passage is
closest in meaning to

,,
,

(A)
(B)
(C)
i' (D)

surround
hide
capture
extract

5.,, Which of the following can be inferred


from paragraph 3 about harvesting kelp?
(A) Kelp can survive if the holdfast is not
destroyed.
(B) Modern harvesting procedures
destroy lots of kelp.
(C) Harvesters used to gather the
holdfasts for use in production.
(D) Harvesting kelp is a new process
that is still changing.
6. The author discusses harvesting
techniques in paragraph 3 in order to
(A) offer instructions on how to gather
kelp
(B) show improvements in gathering
methods
(C) describe the nature of kelp that is
gathered
(D) introduce the process of involving
barges

7. The word Their in the passage refers to


(A) Algae
(B) Procedures
(C) Barges
(D) Rocky surfaces
8. The word manifold in the passage is
closest in meaning to
(A) extreme
(B) limited
(C) multiple
(D) specific
9. All of the following are mentioned in
paragraph 4 as uses of kelp-derived
iodine EXCEPT:
(A)
(B)
(C)
(D)

Creating weapons
Purifying water
Cleaning wounds
Improving food

1O. The word derivative in the passage is


closest in meaning to
(A) impact
(B) replica
(C) product
(D) additive

11. According to paragraph 5, alginate


serves as
(A) a substance used to create paint
(B) a popular garnish for certain foods
(C) an element used in manufacturing
kelp
(D) a thickener for gel-like substances

Preview Test

..

...

21 - -

12. Look at the four squares [l that indicate


where the following sentence could be
added to the passage.

These methods, coupled with the


economic collapse of the Scottish kelp
industry, paved the way for other
countries to begin producing kelp on
a large-scale.
Where would the sentence best fit?
(A) First square
(El} Second square
(C) Third square
(D) Fourth square

13. Which of the sentences below best


expresses the essential information in the
highlighted sentence? Incorrect choices
change the meaning in important ways or
leave out essential information.
(A) Kelp plays a major role in the marine
environment and in industry.
(B) Kelp has grown increasingly
important in manufacturing.
(C) Kelp keeps its ecosystem and the
surrounding marine life in balance.
(0) Kelp is best left in its native
environment because of overharvesting.

14. Directions: An introductory sentence for


a brief summary of the passage is
provided below. Complete the summary
by selecting the THREE answer choices
that express the most important ideas in
the passage. Some answer choices do
not belong in the summary because they
express ideas that are not presented in
the passage or are minor ideas in the
passage. This question is worth 2
points.
Ocean kelp has an important place
in the ecosystem and offers many
benefits to humans.

Answer Choices
(A) Kelp is vital to marine life because it
offers homes and food to sea
dwellers.
(B) Soda ash, which is primarily sodium
carbonate, yields iodine and alkali.
(C) Kelp products are used in a number
of manufacturing processes and
goods.
(D) Groups of kelp exist as kelp forests
and require cool, shallow water to
survive.
(E) Raw or cooked kelp can be used as
either a garnish or a vegetable.
(F) Kelp harvesting has continued for
centuries as humans realized its
many uses.

~ l 22
F

Preview Test

'

:_;

Necessary Skills
o.1m1~,.%l'lli~WMilitf'Xl"-'t.l_ill.B~~f..~1S',1-;,'.!:c,;,i:w,_<,,_L'<;,,: ~-,,,:u--,

-,,

<.~--,-,

Identifying important information and facts stated in a passage


Locating a specific piece of information in a passage quickly
Understanding the distinction between main ideas and supporting details
Focusing on facts, details, definitions, or other information presented in a passage

Example Questions

According to the passage, who/when/where/what/how/why _ _ _ _ _?


According to paragraph X, which of the following is true of
?
The author's description of
mentions which of the following?
According to paragraph X,
occurred because _ _ _ __
According to the passage, why did X do Y?

,,
'

''

Strategies
Look for transitional expressions to locate details such as examples, steps, time,
reasons, or results.
Pay attention to examples and descriptions that provide information and details.
Eliminate choices presenting information that contradicts what is provided in the
passage.
Answer the specific question being asked. Do not select an answer just because
it is mentioned in the passage.

-1
I

'

Fact Questions

',

., .

'

23 - -

linguistics
-~-~m1;t,.'l?IL'i~.:1<~;>."'1t-:~,~,..,;~;0

.>o<>Y.V7"-"-'1::>r',:;-

-.- ,, ... ::,.-~.

' information that you read.


Read the following passage. Then fill in the diagram with th~

The discovery of gold in California in 1848 marked the beginning


of an economic boom known as the California Gold Rush. The miners,
many of whom were originally journalists, lawyers, and businessmen,
preserved their experiences in writing. Much of this was peppered
with colorful phrases related to their new work. Interestingly, many of
these phrases are still in use today in the English language.
The popular phrase "pan out," meaning "to be successful," can
be traced back to the gold rush. Miners would separate gold from
worthless minerals using a pan; gold would settle to the bottom of the
pan, and other sediments would be removed. The likely origin of "pan
out," then, is from the process of gathering all the gold in one's pan.
In addition, the phrase "strike it rich" originated from the Gold
Rush. This phrase has come to mean "to become wealthy quickly." A
strike is literally a discovery, in this case, of gold. During the Gold
Rush, if a miner "struck it rich," he found gold and thus became
wealthy. Having lost its original meaning, the same phrase is used
today when people make money quickly in business or win a lottery.

@t

mark:
to be a defining feature of

boom:
a rapid increase
pepper:
to fill with lots of examples
of something
trace:
to follow or be followed
settle:
to come to rest
sediment:
material that settles to the
bottom of water

-----Theory:

Example 1:

1<
I

I
---------------------!

~--

24 Chapter 1

i------------------------1 Example 2:

1. Which of the following is true according to


'
paragraph 1?

(A) Introducing new phrases into English


was the goal of the writers.
(B) Not all of the Gold Rush miners could
read or write.
(C) The miners wrote about their
gold-mining experiences.
(D) Miners preferred their lives as
journalists, lawyers, and businessmen.

3. According to paragraph 3, what does


"strike it rich" mean today?
(A) To find gold
(B) To break a rock apart
(C) To lose one's wealth
(D) To get money fast

2. According to paragraph 2, where did the


phrase "pan out" originate?
(A) Miners called their findings "pans."
(B) Miners used pans to find their gold.
(C) Gold was found in places called
pans.
(D) Miners called selling gold "panning."

Fill)n the blanks to complete the summary.

The passage discusses English phrases that can be _ _ _ _ _ back to the gold
rush in California. The phrase "pan out" came from the pans that miners used. Gold would
_ _ _ _ _ to the bottom of their pans, and the _ _ _ _ _ would be removed.
The expression ' ' . - - - - - it rich" originated from miners striking the rocks to find gold
and becoming

quickly if they found it.

Fact Questions

25 - -

Read the following passage. Then fill in the diagram with t~e information that you read. 'I'

In Greek theater, the chorus was either a person or a group of


people that served various purposes in supporting the play. It provided
a commentary on the play by emphasizing the major themes and by
clarifying the plot. The chorus also influenced reactions of the audience
by engaging in overt communication with it.
To help the audience understand the events, the chorus helped to
reinforce the main points of the play. It often revealed background
information that the characters could not deliver-such as inner
emotions like fear and resentment-without needlessly interrupting the
play. Through either singing or speaking in unison, the chorus conveyed
ideas that characters were unable to say. Also, an overview of events
was sometimes presented to the audience members to facilitate their
interpretation of the story. This would improve understanding and
enhance enjoyment of the play.
Finally, the chorus could sway audience responses by displaying
embellished reactions to prompt the audience .to react similarly.
Likewise, laugh tracks today can prompt television viewers to laugh.
Another way the chorus interacted with the audience was by
discussing events and characters. This was achieved by either
mocking or sympathizing with them. This clever tool to influence
feelings and reactions of the audience altered the dynamic between
the audience and the play itself.

clarify:
to free from confusion
commentary:
an explanation or
illustration of something
overt:
explicit or easily seen or
recognized
reinforce:
to emphasize
unison:
in harmony; at the same
time
facllltate:
to make easier
embellish:
to exaggerate
laugh track:
a prerecorded segment of
laughter that is added to a
TV or radio program to
mimic audience responses

r;:-------------------------------------Definition:

Role 1:

I
I

<
i.____________.. -----------------_J

- - 26

Chapter 1

[_ ______________________ ___________ ..................... - ........ ,

'

------- - ~ ~--~ ---- - -"----.,----------~u___,__J

1. According to paragraph 1, what was the


role of the chorus in Greek theater?
(A) To introduce new characters
(B) To provide music tor the play
(C) To analyze the audience's reactions
(D) To help the audience understand

3. According to paragraph 3, why did the


chorus present overstated emotions?
(A) To parody the play's characters
(B) To encourage audience reactions
(C) To explain important events
(D) To provide new information

~.
~I

2. According to paragraph 2, how did the


chorus communicate characters'
emotions?
(A) By telling the audience how to react
(B) By summarizing the play alter it
ended
(Cj By presenting the thoughts of the
characters
(D) By repeating their thoughts during the
play

Fill iJtheblanks to complete the summary.

The role of the chorus in Greek theater was to provide a(n) - - - - - - - o n a


play to the audience in order to _ _ _ _ _ the plot. The chorus explained the stories
and _ _ _ _ _ _ _ the audience's understanding of events. By singing or speaking in
_ _ _ _ _ , t.he chorus was also responsible for influencing the way audiences reacted
to a play. It did this by showing

reactions and discussing the play's

events and characters with the audience.

Fact Questions

__ ', -------

27 - - .

Ecology.
'll.\'l.'l'l~~:i'l'.r
..>'IN11~'L'&l:&f.'i!'Hld'U'1";('.dY,.

-,,,.,,,.,,._'I.C-.-.- 'h\'._'.,

.. c ,,,-.

Read the following passage. Then fill in the diagram with the information that you read. +@II

I.:.'I
.i

Global warming is contributing to the shift in weather patterns


across the globe, causing higher air temperatures. In turn, the warmer
air alters the temperatures of the oceans and causes significant
changes to the world's ecosystems. In particular, coral reefs, considered
one of the most vulnerable ecosystems with regard to temperature
changes, could be destroyed forever.
Coral is a brightly colored sea animal that can only thrive in a limited
temperature range. When the water rises above the coral's temperature
threshold for a prolonged period of time, the coral is damaged. While
this temperature threshold differs between different types of coral,
they generally cannot survive in waters above thirty degrees Celsius.
If the ocean water gets too warm, coral goes through a process of
bleaching, in which it loses the algae that give coral its vibrant colors.
Bleaching is an indication of damage to the coral. While it is possible
to survive the damage, most corals do not. In the western region of
the Indian Ocean, rising temperatures have contributed to bleaching
in an estimated ninety percent of native corals.
In addition, changes in weather patterns due fo global warming
have contributed to an increase in tropical storms. The choppy waves
damage coral reefs. Also, the rain can cause flooding on land. When
overflowing rivers deposit their runoff into the ocean, the incoming
sediment clouds the water with tiny particles. These particles act as a
screen that blocks the passage of sunlight. Since the algae that live
within the coral depend upon photosynthesis to live, the decrease in
available sunlight limits their ability to undergo this process. Without
adequate sunlight, the algae die, and therefore, so do the corals.

alter:
to change or modify
ecosystem:
the sum of the organisms
in an environment
vulnerable:
not protected; defenseless
temperature threshold:
the highest or lowest
temperature at which an
organism can live
Indication:
a sign; a characteristic
prolonged:
continued; constant runoff:
. material that is drained off,
such as from a river
cloud:
to make less clear

[-~-~hanges in Coral ~eef;:=J

rct---._
i----~~-----------

Cause 2:

_,..,.,. -

---------~1

'

i_,,__ , , , , . . . . . -----------j
c. - - 28 Chapter 1

"~

____.J

~----o-.-.-~-------------

! Effect:

L_ , ____. _

l
.j

1. According to paragraph 2, how does


ocean temperature damage coral?
(A) By breaking pieces off of coral
(B) By bleaching coral reefs
(C) By killing coral's food source
(0) By introducing harmful sediment

2. According to paragraph 2, what is


bleaching?

(A)
(B)
(C)
(0)

3. According to paragraph 3, what effect


does sediment have on coral?
'
'(A) It covers the algae and causes it to
break.
(B) It makes the water warmer and kills
the coral.
(C) It increases the water pressure and
breaks coral reefs.
(0) It prevents the algae from receiving
enough sunlight.

The loss of algae in coral reefs


The temperature threshold for coral
The loss of color in coral reels
The ability for coral to survive

Fill in the blanks to complete the summary.

,Global warming is responsible for _ _ _ _ _ the ocean's temperatures, which


affects coral reels. Coral reels are

to temperature changes, and they

become damaged if exposed to higher temperatures for _ _ _ _ _ _ _ periods. One


_ _ _ _ _ _ _ of coral damage due to warm ocean temperatures is bleaching. An
increase in tropical storms is another way coral reefs are being damaged because of global
warming. Sediment from overflowing rivers

the ocean, which decreases the

amount of sunlight reaching the corals.

FaG Questions

29 - - .

,.,

L.

The novel was first defined as any long narrative prose that
portrayed aspects of everyday life through fictional characters and
events. The novel appeared during the time that romance was popular.
The invention of Gutenberg's printing press in 1429 significantly
increased the literacy rate and the production and distribution of
written texts. Both the romance and the novel flourished during this
time, though they were very different in content and style.
The novel included different types of characters than those found
in romance stories. The latter described entertaining adventures
about heroes with the purpose of relating moral teachings. Often,
knights were the main heroes of romances, as in the 14'"-century tale
Sir Gawain and the Green Knight. The main theme of the story was
the glories of knighthood. The novel, on the other hand, featured daily
events and ordinary people. The novel often parodied romance's
typical style of impossible, glorified characters. The romance genre
was favored by the nobility, who enjoyed the formal language and
found it perfectly appropriate for describing the deeds of heroes. In
contrast, the prose used in novels was considered "low," and thus
more appropriate for satire.
Yet another difference was that the novel represented human
existence in a way that was closer to real life and included few
exaggerations. It portrayed the human experience through events with
which the average reader could identify. This allowed an opportunity
to engage in a study of human character. In contrast, romance stories
presented a colorful distortion of the characteristics and lives of people
through idealism. Through the novel, one could learn about the real
motives and temperaments of people in a generally realistic way.
With a romance, the point of emphasis is plot, while a novel's main
focal point is character. For example, Daniel Defoe's Robinson
Crusoe, considered to be one of the first "modern" novels, was told
in first-person narrative, through which the reader becomes intimately
familiar with the main character.

---

..

prose:
the ordinary form of written
or spoken language
distribution:
the giving out of things
flourish:
to be successful
parody:
to Imitate someone or
something comically
satire:
a genre that uses irony,
sarcasm1 .and ridicule
temperament:
a range of moods or
personality fluctuations
focal:
related to the center of
attention

-----~--------- ------.,----------------~---------~

1
1
1
i

i"f.t--- 30

Chapter 1

I 1The Romance vs: The Novel I


','

The Romance

Both.'

1. According to paragraph 2, what was the


purpose of romance stories?
(A) To teach morals through adventurous
tales
(B) To show characters who were
r: unrealistic

:\ (C) To educate the public about knighthood


!\., (0) To demonstrate typical daily life

3. According to paragraph 3, which is true


of romance stories?
(A) They tend to show people's problems
and faults.
{B) They have highly developed plot
structures.
(C) They emphasize a character's effect
upon the plot.
(0) Their plots can be formed with
common situations.

2. According to paragraph 3, how did the


novel portray characters?
(A) As exciting and heroic
(B) Idealistically
(C) As honest and tender
(0) Realistically

Fill in the blanks to complete the summary.


The passage discusses the appearance of the novel as it _ _ _ _ _ along with the
romance in the 15'" century. The invention of the printing press increased the _ _ _ __
of written texts to people. The novel often

the impossible characters and

formal style of the romance, instead presenting more realistic


of people. Also, the main
plot in the romance.
-----"-----------------------

and motives

point of the novel was character, as opposed to

----~-------------

.. --------------------------------------------------------Fact Questions

31 ------.

Anthropology
~~~;m<k'\'"tJl~~V-1M:~,_!-\f"jo\\<J,:.'l'f;l<:'1'>J.".:~,,-1 :~-i-'?1-'>':-'I''. ;;

.,_.,'-;'.?:-'ti.:.; ;.__- .. ~;,,, -, -

'

' with the information that you read. lll!!ll


Read the following passage. Then fill in the diagrijm

Ii

i '

People throughout history have developed ways to cope with


uncertainty in life. Symbolic behavior, which can be defined as any
superstitious action that attempts to bring about a supernatural result,
is one way that people have dealt with uncertainty. A recent poll
showed that over half of Americans hold some superstitious beliefs,
even if they know that they are illogical. Nonetheless, superstitions
may have positive psychological effects. Both rituals and taboos-two
common types of symbolic behavior-can help reduce anxiety and
give a person a sense of control.
Rituals involve any behavior that is done repeatedly in order to
produce a desired outcome. Generally, rituals express a person's anxiety
symbolically. This means the behavior is an attempt to lead to a result
through an unrelated action. Those who believe in the power of rituals
often feel that breaking the ritual will produce negative consequences,
perhaps even supernatural punishment. Psychologically, ritualistic
behavior can provide a sense of control over often uncontrollable
situations. For instance, Wayne Gretzky, a Hall-of-Fame hockey player,
tucked in his jersey in the exact same manner before each game. The
outcome of a hockey game obviously cannot be controlled solely by
a single hockey player. However, a ritual may give a player the feeling
that he has control over the team's success, even if it is through the
superstitious. habit of putting on a jersey in a certain way.
Some people do certain things like peFform rituals to obtain the help
of the supernatural. Others do not do certain things for the same reason.
A taboo is a forbidden behavior. Taboos can reduce anxiety in a number
of situations through what a person does not do. Taboos often originate
from beliefs that if a certain action is performed (often in a particular
context), some tragedy or misfortune will transpire. Unlike ritual, taboo
does not involve repetition. Even one incidence of the tabooed behavior
may result in supernatural punishment. This type of symbolic behavior
can be seen in the habits of New England fishermen. These fishermen
have many taboos while at sea, such as whistling on board or saying
the word "pig." Although even the fishermen themselves admit that
they do not believe the taboos have any effect, they continue to follow
them. In essence, these taboos assure the fishermen that they are
doing everything that they can to avoid misfortune. As a result, they are
able to set out to sea with a rnore positive mindset.

cope:
to deal wilh emotionally
symbolic:
representing something
!hat cannot otherwise be
expressed
outcome:
a result
Jersey:
a shirt worn as part of a
sports uniform
solely:
alone; as the only influence
transpire:

to occur

IL_._:__ __~----------------- .. ---,.-----~-'"-----~.--..----------------.. "----"-----~--------"---------------'

r~-- 32
,/

Chapter 1

l.,

!
l
'

Symbolic Behavior

Theory:

ISopport 1
ISopport 2

1. According to paragraph 1, how are rituals

and taboos related?


(A) They both require that an action be
performed.
(B) They are known to ease one's level of
stress.
(C) A taboo can often result from
ritualistic behavior.
(D) Written symbols can be included in
'' both behaviors.

2. According to paragraph 2, why is a ritual


said to be symbolic?

(A) Because it connects actions that are


not related
(B) Because breaking the ritual will cause
problems

(C) Beca.use it involves perceptions of


supernatural powers
(D) Because it gives people control over
their lives

. 3. Which of the following is true according to


paragraph 3?

(A) Taboos cause people to fear breaking


ritualistic behaviors.
(B) Fishermen believe that taboos protect
them from the supernatural.
(C) People may follow a taboo while
claiming not to believe in it.
(D) People believe more in taboos after a
tragedy occurs.

Fill in the blanks to complete the summary.

The passage discusses symbolic behavior as a means for people to _____ with
uncertainty in their lives. Two common types of

behavior are rituals and

taboos. Rituals are behaviors done repeatedly in order to bring about a certain
- - - - - - Although a ritual cannot

bring about a desired result, an

individual can get a feeling of control through performing the ritual. Taboos are forbidden
behaviors that can make some people believe that something terrible will _ _ _ _ __
By avoiding a certain taboo, a person may feel that he or she can also avoid misfortune.

Fact Questions

33 - -

Sociology
l<l.'Ut<~'lr<)m<';'!.';l>:\'}WJ.!;~'.,Y;i~'W"<il"!~,;o.,:..;/~;-1-!',')!'o-; .(.. ', '-" \<';

,t_o _ ,,,(, '" _,.

'

Read the following passage. Then fill in the diagram with the information that you read. lifflll

Globalization is a process of integration. It began centuries


ago and continues to develop today. It can be defined as the
internationalization of political, economic, cultural, and social
systems. Perhaps the most significant example of the early rise of
global culture is the Dutch East India Company. Established in 1602
by the Dutch, who were then colonizing the East Indies of the Indian
Ocean, it was the world's first multinational corporation. The Dutch
East India Company was a key impetus in the trend of globalization,
in that it significantly changed export activity and created an avenue
for extensive cultural exchange.
There was strong competition in the world market for trade
amongst the European nations. One by one, they moved across the
seas in order to expand their empires. The best way they could assert
their power in the growing world was through economic expansion.
The Dutch happened to emerge as a powerful entity mostly through
their maritime trade capabilities and were therefore able to establish their
empire in the Orient. As the Dutch trade grew, so too did its economic
power. Silver and copper were brought from Peru and Japan to trade
with India and China for various textiles such as silk and cotton.
Those items were then traded with other Asian nations for spices
such as cinnamon and pepper, which were in great demand iri Europe.
The company's trade activity also contributed to the spread of culture.
The Dutch government had granted the company a monopoly over
the trade of the "Spice Islands" of the East Indies. Not only did it enter
into trade agreements with the sovereign rulers of the nations of the
East Indies, but it also saw its competition reduced. This rise in power
came when the Dutch government expelled the Portuguese and
British from Indonesia and Sri Lanka. The company even collaborated
with missionaries, and through them was able to trade new cultural
items with Japan and China. In fact, though Japan closed itself to
trade between 1640 and 1854, the company was the only trade
group to have access to Japan. Thus, it was able to transport the
Japanese kimono to Europe, where it became highly . popular.
Because of the route that was established through the Dutch East
India Company, the cultural influences from each country circulated
around the world. Traders along each stop were able to gain a broader
base of knowledge about others involved in worldwide trade.
~---------------------

P<:-\! '

34 Chapter 1

Integration:

the combination of parts


into a whole
Impetus:

a moving force or stimulus

avenue:
a path
expel:

to drive out or send away


collaborate:

to work together
circulate:

to move from place to'


place

I>

Globalization

Cause 1:

. 1
Effect:

Cause 2:

1. According to paragraph 1, what event


occurred at the same time that the Dutch
East India Company was established?
(A) The colonization of parts of Asia
(B) The expansion of the Dutch government
. (C) The increase of the spice trade within
.. Asia
(D) An increased need for inexpensive goods
7.,

i;

2.' According to paragraph 2, how did the


Dutch East India Company gain economic
power?

(A) Through its ability to trade extensively


at sea
(B) By increasing its reliance upon Dutch
funding

(C) By receiving help from the Dutch military


(D) By refusing to trade with other western
countries

3. According to paragraph 3, how did


missionaries affect trade?
(A) They aided in expelling the British
from the East Indies.
(B) They allowed clothing to be
transported to Europe.
(C) They aided the Dutch government's
military.
(D) They helped the Dutch to establish
a trade monopoly.

Fill in the blanks to complete the summary.

The passage discusses the Dutch East India Company as an important _ _ _ _ _ in


the trend of globalization beginning in the 17th century. It provided a(n) _ _ _ _ _ for
cultural exchanges through the trade of goods by sea. Through their _ _ _ _ _ __
trade abilities, the Dutch grew in power and eliminated competition

by----~-

the Portuguese and the British from the East Indies. Through the trade activities of the Dutch
East India Company, highly prized and exotic items such as spice and textiles were able to
_ _ _ _ _ _ _ around the world.

Fact Questions

35 .--

ii

Necessary Skills
~\l;_~\'.l'Jal':ax<H<l."'>.1~W11Li-i::.',!li'<~~<'";-;,_~_,..;,J~;,-c;_.-

'' -,

Recognizing incorrect information as well as information not mentioned in the


passage
Identifying paraphrases that do or do not correctly summarize information from
the passage
Verifying that three answer choices are true and one is false

Example Questions
All of the following are mentioned in paragraph X as _ _ _ _ _ EXCEPT:
According to the passage, which of the following is NOT
?
The author's description of _ _ _ _ _ mentions all of the following EXCEPT:

Strategies
Be aware that often, the three incorrect answer choices are spread across a
paragraph or several paragraphs.
Keep in mind that the correct answer either directly contradicts one or more
. statements in the passage or it is not mentioned in the passage at all.
Check your answer to make sure that you understood the question accurately.

Meqative Fact Questions

37 - -

Psychology
~!'llffl~f'<ili%Yf.l.1:<Jfi">'.V;:0'll>,~;,;;_10 .. ~~,:;'.-:J,>;>;.~;o,Cn:

,.-~.

;h;

' with the information that you read. MMf!j:I


.Read the following
passage. Then fill in the diagrar.n
.
.

i,

il'.

The goal of social conversations is typically the same. Being


friendly, showing concern for others, and talking about topics of
interest are all typical aims for any conversation. However, when
engaged in same-sex conversations, the ways in which women and
men achieve these goals are remarkably different.
Men are far more likely to try to make conversation fun. This fact is
seen in the high incidence of joking by men in same-sex conversations.
Indeed, studies reveal that men are far more likely than women to
include joking and innocent teasing in their conversations. The subject
of personal feelings is seen as a sign of vulnerability and is therefore
broached very infrequently. Further, men do not seem as inclined to
call and chat as women do. Forty percent of the men polled in a
recent survey said that they never call their same-sex friends "just to
talk."
In contrast, women's same-sex conversations include more reference
to feelings, relationships, and personal problems. Some psychologists
have theorized that talk is essential to maintaining and nurturing
female relationships. In maintaining their relationships, women tend
to engage in more frequent conversations with their same-sex friends.
In fact, almost half of the women in a recent survey stated that they
called their female friends on a weekly basis just to chat.

Conversation: Men vs'.Women

t----~-----------

Men

engaged:
busy with a task
Incidence:
the rate at which
something occurs
Innocent:
harmless
vulnerability:
the state of being exposed
broach:
to mention
nurture:
to support

_]

----
Both

..

------~------~--~-- --~-

Women

----'-----------------J_____,___,_________,

1. According to paragraphs 1 and 2, all of


the following are true of conversation
EXCEPT:
(A) Most people converse for very similar
reasons.
(8) Men and women converse in different
ways.
(C) Men often tease women while talking
to them.
(D) Men do not typically like to discuss
emotions.

3. According to paragraph 3, which of the


following is NOT true of women's
conversations?
(A) Women often discuss personal
relationships.
(B) Conversations are essential to female
friendships.
(C) Many females call their friends
regularly.
(D) Most women fear discussing their
emotions.

2. The author's description of male


conversation mentions all of the following
EXCEPT:
(A) Men generally try to use conversation
for amusement.
(8) Men sometimes hurt others through
teasing.
<(C) Men feel that discussing feelings
makes them appear weak.
(D) Many men see emotion as a sign of
weakness.

Fill in the blanks to complete the summary.

According to the passage, men and women

in very different same-sex

conversations. For men, talk is generally not personal, is more likely to include
_ _ _ _ _ teasing and joking, and is not very common. Men often consider personal
feelings a sign of - - - - - - - By contrast, women often _ _ _ _ _ personal
topics such as feelings and problems with one another. Female conversation is thought to
be very important to maintain and _ _ _ _ _ female relationships.

_.Negative Fact Questions

39

-~

Read the following passage. Then fill in the diagram with the information that you read. @U

The lack of clean, suitable water for agricultural purposes is a


growing global problem. One of the best ways to counter this problem
is to ensure that water is used more efficiently. Since agriculture is the
greatest user of water, it is of great importance that farmers and
agronomists alike seek out ways to increase the efficiency of irrigation.
A simple method of water conservation is already being used in
many countries: sub-surface irrigation. When irrigation systems apply
water above the surface of the ground, much of that water either
evaporates or runs off. In warm climates, this waste has been
estimated at nearly fifty percent. In other words, half of all the water
applied is lost. In sub-surface irrigation, as the name implies, water is
introduced below the surface of the ground. This action significantly
reduces water loss. Not only is water saved, but the more uniform
application of water to the plants' root systems leads to hardier plants
with more abundant yields.
Another novel way to use water more efficiently is to grow crops
that are native to the region. Because species have adapted over time
to their native regions, they are often able to handle local conditions
better than non-native species. Corn, native to parts of Mexico, is an
example of how native species can reduce the need for significant
irrigation. In the latter decades of the 20" century, non-native species
of corn from the United States were introduced to and sold in Mexico.
While farmers enjoyed significant yields from these introduced
species, their water usage skyrocketed. This resulted in water-table
depletion throughout many agricultural regions in Mexico. In response,
Mexican farmers have recently reintroduced native corn species that
are better able to handle arid climates.
~-------------------------------

.........--................
1

----------1

counter:
to oppose; to express an
opposite view
agronomist:
one who studies soil and
crops
evaporate:
to convert a liquid into a
gas
uniform:
consistent
hardier:
stronger

novel:
new
depletion:
the act of reducing the
supply of something

--------------

Water Shortage

--]

----~--------------

r----- --
1 Problem:

----------1

I.

r.---------------

I<!
I

i Solution 1:

---_

Solution 2:
"-"- ... .... - ....... ---- ..-.... --
... ...~., _ _ 40 Chapter l

''""'"

.Aa~--- '

' '

,_,.,.-'"'-------------

-----">-.. ~<>~

----.--

--

~U> .-.v,,l

1. According to paragraph 1, which of the


following is NOT true of the water shortage?
(A) It is a problem that occurs all around
the world.
(B) It can be countered by using water
more efficiently.
(C) It is caused partially by farming
practices.
(D) It is affecting worldwide drinking
water supplies.

3. According to paragraph 3, which is NOT

true?

'

(A) Plant species can adapt to their


environments.
(B) Non-native corn in Mexico requires
increased watering.
(C) Mexican farmers are now using more
native corn species.
(D) Mexican corn produces higher yields
than US corn.

I
II

~
~

2. Based upon information in paragraph 2,


all of the following are true EXCEPT:
(A) An above-ground system is less
efficient than a sub-surface system.
(B) Underground irrigation reduces
evaporation and runoff.
(C) Crop yields improve when
underground irrigation is used fifty
:it percent of the time.
;(D) A sub-surface irrigation system gives
t
water directly to the roots.
Fill in the blanks to complete the summary.

The passage discusses ways in which farmers are tryin,g to

shortages of

clean, fresh water. When crops are watered above the surface of the ground, much of the
water
irrigation systems, a more
of the plants. Another

instead of reaching the plants. By installing new underground


application of water can reach the root systems
idea for farmers to use water more efficiently is to

grow more native species of crops. Native crops can grow without excessive watering, thus
slowing the

of precious water resources.

-----------------------------------------t'-----------------------------------------------------------------------

Negative Fact Que~t1ons

41 - -

Marketing
mu;.>o;:w.1~>'.l'>Vl'-<>::'>..,;~<:;r_~-- :.~,;. ,_,,_;.,:,~"::~ ~~->-"''"''''-';,,,-,,

,;,-,.,;,,1" _., ,

Read the following passage. Then fill in the diagram with the information that you read.

Marketers have long advertised their products in newspapers,


magazines, and on the radio. Given the billions of dollars spent
on these forms each year, they must certainly be useful. Yet as
advantageous as such advertising is, there is perhaps no better way
to sell a product than by offering a hands-on demonstration of it.
Doing so allows the potential customer, depending on the product, to
touch, drive, or taste it.
One of the main benefits of product demonstration is that it makes
the product more tangible to buyers. Until most consumers actually
touch a product, it is nothing more than an advertisement in a magazine
or on television. This is certainly the case with new-car sales in the
United States. There, it is common for car shoppers to take a "test
drive" of the vehicle that they are considering buying before making the
actual purchase. It has been shown that car salespeople are very
likely (some estimates put the likelihood at ninety percent) to offer a
prospective customer a test drive within just five minutes of meeting that
customer. There is, of course, a valid reason for this. Empirical studies
show a test drive can increase the likelihood of a purchase significantly.
Another benefit of product demonstration is that it can make a
product's selling points immediately apparent. This makes the product
more appealing in the eyes of the potential customer. While print ads
can certainly list these selling points, customers often want to see
a product in use before they make the decision to purchase it. To
illustrate, consider the situation faced by a leading manufacturer of
non-stick cookware in the 1970s. The manufacturer had just created
a substance that, when permanently applied to-._ pots and pans,
allowed cooked foods to separate easily from the cookware rather
than stick, as was common with previous cookware. Initial consumer
response to the product was very low. However, after demonstrating
the non-stick substance's function at fairs and expositions, sales of
the cookware soared.

@"'

advantageous:
beneficial; favorable
hands-on:
involving human
participation
tangible:
real
prospective:
possible
empirical:
performed through
experience or experiments
exposition:
a large public show

11
;

'1

,,

l:..,,"\,---

.
'
~1
i

42 Chapter 2
.

I
Definition:

.Product Demonstration . .

Advantage 2:

1. According to paragraph 1, all of the


following are true about product
demonstrations EXCEPT:

:!
,,.

LL

Example:

[
[

--+

"

Advantage 1:

IIf

(A) They allow consumers to see how


a product works.
(B) They make customers more likely
to purchase a product.
(<S) They display a product more
effectively than a commercial.
(D) They can be used through various
forms of print advertisement.

2. Which of the following is NOT true


according to paragraph 27
(A) Demonstrations can make a product
seem real.

Example:

--+

(B) Vehicle test drives often increase car


sales.
(C) Car dealers often offer test drives to
customers.
(D) Over ninety percent of car customers
take a test drive.

3. According to paragraph 3, all of the


following are true EXCEPT:
(A) Demonstrations can change a
customer's opinion of a product.
(B) Demonstrations work best after
customers have seen a print ad first.
'(C) Many customers do not buy a product
until they have seen it used.
(D) A cookware company increased sales
with product demonstrations.

Fill in the blanks to complete the summary.

According to the passage, while product advertising i s - - - - - - - in selling goods


and services, a hands-on demonstration is often more effective with - - - - - - customers. Product demonstration makes an item more _ _ _ _ _ to buyers, in that
they can actually touch and use it before they buy it. This shows

what

a product's function is, which can lead to higher sales of the product. For example, a
manufacturer of non-stick cookware made many more sales after demonstrating its product
at a(n) _ _ _ _ _ __

Negative Fact Questions

43 - -

Political Science
Read the following passage. Then fill in t~e diagram with the information that you read. lf!ttll'I

j' '

-!~.

(;~ 44

Dependency theory originated in the 1950s to explain why c.ertain


nations remained poor and failed to integrate into the world economy.
Even then, the world economy was becoming more global in scope.
Poor nations today are still often dependent upon larger, more
developed nations of the world. The theory states that they will remain
so unless they enact major economic reform.
Dependency theory asserts that the economic development of
poor nations cannot continue unless they significantly restructure their
import and export programs. Raul Prebisch, a leading dependency
theorist of the 1950s, stated that developing nations typically rely
upon the export of raw materials or simple commodities for their
economic well-being. Neither of these exports, however, can supply
enough capital for these nations to expand their economies
significantly. Further, these developing countries also import many of
their basic needs, such as clothing, vehicles, and medicine. Prebisch
noted that, in most developing countries, export activity is lower than
import activity. Therefore, the revenue .gained from exporting is
considerably lower than the expenditures required to import such
necessities. This reliance upon imports assures that developing
nations cannot support a healthy economy.
An example of this theory was seen in many Caribbean nations.
Countries in the Caribbean display economic dependence through
their reliance on single exports (such as sugar) or industries (such as
tourism). These countri~s rely on imports in order to fulfill their basic
needs. For example, Grenada exports mainly bananas, cocoa, and
nutmeg, since these grow well there. However, it imports machinery,
fuel, manufactured goods, and many food items. The profits from the
few export industries cannot match the necessary spending for
imports. Therefore, Prebisch would argue, Grenada cannot progress
economically and will remain dependent upon developed nations.
Prebisch and other dependency theorists suggest that the only viable
solution for these developing nations is to reduce dependency upon
imports from other nations. To achieve this, the dependent countries
must increase their level of manufacturing.

Chapter 2

Integrate:
to combine into a whole
restructure:
to change completely

capital:
money
revenue:
an amount of money
broughl in lrom sales
expenditure:
an amount spent; a cost

viable:
able lo be put inlo
practice; practical

:1

lI
j
i

'

!I

J .

'

Dependency Theory

1_
z

Theory:

Support:

Example:

G>

[
[
1. According to paragraph 1, all of the
following are true of dependency theory
EXCEPT:

(A) It was introduced in the 1950s.


(B) It explains why some nations remain
poor.
(C) It says that poor countries cannot
progress.
(D) It claims that small nations are
!' dependent on developed nations.
f;
:~

: 2. Which of the following is NOT true about


dependency theory according to the
passage?

(A) Developing nations do not gain


revenue.
(B) Developing nations must import less.

(C) Developing nations often import more


than they export.
(D) Developing nations get many needs
from other countries.

3. According to paragraph 3, which of the


following is NOT true?
(A) Reliance on a single export or
import may be profitable.
(B) Many Caribbean nations are
dependent on other nations.
(C) Imports must provide for the basic
needs of Grenada.
(D) Grenada spends more on imports
than it gains on exports.

Fill in the blanks to complete the summary.

The passage discusses dependency theory, which explains why some developing
nations cannot _ _ _ _ _ into the world economy. These countries will not succeed
economically until they

their import and export programs. Because the

_ _ _ _ _ gained from exports is much lower than the _ _ _ _ _ _ _ needed


to import basic goods, these nations cannot generate a healthy economy. Dependency
theorists suggest that the only _ _ _ _ _ solution for these countries is to increase
manufacturing in order to decrease dependence upon developed nations for their welfare.

Negative Fact Questions

45 - -

Anthropology

n'>~\'.':1~&:'<><"1trnlfr'.>-~-~"'',,;';-n-1.<.:>;i>f:<''<'1~>-'"t'l"-" 0

"'"

Read the following passage. Then fill in the diagram with the information that you read. "!Al"

: i

Ii

standard:
a basis for judgment

Ethnocentrism is defined as the interpretation of a culture based


upon the standards of one's own culture. Cultural relativity, on the
other hand, seeks to interpret a culture based solely upon that
culture's standards. This is done with no attention given to other
cultures. Increasingly, cultural relativity is the preferred means of
cultural interpretation for anthropologists.
Ethnocentrism is common both as a social and an anthropological
means of interpretation. Generally, ethnocentrism is a universal
phenomenon. For example, ethnocentrism can be seen in something
as simple as an American citizen stating that the English drive on the
"wrong" side of the road. By stating that the other side is wrong, this
hypothetical citizen has judged another culture based upon that of
the United States.
I This becomes a more serious issue in anthropology when it
affects the way entire cultures are represented. One well-known
researcher accused of an ethnocentric view is Erich van Daniken, a
Swiss author and archaeologist. Through his studies, van Daniken
concluded that many historical societies, such as the ancient
Egyptians, were not as advanced as other cultures. Therefore, he
speculated, they could not have completed the Great Pyramids
without assistance. This assistance, van Daniken claims, must have
come from extraterrestrial beings.
Cultural relativism, on the other hand, essentially reverses the
thought process of ethnocentrism. Thus, cultural judgments are
based on the realization that there is no universal standard for culture.
Cultures are distinctive and equal in terms of value-a position many
find difficult to adopt. Cultural relativism requires both knowledge and
understanding of cultures, which involves extensive effort and study.
However, the problem is alleviated slightly within field of
anthropology. Unlike the general public, anthropologists have the
means to gain a deep understanding of cultures through their studies.
Therefore, cultural relativism is more easily practiced. Returning to
the ancient Egyptians, a cultural relativist would examine Egypt not in
terms of the technology understood by one's own culture, but only in
terms of the Egyptian culture itself. Thus, one would accept that the
Egyptians must have built the pyramids. The focus, then, becomes

~ow they built th~~

universal:
applicable everywhere in
the world
hypothetical:
imaginary; not real
extraterrestrial:
outside of Earth
position:
a set of views
extensive:
complete
alleviate:
to lessen the effects of

,,
j

1j
!
j

'
'

---------------

'

_JI

,..

I Cultural lnterpieta(ions
Type 1:

Type 2:
I

' Definition:

Definition:

[f

Example:

Example:

1. According to paragraph 1, which of the


following is NOT true of ethnocentrism?
(A) Anthropologists favor cultural relativity
over it.
(B) ,It is widespread.
:(C) It can ultimately lead to more culturally
'' relative ideas.
'(D). It applies values globally.

2. Based upon paragraphs 2 and 3, all of


the following are true EXCEPT:
(A) Ethnocentrism is seen in
non-anthropologic situations.
(B) Von Daniken believed aliens were.
present in ancient Egypt

(C) Von Daniken was accused of being


ethnocentric.
(D) There is no evidence to refute who
built the pyramids.

3. According to the passage, which of the


following is NOT true of cultural relativism?
(A) It requires an understanding of many
cultures.
(B) It does not believe in universal cultural
standards.
(C) It studies Egypt only in terms of
" ancient Egyptian culture.
(D) It is more common than ethnocentrism.
'

Fill in the blanks to complete the summary.

The passage discusses two ways in which cultures can be understood. The first is
ethnocentrism, which is the interpretation of a culture based on the

of

one's own culture. Ethnocentrism is thought to be a(n) _ _ _ _ _ occurrence. Another


way to interpret cultures is cultural relativism, which judges cultures according to their own
standards. However, this

can be ditlicult to adopt without _ _ _ __

effort and study of other cultures. Anthropologists help

this problem.

Negative r:act Questions

47 - -

Read the following passage. Then fill in the diagram wit~' the information that you read.

I;

The Globe Theatre, located in London, England, is one of the


most famous theaters in the world. During its illustrious history, it was
a stage for many of the greatest plays of all time, including those of
William Shakespeare. The history of the theater, however, is complex
and turbulent. Spanning a period of approximately 400 years, the
Globe Theatre underwent three major transformations.
The original Globe was built in 1599 in London's Bankside district
and was owned by a number of local actors and playwrights, among
them, William Shakespeare. The theater was built using the materials
from an earlier theater called, simply, The Theatre. The Globe was
constructed as an open-air amphitheater that resembled a
twenty-sided polygon. It was three stories high with seats filling its
interior perimeter. In the center was the pit, where attendees (called
groundlings) would stand and watch the performances. This first
structure lasted until 1613, when it was destroyed by fire during a
performance of Henry VII I.
The Globe Theatre was quickly rebuilt and was completed in
1614. Among the changes to the theater was a new tiled roof to
prevent future fire disasters. However, the theater would only remain
. open until 1642. At the time, the Puritans were the main force in area,
closing down theaters and other centers of entertainment. In 1644,
the Globe Theatre was demolished to make room for apartment
buildings. For the next 350 years, the Globe Theatre would continue
to exist only in the minds of its many admirers.
The modern adaptation of the theater was constructed in 1997,
and named "Shakespeare's Globe Theatre." The structure of the
original Globe Theatre was painstakingly researched and measured
to ensure that the new theater would be an accurate replica. Just like
the original, the modern theater is an open-air venue. The main
difference now is the restrictions laid out by the Health and Safety
Department: while three thousand could fit in the original Globe
Theatre, only 1,300 are now allowed to be inside at any given time. In
addition, electric lights were added to allow for night performances to
be held. Unfortunately, the exact location for the Globe Theatre could
not be used, so it is now located about 200 meters from its original
location.

._

;<

.'

Illustrious:
distinguished
turbulent:
chaotic
span:
to re_ach or extend over or
across something
polygon:
a closed shape with three
or more sides
perimeter:
an outline or border
attendee:
someone who participates
in or attends an event
admirer:
one who has a favorable
impression of something
venue:
the location of an event

l--------------------------------------------~-~.-.--- ----- . -------------------- ""' ---"'""-~-------------

.~...""~-- 48 Chapter 2

,I

@!I

'

,,_;,1

. I

The History of the Globe Theatre


Step 1:

Step 2:

Step 3:

[[

[
1. According to paragraph 2, all of the
following are true of the first Globe
Theatre EXCEPT:
(A) It was destroyed in 1599.
(8) It was partially owned by William
Shakespeare.
, . (C), It was built from the materials of
another theater.
(0) The audience stood in a pit in the
center of the theater.

2. According to paragraphs 2 and 3, which of


the following is NOT true?
(A) The theater was rebuilt within a year
of its first destruction.
(8) The Puritans did not allow the theater
to remain open.

(C) The Puritans ordered the theater


destroyed in 1642.
(0) The theater's design was influenced
by a disaster.

3. According to paragraph 4, which of the


following is NOT true?

(A) The new Globe is named after


William Shakespeare.
(B) The new theater accurately
reproduces the Globe.
(C) The Globe now has lights for night
performances.
(0) Over 3,000 people can now attend
performances.

Fill in the blanks to complete the summary.

According to the passage, London's famous Globe Theatre has a complex and
_ _ _ _ _ _ _ history _ _ _ _ _ about 400 years. Constructed as an open-air
amphitheater, its center was a large pit in which _ _ _ _ _ would stand and watch the
performances. After it was destroyed twice, it existed only in the minds of its _ _ _ __
for 350 years. Finally in 1997, the Globe Theatre was again rebuilt. It is an open-air
----~'just

like the original.

----..----- .. ----------"''"-'""'----------------------.. --------- ..----------------------------------------Negative Fact QL1estions

49 - -

i\lecessary Skms
"

~t.\i!;~J:il.\";;T~1''lJi;'!c'.(-'.:~;,/,~,'.'~"' ,,,;,~<.,,,,

,' ,<. ._,-_c.,>:;~ ";t-.; " '

Perceiving ideas that are suggested but not directly stated within a passage
Drawing conclusions based on information given within a statement or section of
a passage
Determining logical implications of the author's v;ords

Which of the following can be inferred about _ _ _ _ _?


The author of the passage implies that _ _ _ __
Which of the following can be inferred from paragraph X about _ _ _ _ _?
Based on information in paragraphs X and Y, what can be inferred about
- - - - -?
It is suggested in paragraph X that--~--

Strategle::;
.-

.-'"_;,'}.(:'''"'~..:,'"

,;: ''F'

.,c,

..

-~ W,,,.,.,,_~,'

"~.

,.-_,--''

Ensure that your answer does not contradict the main idea of the passage.
Do not choose an answer because it seems important or true. The correct arl'swer
must be inferable from the passage.
Check that you can defend your answer choice by referring to explicitly stated
information in the passage that points to the inference you have chosen.

lnff:!rence Questions

51 - -

Literature
;

i.~ ;.:.\

Read the following passage. Then fill in the diagram with the information that you read. +Mfi!llI

An unreliable narrator is any narrator that cannot be trusted to


convey a story accurately. This device is used in literature to force
readers to question the narrator's truthfulness. In addition, many
authors use unreliable narrators to add a sense of realism to a
character.
The use of the unreliable narrator in order to affect the reader's
trust in the tale being told is well illustrated in "The Cask of
Amontillado" by Edgar Allen Poe. The narrator, Montresor, tells the
story of how he was wronged. Already, his view is tainted with
prejudice as he expresses his desire to take revenge. As the story
progresses, the reader begins to distrust his account further as the
symptoms of his psychological illness are revealed. Thus, the reader
must contemplate the character's tale and make his or her owri
decisions about what to believe.
An unreliable narrator can also give a character an added sense
of realism. Many authors realize that people do not always tell the
truth. They may misinterpret situations, and writers structure their
characters accordingly. In the case of Poe's Montresor, had Poe
depicted him as a truthful, reliable narrator despite his mental
instability, the representation would not have been realistic. However,
Poe chose to tell the story as Montresor would have told it, thereby
giving the reader a more comprehensive understanding of his
character.

1--------..--..

device:

atool.

realism:

lhe quality of being true lo


reality
taint:

to contaminate
contemplate:

to think about
comprehensive:

complete

----i

Unreliable Narrators

L__
. --..----------..

.....................- .......................-------------,
1

Definition:

Role 1:

Role 2:

1. What can be inferred about unreliable


narrators?
(A) Most readers believe their lies.
(B) They were first created by Poe.
(C) They are very clever characters.
(D) They may be male or female.

2. Which of the following can be inferred


about Montresor?
(A) His illness is not known at the
beginning of the story.
(B) It is difficult for readers to interpret
his narration.
(C) Poe was not aware that he was an
unreliable narrator.
(0) He is the hero of the story.

3. What does the author of the passage

imply about realism?


(A) Realism cannot be present in normal
characters.

(B) Readers find complex characters


more realistic.
(C) Montresor's mental illness is unrealistic.
(D) Characters that lie are often more
realistic.

Fill\in the blanks to complete the summary.

According to the passage, an unreliable narrator is a literary----~ that is used


to challenge the reader's trust. Unreliable narrators are also used by authors to add a sense
of

to a character. The example used is "The Cask of Amontillado" by Edgar

Allen Poe. The unreliable narrator in this case is Montresor, whose view is _ _ _ __
with prejudice and a desire for revenge. Readers must _ _ _ _ _ _ _ whether or
not to believe the unreliable narrator, thereby gaining a more _______
understanding of the character.

Inference Questions

53 --'

Business
Read the following passage. Then:,fm in the diagram with the information that you read.

. ,,.

i.

An entrepreneur is a person who takes the risk of starting and


managing a business venture in order to make a profit. Many new
businesses fail-some studies estimate the failure rate as high as
seventy-five percent. Nonetheless, experts believe that two things
contribute to success in entrepreneurialism: innovation and a
willingness to take calculated risks.
Innovation is necessary in new businesses. The goal is to meet an
unsatisfied demand. This requires creativity, the driving force behind
innovation. Innovation is particularly important for entrepreneurs
attempting to break into e-commerce, or businesses that buy and
sell over the Internet. The Internet can provide consumers with both
convenience and lower prices. Jeff Bezos, the founder of
Amazon.com, realized that many consumers wanted books at
discounted prices. They did not care where or how they bought them,
and in fact, many preferred to have the books delivered to their
homes. Bezos, certain that online demand could be strong, started
the first major online bookstore, which is now highly successful.
The ability to take calculated risks is another key component of a
new business. Unlike working in an already established business or
organization, an entrepreneur enters a territory in which success is
not guaranteed. Thus, the entrepreneur is taking a unique financial
and personal risk. Debbi Fields, the owner of Mrs. Fields, a highly
successful chain of cookie stores, was told to stay out of the business.
People said that no one would buy her cookies, since cookies are
available in supermarkets everywhere. Nonetheless, she felt there
was a strong demand for fresh, hot cookies. At last, she convinced a
bank to finance her homemade cookie business concept in the late
1970s. Her risk paid off-her company is worth over 450 million
dollars today.

@!li

venture:
a risky business investment
Innovation:
a new or unique product
or method
calculated risk:
a risk taken only after
carefully considering the
results
founder:
a person who starts a
business, organization, or
nation
component:
a part
established:
existing
territory:
an area or field
finance:
to supply with money

~---------------------------~--------------------~

Successful Entrepreneurship

-R-eq-u-ir_e_m~e-n-t

I.__

-1:______H

-2:----~H

L.__R-eq-u-ir_e_m_e_n_t

'"""''"

&ompl"

1. Which of the following can be inferred


from paragraph 1 about business?
(A) New businesses require a significant
financial investment.
(B) Entrepreneurs cannot compete with
larger businesses.
(C) Businesses that are not innovative
never succeed.
i:(D) .Only a quarter of all new businesses
.,,, are successful.

2. What does the author suggest about the


founder of Amazon.com?
(A) He had great difficulty in opening up
this business.
(B) He can interpret whai the consumer
really wants.

(C) He worked in a bookstore before


starting his business.
(D) He hopes to move into other areas of
e-commerce.

3. According to paragraph 3, what can be

inferred about the cookie company?


(A) It is the most successful cookie
company in history.
(B) It was originally not popular with
customers.
(C) Its success is due mainly to bank
financing.
(D) It would not have succeeded
without risk-taking.

Fill in the blanks to complete the summary.

According to the passage, entrepreneurialism is the launching of a business


- - - - - in order to ma,ke money. Two factors that can bring about success in new
business a r e - - - - - and risk-taking. Innovation is crucial in fulfilling a unique
public demand. Risk-taking is another necessary------- of a new business
because many entrepreneurs are competing in a new or difficult _ _ _ _ _ . For
example, the owner of a successful cookie chain had to convince a bank to _ _ _ __
her business concept in order to become successful.
------------------------- .. ------"'- ...-.---<>---- .. -,.---------------------: . -------------------------

lnfPren<:e Questions

55

-'---1.

Biology
~~"!JIB{l'<kli':<'l!l.n:.~101.>.t~ii--0Ml'X<:,'

c;',,1.-,.r.'.- ,-

n-'~;,

. ; ''-'>.'"-'. -.,,

Read the following passage. Then fill in the diagram with'the information that you read. @Sil

ii

,.

. 'I

Carl Linnaeus (1707-1778) was a Swedish botanist, explorer, and


physician. He is best known for establishing a system of species
classification and naming called the Systema Naturae. The creation
of this system was a lifelong process for Linnaeus, and the result of
his work is a system of classification that is still in use today.
Linnaeus's love of plants began during his childhood. Linnaeus's
father introduced Carl to botany at a very young age, and his interest
grew quickly. By the age of eight, Linnaeus had acquired the
nickname "the little botanist" Linnaeus later enrolled at a university to
study medicine. However, much of his time at university was spent
workingwith plants, as plants were common in medicinal treatments.
During this time, he began to record plant species and attempt to
separate them into categories.
The next stage of Linnaeus's career began after his graduation.
Despite financial hardship, Linnaeus embarked upon two botanical
expeditions in 1731 and 1734 in orderto discover more plants that
could be added to his growing list Then, in 1735, Linnaeus's friend
and fellow botanist Jan Fredrik Gronovius published the first version
of Systema Naturae. However, realizing that his classifications were
far from complete, he set out on many more expeditions.
His expeditions ceased, however, when Linnaeus became a
professor at the University of Uppsala in 1741. He soon became
famous in Sweden for his botanical work. During his tenure as a
professor at Uppsala, many of his students set out on similar
expeditions around the world to gather plant species. In addition,
Linnaeus had gained some renown worldwide, and many people sent
him plant specimens from their native countries. His Systema Naturae
grew considerably during this time; what had begun as a small
pamphlet had grown into an extensive, multivolume work. Following
Linnaeus's death in 1778, scientists continued his work.

botanist:
one who studies plants
hardship:
a difficult condition
expedition:
a journey
tenure:
a period of holding a title
or office

renown:
fame
specimen:
a sample of a material

considerably:
to a great extent
pamphlet:
a small published work of
information

-------------------------------------~

.I

The Botanical Career.of Carl Linn,aeUs


Step 1:

..
:~

Step 2:

1. Which of the following can be inferred


from paragraph 2 about Linnaeus?
(A) He was not a good student.
(B) His medicinal plants are no longer
used today.
(C) He was not actually interested in
studying medicine.
;'(D) His father influenced his career.
{':

2. :Which of the following can be inferred


from paragraph 3?

Step 3:

3. Which of the following can be inferred


from paragraph 4?
(A) Linnaeus took his students on
botanical expeditions.
(B) Linnaeus's discoveries were not
famous outside of Sweden.
(C) Linnaeus traveled to new countries
during his professorship.
(D) More species were added to
Systema Naturae after Linnaeus's
death.

(A) Systema Naturae originally contained


many errors.
(B) Linnaeus graduated from university
before 1731.
(C) Linnaeus traveled with Gronovius.
(D) A friend financed Linnaeus's
expeditions.
Fill .in the blanks to complete the summary.

According to the passage, Carl Linnaeus was a(n) - - - - - w h o dedicated his


life to creating a classification system of plants. The process began with a love of plants as
a child. Linnaeus set out on m a n y - - - - - - - to find new plants. As a professor,
he gained worldwide _ _ _ _ _ and collected many plant _ _ _ _ _ from around
the world. His Systema Naturae g r e w - - - - - - - into a multivolume work that was
preserved by other scientists after his death.
---------------------------------------------------------- .. -----------------------------------------------inference Questions

57 - -

Psychology
_<tIM~.;;Z'...'.~1:<>!.'!'1,'.)!B:ll'/.!it'l':'.!.'t;l';'.r'fil:.>;t!lO<t:u.'>1'~.:rn.~c~:-;.-.';,,~;-_-,~ ;,,-,

;,.;._ ' ,

Read the following passage. Then fill in the diagram with the information that you read. *filfl
'

.i

'

'\

ir

i:

I'

ii

~i~~:~- 58

Psychologists have theorized that schemas and other processing


methods play a role in the phenomenon of generating false memories.
A theory presented by Reyna and Brainerd is called the fuzzy trace
theory. Essentially, the theory states that during any event, the brain
makes traces of important information from that event. These traces
can be separated into two distinct categories: .gist traces and
verbatim traces. Each category of traces has a role in the creation of
false memories.
Fuzzy trace theory defines a gist trace as a mental image that
provides a general sense of the entirety of an event. A person
participating in any event, then, would likely remember the most
important parts of it. However, a gist trace does not provide specific
details from the event. Minute details such as names, words, or faces
may not be included in the person's memory of the event. Therefore,
the mind may create a false memory to fill the gap created by the
absence of one or more of those details. For example, after one's
participation in a soccer game, one may remember that the outcome
was a'victory. However, the player may forget a specific detail, such
as the period in which a teammate scored a goal. The player's mind
may then create a false memory: he or she may remember that the
teammate scored in the second half when, in fact, the goal was
scored in the first half.
Conversely, a verbatim trace focuses on a specific detail. These
traces are often item-based. The focus is on a particular item or
perhaps a specific moment from an event. A focus on details can
cause false memories of the overall structure of an event because the
memory creates only verbatim traces that omit the essence of the
events. Many medical patients claim to have forgotten entire events,
though in their memory they can recall specific details. For example,
a medical patient may say that he or she has forgotten what
happened over a number of hours, though he or she remembers
specific details, such as street signs, faces, and so on. In the
absence of the gist of the event, a false memory may be created to
fill in the memory gap. The memory, then, will include the correct
details inserted into an incorrect series of events.

Chapter 3

... .

schema:
a pattern used to aid In
understanding
phenomenon:
an occurrence or
circumstance
gist:
the essence or main part
of something

verbatim:
using identical words
entirety:
the state of being
complete

gap:
an opening or break

outcome:
a result

Definition:

Fuzzy Trace Theory

J.

!!!m
z

Category 1:

Example:

Category 2:

Example:

1. Which of the following can be inferred


from paragraph 1?
(A) Multiple theories exist to explain
false memories.
(B) Psychologists often deal with strange
phenomenon.
(C) Reyna and Brainerd created two
~.. different theories.
(D) Gist traces are more common than
. verbatim traces.

2. According to paragraph 2, what does the


author suggest about gist traces?
(A) They occur often during sports
matches.
(B) They fill in gaps with false memories.

z
G'I

II

(C) They blur the smaller details of an


event.
(D) They include both specific and
general details.

3. Which of the following does the author


imply in paragraph 3?
(A) Gist and verbatim traces are not
related.
(B) Verbatim traces do not create false
memories.
(C) Most medical patients use verbatim
traces.
(D) Verbatim traces are a common
medical issue.

Fill in the blanks to complete the summary.

According to the passage, the

of generating false memories can

be explained by the fuzzy trace theory. This theory states that after an experience, the brain
makes traces of information. A(n) _ _ _ _ _ trace is a remembrance of the event in a
general sense. Because the brain remembers a general sense of the

of an

event rather than specific details, false memories may be created to fill in the
_ _ _ _ _ . A(n) _ _ _ _ _ trace is a recollection of events based on specific
details or moments, rather than the experience as a whole.

Astronomy
--.iJl.!!~wu~-4'-hnW<>'*~"";q";1.!,.w&ai.t;..;,,,..~-,~;;,,;,,1<-""''"'''''" .,~cc

,.,_,--.

Read the following passa~e. Then fill in the diagram with the information that you read.

The moon is the Earth's only natural satellite, orbiting around the
Earth from an average distance of 238,000 miles. According to the

terrain:

an area of land
vast:

giant impact theory, another forming planet collided with our planet
about 4.5 billion years ago, causing large pieces of the Earth to leave
the atmosphere and go into orbit around the Earth. Later, these

very large

pieces underwent a process in which they combined, melted, cooled,


and subsequently formed the moon. After its formation, numerous
craters, which are characteristic of the jagged terrain of the moon,
began to form. The vast majority of lunar craters were created by

eject:

impacts, although volcanic activity may also have caused craters to


appear.
An impact crater is formed when a meteorite hits the surface.
Some planets and moons contain atmospheres that are able to burn
up meteorites due to the heat produced from air resistance as they

barrier:

a structure that prevents


passage
to push out forcefully
velocity:

speed
molten:
melted
viscosity:

the property of a fluid that


causes it to resist flowing
plateau:

a raised area of land that


is flat at the top

enter the atmosphere. The moon does not have an atmosphere.


Therefore, it has no protective barrier to burn meteorites up before
they reach the surface. Instead, meteorites forcefully explode upon
impact, ejecting surface material at great velocity. The force of the
impact causes smaller pieces to shoot out from the impact site. The
ejected surface material falls back down and showers the moon's
surface, creating a raised ring around the crater. Larger craters are
created by the impact of asteroids, which are much larger than
meteorites.
Volcanic craters form when hot magma, which is molten rock
beneath the surface, rises upward, breaking open the surface layers.
Since the moon has no moving plates, there are no upper mantle layers .
as there are on the Earth. Upper mantle layers generally remain
intact, so eruptions from below the surface are less likely to occur.
Because of the lack of mantle layers on the moon, the magma breaks
right through the surface. Through these cracks, hot gases escape
as the magma cools and recedes. Through this process, a cavity
beneath the surface is formed, and it quivers from the lack of support.
As magma has low viscosity, it does not have enough matter to fully
form a plateau above the surface. As the surface collapses, the
magma falls with it, forming a crater. Volcanic craters differ from
impact craters in appearance with their irregular shapes and presence
of volcanic matter.
-------------.
---------------------'

~\.:'1 .....- - 60. Chapter 3

! :<.;~
---:1

1;'J i.'
l

lI
1

I. . _Ty_p_e_1_:_ _ _ _ _ _]

L
..

_Ty_p_e_2_:_ _ _ _ _ _

Lunar.Craters

!::

!!l

E>pl,ooboo

__.H E<pl~"'"'

----'

1. What can be inferred from paragraph 1


about the moon?
(A) It used to orbit much closer to tbe
Earth.
(B) Its atmosphere disappeared after
formation.
(C) Its interior was very hot at one time.
may form there sometime
.(D)
Volcanoes
.
., in the future.

:'2. Which of the following can be interred


based on the information in paragraph 2?
(A) The process of crater creation
continues even now.
(B) Meteorite craters are more common
than asteroid craters.

i
Cl

(C) Meteorite impacts affect the


atmosphere of the moon.
(D) Asteroid impacts do not create raised
rings.

3. According to paragraph 3, what can be


interred about volcanic craters?

(A) They probably create strong


earthquakes on the moon's surface.
(B) Their formation relates to the lack of
a lunar atmosphere.
(C) They can be identified by their age.
(D) They would be more common on
Earth if it lacked a mantle.

Fill in the blanks to complete the summary.


The moon has accumulated a number of craters that form its jagged _ _ _ __
The

majority of lunar craters are caused by the impact of meteorites and

asteroids. Because the moon has no atmosphere, there is no protective

to

burn up meteorites before they hit the moon's surface. Therefore, when meteorites explode
upon impact, surface material is
are formed is when

with great force. Another way lunar craters


rock rises through the moon's surface, breaking it. A

cavity forms beneath the surface, which eventually collapses to form a crater.

Inference Questions

61 ---:

Zool<lgy
'~);i,1/'ktJc;~'.dV ,;,;,;,:,.-,!,,}~'-1!'_,,.,

-';',,;,'(,-:,.-,

;., :-.W:-:i'.-._-;_,

;<:,,,;,_,-, --;,-1 :, .

Read the following passage. Then fill in the diagram with the information that you read.

The wolf is the ancestor of all domestic dogs. Evidence from Israel
indicates that humans domesticated wolves over 12,000 years ago.
Today, dogs and wolves exist as a separate species, even though
their genetic makeup is nearly identical. While most dogs bear little
physical resemblance to the wolf, dogs retain many behavioral

~
'.'.!-
!. :.:

.:_,. '~i

Ii
'

characteristics common to wolves. Modern dogs display similarities


to wolves in social structures and in their displays of aggression.
The structure of the wolf pack can be seen amongst families of
dogs. Both species evolved as a group of social hunters who had to
work as a team to kill large animals. Wolves have hierarchical pack
structures, in which one wolf acts as the leader and other wolves are
expected to be submissive to the leader. Within families, the mother
and father are the alpha wolves, and pups are expected to obey.
Wolves form hierarchies not only within their families, but also within
their packs. Should they try to assert power, those at the bottom of
the scale of dominance risk being ostracized by pack leaders. The
hierarchy and teamwork of wolves can also be seen amongst dogs,
such as the sled-pulling Siberian huskies. The sled-driver acts as the
alpha male or female, giving instructions to the pack. The entire
group forms a pack hierarchy, wh'1ch is vital to the success of the
sled.
Both dogs and wolves are highly territorial and will show
aggressive behavior in order to assert power. Wolves and dogs
similarly curl their lips and snarl when attempting to intimidate or
threaten. Wolves may fight within their hierarchies, often until one wolf

+@

resemblance:
the state of being similar
to something
alpha wolf:
the wolf considered the
leader of the pack
ostracize:
to exclude from a group
territorial:
defensive of a particular
area

Inherent:
existing as a necessary
atlribute
belligerence:
the state of treating with
-disdain or aggression
bare:
to reveal or show

is severely injured. This struggle to show dominance is a common


theme in the history of both dogs and wolves. While domestication
could tame their wild natures to an extent, dogs have still retained
their inherent belligerence and suspicion of strangers, which humans
have learned to take advantage of for guarding, hunting, and sports.
While modern dogs do not howl in the same way that wolves do, their
vocal defense of territory shows many similarities to how wolves
defend their homes. Just as a, wolf bares its teeth and displays
aggression when threatened, a dog behaves similarly if it encounters
an unfamiliar or threatening animal.

------

--p--------~~---,------------'

1 .

w(1ves and Dogs . I

Theory:

Support 1:

<
1. According to paragraph 2, what can be
inferred about wolves?
(A) Both males and females can be pack
leaders.
(B) Wolf cubs often try to become the
alpha male.
'(C) Packs do not exhibit hierarchical
relationships.
!(
(D) The oldest wolf in a pack becomes
'the alpha wolf.

Support 2:

(C) Humans are harmful to both wolves


and sled dogs.
(D) Wolf hierarchy is not present in dogs.

3. According to paragraph 3, what does the

author imply about wolves?


(A)
(B)
(C)
(D)

They howl when they are happy.


They enjoy fighting.
They are better guards than dogs.
They are suspicious of strangers.

2. What can be inferred from paragraph 27

(A) Dogs do not naturally feel a sense of


teamwork.
(B) A sled driver is considered the most
powerful animal.

Fill in the blanks to complete the summary.

According to the passage, while dogs and wolves bear little physical - - - - - - to each other, they demonstrate common traits. For example, their social structures consist
of hierarchies within pacl<s or families. They are also both highly

and

become aggressive to assert dominance and ownership. Wolves and many dogs have a(n)
- - - - - - - - - - - - t o w a r d strangers. They react to a threat by
_ _ _ _ _ their teeth and displaying other aggressive behavior.

lrii2rence 0uestions

63 - - - 1
'

! .

Necessary Skills
~-'-U~-.1>-\.!7.;&l.~.ltl.1l:2J:;.,:,,./.WW~e_:,k>'(:,.,<,1y.t.H

:<.-'.. ;\/:<-.'.'-'?-

' '" -...< - '

'

Determining why the author has presented a piece of information in a certain


place or way within a passage
Understanding the role of a certain statement in a passage
Inferring the author's intention in mentioning certain information
Relating specific information to the main ideas of a passage in order to understand
the purpose of the information

The author discusses _ _ _ _ _ in paragraph X in order to


Why does the author mention
?
The author uses
as an example of

<

Learn and understand the meaning of certain words and phrases often used to
describe rhetorical purposes: definition, example, function, to illustrate, to explain,
to contrast, to refute, to note, to criticize.
Focus on logical links between sentences and paragraphs in a passage.

Rhetorical Pt1rpose Questions

65 _ _:,

literature.

i'

Read the following passage. Then fill in the diagram with the information that you read.

~.

. .

In literature, the theme is the general idea that is expressed


throughout the piece. The theme of a poem is rarely obvious, and it
may be difficult to interpret the theme right away. However, by paying
attention to certain elements in the poem, it is easier to discern what
the theme is.
The first element that can indicate theme is the title of the poem,
as it contains the first words the reader encounters. Therefore, it can
supply information about what the author wishes to express.
Consider the poem "A Boat beneath a Sunny Sky," by Lewis Carroll.
This title suggests a peaceful afternoon and happy times. From simply
analyzing the title, the reader can assume that the theme will involve
a joyful, tranquil experience.
Sometimes, however, the title is not enough to unearth the theme
of a poem. Another way to find the theme of a piece of literature is by
analyzing repeated words or images. In "A Boat beneath a Sunny
Sky," Carroll repeatedly mentions dreams: dreams that linger over the
evening and dreams that help the days go by. In the final line, Carroll
writes: "Life, what is it but a dream?" By analyzing the repeated
words, the reader can conclude that the theme of the poem is to
appreciate the dream-like qualities of life.

IL. _ _
ldentiiying
Them;!
____
__
:'.___j
1------..---..------..--------1
Step 1:

l_ ----- - -- - -"""''"'

Step 2:

+@flti

Interpret:
to find the meaning of
. something
discern:
to tell apart or distinguish
Indicate:
to show clearly
tranquil:
calm
unearth:
to discover by searching
thoroughly
linger:
to take a long time doing
something
conclude:
to decide upon

1. Why does the author discuss the title of a


poem?
(A) To demonstrate how to interpret a
poem's title
(B) To explain how to find the theme of
the piece
(C) To provide an example of poems with
long titles
(D) To argue that the theme is rarely
revealed in the title

3. The author uses dreams as an example of


(A) the ways that titles can be misleading
(B) how repeated words can reveal a
poem's theme
(C) the main theme of "A Boat beneath a
Sunny Sky"
(D) common themes in children's
literature

"O

2. Why does the author mention a peaceful


afternoon and happy times?
(A) To provide a theory about why the
author changed the title
(B) To refute a previous point about the
creation of titles
(C) To show what the title suggests about
,,. the poem's theme
(Qj To note the masterfulness of the poet
in describing the scene

"

Fill in the blanks to complete the summary.

The focus of the passage is on literary themes and how to

~----

themes in

poetry. A theme is the general idea that is expressed in a poem. The author describes two
ways of
often

themes in poetry. First, the reader can analyze a title, which


what the poem may be about For example, Lewis Carroll's "A Boat

beneath a Sunny Sky" suggests that the poem is about something joyful and _ _ _ __
Next, the reader should look for repeated words or images, as they often help the reader
_ _ _ _ _ the theme of the poem.

Rhetorical Purpose Questions 67

--:j

Read the following passage. Then fill in the diagram with the information that you read.~ +@tlM

'

"I

'

_.,:

i'

Meteorologists study weather patterns on the Earth. Many of the


spectacular phenomena that can be witnessed from the Earth do not
originate on our planet. Rather, some are caused by powerful sforms
in space, like Aurora Borealis. Aurora Borealis is an extra-planetary
storm that causes impressive lights to appear in the northern sky.
These lights are only visible from areas in the northern half of the
Earth.
Aurora Borealis begins when a cloud of particles is discharged
from the sun. The cloud, called plasma, travels through space very
quickly. The particles in the plasma are negatively charged, meaning
that they contain electrons. When the plasma approaches the Earth,
it comes into contact with atmospheric gases.
The gases in the atmosphere help protect the Earth from the
sun's rays, but when the charged particles collide with the gases,
electricity is transmitted. This is because the atmospheric gases
are good conductors of electricity. The collision of plasma and
atmospheric gases causes the atoms to become excited, and they
build up energy.
When the stored energy is released, the atoms emit light. The
spectrum of colors changes continuously during the storm. A green
glow can persist for hours, especially if there are no new collisions
between atmospheric gases and plasma. From higher altitudes, a red
glow is often witnessed.

How Aurora

L __J

spectacular:
exciting to look at
originate:
to come from
extra-planetary:
referring to something that
occurs outside of a planet
Impressive:
causing admiration
discharge:
to release or let out
colltde:
to crash into
spectrum:
a range of colors

Borealis~~ Fo~~:_Q
Step 3:
''

l
I

Description:

Description:

I'

I
l____,,~-------J

'---------1

1. The author describes Aurora Borealis as


an extra-planetary storm in order to
(A) emphasize that it is a massive and
powerful storm
(B) show that Aurora Borealis affects the
weather on Earth
(C) contrast Aurora Borealis with other
storms in outer space
(D) note meteorological phenomena
outside of the Earth

3. The author discusses altitude in


paragraph 4 in order to
(A) explain the effect of the Earth's
atmosphere on the plasma
(B) illustrate the height to which the
Aurora Borealis can be seen
(C) show that without electrons, the
storms would lack their colors
(D) describe a factor that affects the
colors of Aurora Borealis

[[I.

I
.
.

[
:E

"'3z

GI

2. Why does the author mention plasma?


(A) To give the proper scientific term
(B) To explain the importance of plasma
(C) To provide an example of solar
discharge
(D) To contend the role of plasma is
minimal

:, ~Fill in the blanks to complete the summary.


i --.. --------------------------------------------------------------------------------------------------------------------.
~

According to the passage, many of the

weather phenomena seen

from Earth are caused by powerful storms in space. For example, Aurora Borealis is an
extra-planetary storm that results in the appearance of

lights in the

northern sky. Aurora Borealis is caused when particles a r e - - - - - from the sun.
Then, the particles - - - - - w i t h gases in the Earth's atmosphere. In the process,
an electric charge is transferred to the gases. The collision causes energy to build up.
When the energy is released, the gases emit lights with a continuously changing
_ _ _ _ _ of colors.

Rhecorical Purpose Questions

69 - - ;

Er1vironmental Scier1ce
Read the following passage.1hen fill in the diagram with the information that you read. +Mflfij

The invention of automobiles in the 19'" century had a profound


effect on modern transportation. Further advancements in the
automotive industry have led to the development of the electric car.
Electric cars are able to function on electric power instead of
gasoline fuel. The merits of the electric car have been debated
since its invention. However, due to growing concerns about the
environmental costs and economic sustainability of gasoline fuel, the
electric car is being hailed as the best alternative available to
consumers today.
Most modern vehicles employ an internal combustion engine. In
- ....

order to run, the fuel reacts with air in a small space and produces
energy. When internal combustion engines first became popular,
there was little concern for the negative effects they might have on
the environment. Scientists have since learned that they produce
gases and fumes that contribute to air pollution. There are al.so
increasing concerns about the role of carbon dioxide emissions on
global warming. Many feel that automobiles are a major cause of the
problem. Electric cars are a good alternative to internal combustion
engines. They do not directly produce any harmful emissions.
According to a scientific study, electric cars reduce carbon dioxide
emissions by as much as one hundred percent, especially if solar or
wind power is the source of electrical energy.
In addition, electrical cars are also better for the economy. They
reduce dependence on petroleum, a non-renewable energy source
that powers most internal combustion engines. As the world's supplies
of petroleum are depleted, it becomes increasingly expensive for
consumers. The fluctuating price of petroleum can cause already
unstable economies to collapse. In contrast, electric cars rely less on
non-renewable energy sources. The increased use of electric cars
lessens the importance of petroleum in the world economy. Moreover,
since it is less expensive to produce electricity than gasoline, the cost
of operating an electric car is more affordable for consumers.

profound:
in an extreme way
merit:
an advantage
sustalnablllty:
the ability to be
maintained
fume:
a harmful vapor or gas
emission:
a substance released into
the air
fluctuate:
to change constantly
collapse:
to fail

--------- ...---------------'

Electric Cars

Theory:

ISupport 1:

Support 2:

J~

S"pport 3

~~~~~~~~~~~~~

t=i

1. Why does the author mention internal


(A) To explain how electric cars function
(B) To criticize the amount of fuel used
(C) To describe the most common type of
"
engine
(D) To note an advantage of non-electric
cars
tJ,,

2; The author discusses carbon dioxide


emissions in order to
(A) argue that electricc9,rs are Q~tter for
the environment
(B) note that they can positively impact
the ozone layer

(C) assert that electric cars do not help


lower emissions
(D) define how environmental standards
are tested

combustion engines?

3. The author uses solar or wind power as


an example of
(A) energy used by internal combustion
engines
(B) a kind of renewable energy used to
run electric cars
(C) a way to make cars run more
efficiently
(D) a type of non-renewable energy for
cars

Fill in the blanks to complete the summary.


The invention of the automobile has had a(n) _ _ _ _ _ efiect on modern transportation.
The passage describes the _ _ _ _ _ of the electric car and its benefits to the environment
and ihe economy. Gasoline engines produce gases and _ _ _ _ _ that contribute to
global warming. Electric cars, meanwhile, can reduce

by one hundred percent.

Electric cars are also better because they reduce reliance on petroleum. Decreased
dependence reduces the impact of

petroleum prices on the economy.

Rhetoncal Purpcse Q1Jestions

71

-~-,

Biolog.y ,,
"'.UMifi!l.WWl:.<'!H\=<'..'l'!SlU111.,k'< '-' D ;>;."'.~.,~,.~;,.).'

';

}"'- ~-../ _,_ )

Read the following passage. Then fill in the diagram with the information that you read. i@fi+

:1

Altruism is selfless behavior that aims t6' benefit others. It has


been examined by philosophers, sociologists, and even biologists.
For some biologists, altruistic behavior.is strange because it seems
to defy Darwin's theory of natural selection. In Darwin's theory, a
species is able to continue to survive based on how well adapted it
is to the environment. Organisms that possess features to better help
them survive are generally regarded as the strongest of the species.
But how does a species benefit from altruism? Kin selection theory
proposes that altruism is an evolutionary mechanism that promotes
species survival by self-sacrifice.
Kin selection is the idea .that an organism will place the
reproductive success or survival of its relatives above its own in order
to help the species. Altruistic behavior that results in the evolutionary
success of related organisms is called kin altruism. The theory was
first explored by evolutionary biologist W.D. Hamilton, who first
observed the phenomenon occurring in social insects, such as ants.
He later prepared a mathematical formula, called Hamilton's Rule,
which explained how kin selection benefits the genetic fitness of a
species. According to Hamilton's Rule, the value of kin altruism can
be analyzed in terms of how it either helps or hurts the entire species.
Kin altruism is best illustrated by the bee, a species believed to
exercise kin selection. In a hive, there are different types of bees:
worker bees, drones, and the queen. Each type has a function that
helps the colony survive. The workers collect food and help maintain
the hive, the drones defend the hive from enemies, and the queen
gives birth to new bees. These types are thought to have evolved
because of kin altruism, which favors the queen for reproduction.
The workers forgo their ability to reproduce. Even so, by providing
safe conditions for the queen, they are able to guarantee the
continued survival of the species.

selfless:
unselfish
defy:

to go against
fitness:
the ability for a species to
survive
exercise:
to make use of
colony:
a group of animals of the
same species that live
together
forgo:
to give up

'
'
!

Altruism and Evolution:,

[I_.

Theory:

Explanation:

Example:

GI

..."'
~
i3
z

I
'

GI

:e

3z"'

GI

...

1. Why does the author mention Darwin?


(A) To introduce the topic of altruism in
evolution
(B) To criticize Darwin's theory of natural
selection
: (C) To question whether altruism and
.,., evolution are related
(D) To provide background information
about kin selection

2. The author discusses Hamilton's Rule in


order to
(A) discredit the scientist's ideas about
kin selection
(B) explain how kin altruism can benefit a
species

(C) note its importance in understanding


altruism
(D) show why some animals favor their
relatives

"'
m

3. The author uses bees as an example of


(A) animals that have not evolved for
many years
(B) why altruism is bad for evolutionary
success
(C) how animals that practice kin selection
get hurt
(D) animals that benefit from kin~selection

Fill in the blanks to complete the summary.

The passage discusses altruism in biology. Altruism, or _ _ _ _ _ behavior, is


considered an evolutionary mechanism that seems to
selection. Some scientists believe that by
promote the genetic

the theory of natural


kin selection, some animals

of their species by sacrificing themselves. For

example, worker bees _ _ _ _ _ their ability to reproduce so that they can protect the
hive and their queen.
--------o>---.--.. --------.,--.. -----------------.. --------------------.. ----,---------------------------Rhetorical Purpose Questions

73 - -

Oceanography
.

~1.:~ll'.'.:'.1-'PliW.i11>."'"',;i;.r""A"'.1'u,u~,,,,_,~~:\\;~-<-rJ,.<-."--~'-"'-'.Y.-,__.,, -~:,.

'

Read the following passage. Then fill in the diagram with the information that you rea'~.

Rogue waves are extremely large waves that are more than double
the average height of most waves. According to mathematical
calculations and various personal accounts, rogue waves can reach
remarkable heights. They appear unexpectedly in calm waters and
can do major damage, even to large ships. Unlike tsunamis, which
are practically undetectable in deep water, rogue waves only occur
far out at sea. Stories about rogue waves have circulated amongst
sailors for centuries, but it was not until recently that scientists
confirmed that they actually exist. What they still are not sure of,
however, is what causes them.
Some instances of rogue waves have been explained by the
interactions of normal wave patterns with ocean currents. Scientists
believe that it is possible for waves to reach the heights described
when they come into contact with strong ocean currents. The wave
heights increase significantly when a normal wave reaches a current
head on. In other words, the wave is built up by the power of the
current. This explanation was first proposed after scientists observed
a high incidence of rogue waves in the ocean surrounding the
southern tip of Africa. Intact, since 1990, at least twenty ships have
encountered the waves, which reportedly reached up to 190 feet. The
waves are thought to be caused by wave interactions with the strong
Agulhas Current, which runs southbound along the east coast of the
continent.
Ocean currents may be responsible for rogue waves in some
parts of the world, but scientists have confirmed their existence even
in areas that are not affected by strong currents. In those cases,
scientists think that the waves are caused by wave reinforcement.
Wave reinforcement is when two or more waves join together to form
one massive wave. When the waves are joined, each height is added
to the others. For example, if a ten-foot wave comes into contact with
a fifteen-foot wave, the resulting wave will be twenty-five feet tall.
According to some evidence, it is possible that many waves can join
together, which would create rogue waves. Scientists still do not
understand which circumstances cause wave reinforcement, but
many propose that the reason rogue waves appear suddenly is
because they are formed by multiple smaller waves randomly.

*@11

account:
a description of an event
undetectable:
unable to be noticed or
observed
circulate:
to pass from one person
to another
current:
a steady flow of water in
one direction
Incidence:
the rate at which
something happens
massive:
very large
randomly:
occurring without any
pattern

.,'

;'

1. Why does the author mention tsunamis?


(A) To explain why rogue waves cannot
be predicted
(B) To describe the similar mechanism
that causes rogue waves
(C) To note that rogue waves and
tsunamis are unrelated
/' (D) To show that rogue waves are
'' comparable to tsunamis

2. The author uses the Agulhas Current as


an example of
(A) how currents become stronger
through their contact with ocean waves
(B) how multiple waves are built up by
ocean currents

(C) a current that flows in the northern


region of Africa
(D) a current that does not affect the
wave height in the region

3. The author discusses


in order to

wave reinforcement

(A) propose another theory for what


causes rogue waves
(B) suggest that ocean currents cannot
be responsible for rogue waves
(C) demonstrate how rogue waves are
created closer to land
(D) explain how multiple waves are able
to interact at sea

Fill in the blanks to complete the summary.

According to many personal

and mathematical calculations, rogue waves

can reach incredible heights. Stories of rogue waves have

for centuries, but

scientist still are not sure what causes them. A high _ _ _ _ _ of rogue waves off the
southern tip of Africa seems to be caused by contact between normal wave patterns and
ocean currents. In other areas, rogue waves may be caused by two or more waves joining
together to form one

wave. However, the exact circumstances in which

rogue waves appear are still unknown, and in fact, they may sometimes occur _ _ _ __
-------hn .... -------------u-- '''""'" ''"""----,. _
- ' "'' ' ,.,, <"------... ,,_.,,._,, ________ ,----- --- --u-.,--------------
_._.,_~

K.hetorical Puroose Questions

75 - - .

Anthropology.
..,v .. :-'.- - --

~1t..~)Ot.~~.~.=~v,;01>:i.'1M:Zl-l!'.Jf,,e<,~,,-,J.Vi;:,~;,-11

Read the following passage. Then fill in the diagram with the.information that you read. +@*fl

Artifacts are ancient objects that have been recovered through


archaeological endeavors. Anthropologists study the cultural artifacts
left by older societies to gain valuable knowledge about their ways of
life. When studying Native American cultures, one common type of
artifact is pottery, which can be found all over the continent. By studying
the pottery found in a region, anthropologists have been able to learn
about ancient techniques, as well as the functions of pottery for
Native American peoples. Years of recovering and studying pottery
have revealed many differences between the pottery made by people
who lived in the southwestern portion of what is now the United States
and those who lived in the northeastern region.
Southwestern pottery remains a popular and revered art form. The
styles featured in art galleries and museums today have an extensive
history. Long before Columbus arrived, southwestern cultures had
formed techniques to produce pieces that were beautiful, long-lasting,
and functional. Using clay commonly found in the, region, Native
Americans first created a type of pottery called bisque, which means
that it was subjected to extremely hot temperatures to make the material
harder, but was not glazed. The pottery was plainly adorned, if at all,
suggesting that it held more of a functional purpose than ritualistic.
The pottery of the northeast differed from southwestern pottery in
many respects. First, due to the disparate geography, the materials
used varied greatly between groups. Rather than just using dried
clay, northei:istern peoples mixed clay with other materials that were
available, such i:is crushed shells, sand, and plants. Next, the form of
northeastern pottery is completely distinctive. Instead of solid pots
like those made in the southwest, potters in the northeast formed a
rope made of prepared clay. The rope was wound around a circular
base, forming coils as the piece was built up. Finally, after the coil
pottery was fired, it was further treated with special stones. The
natives rubbed the piece with the stones in order to smooth the
surface. This final treatment added a polished look to the piece.

recover:
to get something back
revere:
to greatly respect
extensive:
having a great range
subject:
to put something through
glazed:
covered with a layer used
to protect the piece
disparate:
different
fire:
to heat objects made
clay to make them harder

of

:t.

~~~/,...:.--

76

Chapter 4

Iii -

Native American Pottery

'--T-yp_e_1:_ _ _ __. ___. .__M_a-te-ri-a-ls-:_ _ ___.

---+

ITechoiqco

.__Ty_p_e_2_:_ _ _ __. ___. .__M_a-te-ri-a-ls-:- - - - - '

---+

1. Why does the author discuss southwestern


and northeastern artifacts?
(A) To contrast two types of pottery
and their production processes
(B) To present a key point in the
definition of artifacts
() To show the artifacts usually found
~
~
' by scientists
' (D) To explain how different groups
,; influenced each other

2. The author discusses southwestern pot


decoration in paragraph 2 in order to

Tooholqco

(C) explain what types of uses they had


(D) give an example of typical
Southwestern pottery

3. The author mentions the stones used in


the northeast to describe
(A) the reason southwestern pots were
less smooth
(B) the process of decorating the pots
(C) the diverse functions of pots in daily
life
(D) the method used to polish pots

(A) criticize the artistic talents ofthe potters


(B) demonstrate which designs were used

Fill in the blanks to complete the summary.


The passage discusses artifacts
northeastern parts of the United States. A popular and

in the southwestern and the


art form, the pottery

of southwestern Native Americans has a(n)

history. In the beginning, they

created more functional pottery out of clay,

it to extremely hot

temperatures. Northeastern pottery featured many different materials due to _ _ _ __


geography. Native Americans in the northeast also finished pots with stones to add a
polished look.

Rhetorical Purpose QuBstions

77 - -....

Instructions: Choose the best word or


phrase to complete each sentence.

1. After their near defeat on the battlefield,


the corporal brought in more troops to
_____ the unit's ranks.
(A) broach
(8) reinforce
(C) eject
(0) circulate
2. Volunteering to work with the animals at
the zoo gave the biology student some
good
experience.
!'.

(A)
(B)
(C)
(0)

tangible
turbulent
illustrious
hands-on

3. The opening ceremonies featured several


hundred performers singing and dancing
in _ _ _ __
(A)
(B)
(C)
(0)

string
focal
unison
stratification

4. Although the music was really loud, I


could understand the
of
what he said from his body language.
(A)
(B)
(C)
(O)

outcome
merit
chunk
gist

5. It was inspiring to see how well the victim


_____ after losing her daughter
and brother in the crash ..
(A) coped
(B) utilized
(C) settled
(0) bared

6. A common obstacle in intercultural


relationships is overcoming the
language _____.
(A)
(8)
(C)
(0)

terrain
compound
barrier
integration

7. The professor gave a very _____


review before the exam to ensure the
students understood the material.
(A)
(B)
(C)
(0)

undetectable
viable
embellished
comprehensive

8. Young sea turtles, especially those that


have just hatched, are particularly
---~-to predators like birds,
sharks, and monitor lizards.

. (A)
(8)
(C)
(0)

turbulent
vulnerable
innocent
profound
,,

Instructions: Choose the word or phrase


closest in meaning to the underlined part
of each sentence.
9. Because of the widespread damage
done to the area by the earthquake,
the government declared a state of
emergency.
(A)
(8)
(C)
(0)

impressive
novel
extensive
prolonged

'

. -;_-",--

;-.-'

10. The dentist administered laughing gas


to her patient in order to ease his
discomfort.
(A) alleviate
(B) integrate
(C) devastate
(D) generate
11. It was time for the accused to give her
version of events that occurred on the
night of the robbery.
(A) incidence
(B) account
(C) emission
(D) avenue
12. In many western cultures, the white lily
is _a flower that is representative of purity
and sweetness.
(A) verbatim
, (B) universal
(C) advantageous
(D) symbolic
13. Although they were twins, the girls bore
very little physical likeness to each other.

(A)
(B)
(C)
(D)

component
resemblance
specimen
temperament

14. The reporter's weekly review on


Hollywood stars and gossip was the
network's most popular show.
(A) commentary
(B) expenditure
(C) revenue
(D) impetus

15. When the body is stressed or frightened,


adrenaline travels throughout all parts of
the body in the blood, keeping energy
levels high.
(A) ostracizes
(B) interprets
(C) circulates
(D) originates
Instructions: Write the missing words. Use
the words below to fill in the blanks.
discharge speetacular
gap

indication

phenomenon

A thunderstorm is a 16. _ _ _ _ __
. produced during a cold front and usually
lasts an hour or less. The lightning is often
considered the most 17. _ _ _ _ __
part of a storm. It is a 18. _ _ _ _ __
of electricity between clouds and the ground.
When lightning occurs, it instantly heats the
air around it. The hot air expands quickly.
After the lightning disappears, the air cools
and contracts. This process of expansion and
contraction produces sound waves heard as
thunde(. Counting the 19. _ _ _ _ __
between the lighting and the thunder can give
an 20.
of how far away the
storm is.

Instructions: Match the words that are similar


in meaning.

21.
22.
23.
24.
25.

depletion
uniform
hypothetical
prospective
transpire

(A)
(B)
(C)
(D)
(E)

even
potential
reduction
happen
imaginary

Vocabulary Review 1

79 - - -

J.J1 Enviror1mental Science


Read the passage and answer the questions.

'.'-

l@ili

When carbon dioxide (C02) is released into the air, it acts as a blanket. It keeps heat in
the Earth's atmosphere and produces global warming. Global warming causes the Earth's ice
to melt, sea levels to rise, and precipitation levels to change. What many do not know is that
nat.ure has its own method of counteracting the increased release of carbon into the
atmosphere. This takes the form of what climatologists call the carbon sink effect. A carbon
sink is anything that acts as a storage area or reservoir for carbon. Oceans and areas with
vegetation are natural carbon sinks because they absorb carbon from the air.
The Earth's oceans and areas with growing vegetation are natural carbon sinks. At least
one-third of the alllbropogenic carbon released on Earth is absorbed by the world's oceans.
Young forests with actively growing trees are another area where the carbon sink effect is
easily seen. Oceans and forests become carbon sinks through the process of photosynthesis.
Photosynthesis occurs when plants take in carbon dioxide for nutrients and, in turn, release
oxygen back into the environment. Phytoplankton in the ocean and all the vegetation in the
forests are responsible for absorbing carbon dioxide from the air. Carbon can also be
absorbed into much of the Earth's soil in the form of organic material, especially in agricultural
areas.
Since it is important to reduce the amount of C02 in the air, scientists hope to increase
nature's ability to form these carbon sinks rather than try to make artificial sinks. Artificial sinks
require that carbon first be captured and stored, adding steps and expense to the process.
Reforestation is a less expensive way to fight global warming with carbon sinks. The addition
of iron oxide or iron sulfate to the world's oceans is another method of increasing carbon
absorption. Adding these iron byproducts encourages the growth of plankton. Plankton
boosts the amount of carbon absorbed through photosynthesis. As more carbon is absorbed
back into nature, less is found in the atmosphere, and global warming attributed to carbon
gas is greatly reduced.

fmthropog1:mic produced by humans

-------------'

\ ..

_-()}i
f~~//----

~."!'j.l.'

;~t

- ;,

80

Mini Test 1

'

\!.1,'o .- '~~;!lti!li!W~'W#'lV>'W-,t--",;,,

1. Why does the author' mention a blanket in


paragraph 1?
(A) To argue that C02 is responsible for
the carbon sink effect
(B) To give an explanation of the function
of C02
(C) To note that the Earth has too much
C02
(D) To illustrate how C02 affects Earth's
temperature
2. According to paragraph 1, how does C02
contribute to global warming?
(A) It causes the Earth's ice to melt and
sea levels to rise.
(B) It allows more of the sun's heat to
enter the atmosphere.
(C) It reduces precipitation in the air.
6\D) It traps heat in the atmosphere.

'

'"

t"a. Which of the following can be inferred


from paragraphs 1 and 27

(A) C02 is the only cause of global


warming.
(B) More carbon sinks would eliminate
global warming.
(C) Carbon sinks cannot form in cities.
(D) Melting ice contributes to global
warming.

c:
...."'

5. According to paragraph 3, what will


reduce the amount of carbon in the air?
(A) Cutting down on harmful emissions
(B) Reducing effects of photosynthesis
(C) Cultivating healthy soil
(D) Creating more natural carbon sinks

6. All of the following are true about carbon


sinks EXCEPT:
(A) They are storage areas for excess
carbon.
(B) They can reduce C02 in the
environment.
(C) They are found mainly in areas with
rich soil.
(D) They can be created through
photosynthesis.

z"'

"'
"'\::,,

'
I

15

"'
:;;

:!"'z

"'

,,

"'....

"'"'....

7. Which of the following can be inferred


about creating artificial carbon sinks?
(A) Artificial sinks are more beneficial
for the environment.
(B) Natural sinks are preferred over
artificial sinks.
(C) Artificial sinks do not absorb as much
carbon.
(D) Natural sinks absorb more than
enough carbon.
8. The author discusses plankton in

4. Which of the following is NOT true about

nature's method of carbon sink formation?


(A) Oceans absorb over one third of the
carbon released by human actions.
(B) The soil in agricultural areas absorbs
the majority of the Earth's carbon.
(C) Forests with young vegetation absorb
more carbon than older forests.
(D) Photosynthesis allows plants to take
carbon from the environment.

paragraph 3 in order to
(A) refute the effectiveness of artificial
carbon sinks in the world's oceans
(B) explain the role of photosynthesis
in removing carbon from the
atmosphere
(C) illustrate how adding iron byproducts
increases carbon absorption
(D) criticize the methods scientists are
using to try to stop global warming

Mini Test 1

81 - -

02. Marketing
Read the passage and answer the questions. i@fii

.,

Advertising makes an immense difference in the popularity of a product. Studies have


shown that consumers usually pay more for advertised products than non-advertised
products. As a result, the marketing and advertising industry is a multibillion-dollar-a-year
business. Local commercials can cost as little as several hundred dollars to produce, while
national companies have been known to spend hundreds of thousands of dollars on a single
commercial. Interestingly, the source for these expensive commercials is the consumers who
view the advertisements from the comfort of their living rooms. Companies influence the
prices that consumers pay in two ways.
When consumers purchase any product, they are obviously paying for the cost of
manufacturing the product. However, they are also paying for the advertising campaign
devised to convince consumers to buy the product. The more costly the advertising campaig~,
the higher the price paid by consumers. Companies are allowed to set these high prices
because consumers have shown that they will conti,nue to pay them. For instance, a multinational
chain of coffee shops significantly raised its prices a few years ago, due in no small part to
an expensive national advertising campaign. While. regular customers noticed and were not
fond of the hike, one was quoted as saying, "I doubt it will make an impact [on consumers].
It won't change my habit." Indeed, it did not; the company experienced record sales in spite
of the fact its prices were higher than its competitors'.
Another way advertising tends to affect product pricing is seen in the arena of perceived
value to the consumers. The perceived value is the worth that consumers assign ,to a
product. Studies show that often the advertising for a product determines its vah.ie more than
the quality or necessity of the product itself. One way a company may take advantage of this
perceived value is by running commercials during prime time. These ads are usually much
more expensive than others. For example, a national commercial that ran during a popular
American show cost close to $745,000 to run for thirty seconds. If people watching the show
perceive that the product is more valuable simply because it was advertised during prime
time, they are often willing to pay higher prices. So, whether it is seen in the cost of the
commercials or the advertiser-assigned value, it is obvious that advertising drives product
pricing and directly affects consumers.
r,. campaign a planned and organized series of actions intended to achieve a specific goal

M;;:~~~;q,;;ti;F.l.,'.

:> .,.

1. Which of the following can be inferred


about pricing and advertising?
(A) Products that are not advertised
cost more than advertised products.
(B) Customers will pay little for a
non-advertised product.
(C) Companies selling higher-priced
products usually advertise more.
(D) Local companies will not spend
money on advertising.

t'
~

--1

~:_.

,-':

,.

2. How do companies determine the price


they charge for a product?
(A) By determining the overall costs of
making the product
(B) By basing their prices on advertising
costs and competitor prices
(C) By choosing perceived value or
?'
manufacturing costs
.,,_
(D) By including the cost of advertising
in the overall costs

,,,

{-:

c'3, Which of the following is true about the


effect of higher-priced ad campaigns?
(A) They increase the price consumers
must pay.
(B) They are a response to consumer
demand.
(C) They result in competitor price
matching.
(D) They have positive effects on
customers.

4. The author uses the chain of coffee shops


as an example of
(A) the competitive nature of the
advertising industry
(B) a means by which large businesses
deceive consumers
(C) the acceptance of higher costs by
consumers
(D) a costly advertising campaign that
failed

5. According to the passage, which of the

"'

following is NOT true about the coffee


chain's price increase?

,,"'

(A) Many customers stopped buying


their coffee there.
(B) The coffee chain raised prices so
they could advertise more.
(C) Customers acknowledged the
change in prices.
(D) The price increase did not change
customers' drinking habits.

~
z

Q
Cl

:e

"'3
z

Cl

,,

6. Which of the following can be inferred


from paragraph 3?

"'m_,

!!I

(A) Price is not the only factor affecting


buyer decisions.
(B) Consumers will pay less when they
see expensive advertisements.
(C) The lowest-priced products are
usually purchased the most.
(D) Consumers become angry when
prices are raised too high.

7. The author mentions commercials


running during prime time in order to
(A) contrast effective advertising with
unsuccessful products
(B) provide a function of television
advertising on a national level
(C) refute the idea that consumers
pay more for products they see on
television
(D) explain how consumers justify
paying higher prices for products

8. Which of the following is NOT true about


the
(A)
(B)
(C)

perceived value of a product?


It can be higher than the actual value.
It is influenced by advertising.
It is partially determined by the
consumer.
(D) It is the same as the true value.

Mini Test 1

83

---.~

Necessary Skills
~l.1!.l>l::~;.:i:c(tl.J:._;.u;..l.l~'l'l)!;!,\;;.ilir;;\'JlliS'.(l!l'.<i~.<Z~:~,-,c.:~-''''(''<'c'.\.

'.'.

J '. '"'- .,.

Identifying the meaning of individual words and phrases as they are used in a
specific reading passage
Choosing the correct meaning of a word or phrase in order to understand its
relevance within a passage

Example Questions
The word _ _ _ _ _ in the passage is closest in meaning to
In stating
, the author means that

Strategies
Do not choose an answer only because it may be a correct meaning of the word
or phrase: choose the meaning that is being used in the passage.
Try rereading the sentence in which the vocabulary word appears, substituting
the answer choices for the word.

Vocabulary Questions

85 - - . :

.. ,.

Li r'g uistics
~1W!l"/$1A:~rN::'.l'M~tl::.:1w~,_,,o;i'.'U;.{f.-"P"

'1--0-.i.'-:<:;l,<':.: ,:,~

~t-"':1'<-

.,

Read the following passage. Then fill in the diagram with the information that you read. +@fl:M

Communication between humans is largely based upon language.


Learning a new language involves many different processes. One
step is to learn and build a vocabulary. Vocabulary is comprised of
the different words used by speakers to express themselves. Linguists
recognize two different types of vocabulary: passive and active.
The term passive vocabulary refers to the set of words that one
has learned or is familiar with. Words that comprise a person's passive
vocabulary are used infrequently for several reasons. First, their
definitions are not always completely known. In other cases, speakers
avoid certain words or terms that they do not need to use often in
conversation. For most people, their passive vocabulary is much
larger than their active vocabulary. This is because passive vocabulary
continues to expand throughout a person's lifetime.
Active vocabulary is the set of words that a person can produce,
such as when speaking or writing. Many linguists agree that a
person's active vocabulary is usually smaller than his or her passive
vocabulary. This is because words are only added to the active
vocabulary as they are needed. In other words, a person will not add
a word to active vocabulary unless he or she needs it to communicate.

linguist:
a scholar who sludies
language
passive:
not actively in use
avoid:
to stay away from
lifetime:
a period of time that
someone lives
produce:
to make something

,,,_,_J.--

- - - -... ,.______

--

jTYP;~----1

I
I
:

- .

<
<

Vocabulary
.

1Poi nt1:_______ -------- - -- .-----.. - . . - -1


L____,___. ____ .. '- --..---------..----..---- .. . . ..._J

J
--------------------------------]

-Pci::~::~:-::-:::::::-::::::~:_______:_ .::_ :_ _1Pol~;1:--

l_________________ ._____________________. ___

------- ----------------- 1

i.

1-P~i~~;-

i
I_. ____.... _.._ ................ -------------

------------~-~--~-~

_J

1. The word comprised in the passage


is closest in meaning to
(A) learned
(B) used
(C) formed
(D) selected

3. The word expand in the passage is


closest in meaning to
(A) grow
(B) understand
(0) undermine
(D) overtake

Ill
"O

~z

en

:;;

2. The word infrequently in the passage


is closest in meaning to
(A) not correctly
(B) not often
(C) not in a useful way
(D) not in all situations

"'z3
en

Fill in the blanks to complete the summary.

The passage describes two different types of vocabulary that _ _ _ _ _ recognize.

' . Passive vocabulary is the set of words that a person knows but
.,,

using in

:;, s~eech or writing. A person's _ _ _ _ _ vocabulary expands throughout his or her


~----

Active vocabulary, on the other hand, is the set of words that a person can

actually _ _ _ _ _ when speaking or writing.

Vocabulary Questions

87 - - . '

Biology
~~cllliffii;(.11J'."llr.~~;;;>E;;i,).f),.0C:;.;{:~''"'"''-''"''

'
Re~d

."

the following passage. Then fill in the diagram with the information that you read .. @!tk'

Angiosperms are the most common type of plant on the planet. All
plants classified as angiosperms share a number of traits. The most
distinguishing feature of angiosperms, however, is undoubtedly that
they produce flowers. At one point, angiosperms cohabited the globe
with the dinosaurs. However, unlike dinosaurs, angiosperms nofonly
survived, but thrived.
By studying the fossilized remains of older plants, scientists know that
angiosperms evolved to their current form over millions of years. The
earliest angiosperm fossil is estimated to be 130 million years old. Other
evidence suggests that angiosperm traits existed in plants 250 million
years ago. The fossils come from a type of plant called gigantopterids,
which resembled fems. Like angiosperms, the gigantopterid fossils
were found to contain a chemical used to protect flowers. This led some
scientists to believe that the gigantopterids are the early ancestors of
angiosperms-a "missing link" between ancient plants and modern
flowers.
The fossil record of angiosperms demonstrates how they diffused
across the globe by the mid-Cretaceous epoch, about one hundred
million years ago. During this period, the varieties of angiosperms
also increased. With the help of bees to spread pollen, angiosperms
became the most common type of plant on Earth by the Campanian
stage, eighty million years ago.
l--

undoubtedly:
accepted as truth
thrive:
to do well
fossil:
a trace of an animal or
plant preserved in stone
estimate:
to guess or approximate

ancestor:
something from which
others evolved
diffuse:
to spread

'

-----------

,--------.
. ----..----"l
Angiosperms

i
'

------"'"'"--------- - _,~---------- ---- ~

r-----------------. !

L-------------~~--- J

L... ----~,, .....

c ......

~----------~-J

80 Million Years
Ago:

1. The word cohabited in the passage is


closest ill meaning to
(A) lived together
(B) did often
(C) journeyed
(D) continued

3. The word pollen in the passage is closest


in meaning to
(A) a common plant disease
(B) powder from flowers
(C) early types of flowers
(D) leaves and stems

r[

2. The word epoch in the passage is closest


in meaning to
(A) variety
(B) age
(C) climate
(D) change

Fill in the blanks to complete the summary.


,,,.

"

'

The passage discusses angiosperms, flowering plants that continued to _ _ _ __


long after dinosaurs became extinct. The earliest angiosperm
be130 million years old. It is also

is thought to

that angiosperms began to evolve

about 250 million years ago from the gigantopterids. Gigantopterids may be the earliest
_ _ _ _ _ of angiosperms. Bees helped the plants

across the world

by spreading the plants' pollen. Angiosperms are now the most common type of plant on
Earth.

Vocabulary Questions 89 - -

Economics
'
Read the following passage. Then fill in the diagram with the information
that you read.

In economics, inflation is the process that leads to increased


prices for all goods and services. Generally, inflation is seen as
negative because it lowers the value of money. For example, after
inflation, one dollar cannot buy as much as it did before inflation.
Many factors contribute to inflation, and economists have formulated
several theories to explain why it happens. Two popular theories of
inflation are demand-pull inflation and cost-push inflation.
The theory of demand-pull inflation is associated with John
Maynard Keynes. Keynes was an important 20'"-century economist.
According to Keynesian economics, demand-pull inflation occurs
when the demand for a product, or good, increases. Increased
demand means that the product will eventually become scarce. With
less of the product available, it costs more for people to purchase it.
Thus, consumers .effectively bid up the price of the product. Indeed,
demand-pull inflation is usually described as a situation in which "too
much money is spent chasing too few goods." Economists have

@!"'

effectlvely:
in effect
Isolate:
to separate something
from its other parts
export:
to send goods to another
country to sell
production:
the process of making
things to be sold
profitable:
able to make money
raw materials:
goods that are not
manufactured
Import:
to buy products from
another country

isolated several factors that cause demand-pull inflation, such as


increases in the supply of currency, in government purchases, or in
exports.
Cost-push inflation also originates in Keynesian economics. The
theory holds that inflation occurs when the price of production rises
significantly. If it costs more to make something, companies must
increase the price they charge the consumer in order to remain
profitable. This situation is commonly seen when companies increase
their employees' wages. If businesses must pay their workers more,
they must in turn charge more for the goods or services they
produce. Another cause of cost-push inflation is seen when the price
of raw materials rises. For instance, if it costs a company more to
import metals used in manufacturing, it must pass this increase on to
the consumer.
~--------~--~------------~-----------------------

-m 90 Chapter 5

''

Inflation .

---'-----,

[TYPS 1:

- ,

Type 2:

[~

"'

---------

1. The word formulated in the passage is


closest in meaning to
(A)
(B)
(C)
,,,(D)

Definition:

Definition:

researched
decreased
conceived
argued

Definition:

3. The word wages in the passage is closest


in meaning to
(A)
(B)
(C)
(D)

price of materials
money paid to workers
number of employees
kind of workplace

"'

...
2. The word scarce in the passage is
closest in meaning to
(A)
(B)
(C)
(D)

expensive
frightening
popular
not easy to find

Fill in the blanks to complete the summary.


The passage discusses two economic theories explaining inflation. The first is demand-pull
inflation. This is when supplies are unable to meet consumer demand. By their willingness to
pay more for the product, consumers _ _ _ _ _ _ _ bid up the price. Factors causing
demand-pull inflation include an increase in currency or
of inflation is cost-push inflation. This occurs when the

. The second type


price of

something increases. Companies must charge consumers more in order to keep the
businesses _ _ _ _ _ . Cost-push inflation can also be caused by an increase in the
price o f - - - - - - - - - - - - - - ~.,-- -0< ,.,------------'""''"'- '" -- -----.,~------- .. " - - - - - - - - - - - - - ....... - ...... ., ____ .......... _ ----- -- " - - - - - - - -

Vuca!Julary Questions

91

--1

'

Photography
=r,u~;r;;>JJ>1::..:i\J!l\>'-(;;:rr...i:Tw<0tn~,z,z;c:.-,c_::"'";;:.)_

,, ___ ;_ ,

'

'
Read the following passage. Then fill in the diagram with the infqrmation
that you read. 'I"

In photography, contrast is the degree to which the tones in the


picture differ. A high-contrast black-and-white photo, for example,
will feature mostly black and white objects with few grey tones. A
low-contrast photo, meanwhile, will have very little tonal variation.
A photographer who has paid proper attention to contrast in his or
her work will direct a viewer's attention to the most interesting aspect
of the photograph. Also, the picture will be more pleasurable overall
to look at. There are many different techniques that a photographer
can employ to achieve effective contrast in a photo. First, special
care must be taken to provide adequate and appropriate lighting.
Another way to ensure proper contrast is to regulate the amount of
time the film is exposed to light.
When deciding how to light a scene, the photographer must consider
how the lighting will affect the contrast on fiim. Too little lighting and
the tonal differences may not be apparent in the photograph. On the
other hand, too much lighting may make the subjectappear washed
out. If relying on natural light, special equipment called filters can be
used to improve contrast. Filters are pieces of colored glass that are
usually screwed on to the end of the lens. When properly used, the
filter can make certain colors appear brighter and others duller.
Aside from determining appropriate lighting for a scene, pleasing
contrast can also be achieved. This can be done by paying special
attention to the amount of light the film is exposed to when actually
taking the picture. In photography, this is called exposure. Exposure
can be manipulated by changing either the aperture (the opening in
the lens that allows the light to come in) or the shutter speed (the
speed at which the window in the camera remains open). Fortunately,
choosing the proper exposure time does not take a great deal of
technical knowledge of cameras. In fact, by experimenting with the
camera, even an amateur can figure out appropriate exposure times
for different subjects.
If a photographer is unsure of how long to expose the photo, he
or she can also employ a technique called bracketing. Bracketing is
when a subject is photographed multiple times using different exposure
times. Bracketing allows the photographer to choose which of the
exposures will render the best contrast.

tone:
!he quality or shade of
color
variation:
a smal I difference in
somelhing
direct:
lo control something
regulate:
to adjust
apparent:
obvious
filter:
a piece of equipment used
to remove certain kinds of
light
manipulate:
10 control somelhing wilh
hands
render:
to make or cause
somelhing

L--------------------------..---..---........,,_.......---------- ...................--------------'

- 92

Chupter 5

'

-1

'

[I"'....

--------,
Step 1:

Step 2:

Description:

Description:

1. The word degree in the passage is


closest in meaning to

!.

(A)
(B)
(C)
.(D)

angle
variant
amount
temperature

Example:

[
[

Example:

3. In stating that too much light will make a


subject look washed out, the author
means that the subject will appear
(A) clean
(B) faded
(C) darker
(D) in high contrast

2. The word adequate in the passage


is closest in meaning to
(A) satisfactory
(B) bright
(C) attractive
(D) careful
Fill in the blanks to complete the summary.

The passage describes ways that photographers can achieve effective contrast in
photographs. Proper contrast in a photograph will _ _ _ _ _ the viewer's attention to
its most interesting aspects. One way is to
exposed to light. By using lens

the amount of time the film is


, photographers can accentuate certain

colors and reduce others. The other way that photographers control contrast is by exposing
the film correctly. Exposure can be _ _ _ _ _ _ _ by changing the aperture or the
shutter speed. A technique called bracketing can allow a photographer to choose an
exposure that will _ _ _ _ _ the best contrast in a photograph.
------------------~------.

-- "- -- .. -- ----' ........ .... -- ---~-------- ---- ---------~-~----"' ------ .. ----------Voceibulary Questions

93 - - -..,

Health

':>.

Read the following passage. Then fill in the diagram with the information that you read. i@ffl
'

Fatigue is the feeling of extreme weariness or exhaustion. At some


point, everyone feels the effects of fatigue, whether it is after a long
run or a hard day of studying. However, not many people know about
the different types of fatigue. The three main types of fatigue are
physical, mental, and in extreme cases, pathological. Knowing the
types of fatigue can help to identify the behavior that causes it. Or in
some instances, the type of fatigue can also provide clues about
more serious health conditions that may be present.
Physical fatigue is a familiar feeling for many people. Some
describe physical fatigue as feeling drained. It usually occurs after
doing rigorous work, like riding a bike or playing a game of soccer.
Medical experts define physical fatigue as a condition that keeps
someone from performing at a normal level. Physical fatigue is also
called muscle weakness. This is because, in a medical sense, one
who feels physical fatigue is actually suffering from a loss of muscle
function, albeit a temporary one. Physical fatigue usually becomes
obvious after some exertion. When the body is forced to work while
fatigued, one may experience a burning sensation in the muscles.
A different type of fatigue is mental fatigue. People who suffer
from mental fatigue do not always feel the same effects as those who
suffer from physical fatigue. In fact, a person who is mentally fatigued
is often afflicted with drowsiness. In other cases, a mentally fatigued
person may not feel sleepy, but find it very difficult to concentrate.
Mental fatigue can occur because of many different factors. Stress
and overwork are common causes. Interestingly, one of the top causes
of mental fatigue is boredom. As with physical fatigue, mental fatigue
can be harmful if it interferes with daily activities such as driving.
Many people experience either physical or mental fatigue on
occasion. For some people, however, fatigue is a constant condition.
When fatigue causes discomfort or often interferes with daily
activities, it could be pathological. Pathological fatigue may be an
indicator of a more serious health problem, such as chronic fatigue
syndrome (CFS). CFS is a disorder that causes sufferers to experience
constant physical and mental fatigue. A commonly misunderstood
disease, CFS is one example of how fatigue can take on pathological
proportions.

''

I
I

1'

'

- 94

Chapter 5

............, ... _._.

__

,,,_,_,

_____ ___
,,,

,,

..........- ............. , . , _ , ,

weariness:
the state of feeling tired or
lacking energy
pathological:
relating to or caused by a
disease
drained:
lacking energy
temporary:
not lasting a long time
exertion:
the state of using energy;
effort
drowsiness:
the state of being sleepy

_J

_______.

; i

1. The word rigorous in the passage is


closest in meaning to
(A) severe
(B) demanding
s~ (C) precise
(D) forceful

3. The word interfere in the passage is


closest in meaning to

(A)
(B)
(C)
(D)

go into
get in the way of
lose influence
lose concentration

2 . . The word albeit in the passage is closest


in meaning to

(A) although
(B),moreover
(C) yet
(D) all ready
Fill in the blanks to complete the summary.

The passage discusses three types of fatigue, the feeling of extreme _ _ _ _ _ __


or exhaustion. Physical fatigue can be described as feeling
physical exertion. This type of fatigue is a(n)
and is usually pronounced after some type of
fatigue. It is characterized by feelings of

, and is caused by
loss of muscle function,
. The second type is mental
and difficulty concentrating.

The third type is pathological fatigue, which may be caused by a disease such as chronic
fatigue syndrome.

----------------------------------u-.O .... - - - - b

Vocabulilry Questions

95

----I

A~1tt1 ropo~ogy
~~"'f1.'/"1",;,',-:.:u!''''-~v;;_;.r.,'.w;r':.~~~L--,.,,--,

,;,,::: J .:r '"'- . -

--

-. - ---

Read the following passage. Then fill in the diagram with the information that you read. l@jil

:;,'

I
J

The word "aborigines" may bring to mind the native peoples of


Australia. However, the term also refers to the group of people who
originally inhabited Taiwan. The often-ignored indigenous people of
Taiwan have a very rich culture and history. They also share a variety
of interesting creation myths and fascinating traditions. Although
perhaps not widely recognized, according to many anthropologists,
prehistoric Taiwan was most likely the seat of Austronesian languages
and culture.
The Austronesian language group is thought to include the largest
number of languages. One-fifth of all languages in the world belong to
the Austronesian family. It is also the most geographically dispersed
language group in existence. These languages are widely spoken by
native peoples in countries located in the Pacific Ocean. Such countries
include Malaysia and Indonesia (where Malay is spoken), the Philippines
(Tagalog), and New Zealand (Maori). Surprisingly, Malagasy, the
language of the natives in Madagascar off the eastern coast of Africa,
is also considered an Austronesian language.
It may seem hard to believe that one of the largest language
families could have originated on an island as tiny as Taiwan. Yet,
anthropologists have recently uncovered more evidence that
confirms this hypothesis. The most convincing data stems from the
concept of language dispersion in linguistics. The concept holds that
the spread of language can be determined by tracing it to the area
with the most linguistic diversity. In other words, a region that is found
to have many languages is usually the source of related languages in
neighboring regions. In Taiwan, this is certainly the case. In fact, nine
out of the ten branches of the Austronesian language group are
formed by Formosan languages. Formosan refers to languages spoken
by the original inhabitants of Taiwan. This means that languages spoken
on the island were likely the basis for most Austronesian languages.
Another way of tracing Austronesian languages to Taiwan is by
studying migration patterns of ancient speakers. Archaeological
evidence. suggests .that people (presumably those who spoke
Austronesian languages) began leaving Taiwan about 6,000 years
ago. Also, studies have uncovered a genetic link between people
currently living in Austronesian-speaking countries and the indigenous
people of Taiwan. This connection is significant in proving that the
language, like the people, probably originated in Taiwan.

Ignored:
overlooked; disregarded
confirm:
to prove to be true
hypothesis:
a guess that is made by
studying the facts
stem:
to come from or originate
in
diversity:
a variety of something
migration:
a population movement
from one place to another

'

. . . . . . . . . . . . . . . . . . . . . .. . . . . . . . . . . ..... . . . .. . .:................................................................................~------J

~.:\--

.
; ~

;:

,.-; ,,:

96

Chapter 5

'

> '..Origin

Theory:

[
[
[

of Austronesian Languages

Support 1:

Explanation:

Support 2:

Explanation:

1. The word indigenous in the passage is

'
'

3. The word dispersed in the passage is


closest in meaning to
(A) spoken
(B) learned
(C) spread
(D) central

closest in meaning to
(A)
(B)
(C)
: (D)

original
angry
old
shy

' 2. c: In stating that Taiwan was the seat of


Austronesian languages and culture,
the author means that Taiwan was
(A)
(B)
(C)
(D)

a place to stop
the main focus
the base
the last

Fill in the blanks to complete the summary.


The passage proposes that the often-_ _ _ _ _ aborigines of Taiwan are the
descendants of the original speakers of Austronesian languages. Anthropologists have
uncovered evidence that
convincing data
the most linguistic

the origins of this language family. The most


from the fact that languages usually originate in areas with
. Studies reveal that Formosan languages form nine out

. of the ten branches of Austronesian language. Scientists also can genetically link peoples
from countries that speak Austronesian languages to the ancient inhabitants of Taiwan by
studying their

patterns.

Vocabulary

Qu~stions

97 _ _.,j

Necessary Skills
~~=.tN4W!;o;;;;.,r.1r~'illi.'Y.dli(~,;;~\;;c,.;\'><}C\"_,f_._,.,,;.,J,)l11,;:0\''; .. fo, . l', ' '

Identifying the relationships between pronouns and their referents in a passage


Determining to which person, place, or thing a pronoun refers in the context of a
passage

!Example Questions
The word _ _ _ _ _ in the passage refers to

Strategies
Ensure that your answer choice matches the same number (singular or plural)
and the person (first, second, third) as the pronoun being asked about.
Try substituting your answer choice for the pronoun in the passage to see if your
choice is sensible.
Since there is often more than one answer choice that may seem correct, read
the sentences around the pronoun's sentence carefully to ensure you choose
correctly.
Remember that the grammatical referent for a pronoun may appear in a preceding
clause or sentence.

Reference Questions

99 __,

History
~'i:'~"'w1n<11t!&l!i"b'WW1!(11i&mO:.u~-<n.n1\:"':,

!;m;\::-w;

'"'c_,.,

Read the following passage. Then fill in the diagram with the information that you read. @f&I

1'

i1

~.

\
I

"

The United Nations is an international organization. Its primary


goal is to maintain peace arnong the countries of the world. It is
comprised of 193 nations and has existed for over fifty years. The
United Nations officially convened for the first time in 1945. However,
a series of events preceded its formation.
Before the United Nations carne into existence, there was another
organization with a similar goal called the League of Nations. It was
founded in 1919, shortly after World War I. However, the League of
Nations was not effective in enforcing its resolutions. This and other
limitations left world leaders wanting a different type of international
organization.
The main motivation behind the formation of the United Nations
was to avoid another devastating war like World War 11. The idea for
the United Nations came about during wartime conferences in
Moscow and Tehran in the latter part of 1943. When the declarations
were signed, an agreement was rnade that an organization like the
United Nations would be created in the future. Ov~r the next year,
representatives from China, France, the United Kingdom, Russia, and
the United States met in order to outline the basic goals of the
organization. They were agreed upon, and the charter was finally
approved on October 24'", 1945. This marked the official beginning of
the United Nations.

The Formation of the United Nations

primary:
main

convene:
to come together
precede:
to come before

r
l'
J.-

n.

II'
1

enforce:
to uphold

i'.'

resolution:
a formal statement of a
decision made by an
organizatio_n or assembly
devastating:
causing a lot of damage

wartime:
occurring during a war

charter:
a formal statement

I'.
i

J:

!.'

i'

-------------------

1------1919:

L_,_____J
,:.1

Xi---

100 Chapter 6

1945:

1944:

1943:

L____

_J

. f

,_____J

The word its in the passage refers to


'
(A) sovereign nations
(B) the League of Nations
(C) the United Nations
(D) peace

2. The word I\ in the passage refers to


(A) The United Nations
(B) The League of Nations
(C) Goal
(D) Existence

3. The word Tqey: in the passage refers to


(A) Basic goals
(B) Conferences
(C) Representatives
(D) Five countries
.,\,

<:

Fill in the blanks to complete the summary.

:i

_.. .,.:.x...................--------------------.. -----------------------------------------.. --------------------------------------------

The passage discusses the history of the United Nations. Another international
organization called the League of Nations _ _ _ _ _ its formation, but it had been
ineffective in _ _ _ _ _ its resolutions. World leaders wanted to avoid another
- - - - - - - w o r l d war, so they made an agreement to create a new peacekeeping
organization. After a series of _ _ _ _ _ conferences with representatives from around
the world, the United Nations officially

for the first time on Oct 24'", 1945.

Reference Questions

101 - - i

...1.----~.....---.--..-....----....-.-------

Sociology
n~~li.'<t'&U<UIO:,".~\'.-LJ<:~,Mn.\?i;,.:,;;,x,.-s,.<-,,,..,;.,,_ }~

' __ ,,_,

Read the following passage. Then fill in the diagram wit~ the information that you read. l@ifj

Urbanization describes a population shift from rural communities


to large cities. More and more, countries are encountering problems
with the rapidly growing rate of big cities. This is even more of a
problem in areas where the population boom has surpassed the
city's ability to meei the needs of its people. This has been the case
in the Mexican capital, Mexico City. A metropolis of over twenty million
people, its resources have become limited. City administrators have
been left to deal with problems such as water shortages and heavy
traffic. Lckily, city officials are taking new measures to reduce the
problems associated with urbanization.
Having a safe and reliable water supply is important to any city in the
world. In Mexico City, water shortages have always been a problem.
Although the city is situated above an aquifer, it is not enough to supply
the overwhelming number of inhabitants with the water it needs. In
order to counter the water shortage, city officials have begun to explore
options for obtaining water from other sources, namely states that
surround the federal district. Additionally, they have launched a city-wide
campaign to raise awareness about water conservation among the
city's citizens.
The dense population of Mexico City inevitably creates traffic
jams that can last for hours. In order for the city to properly function,
its inhabitants need to have a way to get around quickly and easily.
Mexico City's response to the traffic problem was to form a partnership
with an organization. Their partnership will help city administrators
improve public transportation. Among the changes is the replacement
of old, unreliable buses used for public transportation. This will increase
efficiency by avoiding mechanical failures. Moreover, extra routes will be
added in order to serve a greater part of the population. The improvement
in public transportation is expected to ease traffic congestion by reducing
the number of cars on the road.

rural:
characteristic of the
country
surpass:
to do better than
metropolis:
a very large and influential
city
shortage:
a lack of something
reliable:
able to be trusted or
depended upon
aquifer:
an underground source of
water

------------------------~

: r - - 102 Chapter 6

tl._

Problem 1:

Solution:

,,
~z
VI

GI

Problem 2:

Solution:

1. The word Thi~' in the passage refers to


(A) Population
(B) Problem
(C) Rapid growth
(D) Rural shift

3. The word )hey; in the passage refers to

(A)
(B)
(C)
(D)

city officials
sources
inhabitants
water shortages

\j:

!;

2. , .The word ii in the passage refers to


' (A) Mexico City
(B) population
(C) aquifer
(D) water shortage

Fill in the blanks to complete the summary.

The passage discusses the problems associated with urbanization. A large population
shift from

communities to large cities can cause great difficulties, especially

in areas where population growth


its citizens. For example, in the

a city's ability to meet the needs of


of Mexico City, issues such as water

_______ and heavy traffic are a challenge for city officials. Mexican administrators
are now exploring options that will provide people with a safe and _ _ _ _ _ water
supply and efficient public transportation.

Reference-Questions

103 __,

Binary star systems are pairs of stars that move around the same
center of mass in space. To explain further, the center of mass is the
point on which the mass of a system is concentrated. By studying
binary stars, astrophysicists are able to determine the actual mass of
stars in the system. There are several types of binary star systems.
Each type is classified according to how it is observed.
A visual binary star is one that can be seen through a telescope.
Sometimes binary stars cannot be seen through a telescope
because they are too close together and simply appear as one star.
However, many modern telescopes with high resolving power can
distinguish between binary stars. The brighter of the visual binary is
called the primary star, and the dimmer is called the secondary star.
By measuring the angles of the stars in relation to each other over
time, physicists are able to determine the shape of the binary's orbit.
Spectroscopic binary stars are systems that can only be seen
with a spectrometer. The spectrometer is able to measure otherwise
unperceivable differences in the type of light the stars emit. By
watching for spectral lines, astrophysicists can recognize a system in
which the stars are too close together to be seen with a telescope.
An eclipsing binary star can be seen when the orbit of the binary
is situated along the line of sight for the viewer. As the stars travel
along their orbits, they inevitably eclipse each other. By studying
eclipsing binaries, scientists are able to learn about the distance to
other galaxies with utmost accuracy. Thus, they can increase the
available information concerning the composition of space.

'

''

!!<'
I

!"'I

concentrate:
to bring together; to focus
resolving:

relating to the ability to


show things in detail
orbit:

a path that objects move


around
unpercelvable:

not able to be noticed


spectral line:

a dark or bright line that is


emitted from a continuous
spectrum
Inevitably:

in a way that cannot be


avoided
composition:

the parts that something


is made of

Ii
:;i,

~~\::-r-"_I

11-.,
:'.\
iA

104

Chapter 6

I' '

I
Classification 1:

Binary Star Systems

'>

[I-

Explanation:

.'
;

[f

Classification 2:

II

Explanation:

,__

Classification 3:

Explanation:

3. The word they in paragraph 3 refers to

1. The word it in paragraph 1 refers to


; (A) mass
\ii (B) type
(C) star
1;(0) system

(A)
(B)
(C)
(D)

galaxies
eclipsing binaries
scientists
orbits

2. The word they in paragraph 2 refers to


(A) lenses
(B) telescopes
(C) binary stars
(D) scientists

Fill in the blanks to complete the summary.


-----------------------------,_....,. .. _ .. _______,_.. ..
_.

Pairs of stars that

"''

,.,.

-~----.---

__ ., .... ....... ------------------------------------

around the same center of mass in space are called

binary star systems. The point that the mass of a system is

on is the

binary star system's center of mass. Visual binary stars can be seen through a telescope
with high

power. Spectroscopic binary stars can only be seen with a

spectrometer, which measures

differences in the light the stars emit

Eclipsing binary stars travel along an orbit in which they eclipse each other. Studying
eclipsing binary stars allow scientists to determine the _ _ _ _ _ _ _ of space.

Reference Questions

105 - - '

Biology
'

"

'll'l2tW~tl'U'-,w>--51'U.T<;<J:~f;!;!~zy~1;;;fa,:1',o,fil'V;'.:~'l!'-":w--.u;,~,,.,-.~,,,'.-'

; '''

Read the following passage. Then fill in the diagram with the information that you read. +@!Ill

Compared to the busy lives of humans, it would seem that plants


are just stationary companions that beautify gardens and brighten
rooms. Surprisingly, plants actually move a great deal. From the
interweaving of an ivy plant on a pole to the sunflower that cranes its
neck to reach for the sunlight, plants are in constant motion. Although
their efforts go mostly unnoticed, most piants depend on daily
movement in order to get proper nutrition, repel enemies, or
reproduce. Two common types of plant movements are tropisms and
nastic movements.
Tropism is when a plant moves in response to a stimulus.
Moreover, the stimulus is always directional, meaning that the plant
will move to face the direction of the stimulus. The movement caused
by tropism indicates that the plant is growing, thus making it an
irreversible movement. There are many kinds of tropisms. The different
types are typically denoted by a prefix that goes before tropism. For
example, phototropism is movement that responds to light. A familiar
example of phototropism is when a houseplant turns and grows
toward the brightest source of light available, usually a window. If the
houseplant is moved, one would observe that it changes direction yet
again and starts growing toward the light. This mechanism helps
plants receive enough light to be able to go through the process of
photosynthesis and create food.
In contrast, nastic movements are plant movements that occur in
response to environmental stimuli, such as temperature or humidity.
Unlike tropic movements, they are not directional. Also, because they
do not have to do with plant growth, nastic movements are reversible.
Different stimuli are responsible for nastic movements. When referring
to a specific type, the same system of adding an appropriate prefix
to nasty is used. For example, a nastic movement that occurs in
response to a touch is called thigmonasty. Thigmonasty is the type of
plant movement that is responsible for the rapid shutting motion of the
carnivorous Venus flytrap. The plant is equipped with highly sensitized
hairs on each leaf. When an insect touches the hair, the leaves snap.shut, trapping the insect. Although more dramatic than a phototropic
houseplant, thigmonasty is just a mechanism that allows the Venus
flytrap to secure the nutrients it needs to survive.

1 - - 106 Chapter 6

stationary:
stuck in one place; not
able to move
Interweave:
to combine two or more
elements so they cannot
be separated
crane:
to stretch in order to look
at something
repel:
to force away
stimulus:
something that causes
growth or activity
mechanism:
a method or means of
doing something
photosynthesis:
a process in plants by
which light is converted
into chemical energy

"

IIf
Type 1:

Type 2:

<
<

ICharacteristic 1:
I Characteristic 2:

I
I

IChmct";,i;o 1'

Ll_c_h_ar_a_ct_e_ris_t_ic_2_:_ _ _ _ _ _

1. The word theJr. in the passage refers to

!'

i.

(A)
(B)
,i' (C)
L(D)

~~---"----'I.

ll

[
[

3. The word they in the passage refers to

sunflowers
poles
plants
movements

(A)
(B)
(C)
(D)

nastic movements
tropic movements
environmental stimuli
directions

'". 2. "The word it in the passage refers to


'
(A) indication
(B) growth
(C) plant
(D) tropism

Fill in the blanks to complete the summary.


While plants may appear to be _ _ _ _ _ _ _ objects, they do in fact move a
great deal. For example, ivy plants can

with a pole, and sunflowers

_ _ _ _ _ their faces toward the sun. One type of plant movement is tropism.
Tropisms occur when a plant reacts to a directional

. For instance,

phototropism is when a plant moves \n response to a light stimulus. Another type of plant
movement is nastic movement It occurs when a plant reacts to environmental changes.
Thigmonasty occurs when the plant moves in response to touch. This movement is a(n)
- - - - - - - t h a t allows plants such as the Venus flytrap to feed.
., ___ ., _____________________ .. __________________________________ _______________________ ., _____________ ., _______ ., ________ ., _____ _

Refer:ce Questions 107

---'lI .

Health Sciences
Read the following passage. Then fill in the diagram with the information that you read. +@jl:M

cultivate:
Sugar cane has been cultivated since prehistoric times. Thbught
to grow
to have originated in the Pacific island of New Guinea, sugar has found
suppress:
to prevent from operating
its way to households all over the world. Despite its value, the sweet
crave:
substance has many negative effects, according to some scientific
to want something strongly
reports. It has been proven to cause tooth decay, suppress the immune
Indulge:
system, and contribute to obesity. But perhaps the most serious of
to allow oneself to have
something enjoyable
the consequences that result from eating sugar is the possibility of
withdrawal:
developing a sugar addiction. Sugar addiction is when a person
a period when one
discontinues use of a
craves food that contains sugar and has a negative reaction in its
substance on which the
absence. Scientists have learned that it is possible for humans to
body is physically
dependent
become both psychologically and physically dependent on sugar.
tremor:
Psychological dependence is when the mind becomes reliant
a slight shaking movement
on a certain substance. or activity in order to feel good. Psychological
addictions usually occur when a person tries to relieve other symptoms,
like feelings of loneliness or anxiety. Thus, it is easy to see how one
might become psychologically dependent on sugar. Numerous studies
and surveys have revealed that people seek sugary foods when they
are feeling down or angry. In addition, they often report feeling better
after indulging in a sweet snack. This suggests that the uncontrollable
cravings, anxiety, and even depression experienced when one cannot
have sugar are caused by a psychological addiction to the substance.
The possibility that sugar is physically addictive has been a matter of
debate for many years. A physical addiction is when a person's body
becomes accustomed to the use of a substance. It is characterized
by the appearance of withdrawal symptoms when the person stops
using the substance. The idea that sugar could trigger physiological
changes similar to a drug has been refuted in the past, but recent
studies have provided new insight into the legitimacy of this claim.
Experimenters tested rats by offering them sugar-water solutions
as well as regular food. Initially, the rats ate both the food and the
sugar equally. Within a month, however, scientists observed that they
had stopped eating regular food. Moreover, their sugar consumption
had doubled. When the sugar was taken away, researchers_noted
that the rats showed signs of withdrawal, such as paw tremors and
teeth chattering. Although similar studies have not been reproduced in
humans, the results suggest that sugar might be physically addict._iv_e_._ _ _ _ _ _ _ _

>

'

'~
i:

:.;

J
'

:\

<.i

:~}--;11!

ll

108

Chapter 6

[[

I
Theory 1:

Support:

Theory 2:

Support:

1. The word Thi~: in the passage refers to

"

(A)
(B)
(C)
(D)

Sugar dependence
Report feeling better
Feeling down
People seek sugary foods

[f
3. The word they in the passage refers to
(A) rats
(B) experimenters
(C) scientists
(D) solutions

l:

2. The word It in the passage refers to


(A)
(B)
(C)
(D)

Body
Substance
Use
Physical addiction

Fill in the blanks to complete the summary.


The passage discusses the theory that sugar can be addictive. When a person
_ _ _ _ _ sugary foods and experiences a negative reaction in its absence, he or
she may have a sugar addict'lon. Although sugar cane has been

for

. thousands of years, too much sugar can cause tooth decay and - - - - - the
immune system. According to surveys, people _ _ _ _ _ in sugary foods when they
are feeling angry or sad. This suggests that sugar is psychologically addictive. Evidence
that it may also be physically addictive was revealed after a study on rats. The rats that
were dependent on sugar experienced _ _ _ _ _ and other withdrawal symptoms
when the sugar was taken away.

--------------------------------------------------.. --------------- .. ----------------------------------------------Reference Questions

109 - -.. ; .

Psychology

..

~~'l'~!t~'~a:.ic,~.v.~C!N~"""::,:.o-r._,; ~;:i:

,--~.~-;:

.-,.. ;,,,,,.'<',,-.

Read the following passage. Then fill in the diagram with the information that you read. itl!!'IPI

ii
'

'i.

paralanguage:
Body language can say a lot about what a person thinks or
communication that does
desires. For many years, psychologists have studied body language
not involve speech or
verbal utterances
to learn about how people communicate by using their bodies.
receptive:
Paralanguage is an important part of communication, not only among
able to listen to ideas
humans but between humans and animals as well. Scientists have
cue:
a signal
discovered that some animals seem to be more receptive to cues
document:
from humans. One instance of this is documented in the case of
to record the details of an
Clever Hans, a horse from Germany who would lend his name to the
event
showcase:
phenomenon called the Clever Hans Effect.
to display something
Early in Hans's life, his owner discovered' that Hans had an
adept:
extraordinary talent. Unlike other horses, Hans was able to perform
having natural ability to do
something well
simple arithmetic, tell time, and even keep track of calendars.
Wishing to share Hans's talent with the world, his owner took Hans all
over the continent to showcase his wondrous skills. But when German
psychologist Oskar Pfungst studied Clever Hans, he discovered that
Hans was no more skillful at mathematics than any other horse. Instead,
he was particularly adept at interpreting human body language.
Pfungst came to this conclusion after many different trials. First,
he took Hans away from an audience to make sure the horse was not
receiving signals from people watching him. Then Pfungst had people
other than the horse's owner ask Hans questions. In both trials, Hans
was able to answer the questions correctly, but the horse performed
poorly when he could not see the person asking him questions. He
also did not perform well in tests where the person asking the question
did not know the answer. These results led Pfungst to a groundbreaking
discovery. Without intending to, the people asking questions were
giving Hans cues about the correct answers. As the horse tapped out
his response to a question, his trainer would change his posture
when Hans approached the right number. That was why he could not
give the right answer when the trainer did not know it. Furthermore,
without being able to see the trainer, Hans could not tell when he
should stop tapping his foot.
. Pfungst's discovery was important in many respects. It helped
scientists thereafter design experiments in which testers could not
influence the results through subtle, often involuntary cues. It also
brought to light problems that occurred not only with animal test
subjects, but human subjects as well.
---------------------------------

l - - - 110 Chapter 6

How Pfungil
Disc60~re8thed~Jer
R~n's Ette(;(.'
-_
'
.
..

..

''

i',

'

~':'

'

-,

'

_'

'"

Step 1:

Step 2:

Step 4:

Step 3:

::;;;

"':::;

Result:

Result:

1. The word their in the passage refers to


(A) people
::. (B) horses
"': (C) psychologists
(D) animals

Result:

Cl

Result:

. 3. The word It in the passage refers to

(A)
(B)
(C)
(D)

Experiment design
Pfungst's discovery
Involuntary cues
Test subject

2. The word he in the passage refers to


(A) Pfungst
(B) the trainer
(C) Clever Hans
(D) the person asking questions
Fill in the blanks to complete the summary.

The passage discusses a psychological phenomenon called the Clever Hans Effect. It
was discovered by German psychologist Oskar Pfungst when he

the

case of a horse that could allegedly perform arithmetic and keep track of the date. In order
to _ _ _ _ _ _ _ his amazing skills, Clever Hans and his owner traveled the continent.
Pfungst subjected the horse to a series of tests and discovered that the horse was actually
just

at interpreting human body language. By being more

to

_ _ _ _ _ from the trainer's involuntary body language, Clever Hans was able to

produce the correct answers .

........................... -------- ....

-~--------------------------------

.. ---------- .. -------- .. -- .............. _____________ ........................ --Reference Questions

111

--.i

'i. ,;,:

Necessary Skills
;;;;.o,;.<\"AW;C~~ ..il~!:c:i!i"N.ll~1t;,n'M>;1::/.\;;.W;f,1,>''!..i~W.:;-.tl.'<V,.,,,;-,., '"'"'" c_..;;:;_,_,;:

'"'" ' ' ' ' ' ;,, ,;_, . '

Identifying the answer choice that has the same essential meaning as a
highlighted sentence in a passage
Eliminating answer choices that change the meaning in important ways or leave
out essential information

E;campie Question
Which of the following best expresses the essential information in the highlighted
sentence? Incorrect answer choices change the meaning in important ways or
leave out essential information.

Ensure that you understand the ways in which an answer can be incorrect. Either
it contradicts a detail in the highlighted sentence, or it omits something important
from the sentence.
Be careful that your answer choice does not contradict the main argument of the
paragraph in which the highlighted sentence occurs or the passage as a whole.

Sentence Simplificution Questions

113

----!

Geography
.lKK<~rom\li'Jlri'!i:<&.>:r<;.t:tR'Z1"iiliif.~~~~Jf,{''-7.~"-'-';,n-~-. A.'-'.<->;1-,, i' 'N:.n> \'

. ,5,-_.,

Read the following passage. Then fill in the diagram with the information that you read.

The hydrologic cycle is the process of evaporation and condensation


in which rain is recycled until it returns to the Earth's surface in the
form of precipitation. It is one of the most important cycles on the
globe as it ensures a constant water supply; however, in some areas
of the world, the process is interrupted, which results in the rain shadow
effect.
A rain shadow is created when the hydrologic cycle is interrupted
by mountains. To understand why the rain shadow effect occurs, it is
important to first examine how the hydrologic cycle functions. First,
water on the surface of the Earth evaporates, which is when water
droplets vaporize, or turn from liquid to gas form. When evaporation
occurs, the moist air rises. Near mountain ranges, as the air travels
upward, it is affected by the cool temperatures of the high elevation.
The air is blown toward the mountains by wind and as it cools, it
begins to condense. That means the water vapor is turned into water.
However, the mountain physically obstructs the- rain clouds that
carry the water. Before it can reach the other side of the mountain, the
water usually falls as rain or snow on the mountain; meanwhile, the
leeward side of the mountain receives little to no rain. This area is
called the rain shadow.

--------~--------]

The Rain Shadow Effect

-----~-------------"--~-

Step 1:

i.

114 Chapter 7

..

--~-~

@I"'

preclpl!atlon:

water that falls from clouds


droplet:

a small amount of liquid


elevation:

a height
condense:
to cause a gas to change
to a liquid
obstruction:

something that blocks

Which of the following best expresses the


essential information in the highlighted
sentence in paragraph 1? Incorrect answer
choices change the meaning in important
ways or leave out essential information.
(A) The hydrologic cycle recycles the
water on the Earth unless it occurs
near a mountain, where it cannot
occur.
(B) The rain shadow effect reverses the
processes of the hydrologic cycle by
ensuring constant water supply.
(C) The rain cycle helps provide water for
the world except for rain shadow
areas, which occur when the cycle
cannot be completed.
(D) The process of recycling water and
providing water for the world is
endangered by the rain shadow effect.

[
[
[

(B) If water cannot evaporate on \he


Earth's surface, it cannot turn '1nto
water vapor later.
(C) When water evaporates, it causes the
rain cycle to begin once again by
turning water to gas.
(D) Evaporation is the process in which
water vapor turns into precipitation in
the form of rain or snow.

3. Which ot'the following best expresses the


essential information in the highlighted
sentence in paragraph 3? Incorrect answer
choices change the meaning in important
ways or leave out essential information.

1
2. ' Which of the following best expresses the
; essential information in the highlighted
. sentence in paragraph 2? Incorrect answer
'choices change the meaning in important
ways or leave out essential information.
(A) Evaporation, the first step in the
hydrologic cycle, is when water
droplets turn into a gas in the form of
water vapor.

(A) The side of the mountain facing away


from the prevailing wind usually
receives the most rain because it
cannot travel to the other side.
(B) The leeward side of the mountain
receives the remainder of the rain that
moves over the mountain .
(C) One side of the mountain can only
get water from rain that trickles down
from the top of the mountain.
(D) The mountain obstructs the passing
of the precipitation, causing the
leeward side to remain dry.

Fill in the blanks to complete the summary.

The passage discusses the rain shadow effect, which occurs when the hydrologic cycle
is interrupted. When the water in the air evaporates, the moist air rises. When it reaches the
high _ _ _ _ _ of a mountain range, the cooler temperatures cause it to _ _ _ _ _ .
Usually, water

fall back down to the ground as _______ after

they evaporate. However, if a mountain physically

the rain clouds, the rain

will fall before it can reach the other side. Therefore, the water never returns to that area.

-----------=------.. --------~--.:.- . --------------- ---------------------------------------------------------------~'

f/: ,

Sentence Simplification Questions

115

--i

....11,~,...llll.!llllllll~~llllllllllll!lllllll. . . . . . . . . . . . . . . . . . . . . . . . . . . . . . . . . . . . . . . . . . . ......

Read the following passage. Then fill in the diagram with the information that you read. +tffi@li

The Internet has opened doors for many new and different types
of businesses. One type of internet commerce that has gained
popularity in recent years is the on line auction. The business model for
this type of enterprise is one that allows users to bid on and purchase
items over the Internet. The most popular of this business type does
not actually place company-owned products up for bid. A website
. simply provides a marketplace for users to sell and buy products. The
online auction business type has been proven extremely successful
for many reasons, but the modeLhas its limitations and dr<J.wbacks.
There are several benefits to the online auction business model.
First, since it operates on the Internet, there are no time or
geographic constraints. This means that commerce can occur at any
time of day from any part of the world. In turn, the constant activity
translates to more profits for the business. Second, the company
benefits from the nature of online bidding, which involves the aspect
of "winning" an auction. For many people, online auctions are much
like gambling, and the prospect of winning an auction is ehough to
keep them coming back; therefore, online auction sites can enjoy a
high level of consumer loyalty.
Despite the success of some on line auction websites, the business
model also has disadvantages. First, the anonymity offered to patrons
of the sites often promotes the sale of stolen or counterfeit goods.
With an increased awareness of the frequency of stolen goods being
sold in online auctions, consumers feel less confident about bidding
on products online. Second, because consumers pay for goods
online, the issue of online security is important. If a company cannot
ensure that personal information such as credit card numbers are
protected, customers are wary of using the service.

,,
:\

'

auction:

a public sale where the


person who pays the most
wins the item
bid:
to offer an amount of
money to buy something
constraint:

a restriction
prospect:

a possibility
anonymity:

the state of having an


unknown identity
patron:

a person who uses a


service

counterfeit:
fake

-------..------------..----------------------..-------------'
l_. ___

r Advantage 1:
'

l Advantage 2:
I

L__

;~ --

\'.!

lj

t.

116

Chapter 7

-T~~~ Auction Business Model

r.isadvantage 1:

______________ __J

J1 Lisadvantage 2:
!

..
l

Which of the following best expresses the


essential information in the highlighted
sentence in paragraph 1? Incorrect answer
choices change the meaning in important
ways or leave out essential information.
(A) An online auction business model is
severely hindered by its limitations,
which is why it hasn't been used often.
(B) The advantages of the online
auction business model outweigh
its disadvantages, as can be seen
by its success.
(C) The success of the online auction
business model depends on the
management of its limitations.
(D) The online auction business
model has both advantages and
disadvantages.

, . 2.,i
"
'.

Which of the following best expresses the


essential information in the highlighted
sentence in paragraph 2? Incorrect answer
choices change the meaning in important
ways or leave out essential information.
(A) Online auction sites are able to draw
people in by making them feel as if
every purchase is a gamble.
(B) People who enjoy gambling typically
enjoy online auction sites because
they like to try to win.

(C) Online auction sites can create a


strong customer base because the
nature of the site makes people want
to use the service again.
(D) High consumer loyalty is achieved
when the customer is made to feel
like their purchase is risk-free.

II
"'
~z
"O

Cl

3. Which of the following best expresses the


essential information in the highlighted
sentence in paragraph 3? Incorrect answer
choices change the meaning in important
ways or leave out essential information.
(A) Online auctions are at a disadvantage
because criminal activity makes
customers less likely to buy goods
on line.
(B) The increased public awareness
about the sale of stolen products on
Internet auctions makes people less
willing to participate in online
auctions.
(C) Consumers try to make sure that the
product they have bid on is not stolen
before they make a purchase.
(D) Informed consumers will not want to
bid on products if they suspect that
they might be stolen goods.

Fill in the blanks to complete the summary.

The passage is about the online _ _ _ _ _ business model, which allows people
to _ _ _ _ _ on and purchase items via the Internet. The business model has both
advantages and disadvantages. The model does not have the time or geographic
_ _ _ _ _ _ _ of other businesses, and it provides high customer loyalty. However,
the _ _ _ _ _ _ _ of patrons allows for such criminal activities as the sale of stolen or
i
i

_ _ _ _ _ _ _ items.

'
I'

- - - - - - - - - - - - - - - - - - - - - - - - .. --............................................................................- - - - - - - - - - - - - - - - - - - - - - - - - - - - - - - - - - - - - - - - - -

Sentence Simplification Ql<estions

117

---~I

..........--....................----...................------!

~-:--

Biology
"

~"11'~:<r.;nw.t-\l'C.~~!l.t:.:.:11.U{l>'i,Wf2Jo"'"'1'0.:!''"'""

.,<;,. -. --.,', .. :.

Read the following passage. Then fill in the diagram, with the information that you read. +@lfl

Amphibians are a type of cold-blooded vertebrate that hatch in


water and are born with gills. They are distinguishable from other
animals in that they can live both in water and on land. Uncovering
the evolution of amphibians has been very difficult for scientists;
however, by studying fossils and modern amphibians, they have
been able to guess that amphibians originated from lobe-finned fish.
Tracking amphibian evolution using fossil records is a complicated
task. Very few fossils of early amphibians have been found. In fact,
thereJs a thirty~milHoncyear gap between fossil<; of the crossopterygian
(a type of fish thought to be the closest ancestor of early amphibians)
and the fossil of the first type of amphibian, the lchthyostega. Still, by
using the fossils available, scientists have tracked a number of
transitional species that document the evolution of amphibians from
water to land. This was done by observing the slow changes in
lobe-finned fish as they developed feet-like apparatus. For example,
fossils of the crossopterygian fish have primitive feet that they may
have used to run along the bottom of ponds or lakes. They also have
skull structures and teeth similar to early amphibians.
Fossils of the Panderichthys from the late Devonian period show
further development of legs and amphibian-like body structures.
Unlike their earlier counterparts, their bodies were flattened like
amphibians. They also had straight tails, and their fins had begun to
resemble feet. Fossils from the lchthyostega, the first proper
amphibian, show that the transition from water to land was made not
too long after the Panderichthys. Though much larger than their
modern descendants, their fully developed lung structures and feet
heralded the birth of a new type of animal called the amphibian.

---------

vertebrate:
an organism that has a
backbone

gill:
an organ that allows
animals to breathe
underwater
transitional:
relating to the change
from one form to another
apparatus:
a set of equipment for a
particular use
counterpart:
something that resembles
another thing
proper:
containing all definitive
qualities of a thing

:-;

-------------'

-------------------]

.........

-~-

Species Related to Amphibian Evolution


...

--~~--~~~--~-------------

'

il

118

Chapter7

.1. Which of the following best expresses the

'-t>-\

'.,

essential information in the highlighted


sentence in paragraph 1? lncorrecit answer
choices change the meaning in important
ways or leave out essential information.
(A) By studying fossils, scientists think
that lobe-finnedfish evolved from
early amphibians.
(B) Scientists have had trouble studying
amphibian evolution because modern
amphibians are so different from
lobe-finned fish.
(C) Scientists are still trying to discover
how amphibians evolved from
lobe-finned fish by looking for fossils
of modern amphibians.
(D) Despite the difficulty of studying
amphibian evolution, scientists have
determined from fossils that amphibians
evolved from lobe-finned fish.

2. 'Which of the following best expresses the


essential information in the highlighted
sentence in paragraph 2? Incorrect answer
choices change the meaning in important
ways or leave out essential information.
, (A) The crossop}E)rygian livedthirty million_
years before the first amphibian,

(B) The fossils of the crossopterygians


tend to be more intact than those of the
first amphibian because they are older.
(C) Judging by the fossil record, the
crossopterygian and the first amphibian
lived on Earth at the same time for
thirty million years.
(D) Fossils of the oldest amphibian
ancestor were found thirty million years
before that of the first amphibian.

3. Which of the following best expresses the


essential information in the highlighted
sentence in paragraph 3? Incorrect answer
choices change the meaning in important
ways or leave out essential information.

(A) The large size of the lchthyostega


distinguishes it as the first amphibian.
(B) Modern amphibians bear little
resemblance to the first amphibian.
(C) The /chthyostegas are known to be
the first type of amphibian despite
. their large size and lack of lungs and
feet.
(D) lchthyostegas are considered the first
amphibians because they had lungs
and feet.

lchthyostega.

Fill in the blanks to complete the summary.

The passage is about the evolution of amphibians, which are cold-blooded animals that
hatch in water and are born with - - - - - Few fossils of early amphibians have been
found, but scientists have managed to determine some

species

among them. From the earliest ancestor, the crossopterygian fish, scientists observed the
development of feet-like

. Later, the Panderichthys fish was found to

have amphibian-like features, unlike its earlier


the tchthyostega are believed to be the first

. Finally, the fossils of


amphibian.

-------------~-------- . - .... - . - .............. ____ <0-- .. - - - - - - - - - - - - - - - - - - - - - - - .. ------~=

..----'"-----------.:.----------------Sentt:nce S1111pi1ii..:at10P Questions

119

--.~

[;,

~-1

-'1~1

l)

l'F'

History
_

<'~-'1'.;f~;'.'"f.-!''C"Ui~'{.Se-~;-.

Ft

,,,,., ... '

!.';.

Read the following passage. Then fill in the diagram with the information that you read. @@ii

The Roman Empire was one of the longest-lasting empires in history.


It is often referred to when discussing great civilizations. It has been
studied by numerous scholars, and many theories have been offered
as to the reason why the Roman Empire fell in the year 476. One of
these theories was proposed by anthropologist and historian Joseph
Tainter. Tainter has a slightly differenttake on the fall of Rome, and
attributes its decline to the central idea that societies collapse when
they. become too complex. a.. nd can no longer sustain. themselves.
.
.. - .
According to Tainter, the more problems a society is posed with,
the more complex they become in trying to solve the problems. But
problem-solving requires resources, such as human resources, to
administer and oversee the solution. It also requires monetary
resources to finance the cost of the solution. As a society tries to correct
itself, it continues to deplete resources until the society collapses
because it can no longer maintain itself.
To support this theory, Tainter points out how the Romans dealt
with the problem of limited food. The problem originated because
the Romans had begun to produce less food, but the population
continued to increase. At this point, according to Tainter, the society
began to become more complex. Their solution was to conquer
neighboring lands and take their resources, whether it was food,
slaves, or metal. In solving the problem, the Roman Empire grew
more multifaceted, and the solution to the problem required the
extended efforts of high Roman officials, soldiers, and bureaucratic
officers. In Tainter's view, the cost of solving the problem eventually
caused an internal breakdown of the system. This was because the
energy required to find a remedy distracted from the energy required
to deal with new problems, such as invasions.
-

empire:
a group of countries ruled
by a single person

' :'.~
,.' ;;.

1-~i::
I .i)

~-

'

clvlllzalfon:
the state of developed
human society with culture
scholar:
a person dedicated to
studying a particular
subject
attribute:
to believe to be a result
of a cause
monetary:
relating to money
finance:
to pay for
neighboring:
next to

:i

_ _ __j

----------]
I,-----------------
What Caused the Fall of Rome?
..

-------------------,~-------------

---..----------------1
\ Theory:
.

I
l

'

'

L------------------.I

,,_

L_ ________,._ .. , _ ............--------------------- ........... .

, ..\;

1. Which of the following best expresses the


essential information in the highlighted
sentence in paragraph 1? Incorrect answer
choices change the meaning in important
ways or leave out essential information.
(A) The decline of Rome, according to
Tainter, occurred because the society
could not grow.or become more
complex.
(B) Tainter believes that societies like
Rome fall because they cannot
sustain a declining culture.
(C) Tainter's ideas differ from other
scholars in that he thinks that Rome
fell because they could not sustain
their increasingly complex society.
(D) Tainter proposes that it takes
knowledge of complex societies to
understand why Rome fell.
:

~i.

2. Which of the following best expresses the


r

essential information in the highlighted


sentence in paragraph 2? Incorrect answer
choices change the meaning in important
ways or leave out essential information.
(A) A society will collapse after it has
exhausted all of its resources in
attempts to correct problems.

(B) When a society corrects itself, it is


able to reduce its use of resources
until it collapses.
(C) A society that has fixed its problems
cannot maintain itself without new
resources.
(D) The loss of resources is the main reason
why societies cannot repair themselves.

lt
'

'

...

Ill

"'z
G'>

.z

,,

Ill

>
"';.;

G'>

3. Which of the following best expresses the


essential information in the highlighted
sentence in paragraph 3? Incorrect answer
choices change the meaning in important
ways or leave out essential information.

~I

9
n
"';;l

!!l

(A) The Roman Empire dealt with their


problems in a way that made officials
work harder.
(B) Roman bureaucracy was efficient in
solving problems because they
worked hard.
(C) The Romans were demanding of their
high officials because they wanted to
solve too many problems.
(D) As Romans fixed one problem, they
created several others that the
government could not solve.
.. .

Fill in the blanks to complete the summary.


The passage is about Joseph Tainter's theory regarding the fall of the Roman
- - - - - This great

has been studied by many _ _ _ __

According to Tainter, Roman society was ruined because it became too complex. Complex
societies need resources to

the costs of solutions to their problems. In an

attempt to address their food shortage, the Romans decided to conquer _ _ _ _ _ __


lands and take their resources. Eventually, the Romans only created more problems for
themselves and extended their resources until they could no longer maintain the society.
----------.- .... .,,.,., .,---=- .. -------v----------~-----''''''"'"--'""''""'"''"-" >------'""-""''----- .. ------------"--

II

Sentence Simoliiicat1or1 Questions

121

----1

Psychology
~~-ffe...l[l~'-\i~7.'1'.~"~b1LMN\l'h.\'.-!f_~,;-,;:;~:t~{C_,~;,~_L,"\

--;, . ' , .. '

Read the following passage, Then fill in the diagram with the information that you read. 11@"

,.

i.

cognitive:
For years, intelligence was evaluated in terms of how well a person
relating to conscious
performed mental tasks such as reading, writing, or problem-solving. Yet
mental activity
according to psychologist Robert Sternberg's Triarchic Theory of Human
approach:
a way of doing or. solving
Intelligence, the key to understanding intelligence is recognizing different
something
types. The theory proposes that intelligence is better analyzed by taking
proficiency:
the state of being skilled
a cognitive approach, meaning that more can be learned by studying
standardized:
its different parts. According to Sternberg, the three components of
conforming to an
human intelligence are: analytical, creative, and contextual.
approved model
Analytical intelligence is essentially "book smarts," since one with . Intuitive:
having a deep insight
a high level of analytical intelligence uses skills learned in school or
context:
through books. It is the ability to solve well-defined problems that
a situation
have one solution. Analytical intelligence is measured by one's reading
comprehension or proficiency in academic tasks. In fact, most
standardized college entrance exams test mainly analytical intelligence.
Creative intelligence refers to the cognitive ability to apply existing
knowledge to new situations. In other words, it describes how well
one uses previously learned skills to solve new problems. Creative
intelligence also includes the ability to successfully perform unfamiliar
tasks. A person with creative intelligence is often highly intuitive and
artistic. A task that tests creative intelligence usually has many correct
or open-ended answers.
Contextual intelligence is the ability to behave appropriately
according to a given context. Sometimes referred to as "street smarts"
it is not based on any type of academic learning. According to
Sternberg, there are three processes for making appropriate choices.
The first is adaptation, when a person changes to meet the demands
of the situation. The second is shaping, when one makes changes to
the environment itself. The last is selection, when a person leaves an
environment to find a more suitable one. An example of this is an
emigrant who leaves a place for better prospects elsewhere.
- ..----------------------------------------'

.,'.
)

-;,,

.;
.'
'

,+--

,:..1

122 Chapter 1
'i;-

;,;:;-

: -~ 1. Which of the following best expresses the


essential information in the highlighted
r- sentence in paragraph 17 Incorrect answer
choices change the meaning in important
ways or leave out essential information.
(A) Scientists stand to learn much more
about human intelligence if they can
formulate a better approach.
(B) The three different parts of
intelligence include a heavy
emphasis on cognition.
(C) Without employing a cognitive
approach, people will never learn
the extent of human intelligence.
(D) By examining different parts of
intelligence, scientists can learn
more about human intelligence.

;:.

.i/C

,;'

>;'

'

,"-.

.,

2. ;>Which of the following best expresses the


,,: essential information in the highlighted
sentence in paragraph 27 Incorrect answer
.:., choices change the meaning in important
ways or leave out essential information.
(A) Analytical intelligence only applies to
people who have been educated and
can use the skills they learned in school.
(B) Learning from books is a skill that is .
part of analytical intelligence.

(C) Skills learned in school or in books


makes up the type of intelligence
called analytical intelligence.
(D) Analytical intelligence is promoted
by schools and books because it
helps people learn skills.

3. Which of the following best expresses the


essential information in the highlighted
sentence in paragraph 47 Incorrect answer
choices change the meaning in important
ways or leave out essential information.
(A) Contextual intelligence is also called
"street smarts" and does not involve
analytical intelligence.
(B) Contextual intelligence is called
"street smarts" because it does not
have many practical uses and is not
based on academics.
(C) Even though contextual intelligence
is referred to as "street smarts," it
has little to do with intelligence or
academic knowledge.
(D) Academic learning is also called
"street smarts" because it is not
based on traditional intelligence.

[
[
[
...,

..."'

"'!!I

Fill in the blanks to complete the summary.


---------..-----------------------.---- -------- .. -....----------------------------------- .. ----------------The passage is about Robert Sterberg's Triarchic Theory of Human Intelligence. His
_ _ _ _ _ studies three components of intelligence: analytical, creative, and
contextual. Analytical intelligence is measured by _ _ _ _ _ _ _ in academic tasks,
and is often determined through the use of

tests. Creative intelligence

is how well a person can apply known skills to new situations. Often, creatively intelligent
people are quite - - - - - Contextual intelligence is the ability to behave
appropriately in any given _ _ _ __

------------------------------------------------------------------------------- ..------------------------------------Setitence Simplification Questions 123

"' ,,,,,.-

. .. .

--'1.

Anthropology
~~M!1'.<:lt~~~=~t"~'l'Jft/Ni.h\!~X!,~>;:i<\'.'~:,f:.,,.f",,\. >,

,.,-.(.:

'
Read the following passage. Then fill in the diagram with the i(lformation
that you read. @If!

..
Cultural diffusion occurs when aspects or traits from one culture
are spread and incorporated into another culture. It is a natural process
that does not involve a country forcing its traditions and culture onto
another, as in assimilation. Three recognized modes of cultural diffusion
are direct contact, intermediate contact, and stimulus diffusion.
Direct contact diffusion occurs when one culture directly borrows
from another. This is usually facilitated by close proximity; in other
words, when two countries are located near each other, they are
more likely to experience direct c.ontact diffusion. For example, the
game of hockey, a favorite national pastime in Canada, is now popular
in the United States. The United States has also influenced Canadian
culture by popularizing baseball in the northern country.
Intermediate contact diffusion is when parts of a culture spread
through an intermediary, or "middleman." In the past, intermediaries
were merchant sailors or missionaries traveling to distant countries
who returned with cultural treasures. For example, anthropologists
believe that soldiers were instrumental in the spread of culture during
the Middle Ages. The Knights Templar and the Knights of St. John
traveled between North Africa and Europe. As they did so, they
spread different aspects of their respective cultures. Furthermore, it
can be argued that today, the Internet is a primary intermediary for
cultural diffusion as it connects people from all over the world.
The last type of cultural diffusion is stimulus diffusion, This occurs
when the knowledge of a certain trait is enough to spur the creation
of an equal trait or aspect in another culture. Without direct contact,
this tends to be an interpretation of the culture's traits rather than an
exact replica. Take the case of the arrival of Europeans in North
America. A Cherokee Indian named Sequoya noted that the Europeans
had a writing system. Without directly adopting the English system,
Sequoya developed a completely unique writing system for his people .

. - . Cultural

1Type1:

Diffusi~

Explanation:

-------~

124

Chapter 7

Incorporate:
to include something
assimilation:
the process in Which a
group takes on cultural
and other traits of a larger
group
mode:
a way of doing something
Intermediate:
being between two things
Instrumental:
important in causing
something to happen
spur:
to encourage
replica:
an exact copy of
something

('

IType 2:
IType 3:

I
[I
l

H
H

Explanation:

Explanation:

1. Which of the following best expresses the


essential information in the highlighted
sentence in paragraph 1? Incorrect answer
choices change the meaning in important
ways or leave out essential information.
(A) Cultural diffusion is when one culture
loses its characteristics.
(B) A culture incorporating immigrants
from another is referred to as cultural
diffusion.
(C) The acceptance of characteristics of
'
one culture into another is known as
1 cultural diffusion.
;(D) The diffusion of one culture into
" another usually results in war.

2. Which of the following best expresses the


essential information in the highlighted
sentence in paragraph 2? Incorrect answer
choices change the meaning in important
ways or leave out essential information.
(A) A country's location determines how
much diffusion occurs there.

11
(B) It is probable that neighboring countries
will have direct contact diffusion.
(C) Direct contact diffusion can only
occur between countries that are in
very close proximity.
(D) Countries that are located close
together have facilities to help deal
with direct contact diffusion.

3. Which of the following best expresses the


essential information in the highlighted
sentence in paragraph 3? Incorrect answer
choices change the meaning in important
ways or leave out essential information.

(A) Supporters of the lnterne.t think that it


enables cultural diffusion.
(B) It is unknown if the Internet affects
intermediate contact diffusion.
(C) The 1.nternet puts people in contact in
order to promote diffusion.

(D) The Internet is a modern agent of


intermediate contact diffusion.

Fill in the blanks to complete the summary.

The passage discusses cultural diffusion, when aspects of a culture are - - - - - - into another. There are three different

of cultural diffusion. Direct contact

diffusion occurs when one culture takes on aspects of a culture that it has contact with.
- - - - - - - contact diffusion is when the spread of a culture happens through a
"middleman." Stimulus diffusion is when the knowledge of a certain trait

the

invention of a similar trait in another culture, though it may not be an exact _ _ _ __

.i./. ______________________________..
!

Sentence Simplification Questions

125 - - -..j

Necessary Skills
f-'ll':.W..o/.JI~'n~'!i.'ll'<!Ui.'i~<>~a<>o1>cot'>{=~.,<.-1;10d-:..-',.;i,,;;"'u:<'

.->:. ' ... ; .

Understanding the logic of a reading passage and grammatical connections


between sentences

Example Question
Look at the four squares [] that indicate where the following sentence could be
added to the passage.
[You will see a sentence in bold here.]
Where would the sentence best fit?

Strategie:s
Try inserting the bolded sentence in place of each square.
Pay attention to both the structure of the bolded sentence and the logic of its
placement.
Watch for logical connecting words as they can give important clues about where
the sentence should be placed.
Ensure that the bolded sentence connects logically to both the sentence preceding
it and the sentence following it

Text Insertion Questions

127 ----

Read the following passage. Then fill in the diagram with the information that you read. +iffi!IJti!I
' I

<'

'

A) While fungi can be a nuisance due to their destructive


capabilities, the members of the kingdom fungi have many attributes
that make them vital to environments. B) In fact, ecosystems rely on
fungi to remain clean and healthy. C) Fungi, such as mushrooms,
molds, and yeasts, have two functions in preserving the health of
ecosystems. D)
Unlike other plants that create their own food through the process
of photosynthesis, fungi obtain nutrients through hosts. Hosts are
the plants or animals on which they live. A) In the process of
decomposition, fungi break down organic material from these hosts
into inorganic material. This material can be returned as nutrients
back into the environment. B) Without these nutrients, the soil
would not be able to support new life, and all life in the environment
would eventually die. C) Thus, through their destructive capabilities,
fungi allow life to persist. D)
+.I-\) Fungi also exhibit reproductive behavior that is unique to
their kingdom. + 8) Fungi reproduce through the use of spores,
which are microscopic particles that the fungi release into the air. As
the spores land on a habitable area, such as a damp patch in a
forest, they begin to reproduce themselves. This sends even more
spores into the air. + C) In fact, mold, which is one of the fastest
reproducing fungi, is used in the creation of many antibiotics due to
its ability to reproduce so quickly. This allows the, medicine to be
easily manufactured in large quantities. ~ 0''

,
<0

!"'"--~""

..

~--"

o.o "'~~. - - -

. . . . - , _

--

--~"

,,,

, ...........

Fungi

nuisance:
a source of annoyance
or !rouble
capability:
the qualily of being able
to do something
exhibit:
to show something
microscopic:
too small to be seen
without the use of a
microscope
habitable:
able to be lived in
antibiotic:
a medicine used to
destroy bacteria

-----~-"- M~--~- --~-,,.-l

-~-!

Characteristic 1:

I
II

i
l .-... -...... .............................. ................ ...... ...............
i-------~-~------- -~-~"_

! Characteristic 2:

.j;:\

~~ 128
-~~

d
';l

Chapters

..,.... "'"" ---

--~~

. .....J
>

Role:

.,

Look at the four squares [ l that indicate


where the following sentence could be
added to the passage.

These roles allow fungi to be put to


important and surprising uses.
Where would the sentence best fit?
(A)
(B)
(C)
(D)

First square
Second square
Third square
Fourth square

3. Look at the four diamo~ds [ +l that


indicate where the following sentence
could be added to the passage.

This behavior allows fungi to spread


quickly in an environment and reach
new areas in which they had not
previously been present.
Where would the sentence best fit?
(A)
(B)
(C)
(D)

First diamond
Second diamond
Third diamond
Fourth diamond

,,

2. Look at the four circles [ l that indicate


where the following sentence could be
added to the passage.
For example, fungi can decompose
dead leaves, which in turn will return
the nutrients in those leaves to the soil.
Where would the sentence best fit?
'"'
,
.

(A)
(B)
(C)
(D)

"'
;;l
!!!

First circle
Second circle
Third circle
Fourth circle

Fill in the blanks to complete the summary.


According to the passage, although fungi can be a(n) _ _ _ _ _ _ _ , they are vital
to ecosystems. Despite their destructive

, fungi help to decompose

organic matter so that nutrients will be returned to the environment. In addition, fungi
_ _ _ _ _ reproductive behavior through the use of spores. Once the spores find a(n)
_ _ _ _ _ _ _ area, they reproduce very quickly. This reproductive ability has led to

their use in the development of _ _ _ _ _ _ _ in modern medicine.

I
I,

--1.

~
...Jh.G...----------..........................................--......_.........i
Text Insertion Que5tions

129

Business
Read the following passage. Then fill in the diagram with the information that you read. @#fl

'':

A) Trash disposal is a problem that cannot be solved easily. The


issue has led to the suggestion of various solutions. B) Previous
waste reduction efforts focused on specific groups-consumers or
the government, for example-and their parts in solving the problem.
However, a new theory called product stewardship instead looks
toward cooperation between all groups involved in a product's life.
C) The theory suggests that environmental protection plans must be
implemented at each stage of a product's life cycle: from its conception
to its eventual disposal. D)
Product stewardship recognizes a manufacturer's ability to
reduce a product's environmental impact in a number of ways. First,
manufacturers are rethinking the way in which they package their
products. A) Thus, packaging is increasingly being produced with
recyclability and non-toxicity in mind. B) Manufacturers are also
actively promoting recycling of their products using product labels.
C) Some battery companies have implemented programs that are
aimed at reducing a battery's environmental impact D)
Many retailers are now participating in programs to reduce the
impact of the products that they sell. Retailers are in an important
position because they act as the mediators between manufacturers
and consumers. A) In addition, retailers can create their own
recycling programs that allow consumers to more easily execute the
recycling plans suggested by manufacturers. + B) For example, some
office supply retailers have instated recycling programs for used ink
cartridges. + C) These programs motivate consumers to bring back
used cartridges in return for a small payment. Then, the cartridges
are sent back to the printer company so they may be reused. + D)
While much of the government's role has been to foster recycling,
many programs have been created to allow a more direct role in
environmental protection. One such program has been started in
some US states. It attempts to encourage the recycling of cans and
bottles. In these states, consumers are charged a container deposit
.for each recyclable can or bottle they purchase. This fee is generally
five or ten cents. Then, consumers can get this deposit back if they
take the container to a recycling center .

Implement:
to set up; to establish
life cycle:

the complete process of


change and development
during the useful lite of
something

i"

II
l

'I
I

t
,.

toxicity:
the degree to which a
substance is poisonous

'~r

mediator:
a link between two people
or groups

''
i.

execute:
to put a plan into action
Instate:
to establish
foster:
to encourage; to allow

t:

l'.
r
'

f
}-

i)

::\

:~
:~

-,\
,_-,

'

"

Definition:

Aspect 1:

,_.__ Role:

VI

i
I

"O

Aspect 2:

~
z
Cl

Role:
'---

::e

Aspect 3:

1. Look at the four squares [ l that indicate


where the following sentence could be
added to the passage.
Through these plans, a product's
environmental impact can be reduced.
Where would the sentence best fit?

"'3

,.._ Role:

Cl

Where would the sentence best fit?


(A)
(B)
(C)
(D)

First circle
Second circle
Third circle
Fourth circle

f-'.

",(A) First square


"(B) Second square
(C) Third square
(D) Fourth square

2. Look at the four circles [] that indicate


where the following sentence could be
added to the passage.

In addition to using such visual


reminders, manufacturers have even
begun paying to recycle their products.

3. Look at the four diamonds [ +] that


indicate where the following sentence
could be added to the passage.
For this reason, many retailers aid
manufacturers in transmitting their
messages of recycling to consumers.
Where would the sentence best fit?
(A) First diamond
(B) Second diamond
(C) Third diamond
(D) Fourth diamond

Fill in the blanks to complete the summary.


The passage discusses the concept of product stewardship. It encourages environmental
protection plans that are _ _ _ _ _ _ _ throughout a product's entire life cycle.
Manufacturers have begun using new packaging that is recyclable and non-_ _ _ _ _ .
Retailers are also acting as
are also

between manufacturers and consumers. They


recycling programs. Lastly, state governments are

_ _ _ _ _ recycling through many programs, such as the container deposit program.

------------------------------------ ..

------~---------------------------------------------------------------------------

Text Insertion Question<;

131 - -

'

Read the following passage. Then fill in the diagram with the information that you read. +MMll;I

Earth and Venus are quite different atmospherically despite their


relative proximity in our solar system. A) The most striking difference
is the atmospheric temperature on Venus. It is about five times the
average temperature on Earth. B) In addition, the atmosphere of
Venus is about 96.5 percent carbon dioxide, with almost no water
vapor at all. C) The answer proposed by many astronomers is that
Venus experienced what they refer to as a runaway greenhouse
effect. D)
A) Because Venus is not close enough to the sun for its solar
proximity to explain the planet's heat, researchers developed the
theory of the runaway greenhouse effect to explain the temperature.
B) Just as greenhouse gases serve to heat Earth, they do the same
to Venus. However, the effect is exacerbated on Venus due to the
planet's high concentration of carbon dioxide. In essence, the carbon
dioxide acts as a barrier. It prevents the infrared radiation of the sun
from escaping the atmosphere. C) While one can imagine Earth as
having a thin barrier that traps solar radiation, Venus .contains a very
thick barrier due to its atmospheric makeup. D)
The process that prevented oceans from appearing on Venus
must be traced back to Venus's formation. It is very likely that Venus
contained water during the time just after its formation. + A) Due to
the heat, clouds never produced rain, and water vapor remained in
the atmosphere. + Bl Its presence served much the same purpose
as carbon dioxide. Therefore, as more water vapor was. introduced
into the atmosphere, the surface temperature on Venus rose. + C) This
process would then have "run away." In other words, the problem
reached such a level that any liquid water evaporated. Once in the
atmosphere, this water vapor escaped as it was broken down by
the sun's radiation. Then, even more carbon dioxide entered the
atmosphere. + D)

proximity:
nearness
runaway:
out of control
greenhouse effect:
the heating of a planet due
to the Inability of heat to
leave the atmosphere

serve:
to have the function of
exacerbate:
to make worse
concentration:
density

-----------

\l

~1---

132

Chapter S

..

. ''r.

.The

Runa~ayG;~~ilho~s~ Effect

,.\

~ -------------'~~-Ef-fe_c_t:--------~
[:::_se_2_:------~-------'~~-Ef-fe_c_t:
~

1 C_a_u_s_e_1_:

________

..,
"'

~
25

Cl

:e

"'3z
Cl

1. Look at the four squares[] that indicate

Where would the sentence best fit?

where the following sentence could be


added to the passage.

These factors suggest an effect that, at


some point in its history, caused Venus
to develop its current atmosphere.
Where would the sentence best fit?
:,

(A) First circle


(B) Second circle
(C) Third circle
(D) Fourth circle

"'

3. Look at the four diamonds [ +l that

(A) First square


:!2 (B) Second square
"'(C) Third square
... (D) Fourth square

indicate where the following sentence


could be added to the passage.

However, due to the heat caused by


the greenhouse effect, liquid water
that might have existed slowly began
to evaporate into water vapor.
Where would the sentence best fit?

2. Look at the four circles [ l that indicate


where the following sentence could be
added to the passage.

(A)
(B)
(C)
(D)

Thus, a lower amountof heat is


allowed to escape Venus, causing the
temperature to remain constantly hot.
Fill in the blanks to complete the summary.
--.................... -----------------.. ----,,----------"___ ---- .. -- ......
,,

First diamond
Second diamond
Third diamond
Fourth diamond

I
!

_.

................................ " ____ ,. ...... --------------------------

The passage discusses t h e - - - - - - - greenhouse effect on Venus.


Greenhouse gases

to heat both Earth and Venus. However, their atmospheres

are much different despite their relative


- - - - - - - of carbon dioxide acts as a barrier. This

to each other. Venus's high


the

heating effect by preventing the sun's infrared radiation from escaping Venus's atmosphere.
The heat caused by the gases allowed water to evaporate and escape the atmosphere,
which is why oceans never formed on Venus.

----------.. ---------------------------~----------- .. -----"------------.. ------------.. ----'"----------------


i

Text Insertion Questions

133 - - '

History
~wa.~,l;IEil\%}\'o~!:U'<.(~N.'Af~-Y;.1'1",(e',:,,c_~.;,;-;,

;-,,-,~-:

.-.-, . -,,_, / .-,

Read the following passage. Then fill in the diagram with the information that you read. @@ii

,,,
i'!

In the mid-1800s, American cities were experiencing rapid


population growth. At this time, the idea of beautification of the urban
landscape became an important consideration in city planning. A)
The pioneer of the design and construction of preserved public
space was Frederick Law Olmsted. B) He is considered to be the
leading landscape architect following the Civil War period. He effected
many changes in the conception of public space. C) The primary
purpose o.f public parks suggested by Olmsted was to create a quiet,
passive, open space-a kind of sanctuary from the bustle of the city.
D) Olmsted also made public space accessible to everyone, rather
than a specific group of people.
The need for public spaces arose from the realization that cities
lacked a cohesive balance of areas for work, play, and living. A)
Cities had become crowded and stressful, and city planners
recognized the desire of citizens for an escape from city life. B)
Thus, Olmsted planned his park with escape in mind. C) In fact, his
goal was to create a space that differed from New York City as much
as possible while remaining integrated in the city's overall structure.
D) Olmsted planned the park very carefully, though the end result
looked very natural, as if it had always been a part of New York City.
+ 1~) While many previous public spaces catered to specific
groups of people, Olmsted planned Central Park as a truly public
space: + B) Many earlier parks had been similar-in design, though
they catered either to the wealthy or to members of the government.
+ C) Even in the mid-19ili century, public space was not truly public.
Instead, its use by the homeless, the poor, and the young was
discouraged. + D) Olmsted, however, wanted public spaces to be
available to all people. Thus, he created Central Park to be truly
centralized in New York City. Its location in the middle of the city
ensured that traveling to the park was practical for all citizens.

beautification:
the improvement of the
visual appeal of something
pioneer:
someone who introduces a
new idea
effect:
to make happen
sanctuary:
a place to find rest and
relaxation
bustle:
chaotic activity

cohesive:
connecting parts in a
practical way
cater:
to provide something that
is desired

"f.

j ,olmsted andP,ublic Space


Topic:

Point 2:

'

where the following sentence could be


added to the passage.
The most notable of these
transformations is Central Park
in New York City.
Where would the sentence best fit?

.,

I [f

Explanation:

1. Look at the four squares [] that indicate

,;;'(A)
'~' (B)
. '(C)
'(0)

ll

H"'' '"""

~ Poict 1

"

First square
Second square
Third square
Fourth square

2. Look at ihe four circles [ l that indicate


where the following sentence could be
added to the passage.
The motivation behind New York's .
Central Park reflected this desire.

Where would the sentence best fit?


(A)
(B)
(C)
(D)

-a I

First circle
Second circle
Third circle
Fourth circle

"'

3. Look at the four diamonds [ +l that


indicate where the following sentence
could be added to the passage .
For example, King Louis XIV of 17'"century France created many beautiful
parks, though they were intended for
him and his royal counterparts.
Where would the sentence best fit?
(A)
(B)
(C)
(D)

First diamond
Second diamond
Third diamond
Fourth diamond

Fill in the blanks to complete the summary.


The passage discusses the changes in public spaces _ _ _ _ _ by landscape
architect Frederick Law Olmsted. He realized that cities lacked a(n)

balance

of places in which to work, play, and live. Olmsted planned Central Park in New York around
the idea that people needed a(n)

in order to escape the

city life. While previous public spaces had

of

to certain groups of people,

Olmsted wanted Central Park to be accessible to all members of the public.


--------------------------------~--------

.. ----

~-

- -------------------------------------------------------------Text

l~ertron

Questions

l
I

'

135 - - \

Psychology

V~=~t.::~.\>>J~.\>.Tu-rx:.~-,.;.""\!'"'':f.~~-l)!M'<c<:.-.1;; ''~'"'I'

Read the following passage. Then fill in the diagram with the information that you read. @m

Why do people focus their attention on particular elements of their


surroundings while completely ignoring others? Selective attention
theories propose answers to this question. A) Most people have been
in .a situation where their attention was focused on a difficult issue,
such as a challenging homework problem, and b.ecame inattentive to
other stimuli in the environment. B) Selective attention theories,
then, attempt to describe how one can focus on a single consuming
issue and ignore other stimuli. C) These theories can be separated
into two distinct categories: filter theories and attentional resource
theories. D)
Donald Broadbent proposed the first filter theory in 1958. It would
become the basis for subsequent filter theories. A) In his theory,
Broadbent suggested that humans begin to filter information
immediately after it is heard. 13} For example, Broadbent believed
that humans disregard the chatter in a busy room if it has no relevance
to their current train of thought. G) Therefore, the brain must first
process the information and regard certain stimuli (one's name, for
example) as having higher importance. D) The brain, then,
processes anything that enters through the ear. However, not all
information enters the consciousness of the subject. Essentially, the
brain filtBrs anything that is unfamiliar. A familiar name or voice can
redirect one's attention.
In contrast, attentional resource theories state that attention is
dependent not upon filters but upon a fixed amount of attention that
one can deyote to tasks. + f..\) Imagine attention as a percentage.
Now, imagine that at any given time, one can devote one hundred
percent of his or her attention to a given task. + '8) Returning to the
example of the crowded room, a person would have to divide his or
her attention according to all stimuli in the room. + C) This person
might devote, for example, seventy percent of his or her attention to
a conversation with a friend. ~ !J) Thus, thirty percent of his or her
attentional resources would be available for other tasks. These tasks
could be listening to other conversations in the room or even thinking
about a baseball game. If he or she wanted to devote more attention
to the conversation, attentional resources would have to be redirected
I from another task.
L___________________ ---~------

j - 136
Af

Chapter 8

selective:
characterized by !he
ability to make choices
stimulus:
something that causes a
person to perform an
action
subsequent:
next in a series
disregard:
to ignore
relevance:
the telation to current
topic or matter

redirect:
to move from one place to
another
devote:
to commit to a particular
purpose

-------'

L
e

Selective Attention Theories ..

F' I-+ "'' "'"'

-+IL. .

'

(A)
'(B)
'.'; (C)
. i:(D)

'1

First square
Second square
Third square
Fourth square

J_..

-E-xa_m_p-le_:_ _ _ _ _ _

1-, =:]---+ ~'P''":=l-+


1. Look at the four squares [ l that indicate
where the following sentence could be
added to the passage.
These might include a conversation in
the same room or the noise from a
construction site outside.
Where would the sentence best fit?

__,I

l.__Exa-mple-:_

room, even if it was not a part of their


current conversation.
Where would the sentence best fit?
(A)
(B)
(C)
(D)

::;;

"'3
z

G'I

First circle
Second circle
Third circle
Fourth circle

3. Look at the four diamonds [ +l that


indicate where the following sentence
could be added to the passage.

2. Look at the four circles [ l that indicate


where the following sentence could be
added to the passage.
However, later filter theorists
recognized that people will still
respond to their na!lle in a crowded

This attention can be thought of in


numerical terms.
Where would the sentence best fit?
(A)
(B)
(C)
(D)

First diamond
Second diamond
Third diamond
Fourth diamond

Fill in the blanks to complete the summary.

..........-------------------- ..

------------------------"-~-------

.. ---" .... ,_.............. ~ ...... ----------------------~-~----

The passage discusses _ _ _ _ _ attention theories, which try to determine why


people focus their attention in different ways. Filter theories say that the brain filters out
unwanted information. For example, a person
they have no

conversations in a room if

to his or her thoughts. One's brain can filter unfamiliar

information and _ _ _ _ _ attention to familiar names or voices. Attentional resource


theories describe attention as a fixed amount to divide according to how much attention one
wants to pay to something. A person can _ _ _ _ _ a certain amount of attention to
certain stimuli.

rext Insertion Qu~st1ons

------------11111111.......--..........................................., -

.iY>

137 - - ,

!-''Y~i-:>:-,.

, - - - 138

Literature
Read the following passage. Then fill in the diagram with the information that you read.

i
i

A} Authors use many storytelling techniques to give the basic


events of their story a deeper meaning. B} In an allegory, the author
uses metaphor to such an extent that the entire story becomes an
extended metaphor for something completely unrelated to the events
of the story itself. C) Allegory can have many uses, though it is most
often used either to provide another way to understand a difficult
concept or to represent a story in a new, entertaining way. D)
Plato's allegory of the cave is an example of using an allegory
in order to help readers understand the concepts being discussed.
A) Plato believed in a world of forms, in which everything existed in
its "real" state. B) Realizing that these concepts are difficult to
understand, Plato wrote the allegory of the cave in his philosophical
. work The Republic. In it, Plato uses an extended metaphor to explain
his concepts. C) The allegory involves prisoners chained up in a
cave who can look only at the wall of the cave and cannot turn around
to view the entrance. The prisoners can see the shadows of the activity
happening outside, and they mistake these shadows tor reality. D)
However, one prisoner manages to escape his bondage and leave
the cave. He can now see the real world instead of shadows. The
allegory, then, is attempting to explain philosophical knowledge. The
prisoners represent the public, who can only see the representations
of the forms (the shadows on the wall}. Only by escaping the cave
can one see the real repr.esentation of the world as philosophers do.
+ A) In addition, writers use allegories to create a new telling of
a familiar story. An example is Animal Farm by George Orwell. + B)
Orwell wanted to write a story about the political situation in the
1940s. + G) Instead, Orwell placed his story in the world of a farm of
animals. Many of the characters were pigs, horses, and donkeys. In
the story, the animals stage a revolution and evict the humans from
their farm. However, the animals soon find that they are not fit to
govern the farm themselves, and their situation turns into a disaster.
Thus, Orwell's story is literally about animals taking over a farm. + D)
However, the allegorical meaning of the story describes Orwell's
viewpoint on a popular topic in 1940s politics. In fact, most allegories
can be understood literally, according to the events that transpire in
the book, and figuratively, in terms of what those events represent.

'-----------,--~-----------------.,~-----

Chapter 8

@1"

metaphor:

figurative language; a
conceptthat is used to
represent an unrelated
idea
bondage:

the state of being


restrained
stage:

to plan and carry out


evict:

. to put out from a building


. Ill: .
well suited
transpire:

to happen
figuratively:
using a metaphor; not
literally

"'
'

.
j

!'
;l

,.

f
f

''
}

)'.

'
;>

I'

_J

Definition:

.'

Role 1:

"'

Allegory';

Role 2:

~2

Example:

.....

Example:

Cl

"'
~
"'O

2
Cl

'

1. Look at the four squares []that indicate


where the following sentence could be
added to the passage.

One such literary technique Is called


allegory.
Where would the sentence best fit?
-'

,i

.~.

'

~;

(A)
.(B)
: (C)
'". (D)

First square
Second square
Third square
Fourth square

2. Look at the four circles [ l that indicate


where the following sentence could be
added to the passage.
Everything in our world, however, was
just a representation of the real version
of the object In the world of the forms.

Where would the sentence best fit?


(A)
(B)
(C)
(D)

First circle
Second circle
Third circle
Fourth circle

3. Look at the four diamonds [+l that indicate


where the following sentence could be
added to the passage.
However, had he written about politics
directly, the story would have been
simply a telling of current events.
Where would the sentence best fit?
(A)
(B)
(C)
(D)

First diamond
Second diamond
Th'1rd diamond
Fourth diamond

Fill in the blanks to complete the summary.


The passage discusses the use of allegory in literature. Allegory is used as an extended
metaphor for two reasons. The first is to explain difficult concepts. For example, Plato uses
the

of prisoners to explain philosophical knowledge. The second purpose of

allegory is to tell a familiar story in a new way. For instance, in George Orwell's Animal
Farm, farm animals - - - - - a revolution to ----~ humans from their farm.
They soon discover that they are not _ _ _ _ _ to run the farm themselves. The story's
events _ _ _ _ _ _ _ represent the political situation of Orwell's time.
-----------~--~.-----------~--------------

.. -------------------------------- .. -------,.-------------------------Text Insertion OuestiOns

139

--.1

I.

Instructions: Choose the best word or


phrase to complete each sentence.
1. The article was written by a famous
_ _ _ _ _ from a university in
Australia.

(A)
(B)
(C)
(D)

ancestor
scholar
pioneer
barrier

2. The people in the crowd had to


_ _ _ _ _ their necks in order to see
the fireworks display overhead.

(A)
(B)
(C)
(D)

precede
foster
manipulate
crane

3. The new law will be put into effect when


the council
next month.

(A)
(B)
(C)
(D)

convenes
renders
postulates
serves

..!
4. When I said that I was so hungry I could
eat a horse, I was speaking
(A)
(B)
(C)
(D)

passively
inevitably
actively
figuratively .

5. Side effects of this medication may


include headaches,
, and
nausea.
(A)
(B)
(C)
(D)

;:.

weariness
diversity
exertion
drowsiness

'

~t~~

.~~FF

'il:.ci
j\.'~<

140

Vocabulary Review 2

6. The maritime museum features a


miniature
of the Titanic
on display.
(A)
(B)
(C)
(D)

replica
fossil
counterpart
mechanism

7. Some slimming drugs are designed to


_ _ _ _ _ your appetite so you do
not feel hungry.

(A)
(B)
(C)
(D)

surpass
isolate
suppress
implement

8. The
that were caused by
the earthquake were so strong that the
pictures on the wall crashed to the floor.

(A)
(B)
(C)
(D)

migrations
tremors
stimulus
mediators

Instructions: Choose the word or phrase


closest in meaning to the underlined part
of each sentence.
9. Hikers may have trouble breathing once
they reach the mountain's peak due to
the extreme b.filgl]1.

(A)
(B)
(C)
(D)

elevation
composition
variation
proximity

-i-"~(,> -~ey""'''""''"'"'""''''"'" ,,,

. ,, '

f .. ',10. You are not allowed to bring in fruits,


ef

'\;, .
1,

:;

vegetables, or meats when you travel


to another country,
(A)
(B)
(C)
(D)

,,

'

.,

'

,.

15. The restaurant's policy not to permit pets


inside is strictly imposed.
(A)
(B)
(C)
(D)

import
cater
evict
repel

enforced
stationary
confirmed
cohesive

11. To ensure fairness, the teacher should 1!Y


llil1JQ call on the same student over and
over during class.
(A) adorn
(B) avoid .
(C) ignore
(D) spur

12. It is thought that about 44,000 Americans


. die each year in hospitals as a result of
'c' preventable medical errors.
W'
. (A) incorporated
>(B) instated
.: (C) disregarded
(D) estimated
13. Because the main road to the lake was
blocked by a fallen tree, we had to find
. an alternate road to the.cabin.
(A) cue
(B) terrain
(C) route
(D) sanctuary
14. My health is the main reason behind my

choice to quit smoking.


(A)
(B)
(C)
(D)

apparent
primary
proper
intuitive

Instructions: Write the missing words. Use


the words below to fill in the blanks.
fossils.

. preservecJ

instrumental

devote

identify"ing

Paleontology is the study of the life of Earth's


history as reflected in fossil records. From the
Greek "paleo" and "ology," paleontology
literally means the study of ancient beings.
16.
are the remains or
traces of organisms, such as plants, animals,
fungi, and bacteria that lived in the geological
past and are 17.
in the Earth's
crust Paleontologists 18. _ _ _ _ __
much of their time to digging up and
19.
fossils. Many of the life
forms that paleontologists study are extinct
These fossils are 20.
in
providing us with information about life that
existed on Earth long before humans.

Instructions: Match the words that are similar


in meaning.
exacerbate
subsequent
counterfeit
metropolis
25. regulate

21.
22.
23.
24.

(A)
(B)
(C)
(D)
. (E)

city
worsen
fake
adjust
following

''

'r-'

\~f,',,,~,

--1

j........- -............-.........- - -.......---.....Ji


Vocabulary Review 2 141

.- j"":~. ~

'k

n_

01 Zoology

~;~

H'

Read the passage and answer the questions. @it'

IG

.fj

;,n

J-

\,
i:

'"

'i

;'I

.,

"

Animals have their own methods of communication, whether they communicate through
body language or through rudimentary grunts and yelps. For example, giraffes press their
necks together to convey affection. Dolphins communicate with unique whistling noises.
However, can animals be taught to communicate using a form of human language? Many
scientists believe that chimpanzees are so genetically similar to humansthat chimps can
indeed be taught to communicate with human language. They have .spent years conducting
research to prove their theory.
To further explore the theory, the Language Research Center in Atlanta, Georgia, has qeen
performing language tests on chimpanzees since the 1970s. A) The most successful finding
occurred with a chimp named Kanzi. Researchers had been working with an older primate
when they discovered that Kanzi had learned the method of communication they had been
teaching the other chimpanzee. B) Using a keyboard with geometric symbols that stood for
certain words, Kanzi developed a vocabulary of two hundred words. C) Kanzi used the
keyboard and some gestures to communicate basic sentences. D) He even. seemed to
understand instructions. Scientists were astounded when Kanzi was instructed to "give the
dog a shot." Kanzi picked up a hypodermic needle and injected a stuffed toy dog.
While Kanzi's achievement was important, another chimp would later make a breakthrough
that showed true progress in the scientists' attempts. o A) A chimp named Panbanisha had
been taught to communicate with a keyboard similar to the one Kanzi used. B) One morning,
Panbanisha grabbed the keyboard and repeatedly typed in "fight," "mad," and "Austin." o C)
Another chimpanzee named Austin lived in a building near Panbanisha. That morning, Austin
had had a loud disagreement with another chimpanzee. D) It appeared that Panbanisha
had overheard his fight and decided to talk about it with the researcher.
Panbanisha's ability to create a crude sentence proved what researchers had hoped-that
chimpanzees could be taught to communicate about something more than just their basic
needs. Panbanisha's breakthrough showed that chimps could communicate about things
around them or, in this instance, could communicate gossip. Up to this point, researchers had
not seen such communicative similarities between animals and humans. The research with
Panbanisha proved that chimpanzees can be trained to communicate with people through
language. While this is a long way from having true conversation, the evidence shows that
animals can use human language to communicate on more than just a basic level with people.

r
-~

Ii
N

w
~

-f;

')

r
"

--~,

ges'\L1re an action intended to communicate feelings or intentions


----------------------------~---

.~..: - - 142 r:-'lini Test 2


'( .

1':;!

___________

~.:

;~

!_.-__ ,

~;i-,{1';
. 1. The word rudimentary in the passage is
. closest in meaning to
(A) peaceful
(B) basic
(C) plain
(D) authentic
Which of the following best expresses the
essential information in the highlighted
sentence in paragraph 1? Incorrect
answer choices change the meaning in
important ways or leave out essential
information.

;,

'

(A) Because chimps and people are able to


communicate using human language,
they have become genetically alike.
(B) The genetic connection between
people and chimps allows them to
communicate with one another
through human language.
Genetic links between chimpanzees
and people have led scientists to
think that chimps can communicate
using human language .
(D) It is genetically likely that chimpanzees
and people will communicate with
each other.

\,y)

.;,i

3. The word their inthe paragraph 1 refers to


(A) humans'
(C) animals'

(B) chimpanzees'
(D) scientists'

4. Look at the four squares []that indicate


where the following sentence could be
added to the passage.
Like human children, Kanzi learned
communicative skills through imitation.
Where would the sentence best fit?
(A)
(B)
(C)
(D)

First square
Second square
Third square
Fourth square

5. Look at the four circles [ l that indicate


where the following sentence could be
added to the passage.
Upon investigation, the research team
discovered what the chimp was trying
to communicate.
Where would the sentence best fit?
(A)
(B)
(C)
(D)

First circle
Second circle
Third circle
Fourth circle

'

[
['
c

...

'

6. The word his in the passage refers to


(A) Austin's
(B) Panabanisha's
(C) researcher's (D) Kanzi's
7. The word crude in the passage is closest
in meaning to
(A) short
(B) simple
(C) offensive
(D) unprocessed

a.

Which of the following best expresses the


essential information in the highlighted
sentence in paragraph 4? Incorrect
answer choices change the meaning in
important ways or leave out essential
information.
(A) Scientific proof indicates that animals
communicate with people using true
human language.
(B) Human language allows for meaningful
communication to occur between
animals and people.
(C) Because of scientific research, people
can now communicate with animals
using human language.
(D) Scientific proof reveals that some
meaningful communication between
animals and people is possible.

Mini Test 2

143 - -

]
I
I

c:2 linguistics
Read the passage and answer the questions.

!\'_,

@iii

-~--

~
1>,

l'
It is thought that at one point, there were over 200,000 languages in use around the world.
Today, there are approximately 6,800 living or actively spoken languages in existence. This
number includes 9bscure languages spoken by only a handful of people in remote parts of
the world. Even with the immense number of languages in active use, researchers in the field
of linguistics feel there are literally hundreds
of languages in danger of extinction. at a rate
.
faster than that of many endangered animals. A) In fact, linguists estimate that one language
falls out of use about every two weeks. B) The circumstances surrounding the disappearance
of languages can be attributed to different causes. C) Many of these languages also have
no written form, which makes them particularly susceptible to being lost and forgotten. !J)
Several cultures of the world are seeing a decline in their native tongues. A) Australia,
a very language-rich country, has already lost many Aborigine languages. Several more have
only one or two people still alive who can recall their specific intonations and word meanings.
e B) A similar situation is seen in North America. Native Americans in several areas have
tribal languages spoken on reservations that are becoming obsolete. e G) The absence of
those with knowledge of ancient languages leaves no one who can articulate the words
spoken by their ancestors. 0)
Many other countries are seeing indigenous languages being choked out by- the ..
prevailing languages. The Amazon region in South America has seen the rise of Spanish and
Portuguese as the languages of choice. The languages spoken by many people in remote
regions are no longer being learned by new generations. The reduction of minority languages .
in Siberia can be directly attributed to the Russian national government, which established.a
policy that forces speakers of minority languages to use only the declared national and
regional languages. This decreases the use of minority languages in Siberia and will eventually
lead to language extinction.
Of the thousands of languages still in existence, only eighty-three are considered to have
global influence. These are used by the vast majority of people in the world. Though several
groups have been working to capture and preserve endangered languages through written
and auditory means, most will be lost to daily use forever. Their regression is viewed as a
great loss of cultural understanding and heritage from the past.
.

t> ob""~ure

unknown to most people

&

t~
;

{'.j

ft

't
.iii
~;
}
j

rr

':,-

1~

..

The word remote in the passage is closest


in meaning to
(A) reserved
(C) available

(B) isolated
(D) dominant

2. Which of the following b~st expresses the


essential information in the highlighted
sentence in paragraph 1? Incorrect answer
choices change the meaning in important
ways or leave out essential information.
(A) As many languages are in danger of
being lost, so too are many animals at
risk of becoming extinct.
(B) Although there are many languages
in existence, they are becoming lost
more often than endangered animals.
(C) Although many languages are in use,
hundreds are at risk of being lost
faster than many endangered animals.
,.
" (D) While many animals face extinction,
linguists feel that the danger of dying
languages is greater.
3. Look at the four squares [ l that indicate
where the following sentence could be
added to the passage.
These causes range from the death of
the speakers to the more gradual
decrease in use as people learn more
widely accepted languages.
Where would the sentence best fit?
(A)
(B)
(C)
(D)

First square
Second square
Third square
Fourth square

-~'

First circle
Second circle
Third circle
Fourth circle

[I
"'>'"
"O

"'z
Cl

5. Which of the following best expresses the


essential information in the highlighted
sentence in paragraph 3? Incorrect answer
choices change the meaning in important
ways or leave out essential information.
(A) The Russian government is responsible
for a policy under which many people
cannot use their own native language.
(B) Minority languages are illegal in
Siberia due to the government's policy
that favors speakers of the national
language.
(C) Russia's government introduced a rule
favoring the use of local languages in
favor of li\tle-used languages.
(D) Speakers of Siberia's national
language are part of a policy made
by the government seeking to reduce
minority languages.

:;;

"'3z
Cl
"O

('I

'"....

....
"''"

6. The word This in the passage refers to


(A)
(B)
(C)
(D)

National government
Policy
Minority language
National language

(A) languages
(C) countries

(B) people
(D) groups

8. The word regression in the passage is


closest in meaning to
(A) decline
(B) slump
(D) collapse
(C) failure

Mini Test 2

i;.,,.

.J;.~.1(J

(A)
(B)
(C)
(D)

7. The word most in the passage refers to

4. Look at the four circles [ l that indicate


where the following sentence could be
added to the passage.
This is due to the death of the older
members who grew up speaking the
language.

Where would the sentence best fit?

:.

145 - -

.
i,; .- - - - - - - - -....- . . - - - - - - -. . .- - - - - - - - -

Necessary Skills
ll,X~~"W.ill!ll=~*n;;~"'"~-:.>;-Al;XT'~'"t{'<"4~1t-mi:ui;;;"'"'j~''H.t-f,;~~ ..

_,, .

Recognizing the organization and relative importance of information presented


in a passage
Understanding and locating specific points in a passage key to the gist of a
passage as a whole .
Organizing information presented in a passage into a mental outline

!Example Question
An introductory sentence for a brief summary of the passage is provided below.
Complete the summary by selecting the THREE answer choices that express the
most important ideas in the passage. Some sentences do not belong in the
summary because they express ideas that are not presented in the passage or
are minor ideas in the passage. This question Is worth 2 points.
[You will see a sentence !n bold here.]

Strategies
Distinguish main ideas from minor ideas, and essential information from nonessential information ..
Remember that correct answer choices will not be identical to any particular
sentence in the passage.

~:

:i1

): "''" ~~j
"

Prose Summary Questions

147 - - . .

Botany
-

C\!l~J'FtJll"<$'.:R~'Cit><<'""";1;:r-,-,1..-.>0.:\:c. ;: ~"

;s. , :.:.,-~,, '""_-,

'-~~

Read the following passage.' Then fill in the diagram with the information that
you read.
.
-

' .:

': '.I

'i .

.i

.,

Stems are the structures that support a plant's buds and leaves,
carrying nutrients to all parts of the plant. Water and carbohydrates
(sugars) are transported through the stem's interior tissues, which are
typically arranged in a concentric ring formation.
The outermost ring, called the phloem, is where carbohydrates are
transported from the leaves to the roots. This occurs through a process
.called the pressure-flow mechanism. A movement called translocation
transports nutrients from the source, where carbohydrates are made
and stored in the plant, to the sink, where they are needed. Sinks can
be roots, flowers, fruits, stems, and young leaves. As carbohydrates
enter the phloem from the leaves, water is transported in by osmosis,
which causes pressure to build up, pushing the nutrients downward
toward the roots.
Conversely, in the innermost ring of the stem, called the xylem.
water moves in an upward direction according to the cohesion-tension
theory. Since water molecules attract each other (a property called
cohesion), more water is pulled up from the plant's root system toward
the leaves. This movement causes tension within the stem's xylem,
creating continuous, thin columns of water that extend through the
stem. As water evaporates within the plant, the tension becomes
greater, moving the water molecules up the plant. Additional water
from the soil enters through the roots to sustain the process and the
life of the plant.

@ii'

bud:
an undeveloped stem
of a plant
concentric:
having a common center
source:
the origin; the place from
which something has been
obtained
osmosis:
the passing of fluid
through a porous
membrane
tension:
an opposite force that
results from stretching

attract:
to draw toward or pull by
physical forces
sustain:
to keep. an action or
process going

[_r~~-t-St=em=-s_-_-_,J
Definition:

~ r:1_:-~] ~

"

l._un-ct-ion-:- - -..--'

1. An introductory sentence for a brief summary of the passage is provided below. Complete
the summary by selecting the THREE answer choices that express the most important ideas
in the passage. Some sentences do not belong in the summary because they express ideas
that are not presented in the passage or are minor ideas in the passage. This question is
worth 2 points.

'

"'
~z
"O

Stems are the support structure of a plant's buds and leaves, providing a means of
nutrient transport and water replacement.

GI

Answer Choices
;: (A) The cohesion-tension process within the stem is essential for plant survival.
Nutrients are provided to the plants from the root system .
.. ( C) The xylem of a stem guarantees water delivery to the entire plant.
,. (D) Carbohydrates are transported through the phloem.
( E) The primary function of a stem is to support the weight of the buds and leaves.
( F) Concentric rings allow for the exchange of water and nutrients.

ti. (B)

Fill in the blanks to complete the summary.


Stems are the support structure for a plant's buds and leaves. - - - - - - rings within the stem transport nutrients downward from the _ _ _ _ _ to the plant's
roots through the pressure-flow mechanism. Water evaporates from the leaves, which
_ _ _ _ _ more water molecules upward from the roots. This process creates
_ _ _ _ _ in the interior of the stem for continuous water movement. More water enters
through the roots from the soil, - - - - - - - t h e life of the plant.
..........----------------- ..---------""---------------------------------.................... ,..................................... __________________ .. ___ _

It .. '.

Prose Summary Questions

J.1.('11111...

149

--1

111111111!11111111111111!11!!-li!!!!l"!l!lllll---------------.....:.-

d ...!l!lll,.
_11111..

Archaeology
~~;::w.M\H.');tii>"'f'H'.J>:,m~."'~J.-i'Cl"'";.,,,.

"-"'';_'.'.: . .,, : . .

Read the following passage. Then fill in the diagram with the information that you read. i@f}J

Experimental archaeology is a subfield of archaeology. It attempts to


replicate certain skills of past human cultures. Since excavated artifacts
can date back thousands of years, it is important for archaeologists
to determine what made them so durable. This information provides
essential insight into the lifestyles of past civilizations.
Experimental archaeologists want to understand the relationship
between ancient human activities and artifacts left behind. They
perform experiments to try to determine how the original articles were
created. Their goal is to recreate the entire process using only the
primitive materials and .methods that would have been available at
the time. By studying the artifacts, their approximate age, and the
region in which they are found, scientists are able to determine what
resources would have been available. They also assess what methods
would have been employed in their manufacture using only naturally
occurring or handmade implements.
An experiment was conducted to replicate the ancient art of paint
making in the southwestern United States. Experimental archaeologists
found that minerals, ores, and sedimentary deposits most likely provided
the pigments. These rocks were ground into fine, colorful powders
with a grinding stone. In order to produce a liquid paint that would
adhere to a surface, binding agents were needed. Studies of the
paint samples showed that animal fat was the most likely binder
used. Fat was extracted from the bone marrow of deer. This was
done by an archaeologist experimenting with a hammer stone and
granite anvil, tools that the original makers also would have used. The
test also proved that the fat and mineral pigments blended together
well. However, they were too thick to be applied. Therefore, the roots
of yucca, a plant plentiful in the area, were pounded with stones.
Then they were soaked with water and squeezed out as a liquid. This
substance was combined with the pigment and fat ingredients to
create a silky, fluid, durable paint. The entire process replicated the
methods and materials rnost likely used by the ancient paint makers.

::

f"'<f--- 150 thaptor 9

!;

1ji:
f;.,

replicate:

to copy or reproduce
durable:

able to last
primitive:

the first or earliest of its


kind
sedimentary:

setlling to the bottom


pigment:

a material that provides


color
extract:

to pull something out


anvil:

a large block used to


shape metals

Definition:

Role:

Example:
Ill '
"'O

~z

Cl

1. An introductory sentence for a brief summary of the passage is provided below. Complete
the summary by selecting the THREE answer choices that express the most important ideas
in the passage. Some sentences do not belong in the summary because they express ideas
that are not presented in the passage or are minor ideas in the passage. This question Is
worth 2 points.

Experimental archaeology attempts to recreate the methods and materials used to make
ancient artifacts.

Answer Choices
(A)
( B)
(C)
(D)
( E)
( F)

Experimental archaeologists use the actual tools and techniques of ancient peoples.
Ancient paint-making techniques utilized plant, animal, and mineral ingredients.
Animal fat is the prinCipal color-producing agent in most ancientpaints.
Experimental archaeologists combine modern techniques with ancient knowledge.
Experimental archaeology attempts to discover how ancient peoples lived.
Studying past human behavior reveals the efficiency of using available materials.

Fill in the blanks to complete the summary.


Experimental archaeology attempts to _ _ _ _ _ certain skills of past cultures. By
doing this, we can determine what made ancient artifacts so
use only the

..Archaeologists

materials and methods available at the time the items were

made. One experiment recreated the process of paint-making using ores, minerals, and
_ _ _ _ _ _ _ deposits. Then, animal fat was

using tools

available at the time. Finally, yucca roots were added to recreate the substance likely made
by ancient paint makers.
-----------------------------------------------------------~------

~
,I,

r . ,.

. --------------------------------------------------Prose Summa(y Questions 151

__:__j

. -~\, '.f~i:,~. '-

ic --.~->---;;---

. . . . .--.(J/
. . .llllllllllllllllllllllllllllllllll!llll!llll!ll!lillllll
. . . . . . . . . . . . . . . . . . . . . . . . . . . . . . . . . . . . . . . . . .,
-- ----- ' - -

History
-=~-===.t,,tr~~~"'l:<'J'J=c~-..n:.::c:1'.o",-:;:;:;!~1'.:>C;C _-d-.tv~,,:: -~-c,

Read the following passage. Then fill in the diagram with the information that you rea~. @!iii

: i'

The gold rush of 1849 in California brought thousands of


newcomers to the state. They came in search of fortune and a better
life. Although most of them came to dig for gold, other opportunities
were inadvertently created for entrepreneurs of a different kind.
The gold rush caused crowded camps to appear all over
California, giving rise to competition for basic daily needs. Food,
clothing, and other necessities could hardly be produced fast
enough to keep up with the growing masses. People from all types of
backgrounds quickly discovered that there was just as much fortune
to be made serving the gold diggers as there was in digging for gold.
With fierce and increasingly violent rivalry to find more gold happening
all around them, some people saw an opportunity to make a better
living by providing other goods and services to the rapidly growing
population. A few had inflated their prices and taken advantage of
the miners' plight. However, most were simply honest, hardworking
businessmen with creative ideas.
One such entrepreneur was a young merchant named Levi
Strauss. He was best known for a prosperous dry goods business
that manufactured various types of apparel. Strauss created a pair of
sturdy pants out of canvas, which became very popular among the
miners. The pants were durable enough to withstand the harsh
conditions of the miners' activities: stooping, kneeling, bending, and
crawling in mud and on rocky surfaces. Ultimately, he added a
critical element to the trousers: the metal rivet, which provided the
best reinforcement for the laborers' work wear. This feature, still used
in the manufacture of jeans today, changed the course of American
fashion and put Mr. Strauss on the road to unparalleled success. Levi
Strauss was one of the first of many entrepreneurs to make hisfortune
during the gold rush without digging for a single nugget.

Inadvertently:
unintentionally
entrepreneur:
a risk-taking
businessperson
necessity:
something required
rivalry:
a feeling of competilion
Inflate:
to raise prices
plight:
an unfortunate situation
apparel:
clothing
rivet:
a bolt used lo laslen two
pieces of material together
reinforcement:
something that
strengthens

.t

:-~

------------------------------------------------'

r------------------------------

1___ Ent~epreneurs~~~_:irin~~ California Gold Rush

-----------1

Cause:

_ _J

[E!i;;;~---------.----1

I Example: --

--+l

L__
_

!--+!
r

l
!

~--------J

_J

'

An introductory sentence for a brief summary of the passage is provided below. Complete
the summary by selecting the THREE answer choices that express the most important ideas
in the passage. Some sentences do not belong in the summary because they express ideas
that are not presented in the passage or are minor ideas in the passage. This question is
worth 2 points.

The rapid arrival of people during the California Gold Rush created a pressing need for
goods and services.

~z
z
Cl
111

-o

~z

Cl

"'O

'"

Answer Choices

,, -

,,,(A)
,, ( B)
. ( C)
, (D)
' ( E)
( F)

The fortune seekers of the gold rush were unable to meet the miners' daily needs.
Miners became hostile over inflating prices of goods and services .
Opportunities arose for new ways in which creative people could earn a living.
Levi Strauss made his fortune by manufacturing durable workmen's trousers.
The rapidly growing population created thriving communities.
The demand for goods and services increased faster than the supply.

, Fill in the blanks to complete the summary.


The 1849 gold rush in California------- created opportunities for
entrepreneurs to make their fortunes in ways other than mining. A rapidly growing
population increased the demand for basic _ _ _ _ _ _ _ such as food and clothing.
While some merchants _ _ _ _ _ their prices to take advantage of the miners'
_ _ _ _ _ , many were honest and hardworking. Levi Strauss was a merchant who sold
various types of

, including a type of work trousers that used metal rivets. This

provided a type of pants that fulfilled miners' needs in a new way and led to his success.
---------------------------------~:..--------------- ...

------------------------ . ----------------------------------

r
Prose Summary Questions

.---'-

---

,,

.,

153 - -

Read the following passage. Then fill in the diagram with the information that you read.

@it

;:;

'''

;:

The scattering of sunlight off the molecules in the atmosphere is


responsible for the blue appearance of the sky. Usually, we see light
in its full visible spectrum, which appears white to the human eye.
However, when light rays are broken or refracted by water in the
atmosphere, its colors appear separated. This is because each color
has its own wavelength.
The reason the sky appears to be blue was first investigated by
British physicist Lord Rayleigh. He discovered that sunlight is
scattered by water molecules in the air in the same way that white
light is scattered by a prism. The visible light spectrum occupies only
the portion of the entire electromagnetic radiation field that we can
see. The spectrum includes the colors of a rainbow we can sometimes
see in the sky. It ranges from red, classified as the longest wavelength,
to violet, classified as the shortest Since this scattering in the sky is
more effective at shorter wavelengths where the blue end of the visible
spectrum is, the sky appears to be blue. Although all the colors of the
visible light spectrum are present in white sunlight, the blue wavelength
is most easily detected by the human eye, due to the angle at which
we view sunlight.
As we view the sunlight away from the sun, it is scattered through
more atmospheric molecules than if we looked more directly toward
the sun. This scattering, called Rayleigh scattering, is predominantly
in the blue end of the light spectrum. It is because of the shorter.
wavelength of blue that the light is more effectively scattered. If we
look toward the sun, the full spectrum of white light is more dominant;
further from the sun, a deeper blue is visible. In other words, the more
air molecules light travels through, the more saturation of color we
can detect.

scattering:
the process by which
things are split apart

~:

spectrum:
the band of colors present
in light
refracted:
changed in direction and
speed
wavelength:
the distance between two
peaks of a wave
range:
to vary within certain limits
predominantly:
mainly; primarily
saturation:
the degree of purity of a
color

. .i

r--------~

--1

\Theory~---.

I~

Su~p-or~1-~_

I~ I Support 2:
I

IL ___

'

L
''

..

,,_

Why the Sky Is Blue

.. ___________

--i
l

,'l

!"

--~

';,,

----

~!)

.;

1. An introductory sentence for a brief summary of the passage is provided below. Complete
the summary by selecting the THREE answer choices that express the rhos! important ideas
in the passage. Some sentences do not belong in the summary because they express ideas
that are not presented in the passage or are minor ideas in the passage. This question is
worth 2 points.

The passage discusses the scientific reasons for the appearance of a blue sky to the
human eye.

Answer Choices
;(A) The closer we look toward the sun, the more colors of the visible spectrum we can see.
c:i(B) Sunlight scatters off of molecules in the air more effectively at shorter wavelengths.
( C) The sky looks bluer farther from the sun because of the decrease in light waves.
(D) The visible spectrum includes all colors of the rainbow.
( E) Water molecules in the air scatter light in the same way as a prism.
( F) The blue range of colors have short wavelengths.

.t

Fil.I in the blanks to complete the summary.

'

!: .

The sky appears to be blue because of the _ _ _ _ _ _ _ of light waves against

water molecules in the atmosphere. The colors of the light spectrum we can see
_ _ _ _ _ from red, which has the longest wavelength, to violet, which has the
. shortest. When light rays are broken or _ _ _ _ _ by water in the atmosphere, colors
appear separated because of their wavelengths. The scattering of light molecules in the
'

sky is

at the blue end of the light spectrum because of its shorter

. wavelength, most easily seen by the human eye. Farther from the sun, the greater
- - - - - - - o f color makes the sky appear a deeper blue.

----------------------------------------- ..

-----------~---------

.. ------------------------------------------------

--1

Prose Surnrnary Questions 155

Art History
..

~~"t.."Wi!i0'4.'1f>~;~.li!<'9:.:'<fl'.H~.th<WWa.;~-'<,_,~f;A'

J\;,-l'l'JJi' ,_,,,.,-

Read the following passage. Then fill in the diagram with the information that you read. l@fl:I

Alfred Stieglitz made art history in the 1920s by advancing his


photographic work from simple images of the world captured on film
to expressions of artistic interpretation as seen through the lens of his
camera.
While spending time in Europe, Stieglitz's photographic skills
became widely respected. He returned to New York City in 1890 to
become a partner in the Photochrome Engraving Company. The art
movement in Europe had been a source of inspiration to him. He soon
found himself more interested in promoting photography in the US as
an artistic expression than producing standard photographs. His
enthusiasm, however, was not as well received in the American art
world. The brief but important Dada art movement had gained broad
recognition in Europe as a creative social outlet. Stieglitz became
involved with the movement in New York. However, it did not have a
comparable impact and quickly disbanded. Dada art, however, had a
profound impact on the way Stieglitz viewed his work.Consequently,
he helped to establish the Camera Club of New York. He also edited
and published the periodical Camera Work. Both of these endeavors
served to promote the artistic side of photography to the public.
Stieglitz focused much of his work on the urban surroundings of
New York City. He used the play of shadow and light rather than
objects as the main content of his photographs. Stieglitz's series of
photographs of clouds and sky was called Equivalents. He undertook
the project in an effort to demonstrate that form rather than subject was
what mattered in creating an expression of visual art. The incorporation
of abstract patterns was his way of conveying emotion. By capturing
these simple images so completely, Stieglitz believed that all who
looked upon it in the future would experience a feeling equivalent to
the one he experienced viewing the scene in person. In an unparalleled
career that spanned the transition from the Victorian to the modern
world of art, Alfred Stieglitz has likely had a more profound influence
on the shifttoward aesthetic photography than any other individual.
His sensibilities toward this evolving art form showed true maturity in
his craft. Stieglitz's photography embodied his desire to try new
things in order to exhibit beauty in that which previously appeared

L--~~-v-~~~-~'.'.:

'~

'

promote:
to advance
Dada:
an art movement of the
early 2Qlh century
concerned with strange
and nonsensical art
outlet:
a means of expression or
satisfaction
impact:
an influence
disband:
to split apart
periodical:
a magazine distributed at
regular intervals
equivalent:
.similar
aesthetic:
concerned with emotions

__________________________________________________________.________________________

156 Chapter9

...

expression:
the communication of
thoughts and feelings

'----'"----';

__

, ,

\'

'

__j

Definition:

Role:

Example:

1. An introductory sentence for a brief summary of the passage is provided below. Complete
the summary by selecting the THREE answer choices that express the most important ideas
in the passage. Some sentences do not belong in the summary because they express ideas
that are not presented in' the passage or are minor ideas in the passage. This question Is
worth 2 points.

Alfred Stieglitz attempted to transform photography to an expressionist art form .

.;~.__

.;'!:

'i,\

~>

t'),

'

'

-;~~'

Answer Choices
(A)
( B)
( C)
(D)
( E)
( F)

The modern shift of photography was not well accepted in Europe in the 1900s.
Images of clouds and sky were meant to recreate an emotion for the viewer.
Stieglitz was inspired by the non-traditional approach of the Dada art movement.
Stieglitz's partnership in the Photochrome Engraving Company launched his career.
Photography as an art form quickly became popular in the United States.
Stieglitz sought to show beauty in images that were not ordinarily seen as beautiful.

Fill in the blanks to complete the summary.

Alfred Stieglitz was a photographer who made his work a(n) - - - - - - - of


artistic interpretation of the world around him. Dada art had a profound _ _ _ _ _ on
the way he viewed photography, so he became interested in the _ _ _ _ _ _ _- of photography as art. Stieglitz edited and published a photography

and

focused much of his work on urban surroundings. His Equivalents series was intended to
completely capture a moment in time, thereby affording the viewer a(n) - - - - - - experience of the feeling of that moment.
;

..

Prose Summary Questions

157 - -..

a.,:' .
_L

Sociology
~l!ll:l:ma..\!/:'.l,-,'{S'l'7"'i'Y>H""'~l>li!-i'"<i::<,).il'..1!1;,).>,~~_,.~,-_- "/.-. FAi".',0.,;._'__::,; ';, :~,,;:

Read the following passage. Then fill in th~ diagram with the information that you read. +@!!iii

The debate surrounding media's role in socialization intensified


after the television was first made commercially available in the
1930s. Today, the role of television is even stronger. According to a
leading polling firm, ninety-nine percent of American households own
at least one television. The people in those households watch an
average of seven hours of television each day. Thus, television is
undeniably an important part of modern culture. The benefits of
media such as television, however, are debated. Many believe that
media provide many social benefits. Others feel that their effect on
socialization is entirely negative.
Those in support of media as socializers cite their educational and
interactional benefits. Especially for children just learning to socialize,
many popular educational programs can help teach valuable life
lessons. For example, Sesame Street, a popular American television
show, focuses on teaching children academic information such as
mathematics and valuable social skills like sharing. Various studies
have indicated the benefits of this program. One study in particular
showed that children who regularly watched Sesame Street
consistently received higher grades in school than those who did not.
Moreover, media can provide people of all ages with topics that allow
them to socialize more effectively. For example, television provides a
number of popular programs-from news to comedy-that children
and adults alike can discuss. A person in a social situation can
certainly find a shared topic to discuss from one of the many television
programs being broadcast daily.
Those against the media as a socializer find grounds for contention
in these same areas. They claim that the education and interaction
offered by the media are often negative rather than positive.
Programs on television are often not educationally based. This is true
even among those designed for children. Many animated children's
shows are based on violence. In addition, acts of violence can be
witnessed on any local news program. Statistics suggest that the
average American will view over 200,000 acts of violence on television
by the age of eighteen. These statistics suggest that television teaches
violence rather than academics or social skills. Critics of television as
a socializer also suggest that, rather than giving people a common
ground to discuss topics, it implants often unrealistic stereotypes.
These stereotypes can then be perpetuated by those who watch
them. For example, a violent television show may be set in a city that,
in real life, is very peaceful. However, viewers may develop a bias

l---~~ar~ ~~~~~i~~~~8. t~ '.~s-re~r_8.~~-~'.~tio~~: the sh-~w~------------

soclallzatlon:

the process of learning to


socialize with others
Intensify:

to become stronger
undeniably:

unquestionably
lnteractlonal:

relating to how people


converse
grounds:

reason or basis
contention:
an argument; a dif?pute
implant:

to place into
stereotype:

a simplified or distorted
image

t -:~
'

___ I
- f.

,:.. , . - - 158

..
i_

:l-

,,1-:

Chapter 9

_A_d-va-n-ta_g_e_1_:_ _ _ _ _ _ _

__,J JDi'""~lgo i

_A_,d-va-n-ta_g_e_2_:_ _ _ _ _ _ _

__.l IDi"'""""'' ''

._I

._J

!; .
f

:~

::;,.,.-:

i--

"

1. An introductory sentence for a brief summary of the passage is provided below. Complete
the summary by selecting the THREE answer choices that express the most important ideas
in the passage. Some sentences do not belong in the summary because they express ideas
that are not presented in the passage or are minor ideas in the passage. This question is
worth 2 points.
This passage discusses some of the ways in which media act as a socializer.

i
w_:

jy
,'

Answer Choices

r ,

'

(A)
( B)
( C)
(D)
( E)
( F)

Many believe that children should not watch television due to the violence it portrays.
Television viewers often hav.e stereotypes about cities due to negative depictions ..
Many television programs benefit children by teaching school subjects and social skills.
Television contains many stereotypes that can be learned and spread by viewers.
Many children's programs contain violence and can teach children the wrong lessons.
While some feel media have a role in socialization, others argue they have no role at all.

___ ,, .... ,.,..,-------------------------m.----

The passage discusses the media's role in socialization, which has only _ _ _ _ _ __

as television's role has grown in society. Television is

'

part of modern culture. The media have educational and

---.

Fill in the blanks to complete the summary.


----------------------------------<>---'<-<>----~-----wwM

.-1;_

an important
advantages

and disadvantages. Many people believe that the media help educate children and give
I

f'

people common topics to discuss in social settings. However, others believe that the media.
promote violence and _ _ _ _ _ often unrealistic _ _ _ _ _ __

......................------------------------------------------.--------------------------------------------------------------Prose Summary Questions

- --

-~---,---

--,,--,-

159 - -

;]
l:'

i"'i1

l\lecessary Skills
-,

~--if;,_;;:-<'.'ff!m>J11:>:,2:,,o

__ ,:_";J.C;_~,_,,,_, ,y-, . , ,

,.- ' . , - -

Recognizing the organization and purpose of a passage


Understanding rhetorical functions such as cause-effect relationships, comparecontrast relationships, and arguments
Identifying and organizing important ideas and points from a passage and
placing them in the appropriate context

Complete the table below to summarize information about concepts discussed in


the passage. Match the appropriate statements to the concepts with which they
are associated. TWO of the answer choices will NOT be used. This question is
worth 3 points.

Separate main ideas from minor ideas, and essential inform?tion from non-essential
information.
Remember that major ideas are ones that would be included in a fairly detailed
outline of the passage.

Table/Chart Qutlstions

161 _ _ _,

Geology
~l!J.~.~;m!wmv...imtti'lJl'lll'J!J:l!.';,,S'll..\i't'l"...:iw.'N~':\:fa:>/O",;>;n:w,,,.,_,. ._.,~~,-N;

, ... _:

~"'''--

Read the following passage. Then fill in the diagram with the information that you read.

An earthquake generates shock waves traveling out in all directions,


both in the Earth's interior and along the Earth's surface. These shock
waves are called seismic waves and can be felt by people living
many miles away from the earthquake.
P waves and S waves are the two kinds of interior seismic waves.
P waves are primary waves that travel outward from where the
Earthquake originates beneath the Earth's surface. The fastest of all
seismic waves, P waves, can move through any medium of the
Earth's interior: solid, liquid, or gas. They alternately compress and
expand the medium so that P waves vibrate in the same direction in
which they travel.
On the other hand, S waves, or secondary waves, can only travel
through solid mediums. They create vibrations at a right angle to the
direction in which the waves are traveling. This results in the shaking
characteristic of an earthquake, where rocks move up and down or
from side to side.
Because the speed of S waves is always slower than that of
P waves, scientists can compare the arrival times of the two types
of interior waves. This allows them to pinpoint the location of an
earthquake, even if it is thousands of miles away.

r- Seismic Waves _]

ili!oR

generate:

to cause
seismic:
relating to vibrations of the
Earth
originate:
to begin; to start
medium:
material or surroundings in
which something exists or
moves through
alternately:
by taking turns; first one
and then the other
compress:
to squeeze together
pinpoint:
to identify; to locate

Directions: Complete the table below about the types of seismic waves discussed in the
passage. Match the appropriate statements to the type of seismic waves with which they
are associated. TWO of the answer choices will NOT be used. This question is worlh 3 points.

Seismic Wave

Statements

P waves

S waves

'

Answer Choices
.'

' ~

(A)
,,, ( B)
,. ( C)
.: (0)
, ( E)
( F)
(G)

Travel on the surface of the Earth


Produce vibrations that move in the direction of the waves
Can only move through solids
Produce vibrations that move up and down or side to side
Move through underground water
Are the fastest moving waves
Can vibrate at any angle

Fill in the blanks to complete the summary.


................................................................................................................................................

---------------------------~-------

.. ---------

According to the passage, earthquakes _ _ _ _ _ shock waves known as seismic


waves. Two kinds of seismic waves travel through the Earth's interior. Primary waves, or P
waves, travel outward from where an earthquake _ _ _ _ _ . They can move through
solids, liquids, or gases under the surface of the Earth, alternately------- and
\

expanding them. S waves are secondary waves that can only move through solids. They
vibrate at right angles to their direction. S waves travel more slowly than P waves and can
only travel through solid

. Scientists can compare S waves and P waves to

_ _ _ _ _ the location of earthquakes, even thousands of miles away .


.................................................... .. ___________ ., ............................................................................................................................ _.. _____________ ., ___ _
~

Table/Chart Questions

163 - -

Anthropology
:i:~&~~tm<-0<:~0.~,;r;J1rn:o\vt:1\.'.; :,:_;-;.;r;;~

c<:.::,,-_,."";; >C'',-_,- "'"' -, ..

-;~-

Read the following passage. Then fill in the diagram with: the information that you read. +@!!fill
'

-:,'.':_~=-. \

.,,,.

The strategy a non-industrial society uses to obtain the resources


its members need to survive-especially food-affects its social
organization. Thus, societies using the same system of economic
production will share some social characteristics, even though they
are located in different parts of the world.
For much of human history, human beings were exclusively
hunter-gatherers, foraging to meet their dietary and other needs. In
such egalitarian societies, the basic social unit is the band. A band is
a group of usually fewer than a hundred people, whose members are
related through marriage or kinship. Everyone in the band gets an
equal share of meat, and there is great social mobility. People marry
outside their own band, so a person can choose to live with bands of
his or her parents or grandparents. People can change their band
membership several times in a lifetime. After marriage, a woman may
move with her husband between her band and his.
About 12,000 years ago, .humans developed food production in the
form of farming and herding. As societies adopted this new economic
strategy, social structures changed. A pastoralist, or herding society,
is also based on small groups but is much more hierarchical. The
main social unit is the extended family, with a male elder leading his
sons and their families. Pastoralists rely on a domesticated herd of
animals, for instance, cattle or camels, for food and other products. The
herd must be moved from place to place, depending on the seasons,
to find food. When the entire group-men, women, and children-move
with the herd, anthropologists categorize the movement as "pastoral
nomadism." In some herding societies, only part of the group moves
with the herd, leaving most of the members behind in a home village.
Anthropologists call this "transhumance."

~i
I

'

'
.
Hunter-Gatherers

exclusively:
only
forage:
to look for food
egalitarian:
having equal social,
political, and economic
rights
kinship:
a family relationship
adopt:
to choose to follow
something
hierarchical:
separated using different
levels of ranking
domesticated:
tamed; brought under
control for human use

. -. 'f;. _'.~_

.
----1----------. f-'-----!-~~~_:.-~-----r---Pastoralists

--~-----

=i-

'

l__:.________________ L_. ________~------------l _______ ,___J


:~,____

~!i

'j'

i~ )

164

Chapter 10

1. Directions: Complete the table below by summarizing the characteristics of hunter-gatherers


and pastoralists discussed in the passage. Match the appropriate statements to the society
they describe. TWO of the answer choices will NOT be used. This question is worth 3 points.
Society

C\

Statements

Hunter-Gatherer

Pastoralist

Answer Choices
. (A) A married woman has a choice of several different groups with whom she can live .

. (B) Herds often split into groups that move separately .

. '
l

.;,;(C) Women and elders are the most important members of the group .
' (D) About 12,000 years ago, humans thought to develop this method .
. ( E) Members of the group are treated as equals .
( F) Members own an equal share of herded animals.
(G) Sons usually live in a group with their fathers, even after they marry.

-~

---.

Fill in the blanks to complete the summary.

-. ------------------------.. ----..--.. ---------------------.----"--------------------------------------The way in which a pre-industrial society finds or produces its food affects its social
structure. Hunter-gatherer societies, in which members

for food, are

- - - - - - - Members are usually related through marriage or _ _ _ __


Everyone gets an equal share of the meat, and they can easily move from one band to
another. Pastoralist societies are

. Usually an older male member

heads up the group, which consists of his sons and their families. Pastoralists also rely on
- - - - - - - animals for food and other products.

..
(
k
r---,.

"--------------------------------------------------------------------------------------------------------------Table/ChJrt Questions

165 __,.

Philosophy
~~w;~l<;lllS~}y~,:o/'~4'-Uo'rl"'c,\,J~'>'!.M,\Ci.\+.::_,

~~---c~ ~

".

Read the following passage. Then fill in the diagram with the information that you read. @iH

Plato and Aristotle were two of the earliest Western political


philosophers. While both shared many beliefs-Plato was a teacher of
Aristotle-Plato and Aristotle differed on how to achieve a good
government.
Plato believed there was a world beyond human senses, a world
of what he called Forms. Human senses-sight, hearing, touch, smell,
taste-are not perfect and therefore cannot give man a perfect
rendering of the world. Instead, it is only through the human mind that
man can truly know Forms, that is, reality. In Plato's view, only a small
minority of men, genuine philosophers, are capable of obtaining this
otherworldly knowledge. For Plato, therefore, the ideal form of
government was the philosopher-king. Only a philosopher-king could
ensure that the state promotes the ideal of the good life, a concept
derived from the world of Forms that only a philosopher could
understand. In the absence of a philosopher-king, Plato turned to the
rule of law as the subsequent guarantee of good government.
Aristotle defined the problem of governing differently from his
teacher, rejecting Plato's theory of Forms. For Aristotle, what is
perceived by the senses is indeed the real world, and man can gain
knowledge of it through observation and study. Aristotle agreed with
Plato that government should promote the good life and the rule Of
law was necessary to that goaL For Aristotle, however, the rule of law
was not a second choice but was preferable to rule by any one man,
the concept of a philosopher-king notwithstanding. Aristotle's theory
that even the ruler of a state must be subject to its laws formed the
underpinning of modern constitutional government.

rendering:
an interpretation

ensure:
to m~ke certain
derive:
to create from an original
source
rule of law:
the idea that people are
governed by laws that
apply to and are known
by everyone, and not by .
orders of rulers
subsequent:
next in a series
underpinning:
a foundation; a support

--]

1. Directions: Complete the table below to summarize information about the two philosophers
discussed in the passage. Match the appropriate statements to the philosopher with which
they are associated. TWO of the answer choices will NOT be used. This question Is worth
4 points.
Philosopher

Plato

Aristotle

Statements

[
[(
[

Answer Choices

\l

1)

;;(A) Man perceives reality through his senses.


v(B) Only a philosopher can understand the ideal of the good life.
~
. (C) A world exists that is beyond the human senses.
f
1;. ' (D) A constitution is necessary for good government.
( E) The world of Forms is the true reality.
'
( F) A government based on the rule of law is the best method.
(G) Rule of law is necessary for good government in the absence of a philosopher-king.
(H) A person with well-developed senses can perceive the world of Forms.
( I ) Even ruler.s must follow the laws of the. state.
Fill in the blanks to complete the summary.

According to the passage, Plato believed reality existed in a world of Forms that could
only be understood by philosophers. Because human senses cannot give a perfect
_ _ _ _ _ of the world, it is only through the mind that humans can truly know reality.
For Plato, the ideal form of government was a philosopher-king, who
that the concept

could~----

from the world of Forms was promoted. Without a

philosopher-king, Plato believed that the rule of law was the

form of

good government. Aristotle, however, believed that what people perceived through their
senses was reality. His theory that even rulers must follow the rule of law formed the
- - - - - - - of constitutional governments today.
________ .:. ___________________________ n------------<'""-----------.. ----~---1..--------.,-------------------------

Table/Chart Questions

ii.;--------...............

:.t.

---1111111------.. .

1111111. . . . . . . . . . . . . . .- -. . .

d.>-',c

167 ---...

1 '

Art riistory
.U:t;'llf);l'l ,;,J

,J

'l~'Ji.<.t:<.~~'-'.>><<l>!.','./ ,:,_~~'-"

Read the following passage. Then fill in the diagram with the information that you read. I@@

'

'

The Baroque period in Western architecture spanned the 171h


century and first half of the 181h century. The Rococo style then
emerged in the early 18th century as a reaction against Baroque
excesses. However, it actually was a refinement of Baroque elements.
Four key elements identify Baroque architecture. First, buildings
tended to be massive, creating a sense that their importance was
larger than life. For example, palaces from the era were visual
reminders of the poVl(er of the state, and impressive churches
encouraged the faithful in their beliefs. Second, painting and
sculpture combined with architecture to create a decorative Baroque
unity. Sculpture, for instance, was bold and dramatic, like the
buildings themselves. Third, to bring that same drama to the interior,
Baroque architects used vivid colors and .luxurious materials of
different textures. One example is the covering of church ceilings
with brightly painted scenes. A fourth element was the design and
decoration of the interior space. This offered a variety of views
leading off the main visual axis of the building, usually'leading from the
entrance. Here, too, powerful sculpture and lighting effects were used
to draw the eye to various parts of the interior. This was done so that the
interior offered a series of dramatic and distinct visual experiences.
The Baroque style proved to be too robust for the more graceful
tastes of the 181h century. In response, the Rococo style emerged.
Rococo, rather than being a style apart from Baroque, is better
viewed as a style of decoration applied to Baroque elements. Rococo
decoration was more delicate and subtle. By using smaller columns
and continuous smooth surfaces in place of contrasting textures,
Rococo buildings appeared less massive and imposing. While the
integration of painting and sculpture with architecture remained
important in Rococo design, Rococo sculptors preferred to work on a
smaller, more intimate scale. Pastel tones replaced the vivid colors of
the Baroque style, and mirrors were popular decorations. Graceful
S-and C-shaped swirls and curves decorated walls and ceilings so
much .that critics attacked the style for its overuse of ornamental
decorations. Windows were added to building designs to let in more
light. This either softened the impression of the interior or created a
dramatic effect. In general, though, interior space became more
unified, rather than drawing the eye to various dramatic views.

refinement:
a slight change; a small
improvement
axis:
an imaginary slraight line
through Iha. cenler of
something
robust:
forceful; strong; tough
subtle:
not very obvious;
suggestive
imposing:
overwhelmirlg; impressive
Integration:
Iha state of combination; a
mixing together to make a
whole

.
/
l---~--nn--------------------~~-----<>v~,.---------~-">------~----~~------J
;

f ' i - - - 168 Chapter 10

n
j!

l'

'

'

'

-;-\.

-----------------

Baroque vs.

Rococo~

-----------..--~-----_,....,---,,

Rococo
Both
-l-------.----------1--------------i

Baroque

~------------

---

1. Directions: Complete the table below with information about the two styles of architecture
discussed in the passage. Match the appropriate statements to the style of architecture they
describe. TWO of the answer choices will NOT be used. This question Is worth 3 points.

[
[

.
i

'

Architectural
Style
.

Statements

Baroque

Rococo

<

. 'i

Answer Choices
(A) Swirls and curves were used in interior design.
( B) Sculptures tended to be smaller and less dramatic.
(C) Palaces were especially massive and imposing.
(0) Painting did not contribute to the overall style.
(E) Different, dramatic views were used in the design of the interior.
( F) Style emphasized simplicity and plain designs .
(G) Paintings using bright colors decorated church interiors.

Fill in the blanks to complete the summary.


There are four key elements of Baroque architecture. Buildings were massive.
Architecture, painting, and sculpture worked together. The interiors used bright colors and
different materials, and there were dramatic views off the main visual _ _ _ __
Baroque architecture was too

for the tastes of the time. However, Rococo

architecture was a(n)

of these elements. Its decoration was more

delicate and _ _ _ _ _ . It also featured the _ _ _ _ _ _ _ of painting and


sculpture with its architecture.
"-''

~- .~

"-~"'

- -

-- .... a -,.- -

--- - -

-- ----- -

---~~---

---------

rable/Chart QuestionS

;;-- -

. ,,

- .-.

169 - - -

Sociology
..

~~"ffA'"i!i-~;,:;w~.~;.~)\U;'.<-Y,,,;;.,;-.>:'"'-c'_',,-,-,,,_,.,

'
Read the following passage. Then fill in the di11gram
with the information that you read.

i'

Centuries of conventional wisdom held that human beings could


be divided into races that have a basis in the science of biology. Yet early
usage of the word "race" did not assume any biological foundation.
As its definition changed over time, however, it eventually claimed a
scientific foundation.
When the word "race" first entered the English language, it had a
meaning quite different from its more current association. Race simply
denoted a group of people who shared an identity. "The human race"
included all human beings. The term could also identify people vJho
shared a national interest, as in "the Czech race," or "the Brazilian race."
It also referred to a way of life, such as "a race of hunter-gatherers."
Explorations in the 161h and 17 centuries affected the definition of
race. These journeys brought Europeans into contact with cultures
quite different from their own. The people they encountered were also
noticeably different in physical characteristics. It was during this time
of European conquest and colonization that racial categories
became defined by physical appearance. At the same time, science
was evolving, and attempts were made to measure "racial differences."
As people were racially categorized by their physical attributes
such as skin color, head shape, and hair texture, another important
idea associated with race developed. A "racial worldview" assumed
that each race had its own behavioral and physical traits that were
passed on from parents to children. These traits could therefore be
used to distinguish each race. Three commonly used categories were
Caucasoid, Mongoloid, and Negroid. A mid-19 century treatise,
"Essay on the Inequality of Human Races," lent further support to the
idea that race was a biological explanation for human differences that
could thus be legitimately used to rank human beings.
As the 201h century dawned, the idea behind the concept of racethat humans could be readily divided into biologically determined
races-was widely accepted. However, scientists found it increasingly
difficult to actually measure those biological determinants. As genetic
studies in the late 20th century were to confirm, the reason was simple.
Races have no biological foundation. There is no "race" gene. It turns
out that so-called races are neither based on biology nor definable in a
scientific manner. Today, sociologists and other scientists recognize
that "race" is a cultural construct. It identifies a group that shares some
visible physical traits, as well as some cultural and historical experiences.
.

i@IH

conventional:

following generally
accepled beliefs
association:

a connection of ideas
denote:
to mean; to refer to
distinguish:

to tell apart from


something
treatise:

a formal, written work


addressing a subject
extensively
J;

rank:

;:.

to assign a certain position


or place, for instance, a
social class

'

construct:
an idea or perception
based on impres_sions
from the senses

i~'i
c~

------------~---------v -c->v.__._..------.--'---'-"--~---~--------------

{'
;,f

~1

ti

'ff.

1:,,,
I!'

tt:

'fi

"
E~-

:r
,,f_i;

i"

t-.')"

(:''.

t'i~
v.t~!!~
f~'

i
!'.

tf:'',~

l\.

v'
.t.:~~f
'!
\\

' '"

:,
' ,,

I t;
l

I''<
-~\

I <:,

I"

['I

'i

'

.':'

i
!

J'
)

[.

[['.

The.yharygingld~as of. Race

'j'
'

Meaning 1:

'

. Meaning 3:

Meaning 2:

~~

1. Directions: Complete the table below to summarize information about the concepts of race
discussed in the passage. Match the appropriate statements to the era with which they are
associated. TWO of the answer choices will NOT be used. This question is worth 3 points.
Period
Original Usage
16lli and 1?'h Centuries
,\
.\.

i"
,,

2Qlli Century

[[

Concept of Race

Answer Choices
(A)
( B)
(C)
( D)
( E)
( F)
(G)

Divided people into races according to physical traits


Used definitions for race acquired from foreign countries during the European colonization
Stated tbatparents pass racial characteristics to their children
Said that a race of people shared a lifestyle
Proved that races are not determined by biology
Described race as a cultural creation
Changed the definition of race in reaction to a 19'h-century essay

Fill in the blanks to complete the summary.


According to the passage, it w a s - - - - - - - wisdom that race simply
_ _ _ _ _ a group of people who shared an identity, such as being from the same
country. Race meant s-omeilling much different from its current

. Later,

people thought race was a biological fact It was believed that certain physical and behavioral
traits could be used to _ _ _ _ _ _ _ each race. Finally, scientists showed there was
no scientific basis for the idea of race, and it was actually a cultural _ _ _ __

-----------------------------.. -----------------------------------------------------------------------------------------Table/Chart Quesfrons 171 - - . - " l

Economics
Read the following passage. Then fill in the diagram with the information that you read. +@If
"

Until the 1930s, governments trying to improve a country's economy


followed the "quantity theory of money." It argues that the supply of
money has an effect upon the strength of the economy. In the 20th
century, two Western economists revisited the quantity theory to
develop new yet conflicting theories on how government actions can
influence the economy.
In 1936, John Maynard Keynes criticized the quantity theory in his
General Theory of Employment, Interest, and Money, He argued that
the money supply did not have a significant effect on prices or the
strength of the economy. Keynes believed that the key to a healthy
economy was the demand for goods and services produced. His
theory is known as a "demand-side" theory. If there is not enough
demand for the goods and services companies produce, the
companies will produce less. This means that they will employ fewer
people. For Keynes, three variables affect demand: consumer spending,
government spending, and investment spending. Therefore, to improve
the economy, a government can alter its tax and budget policies and
encourage investment For instance, it consumers pay fewer taxes,
they are likely to spend more on goods and services, thereby
increasing demand. Keynes's work formed the basis of a new branch
of economics called Keynesian economics. It became so widely
accepted that for several decades, economists rejected monetary
policy as a way of managing the volume of economic activity.
In the 1960s, Milton Friedman introduced his "supply-side" economic
theories. It brought focus back to the supply of money in an economy.
Friedman argued that the money supply has a causal effect on the
economy. For him, the fiscal disaster of the Great Depression was not
a result of low demand as suggested by Keynes. Rather, it was a
contraction in the money supply. The Federal Reserve, the centralized
banking system in the US, supplied less money than was desired.
This caused a drop in consumption and thus a decline in the overall
health of the market. A detailed analysis published by Friedman and
a colleague, A Monetary History of the United States, 1867-1960,
persuaded many that his theories were valid. According to this
monetarism school of thought, a steady rate of eco_nomic growth can
be promoted by a steady growth of the money supply. Simply put.
more available money for spending will increase. production and
employment Friedman argued against the Keynesian focus on
government fiscal policy. He and his fellow monetarists believed that
the government should only intervene in the economy by controlling
the rate of growth of the money supply.

----~---------------------~----~---------------------~

revisit:
to consider again
variable:
something that changes or
can change
monetary:
of or about money
causal:
being a cause of
something

fiscal:
financial: having to do
with government policies
of spending and taxing
contraction:
a reduction in size; a
shrinking
consumption:
the purchase or use of
goods and services
Intervene:
to enter or come
between in order to
aflect something

Keynes vs. FriJdman

I
. ~eynes

"i..

Fri~dman

1. Directions: Complete the table below to summarize information about the economic theories
discus.sed in the passage. Match the appropriate statements to the theory with which they are
associated. TWO of the answer choices will NOT be used. This question is worth 3 points.

....

Theory

Keynesian

Monetarist

)f
l'~

';,

,,
,.

Statements

Answer Choices
(A)
(B)
(C)
(D)
( E)
( F)
(G)

Governments need to control the supply of money in order to affect the economy.
Governments should not try to influence the economy.
Governments can change tax policies to change consumer spending.
The employment rate will fall without sufficient demand.
The health of an economy depends on three kinds of spending.
The economy will be negatively affected by a contracted supply of money.
Increases in the tax rate will promote a healthy economy.

Fill in the blanks to complete the summary.


Two economists developed new theories on how governments can influence the economy.
Keynesian economists believe that three _ _ _ _ _ affect economic activity: consumer
spending, government spending, and investment. Keynes's theories became so widely
accepted that economists rejected

policy to manage economic activity. On

the other hand, Friedman's monetarists believe that money supplies have a(n) _ _ _ __
effect on the economy. They attribute the
to a{n)

disaster of the Great Depression

in the money supply, causing the market to decline.

t .
\

~"

Table/Chart Questions

173 - -

'

Instructions: Choose the best word or


phrase to complete each sentence.

1. During the winter, animals such as deer


and fire ants
for food that
will feed them through the cold months.
(A)
(B)
(C)
(0)

distinguish
forage
discern
foster

2. The new health club downtown caters


_____ to women and their needs.

(A)
(B)
(C)
(0)

sedimentary
randomly
inevitably
exclusively

3. Astronomers were surprised when they


finally pinpointed the
. of
the intense microwaves from space.
(A)
(B)
(C)
(0)

source
reinforcement
spectrum
kinship

4. Glazes used on materials such as


stoneware and porcelain make pottery
very strong and _ _ _ __
(A)
(B)
(C)
(0)

profitable
durable
recreational
variable

5. The employee's open defiance of


company policy was
for
dismissal, so she was fired immediately.
(A)
(B)
(C)
(D)

.-.:...- -

'i:1)

}i

treatise
proximity
grounds
subject

174 Vocabulary Review 3

6. Scientists believe that leprosy, a disease


widely thought to have spread from India,
in fact appears to have
in
Africa or the Near East.

(A)
(B)
(C)
(D)

originated
cultivated
envisioned
replicated

7. The long-standing _ _ _ __
between the two baseball teams made
their games more exciting for their
devoted fans.
(A) dissipation
(B) deluge
(C) contention
(0) rivalry
8. The Czech Republic has the highest
annual per capita
of beer
in the world.

(A)
(B)
(C)
(0)

expression
rationale
consumption
territory

Instructions: Choose the word or phrase


closest in meaning to the underlined part
of each sentence.
9. The local department store sold all types
of clothing: work, casual, and dress.
(A)
(B)
(C)
(0)

apparel
tension
bondage
hardship

10. It is possible for some lizards to grow new


tails after losing their tails.
(A)
(B)
(C)
(0)

sustain
generate
promote
ensure

~
, _},

~- -'.'

11. Because the sisters fought constantly


over which television show to watch, their
mother finally had to mediate and choose
one for them.
;

'

'
'

(A)
(B)
(C)
(D)

adopt
intervene
thrive
indulge

12. Because of its large surface area,


temperatures in parts of Canada can
~ from forty degrees below zero to
thirty-five degrees above zero.
(A) implant
(B) shape
(C) execute
(D) range

. ;.:

. , 13.,,Mahayana Buddhism is the main religion


.
.iri'China, Japan, Korea, and much of
;
Vietnam.
(A) commercial
(B) subsequent
(C) predominant
(0) inherent
fl

,.

14. Phil unintentionally told his aunt about her


surprise birthday party because no one
had told him that it was a secret.
(A) formally
(B) undeniably
(C) considerably
(D) inadvertently

15. Many scientists believe that the

[
[
[

Instructions: Write the missing words. Use


the words below to fill in the blanks.
stereotypes

denotes

ranking

underpinning

construct. '

The term caste 16.


one's
rank or position in society according to birth,
occupation, or some other criterion. Many
nations face issues of social inequality.
Perhaps nowhere else in the world is it such
an elaborate 17.
as the
Indian institution of caste. Caste has long
been an 18.
of Indian
culture, but in recent years it has been
severely criticized by both Indian and
foreign observers. Although some claim that
caste has been abolished, such statements
do not reflect reality. Caste has undergone
significant changes, but it still involves
19.
and discrimination
against hundreds of millions of people.
India's constitution forbids negative public
discrimination on the basis of caste. However,
caste 20.
has occurred for
centuries and will likely continue.

I'

Instructions: Match the words that are similar


in meaning.

21.
22.
23.
24.
25.

extract
imposing
robust
primitive
necessity

(A)
(B)
(C)
(D)
(E)

strong
take out
overwhelming
requirement
simple

descendants of wolves, jackals, or


coyotes were interbred and evolved
into ill!Ilfili companions for humans.
(A) domesticated
(B) conventional
(C) tranquil
(D) fundamental
f_

J.:Jl ;~!l 'l l l l l l l

17 5

-~:

""11111111..lllllllllllllllllllllllllllllllllllllli!!!lllllllllllllllllllllllllllllllllllllllll!illlllllllllllllllllllllllllllllllllllllllllllllllllllllllllllll!!VollllcalllblllulalllrylllRlllevllliewllll!!l111111111111-111111111111

I
i

Hearing is a complex process involving bones, cells, and blood vessels. When these do
not work correctly, some people may experience distracting humming, buzzing, ringing,
or whistling sounds inside their heads. This perception of sounds, known as tinnitus, has
frustrating complications.
Not a disease itself, tinnitus is a symptom caused by various conditions. These may
include injury due to loud noises or foreign objects, or ear infections. Millions of people
experience tinnitus on some level, and by treating the underlying problem, it can disappear
in a short period of time. Those suffering from it experience a ringing noise that is usually not
an actual sound. The noise may be in one or both ears and varies in pitch and volume. In
some cases, the sound is so loud it disrupts concentration or hampers proper hearing. People
can suffer from two types of tinnitus: objective or subjective.
Objective tinnitus is unique because, unlike most cases of tinnitus, patients suffering from
it may have an audible sound emanating from their, ears that a clinician can actually detect.
This arises from muscle spasms around the middle ear, which can emit a clicking or cracking
reverberation. Some experience a rhythmic throbbing, repeating in time to a pulse. pulsatile
tinnitus, usually objective in nature, is caused by altered blood flow or increased blood
turbulence in proximity to the ear, resulting in the audible sound of blood moving through the
veins. Objective tinnitus is very uncommon; most people suffer from subjective tinnitus.
Although there are many causes of subjective tinnitus, the most typical are conditions that
can cause hearing loss, such as exposure to excessively loud noises or natural aging. These
may have lasting effects or may only cause brief hearing problems. Other causes of subjective
tinnitus can include lead poisoning, chemotherapy, and Lyme disease. The mechanisms of
subjective tinnitus vary. Microscopic hairs located on the end of auditory cells in the ears can
be bent or broken, thus misreading sound or sending signals to the brain of sound that is
nonexistent. Direct trauma to the ear can also create the ringing sound, but other causes are
relatively unknown.
Researchers recently introduced two categories of subjective tinnitus. Symptoms caused
by disorders of the inner ear or acoustic nerve are
Somatic symptoms are caused by
nerves within the head and brain but outside of the ear. Although there are no specific cures
for tinnitus, therapy and adequate rest often help the symptoms recede over time.
; :: ;; of, relating to, or located near the ear
--------J

'

,,

I
Directions: An introductory sentence for a brief summary of the passage is provided below.
Complete the summary by selecting the THREE answer choices that express the most
important ideas in the passage. Some sentences do not belong in the summary because
they express ideas that are not presented in the passage or are minor ideas in the passage.
This question is worth 2 points.
The condition known as tinnitus causes ringing in the ears and is a symptom of other
medical conditions .

Answer Choices
r

t
t:i;

(~) Blood flow interference can cause pulsatile tinnitus.


(B) Buzzing within the ear indicates that tinnitus is present.

(C) An audible sound coming from the ear characterizes objective tinnitus.
(D) Antibiotics are helpful in treating medical conditions, whether natural or injury-related.
( E) Subjective tinnitus has many causes but few explanations for the resulting symptoms.
(F) Symptoms within the ear are worse than those from outside sources.

ri
'

'
"
. -\,-.:_;_
$,-

If,, ' ',,,

MiniTest3

177 - -

'

02 Business
Read the passage and answer the questions. +@id

;;~

l';
)

_;:

~r

..l

'."t

1... -.

)'

'-.:

,;

To sell manufactured products, companies must establish a need or an incentive that


persuades consumers to buy. Many businesses have provided an appealing answer to the
challenge. By offering. coupons that provide a discount on specific goods, consumers feel
better about their spending and companies still receive a profit
Couponing is not a new concept C.W. Post Co. first introduced coupons in 1895 when it
offered penny-off coupons to increase sales of its breakfast cereal. Such cents-off coupons
remain the oldest, most widely used, and most effective sales promotion tools available to
suppliers. Manufacturers have used all varieties of coupons, from direct mail to instant
coupons to newspaper inserts, the most common couponing choice. Stores have increased
couponing in the past several decades, and subsequently, shoppers are responding. In the
US, over eighty percent of consumers use coupons, and nearly twenty-five percent claim to
use them every time they shop.
Many advantages exist in the production and us~ of coupons. Each coupon serves a dual
purpose. While offering the customer a price break, it also increases profits and brand
recoanition for the manufacturer. Popular with new as well as established companies,
coupons make it possible to offer a price reduction only to price-sensitive consumers, without
lowering the overall price of the product Additionally, consumers are more likely to try a new
product with a discount coupon because the perceived spending is less. When experiencing
a decline in sales, established companies can issue coupons to encourage past buyers to
purchase again and entice new users to try the product
Disadvantages of coupons do occur for the companies involved. To print and distribute
coupons, businesses undertake certain costs. Usually, sellers print an expiration date on their
coupons. However, they intend for consumers to purchase the product as soon as the
coupons are issued rather than at the end of the specified time frame. Until consumers
choose to buy, the company's incurred costs are not offset by their purchases. Thus, the
consumers dictate when the companies receive their profits. Additionally, companies whose
intention was to increase sales may not notice an increase for several months. Much
responsibility lies with the consumer. Consumer response to a coupon usually takes between
two and six months, and coupons are most frequently redeemed just prior to the expiration
date that is stated on the coupon.
~
~

ti
'

.,,'
I

J.

"

-,zJ,
R

J
I

"
~

direct mail advertising sent directly to prospective customers via the mail
brand recognition a customer's awareness of a particular brand

'-'.-

178 Mini Test 3

.i</:Z . . . . .

t:::~,:~:~::::::,:;: b;:~~:::ofmipo~ogo lo pro,ldod bolow


Jr \
);

t
.

~ .

~.
i'
:~~ .. ~--::- ;_-

P,

''

Complete the summary by selecting the THREE answer choices that express the most
important ideas in the passage. Some sentences do not belong in the summary because
they express ideas that are not presented in the passage or are minor ideas in the passage.
This question is worth 2 points.

~
I;>
,,

~z

Cl

Couponing is ?n effective sales tool used by companies to increase profits by offering


the consumer incentives to purchase their product.

.,

3z

Ii

l'

..,"'

Cl

y.

.~::.-,_ . ~'

Answer Choices
Manufacturing coupons costs the company and reduces profits.
Couponing has increased significantly over the past few decades.
Expiration dates and coupon use affect company profits.
C.W. Post Co. initiated the use of coupons as a successful promotional tool.
Companies offer coupons in order to advertise and raise buyer interest.
Customers who use coupons are more conscious of price.

j;1
.

-~.'fil1,.11... ,.......-

. Mini Test 3

...- .........- ..--!1!1!111111111!-..111!--.l!l.ll!!L!!ll.i. . . .-

179 - -

. . . . . . . . . . . . . . .Lc

03 Biology
Read the passage and answer the questions. @SH

Infections are caused by microorganisms that invade the cells of the body. They cause
damage to the tissue and interfere with normal cell activity. Two types of infectious particles,
virions and prions, cause multiple health problems. These can range from minor to fatal. While
both infectious agents cause disease, their inherent structure and behavior are different, so
the effects on the human body are diverse.
A virion is the infectious form of a complete virus particle. It consists of genetic material
and a protective protein shell called a capsid. Virions are not capable of reproducing alone,
so they infect a host cell in order to replicate themselves. On the other hand, a proteinaceous
infectious particle, commonly referred to as a prion, is a microorganism composed solely of
protein that causes serious infection. These diseases affect the structure of the brain and
other neural tissue. They can disrupt normal nerve communication within the brain.
Regarding its structure, a virion is essentially a gene transporter existing in its simplest
form ?S nucleic acid. It is surrounded by a protective capsid that serves as the shape of the
organism. Conversely, prions are made up of proteins found in the human body that have an
altered structure. They are resistant to the enzymes that normally break down protein.
Scientists believe that prions infect the body by refolding abnormally and converting normal
protein molecules into an irregular structure.
Human diseases caused by viruses include influenza and chicken pox, as well as more
serious infections such as AIDS and Ebola. Because virions inhabit a host cell, researchers
find it difficult to eradicate the infection without killing healthy human cells. Vaccinations prove
to be the most effective preventative for infection. Medications exist that treat symptoms of a
viral infection, although the virus itself cannot be destroyed. Whereas researchers have found
ways to prevent virions from infecting the body, prions are resistant to all medication and
procedures. They cause neurodegenerative diseases, which are evident by such symptoms
as convulsions (violent, uncontrollable body movements), dementia (an illness that gradually
destroys brain function), and personality changes. These ominous diseases that degenerate
the body include mad-cow disease and other forms of Creutzfeldt-Jakob disease. The
incubation period in which a prion-induced disease develops is slow. However, it progresses
rapidly once symptoms appear, causing brain damage that leads to death. Thus, there is no
effective treatment for diseases caused by prions.
~ nucleic acid

an acid such as ONA or RNA that is found in the cells of all living things
> neurodegenerative disease condition in which cells in the brain or spinal cord are lost

;.;_.~..,____

180

Mini Test 3

' -

.. >-~; '

Directions: Gomplete the table below to summarize information about the infectious agents
discussed in the passage. Match the appropriate agents to the characteristics with which
they are associated. TWO of the answer choices will NOT be used. This question is worth
3 poin(s.
Infectious Agents

Characteristics

Virions

Prions

., Answer Choices
i: :rt (A) Can be prevented before infection
t
( B) Have effects that last a limited amount of time

:; ' ( C)
1
( D)

(E )
( F)
(G)

Cause irreversible damage to tissues


Are composed of matter found in the body
Can be eliminated effectively with medication
Form an abnormal and irregular structure
Include a piece of genetic material that is replicated

I
i

"i'

Ji.IT..--------..... -1!11111

ll!ll.-
....

..Mil.1il..'3.181.......I

04 History
Read the passage and answer the questions. @1'

The prehistoric dinosaurs that roamed the Earth millennia ago vanished from history
without leaving clear indications of what transpired. It was widely believed that dinosaurs and
other prehistoric species met their end as a result of the meteor that formed today's Chicxulub
crater. The meteor shook the Earth millions of years ago when it landed on the Yucatan
Peninsula. It may have been a piece of a much larger asteroid that broke apart in distant
space. The theory explains that when the meteor crashed into Mexico, leaving a 112-mile-wide
depression and worldwide destruction in its wake, it caused the extinction of the dinosaurs.
However, growing evidence supports a new theory. It states that the extinction that took
place at the end of the Cretaceous Period approximately sixty-five million years ago was
caused by a series of events. These include the impacts of multiple meteors, heavy volcanic
activity in India, and climate changes. Paleontologist Gerta Keller, supported by the National
Science Foundation, has been building evidence to prove the new theory according to
scientific findings.
Keller gathered information proving that the Chicxulub impact could not have caused the
mass extinction. It appears to have predated the extinction by 300,000 years. Paleontologists
extracted samples of marine microfossils from .sediment layers above and below the
Chicxulub impact layer. No significant biotic changes were found in the samples.
Given the magnitude and number of species that became extinct, Keller's theory more
likely explains what truly happened. The Chicxulub crater undoubtedly had an effect on the
Earth and its inhabitants. However, it would not have been enough to cause a mass extinction,
wiping out thousands of species in a relatively short time. Fossils show that many species
lived beyond the meteor's impact. Keller describes other factors that played a part. For example,
constant volcanic activity released greenhouse gases, which caused a warming effect, produced
acid rain, and raised the temperature of the ocean. At this time, marine species evolved to
survive in the changed atmosphere. Neither Chicxulub nor volcanoes caused extinction, but
they placed great stress on most species. It was the impact of a larger meteor that was the
final factor.
All these events weakened Earth's inhabitants and culminated in an impact of colossal
proportions. By the time the larger meteor hit, the creatures could not withstand the changes.
Although it is not confirmed, some believe the large meteor landed in India, leaving a crater
an estimated 300 miles wide. The meteor and resulting crater had to be of great significance
given the resulting destruction.

'!

1.., : - -

!
I> biotic pertaining to life or living

v colossal
182

Mini Test 3

of great size or extent

_______________J

:'

; J'.

I
Directions: Complete the table below to summarize information about the two theories behind
the extinction of the dinosaurs discussed in the passage. Match the appropriate statements to
the theory with which they are associated. TWO of the answer choices will NOT be used. This
question is worth 4 points.
Theory

J.

'

j; t

;_

Statements

Chicxulub Theory

Keller's Theory

:IE

"'3z

Answer Choices
(A) Multiple volcanic eruptions caused dinosaurs to die.
( B) A piece of a large asteroid may have broken away in space and hit Earth.
(C) An object from outer space left a crater on the Yucatan Peninsula.
(D) Global warming caused by volcanic activity contributed to extinction.
( E) Dinosaurs became extinct because of a meteor's impact on Earth.
( F) Volcanic eruptions released toxic gases and caused acid rain.
(G) Tl:le end of the Cretaceous Period showed an increase in dinosaur species.
(H) Dinosaur species weakened after years of stressful living conditions.
( I ) The Earth was bombarded by multiple meteors that diminished dinosaur numbers.

f
f,

Mini Test 3

183 - -

',-,.v,,,. ,_

';\.

~-.~' ,:~

\
I

l
'

l'

'kI

. '~ . ~ .,

,,

i' .

The TOEFL iBT Listening Section--------------------- 188


Preview Test ---------------------------------- 190
Chapter 1 Main Idea Questions -------

. 197

Chapter 2 Detail Questions ---------------------- 209

Vocabulary Review 1 -------------------------------- 220


Mini Test 1 ------

------------------------ 222

Chapter 3 Function Questions ----------.------------- 225


Chapter 4 Stance Questions - - - - - - - - - - - - - - - - - 237

Vocabulary Review 2 ---------------------------- 248


Mini Test 2 --------------------------------------------- 250

Chapter 5 Organization Questions ----------------------- 253


Chapter 6 Content Questions -------------------------------- ---- 265

Chapter 7 Inference Questions --------------------------------------------- 277

Vocabulary Review 3 ---------------- ------------------- ------ 288


IVlinl Test :J

----

Answer Key Listening

290

In the listening section of the TOEFL iBT, you will hear a variety of conversations and lectures, each
of which lasts from three to six minutes. A total of six listening passages will be presented. After
each passage, you will then be asked to answer five to five to six questions about what you heard.
Like the reading section of the TOEFL test, the questions are designed to assess your understanding
of the main idea, factual information, and inference. You will not be asked questions regarding
vocabulary or sentence structure.

Passage Types
1. Conversation - Two people discussing a campus-related problem, issue, or process
2. Lectures - A professor presenting information related to an academic topic
3. Classroom interaction - Similar to a lecture, but with some interaction between the professor
and one or more students
Question Types
Questions for the listening section of the TOEFL typically appear in the following order:

r~'.~-- 188

Question

Type

Main Idea

2-3

Factual Information I
Detail I Content

Choose the statement that is true according to what was said.


Select multiple answers to complete a chart.

Purpose/ Inference I
Organization

Recognize the speaker's purpose, draw an inference,


or explain how the speaker communicated certain information.

5-6

Repeated Listening

Description
Choose the best phrase or sentence.

Hear a particular portion of the listening passage again and


recognize the speaker's purpose, attitude, or the implied
meaning of a statement.

The TOEFV iBT Listening Section

~~.,.,,,,,.,_,;;::'~':.'cc~.__,,-'-"' ''--""'~
'c.:._:

~''

.::c;:_":::.:c:'

'"==~~-.........._.......,...._......_.,..._.....

;;;;:.;:;,,;;;,:;:.

Study Tips for Listening

,, ., .,, .

Jr

Practice listening to North American English as much as possible. For the purposes of the
TOEFL, educational programs, documentaries, and news programs are excellent sources.
When you are practicing for the listening section of the TOEFL, listen to the material only once
and then answer the questions. Then review the answers while listening a second or third time.
Remember, though, during the real test you are only permitted to hear the conversation or lecture
once before answering the questions.
Pay attention to how pauses and intonation are used to organize the passage, emphasize
important information, and show transitions.
1 Make a recording of the programs you use to practice listening. Replay any sections you have
difficulty understanding.
Keep such things in mind as the main idea, the development and support of the main idea, and
the speaker's reasons for mentioning certain points.
Develop your note-taking skills. While you are listening, try to write down key words in an organized,
graphic way that makes sense to you.

i' . .
:it: i ;;:;::. ['' of'"""""'"'_,, '"""'to ""''"'"' campoo
' .j' .

<l' 't
''l.

Ille "woll

Ill
"O

~z

G'I

""''oo' "'domic

Test Management

;~

~
I

'

I~

l'-

A picture will be shown on the screen to allow test takers to recognize each speaker's role and
the context of the conversation. Along with this picture, a subject title will be given for each lecture.
Before you begin the listening section, listen to the headset directions. Pay particular attention to how
you change the volume. It is very important that you are able to hear clearly during the listening
section of the test.
If you miss something that is said in a conversation or lecture, do not panic. Simply keep listening.
Even native speakers do not hear everything that is said.
Note-taking during the lecture is permitted. Paper will be provided by the test supervisor. These
notes can be studied while answering the questions, and will not be seen by test graders.
Like the reading section, questions cannot be viewed until after the lecture or conversation has
been completed.
In the listening section, you must answer each question before moving on; you cannot return to
a question later. If you are unsure of an answer, guess.

1'

r\
I

The TOEFL' iBT Listening Section

189

--,J

@'

You will listen to two lectures and one conversation. You will hear each lecture and conversation
one time.
After each listening passage, you will answer some questions about it. Most questions are worth
one point, but some questions are worth more than one point. The directions indicate how many
points you may receive.
You will have 15 to 20 minutes to both listen and answer the questions. The questions ask about
the main idea and supporting details. Some questions ask about a speaker's purpose or attitude.
You may take notes while you listen. You may use your notes to help you answer the questions.
Your notes will not be scored.
In some questions, you will see this icon:
conversation again.
When you are ready, press Continue .

.:...~.-- 190
:i_\.

Preview Test

n. This means you will hear part of the lecture or

Listen to a lecture in an anatomy class.


Ill
"'O

~z

Cl

:.
'.;i''

;
t

!.'

!''
t-

r
1,.
. I

I
PreviewTest 191 - '

''

1. What is the main topic of the lecture?


(A) The role of chemoreceptors in
distinguishing differences between
tastes
(B) How chemoreceptors allow humans
to smell and taste
(C) How chemoreceptors have evolved
over time in animals
(D) The reason why the brain is able to
receive electrical signals

2. Why does the professor say this:

(A) To prove that the epithelium is an


extremely complex part of the nose
(B) To describe the structure of the nose
in detail
(C) To give an example of how
chemoreceptors in the nose affect
the ability to smell
(D) To explain that the epithelium is the
part of the nose that detects smells

3. Listen again to part of the lecture. Then


answer the question.

Why .does the professor say this: (/


(A) She wants the students to draw a
logical conclusion.
(B) She finds it odd that humans smell as
well as they do.
(C) She wants students to guess how the
study was performed.
(D) She knows that the students will find
the concept confusing.

4. According to the professor, how is the


sense of taste important to humans?
(A) It indicates whether or not food is
good to eat.
(B) It makes food more enjoyable.
{C) It helps people find delicious food.
(D) It identifies poisonous food.

5. What is the professor's attitude toward


buying food at the supermarket?
{A) She thinks it has improved people's
sense of taste.
(B) She feels it has made the sense of taste
less important.
(C) She believes it has decreased the
quality of food.
{D) She assumes that it provides
better-tasting food.

6. How does the professor organize the


information presented in the lecture?
(A) She classifies two key types of
chemoreceptors.
(B) She compares the various human
sensory systems.
(C) She explains the role of chemoreceptors
in animal survival.
(D) She describes problems associated
with chemoreceptors.

: Listen to a lecture in an art history class. @ti

.~-

. 1f
.

, !i. ,.

,I
~
[

Preview Test

193

--1

"

[.;c

1 .

t;;

--

ilil\!ilhiiL

7. What are the speakers mostly discussing?


(A) How El Greco's work contributed to
later art movements
(8) The ways that El Greco's art was
influenced by his astigmatism
(C) The theory that astigmatism causes
people to see objects as elongated
(0) The reasons why El Greco's art was
different from his contemporaries

F;
: ,;
L

,l,:jj
,,_,

""
r
"i

~JJ--- 194

"-1

th:
[Ii'I

8. Why does the professor explain the


effects of astigmatism on perception?
(A) To refute the argument that astigmatism
was responsible for El Greco's style
(8) To demonstrate that there are many
factors that contributed to El Greco's
style
(C) To give an example of a condition
that can affect an artist's vision
(0) To show how El Greco's style was
influenced by his astigmatism

9. According to the lecture, what element of


El Greco's work demonstrates his ideology?
(A) The dreamlike quality
(8) The elongated figures
(C) The imaginative scenes
(D) The perfectly proportioned subjects

Erm&!

~~~M.l:i'Hil!iil1~,;~~

10. Listen again to part of the lecture. Then


answer the question.
Why does the professor say this:
(A) To concede that the professor was
correct about El Greco's ideology
(8) To support the professor's claim
about the Cubists
(C) To maintain that the astigmatism
could have caused El Greco's style
(0) To deny that astigmatism would make
horizontal figures thicker

11. What is the professor's opinion of El


Greco as an artist?
(A) She thinks that his innovative style
was a result of a physical condition.
(8) She considers him a creative artist who
influenced artists after him.
. (C) She believes that there are other
artists that were more influential on
the Cubists.
(0) She feels that modern science is
unable to explain art

12. Based on information from the lecture,


which characteristics describe El Greco's
style? Place a checkmark in the correct
box.
YES
Elongated subjects
Highly realistic
Representative of
Renaissance art
Inspired by his
imagination

Preview Test

NO

.'

:'' -

"l

.J

'~f

./'

llA~'"'.),,.

Listen to a conversation between a student and a university employee.


VI

"II

>

;::!

Cl

~
"II

n
m
m

...
...
VI

II

I
I

!I

I'
l
!I

Preview Test

195 _ _i

13. What are the speakers mainly discussing?


(A) The high cost of identification cards
(B) How to replace a lost identification
card
(C) Where to obtain a temporary
identification card
(D) Why students need to have an
identification card

14. Why does the student visit the office?

(A) Because she has found another


student's ID card
(B) Because she is unsure about how to
use her ID card
(C) Because she cannot use her ID card
tor campus services
(D) Because she has misplaced her ID
card
!'

15. According to the conversation, why can't


the student have a temporary ID card?
(A) It is too expensive.
(B) It is a security risk.
(C) Sh.e has lost hers too many times.
(D) She cannot access university
buildings.

!).
~~).--_ 196
\1'

11. -

Previl"lw Test

-.--;-,-;-_-->;_;_,-_

16. Li~\en again to part of the lecture. Then


answer the question.
Why does the student say this:

(A) She does not understand why she


cannot have a temporary card.
(B) She is frustrated that she has to pay
tor a replacement card.
(C) She is worried that she will lose her
new card.
(D) She is glad that the man will allow
her to replace her card.

17. What can be interred about the student?


(A) She does not like it when her parents
give her money.
(B) She has never had to replace her ID
card before.
(C) She is usually not busy in the
afternoons.
(D). She often misplaces her things.

Necessary Skills
9

.,......~www,;rn,~"!1~,ffl..~"lJ',t~.>i;il'.>J.l'!'i:.f'-<.av.-:i~'><'rt:'c'.f-''""T

Understanding the overall topic or basic idea of a lecture or conversation


Understanding the speaker's general purpose in giving a lecture or having a
conversation
Inferring the speaker's purpose or main idea when it is not directly stated

Example Questions
R\U:ll!lR':l'X>.~-r.nm1m:t'.'.-.l3:>.u:.;.%>\'.;"VUEJ;\~f');(IJ->:!i'.l"~-'."J'S>'il'-/.','.".1o''"-

' <,' '

What is the main topic of the lecture?


What is the main topic of the conversation?
What are the speakers mainly discussing?
What aspect of
does the professor mainly discuss?
What aspect of the problem does the
help with?
What features of each type of _ _ _ _ _ does the professor focus on?
What is the woman's main concern about - - - - -?
What concerns does the student have about _ _ _ _ _?
Why did the professor mention
?
What is the student's motivation for
?

Strategies
Pay attention to expressions that indicate the topic:
- Today's talk is on ...
- Today we're going to talk about .. .
- Now we are going to discuss .. .
In a conversation, listen for cues that will indicate a speaker's main purpose.
- How can I help?
- What do you need?
- Can you help me with ...
Listen for key words that are emphasized or repeated.
Keep in mind that two or more rnajor ideas together may define the overall topic.

Main Idea Questions

197 .___,_,,

Linguistics

_..,._._;;,_...,.,,.,.~-:t~~..tJl~~H~J.!i-Z'~'~'=,";>:il:'~';~r.c.,,,;,~-~'..;.--'.,----

'.
l,
i_:

Listen to a lecture in a linguistics class. Fill in the diagram with the information that you hear.

Key Vocabulary
acquisition: the act of getting something
formulate: to come up with an idea or plan
emphasize: to pay special attention to a certain aspect

manipulate: to influence or contra.I SOfr!elhing


habitual: occurring repeatedly .

.The TrarisactionaLMod~Fof L.aHgtlage


"'

: :. '

---- '<-

> -:.. ".' ' :--. ,,,.--...._-"-''

AcquisltibA'~/.ij
-.",,_ -;"',;,- .. _, '..;to;::-.---.:>

Point 1:

Example:

Point 2:

Example:

1. What is the main topic of the lecture?


(A) Differences between language
acquisition models
(B) Academic fields that have been
affected by the transactional model
(C) How the transactional model
influences our understanding of the
environment
(D) A theory that linguists use to explain
language acquisition

2. What aspect of the transactional model

does the professor mainly discuss?


(A) How it emphasizes the importance of
language acquisition
(B) How environment affects language
learning
(C) How it differs from previous models
(D) How its application has been
controversial

Fill in the blanks to complete the summary.

---------------------------------------------------- .. -------------------------...... ----..,..,_------------------ .. -------The transactional linguistic model helps linguists understand the process of
_ _ _ _ _ _ _ _ _ . The theory says that
is influenced by
_ _ _ _ _ _ _ _ _ It also says that ______ can have a similar effect on

------ within the environment.

! :.

"''(f

History
-~~--~,~~~~'"-uru~.whl>lfl:I."''~""~;,~-"'--,::,

''"-:,

,,,,.,.;:;Listen to a lecture in a history class. Fill .in the diagram with the information that you h~ar.

.i'-i-'ti'~/:

~ff~~-~~-:-;: - tt1Gf'
'.-. '..~.w :, .~.: ;J. p~'_'_--""""
....

...

4.(',_ !.'/,'" fi';


Key Vocabulary
"lj/'.;._>- :-it.z.,4. :; .:,::;.~:. ~.~:tJ.-;_~b. n;~uer: to take c.9ni~ol. o.t land o.r ~eople.b.y_fore~ .]'-' . -.;' . fli'!!? legendary: very famous fora long period of time

;. ~).h_~-..}k ~.~j'..~ss
.. lm-11.atlon: the pr~cess of b8coming similar to sorrjet_h)ng,
-.:; --.... -_
:.x~.:}_~:.- ~~-;--> >.

.-;'.'<~-\-:'." t~4F:

especially a culture

esteem: to have a good a.Pinion of someone .. . .


.pride oneself on: tci be satisfied with 9rproud qi oneself tor .

__
...

:-.->;;

:;-

Contribution 1:

Contribution 2:

Contribution 3:

Contribution 4:

",,

''-,,-'

,'.{_

>\"
1;.,,;.;_;--

" - ;1, : ~: _:: .

',-, --

"~~

..

1. What is the main topic of the lecture?


(A) The Persian influence on Greek culture
(B) Why Al.exander the Great was admired
by the Romans
(C) How a famous military leader helped
spread Greek culture
(D) How Greek culture came to be globally
appreciated

2. What aspect of Alexander's reign does the


professor mainly discuss?
(A) His treatment of the people that he
conquered
(B) His effective organization of the army
(C) How his conquests made him famous
in the ancient world
(D) How his policies brought Greek culture
to other countries

Fill in the blanks to complete the summary.


..........................................---------------------------------------------------------------------------------------------------Alexander the Great spread _ _ _ _ _ to many parts of the world. To promote cultural
assimilation, he had ______ marry------ The professor also mentions
Alexander's _ _ _ _ _ , giving the example of Romans trying to associate themselves
with Greece

Greek.

----------------------------------------------------------------------------------------------------------------------Main Idea Questions

199 - -

Office Hours
I

1~

Listen to a conversation between a student and a professor. Fill in the diagram with the
information that you hear. li'!fflffj

\i
i

Key Vocabulary
sentiment: thought or opinion about a situation
setup: the way something is arranged
go on: to continue

Problem:

1. What is the main topic of the conversation?

(A) What the woman must do in order to


pass the course
(B) Why the woman is not interested in the
lectures
(C) How the professor can improve his
teaching style
(D) The student's feelings about a recent
lecture

auth~;lty: expert on a subject . .


.
' Input: comments or suggestions made on a subject

Solution:

2., What problem does the woman have with


the professor's class?
(A) He does not conduct class discussions
well.
(B) He does not encourage group
discussions.
(C) He ignores information in the textbook.
(D) He refuses to ask for input from the
students.

Fill in the blanks to complete the summary.


The student goes to see the professor because she thinks she is not _ _ _ _ __
effectively in the class. In particular,
thinks that the _ _ _ _ _ waste
time. She feels that class time could be better spent by _ _ _ _ _ . The professor agrees
to _ _ _ _ _ changing the way he teaches the class.

History

~~~1'll'lrutr..i~m'?.lFl~'l.!il!t<t'li:'l-~W"0"6~r,;J>'

_-;,:

~o-';

__ ,;.>

Ji!'"' '"" Listen to a lecture in a history class. Fill .in the diagram with the information that you h~ar.
{:<;;:~ --1- ~:

f.'Yc';'Key Vocabulary

'lf,1i.>._ .
~tr con~uer: to take control of land. or people by force

>

.
,. . .
W,~'.\'~sslmllatlon: the process of becoming similar to SOO)Bth.ing,
,f.\'ii/C 011' '
especially a culture

~!;:-legendary: very famous for .a long period oftirne

esteem: to h.ave a good opinion of som~one . / ,. '.


>pride oneself ~11: to be.saU~fied with. o~ proud oi ones~lf for.

"'

~z

GI

"~\(~ff:\ \lli:<.:.

:E

"'3
z

GI

Contribution 1:

Contribution 2:

Contribution 3:

1. What is the main topic of the lecture?


(A) The Persian influence on Greek culture
(B) Why Al.exander the Great was admired
by the Romans
(C) How a famous military leader helped
spread Greek culture
(D)' How Greek culture came to be globally
' appreciated

Contribution 4:

2. What aspect of Alexander's reign does the


professor mainly discuss?

(A) His treatment of the people that he


conquered
(B) His effective organization of the army
(C) How his conquests made him famous
in the ancient world
(D) How his policies brought Greek culture
to other countries

Fill in the blanks to complete the summary.

--..-------------------------------------------------------------------------------------------------------------------I

f,

i.!

Alexander the Great spread _ _ _ _ _ to many parts of the world. To promote cultural
assimilation, he had ______ marry _ _ _ _ _ . The professor also mentions
Alexander's ______ , giving the example of Romans trying to associate themselves
with Greece ______ Greek.
{
---------r---,--------------------,-----------------------------------------------------------------------------------

Muin lclea Questions

199 - -

Office Hours
Listen to a conversation between a student and a professor. Fill in the diagram with the
information that you hear. +Mf!fbj
Key Vocabulary
sentiment: thought or opinion about a situation
setup: the way something is arranged
go on: to continue

Problem:

1. What is the main topic of the conversation?


(A) What the woman must do in order to
pass the course
(B) Why the woman is not interested in the
lectures
(C) How the professor can improve his
teaching style
(D) The student's feelings about a recent
lecture

i(

. autho;lty: expert on a subjeci


.
..
.
.. Input: comments or suggestions rnade on a subject .. .

Solution:

2 .. What problem does the woman have with


. the professor's class?
(A) He does not conduct class discussions
well.
(B) He does not encourage group
discussions.
(C) He ignores information in the textbook.
(D) He refuses to ask for input from the
students.

Fill in the blanks to complete the summary.


The student goes to see the professor because she thinks she is not _ _ _ __
effectively in the class. In particular,
thinks that the _ _ _ _ _ waste
time. She feels that class time could be better spent by _ _ _ _ _ . The professor agrees
to _ _ _ _ _ changing the way he teaches the class.

~-

Service Encounter
Listen to a conversation: between a .student and a university employee. Fill in the diagram
mr''''< with the information that you hear. @ti
l}/,''9~, ...

"'v'""'' ~\f:fey Vocabulary

.
.
to find and eliminate a problem
l~!ffhardware: the physical components of computer or other
~if;::. . .
instrument
,,
.
.

l!ill~troubleshoot:

I'>{'.<'.

In working order: able to operate normally


Incoming: coming In or arriving
directives: orders or instructions

::;;
Problem:

"'3z

Solution:

GI

1. What are,\be speakers mainly discussing?


' ,

tfv.:

(A) Why the student cannot remember his


. network password
(B) The $fudent's computer hardware
problems
(C) Why the campus network requires
a password
(D) How the student can connect to the
campus network
{;.

f..

2. How does the woman help the student?


(A) By installing a program on the
student's computer
(B) By applying for a campus email
address
(C) By helping the student get a network
password
(D) By verifying the student's IP address

.-\'

Fill in the blanks to complete the summary.


The student called _ _ _ _ _ _ _ because he needs help connecting to
- - - - - - - The woman discovers that the student has _ _ _ _ _ _ _ a
network password. The woman walks the student through the steps of _ _ _ _ _ __
and logging on to _ _ _ _ _ __

-----------------------------------------------------------------------------------------------------------------------

I
Main Idea Questions

201 ----;

literature
'
Listen to a discussion in a literature class. Fill in the diagram
with the information that
you hear.

+iffifii

Key Vocabulary
blaze the trail: to do something that was never done before .
contemporary: someone who lived during the same time as
someone else

departure: something different from what i.s common or


expected

fragment: an incomplete part


refreshing: different and interesting in a pleasant way

.Emily Dickinson vs:. Her Female Contemdoraries .

<

,-

--

-'

Emily Dickinson:

Both:

Female Contemporaries:

1. What are the speakers mainly discussing?


(A) Why Dickinson was not popular
during the Victorian period
(8) The special qualities of Dickinson's
writing style
(C) How Dickinson's style changed
during her career
(D) Dickinson's role in the creation of a
new style of poetry

2. Why does the professor mention


Dickinson's grammar?
(A) To give a reason why it is difficult to
read her poems
(8) To demonstrate her unique writing
style
(C) To explain why she did not use a .
conventional writing style
(D) To compare her poetry with her fiction

Fill in the blanks to complete the summary.

The lecture compares


of Emily Dickinson to that of other poets of her
era. The student says that Dickinson's style is marked by
. The professor
adds that she used
patterns as well. The professor also says that
Dickinson focused on very _ _ _ _ _ _ _ , while other writers focused primarily

---- --------------------------------------------------- ------- ''

;.i..-i

202

~-------------

------------------------------------ .-----

Chapter 1

_.

----~-.

""
'!-

Medical Science
Listen to a lecture in a medical science class. Fill in the diagram with the information that

you hear. +iffli'='


If
.
~c,, Key Vocabulary
~t)

-.

Ui Infectious: capable of spreading to others


. 1;"-.{.
~''.;'transmit:
to carry from
one place to another
'
.
~~;~yglene: the practice or principles of cleanliness

contract: to catch an Illness or disease


mortality rate: a statistic that describes the number of .
. deaths during a period of flme

"" .-- -~ - ,

\;{,;

Cause 1:

Effect:

Cause 2:

1. What is the lecture mainly about?


(A) How Europe protected itself from
outbreaks of disease
(8) The reasons why mortality rates are
higher in underdeveloped countries
(C) Why so many people in the past died
from treatable diseases
(0) Different ways to prevent the bubonic
plague and smallpox

2. What aspect of infectious diseases


does the professor mainly discuss?
(A) How they have affected world
populations
(8) The different ways they are transmitted
(C) The reasons why they were spread so
easily in the past
(D) How to prevent them by making
vaccinations accessible

Fill in the blanks to complete the summary.


The professor discusses why diseases that a r e - - - - - - - once killed so
many people. The first reason mentioned is the
- - - - - - Another factor is the
within some

of how diseases are


to certain types of diseases

------

----------------------------------------- ---------------------------- ------------ ..-----------------------------/vlain Idea Questions

203

--~..,

Office Hours
''

Listen to a conversation between a student and a professor. Fill in the diagram with the
information that you hear. 'f1
Key Vocabulary
keep track of: to pay attention over a period of time
virtually: almost but not quite
comprehensive: including everything that is necessary

Problem:

1. What are the speakers mainly discussing?


(A) Whether or not the course will repeat
content from a previous course
(B) Alternatives to the current textbook in
the course
(C) Why the student should take a more
advanced class
(D) The reasons why the student must
take a prerequisite course

prerequisite: something required in advance


overlap: the amount that two subjects extend over the
same area

Solution:

2. What concerns does the student have


. about the professor's textbook choice?
(A) It may not cover all the material in the
course.
(B) It might repeat material that he learned
in another class.
(C) There is a newer edition of the
textbook available.
(D) There are other books that are more
suitable for the subject.

Fill in the blanks to complete the summary.


The student approaches the professor to ask about what material _ _ _ _ _ __
in the class. He is concerned because the professor assigned _ _ _ _ _ _ _ that is
used in - - - - - - - - course. The professor reassures the student that
- - - - - - - - w i l l be covered in her course.

"

~:~

/j:,
i

204

Chapter 1

Service Encounter
Listen to a conversation between a student and a university employee. Fill in the diagram
'
with the information that you hear. +@u
Vocabulary
bewllde1lng: causing confusion
.vouc:her: a paper that is exchanged for goods or services

Problem:

. 1.

W~at

are the speakers mainly discussing?

(~);

Finding out which meal plan options


are available to upperclassmen
(B) Changing meal plans after the
deadline has passed
(C) Deciding on a suitable meal plan
(D) Cancelling a meal plan and getting
a refund

[[

excessive: too much


downgrade: to lessen something

Solution:

2. Why does the student want to change his


meal plan?
(A)
(B)
(C)
(D)

He does not enjoy the food.


He wants more dining points.
He wants to cook for himself.
His parents are upset about the cost.

Fill in the blanks to complete the summary.


The conversation takes place in the
administrator's office. The
student asks for more information about the university
. The administrator
explains that as
, he must remain enrolled in one of the three
- - - - - - - plans. The student listens to the information and chooses a plan with
_ _ _ _ _ _ _ than the default plan.

-------------------------------------------------------------------------------.. ----------------------------------

i
Main Idea Questions

205

--jI .

Geology
0

"""""""'"""=~m~-{'ll<ll'~~Wl><',~fl~~'<'f'>:.-"T.:<W:Or'J;<''<:>J~~~,-~~.-.''<l"':/TM:c:o...>n::<''~',- )',"'"'

.,,-

Listen to a lecture in a geology class. Fill in the diagram with the information that you hear.
+ifiAS:tw
. Key Vocabulary .
debris: fragments of something that has been broken
phenomenon: an occurrence that caii be perceived
grade: the degree of a slope

be comprised ol: to be made of .


.
saturate: to fin completely so that no more can be added .

Characteristics:

Typo 3

1. What is the lecture mainly about?


(A) The characteristics of different
landslide processes
(B) The different causes of landslides
(C) Why the term "landslide" is not
appropriate for all geological processes
(0) How different kinds of landslides
can be avoided

2. What features of each type of landslide


does the professor focus on?
(A)
(B)
(C)
(D)

Composition and speed


The damage caused by each type
Typical locations and causes
Statistics about how often each occurs

Fill in the blanks to complete the summary.


- The-professor discusses the three different types of landslides, beginning with
_ _ _ _ _ , which moves slowly and can be
. The professor then
discusses
, which moves ______ creep and is caused by soil being
saturated with water. The professor then describes a
the same process as creep but moves much faster and causes

, which follows
-~------

---------------------------------------------.. --------------------.---------------------------------------------,.---

206

Chapter 1

..

Zoology
l'.l:lr.4Wl.w=l\''.l'a!l:f"":4'l~?<hil'i=1:~'1>>":?.ill'Mr. <_\"~-" ''-.>'"-

>f'-''"-.'--;' : -,

Listen to a discussion fo a zoology class. Fill in the diagram with the information that you

: hear. +Mf!j:fl
Y,(t~i.wltn1ess: to see something happen

hlerar~hy: a system where people or things.are ranked

fi'l'~/.explllclt: particular or; exact .

. according to their importance


dispute: a disagreement oc fight

%!!~1'f<iir lf!'iiconcluct: a kind of ~ehavior.

"'
~z
"O

Gl

~
3z

Gl

Reason 1:

Example:

Reason 2:

Example:

2. What aspect of play-fighting does the

1. What is the lecture mainly about?

.'
I

professor mostly discuss?

(A) How hierarchical differences cause


animals to play-fight
(B) The similarities between play-fighting
and genuine fighting
(C) The reasons why animals play-fight
(D) How young wolves learn to play-fight

r<

'

,,

1:

;/
l.

--.... '"

(A) Why it causes specific behavioral


changes in adult animals
(B) How it prepares young
animals for adulthood
(C) How different animals interpret and
participate in it
(D) Why animals stop doing it when they
reach adulthood

Fill in the blanks to complete the summary.

~ -------------------~-----------------------------------------------------------------~------------------------------..

'

The professor gives reasons for play-fighting, which is when animals appear to be
_ _ _ _ _ , but are actually just
each other. The first function of
play-fighting is to help animals learn
says that animals fight to

later in life. The professor also


is.

who the

fvla1n Idea Questions

______

- - --

-----"'=-- -

207 - -

--~~-=-~~~~=------------------------=

.I

j'

l
l

'j
:

~3

:I~

ijr1
'f~!

,:_1,

J
'

:"

,.

l1i1:

.,4

lji

::i

,!.~'/.,'.:
! :

~i .

~,

! ~j~'.'.
: I::

;J:y;
'Ji

-,,,

::

'.'

Necessary Skills
=.;-girarn,J!~.;w:it'<!.l;..xr.ttmY-.r;;~''A\'~:i:v;il'W-'<c\'r.~"""'''.'., t-~o;~t,cq~!_--

,,,.,,_, './"""

Taking note of major points and important details of a lecture or conversation


Listening for signal expressions that identify details, such as the following: for
example, the reason is, on the other hand, I would say
Eliminating incorrect answer choices
Identifying a statement that is not mentioned

i
i

According to the lecture, what is


?
Which of the following is true, according to the lecture?
What does the speaker say about
?
What connection does the speaker make between _ _ _ __
?
. and
What does the professor suggest the student do?
Which of the following is true of
?
What advice does the professor give to the student about
?
According to the discussion, how did
?
According to the speaker, why do
?
According to the conversation, why must the student
?
According to the speaker, who were
?
According to the lecture, what kinds of
? Choose 2 answers.

Since answers to questions are generally found in order in the passage, it is


helpful to take notes in the order of what you hear.
Detail questions do not require inference. Choose what speakers actually say.
In a lecture, detail questions are about information related to the following: new
facts, descriptions, definitions of terms/concepts/ideas, reasons, results, and
examples.
o Incorrect choices may repeat some of the speakers' words but do not reflect
correct information from the lecture or conversation.

Detail Questions

---------- '-~-==------- - - - -

209 - -

- - ---- - =

Sociology
~

!C;p&=~ttm'>J'.,~MJ:\'~:.<.~:'!-'\'J,'i/)!!ifl.<~9.W,\'l':""'.O:-;:

(Co'l'"Y\.o''.:,-;.

Listen to a lecture in a sociology class. Fill in the diagram with the information that you hear.
liijj:jl
Key Vocabulary
dynamics: a set of forces that exist, especially in a
relationship
set off: to show the difference between two things

consequentfal: important
unstable: easily made to fall or come apart
cease: to stop

.. Dyads
Definition:

Characteristic 1:

<
1. According to the professor, what is a dyad?
(A)
(B)
(C)
(D)

A social problem
A group of two people
A circle of friends
A rule for marriage

2. According to the professor, how are dyads


different from other relationships?
(A) They usually have negative
consequences.
(B) They are more stable.
(C) They require a specific number of
people.
(D) They are much more intense.

Characteristic 2:

3. According to the professor, which of the


following is true?
(A) Groups with several members are
likely to be unstable.
(B) A club may not survive if a single
member leaves it.
(C) A photography club is a common
type of dyad.
(D) Dyads end when one person leaves
the relationship.

Fill in the blanks to complete the summary.


--~

:i

ii

F.U.,

l."i',

...-..., ..... ____ .,.,_ . ______ ... _,,.,.,.,. __ ,..... __ ....


,

~,;11;

''It'

,__

.. ---- ... , .. ___ ,,_, ____________,., ____

~---------~-.--------------

.. -----.....

--

The professor discusses dyads, which are groups of _ _ _ _ _ . This type of group
is ______ for two reasons. First, it is
, since each member has only
_ _ _ _ _ _ _ to focus on. Second, it is a very _ _ _ _ _ _ _-if one person
leaves, the group ______

------- -- ------- -------- -.... --- - --- ----------------------------------------------------- .. ------------- --

i\j'1

"\I~;_,- 210

._

Chapter 2

Psychology
-.:ttw:wM'~.r;;:m.~wa"!'.lifn;;51;'t,"lf.>'7'A11t~~'l,,!;'1'1.w.w-"X:m<t..1\V;'C4>'.t~

'l' -~'" ,.- .,,

Listen to a lecture in a psychology class. Fill in the diagram with the infqrmation that you
hear. @S:M
Key Vocabulary
Inexplicable: unable to be explained
. approximate: to guess
manifestation: an outward appearance

accurately: correctly: exactly .


prone: likely to do or have something

;.Children an(lPanrc'Disbrder ',

Theory:

Support 1:

"

<
1. According to the professor, what is panic
; disorder?
(A) Feeling stronger pain than normal
(B) Powerful and unexplained feelings of
anxiety
(C) Rapid mood changes
(D) Unusual sensitivity to the body

'

I
I

l!

,I'.

,,..

2. According to the professor, how are body


awareness and anxiety related?
(A) An increased heart rate can cause
panic disorders.

'"_,

Support 2:

(B) Children feel anxiety more intensely


than adults do.
(C) Children who experience pain can
develop panic disorder.
(D) Being worried about one's body can
lead to anxiety.

3. Who were the subjects of the study?


(A) A group of randomly chosen children
(B) A group of children with panic disorder
(C) A group of both adults and children
(D) A group of normal children

It

Fill in the blanks to complete the summary.

'

The professor discusses a theory that links panic disorder with being aware of
- - - - - The professor describes the study, in which
were
asked to guess their
. The study showed that the children
_ _ _ _ _ _ _ their heart rates accurately were more likely to _ _ _ _ _ __
The professor also discussed how people who worry when they feel pain can develop

--------------------------------- .. ------------------------------------------------------------------------Deti!il Questions

211 - -

Office Hours
Listen to a conversation between a student and a professor. Fill in the diagram with the
information that you hear. @UI
Key Vocabulary
bring up: to raise or make better
be one's best bet: to be the best option or plan of action
take In: to understand

Problem:

hit: to suddenly understand something


clear up: to clarify or explain

Solution 1:

~--~ ISolution 2:
I Solution 3:
1. Why is the student worried about the
class?

,;

(
j_'

'

'

;' . ' t

(A) He does not know how to find out his


grades.
(B) He thinks his grades are too low.
(C) He has not studied enough for the final
exam.
(D) He wants to do extra work to improve
his grade.

3. What does the professor suggest the


student do before the final exam?

(A) Make an outline of the class's main


topics
(B) Do extra homework to get more grade
points
(C) Go over the textbook before the exam
(D) Visit the professor to find out what
material to study

. 2. What advice does the professor give to the


student about studying for the final exam?

(A)
(B)
(C)
(D)

Study the material in more depth.


Study the material in the book.
Study the main points of the material.
Study with a group of other students.

Fill in the blanks to complete the summary.


The student goes to see the professor because he is worried that he is not _ _ _ __
. In addition,
. in the class. The professor suggests that he try do well on
the professor suggests studying only
. The professor also recommends
that the student _ _ _ _ _ her to _ _ _ _ _ any remaining confusion about the
material.
~_:-;,<.- 212 Chapter 2

Service Encounter
Listen: to a conversation between a student and a university employee. Fill in the diagram
. with the information that you hear. i@j:IM
,.,., .., -

fil.

Key Vocabulary

l~{'c\,.'

ffi:.

check out: to borrow from a library or similar place


subscribe: to pay for and receive something over a fixed
>
period of time

demand: desire for or interest in


reserve: something saved for later use
priority: position ahead of others

~-::

Problem:

Solution:

Option 1:

/
i--

. _.,

l'

--1.:;,c_ What problem does the student have?

.
.f

l
~~'
-1

I
f

I
!

I
.

Option 2:

,,,

: (A) He is trying to find the location of a


different library.
(B) He does not know which journal
contains the article he wants.
(C) He is unable to locate a particular
journal that he needs.
(D) He does not know which library has
the journal he needs.
2. According to the woman, what is the
interlibrary loan system?
(A) A system that allows students to
borrow from other libraries
(B) A network that allows students to
locate checked-out books

(C) A system that rents books to other


people for a small fee
(D) A database that tracks what books
students have borrowed
3. Which of the following is true according to
the conversation?
(A) The student will have to go to a public
library.
(B) The woman will reserve a journal for
the student
(C) The student will not be able to get the
journal issue.
(D) The university library charges a fee for
each loan.

Fill in the blanks to complete the summary.

----------- ----------------- ........ --- _.,._ .......------ .. ----- ---------------------------- ----- .... -----------------------A student needs a
at the library but cannot find it The librarian suggests
something called an
, which is a system that allows the library to
_ _ _ _ _ books and journals from other libraries. She can look for the journal at a
_ _ _ _ _ library, where it will probably be in stock. However, the student asks her to
look at a
library, which will be-~------------------------- .. --------- -------------------------------------------------.. ----------~~bt>tail Questions

213 - -

;-- 214

Botany
JUCPmm.1'.t"<lli!Sm:JM!.1/~:WiMf>lfo.."'<l.W:l'!MX6""~qt<",'f><~MA::.-..,~''-f'-'-.-1-:a',-r-"'''';'~'_;

<''>""~o~<f'

.:, -

' Fill in the diagram with the information that you


Listen to a discussion in a botany cla~s.

hear. @s:+
Key Vocabulary
blossom: to produce a flower
measure: an actiOn taken

widespread: occurring often or in many places ..


vulnerable: easy to attack
prematurely: too soon

j . Diseases Affecting Roses ' '


I

Disease 1:

Disease 2:

_ __.__r-----_______

lc
Lha-ra_c_te-ristic~------

1. According to the professor, what are the


symptoms of black spot? Choose 2
answers.
(A)
(B)
(C)
(D)

A weakened plant
Dark circles on the leaves
Dust covering the plant and the ground
The loss of leaves leading to brown
growths on stems

2. What does rose rust cause?

--------.-,1--------'

I Characteristics:
(C) A plant that suddenly loses all of its
leaves and turns orange
(D) Orange growths that appear near the
base of the plant
3. What are the two rose diseases caused by?
(A)
(B)
(C)
(D)

Viruses
Bacteria
Fungi
Insects

(A) Orange spots and orange dust on


stems and leaves
(B) A dehydrated plant with orange rings
around the stems
Fill in the blanks to complete the summary.

.. -.. -------"'''>------------------------.----"'""
Roses are vulnerable to - - - - - - - t h a t are caused by fungi, bacteria, and
viruses. One of the diseases is called
, which causes round black
spots to appear on the
, causing eventual damage to the entire plant.
Another disease called
causes orange spots to appear on the leaves,
as well as _ _ _ _ _ __

~M---'"-H----,.--------"'~-------'-'----------~-------------------------

Chapter 2

{:

Anthropology
~_.,.-~.Zo!6'Ull'lO!>l'Cil'-fll.~<bC.-,;"1,'.!tii:<::-'1-<xH'\'.<h<:<~!1.1;_;;-,.,'.-\l'""''-'"~..'-1'>'-"""'~..

~:,--

'

Listen to a lecture in an anthropology class. Fill in the diagram with the information that you

.,.. ,..,,,,, .. hear. Af:t.

'

!!'l~E'){~y Vocabulary

.~~'.~dapt: to adjust to conditions

,,,,,, ' . :i\i'J'lillck (something) up: to support or give evidence for

~?J.~~chor: to ground or hold in place

Innovative: creative
retrieve: to go get and bring back

Theory:

<
1. According to the professor, why did the
,Aztecs build floating gardens?

'(A) To allow easy travel between islands


(B) To provide space for people to live
(C) To provide fresh water to their city
(D) To grow certain kinds of plants
; 2. How did the floating gardens stay in place?

(A) They were held in place by tree roots.


(B) The Aztecs built aqueducts to hold
them.
(C) Tree branches kept them from moving.
(D) They were between channels of flowing
water.

3. According to the professor, how did the


Aztecs obtain fresh water?
(A) By travelling to springs outside of the
city
(B) By replacing swampland with floating
gardens
(C) By constructing artificial channels to
fresh water sources
(D) By building structures to separate
fresh water from swamp water

Fill in the blanks to complete the summary.


The professor discusses how ancient civilizations sometimes had to _ _ _ _ _ __
environments that were difficult to live in.
adapted to their environment
in a couple of different ways. They had to build.
over the swampland to
make room for people to live. Second, they built aqueducts to provide _ _ _ _ _ __
to their city, which was surrounded b y - - - - - - -

----------------------------------------------------------:..------------------------------ .. ---------------------Dewil Questions

215 - - "

Office tiours
Listen to a conversation between a student and a professor. Fill in the diagram with the
information that you hear. @U
Key Vocabulary
permit: to allow
right up one's alley: suiting one perfectly
oriented: favoring a particular point of view

construction: formation; something that is created


dig Into: to explore

"Introduction to International Relations" vs. "Representatio_ns ~~~~ce ;dP'oiitics"


International Relations:

Both:

Race and Politics:.

_._J

_________________ __

1. According to the student, why can't she


take both classes?
(A) She cannot afford to pay for both of
them.
(B) The university will not allow her to take
both classes.
(C) She is worried that she will be too
busy next semester.
(D) She has time in her schedule for only
one class.
2. According to the professor, the class
"Introduction to International Relations" is
best for which of the following?

,,._,

(C) International students


(D) Political Science students
3. What advice does the professor give to the
student?

(A) She should take the class she is most


interested in.
(B) She should take a class in a different
department.
(C) She should take a less advanced
class.
(D) She should get more personal
experience outside of class.

(A) Freshman
(B) Business students
Fill in the blanks to complete the summary.
.,-

.------------.--------------"''" --""" ---- .. - ............. - .. -"- __. ___ ,---- - <>---------------~-- ..... .

The student goes to the professor's office _ _ _ _ _ _ _ . The student cannot


decide which class ______ . She likes both classes, though one class is more
appropriate for her
. However, the other class
to her. The professor recommends _ _ _ _ _ _ _ class.
;

_l{;{;':!

fh#--:~;1~H

"

216

Chapter 2 .

is_~-----

~-

Service Encounter
Listen to a conversation between a student and a university employee. Fill in the diagram
. with the information that you hear.

@!"'

Key Vocabulary

\~;_!'-?

i):..--.. scholarship: an amount. of money awarded to a student ..


\, .

~~~f- f8e: the amount of money paid for a privilege or service

[
[

dlsenroll: to remove from a roll or list


extension: extra time given to finish something

\;(:f: 'tuition: a sum charged for instruction at a university

Problem:

Solution:

r.
ft
.

,,,.,

1+. Which of the following is true of the WOW


Scholarship?
, (A) It does not include money for tuition
costs.
(B) It provides $200 toward the student's
education.
(C) It pays for the student's entire
education.
(D) It does not pay for class fees.

3. What does the worker suggest to the


student?
(A) Look for scholarship information online
(B) Ask her grandparents for money
(C) Wait until she gets paid from her job
(D) Visit the scholarship office

2. According to the student, when should she


pay the fee?

r.

(A)
(B)
(C)
(D)

By the
By the
By the
By the

end of the day


end of the week
first day of class
end of the semester

Fill in the blanks to complete the summary.


The student did not realize that certain university classes had _ _ _ _ _ ,.and she
now has _ _ _ _ _ she cannot pay. She does not know how she will pay
I

_ _ _ _ _ _ _ . The student did not know that she - - - - - - - for more than
one scholarship, so the worker advises her to
additional scholarships.
----------------------------------------------------------------------------------------------------------

Detail

Qu~stions 217 _.-

j ,_.______________.................................................................
L

i.1'' .;,q. J'

Political Science
Listen to a lecture in a political science class. Fill in the diagram with the informat!on that

you hear.

@''

Key Vocabulary
sovereign: having supreme power or highest rank
face: to experience or to encounter
abolish: to get rid of

goodhearted: kind and generous


succession: a process in which things or people
follow each other

"' ~Ab$dlqfo,Mo,n~r,9hy d,, I

l~A-d-v-an-ta_g_e_1_:---~ -~~1~S-u-p-po-rt-:--------~---1

Etago

--1

S"ppcrt

1. What does the speaker say about


Catherine the Great?
(A) She used her power mainly to aid the
upper class.
(B) She is proof that a bad monarch will
hurt a country.
(C) She was a good monarch who helped
her country.
(D) She felt that increasing taxes would
strengthen her country.
2. According to the professor, which of the
following is an advantage of absolute
monarchies?

(B) They offer the people a larger voice in


government.
(C) They bypass the election system and
simplify government.
(D) They minimize the chance of negative
reforms and changes.
3. According to the professor, why was it
good that monarchies did not have
elections?
(A)
(B)
(C)
(D)

Many people were illiterate.


Few people wanted to vote for rulers.
Elections do not require participation.
Rulers often ignored the results,

(A) They provide unlimited power for long


periods of time.
Fill in the blanks to complete the summary.

The professor discusses the _ _ _ _ _ of absolute monarchies. First, absolute


monarchies can give
the power to make good changes. The professor
uses the example of
. With an absolute monarchy there is
______ for elections. This made government easier in the past, since many people
could not ________

---------------------------------------------------------------------------------------------------------------------

Music History

~il:llf.6~~W&:i~%:~./\ll>1N,,H<U!f'M>~~o~-~'1-~~''~yx;<."'~-'.,_-,

"'"'''''' Listen to a discussion in a music history class. Fill in the diagram with the information that

you hear. ''Ml''


-~/

,''

rf' Key Vocabulary

tL 'epic: having heroic or monumental qualities.

,
'
;f;i. folk: relating to the culture and history of common people
.
,.,, commit: to keep for future use
' tt:.>'.:.i
't:t"!-;_'

.,.

'

Definition:

1. Apcording to the professor, which of the


fciliowing defines Celtic culture?

l
r

I
r

!t

'

(A) Western European culture up to the


Middle Ages.
(B) The culture of Eastern Europe where
bards originated
(C) Modern European culture including
Ireland and Scotland
(D) The culture of Central Europe during
the Middle Ages.
2. According to the professor, what kinds of
stories did bards tell in their songs?
(A) Stories about ancient European
cultures

melody: the arrangement of musical notes to create


a pleasurable musical passage
catchy: easy to remember, as in a song

Role:

(B) Epic stories about real and fictional


events
(C) Historical accounts of Celtic kings
(D) Original stories created by the bards
3. How did music help people remember
history?
(A) Memorable melodies helped people
remember stories.
(B) The bards' music inspired people to
write stories about history.

(C) Bards encouraged people to continue


singing their songs.
(D) The music of the bards used elements
found in rnodern literature.

'
l

I
i

Fill in the blanks to complete the summary.


.. - - - - - - - - .. ---'-------<ouu~--."''''"' __ ,.,,_,------n-n---,._., _____ - - - - - - - - - - - - - - - - - - - - - - - - - - - - - - - - - - - - - - - - - - - - - - -

The lecture defines bards as _ _ _ _ _ that played instruments and sang. Bards
are a part of Celtic
and had an important role in its history. The professor
discusses the role of bards, saying that they helped the people
the stories
that described their
. The catchy qualities of music helped the people
remember ________

Instructions: Chaos~ the best word or


phrase to complete each sentence.
1. Because the king did not have a son, his
eldest daughter became next in line of
- - - - - for the throne.

(A)
(B)
(C)
(D)

t {'
I

scholarship
sentiment
succession
sovereign

2. I believe in life after death, but I cannot


_ _ _ _ _ my claim with any hard
evidence.

(A)
(8)
(C)
(D)

back up
take in
dig into
keep track of

3. Research League International continues


to
in innovative strategies
for medical research.

(A)
(B)
(C)
(D)

blaze the trail


bring things up
check out
set off

4. Some things in nature seem


_ _ _ _ _ , such as the almost
telepathic bond that develops between
some owners and their pets.
(A)
(B)
(C)
(D)

5. The _ _ _ _ _ of two small, older


restaurant chains cost the corporation
$500 million.
(A)
(B)
(C)
(D)

acquisition
assimilation
construction
manifestation

6. The website has a great video showing


the inside of the pyramid. It's like
_ _ _ _ _ being there!
(A)
(B)
(C)
(D)

approximately
notoriously
prematurely
virtually

7. There is
support for the
president's new environmental policy.
(A)
(B)
(C)
(D)

prone to
one's best bet
right up people's alley
widespread

8. Without knowing how much was spent


last year, we cannot
a
new budget for the coming year.
(A)
(B)
(C)
(D)

charge
dispute
formulate
manipulate

habitual
inexplicable
unstable
vulnerable

:'

i<"i'

~~~~-- 220

i\';.l.1'.
'<

Vocabulary Review 1

.,
"

Instructions: Choose the word or phrase


closest in meaning to the underlined part.

14. The students came up with a new and


clever way to do their group presentation.
(A)
(B)
(C)
(D)

I probably wouldn't eat it everyday, but


tofu burgers for lunch made a fun and
interesting change to my usual routine.
(A)
(B)
(C)
(D)

bewildering
incoming
overlapping
refreshing

~
2S

G'I

"'3
z

15. The website's security will not allow you to


access the database without first entering
a valid user ID and password.

(A)
(B)
(C)
(D)

G'I

...

permit
reserve
setup
transmit

'"-I

Instructions: Write the missing words. Use


the words below to fill in the blanks.

, 11 ....Someday we will completely defeat such


.diseases as cancer and heart disease.
j

contract

commit
comprise
conduct
conquer

fees

disenrolled

12. The bus driver seemed very kind and


tIBQilll and always had a smile for anyone
who got on the bus.
(A)
(B)
(C)
(D)

...

Ill

:;;

10. It was difficult for women to rise very high


within the company's ranking system.
(A) authority
(B) hierarchy
(C) melody
(D) priority

.(A)
(B)
(C)
(D)

explicit
infectious
innovative
sanitary

anchored
disquieted
good-hearted
oriented

13. The principal told reporters that the


school had taken "appropriate serious
action" with regard to the misconduct of
cafeteria employees.
(A) catchy
(B) consequential
(C) comprehensive
(D) contemporary

extensions
tuition

Above the basic 16. _ _ _ _ _ __


paid by students, some classes also include
special 17.
. According to
the student 18.
that
applies to anyone accepting admission to the
school, these fees must be paid before the
first day of class. No 19. _ _ _ _ _ __
for paying late fees will be given. Any
student who has not paid his or her special
fees by the first day of class will be
20.
from the class.

Instructions: Match the words that are similar


in meaning.
21.
22.
23.
24.

debris
cease
departure
retrieve
25. voucher

(A)
(B)
(C)
(D)
(E)

conclude
collect
exit
form
fragments

'

.A
\. ., ,;. .~.;

Vocabulary Review 1

221

-~.1

I l!!llll!!!ll!!!l!!!lll!!!lllll!!!ll!!l!!illl!!!ll!!!!!!!!!l!!!!!!l!!!!!ll!l!ll!lll!!!ll!l!llllllllllllll......111111111111...............- ..................
.... . .....\'.){
._,''""
- - ---

;~ :; __

---

!_:,_'-

01 Anthropology
Listen to a lecture in an anthropology class.

@1

Key Vocabulary
distinct: clearly different
descend from: to come from; originate from
refute: to argue against; to dispute
reconstruct: to build again
ln~ablt: to live in

1. What is the lecture mainly about?


(A) The ways that Aztec and Mayan
mythology are similar
(B) How aspects of Aztec mythology
were incorporated by the Mayans
(C) Why the mythologies of both cultures
share some similarities
(D) The reasons why the Aztecs never
conquered the Mayans

2. What theory about the Mayans and


Aztecs does the professor support?
(A) Their mythologies were similar
because they descended from the
same peoples.
(B) Living near each other and seeing the
same landscape made thern similar.
(C) The Aztecs were direct descendants
of the Mayans.
(D) Only Mayan beliefs are supported by
archeological finds.

3. According to the professor, which of the


following statements is true?
(A) The Aztecs predate the Mayans by
several hundred years.
(B) The Aztecs sold com to the Mayans.
(C) The Aztecs and the Mayans lived in
the same territory.
(D) The Aztecs and the Mayans
originated at different times.

negotiation: the reaching of agreement through


discussion
expanse: a wide, open area
staple: a basic, necessary food or product
Inevitably: certainly; unavoidably

4. Which modern countries are in the same


areas the Mayans used to live?
Choose 2 answers.

(A)
(B)
(C)
(D)

Guatemala
Mexico
The United States
Brazil

. 5. What was a result of the trade relationship


between the Aztecs and the Mayans?

(A) They intermarried.


(B) They developed new kinds of
products.
(C) The exchanged customs and beliefs.
(D) They were suspicious of each other.
6. According to the professor, what food

staple did the Mayans and Aztecs have


in common?
(A)
(B)
(C)
(D)

Corn
Beef
Bananas
Cactus

02 Business
Listen to a discussion in a business class. +@@II
({t-.q'-___ :.

~J~ l{ey Vocabulary

[~(;.:reinforcement: a repeated process to aid learning.


ij'.)'i'.the
floor Is open: anyone is allowed to speak
-'.;;y_ ..
~tt1t; fare: the cost of transportation

rf:i{' m.alntenance: work lhal is done to keep a machine


~~~fi:''-'
f~.;,-

fuel: something used lo power a machine; like gasoline


fluctuation: changes over a short period of time
hedge: to try to avoid possible losses in a financial
transaction

:e

"'3z

1n working order

Cl

7. What is the main topic of the discussion?


(A) The reasons why the low-cost carrier
model has failed
(B) How a company can save money by
eliminating services
(C) The ways that low-cost carriers can
improve
(D) What a low-cost carrier is and how it
works
-~:
! 8. , What aspect of the business model does
\
)he professor mainly discuss?
t

(A) The ways the company can attract


more customers
(B) The disadvantages of operating a
low-cost carrier
,
(C) How a low'Cost carrier keeps. costs
low
(D) Why low-cost carriers lose so much
money

9. According to the professor, what content


will be on the test?
(A) Information about a certain airline
business model
(B) The common business models for
airlines
(C) Ways to improve airline business
models
(D) The cost of opening and maintaining
a low-cost carrier

10. By definition, how are low-cost carriers


able to provide low fares?
(A) They do not have to pay for fuel.
(B) They get rid of extra services.
(C) They operate a limited number of
planes.
(D) They get rebates from the government

.,,

n
rn

...
~

11 What are some of the stated benefits of


using only one kind of aircraft?
Choose 2 answers.

(A)
(B)
(C)
(D)

Ease of maintenance
Less fuel is used by the airline
Better service
Discounts on aircraft

12. What is an advantage of using secondary


airports?
(A) They are busier than major airports.
(B) They are open at convenient times.
(C) There are fewer regulations.
(D) There are reduced landing fees.

13. What is fuel hedging?


(A) The practice of buying fuel before it
is necessary
(B) The state of changing fuel prices
(C) Buying aircraft fuel in bulk
(D) Buying fuel cheaply and selling it for
a higher price

Mini Test 1

223 - - ,

--;1;

. ,,,

03 Service Encounter
Listen to a conversation between a student and a university employee. ljffi@&i
Key Vocabulary
graduate: to receive a degree after completing a
course of study
Independent: free from external control or inlluence
offlclally: in an approved, recognized role

14. What are the speakers mainly discussing?


(A) How to gain access to the library
storage room
(B) Why non-students are not allowed in
the library
(C) How the student can use the campus
libraries
(D) Why the student should be allowed to
use the facilities
,,

(A) The cost to obtain the pass


(B) The types of access and services the
student will have
(C) The parts of the library that will
remain off-limits
(D) The ways to speed up processing of
the application
16. Which of the following is true about the
man?
(A) He wants to study library science.
(B) He cannot locate his library card.
(C) He has already graduated.
(D) He wants to take more classes next
semester.

i,

'

~~-iy~:.1.i,"- 224 Mini Test 1

J'''i''

[;ll;.;

''ill
~

17. Why does the man still need to use the


university library?
(A) He cannot get into any of the other
libraries.
(B) He is working on an independent
study project.
(C) He needs to finish a project before he
can graduate.
(D) He is taking very difficult courses this
semester.

15. What feature of the non-student pass


does the librarian mainly discuss?

administer: to manage or control


confirm: to check that something is correct

'

'

.'

18. What is a disadvantage to having


non-student access?
(A) It only includes access to the main
library.
(B) It costs $50 per semester.
(C) It requires the student to get a new
email address.
(D) It is only free for graduates of the
university.

i\lecessary Skills
l.'ll"lS.fJ.=..Y.=~oti'Sc2'Jr.1J.i"'~'<'<'?.-Ei,:.'-'>.\O.'i-'>'-':~;~!..i-",.,c.\"if!<'J\';(.d,,

_,-,: o',' -,,,

Understanding what a speaker is trying to achieve through what is said


Inferring a speaker's reason for saying a certain sentence or phrase
Using the context to figure out the real meaning of a sentence or phrase
Recognizing the tone of voice, intonation, and sentence stress that a speaker
uses to show his or her intended meaning

Function questions will not appear during the test. You will only hear them.
Listen again to a part of the conversation. Then answer the question.
You vyill hear a few lines of the lecture or conversation again.
Why does the
say this:
You will hear part or one line of the previous excerpt again.
The following types of function questions may appear during the test:
- What is the purpose of the
response?
- What does the
imply when he/she says this:
- What can be inferred from the _ _ _ _ _ response to the _ _ _ _ _?

Listen for the overall organization of the lecture or conversation and think about
whether the purpose is to describe, explain, compare, or give an opinion.
Consider the relationship between the speakers and the context in which the
speakers meet.
Use clues like intonation to help you understand the meaning behind the words.

Function Questions

225 - -

literature
Listen to a' lecture in a literature class. Fill in the diagram with the information that you

hear.

+Mfti'*

Key Vocabulary
flaw: a bad feature or fault
fortune: situalion in life
lighthearted: cheertul; not serious

clever: intelligent or inventive


ruse: trick

<Genre .

[Definition:

Example 1:

Example 2:
I

Definition:

'

Example:

1. Listen again to part of the lecture. Then


answer the question.

Why does the professor say this:


(A)
(B)
(C)
(D)

... I
I

,, I
!

To
To
To
To

provide an example
return from a digression
define an important term
give an opinion

2. Listen again to part of the lecture. Then


answer the question.

Why does the professor say this: \)

Example:

(B) To see if students are ready to continue


(C) To ask the students to participate
(D) To make sure students are listening
3. Listen again to part of the lecture, Then
answer the question.

Why does the professor say this:


(A)
(B)
(C)
(D)

To support a theory about the play


To introduce a quality of comedy
To suggest the information is wrong
To give her opinion of the play

(A) To announce a digression


Fill in the blanks to complete the summary.
.. --

--------------------,.----_,._w-----~--'4------,.-------"--~--'"'"------------------------,...,----

.-1

'.,\'!.i

+:1
it;i:'li

~\'b'

r~~iL 226

The professor defines a ______ as a category of art, music, or literature. She


mentions two different genres. First,
features a hero who experiences
bad things due to
in his or her character. Euripides was a writer of tragedy.
The professor also mentions comedy, which dealt with
. Shakespeare
wrote
comedies about confusion in love.

.
---..-------..-----------------....----- . ---..--.. -..--.. --............ _. ___ .............................
_______ ,, ______ ,, _________ ................ -------Chapter 3

',,
---------- - ---- --- -- ----- - ;------

History
'

~.11~;m~=~<>;-0gff~'~inlt"l~R-~-"'-lru"""'"'"Pi'J.~'.'':'-'"'"''''"

Listen to a lecture in a history class. Fill in the diagram with the information that you hear.

@@
Key Vocabulary .
employ: to hire
behind: the cause of
daring: fearless or adventurous

exploits: famous achievements.:


break out: begin suddenly

to

Definition:

Example 1:

<
; 1. Listen again to part of the lecture. Then
.
answer the question.

Why does the professor say this:


1

(A)
(B)
(C)
(D)

To compare two types of pirates


To explain the importance of pirates
To describe who the Sea Dogs were
To say how pirates earned money

2. Listen again to part of the lecture. Then


answer the question.

Why does the professor say this:

(A) To clarify a statement

.Example 2:

(B) To define a term


(C) To compare two points
(D) To correct a mistake
3. Listen again to part of the lecture. Then
answer the question.
Why does the professor say thi.s:

(A) To emphasize her impartiality on


the topic
(B) To criticize Queen Elizabeth
(C) To express her doubt about a theory
(D) To give an example of piracy at sea

Fill in the blanks to complete the summary.


The professor discusses a group of
pirates called the Sea Dogs, who
stole from
. The Sea Dogs were hired by
of
England. They stole money from Spanish ships to try to hurt
. They also
took over Spanish _ _ _ __
Function Questions

227 ___,

Office Hours
1'
I

,ti~.

,,......

Listen to a conversation between a student and a professor. Fill in the diagram with the
information that you hear. @*'
Key Vocabulary
fulfill: to satisfy or complete
rigorous: requiring a lot of discipline and obedience
speclallzatlon: a chosen focus of work or study

Problem:

grant: to offer or give


advocate: to argue for

Solution:

1. Listen again to part of the conversation. Then


answer the question.

Why does the professor say this:

(I

(A) To suggest that the student change


her major
(B) To show his happiness at the student's
comment
(C) To suggest that the student has insulted
him
(D) To provide an answer to the student's
question
2. Listen again to part of the conversation. Then
answer the question.

Why does the professor say this:

(I

(A) To see if the student has time to


complete the major
(B) To recommend that the student not
add the major

(C) To determine if the student has taken


enough classes
(D) To see if the student has completed
the lab requirements
3. Listen again to part of the conversation. Then
answer the question.

Why does the professor say this:

(I

(A) To express doubt about the student's


abilities
(B) To provide a solution to the student's
problem
(C) To recommend a plan of action for the
student
(D) To ask the student to make a decision
quickly

Fill in the blanks to complete the summary.


The conversationtakes place between a student and a professor. The student
approaches the professor in order to
about her academic plan. She
______ ti is class, so now she is considering _ _ _ _ _ _ _ . The professor
expresses concern that the student is considering doing this so
in her
academic career. He thinks that a
in psychology might be better.
------------..----- .. -------- .. ---------~-------.------------------------ .. ---------------------------------- --'"'
~\;_.~. ..--- 228 Chapter 3

Service Encounter
Listen to a conversation be.tween a st~dent and a university employee. Fill in the diagram
with the information that you hear. lit!ll"'
J{Y;;.c,: __-.

[:_;/,J<eyVocabulary
.
'\(W(''.->:,,-- .- ,
.
- _.-_-_.
~iit\?ruclal: extremely Important
~;,;:,tutor: private instructor < . .

get In touch:. to contact someone ..


via: through or using

- f~~f;S~-~-k up:._t~ get m8t91ied_ ~ith'


nl;.-,,,.,

Problem:

Solution:

1. Listen again to part of the conversation. Then


answer the question.
Why does the student say this:

(A) To describe his problem for the


woman
(B) To complain that the center does not
have enough tutors
(C) To find out ii the woman can help
him
(D) To ask the woman to help him write a
paper
2. Listen again to part of the conversation. Then
. answer the question.
Why does the woman say this:
(A) To assure the student that his request
for help will not be ignored
(B) To demonstrate how competitive the
writing fellow program is

(C) To show that the application does not


ask personal questions
(D) To explain how he can finish the
application quickly
3. Listen again to part of the conversation. Then
answer the question.
Why does the student say this:
(A) To find out why it will take so long
(B) To ask how he will be contacted
(C) To show his happiness about the
solution
(D) To answer the wom.an's question

Fill in the blanks to complete the summary.


-------------------"-'" '"""" .. '....... ,, .............. ---..
....... --
----------------'"---.-------------------The student visits the Academic Support Center to request help with _ _ _ __
The coordinator suggests that he work with a
, who would be able to
walk him through the entire
process. The student fills out
_ _ _ _ _ _ _ , which the coordinator tells him will be used
a writing
----~------

~-----

fellow for him.

------------------------.

... ,. .... -........... -----------.. ------------------"-"--"--"-----~----------- Function Questions

229 __,

Environmental Science
,

Listen to a discussion in an environmental science class. Fill in the diagram with the information

that you hear. @sm+


Key Vocabulary
pop quiz: a quiz given without prior notice
evaporation: the process by which liquids change
into water vap9r

gaseous: made up of gas


current: a steady flow of air or liquid
. condense: to change from gas to liquid

>.rile Water cjC!e >.. I


,

-.,

,'

.,

,_.

'

Step 1:

Step 2:

Description:

Description:
)c

.,'

,.f,

Step 4:

Step 3:

Description:

..---

Description:
[

------1. Listen again to part of the discussion. Then


answer the question.

Why does the professor say this:

(A) To explain why she is revisiting a topic


already covered
(B) To show her disappointment with the
students' quiz grades
(C) To introduce a difficult concept
(D) To ask students to pay better attention
2. Listen again to part of the discussion. Then
answer the question.

Why does the student say this:


(A) To make sure the professor
understands him

(B) To see if his answer is good enough


(C) To ask for a more detailed explanation
(D) To show that he understands the
process of evaporation
3. Why does the professor say this:

(A) To offer another way to understand a.


concept
(B) To see if the students are paying
attention
(C) To describe the differences between
two processes
(D) To emphasize that the processes are
easy to understand

Fill in the blanks to complete the summary.


-----------------------,-"''----------'--------------------------.. ---------------------------------- .. ,,The discussion is about the water cycle. The professor explains the process, beginning
with
, when water becomes
. Next, the water
________ and turns into a liquid again.
is the third step, in
which water moves in air currents. Last, precipitation happens when water

-----------.------------------------------------------------------------ .

\f1~--- 230
r;
'I~

Chapt.cr 3 _

-------------------------------~----

.. -

'

Psychology
...

L~~-.l':~~-~~~~ ')l;'~,'::\il>.'l=~ui;;\1,~l.m.'.;\<\,>;cV<;'1')',f''>'.,_-._-

;,._.,.,,--'

Listen to a lecture in a psychology class. Fill in the diagram with the information that you

hear. 'Ml'"'
~;)'

: .. Key Vocabulary

\~.;
f,:J:

famlllarlty: knowing someone or something well


convene: to come together
r,::::, consensus: a state of agreement within a group
~--'"'-

..

'!'...

Step 1:
Description:

Step 2:
Description:

Step 4:

Step 3:
Description:

Description:

<,::':. .

i~

,.f.\

~).

etncacy: the ability to produce a desired outcome


competence: being qualified or able :

1. Why does the professor say this: (I


.(A) To summarize "forming"
(B) To introduce group development theory
(C) To give an example of how groups form
(D) To explain how Tuckerman chose the
first stage in his model
2. Why does the professor say this: (I
(A) To provide another way to understand
the concept
(B) To describe the role of "growing
pains" in group development
(C) To compare two different stages of the
model

(D) To explain why this stage of the model


is called "storming"
3. Listen again to part of the lecture. Then
answer the question.
Why does the professor say this: (I
(A) To emphasize the difficulty of this
stage in the model
(B) To illustrate why groups become
stable at this stage
(C) To describe how group members
come to trust each other
(D) To show how one stage leads to another

Fill in the blanks to complete the summary.

The professor introduces the Tuckerman model and describes four stages in
~---~--The first is "forming," which is when the group is _ _ _ _ _ . The
second is "storming," which is the time of
as the group struggles within
itself. In the third stage,
, the group begins to mature and form shared values.
The final stage is
, which is when a mature group is able to work together in
an effective manner.
--------------------------------------------~-------------

.. ------------------.-----------------------------------Function Questioiis

---- -,_. __ --:

231 - -

Office Hours
'
Listen to a conversation between a student and a professor. Fill in the diagram with the
information that you hear.

'"'"

Key Vocabulary
proctor: a person who watches over siudents during an .
. _ . ,'.: ~xaminat.ion

component: part

spare: to give or grant


object: to have a problem with
commitment: an obligation or promise

--------Solution:

Problem:

~---------------

1. Listen again to part of the conversation. Then

answer the question.

Why does the professor say this:


(A) To offer a solution to the student's
problem
(8) To try to understand the student's
problem
(C) To apologize to the student tor the
mistake
(D) To tell the student when the final will
take place

2. Listen again to part of the conversation. Then


. answer the question.

Why does the student say this:

(C) To explain her theater class to the


professor
(D) To prove to the professor that she is
very busy

(A) To clarity her problem to the professor


(8) To apologize for the misunderstanding

3.

~isten

again to part of the conversation. Then


answer the question.

Why does the professor say this:

(A) To assure the student that he will solve


the problem
(8) To explain why he cannot solve her
problem
(C) To recommend that the student find
another exam time
(D) To provide the student with a solution
to her problem

Fill in the blanks to complete the summary.


The conversation is between a student and a
. The stude_nt points out to
the professor that he has made a mistake in scheduling
. The professor
is surprised to learn of his mistake, but assumes that it will not change a lot for his students.
The student does have
, though. The professor agrees to_ _ _ _ _ __
if they can _ _ _ _ _ the exam time.

:t

Service Encounter
Listen to a conversation between a student and a university employee. Fill in the diagram
with the information that you hear. +!f!t!li1!i
" ''""'

lfff;'i(~yvocabularv
~J~.';>:,.

- .

~;;c;.accommodate:

~i;:;abroad:

. .. : - .

overseas .. : .

"-

practically: considering facts; realistically


suffice: to meet the requirements

to provide a place fo( ' .

.-_:

lJ!i::
Institute: an organization with a specific purpose<
()Cb.:'\.
' ':
'
. '
Problem:

1. Listen again to part of the conversation. Then


answer the question.

Why does the woman say this:

(A) To imply that she cannot help the


student
(B) To ask the student to explain the
' problem
(C) To show the student that she is
concerned about his problem
(D) To change the subject of the
conversation

Why does the woman say this:

(C) To convince the student to live


off-campus
(D) To criticize the student for not coming
to the office earlier
3. Listen again to part of the conversation. Then
answer the question.

Why does the woman say this:

, 2. Listen again to parfof the conversation. Then


answer the question.

Solution:

(A) To inform the student of a housing


shortage
(B) To explain why she cannot grant the
student's request

'

(A) To give advice on how to solve the


problem
(B) To explain why she cannot solve the
problem
(C) To say that she will solve the student's
problem
(D) To insist that she is telling the truth to
the student

Fill in the blanks to complete the summary.


The conversation takes place in the _ _ _ _ _ _ _ office. The student has come
there to request permission to
because he will be returning to campus

- - - - - - - The director agrees to


for the student granting
permission for him to
off-campus.
-. ---------- --------------- . .. --- -- .............. . .... ----- ---- .. ------ .... __ __ --------- ---------------- ...------.,

"

"'"

.,

Function Questions

233 - ' '

Astronomy
m'l&ii:;:i~~-!$1,g~~-\iiO.fil..'<J.\>:!N<f:>"i:!-.':""""'-'fl'-IN.'i';:w;;;,;

r:,-'<.;1:. .i',,,,:.~ .,\,

Listen to a lecture in an astronomy class. Fill in the diagram with the information that you

hear. 'i71i"I'
Key Vocabulary
undeirgo: to experience
reiterate: to repeat
luminosity: brightness

collide: to crash into


fusion: the process by which two different parts
are combined

Step 1:

Step 2:

.....

1. Listen again to part of the lecture. Then


answer the guestion.

Why does the professor say this:

;:.

(A) To express disappointment in the


students
(B) To explain why he cb9se the
topic of the lecture
(C) To suggest that the students study
harder
(D) To ask students to contact him with
questions

.-

Step 4:

Step 3:

'

2. Listen again to part of the lecture. Then


. answer the question.

Why does the professor say this:

.....

(B) To refute a common theory about star


formation
(C) To describe the characteristics of a
main sequence star
(D) To explain how stars increase their
energy output
3. Listen again to part of the lecture. Then
answer the question.

Why does the professor say this:


(A)
(B)
(C)
(D)

To ask students to participate


To make sure the concept is clear
To remind students to take notes
Toreturn from a digression

(A) To contrast the energy output of


younger and older stars

Fill in the blanks to complete the summary.


------------h----"'"'------------,-------r----- .. --------~--------------------------------------------------~--The process describes the different
in the life of a star. First, the star
begins life as a protostar when energy is created by
. In the next
stage, fusion in the star's ______ begins, and the star becomes a
_ _ _ _ _ _ _ star. Next, reactions cause the star to get hotter and -_ _ _ __
'

''

:~!i'; Ii
,\l1 J1

Lastly, when the star runs out of energy, it.begins to die.

--------------------------------------..--.. -------------.. . . . . . . . . --------.. --..--..------------.. . . . . . . . . . . .------------.. . --

ffi,~-- 234 Chapter3

. d_:

~ :\

- ..... __-\_

_-,

,_, . -

,~--

1:._,.,-

Biology
~~~>'"&=<l'..tn;.il'i't1<W~li...-~~'-'W;~e;;f""-'1:'S,'.V)?,.~'i'<"';c''C:-"I'<'

..

Listen to a discussion in a biology class. Fill in the diagram with the information that you

hear.
'

15+u~1

Key Vocabulary
wrap up: to finish
order: a classification used in biology
aquatically: of the water

hind: at the back


evident: easy lo see

. OrderCetecea 1<j

I Point 1:
I Point 2:

=================================::.==;
~~int 3~--------.
,.
i'

I
I

1. Listen again to part of the discussion.


: Then answer the question.

"

Why does the professor say this:


(A) To help the students remember the
meaning of the word "cetacean"
(B) To explain why whales are cetaceans
(C) To note the origin of the word "whale"
(D) To clarify a difference between whales
and other anir:nals

. . 2. Listen again to part of the discussion.


Then answer the question.

Why does the student say this:

(I

{A) To show the professor she did the


reading
(B) To refute the professor's point about
dolphins

(C) To ask if cetaceans are similar to land


animals
(D) To give an example of a way to
identify cetaceans
3. Listen again to part of the discussion.
Then answer the question.

Why does the student say this:

(A) To ask the professor to explain in


more detail
(B) To tell the professor that she
understands
(C) To imply that she is not interested
in the subject
(D) To confirm that she already knows
the information

Fill in the blanks to complete the summary.


__________, .... _.,_--------

' ' " " ' ' ' oo-'"'"' "'" , . , , , , , _ , . , . - . - - - _, Mc

" > - - - n M - - . . - - - - - - - - - - - - ' " - OMM- M0000 _ _ _ _ _ _ _ _ _ _ _ _ _ _ _ _ _ _ _ _ _ _ _ _ ,, _ _ _ _ _

The discussion is about the Order Cetecea. The professor gives examples of
_ _ _ _ _ included in the order. Cetaceans evolved from _ _ _ _ _ _ _ about
fifty million years ago. He describes some of the _ _ _ _ _ indicating that cetaceans
evolved from mammals. However, he points out that they have
other mammals. They do not have external _ _ _ _ _ , either.
----------------~

.. ------------

, unlike

------.. --------------------------------------------------------Function Questions

235

--1

I.

Necessary Skills
~a=1~ll!'.J=wRi>=mot1;1;~.c.<1fi~'')""'";;

~:c;,::,'.-''''''''

Understanding the speaker's general feeling about what is discussed


Recognizing words or phrases that indicate the speaker's feeling or opinion
Recognizing tone of voice, intonation, and sentence stress that the speaker uses
to show his or her feeling or opinion

Example Questions

Which of the following best describes the professor's opinion?


What is the professor's opinion of
?
What is the student's attitude toward
?
What is the woman's initial attitude toward the student's request?
What is the student's attitude toward the suggestion of
?
How sure is the man that the woman can
?
How certain is the professor that the student can
?
What can be inferred about the student?
Listen again to part of the _ _ _ _ _ . then answer the question.
You will hear a sentence or a few lines again.
What does the professor mean when he/she says this:
What does the student mean when he/she says this:

Strategies
Pay attention to adjectives and verbs related to feelings. These may help you
recognize words or phrases that indicate the speaker's feeling or opinion.
-> Example: A: The course Chemistry 204 was very helpful.
B: Yeah. I really enjoyed the classes with Professor Jones.
Guess the speaker's attitude by the tone of voice, intonation, and the sentence
stress that the speaker uses to show his or her feeling or opinion.
-> Example: (With surprise) You liked it? (The speaker does not agree.)
(Happily) You liked it! (The speaker is pleased.)
Consider the degree of certainty in what a speaker says.
-> Example: You want to know when it was discovered? Hmm, let me think.
Probably around 1600. (The speaker is not sure of the information.)

Stance Questions

237 - - ,

literature
Listen to a lecture in a literature class. Fill in the diagram with the information that you hear.

IMijjH!i

'

Key Vocabulary
groundbreaking: original; innovative
Implausible: not realistic; not likely to be true
narrator: someone who tells a story

consciousness: awareness of.one's own thoughts and


feelings
censor: to remove or change content considered to be
oflensive or illegal

I' Reafi~~'.andHenryJarTies I
I 1
'

Theory:

8"ppmt

<I

J
Support 2:

L.------------------

1. Which of the following best describes the


professor's opinion?
(A) She does not feel James's work is
given proper credit
(B) She is surprised by James's
capabilities as a writer.
(C) She admires James's character
skills as a writer.
(0) She is critical of James's unusual
characters.
2. What is the professor's attitude toward
unreliable narrators?

(B). She is indifferent about their existence


in James's writing,
(C) She believes they are an effective tool
to create realistic characters.
(0) She feels that modern writers should
use them......mqre often,
3. What is the professor's attitude toward the
stream of consciousness method?
(A)
(B)
(C)
(0)

She
She
She
She

considers it very unusual.


finds it easy to understand.
feels it increases realism.
does not really like it

(A) She does not believe that they are


different from other narrators.

Fill in the blanks to complete the summary.


The professor discusses

and his creation of realistic _ _ _ _ _ ,

James used something called an


narrator to make his characters seem
more like real humans. He also used a method called
, which resembles
how people actually _ _ _ __
----------------------------------------------.---------->----------------------------------------------------

Agriculture
--""'"'"-"""'~=ii',~iln1m""11tmWPi'fl',1J'l;l4f'.'>'';..'.~'-.,,,;~-_;Mwr: -,-.,,: .-

- .. _.,_, .. ,,

Listen to a lecture in an agriculture class. Fill in the diagram with the information that you

>hear. 1 1"''
A.,'.
--,{!};'Key Vocabulary
rt:~~'.,

,t.i,i ,renewable: able to be restored

,_
1

._ .

II

smog: air poliution made of fog and. smoke


'..extract: to pull out or obtain from something

!':~1Hossll fuels: mineral fuels found near the Earth'.s surface.

~;;emission: a substance released into the air


H-'l;.'.

_,

.. ;.:~(Bi.dfr4~)'!,,,~f,;I

-----~I

I._A-d-v-an-t-ag_e_1_:

Advantage 2:

,_.,

o;,oo-rago 1

l~D-is-ad-va-nt-ag-e-2:--~~~~--~~I ~

fuels?

(A) He believes that they have good and


. bad points.
(B) He feels their advantages outweigh
their disadvantages.
(C) He thinks they cause too many
problems to be useful.
(0) He considers them unimportant.

'.

"
;

II
t '

I
'

'

,,: 1. What is the professor's attitude toward bio


'

,_. __ ._,.

2. What is the professor's attitude about the


connection between bio fuels and smog?
(A) He is excited that it can reduce
pollution.
(B) He is skeptical that bio fuels will be
helpful.

(C) He is worried that bio fuels contribute


to the problem.
(0) He is happy that people are using
more bio fuels.
3. What does the professor mean when he

says this:
(A) Any amount of gases contributing .to
global warming is undesirable.
(B) The term "renewable" is overused and
meaningless.
(C) The Earth is generally unharmed by
greenhouse gases.
(0) Global warming is an acceptable and
inevitable part of life.

. Fill in the blanks to complete the summary.


........ _____________ . , _______________ ,______ ,__. _____ ----------------------------N-----------------------------------The professor presents the advantages and disadvantages of bio fuels, which are fuels
that are derived from any sort of
, such as corn. Some of the advantages
include less
and reduced
. On the other hand,
they still contribute to
and use up agricultural _ _ _ _ _ .
........ ____________________ ,, ________ --------------------------------------------N-----------------------------------Stance Questions

239 ._ _,,

..................................................................

,~

Office Hours
r

Listen to' a conversation between a student and a professor. Fill in the diagram with the
information that you hear. i@llm'
Key Vocabulary
go over: to discuss in order to clarify
upcoming: about to happen or take place
shaky: not very good; inaccurate

grasp: understanding
perception: the way something is understood or viewed

. Parts of the Exam

,.'f'

'---------,----1. What is the student's attitude toward the

exam?
(A)
(B)
(C)
(0)

He
He
He
He

is not concerned about it


expects to do well on it.
is worried about it.
thinks it is unfair to students.

3. What does the professor mean when she

2. Which of the following best reflects the


professor's attitude toward the exam?

i.'
,-,.

>1

; ' ! :1

jr-:-I

;;

(0) It will be fairly easy for students who


study the lectures.

(A) It will be the most difficult test of the


semester.
(B) It will require a great deal of additional
reading.
(C) It will not be very difficult for most
students.

says this:
(A) Many students write essays that are of
superior quality.
(B) Many students cannot effectively
reflect upon class material in an essay.
(C) Many students are effective listeners,
but do not understand the topics
discussed in class.
(0) Many students continually restate the
same information in their essays.

Fill in the blanks to complete the summary.


_________________ ,.,., _____ ,.......... _______ ,, ........................ ______ ,, ____ ,, ______ .... ,... --------- ,,. ______ ,, __ ,, ______ ,,_, ..................... ..

The student has


a class where the professor went over material for
______ . He approaches the professor in order to get
of what to
expect on the exam. The professor explains that the exam has a simple _______
section and an in-depth ______ section.
. ...

,.,-- ... - _.- "

~~

"'
~
z

Service Encounter

Cl

Listen to a conversation between a student and a university employee. Fill in the diagram
with the information that you hear. @I""

.. ..,.....

~~{";<~~Vocabulary

.. . . .. .

~~~emester: half (or part) of the academic year


~\"''''"'

..

'- "-"

'

--:

h\\\{transfer: to move from one place to another


[4!,~~rolled: having joined a cl~ss ,. .. .
i'!'>i>'>L'

. ::::'._,_ .....,.. ~ .. '-::--,- ___ . '--

_ ,--- ,: ___

.:.- '-.'i-:r;-i::_,:

VI

surplus: an e~cessive ~mount; more than is needed


title: book
' ". . .

,,-, ___

"''"

Problem:

>

'5
z
Cl

::e

Solution 1:

"'3z

Cl

~<
Solution 2:

1. What is the bookstore employee's initial

;\'

f
.

'

..
~

;:;

...m

!!l

3. How sure is the bookstore employee that


the student can find the book for sale
on line?

altitude toward the student's request?


(A) Annoyance
(B) Interest
(C) Disappointment
(D) Surprise

(A)
(B)
(C)
(D)

He is not very sure.


He doubts it seriously.
He is somewhat sure.
He is quite confident.

2. What is the student's attitude toward the


suggestion that she use the library?
(A) She dislikes.the employee's suggestion.
(B) She is unsure of what the employee
means.
(C) She does not think the library has the
.book.
(D) She thinks the suggestion will solve
the problem.
Fill in the blanks to complete the summary.
............ ------ ....... --- -.. -- ---- -- --- .... -.. - -. .- .... --- ---- .... -- -- .. - --- -- -- --- ----. .. - --- ........ --- ........ -- -- ---- --- ---- .. ---------~

.,

-~

The student attempts to find - - - - - - - at the university bookstore. An


employee tells her that textbooks are no longer
this late into the semester.
The student then asks him to recommend some
that would carry the book
and is advised to visit
. Another option for the student is to make a
purchase ______
. ________________ " _______________ __________ _______ .... -----------------------------------------------------------,.

.,

Stance Questions

241 - -

--

Environmental Science
Listen to a lecture in an environmental science class. Fill in the diagram with the information
that you hear. +511n1
Key Vocabulary
Invasive: entering or spreading in a place that belongs to
someone or something else
native: belonging to a place or environment

competitor: something that challenges others to su"!ive


pen: a place where animals are farmed or raised
ecosystem: a community of organisms that live together

.<lnv0siveSpecie .,

Problem (Cause):

1. What is the professor's general attitude


toward invasive species?
(A) He believes they have had a negative
effect on fish farms.
(B) He is concerned that people are
ignoring their harmful effects.
(C) He believes that they always have a
harmful effect on ecosystems.
(D) He is hopeful that invasive species are
not as harmful as some believe.

2. What does the professor think of the


invasive salmon?
(A) He believes that the salmon should not
be considered an invasive species.

Effect ,

(B) He believes that the cause of th.e


salmon invasion is likely incorrect.
(C) He recognizes the potential dangers
that the salmon present.
(D) He does not have a specific opinion
about the theory.
3. What does the professor feel is the
greatest danger from invasive salmon?

(A)
(B)
(C)
(D)

Some types of salmon will die out.


Many fish will not have enough food.
Invasive salmon will eat local ones.
Farmed fish are less healthy than
others.

Fill In the blanks to complete the summary.


The professor discusses invasive species, which he defines as species of animals that
______ environments they are not ______ to. The professor uses
________ as an example of the effects that an invasive species can cause. First,
these farmed fish introduce
that were not found in the ecosystem before
they came. Second, the farmed fish increased competition for
and other
resources.
242

Chapter 4

History
~ru~.n:~=w.:.<ri'l.X>~V--=.l,,,."'<l''ilN?-'1>'-'irn_,J>:il<,,~.~t1';!"<-~-'>-,:., - 0

. - - . , --

Listen to a lecture in a history class. f,ill in the diagram with the information that you hear.
mw
.
.
:; Key Vocabulary
' . touch: to affect emotionally
baokground: important information .
evoke: to cause or inspire

"'

appeal: a request for something, often sympathy or


support
split: a separation or division

N'

iE;

"O

zGI

j The Gettysburg Address I

ISupport,

Theory:

~--t'"'

"O

~
n

;;l
.

..."'

'

l~S-u_p_p-or-t-3:~~~~~~~---------__J

?, 1. What is the professor's general attitude


toward the Gettysburg address?
'!
(A) She believes that it was highly effective.
(B} She feels that it should have been
longer.
(C} She regrets that so few people heard it.
(D) She believes it made Lincoln popular.

2. What can be inferred about the professor


when she says this:

(A) She thinks that people did not have


the ability to learn new information at
that time.

(B) She supposes that the country's old


ideas represented Lincoln's political style.
(C) She feels that new ideas or concepts
may have confused the people.
(D) She believes people felt deeply about
things that were already in their minds.
3. What is the professor's opinion of Lincoln's
use of the word "nation?"
(A) It was overused in the speech.
(B) It was used in a new and unusual way.
(C) It reminded people of what they
already knew.
(D) It was not necessary in the speech.

Fill in the blanks to complete the summary.


The professor discusses why the Gettysburg Address, a speech by _ _ _ _ _ __
is so memorable. She says that it is largely because it appealed to _ _ _ _ _ __
Lincoln reminded people that they fought to secure
. He also said that the
country would one day be _ _ _ _ _. Finally, he did not bring in any new

..

--~--------- ------------~----

...... -.------- .. --------- ...

-------------------- . --- ..

--------------~--

..-------- .. --Stance Questions

243 - -

Office Hours
Listen to a conversation between a student and a professor. Fill' in the diagram with the
information that you hear. lillfljiti

Key Vocabulary
dean: the head of.a school .
tend: to be likely to do something
leave: permission to be gone .

.recuperate: to recover from illness ,


.draw near: to approach

Problem:

Solution 1:

.--------,
<
Solution 2:

1. What is the student's attitude toward his


situation?
(A) He is annoyed with the professor's
policies.
(8) He is worried that he will miss the rest
of the semester.
(C) He is happy that he can take a break
from class.
(D) He is hopeful that he can come back to
class soon.

2. What does the professor think of the


possible arrangements?
(A) She is annoyed that she has to make
special arrangements.

. (8) She is unwilling to discuss changing


the student's grades.
(C) She is prepared to compromise with the
student.
(D) S.he is happy to do anything that the
student wants.

3. How certain is the teacher that the


student can complete the course this
semester?
(A) Somewhat unsure
(8) Very sure
(C) Rather sure
(D) Very doubtful

Fill in the blanks to complete the summary.


The student has been ill and has missed
, so he is worried that his
grades will suffer. The professor suggests
, but that will mean the
student will lose his
. The professor decides to give him an "O.H.," which
means "
" at the end of the semester. This will allow him to finish his
coursework at some time - - - - - - ----------- .. -------------------------- .. ---- .. -------------------- .. --.~---- .. --------------------------------~
f~i~~..__- 244.
'.!'!;'

Chapter 4
-

,-q
' ._' Ji:_.:__ __ ~-'----- ~------'-----'-'---_.- -

Service Encounter
Listen to a conversation between a student and a university employee. Fill in the diagram
with the information that you hear.

"ttli"'

Problem:

Solution 1:

.-----<
Solution 2:

1. What
does the student think about the
f
hdDsing lottery process?
(Al,; He would like to participate in it.
(B) He does not like it.
(C) He is worried that he does not
understand it.
(D) He believes that it does not work.

3. What is the student's attitude as he is


leaving?
(A) He is relieved the process is not
difficult.
(B) He is excited to fill out the forms.
(C) He is confused about what to do next.
(D) He is happy about the change in .
housing policy.

2, What is the woman's opinion of off-campus


housing?
(A) She is reluctant to suggest it.
(B) She thinks it is unfair to students.
(C) She believes it is an easy process.
(D) She recommends it to all students.

Fill in the blanks to complete the summary._


--------:..-------.,-------<---------------------,.----"'"'--------------------------------------------The student plans to
and asks about his options. He states that he
would like to live
. The employee explains how the process works and
provides
for the student to
. The student decides to
_ _ _ _ _ for off-campus housing, but he will return the form later.

Stance Questions

245 _ __:.,

Listen to a lecture in a biology class. Fill in the diagram with the information that you hear.

'''''

Key Vocabulary
vocalization: a sound made using the voice
olfactory: relating to the sense of smell
primal: existing from the beginning

glandular: having to do with glands


territorial: tending to defend specific areas br things

Scent Communication In Animals

Example 2:

'-------c----

11

1. What is the professor's attitude toward


the students' knowledge of animal
communication?
(A) She thinks that they would rather talk
about vocalization.
(8) She feels they do not know about
smell communication.
(C) She hopes they are familiar with the
types of communication.
(D) She considers their knowledge greater
than that of other classes.
2. What does the professor mean when she
says this:

(A) She enjoys the feeling of a cat rubbing


against her.

")j

(8) She is sorry for being exhausted and


irritable today.
(C) She wants to be very honest about her
feelings with her students.
(D) She is not fond of cats and their
marking habits.

3. What is the professor's attitude toward the


communication system used by ants?
(A) She is relieved that humans cannot
detect their smells.
(B) She thinks her students should study
it in detail.
(C) She finds it to be a common
communication system.
(D) She considers it a theory that has
little support.

Fill in the blanks to complete the summary.


--------------------------oO~-O

.. <-~~O<- .. -~-"'--'"---------------- .. -,.----------------n---<>------N-

The professor talks about how animals can - - - - - - - u s i n g smells. The


professor first uses the example of
and states that these animals will rub
against things in order to show affection or to ______ territory. The professor also
talks about
,.

ti.:

~ .

and says that they release

to transmit

different messages.

------"----------"----... --............................................... ------"'"---- ................................................... ______ ,, ______,............ ..

~46 Ch~pter

If

Environmental Science
Listen to a discussion in an environmental science class. Fill in the diagram with the
information that you hear. @SUJ
Key Vocabulary
hydrological: relating to water and its movement
dump: to unload or drop
head for: to leave in order to go somewhere else

hydrothermal: having to do wilh hot water


Intriguing: causing interest

'Whyocearis are Salt)

Cause A:

Cause B:

1.

1.

2.

2.

3.

3.

~fleet:
1. ,What can be inferred about the professor
:
when he says this:
(A) He wants to share some particularly
interesting information with the student.
(B) He is unsure of how to explain
hydrothermal vents to the student.
(C) He does not know if the student has
prior knoyvledge of hydrothermal vents.
(D) He thinks the student should already
know the information.

1
'

I
I

I
I

I
I.

2. What does the professor think of


hydrothermal vents and ocean salinity?

(C) The connection between the two is


obvious.
(D) The subject would be interesting to
study more.

3. Which of the following best describes the


professor's opinion of the fact that the
ocean does not get saltier over time?
(A) it is obvious.
(B) It is beneficial.
(C) It is complicated.
(D) It is surprising.

(A) The relationship is very interesting.


(B) The concept is complex.

Fill in the blanks to complete the summary.


The professor explains why the oceans are
. The first reason is related to
the
process. Rain falls on land and dissolves _ _ _ _ _ __
which accumulate in rivers and eventually flow into the ocean. Hydrothermal vents are another
reason why the oceans are salty. Very hot water on the ocean floor _ _ _ _ _ mineral
salts found there. However, the professor notes that the oceans are not becoming
_ _ _ _ _ over time.
Stance Questions

247 --"'

!fl fil.~.! ! l! l! ! ! !l!Jl ! !l l!l l l !l!l !l l l !l!l !l l l !l!l l fl!l l ll!lllllll!i!~.......................- -.....- - - - - - - -........
........

.I

II.'

~~lll>'!Ul1-\~~Afs;J!~~l!9'.l'lil~~IJl}Y~l>l;~;.,,~~~

:~

Instructions: Choose the best word or


phrase to complete each sentence.

1. After the storm, the farmer discovered


that one side of the sheep's
had fallen down.
(A) flaw
(B) leave
(C) pen
(D) ruse

' ' ..
,.,;;
;'J'I

2. The evidence presented in the trial was


, so it was not
rather
surprising that the jury returned a verdict
of not guilty.
(A) gaseous
(B) invasive
(C) renewable
(D) shaky

3. Although many breeds of dogs are


, African hunting dogs are
not, which explains why they don't mark
areas.
(A) hydrological
(B) hydrothermal
(C) primal
(D) territorial

,':.i

. "

4. The
of sweat from the skin
is slower on humid days than on days
when the air is drier.

(A)
(B)
(C)
(D)

evaporation
perception
specialization
vocalization

, more
5. As the exam date
and more students began attending
weekly review sessions.
(A) drew near
(B) filled out
(C) went over
(D) headed for

6. The solution to the problem was not


, so very few members of
the class were able to figure it out.
(A) background
(B) groundbreaking
(C) lighthearted
(D) straightforward
7. I had my doubts at first, but the online
has actually
writing
helped me write better essays.
(A) competitor
(B) narrator
(C) proctor
(D) tutor
8. The thoughtful gift really
the parents of the new baby.
(A) extracted
(B) grasped
(C) split
(D) touched
Instructions: Choose the word or phrase
closest in meaning to the underlined part.

9. At this time, the website that you are


attempting to access is not available.
(A)
(B)
(C)
(D)

~\"/,____

248

Vocabulary Review

Consequently
Currently
Practically
Rigorously

..

,4:
l . ,.
j ,

I"
I

I
I

I
1

10. In the speaking ruill ot'the test, students


must answer five questions asked by the
instructor.
(A)
(B)
(C)
(D)

commitment
competence
component
consciousness

11 .. The moon does not always look the same.


On some nights, its brightness can be
greater or less than what you see tonight.
(A) efficacy
(B) familiarity
(C) luminosity
(D) certainty

12. It was clear from the student's question


that she did not read the homework
assignment.
~,

I
I

I
I
I

(A) clever
" (B) crucial
'. (C) 13vident
{D) implausible

13. More people died in the New York riots


that lIBggn during 1863 than in any other
riot.
(A) came up
(B) broke out
(C) took off
(D) objected to
14. The wife's brave rescue of her husband at
the end of the movie was very exciting to
watch.
(A) daring
(B) fulfilling
(C) intriguing
(D) upcoming

15. When you sign up for a conversation


partner, the teacher will put you in contact
with a native English speaking student.
(A) give you up
(B) hook you up
(C) show up
(D) wrap you up
Instructions: Write the missing words. Use
the words below to fill in the blanks.
administered
descerided'trom
inevitably maintenance territory
Although the national park at Chaco Canyon
in New Mexico is 16. _ _ _ _ __
by the US government, the entire
17.
of the canyon
belongs to the Native American tribe
18.
the original people of
that area. The forces of nature at work in that
desert-like region 19. _ _ _ _ __
damage or bury the historic ruins found in the
canyon. It is the government's job to oversee
20.
and protection
projects related to the ruins.

- -r--

~--

.- ;; .Y --

~
n

"'

Instructions: Circle the one word which does


not belong.

21. {A)
{C)
22. {A)
(C)
23. (A)
(C)
24. (A)
(C)
25. {A)
(C)

accommodate
exploit
advocate
collide
dean
semester
emission
smog
expanse
lottery

(B)
(D)
(B)
(D)
(B)
(D)
(B)
(D)
(B)
(D)

consensus
grant
appeal
reiterate
institute
surplus
fossil fuel
title
fortune
receipt

Vocabulary Review 2

. ---

"O

249 __.,

01 Astronomy
Listen to a lecture in an astronomy class.

iiffil"i

Key Vocabulary
makeup: the things inside; composition
core: center
elements: the most basic chemical substances
solid: hard or touchable

molten: melted
rotate: to spin; to turn on an axis
band: a stripe

1. Listen again to part of the lecture. Then


answer the question.

What does the professor mean when he


says this:

(A) To remind the students of the material


discussed previously
(B) To clarify students' confusion about
the names of the planets
(C) To explain the difference between the
inner planets and the Jovian planets
(0) To make sure the students are ready
to move on to new content

2. Listen again to part of the lecture. Then


answer the question.

Why does the professor say this:

(A) He wants them to relate the new


information to older material.
(B) He knows the students do not know
all of the elements.
(C) He thinks the students will make an
incorrect association.
(0) He wants the students to read more
about helium and hydrogen.
3. Listen again to part of the lecture. Then
answer the question.

Why does the professor say this:

(J

(A) To question an idea about planet


formation
(B) To imply that the textbook is correct
(C) To show support for a theory
(0) To refute a point made earlier
750

Mini Test 2

4. What is the professor's attitude toward


the term "gas giant?"

(A)
(B)
(C)
(0)

He thinks it is inaccurate.
He finds it appropriate.
He questions its usefulness.
He prefers the term "Jovian planet."

5. What is the professor's opinion of the


textbook's reference to a "rocky core?"

(A) The. term is misleading.


(B) The term is somewhat outdated.
(C) The term does not relate specifically
to Jupiter.
(0) The term he uses is more appropriate.

6. What does the professor think about the


Jovian planets?
(A) They are the most likely places in our
solar system to have other forms of
life.
(B) They are impossible to explore, so we
should focus on the inner planets.
(C) They could be important sources of
fuel for space travel.
(0) They are more beautiful than the other
planets.

~'

"-.-.,,._,

02 Philosophy

f"\(
'f._>r,

Listen to a discussion in a philosophy class. @llfl


Key Vocabulary

I
[

l
t

I
I
I

1O. Listen again to part of the discussion. Then

answer the question.

(I

(A) He is about to share his ideas.


(B) He wants to try answering the
question.
(C) He is going to fulfill the professor's
request.
,, (0) He knows the others will not agree
with him.

7. Listen again to part of the discussion. Then


Why does the student say this:

lntrlnslcafly: by nature; essentially


prescriptive: giving direction: setting rules
morality: concerned with the difference between good
and bad: good conduct

answer the question.

I
I

excerpt: a short piece taken out of a larger work


disruptive: able to cause problems or break up order
ensure: to guarantee
. tarnish: to damage a reputation or good name

t.
:~~

8., Why does the student say this: (I


(A) He thinks the social contract theory
is too negative.
(B) He understands Hobbes's theory
through a popular expression.
(C) He finds the thegry is different from
modern ideas.
(0) He believes the theory does not help
maintain peace.
9. Listen again to part of the discussion. Then
answer the question.

Why does the professor say this:

(I

(A) She is disappointed in the student's


answer.
(B) She finds the comment too simplistic.
(C) She appreciates the student's
response.
(0) She wants the students to give more
detailed answers.

Why does the professor say this:

(I

(A) She wants to clarify the student's


misunderstanding.
(8) She thinks the student has found a
legitimate problem.
(C) She finds the student's logic to be
poor.
(0) She thinks the student did not
prepare for the discussion.

11. At the end of the discussion, what is the


student's opinion of Hobbes's idea?
(A) He thinks it does not say anything
about morality.
(B) He wants it to treat the matter of
government more.
(C) He thinks Hobbes accurately
describes human nature.
(0) He is impressed by the depth of
Hobbes's thought.
12. What is the professor's attitude toward the
idea of altruism?
(A) She thinks that Hobbes ignores it.
(B) She wants to spend more time
discussing it.
(C) She thinks the topic is uninteresting
to the students.
(0) She believes it is unrelated to the
discussion.

Mini Test 2

251 - -

!!!l!!!!!l!!!!!!!!l!!!l!!!l!!!!!!!!!!!!!!!!!!!!!l!ll!llll
........._..................................................11111111111111111111................J-oo'-

' 03 Service Encounter


Listen to a conversation between a student and a university employee. 'I"''
Key Vocabulary
swipe: to quickly put through, as in putting a card ttirough
a reading machine
misread: to not analyze or record correctly

13. Listen again to part of the conversation. Then


answer the question.

Why does the man say this:

(A) He is impatient with the student's


questions.
(B) He has limited methods of helping the
student
(C) He cannot contact Dining Services
while at work.
(D) He is worried that the student is not
eating.

252

Mini Test 2

Why does the man say this:

14. Listen again to part of the conversation. Then


answer the question.
Why does the student say this:
(A) She does not enjoy the cafeteria food.
(B) She does not eat at the cafeteria often.
(C) She likes to eat at the cafeteria.
(D) She can only afford five meals on
campus.
15. Why does the man say this:

16. Listen again to part of the conversation. Then


answer the question.

(A) People tend to pay less attention on


Mondays.
(B) The cafeteria is busier on Mondays.
(C) He usually does not work on
Mondays.
(D) The student only eats at the cafeteria
on Mondays.

override: to bypass or counteract


option: a choice
.
.> '
.. . .
reimburse: to pay back; .to coinp'eirsate .

(A) He is explaining why he cannot offer


any other options.
(B) He wants to understand why the
student would lie to him.
(C) He is upset that others try to cheat
the system.
(D) He thinks the student wants him to
give her a free meal.
17. What is the student's opinion about filling
out the form?

(A) She thinks she might not get her


money back.
(B) She feels it is a waste of time.
(C) She would prefer to speak to
someone at Dining Services instead.
(D) She is confident thatthe issue will be
worked out.
18. Listen again to part of the conversation. Then
answer the question.

What does the man think about Dining


Services?
(A) Their customer service is poor.
(B) Their compl9ints are usually settled
quickly.
(C) They are competent in resolving
issues.
(D) They make too many mistakes.

Necessary Skills
Jm?,mt1.:\1!.~~~"ffl".l:'>"'m'<~1<."'1':P.''.'l!t::<!<n:i;-=c;<:P~l'<.~~ii~'<~:cF:.'"'''-"

'-('''- ';: -

Recognizing the organization of information in a lecture or a conversation


Recognizing the sequence of information
ldentifying'the main steps of a process
Summarizing a process with its main steps

!Example Questions

Why does the professor mention


?
Why does the professor tell the students about _ _ _ _ _?
Why does the professor discuss
?
Why does the professor make a distinction between _ _ _ __
and
?
How is the discussion organized?
How does the professor organize the information about ______?
How does the professor support the idea that _ _ _ _ _?
How does the professor clarify her point about
?

traiteg l::s
.-,i;;r'.f'.->'f.'i::;.<f_'. '',ti;'.. . L;;-

Use diagrams, arrows, and outline format while taking notes to indicate the
organization and relative importance of information.
Listen for transitions that indicate sequencing of information:
- first, now the first step is
- next, (and) then
- so now
- the last step is, finally

t\

;-1r_

'

sd

Organization Questions

253 - - .

Linguistics
---~~"."'-l-l:l'OJ~"C=rt.Wl:="J,V.t.'!.~."i,"'l;~.,-

-"R.'.

.-.~-':

Listen to a lecture in a linguistics class. Fill in the diagram with the information that you

hear. @i'F'

'

Key Vocabulary
linguist: someone who studies languages
prefix: a group of letters that are added to the beginning of a
word to change its meaning

IClassification 1:

I Classification 2:
[Classification 3:

suffix: a group of letters that are added to the end


of a word to change its meaning
enrich: to make something better
Imitate: to copy or try to be like

IExplanation:
Explanation:

-~

Explanation:

1. How is the discussion organized?


(A) The professor describes the process
by which we create words.
(B) The professor compares different
ways to create words.
(C) The professor classifies the ways
words are added to languages.
(0) The professor gives an example of the
use of loanwords.
2. Why does the professor mention the
subject of the previous lecture?
(A) To remind students of the importance
of the last lecture
(B) To review the same topic in this lecture

J
J

(C) To connect that subject with today's


lecture
(0) To refute a key point
3. Why does the professor mention the word
"skyscraper"?
(A) To demonstrate how derivations are
created
(B) To give an example of a compound
word
(C) To illustrate how many new words
were created
(0) To show why some words cannot be
translated

Fill in the blanks to complete the summary.


-----------------------------------------------------u--------u-------------------------------------~-

The professor describes ways that new words are ______ to a language. One
common way of creating new words is through
. These are words that
are directly taken from another language without
. Another way that
new words are added is through
or derivation. A third way that words
are created is when speakers try to ________

---------..----------------.,-------------------------..
, - - 254 Chapter 5

-------------.,.---~---"-----------------------------

Biology

,i~_~_lfW -

._'!U/.l~l.ll:f~W".W.""--i<-~~~1?>:-:nwi;;;.-~H,,;;:c,,,;,";~"-'>,..,,_,,,_c

<:''"'''"' -

Listen to a lecture in a biology class. Fill in the diagram with the information that you hear.

'Mf!lb'
Key Vocabulary

1~}1,;

degradation: the process by which something is destroyed


generate: to 9reate
entail: to involve something

Imperative: extremely important


take place: to happen or occur

,.I(
~,;;

., f,!'"1

' r-:..~~-.
,,

Step 1:

-,;,

Step 2:

: '1.0;.'<.."-

- .'.r..

},

1. How does the professor organize the


information about the production of new
blood?
(A) She explains why the cells are
produced in red bone marrow.
(B) She describes what the process
entails and where it takes place.
(C) She defines blood and explains how it
works in the human body ..
.(D) She gives an example of how long it
takes for new cells to be produced.
2. Why does the professor make a distinction
between red bone marrow and regular
bone marrow?
(A) To emphasize the importance of both
in hematopoiesis

(B) To make a connection between


red bone marrow and blood
(C) To clarify which type is responsible
for the production of new blood
(D) To compare the functions of each in
the degradation of blood cells
3. Why does the professor mention liver
cells?
(A) To include the liver's role in the
degradation of blood
(B) To give an example of the factors that
help increase blood production
(C) To emphasize the spleen's role
in degrading blood
(D) To suggest a different theory about
how the body gets rid of old blood

Fill in the blanks to complete the summary.

The lecture _ _ _ _ _ _ _ a previous talk about the quality and function of blood.
Hematopoiesis is the process of
red blood cells in the human body. New
red blood cells are created
, which is the soft tissue found inside
_ _ _ _ _ . Blood cells usually degrade in the _ _ _ _ _ or the liver.

Organization Questions

255 - - .

Ljsten to a lecture in a zoology class. Fill in the diagram with the information that you hear.

+dJDifJI
Key Vocabulary
pertinent: related to the topic being discussed .
anthrozoology: the study of the relationship between
humans and animals

ii

decree: to give an order . . ... >


. correlate: to have a similar relationship
dlve:ge: to split up and go'ln different dire.1otion1s

Hunting Dogs:

Both:

Falcons:

:i
l
,1

:I
:1.

!!

::i

,i

'.I

1. How does the professor organize the lecture?


(A) He discusses the process of training
falcons.
(B) He defines falconry and explains its
history.
(C) He compares falconry to another
animal/human relationship.
(D) He offers a theory of animal bonding
and refutes it.

i-,

2. Why does the professor mention the


relationship between dogs and people?
(A) To contrast that relationship with another
(B) To compare the abilities of dogs and
falcons
(C) To compare the popularity of each
hunting animal

l~

3. How does the professor support the idea


that falcons do not form bonds with
humans?
(A) By mentioning that falcons hunt only
for food
(B) By explaining that falcons see humans
as dangerous
(C) By describing the bonds made
between falcons
(D) By comparing the hunting styles of
dogs and falcons

Fill in the blanks to complete the summary.

-------------------------------------------------------------- .. ----------------- .. --------


The professor descri_bes the sport of falconry. He compares the _ _ _ __
between falcons and
to that of dogs and - - - - - Both relationships
involve hunting, but falcons only hunt because they
, while dogs will
hunt in order to ________
--------------------------------------------------------------- .. ---------------------------- ..--------

" ' - - - 256 Chapter 5

ljj

. (D) To contrast the usefulness of the two


animals

Psychology

i!.>i-~Wt;;,~~m:;.-m~l'ih'<!Z.n'.<'!m;'ln">.W<Wl<fl.'ffi1'"l;;'-9.'.>')~\0l,o'.;'.',,:~ .'i'7' -: - "',

Listen to a discussion in a psychology class. Fill in the diagram with the information that you

hear. @I'll
Key Vocabulary
excessive: more than _neces_s13ry
random: not following a pattern
Intensity: strength or degree

Ill
"O

. undemanding: easy
cloud: to confuse

Cl

Experiment:

Results:

1. 1How does the professor organize the


lecture?
(A) She describes an experiment to
support a theory.
(B) She describes a theory and supports
it with examples.
(C) She defines multiple aspects of a theory.
(D) She compares two theories to create
a new theory.

2. How does the professor clarify her point


about electric shock?
(A) By explaining that it produced bad
results
(B) By offering a definition of electric shock

(C) By comparing it to a recognizable


feeling
(D) By giving examples of different shock
levels
3. Why does the professor mention taking a
test?
(A) To support the theory that stress can
hinder perfbrr'nance

(B) To show that the theory also applies


to humans
(C) To show that motivation cannot help
performance
(D) To introduce another experiment that
supports the theory

Fill in the blanks to complete the summary.


The professor discusses an experiment that
the theory that too much
motivation can hurt one's performance. The experiment involved
that were
given different levels of
when they made a mistake trying to choose a door.
The study found that when the mice did
tasks, their performance was
harmed by _ _ _ _ _ __

----....-------............

Organization Questions

..................

~
... ~
....~

..........................

~.. ~
...~~
...

257 -

~.

Economics
Listen to a discussion in an economics class. Fill in the diagram with the information that you
hear. +t@IEJ+
l<ey Vocabulary
spark:tocause
,
.
,., ..,.
trite: overused and uninteresting
maxim: a statement that expresses a general truth

-. erroneous: inco_rre_t; _wrong. _,~:; :_,-: :-- _


. _ _,. ,
naive: exlreniely simple; notshrewd
or
sophisticated
....
,_
-_.
.
'

i :fl~Ws Of/M.i,xMrfi[1;

~l"g-ume-nt1:~~~1 l~Sup-port-:~~-]~
IA'gcmoct2._:-

1. How does the professor organize the


information in the lecture?
(A) He defines Marxism and gives
examples of how it applies.
(B) He offers two specific reasons for the
failure of Marxism.
(C) He describes two different effects of
Marxism's failure.
(D) He classifies the different aspects of
Marxism.

2. Why does the professor mention


economists?
(A) To refute his theory that Marxism
was before its time
(B) To support the idea that Marxism is
widely rejected

l,____Sup-port:--~
(C) To disprove a popular theory and
propose another
(D) To support a theory with real-world
evidence

3. Why does the student mention the value of


management?
(A) To give an example of why Marxism is
flawed
(B) To support the professor's theory of
class struggle
(C) To agree that Marxism was correct on
some points
(D) To refute the main idea presented by
the professor

Fill in the blanks to complete the summary.


The discussion is about why _ _ _ _ _ failed. The first point was that this economic
theory did not include other factors that divide a society, like _ _ _ _ _ . The professor
continues by saying that another key premise of the theory was completely wrong. Marx said
that the value of a company equals
plus the value of
to
make the products. Marx failed to account for the added value of _ _ _ _ _ __
------------------------- ----------------- n----------""'"""'""'"'--------------~---------------------------
2~8 ChaPter 5

"'c;

Art History

C\

~~:i!f!W";\':f~'l'.W."''l;P.;;;:;,,_.w.~\\"~1\;'f~QU.i.s!~,;,m<;'<'.f'H~'i:".'.<,,"lei'l'.'<>Y--1'

I
I
I

,-,: < .,;,, ,--.

' history class. Fill in the diagram with the information that you
Listen to a lecture in an ~rt

hear. @Ml'
Key Vocabulary

distinction: a difference between things that seem similar ,


prescribe: to set down as a rule

depart: to move away from

. trivial: having no impact or Importance


stress: to place special attention on something

I
I
I
I

I
I
I

I
I
I
I
I

Impressionism:

Both:

Post-Impressionism:

1. HOV\! does the professor organize the


.'.information presented in the lecture?
'(A) She highlights the similarities between
the two movements.
'(B) She compares the two movements
focusing on their dissimilarities.
(C) She describes the history of the
movements to show their differences.
(0) She defines each movement and
describes their unique characteristics.
2. Why does the professor discuss the
Academy of Fine Arts?
(A) To compare Impressionism to art
that came before it
(B) To compare Impressionism and
Post-Impressionism

(C) To compare two specific techniques


of landscape painting
(0) To compare two definitions of
Impressionism
3. Why does the professor mention artistic
methods?
(A) To emphasize the similarities of the
movements
(B) Tb give an example to support her
comparison
(C) To contradict her previous comparison
(0) To compare the composition of two
paintings

Fill in the blanks to complete the summary.


The professor clarifies the differences between Impressionism and Post-Impressionism.
The professor first says that
had bright color and thick applications of
paint However, the Impressionists painted
and focused on fast
brushstrokes and
, while the Post-Impressionists painted _ _ _ __
and focused on more careful brushstrokes and more - - - - -----------.. ---.,-------------------..
..
-----------~---------------------------

--.--~-------------------

Organization Questions

259 - - '

'
Listen to a discussion in a business class. Fill in the d,iagram
with the information that you

hear. +iffii'fl
Key Vocabulary
spokesperson: a person who is chosen to represent a
group or organization
authoritative: seeming to be complete and accurate

;;owh.er;spoi<e~peopie:F, I

1...-A-'-d-va-n-ta_g_e_1:_ _ _ _ _ _ _ ____,]

l~A-d-v-an-ta_g_e_2._:

monotone: speaking in only.one tone


egotlstlcal: being overly self-important
alienate: to make others hostile to oneself

. I

o;,,.d~tego

DlSadvantage 2:

_ _ _ _ _ _ _ _].

1. How does the professor organize the


lecture on owner spokespeople?
(A) By focusing on their advantages and
disadvantages
(B) By defining a spokesperson and
describing the benefits of using one
(C) By describing an advertising theory
and its originator
(D) By introducing a theory and
highlighting its drawbacks

2. Why does the professor mention likeability?


(A) To disprove a previously stated
advantage
(B) To introduce a new aspect of the
disadvantages

------------------~

(C) To add to his description of


disadvantages
(D) To provide an example of a
disadvantage
3. Why does the professor mention actors?

(A) To expand upon the professor's


example
(B) To show the benefits of an alternative
spokesperson
(C) To contrast the disadvantages of two
elements
(D) To help define an important term

Fill in the blanks to complete the summary.


The talk is aboutthe advantages and disadvantages of an owner spokesperson in
______ . The professor gives two pros and two cons. As for advantages, the
professor says that it helps to _ _ _ _ _ _ _ , and it can make people

i~: :

\;f'.f;

-'------the message. However, some people are not good spokespeople, either
because they are
in front of the camera or their personality sends
about the company.
-----------------------------..----------------------------------------------------------....................,,, __________ _

i11j

~1 1--dl

'\j

260

Chapter 5

Zoology
.&'ll);~&mun>.io:&;:t1;~Jt)<.""1>.\v.t:.!.''-~'.-.~_,_,M>-o~;-

;--:'""-'' ' .

Listen to a discussion in a zoology class. Fill in the diagram with the information that you
hear. @jtD
Key Vocabulary
Involved: complicated and hard to understand
concrete: completely certain
anticipate: to expect

Experiment:

s.ubstantlatlon: the provision of proof or support of.


something
deduce: to reach an answer by reasoning

Result:

Conclusion:

[l
[

---------~

1. Ho'A' does the professor organize the


disc;,ussion?
(A) She offers a definition of a particular
\heory about memory.
(B) She compares animal memories
using the presented theory.
(C) She makes a claim about memory
and supports it with a study.
(D) She describes a theory and explains
the importance of its processes.
2. Why does the professor mention the
relationship between memory and animal
size?

(C) To suggest a contradictory theory


(D) To define her theory for the students
3. Why does the professor discuss the study
of a female wasp?
(A) To criticize the theory that small
animals have better memories
(B) To illustrate the theory that animals
have only short-term memories
(C) To refute the theory that animals do
not possess memory
(D) To support the theory that animals
have good memories

(A) To refute a claim made by researchers


(B) To introduce the conclusion of the study

Fill in the blanks to complete the summary.


The discussion revolves around the professor's statement that animals have more
developed memories than
. She supports this idea with the example of

a
. Researchers used a circle o f - - - - - - - to test the memory of
the insect. The insect remembered the location of
in relation to the
pine cones. The experiment indicates that even small insects have some kind of

----------------------..------ .. ----~--------. ------------------------------------------------ .. ---------

Orgarnzotion Questions

';I:

'_.,.-.

261 _ _. ,.

Psychology
>lf.l.u;;T.tl"V.U>\!Oillli~~mt~v=''(;'.'_'.:'HeJ~;,v;,;t_;:.:"-.~~:.

c--<'-!

;. ,;;

. _,,,

Listen to a lecture in a psychology class. Fill in the diagram with the information that you
hear. l@itji
Key Vocabulary

.mutually: having th~ sa~e relationshipto each


exclu~lve: limited to only cine. > .

nurture: to help develop


Intrinsic: being a natural part
blank slate: a clean, empty board for writing on

Nature vs: Nurtdre ..f

Nature:

Both:

Nurture:

1. How does the professor organize the


information about the nature versus nurture
debate?
(A) She compares the aspects of two
theories on human personality.
(B) She explains the basics and provides
support for each side.
(C) She shows that each side has its
scientific limitations.
(D) She defines two theories on human
personality and refutes them.

2. How does the professor clarify the points


she makes about the nature theory?
(A) She presents a typical experiment
used to prove that traits can be innate.
(B) She talks about conditioning and
suggests that traits are learned.

(C) She explains the concept of tabula


rasa and the idea of humans as a
"blank slate."
(D) She rejects the possibility that
personality and intelligence are
inherited.
3. Why does the professor discuss B.F.

Skinner?
(A) To give students insight about
genetics and personality theory
(B) To give an example of a psychological
method based entirely on nurture theory
(C) To counter the idea that personality
could arise as a result of conditioning
(D) To explain why conditioning can
reverse innate personality traits

Fill in the blanks to complete the summary.


------.. ------------..- .. - ...A-----------------------~-------------<0---.----------------------------~~-

The professor explains the nature versus nurture debate. She clarifies what the debate is
about and discusses
that suggest personality traits are---"'--passed through the genes. Next, she talks about tabula rasa, the idea that humans are born
as
. She mentions _______ work on conditioning to support
that people are more affected by their
or experiences.
------------------------ -- --------- -- .. ---------------------. --- ---------------------------------- ------ -o;--- 262 Chapter 5

Art History

~~rNf;.R~:f!!.'&~-~~lJ;U-'.><X';

:io~_,_,,;;;.;.;~,~\.i'" -'~,--,~-

Listen to a discussion in an art history class. Fill in the diagram with the information that
you hear. tlffjifl:i
.~ey

,,,

Vocabulary

reassemble: to put back together


.
:'abstract: unrealistic; apart from reality
. recall: to remember'

distort: to change from a natural shape


merge: to bring together

j: cubism ~s, surrealism I


'1__,

Cubism:

Both:

Surrealism:

.I'. '
'

1. How does the professor organize the


information iJbout the two art movements?
.(A) He defines each art movement and
describes their originators.
(B) He compares the two art movements
,,
and their dissimilarities.
(C) He uses two examples of art to
contrast the two movements.
(D) He describes the different influences
that the movements had.

2. Why does the professor mention geometric


shapes?
(A) To explain an important aspect of an
art movement
(B) To provide the historical context of
the movement

(C) To show how one art movement


affected the other
(D) To compare two characteristics of the
Cubist movement
3. Why does the student mention distorting
reality?

(A) To contrast the style of the two art


movements
(B) To explain the influence of the art
styles on other art
(C) To note a similarity between the
two art movements
(D) To explain how the two art
movements began

Fill in the blanks to complete the summary.


The class compares the artistic aspects of two art movements: cubism and surrealism.
In the discussion, it is stated that they both
reality, though they did this in
different ways.
focused on changing the shape of the subject and distorting
it
. On the other hand,
distorted reality by
_ _ _ _ _ _ _ that would usually not appear together.
-----------------------------------.. -,----------------------- ---------------------.. ------------------------
Organization Questions

263 _ _.:,'.

., --~.

'.

l\lecessary Skills
i>';~1=NX!Ll'<l_~r,,,;x~.1~1ili>if<.;:.:.:1;,'.~''r"''"0"-'i:''!""'

Understanding relationships between different pieces oi information


Identifying key category words in a lecture or a conversation
Understanding the characteristics of different categories
Comparing the characteristics of different categories
Determining if a certain point is discussed in relation to a category

if1.\i:im~i,e
.-

"':;+.--.;.;,_,-- ,-,.- ,_

Que!Dtloin:s
.,c,-~.'."f<-:O ,: -_,-,.;:. n ,- c,; .'

.;_\,,':-,,:v;~.~i;r;,.,~~;t:;~:

,.; .-

What does
demonstrate?
What does the professor demonstrate by discussing
?
Based on information from the lecture, indicate whether or not each statement
is correct. Place a checkmark in the correct box.

1-----------------------------

Yes

No

-----~------------'---------+----t------1

(statement)

"-~~-~--------A

,/
_____...,. __________________ ---+---

(statement)

--------------------------------------

,/
~f---

,-

,/
(statement)
---------------------------------------------'"-----'- ----~
Based on information from the lecture, to which feature does each example
relate? Place a checkmark in the correct box.
1----------------------------------------------(iS~!Ure)(teature)

J(~x~~pje)

,/

.;
__
_
! __~e!.~'!.1.~~~)__ ------------------ . -----------------------------[___(.____ -------~[~~~p1e)-:~~--~:==--=-~-

-----==~-:=]=~

Take notes as you listen, as questions with tables only appear after the lecture
or conversation.
Pay special attention to category words, the characteristics of categories, and
examples.
For questions asking if or how each phrase or sentence applies, be sure
to click the appropriate box for each answer choice.
" Keep in mind that there are different types of tables to complete: some in which you
need to click Yes or No, and others in which you need to click the correct category.

Content Questions 265 ____.,_

Listen to a lecture in a biology class. Fill in the diagram with the information that you hear.

@pp+
Key Vocabulary

down: sad
trend: the geneml direction 91

essential: necessary
absorb: to take in
grasp: to understand

rnqve'me~t '
.. ,,;\;.-__ :-_<::. ,-- ".: i.':

Support 1:

Theory:

Support 2:

1. Based on information from the lecture, indicate whether or not each statement is correct. Place
a checkmark in the correct box.

-----.....-------------.-..---------------..

The sun is a better source of Vitamin D than food.

-~--~-----

'

Yes

No

---~--------- -~------

The human body requires two hours of sunlight every day.

1---------'---~-----------~-------------------+--1

____________ _,__ _ _t ____J

Fill in the blanks to complete the summary.


-v-----------------------------------------------------~--"----------------------------ri----

The professor discusses the benefits of sunlight to

. The professor also


for mental health, helping to make

people-------

-------- ____ ., ____________________________________________ ----- --..

- - 266 Chapter 6

,(<

. He says that it provides

_ _ _ _ _ _ _ , which helps our bodies absorb


says that exposure to the sun can have

.. --------u

------~-----------

.. ----------------- ------------- -

\'

History
l:.<&!lila~~<;,m1>:'1!J'cCi~,-;,)'1W~,_,~,lf.tMll;;~~);.>'[(~;'Cf."'l'-".':,>;:!C'h'"\'f.1'

;,CJ,.;' '_- ;,, '-~,-: .' -

Listen to a lecture in a history class. Fill in the diagram with the information that you hear.
MMfjifUI
Key Vocabulary
. symbolize: to stand for another thing; to represent
. .. rebelliously: against 9ontrol or authority
.
oppresslon:Jhe state of cruel.treatment.

[
[

storm: to auack or capture a place suddenly and with ..


great force

trigger: to activate or cause .

:' Symbolism ofthe Bastille .


Key Point 1:

Support:

-------------------~

Key Point 2:

Support:

..

----~-

1. Based on information from the lecture, indicate whether or not each statement is correct. Place
a checkmark in the correct box.

I-The eve,ntsat

;h~-Bastille ;,~~

caused by the

re~~lution.

.Yes

No

~}~e Ba~ti~7e"~~f ned_a s}'.~b~I oI;?ii~iZa:i::EE!_~-~~~~----------t----+---1


l_!ll_e...9~'.i~l_e wa~9_p'.!_s_~_.!~<3._t . W.9~_<3._tt_<3._~~~-~:....---------------------'---'---'
Fill in the blanks to complete the summary.
The professor talks about how certain things can
greater ideas for people,
and he uses the example of the Bastille. Originally it was a place that _ _ _ _ _ __
It was a sign of
people freed

and oppression. However, after an event where the


of the Bastille, it came to represent the French Revolution and

I
I

=.
~p!;

Ji l;~ !l !lll\i.i'.;
._. ......... .,...._........................................................................................_
__

'

Content Questions

..............-

267

--1

------------.-i"--;-.

...

. ,-.0

Office Hours
Listen to a conversation between a student and a professor. Fill in the diagram with the
information that you hear. 'I""
Key Vocabulary
organic: having to do with carbon-based chemicals
acknowledge: to recognize and accept
tutorial: help for learning

caught up: very Involved; busy


supplementary: for support; extra

Support 1:

Problem:

Support 2:

1. Based on information from the lecture, indicate whether or not each statement is correct. Place
a checkmark in the correct box.

---------------------.

Yes

'

No

The student is worried about the next test.

-------

-------~

'"

The professor posts tests on line.


~----~--

"'

---~-----

des online tutorials.


---~-~--~"-"---~~-~--------~---~-----

Fill in the blanks to complete the summary.


The student visits the professor to talk about _ _ _ _ _ _ _ . The professor
recommends two ways that the student can
. First, the professor keeps
copies of ________ on line for students to review. Second, the professor says
that there are
tutorials that can help the student as well. The professor also
mentions that
the textbook is important.

268

Chapter 6

'

Service Encounter
.

!1'4~~ooi'o\ii~tl#.$'lfl;)!;1';Wl':".I/Ul'i<"<;!..1W<'W5"~:..-.~o:..1,~.-.;_._..,,_;,;,,.o,-.. :;,..,

Listen to a conversation between a student and a university employee. Fill in the diagram
with the information that you hear. i@Sftl
Key Vocabulary
cater: to provide food for events
afflllated: associated; related
orientation: an introduction to a new situation or
environment

hire: to accept for employment


pencil In: to write in

[:'
I I
1.

2.

3.

I
I
I
I
I ..

1. Based on information from the lecture, indicate whether or not each statement is correct. Place
a checkmark in the correct box.
-

Yes

The stud ent is lo9king for a job that begins imme_9iately.

The stud ent will work in the bakeshop.

--

No
.

---

The stud ent hopes to b~9in workin_g as soon as possible.

----

I
Fill in the blanks to complete the summary.
A student wants to sign up for an on-campus job, so she visits the _ _ _ _ _ __
office. The employee tells her about the
process. First, she must go through
an official
. Then she must go to an
session. Last, there
is a - - - - - - - that occurs after she begins work.
------------------------------------------------------------.. ---"''''~- .. --------------------------------------------------

Content Questions

269 - - .

f'

Environmental Science
,-,-..,_.,.,,..

Listen to a discussion in an environment~! science class. fill in the diagram with the
information that you hear. l@jE!i
Key Vocabulary
plate: a piece of the Earth's surface
.
collision: tho action of two moving objects hitting each other

b~caus~ ~ther 'a9tio~s

result:. to happen
of
pressure: the state of being pushed together;
spring: to move suddenly '

Cause 1:

t
11

I,
I'
1.:

I'
I:
I;

Effect:

H
I'

Cause 2:

--------------1. Based on information from the lecture, indicate whether or not each statement is correct. Place
a checkmark in the correct box.

Fill in the blanks to complete the summary.


------------------------.. ------------------------------------------------,--- .... ------.. ------------------------ .. The professor talks about why some mountain ranges are
than others.
He says that this is sometimes caused by
collisions. This happens when
two of the Earth's plates come together. The professor also discusses _ _ _ _ _ __

as a cause.

push down on the Earth. But when they melt, the Earth
_ _ _c___ _ _ _ and mountains grow taller.
,,_,. ________ MMMMMO--------------M""'"'"'""riM-<>"''""'-,.-MOMMh-M--

_ _ 270

Chapter 6

,,.

I:

~-

...,.

History
.

~~Wf!S~;{.f>7,";,;lt',l.<<U>t"ti<!!H1''.o\'.WN:~:>..lli'T~Y<J."CT.:~":i'''?:l_,,'.: '~"-'" , ? ' , ' '

Listen to a lecture in a history class. Fill in the diagram with the information that you hear.

Key Vocabulary

[
I<; Confucianism .. ,

I'"""
!..... .

-----'I ~

".
I

r;:

1. What does the relationship between


parents and children demonstrate?

(A) How Confucianism replaced


traditional values
(B) How Confucian societies view laws
and rules
(C) How familial roles are affected by
Confucianism
(D) How Confucianism can be challenged
at home
2. What does the professor demonstrate by
discussing profit in business?

(A) How to apply Confucianism to business


(B) How Confucianism is not always
helpful for business

(C) How Confucianism is opposed to


business
(D) How to operate a business based on
Confucianism
3. What does the professor demonstrate by
discussing power struggles?

(A) Businesses operate smoothly without


them.
(B) Confucianism cannot offer a solution
to them.
(C) Power struggles can help a business
operate.
(D) Confucianism changed the concept
of business.

Fill in the blanks to complete the summary.


The professor discusses the effects of Confucianism on
Asia. In
particular, the professor mentions how Confucianism affected the culture of _ _ _ __
First of all, its influence can be seen in how people
each other. It emphasizes
living in
with others by knowing one's place in society. The professor then
discusses how Confucianism applies to - , . - - - - - - -

Content Question<;

271

--.1
i

Office Hours
Listen to a conversation between a student and a professor. Fill in the diagram with the.
information that you hear. @S&J
Key Vocabulary

commute: to travel regularly between two places.


afford: lo have enough money for something
conventional: traditional or common

Problem:

come across: to meet: to find


ponder: to think about

~-

Concern:

Refutation:

Concern:

Refutation:

Solution:

1. What does commuting demonstrate?


(A) The student cannot take summer
courses.
(B) The student's trip to campus takes a
long time.
(C) The student does not like online
courses.
(D) The student cannot afford an apartment.
2. What does the professor demonstrate by
discussing her own experience with online
courses?

(A) Online courses offer a better


education.
(B) Taking online courses is convenient.

(C) Taking summer courses online is


affordable.
(D) Online courses do not offer classroom
experience.

3. What does the professor demonstrate by


discussing online message boards?
(A) Why she favors online classes over
regular classes
(B) How her opinion of online classes
changed
(C) How message boards made her
class easier
(D) How Internet classes offer unique
advantages

Fill in the blanks to complete the summary.


The student asks for advice from his professor about - - - - - - - The
professor mentions
, but the student is worried that such classes are
----~--

The professor says the student would actually save money and
______ earlier. The student then worries that he will miss the _ _ _ _ _ __

l'

but the professor says that certain aspects of these classes could be helpful.
-----.. -------.. ------ _. __._ .. ___ '"--- . ------". --.---.........,.,_, ___ - .... ------------------------- ~~ -.. ______ _._ . _

11.

J!.L.1

p1

,!1il
ij
I

I.

':I---

p'I

272

Chaple< 6

"'
~z

Service Encounter

Cl

Listen to a conversation between a student and a university employee. Fill in the diagram
with the information that you hear. i@jiji
Key .Vocabulary
..
. -

'credit: a numericafrepresentation of academic work ' '


steep: expensive .;
:. , ..
waive: to ignore or get.rid of.

...

IJ>

. mandatory:_ required
' escort: to guide or lead

'"
)>

~-:-

..

Cl

Question 1:

:e

Answer:

"':::;
z
Cl

...

Answer 1:
------~-----------~

Question 2:

";;j'"

< ,- -;n-: -s_w-=e-~

....IJ>

2--:--

1. Based on information from the lecture, indicate whether or not each statement is correct. Place
a checkmark in the correct box.
~--~-----

Yes
;/-.

No

The student wants to find out how much the fee is and what it is used for.
The student

activi~y

fee does not include can:p_us events.

_!_~-~-~'.U..~.en~?.t~~ity !_ee is used mostly f~'.- cam~-~~ _s_~r-~!ces .__

----~-~----

---- - -

Fill in the blanks to complete the summary.


-------------- .. ------------------------------------"---- .. -.. -----------------------------------------The student asks about the student - - - - - - - He wants to know how much it
is and what it is - - - - - - - He is told that it helps pay for both university
_ _ _ _ _ , which are arranged by student organizations, and is also used for various
_ _ _ _ _ .A few services mentioned by the woman are the weekly newspaper,
computer and library facilities, and free on-campus _ _ _ _ _ __
--------------,.-------------------"-----w---------------------.----- .. --- '' -- - - ... ,,,,,-------n-->-

Content Questions

. _:._._,_ __ .:-

273 --

Astronomy
""""~''llll'IID!:!W>'i'~~(;;l:-t'fmm1Y(~;?.>~l~~"'r.',~-"''Y.i<. v ''"'~' -~'.i'NH:~'''L""' -'- s", : . ,, - ,

Listen to a lecture in an astronomy dass. Fill in the diagram with the information that you

hear. @iii
Key Vocabulary
striking: noticeable
attribute to: to understand as resulting from; to consider as
caused by

follow: to understand
build: to increase
dissipate: to disappear over time

WeatheF Pattern_s onJupitef


Classification 1:

Description:

Classification 2:

Description:

1. Based on information from the lecture, to which feature of Jupiter does each example relate?

Place a checkmark in the correct box.


.

Atmospheric Bands

Similar in nature to a hurricane

Great Red Spot

~-------'---1
-==~

Caused by convection
c_p~rk
---- _____, _
------and lig~t red color .--- __________ . _________L ___________
-------"------~-

Fill in the blanks to complete the summary.


The professor explains the weather patterns on the surface of _ _ _ _ _ She
describes its unique colors, which are caused by the
of the planet
along with the process of
. These two things together cause bands of
_ _ _ _ _ gases to wrap around the planet. The professor also discusses the Great
Red Spot, which is a

maintained by the planet's fast rotational speed.

-- .. ----------------------------------------,n-- .. ----------------------- .. --------~-------------------------~"

- - 274

Chapter 6

Biology
~~tt;w>~!;l'.;::rn:lW..!!~~'tW.t'<~~1\-";;Ri'.'lffK~l'\;~J<!l.'.-~:, <--~~'''.'-'

-. ',- -.-,-

1'

Listen to a dis~ussion in a biology class. Fill in the diagram with the information that you
hear. +ifi!jii:H
Key Vocabulary
strand: a thin hair-like part
frantically: in an uncontrolled way
Inherit: to receive from one's parents or ancestors

[
[

dominate: to rule over


materialize: to give physical form to

\ !AHeies - 1
Definition:

Role:

ll

Type 1:

Type 2:

1. Based on information from the lecture, indicate whether or not each statement is correct. Place
a checkmark in the correct box.

----------------..----------------------------------.,..--.....----

Yes.

No

--

Fill in the blanks to complete the summary.


The professor discusses
, which are a pair of
that are
located on a specific chromosome. The professor talks about their
, which
is to determine specific
. The professor then discusses how this occurs by.
defining
and recessive alleles.

Content Questions

275 --:.

-,.

'\.;

Necessary Skills

Guessing the implied meaning of a sentence or phrase


Making a generalization from what is said
Drawing a conclusion based on the main points of a lecture or a conversation
Recognizing how intonation or stress indicates implied information or opinions
Inferring what is likely to happen from what a speaker says

Example Questions
Some inference questions will not appear during the test. You will only hear them.
Listen again to part of the conversation. Then answer the question.
You will hear a few lines of the lecture or conversation again.
What does the professor imply when she says this:
You will hear part or one line of the previous excerpt again.
The following types of inference questions may appear during the test:
What does the professor imply about
?
What can be inferred about
?
What will the professor likely discuss next?
What will the student probably do next?

Try to guess the implied meaning of the given information. The correct answer
is not directly stated.
Pay attention to clues expressed by certain words, word stress, intonation, or
pace of what is said. The same sentence can express different meanings when
said in different ways.
-> Example: Oh, you've never heard of that. {I may need to explain more than
I thought.)
Oh, you've never heard of that? {I'm surprised that you've never
heard of that.)
Pay attention to the last part of a conversation. For example, if a speaker
agrees with the other speaker's suggestion at the end, we can infer that the
speaker will do what is suggested.

r
Inference Questions

277 -

Environmental. Science
Listen to a lecture in an environmental science class. Fill in the diagram with the information

that you hear. @@@

Key Vocabulary
exchange: lo give and receive
circulate: to pass from place to place
enclose: to surround

. marginal: being nexl to somelhing geographically


density: lhickness; a measure al mass by volume

I . Types
. of seas "I
.

Type 1:
I

Explanation:

[----:y

Explanation:

Key Point 2:

Poiot 1,

'--

L----]

1. What does the professor imply when she


says this:
(A) Most people understand what makes
up a sea.
(B) People might not know the
characteristics of seas.
(C) Her students likely know all about seas.
(D) There are many definitions of a sea.

-"--~-----

----------

3. What does the professor imply about the


Mediterranean Sea?

(A) It shares its name with another famous


sea.
(B) It is not appropriately named.
(C) Its currents are controlled by ocean
currents.
(D) Its name may confuse their discussion.

'I

2. What does the professor imply when she


says this:

Key Point 2:

(C) Marginal seas only have one


distinctive quality.
(D) All bodies of water have ocean
currents that run through them.

,,

ey Point 1:

Type 2:

{A) Marginal seas are the most common


type of sea.
{B) This difference is more important than
others.

Fill in the blanks to complete the summary.


------.. -------------------------~----:,-;;, .. ~--"-"----""- ..
The professor discusses the different types of seas. The first type is called a
_ _ _ _ _ _ _ sea. The water in this type of sea circulates through changes in
_ _ _ _ _ _ _ and water density. The second type is called a
sea,

""e----- ---"-'" .. --m--------TI,,-.o-"'''

which is nearly enclosed by


this kind of sea are circulated by

f.

IR
:1

Wt-~.

1
\

or some other kind of land. The waters within


caused by wind.

------.,.---------------~---------------'"-"--_._.

278.

Chapter 7

.... .,.,_.-nn----.,---------------------'"-->----<0--.,---n-

Biology

[ri

[
.''
I'"'
._,.

~~V:~~/G,ftl!;1"m.Y.'<;U'il'.."3!U..Y.l'.<~"\V:<il1oli-''-">"N''i,y:,;;~h_--,_~--

Listen to a lecture in a biology class. Fill in the diagram with the information that you hear.

''II"'
Key Vocabulary
marine: relating to the ocean ..
respiration: the act of breathing
extract: to draw out of something

[
[

swap: to trade or exchange


vertebrate: an organism with a backbone

--'~-.

'. .. . .,- .
~

!'

Respiration Wncierwaforvs:
Respir<.1tion
on Land
-'"''
,,.,,

' ,,

Underwater:

_,

--

-;

Both:

Land:

.I

1.. What does the professor imply about


, oxygen content underwater?
. (A)
' (B)
(C)
(0)

It is more abundant in cooler water.


It is not as plentiful as on land.
It is virtually non-existent.
It is less difficult to process.

2. What does the professor imply when he


says this:
(A) The exchange of gases occurs in
various aquatic animals.
(B) The environment of land animals is
rich in oxygen compared to aquatic
environments.

(C) Many aquatic animals can live without


oxygen for long periods of time.
(0) The students will certainly be tested on
this information on the next exam.
3. What will the professor likely talk about
next?
(A) The way gills extract oxygen from'
water
(B) The advantages of underwater
respiration
(C) The reasons why lungs work better
than gills
(D) The development of gills in fish

Fill in the blanks to complete the summary.

----------------------------------- ..----"--.. ----------------

._,~-----------

.. -----------------------------

The professor discusses two types of respiration: underwater and on land. He explains
the need for different respiration
for animals that live underwater.
_ _ _ _ _ are adapted to extract oxygen from water. For animals that live
_ _ _ _ _ _ _ , there are different ways of breathing. Some animals can breathe just
by having their _ _ _ _ _ exposed to the air. Some animals require special organs to
breathe, like _ _ _ __
______________________________________ ,,,,_ ...... -------------------- .. ---------------- ..----------------------------------Inference Questions

'--'-,----,----,-'

----'-----------'-

-,.- --

'------o---"--~----

279 _ _

Office Hours
.

Listen to a conversation between a student and a professor. Fill in the


information that you hear. +Mi''''

'

diagr~m

with the

l<ey Vo~abulary
':coord.lnate: to match an,d manage:,
roster: a. list of people's names.

mind: to be offended or irritated


confidential: done in secret
appreciate: to be grateful for

Problem:

Solution:

1. .What does the professor initially assume

about his teaching assistant?


. (A)
(8)
(C)
(D)

She
She
She
She

is not doing a good job.


is not completing her work.
is satisfying the students' needs.
does not understand the class.

2. What does the professor imply when he

says this:
(A) He does not want to meet with the
student
(8) He is sad that he cannot stay longer.
(C) He does not have much time.
(D) He has another class to teach.

3. What does the student imply when she


says this:
(A) Her professor's talks are difficult to
understand.
(8) Her professor does not end class
on time.
{C) The scheduling is not a part of her
problem.
(D) Her professor does not know when the
class ends.

...

Fill in the blanks to complete the summary.


The conversation takes place between a professor and a _ _ _ _ _ . She is
approaching the professor to change thetime of her
. The professor
asks her why, and she explains that it is a problem with her schedule caused by
_ _ _ _ _ _ _ . She has trouble _ _ _ _ _ to her group on time. The professor
_ _ _ _ _ to change her to a different group.

I I - - - 280 Chapter 7

\---.

"'
~z

Service Encounter

Cl

Listen to a conversation between a student and a university employee. Fill in the diagram
with the information that you hear. ltlffllotl

Key Vocabulary
coordinator: someone whose job is to manage or
supervise
complaint: an expression of irritation or disapproval

shame: an unfortunate situation


assume: to accept something to be true without proof
straightforward: simple; easy .

Problem:

I
I
I
I

::;;

Solution:

"'3z
Cl

1. What does the woman imply when she


says this:
,(A) Few students come to the housing
..
office.
(B) Most students come to the office to
complain.
(C) The rooms in the dorms are widely
disliked.
(D) She does not know why students make
so many changes.

(C) He does not want to change dorms.


(D) He thinks the housing situation is
unfair.

I;
'

I
,.

I
\

I
I
I

I
I

I
I

-~,

3. What does the student imply when he says


this:
(A) He never received a social security
card.
(B) He does not understand the form.
(C) He is surprised by the requirement.
(D) He thinks the employee made.a
mistake.

2. Listen again to part of the conversation. Then


answer the question.
What does the student imply when he says
this:
(A) He would have liked to live in the dorm.
(B) He finds the other dorms unacceptable.

Fill in the blanks to complete the summary.


---n------ -----'-"'"'---

,.,,.----.o~-------.,----------n-----------------------------MMMMMMMMMMMMMMMMMMM

The conversation takes place between a


employee and a student. The
student has accidentally been assigned to a dorm that houses
, and
he needs to change dorms. He says the room is
, but he obviously cannot
stay there. The employee provides him with a form that will allow him t o - - - - - - a different dorm. She also tells him that he can leave part of the form------~
---------------- .. ---- .. -------------------- .. ------------------------------------------------------------ .. -----Inference Questions

I
__

, -

..

',

LC

281 _

_;,:

Business
Listen to a discussion in a business class. Fill in the diagram with the information that you

hear. @i
Key Vocabulary
Incorporate: to use or include something
feasible: able to be done
following: a group of admirers

logistical: related to managing the details involved in


achieving something
percetved:based on assumptions; inferred

The Bricks and Clicks Business Model ... I


Definition:

I""'"" ,
I"''"' ,,

1. What does the professor imply when he


says this:

(A) He finds the name of the business


model confusing.
(B) He thinks the students might not take
the model seriously.
(C) He does not understand why that name
was chosen.
(D) He thinks the name clearly defines the
model.

2. What does the professor imply when he


says this:

(A) Customers do not like to do business


with new retailers.
(B) New businesses are more likely to use
this business model.

(C) Retail businesses need a supply


network to use this model.
(D) Established businesses are more
likely to make this model succeed.

3. What will the professor likely discuss next?


(A) The extremely high costs associated
with the model.
(B) A popular business model for smaller
businesses
(C) Another set of advantages of bricks
and clicks
(D) Another business model option for
retailers

Fill in the blanks to complete the summary.


The discussion is about the _ _ _ _ _ _ _ business model. It is a model that
includes both offline and
elements. The professor explains a couple of
_ _ _ _ _ _ _ for this business model. First, he says that it allows _ _ _ _ _ __
more freedom in how they operate. He also says the system is stable because established
businesses already have ______ and distribution chains set up.

--------------------------------------------------------------------------------------------------------------
I
'

~-------'----~----~---------'---"-

'

'

~-"--

'-/','
I

-~<~'.r-

1:

I
I
I
I
I

Anatomy
"'~.JK'<l~'?ll!l'..1Cl,",H,G;'l'.D'.-"(,;;o.<'!;1~i1-'>'1oVY.'.Jk~:.<%:-,_,o.,<-:''''-'->1"'.

Listen to a lecture in an anatomy class. Fill in the diagram with the information that you

hear. +ffiNiM
Key Vocabulary

comprise: to include or be composed of


regulate: to control something
Insulator: something that stops heat from escaping or entering

abrupt: unexpected or sudden .


receptor: a kind of cell that responds to sensory
stimuli

The lntegumentary System

'I

---------=]

Definition:

I
I

I
I
I

l
I
I
I
I

Gl

J
J

Function 2:

"'3z

Function 1-:

I
I
I
I,

:e

1, What does the professor imply when he

(D) Skin is important to the integumentary


system.

' says this:


(A) He has discussed the topic before.
(B) The topic is very advanced.
(C) He is going t.o offer new information.
(D) His students know the topic well.

3. What does the professor imply when he

says this:
(A) Perspiration is the most effective way
of regulating body temperature..
(B) The professor expects the students to
read about perspiration on their own.
(C) The body controls its temperature by
many methods other than sweating.
(D) The mechanisms for controlling body
temperature belong to a different system.

2. What does the professor imply when he

says this:
(A) Humans would be in danger without
an integumentary system.
(B) The class should be happy to learn
about the skin's protection.
(C) The skin protects human beings from
injury.

Fill in the blanks to complete the summary.


The professor discusses the integumentary system, which
the
body's external coverings, such as
, hair, and nails. First, the integumentary
system provides
for our internal organs. Second, it helps us to
- - - - - - - our body temperature. Last, it acts as a ~------for pain
and temperature changes.
.. ---- "---------- ---- --- ------ -------------- .... - __ .. ________ -------- .. --- ----------

----~--------

Inference Question~

,c.,~;"_

'-

__, 0

283 - - .

Listen to a conversation between a student and a professor. Fill in the diagram with the

information that you hear.

@ifi

Key Vocabulary

..

)-

fascinating: very interesting


tangible: solid and able to be experienced
aim for: to hope to accomplish; to try

Problem:

motivate: to cause someone to behave in a certain.


- ./
man.ner-- ,. <:.-.,; ,~:-:::._-::--;, _ ;!/:;;'./!
submit: to give somethingto be iuciged.

>:- _

Solution:

---------------
1. Listen again to part of the conversation. Then
answer the question.
What does the student imply when she
says this:
(A) She did not understand the
professor's topics.
(B) She was interested in the professor's
topics.
(C) Her problem is unrelated to the given
topics.
(D) She is upset about having to choose a
new topic.

2. What does the student imply when he says


this:
(A) She is well acquainted with the
professor.

:1 I

11

11

(B) She does not remember a key point


from the class.
(C) She will take more classes with the
professor later.
(D) She thinks the professor knows what
she has come to discuss.
3. What does the professor imply when he
says this:
(A) He believes the other studerits will
benefit more than the woman.
(B) He worries that the student's
assignment will be too hard.
(C) He thinks the student will not work
hard enough on the paper.
(D) He thinks other students will be upset
if he agrees.

II
Fill in the blanks to complete the summary.

-- .. --"---------- .. -..... -----------------------------;-------,,-------------------------------------------A student approaches her professor to discuss the


he assigned.

The student wants to


a previous paper that she wrote rather than
choose from the
the professor presented. The professor
_ _ _ _ _ to let her do this because the student plans to submit the paper to

-------------------- ---------------- ------------------------------------------------------------------ ----..,y,;--

284

Chapter 7

Service Encounter

l
I

I
I
I

Listen to conversation between a student and a university employee. Fill in the diagram
with the information that you hear. +@M"I
Key Vocabulary
policy: a plan or course of action
burur: a person at a university whose job is t~ ~antral
money
comprehensive: large iri scope and including much

:e
Covered by Insurance:

Not Covered by Insurance

J,I
.,

3"'2
Gl

I
I

i.

routine: regular; normal


out-of-pocket: paid out or owed in cash

1. What does the woman imply when she


says this:
~. (A) The insurance policy is difficult to
understand.
(B) The student does not need an
explanation .
(C) She needs more information about the
student's problem.
(0) She thinks the student is confused by
her explanation.

2. What does the student imply when he says


this:
(A) He was unaware that there was a
health insurance fee.
(B) He is not willing to pay a fee for health
insurance.

(C) He believes that the health insurance


charge is unfair.
(0) He had planned to dispute the charge
with the university.
3. What does the student imply when he says
this:
(A) He will find more information about the
policy on his own.
(B) He now fully understands his
insurance policy.
(C) He finds the secretary's explanation
confusing.
(0) He plans to cancel his health
insurance policy.

Fill in the blanks to complete the summary.


The conversation takes place between a student and an employee at -----~
The student approaches the employee with questions about the university - - - - - - The employee explains the cost as well as some of the _ _ _ _ _ that are covered and
_______ in the plan. The student also finds out that _ _ _ _ _ insurance is
not covered by the university plan at all.

-----.------------------------------------------------------------- .. -----------------------------------.--------------Inference Questions

285 _

History
_=t;,Wi;aJ,;t~'!8\\l!,~eo<~t";.)'~1i''l-"~n-;>H"O.~:;<;.p;_,-,~-,--:, ..

,__,.<,<,,,.,_ );_>;(.'i.<Y,Y-:,,;;_,._<-:' ;~t-. _, . . l

Listen to a lecture in a history class. Fill in the diagram with the information that you hear.

'MDI''
Key Vocabulary
Indigenous: occurring naturally in an area
segue: to move smoothly onto another topic
reconstruct: to rebuild

metallurgy: the study of metals .


advanced: ahead of others in technology

LThe s.ucGess oUhe Purepecha

Poict

'_:-~--------~I-----

l~Poin-t--~
2:

-
,.\<I

i"1L .
1,i

".

1. What does the professor imply when she


says this:
(A) There is little information available
about the Purepecha.
(B) More is known about other tribes
than the Purepecha.
(C) The history of the Purepecha has been
hidden or covered up.
(D) Other tribes have longer histories for
scholars to study.

,.
,!

'

2. What does the professor imply when she


says this:
(A) The Aztecs went to war with many
peoples.
(B) The Aztecs survived longer than the
Purepecha.

.__Ex_a_m_p-le_:_________

---L""'' " -~

I!

(C) The Purepecha were a very peaceful


tribe.
(D) The Aztecs had a very advanced
military.
3. What does the professor imply when she
says this:
(A) Other indigenous. groups attacked the
Purepecha.
(B) Other cultures did not have highly
organized militaries.
(C) The Purepecha's military was the
largest in Central America.
(D) Little is known about other indigenous
groups.

Fill in the blanks to complete the summary.


--~----------------- .. -------~---------------- .. -.............. ---------------------.. ------.. ----'"---.. ----The Purepecha are a tribe living today in
. The professor presents
two theories about why the Purepecha peoples were able to effectively _ _ _ __
Aztec and Spanish forces. First, she argues that the advanced Purepecha knowledge of
_ _ _ _ _ _ _ helped them produce
. She also proposes that
the Purepecha army was relatively

and well-organized.

. .':.
!>
;)'

'~'.'.

,,

Ecology
""""-~~.Wf>\'-'i!l'J}J)';Ql:~~-"'xt.>E~mu;.:;;-;:m;,,f

...;o;;wxe"' '" "''- ._.,, . '

Listen to a discussion in an ecology class. Fill in the diagram with the information that you

hear. 'iffiiE'
Key Vocabulary
direct: having nothing to prevent contact
organism: a living thing
benefit: to help; to improve

predator: an animal that hunts am;! eats other animals


sustenance: food

I >Par~sitism vs. Mlitualism:

~,

J:

Parasitism:

Both:

Mutualism:

. 1. What does the student imply when he says


,. this:

(A) Parasites are not really predators


because they are smaller than their
hosts.
(B) He is confused about the difference
between parasites and predators.
(C) Something that kills an animal could
be larger or smaller than the victim.
(D) Conventional predators in nature kill
their prey quickly.

2. What does the professor imply when she

:. j

says this:
(A) The role of parasites is often
misunderstood.
(B) Parasites do more harm than good.

(C) Parasites are more common than


generally known.
(0) Animals would be better off without
parasites.

3. Listen again to part of the discussion. Then


answer the question.

What does the professor imply when she


says this:
(A) The student has provided information
that is off-topic.
(B) The student's answer is not detailed
enough.
(C) There are certain steps in the process
suggested by the student.
(D) The two relationships are not
opposites at all.

Fill in the blanks to complete the summary.


The discussion begins with a definition and example of
and ends
with a definition and example of
. In the first type of relationship, one
animal
of another animal, and by doing so,
the
other animal. By contrast, in the second relationship, two species are able to
_ _ _ _ _ each other by either providing
or nutrition.
--------------~----~--------- ----- .. ------ ----- .... -- . - -- - --------------- --------------------------------lnfe(ence Questions

287 _ _:

Instructions: Choose the best word or


phrase to complete each sentence.

1. The book explains an easy way for a


person to
the number of
calories he or she needs to consume
each day.
(A)
(B)
(C)
(D)

alienate
anticipate
appreciate
approximate

6. The woodpecker's bright red crest gives


it a(n)
appearance.

2. A molecule that interacts with other


molecules and finally bonds with them is
called a(n) _ _ _ __
(A)
(B)
(C)
(D)

coordinator
insulator
predator
receptor

(A)
(B)
(C)
(D)

discouraging
following
striking
undemanding

7. We doubted that such a(n) _ _ __

candidate would pass the personal


interview of the hiring process.

3. The current exhibit at the Children's


Museum is sure to
the
imagination of visitors, both young and
old.
(A) segue
(B) spark .
(C) spring
(D) swap
4. In our physics lab today, we dropped
various substances into water in order to
measure their _ _ _ __
(A)
(B)
(C)
(0)

5. The speaker wanted to make the point


that, although we have come a long way,
the battle against
is far
from over.
(A) aversion
(B) collision
(C) distinction
(D) oppression

densities
harmonies
intensities
policies

(A)
(B)
(C)
(D)

confidential
conventional
egotistical
logistical

8. The university plans to _ _ _ __

its events on campus with the city's


Spring Festival events so that students
will be able to participate in both.
(A)
(B)
(C)
(D)

coordinate
dominate
motivate
reiterate

Instructions: Choose the word or phrase


closest in meaning to the underlined part.
9.. The findings of the two experiments did
not agree with one another.
(A)
(B)
(C)
(D)

.. '-

-;._,

.~

makeup
results
signals
trends

'

:~

1O. The professor spent the entire week


before finals giving the class a complete
review of everything covered during the
semester.
(A)
(B)
(C)
(0)

authoritative
comprehensive
excessive
exclusive

11. The presentation will be very informal, so


if you have questions or comments, feel
free to interrupt at any time.
(A)
(B)
(C)
(O)

aim for
come across
jump in
take place

12. This is a rather complicated problem, so


,. we have to take things one step at a time
to solve it.

~'.

(A)
(B)
(C)
(0)

advanced
affiliated
enclosed
regulated

13. The professor said goodbye \o the


students before they Jfill for spring break.
(A) circulated
(B) commuted
(C) departed
(O) diverged

14. The translation of the story is basically


correct, except for a few problems at the
end.
(A)
. (B)
(C)
(0)

essentially
frantically
mutually
rebelliously

15. Very few students are able to really


understand the differences between the
two philosophies the first time that they
study them.
(A) acknowledge
(B) grasp
(C) mind
(0) ponder
Instructions: Write the missing words. Use
the words below to fill in the blanks.
complaint

comprises

enrich

deduce

feasible

Studying vocabulary 16. _ _ _ _ __


a major part of preparation for college
entrance exams. An often heard
17.
among students is
that it is not 18.
to
memorize 5,000 vocabulary words for such
tests. However, there is an easier way for
students to 19.
their
understanding of college level vocabulary. By
studying Latin and Greek prefixes and suffixes,
students can learn to 20. _ _ _ _ _ __
the meanings of many unfamiliar vocabulary
words presented in lists or within sentences.

Instructions: Write the letter choice of the


opposite word in the blank.

21. _
22. _
23. _
24.
25. _

abstract
dissipate
mandatory
feasible
vertebrate

(A)
(B)
(C)
(0)
(E)

absorb
concrete
invertebrate
optional
impossible

Vocabulary Review 3

289 _

..

11;
"

J'

li:i

J,;1U'
,d.

;f;

01 Computer Science
Listen to a lecture in a computer science class.

'i'P'

Key Vocabulary
delve Into: to go into; to dig into
scheme: a plan; a design
device: a machine; a thing that does work .
accessible: capable of being reached .

::,::

1. How does the professor organize the


information about computer networks that
he presents to the class?
(A) He describes why LANs are more
popular than WANs.
(B) He demonstrates how each network
relates to the Internet.
(C) He explains the scale and basic
setup of each type of network.
(D) He talks about the advantages and .
disadvantages of LANs.

4. The professor describes various ways in


which LAN and WAN networks are used.
For each example, indicate what type of
network is utilized. Place a checkmark in
the correct box.
.

.Local area , Widearea


network ' :.. network

2. Why does the professor mention the


campus library?
(A) To show how WANs are used by
private institutions
(B) To give an example of the uses of
LANs
(C) To describe why WANs cannot be
used in small spaces
(D) To explain why LANs only use a
single server
3. What can be inferred about the speed of
WANs?
(A) They are slow unless a leased line is
utilized.
(B) They are typically slower than a LAN.
(C) They are faster when more computers
are connected.
(D) They are not affected by interference
withLANs.

Private
organization
Home office

r-

Ii

configuration: arrangement ol parts or settings; setup


leased: reserved for exclusive use
relay: lo pass along from one place to another

---

Internet service
provider

--

>-

Small office
~--

__.. _____

-- ---------

Dormitory
..____,

~-----

'---------

5. What does the professor imply about


leased lines?
(A) They are not worth the extra cost.
(B) There are cheaper ways of building a
WAN.
(C) They are the reason why WANs are
not secure.
(D) They do not have any advantages
over other methods.

"'~

z"G\

02 literature
Listen to a discussion in a literature class. M!!SH'
Key Vocabulary
plot: storyline; main story
obsession: an idea or desire that is always in the mind
demise: death
gruesome: scary and disgusting

6. How does the professor organize the


information about The Sandman that she
presents to the class?

(A) She allows the students to discuss


the themes and motifs.
(8) She talks briefly about the author's
background.
(C) She explains why the author uses the
eyes as a motif.
(D) She shows how the story is a good
example of the horror genre.
7. Why does the professor mention the
Italian word coppo?
(A) To connect the use of names to the
motif of eyes
(8) To give an example of a motif in the
short story
{C) To describe how the main character
was traumatized
(D) To describe the source of the author's
ideas for the story

8. What does the male student imply about


the class's understanding of the term "motif?"
(A) The other students are familiar with
the term.
(B) The other students have never heard
the term.
(C) The other students have studied
motifs before.
(D) The other students are unsure what
the term means.

metaphor: an image or idea used to represent


somelhing else
; eyuocket: a hole in the skull in which the eye sits
",-vacant: blank; empty .

::e

"'::;

Zi

9. Based on information from the lecture,


indicate which of the statements below
are included in the class's discussion of
The Sandman. Place a checkmark in the
correct box.

G\

The names of some characters in


The Sandman refer to the eyes.
The main character, Nathanial,
became blind as a young boy.
One of the story's themes is
obsession.

-------+--r-_,

The sandman traditionally


has a very different meaning
from that in the story.

The professor discusses vision


and optical illusions.
-~----------'---'----'

10. What does the student imply when he


says this:
(A) The students have many questions
about the motif.
(8) He does not understand the meaning
of the eye motif.
(C) The eye motif is very prominent in the
story.
(D) The student is not sure if the eyes are
a motif.

Mini Test 3

291 - -

f!.

'<v3 Service Encounter


Listen to a conversation between a student and a university employee. i@lfil
Key Vocabulary
syllabus: an outline of a course of study
on par with: equal to
legwork: walking

(''

....

11. Why does the man mention department


heads?
(A) To explain a confusing part of the
forms
(B) To tell the student that the deadline is
coming soon
(C) To recommend that the student study
abroad
(0) To explain an important step in
transferring credit
12. Why does the man mention a syllabus?

(A) To give an example of what the


student needs for her classes
(B) To give information about what
materials the department heads
require
(C) To illustrate why most students are
not granted credit for their courses
(0) To explain why the process of
receiving credit is complicated

make the time: to find enough time to do something


on the spot: right away; at that moment

13. Based on information from the conversation,


indicate which of the following statements
reflect an action the student must take in
order to receive study abroad credit
Place a checkmark in the correct box.

..
'

"

...

..

Yes . No

Receive course approval from


the head of the department

it

Submit the credit request form


by the deadline
Contact her professors about
_the cours~s she plans to take

,__._,.

Send her professors copies of


her final exam

14. What does the man imply about dealing


with the department heads?
(A) It helps to know what documents they
specifically ask for.
(B) They are hard to contact at the end of
the year.
(C) It is difficult to gain approval without
meeting with them first
(D) They are accommodating to the
schedules of students.
15. What can be inferred about the student?

(A) She studied abroad for two semesters.


(B) She is supposed to graduate in May.
(C) She is worried that her forms will be
rejected.
(0) She is upset that she will graduate
soon.

.f'

The TOEFL iBT Speaking Section-------------------------------- 296


Preview ----------------------------------------------------------------------- 299

Part 1

Thinking and Speaking

Chapter 1 Independent Speaking: Organizing Speech---------- 311


Chapter 2 Integrated Speaking: Synthesizing Information ------------- 325
Chapter 3 Integrated Speaking: Stating Opinions and Summarizing --- 345
Vocabulary Review 1 -------- ----------------------------------------- ------ 358
Vocabulary Review 2 -- ----------------- ---- --- ---------------------------------- 360

Making Speech Coherent


Chapter 4 Independent Speaking: Test Questions 1 and 2 ------------- 365
Chapter 5 Integrated Speaking: Test Questions 3 and 4 ---------------- 387
Chapter 6 Integrated Speaking: Test Questions 5 and 6 - Vocabulary Review 3

- -----

Vornbulary Revi<lVll 4 -----

----------- 401

----- - ------- -- ------------- 41 O

--- --------- .. ______ --- - ----- -------------------- 414

Chapter 7 Pronunciation -- - Chapter 8 Stress and Intonation -


Chapter 9

Pausing -

Answer Key Speaking -

765

The prompts for speaking questions on the TOEFL iBT can be categorized into six types:
Time

Question
Reading

Preparation

Speaking

15 seconds

45 seconds

1-2 minutes

30 seconds

60 seconds

1-2 minutes

30 seconds

60 seconds

Listening .

Independent 01
Independent 02
Integrated

03

Integrated

04

Integrated

05

Integrated

06

45 seconds

I'

The purpose of the speaking section is to evaluate your ability to speak coherently both on your opinions
and experiences as well as on information that you have read or heard. The speaking questions fall into
two categories: independent and integrated. For the two independent speaking questions, you should
draw upon your own experience and knowledge. For the remaining four speaking questions, you will
speak about what you read and/or hear. Your ideas need to be well organized, and the vocabulary and
grammar you use must be accurate enough to be easily understood.
In particular, each question type will require test takers to organize their ideas and speak toward different
goals:

'

'
Question

Task

Independent

none

Describe your experience.

Independent

none

Give your opinion and explain why you think tt1is.

Integrated

Reading
Conversation

100 words
200 words
60-90 seconds

Restate the opinion of the speaker and the


examples used.

Integrated

Reading
Lecture

100 words
200 words
60-90 seconds

Explain how the example from the lecture


supports or refutes the passage.

Conversation- Conversation 300 words


Restate suggestions and tell which you think is
bai1,-ed
90-120 seconds better.

Materials

Length

Tasks
I

'

'

'

I
'

. Lecturebased

Lecture

Summarize what you heard.

300 words
90-120 seconds

,;;
,!'''

!H

: ' _,

-~'

-,,

i[::u~ ;;p;;:;;::~k;ng
["''

:':. . .
!.

~;~~; ~;i~c~:~::~~c=~ ~;v~~;~, ~~o;;~i.~/~~:~;:i.~~:~~~:~ :a:~~:~~ ~=~bs:i~~~a~~:~;~~i~

short. Also, practice pronouncing the diphthongs (combined vowels) as one short, continuous sound
rather than two separate ones. These include the sounds in the following: ail, bye, boy, and house.
Practice speaking with a North American inflection. This involves moving the lips and opening the
mouth wider, and speaking more from the mouth and nose than from the back of the throat.
Practice using the pauses and intonations you learn when studying for the listening section of the
TOEFL iBT.
Practice speaking at home. Use one of the independent writing topics as a speaking topic. Give
yourself 15 seconds of preparation time. Use this time to think of your main idea and details or
examples to support it. Speak for approximately 45 seconds on the topic. Also practice with 30
seconds of preparation time and one minute of speaking time, as this will be the case for the integrated
exercises.

T~ist

Management

'1You

will speak into a microphone attached to a headset.


Independent Speaking questions come first.
:You can take notes and then use them when preparing your response.
Check the time with the clock shown in the title bar on the computer screen.

The TOEFL iBT Speaking Section

-'-'-Co';"-_'c;.-Co'--'-c:c~---~-c:-,_

------

---~---

---c--::::-

----==-----=-:..::==-==-=-

------------------

297 - - : .

,1

How Speaking Will Be Scored


ETS graders will score test takers' responses according to the following scale:
Score

General Description

Key Points

The response answers the question or prompt


well. The speaker is easy to understand and
there are only minor mistakes with grammar
or pronunciation.

Fluent speech that is easy to understand and follow,


appropriate use of grammar and vocabulary, ideas
are explained clearly

The response answers the question or prompt,


but not all of the ideas are fully developed. The
speaker can be understood, but there are
some clearly noticeable mistakes in speaking.

At least two of these problems are present:


poor pronunciation, poor pace of speech, wrong
word choice, limited use of grammar structures,
or incorrect grammar

The response gives only a basic or minimal


answer to the question or prompt Most
sentences can be understood, but some effort
is required by the listener because speech is
not fluent and pronunciation is not accurate.
Some ideas are not clearly explained.

At least two of these problems are present: poor


pronunciation, choppy speech (not fluent), incorrect
word choices, basic grammar, poor use of grammar,
only basic ideas are presented, explanation is absent
or limited

The response is very short, does not show


full understanding of the question or prompt,
and is hard for the listener to understand.

At least two of these problems are present: poor


pronunciation, choppy speech (not fluent), tong or
frequent pauses, poor grammar, use of obviously
practiced or formulaic expressions, lots of repetition
of expressions in the question or prompt

There is no response or the response is not


related to the question or prompt.

No response to grade, or response is not related


to the question or prompt

lL~-.

, ,,,,-- 298 The TOEFL' iBT Speaking Section


1:1 .1
'.,;

The following is a sample Question 1 similar to the one you will see when you take the test. You will first
see a prompt on your screen describing the task similar to the one below.

Describe a study strategy that you use. Why do you feel that this strategy is effective? Include
specific reasons and examples to support your answer.
You will then have 30 seconds to prepare a response and 60 seconds to speak your response into the
microphone. A sample outline of a response can be found below.

Sample Outline
Introduction:
Thesis statement
Body:
Reason 1
'' Topic sentence
Reasons, examples, and details
.'.Reason 2
Topic sentence
Reasons, examples, and details
Thesis statement

When I am studying a difficult subject, I find it very helpful to play slow music at a low volume in the
background.
Tcpic sentence
;J

For one, this helps me relax. I tend to get very stressed when I study, and playing music keeps me calm
so I can focus on my task. ---..,

Reasons, examples, ancl details

Topic se; :anc''

Furthermore, I think it helps me remember the material more easily. When I try to recall information that
I learned while studying, sometimes I can remember it by thinking of what song I was listening to when
I learned it.

</

:=ieasons, exarnples, o.r1c details

Preview

299 - -

"'

'

"

The following is a sample Question 2 similar to the one you will see when you take the test. You will first
see a prompt on your screen describing the task.
Do you agree or disagree with the following statement? Reading books about real events is
more valuable than reading fiction stories. Include specific reasons and examples to support
your answer.
You will then have 30 seconds to prepare a response and 60 seconds to speak your response into the
microphone. A sample outline of a response can be found below.

Sample Outline
Introduction:
Thesis statement
Body:
Reason 1
Topic sentence
Reasons, examples, and details
Reason 2
Topic sentence
Reasons, examples, and details
Thesis statement

Sample Response

Though I often enjoy reading fiction, I do agree that it is more valuable to read books about real people
and events.
Topic sentence
/

/1

Firstly, I think reading about real people and events teaches practical skills. For example, reading
about a businessman and his strategies could teach you more about business that a fiction story
would not.
Topic sentence

~
Reasons, examples, and details. -

Furthermore, I think reading is more interesting if you read about real things. Just last week I read a story
about a real person who traveled across the world in a hot air balloon. Because the story was real, it
was more interesting to me.

.,/
Reasons, examples, and details

~--

c:

"'

The following is a sample Question 3 similar to the one you will see when you take the test. You will first
see a short reading passage about a campus-related topic. You will have 45 seconds to read the
announcement.

--------

~
3z

University Athletics Department Announcement


We regret to inform students that the university will no longer offer intramural sports programs
effective next term. Despite the university's best efforts to support the program, interest in all
individual sports is currently very low. Thus, it is no longer able to sustain the program. Furthermore,
many of the faculty sponsors responsible for intramural sports have recently relinquished their
positions. Without sponsors, it is impossible to continue to offer these programs.

"'

After 45 seconds, you will be asked to listen to a conversation about the passage you just read. Read
the conversation below. Keep in mind that this conversation will only be heard on the real test and will
not appear on your screen.
W: Wow, I'm really glad that they finally decided to stop supporting those terrible intramural
programs.
M: Really? I'm surprised you would say that. I mean, weren't you a team leader for the intramural
volleyball team?
W: Yes, and that's exactly why I'm glad that they're stopping all of the teams. Since I was on a
team, I got to see first hand how bad the program had become. It was time to end it.
M: So, then, you agree with why they're stopping the program? I remember that the announcement
said that student interest in the sports was really low.
W: Yeah, and that's a main reason why the teams had become so bad. I mean, I had been really
interested in keeping the volleyball program going, but no one else seemed to want to. In fact,
I tried to recruit a lot of people to join teams, but they were all happy just playing for fun on the
sand volleyball court by the dorms. Now that they've officially ended the intramural program,
I'm thinking that I can convince more people to just play for fun by the dorms.
M: That actually sounds like fun. I'd be up for doing that. Don't you feel bad for the faculty sponsors,
though? The announcement said that a lot of them had given up on the program, but I know
a few that still really liked it.
W: I actually think it is better off this way. I mean, most of the faculty sponsors were professors,
and they're busy enough as it is. Even the sponsor for my old team complained that he didn't
'
have enough time for the team when he had to grade papers. So, really, it's better for the

LM!~;,"~;=""'-

-- -----

--- . __j
Preview

301 - -

After the conversation concludes, you will see a prompt on your screen similar to the one below.

The woman gives her opinion of the announcement made by the Athletics Department.
State her opinion and explain the reasons she gives for holding that opinion.
You will have 30 seconds to prepare a response and 60 seconds to speak your response into the
microphone. A sample outline of a response can be found below.

Sample Outline
Introduction:
Speaker's opinion
Body:
Reason 1
Topic sentence
Reasons, examples, and details
Reason 2
Topic sentence
Reasons, examples, and details
. Speaker's opinion

Sample Response

"'

The woman says that she agrees with the announcement from the Athletics Department stating that
intramural sports are being discontinued.
Topic sentence

She begins by stating that she agrees that interest in the program has been dropping lately. She points
out that she used to lead an intramural volleyball team, and she had trouble finding other people to
participate. She now believes it is best to stop the program so people can organize volleyball games for
fun.---+ Reasons, examples, and details
Topic sentence

Furthermore, she agrees with the fact that faculty sponsors have begun to quit their positions on
intramural teams. She says that this is better for the professors because they had often been unable to
spend much time on the team when they had other work, like grading papers.

/
l~easons,

examples, and details

The following is a sample Question 4 similar to the one you will see when you take the test. You will first
see a short reading passage about an academic topic. You will have 45 seconds to read the passage.
Dormancy
When faced with the many threats of the natural world, many animals enter a state of
dormancy to avoid danger. When animals become dormant, they enter a state of inactivity that
allows them to conserve energy. Many factors can lead an animal to enter a state of dormancy.
Some animals must become dormant due to an environmental strain-if the environment becomes
harsh, dormancy can allow an animal to survive. In addition, some animals may become dormant
when the risks of predation are higher. For example, some slow or defenseless animals must rely
upon the cover of night to avoid predators.

;j
3
z

Cl

After 45 seconds, you will be asked to listen to a lecture about the passage you just read. Read the
lecture below. Keep in mind that this lecture will only be heard on the real test and will not appear on
your screen.
W: While we've spoken at length before about animal defenses, we have yet to cover how animals
with limited defenses manage to protect themselves against harsh conditions and predators.
One strategy employed by many animals is called dormancy. I'd like to show you today how
one animal-the snail-uses dormancy to stay protected.
Especially in very warm climates; the snail must enter a state of dormancy during the daytime
to avoid the harsh conditions of the environment. One concern for the snail is temperature.
See-as I'm sure you're all aware, the snail is very slow. Now, for many other animals, it's easy
to move around during the day because they can quickly move to shade if they get too hot.
For the snail, this isn't the case. The snail solves this by remaining in the shade-and inside its
shell-all day in a dormant state. Then, when night falls, the snail has enough energy to find

food without the threat of overheating.

Furthermore, the daytime is considerably more dangerous for the snail, so it must remain
dormant during the day to avoid predators. The snail's movement speed-or lack thereofmakes it an easy target for predators. Rather than risk exploring the ground during the day,
it remains dormant until the sun goes down. The night affords the snail enough protection to
remain hidden from many predators. Thus, it can search for food in relative safety.

Preview

__ ,,.

-~---

_-

_________ ,___ . ----------.----

----

--'-'----~-----~----------- - - - - - - - - - - - - - - - - -

303 -

After the lecture concludes, you will see a prompt on your screen similar to the one below.
Using points and examples from the lecture, explain how the snail exemplifies the use of animal
dormancy.
You will then have 30 seconds to prepare a response and 60 seconds to speak your response into the
microphone. A sample outline of a response can be found below.

Sample Outline
Introduction:
Summary of the lecture's main point
Body:
Reason 1
Topic sentence
Details from the lecture
Reason 2
Topic sentence
Details from the lecture

Sample Response

Summary of lecture's main point


~

The lecturer uses the snail as an example to demonstrate the reading passage's claim that animals use
dormancy to avoid harsh environmental conditions and predators.
/ ' Topic sentence
The lecturer first states that the snail must remain in the shade and inside its shell during the daytime
because the environment is too harsh. More specifically, she states that the snail is unable to easily
move to shade to avoid getting too hot, so it instead remains dormant during the day and waits until
night to come out.

-----.-

Details from lecture

Topic sentence
~

The professor then discusses the fact that the snail must remain dormant during the day to avoid
predators. She explains that, because of the snail's slow movement, it is safer to explore the environment
I

late at night when the darkness can hide it. Thus, it remains in hiding during the day.

Deta!ls fr:Jrr: lecture

~1
l
I
i

The following is a sample Question 5 similar to the one you will see when you take the test. You will
listen to a conversation between two students about a campus-related problem. Read the conversation
below. Keep in mind that this conversation will only be heard on the real test and will not appear on your
screen.

I
I

'
.

W: Nathan! Hold on ... I wanted to congratulate you on making it to the championship game. You
must be so excited!
M: Well, I wish I were still excited. See ... I have a really serious problem that just came up today.
Hey, maybe you could help me out.
W: Sure, I'd be glad to. What happened?
M: Well, as you know, my football team's championship game is this weekend. And, of course,
the team is really counting on me to attend. But I got contacted by my top choice for university
yesterday, and they want me to come out for an interview this weekend. They seem really
interested in me, and I don't want this to affect my chances of getting in.
W: Wow, you certainly do have quite the predicament. Well, hey, why don't you just talk to the
university and let them know that you have a championship game this weekend? I'm sure they
would understand.
M: That was definitely the first thing I thought of, but I worry that ... well, if I try to get out of it, that
they'll think I'm not serious about going to their school. And that's the last thing I want to do.
W: True. I guess your other option then is to tell the team that you can't play on Saturday. That
way, you'll get to go to the interview.
M: Right. But ... no! to sound conceited ... but I know my team really needs me, and I don't
want to be the reason that we lose the championship game. I mean, I could end up letting the
whole school down!
W: That's true. Wow ... I wish I could be of more help, but I really don't know what you should
do!
M: Yeah, neither do I ...

'.

---------------'

I
I

I
I

After the conversation concludes, you will see a prompt on your screen similar to the one below.

The students discuss two possible solutions to the man's problem. Describe the problem.
Then state which of the two solutions you prefer and explain why.
You will have 30 seconds to prepare a response and 60 seconds to speak your response into the
microphone. A sample outline of a response can be found on the next page.

Preview

L - - - - - - - -----~~--~---'-~- - -

--------------- -----------

-- - ------ -

---- -

305 ----:

----

-~-

-- -

'

-~

Sample Outline
Introduction:
Summary of the problem
Suggestions
Your preferred solution
Body:
Reason 1
Topic sentence
Reasons, examples, and details
Reason 2
Topic sentence
Reasons, examples, and details

Sample Response

Summary of problem

The man states that he has been invited to his preferred university for an interview on the same weekend
that he is supposed to play in a championship football game. He can either ask the university to
change the date of the interview, or he can skip his football game. I believe that he should skip the
football game.
\.

\.
Suggestions

Your preferred solution

Topic sentence

For one, I believe that his interview is more important than the football game. It is true that this decision
would likely disappoint his team members and perhaps even the entire school, but this will still pale in
comparison to the personal disappointment he will experience if he does not get into his top-choice
university. He should do everything he can to make sure he gets in.

Reasons, examples, and details


Topic sentence

'
,~

I also think that this is the best solution because the university is unlikely to change his interview date.
Most university officials are very busy, and they do not have time to reschedule interviews. In fact, the
man is right that the universit'j;;,ight think he was not serious about attending if he tried to change the
date of the meeting.

Reasons, examples, and details

I''(;';(.

:!ii;t]'----

j .

~.ip-~

306

Preview

~ J

f-------------1!!1!!1!1111_1111___1___1 1__1 l_l !!!___l l _!l l_

!11111_!111
_ _ _ _ _""""""""-..:..............
__. ._. ._._ .._.---_
...
_ ._.111111111111
____ _

c:

zGl
The following is a sample Question 6 similar to the one that you will see when you take the test. You will
listen to a lecture about an academic topic. Read the lecture below. Keep in mind that this lecture will
only be heard on the real test and will not appear on your screen.
M: Among the many different aspects of a stage production, one of the most important
considerations is often ignored: stage lighting. When you watch a play, you probably don't
consciously notice it, though it has a profound effect upon the way you interpret a scene ..
Experts use theories of color and lighting position to gain these effects, which typically go
unnoticed by the audience.

1'

I~;

The color of the lighting can complement the actor's portrayal of emotions and actions,
and can even provide a certain context that the actors themselves cannot convey. Let me give
you an example that exemplifies both of those qualities. In one of my favorite scenes from the
theater, the tension in the scene has slowly been building as the characters begin to learn
about a crime that has happened. But, see, .in the scene, the colors of the lights slowly get
redder. So, even though the characters aren't standing there stating that the scene is getting
.more tense, the audience can tell this from the intensity of the lights.

1:'.

In addition, the position of the lighting can affect how the audience sees a scene. This is
especially apparent in the use of lights to make certain parts of the stage bright while others
remain dark. Now, in that same scene I've just described, the lighting technician used a type
of lighting known as down lighting-light that points straight down. This type of lighting is used
to create the illusion of depth~in this case, it was used to make two characters seem farther
away from each other than they were. This helped the tension that led into the climax, where
one of the characters was attacked while the other wasn't looking.

After the lecture concludes, you will see a prompt on your screen similar to the one below.
Using points and examples from the lecture, explain how stage lighting can be used to control
the audience's understanding of a scene.
You will have 30 seconds to prepare a response and 60 seconds to speak your response into the

microphone. A sample outline of a response can be found on the next page.

l
Preview 307 - -

:1. '
'

Sample Outline
Introduction:
Main topic
Body:
Key point 1
Topic sentence
Reasons, details, and examples
Key point 2
Topic sentence
Reasons, details, and examples

Sample Respon!SC

Main topic

The professor states that stage lighting can be used to cqntrol the audience's understanding of a scene.
This can be achieved through the use of color and the direction of the lighting.

//1

Tooic sentence
'

The speaker first says that color can be used to change how the audience interprets a particular scene.
He uses an example of a scene from one of his favorite plays. The scene was very tense, and the director
used red lights that got progressively brighter. This added to the feeling of tension in the scene.

~
Tc;pic

serilnnct~

Fleasons. exampl2s, ancl cleta1ls

;"-_

The direction of the lighting can also be used to control how the audience interprets a scene. The
speaker discusses down lighting, which is lighting from directly above. This type of lighting creates the
illusion of depth, and in his example, the professor says that it was used to make two people seem far
away from each other. _
',)

308

Preview

-,--,
,_ .-,'.--'
'":f'-/}f;;r.~-'\,__ ,:-~,'.~:~[i~'J"-,

. f[:iii,.);

f>~~[S~~R~e,r,;1.',;;; lridependentspeaking:.
<,j,

:;fi ' , ~~C:hapter2


.

:-.~;r;;/.,:_:,_- - ,_-_.,,,, _-

Organizing Speech
Integrated Speaking:
Synthesizing Information

'chapter 3

Integrated Speaking:
Stating Opinions and Summarizing

Vocabulary Review 1
Vocabulary Review 2

i_,
--~c:--__ '.:_------=-=--::_-~_:

,;-.i'-_.,

_ ___: __

===-------------------

'\'
,,... ,
1~;'

~:

m; """"''' ":... .'

"'===,="'c=='' "'==" "' " ........ ;,, ..

Necessary Skills
ug~ll\lillollil'#l".!S[!i!~i!~M.,,i~l!a~l:!.~.li'!.l~~'f'ielH<U'"i-"!~~-";:;f,-<lJ'>:'-'--l;-1i;,.,,

'

Describing a personal experience or expressing a personal preference


Organizing ideas
Expressing a clear topic statement and supporting points
Speaking clearly and accurately with knowledge of grammar, vocabulary,
and pronunciation

Strategies
Though preparation time is limited in the speaking portion of the test, it is nevertheless
important to use this time to plan the organization of your response. This way, your
response will be more relevant and coherent. An organizational process for preparing your
speech is detailed below. Jn each step, there are certain things to keep in mind.
Process , .,.,;.

IJ~ -

Read the prompt and


understand the task.

_S_t_ra_te_g_y_-~--"''-""~'_<_ii~''.'~;2:~;;j

;_
..._._.._ . _._<___

Be sure that you understand the question and what it


requires you to do.

ecide on the main id~a or choose a position. Use the .

~ - ' ----
:J ~

dentify the main idea


and create a topic
[ statement.

elevant parts of the prompt in making your topic


tatement.

----------~----

. t
d
Brains orm an s
. 'd
suppo rt 1ng 1 eas.

-----

Quickly think of supporting ideas from your own


.
.
experiences. Choose ideas that most clearly support
.
your topic statement.

-----------------

r-.-~-~---J IA~an~e y~~~~:~~~utting th~;n order from most

I urgarnze t e

eas.

---------~-----

important to least important.


--------------------

--~

Independent Speaking: Organizing Speech

~~~~

-----------

311 - -

Practice 1 - Personal Experience .


~Step

'

Read the prompt below. Write down your answer and related key points in the blanks.
!- ~;;~~~;;b~~-~~;~~~~h~--h~~1,~;~;d-~;~b~~~~~--~h~~;~~;~;~ci~;.1~~1~ci~'";;~~~i~-

reasons and examples to support your answer.

\.,..,_ .. ~-..,.,,,,.,_,,,,"'~"''-'"'',_,""i>''"~'"

~'""""''"'"''-'-='"""~u<Y~_,,...,_.,,,"'"'"-"""~ ..,.,,....,.,..,.,.,.,._,,._,~,.~.,,.,._ _ _ _ ..,,,_,_.,...,_,.,,,,.__,~~'~-'-'~~=<EOvk..._.."

One person who has helped me become who I am today is_ _ _ _ _ _ _ _ _ _ _ __


Reason 1:

------------------------------~

Reason2: ______________________________

Read the related ideas and expressions below. Add at least two of your own.

hard work:
effort, dedication, enthusiasm, ____________

good at something:
talented, gifted, skilled, _ _ _ _ _ _ _ _ _ __

dedication:
the hard work and effort
spent Off something
talented:
very good at doing
something
encourage:
to give hope or support
generous:
giving often to other people
volunteer:
to do work without getting
paid

encourage:
suggest, persuade, convince, ____________

giving:
generous, kind, charitable, ______ , ______

- - - - - - - - - - - -- - - - - - - - - - -

------ -

---- -

- - - - - - - - - - - - - - - - - - - -- - - - -

(I Listen to a sample response. Write down any useful expressions.

"'"**'

Gl

---

------------------------------

-!(!!'

Now create your own response using words and expressions from Steps 1 and 2. Use the
prompts below to help you.
One person who has helped me become who I am i s - - - - - - - - - - First of all,
- - - - - - - - - - - - - - - - - - - I used to ___________
________________ . But _______________
and I - - - - - - - - - - - - - - - - - - Second, _ _ _ _ _ _ _ __
- - - - - - - - - - - - - - - - - - - B e c a u s e of __________ ,

- ~Step 4
Read the response you wrote above out loud. Try to read slowly and clearly. Practice saying
the whole response several times. Then close your book and say the response without looking
at the words.
independent Speaking: Organizing Speech

---~--

- - - - - ---

313

~.

---------------------- - - - - - -

------ - ---- - --- - - - - - - -

--------

--

-------- -

Practice 2 - Personal Preference


;~-I

~Step

Read the prompt below. Write down your answer and related key points in the blanks.

--Des~rib~;;~;~~~~ kno:i-~h~do~a j;~~~~ir;,Why d;yo~~dmir~~h;-J

person's job? Include specific reasons and examples to support your answer.

. . . . . -----..

---------->->-~

--~---H----~-='"'"""

-,

_ _ _ _ _ _ _ _ _ _ _ _ _ _ _ _ _ _ _ _ _ _ _ _ has a job that I admire.


Reason1: ______________________________

Reason2: - - - - - - - - - - - - - - - - - - - - - - - - - - - - - -

Read the related ideas and expressions below. Add at least two of your own.
Related Ideas and Expressions

have a good opinion of someone:


respect, admire, think highly of, ____________

brave:
courageous, fearless, daring, ____________

help:
aid, assist, lend a hand, ____________

work:
compensation, paycheck, salary, ____________

314 Chapter 1

_-,_,.,

admire:
to have a feeling of respect
for someone
courageaus:
very brave and willing to do
dangerous things
aid:
to help someone
compensation:
payment given to you for
work you've done

.-;~

~Step

n Listen to a sample response. Write down any useful expressions.

i@IM

------------------11~mmm111m11_

--------------------------.

-------------------------------

--------------------~-------------

---------

~S$ep 3
Now create your own response using words and expressions from Steps 1 and 2. Use the
prompts below to help you.
- - - - - - - - - - - - - - - - - - - - - - h a s a job that I admire. I
admire this job for two reasons. F i r s t , - - - - - - - - - - - - - - - - - - - - _________________ for the work t h a t - - - - - - - - - ________________ .Second, ______________
_______________ wanted t o - - - - - - - - - - - - - - -
Being a ______________ is how ______________

Plus,-------------------

~Step

Read the response you wrote above out loud. Try to read slowly and clearly. Practice saying
the whole response several times. Then close your book and say the response without looking
at the words.
Independent Speaking: Organizing Speech

--

... ,

----------~----

315 - -

'(

Practice 3 - Personal Experience

~.
f

~Step

Read the prompt below. Write down your answer and related key points in the blanks.

--W~~-;;meone you feel comfortabl;talki~~ to-;bout ;;;ble~s?~:;~j""-1


comfortable with this person? Include specific reasons and examples to support your
answer.

I
J

..,___,.,...,......,.,._,.,_."""""'-=-=.._ ......,..,,..,. ...........,..,,..,,,,.,....,,.. _ _ _ _ _ _~--,..,.......--=--~~-.""'""'"" .......,...............'""""'~"""'""'"'''~=~-~,,~._,,/

I feel comfortable talking with _ _ _ _ _ _ _ _ _ _ _ _ _ _ _ _ about my problems.


Reason 1:

-------------------------------

Reason2: ______________________________

Read the related ideas and expressions below, Add at least two of your own.
Related Ideas and Expressions

comfort:
comfortable, relaxed, at ease, ____________

problem and solution:


I.

j.;

issue, concern, resolution, ____________

types of knowledge:
I

experience, fact, information,

!'.

related:
similar, alike, connected, ____________

comfortable:
relaxed or at ease
patient:
able to deal with difficult
situations without becoming
upset
solution:
a way to fix a problem or
bad situation
experience:
knowledge gained through
participation or work
similar:
have some of the same
parts or qualities

~Step

n Listen to a sample response. Write down any useful expressions.

i@ibl

1114

j;;)

I'." .
.:;,

I"
[' ,

:; ..;
I:.-,,.

I,

IL,.,
I

.;,

~Step
.;.

Now create your own response using words and expressions from Steps 1 and 2. Use the
prompts below to help you.
I feel comfortable talking with _____________ about my problems. First,
,
--~---------is _____- ' - - - - - - - - - - - - - - -

__________ is __________ and ___________


when I tell _ _ _ _ _ _ _ _ _ _ _ _ _ _ my problems. That way, _ _ _ _ _ __
_____________ . Second, _________________

so _____________ knows what to do.

~Step

I,

Read the response you wrote above out loud. Try to read slowly and clearly. Practice saying
the whole response several times. Then close your book and say the response without looking
at the words.
Independent Speaking: Organizing Speech 317

:__,. -

-_.__::.....:..___.

_:..::..:-.:.::~,::~~----.---_

__
.

_ _ ,,

___

_,_:

________ ---

-----

...,..

----- -------

-----------------------

--j

Practice 1 - Personal Preference


Step 1
Read the prompt below. Write down your answer and related key points in the blanks.

Sh~"~ld;t-u-de_n_t;h-a-ve_t_o_~ay to attend

university, or should

~~rnments provide free

education for students? Include specific reasons and examples to support your answer.

----~----

..-------------

-------

I feel that _ _ _ _ _ _ _ _ _ _ _ _ _ _ _ _ _ _ _ _ _ _ _ _ _ __
Reason 1: One reason I think this i s - - - - - - - - - - - - - - - - - - - - -

Reason 2: Another reason I feel this way i s - - - - - - - - - - - - - - - - - - -

Read the related ideas and expressions below. Add at least two of your own.
Related Ideas and Expressions
money:
funds, finances, currency, ____________

worry:
fret, be anxious, be concerned, ____________

paying attention:
focus, concentrate, think about, ____________

education:
school, university, teach, ____________

318 Chapter 1

funds:
money
responsibility:
.being required to meet the
'expectations of others
available:
able to be used
fret:
to worry about something

concentrate:
to focus your attention on
one thing

~Step

--------

Listen to two sample responses. Write down any useful expressions.

_,

__"'___

Mt!MM

----

I
I

'
'I ,I'
I
I

~;Step

Now create your own response using words and expressions from Steps 1 and 2. Use the
prompts below to help you.

I think that students _ _ _ _ _ _ _ _ _ _ _ _ _ _ _ _ _ _ _ _ _ _ __


for two reasons. F i r s t , - - - - - - - - - - - - - - - - - - - - - - - - - '
Since _ _ _ _ _ _ _ _ _ _ _ _ _ _ _ _ _ - - - - - - - - - - - _ _ _ _ _ _ _ _ _ _ _ , If universities did not charge, they _ _ _ _ _ _ _ _ __
____________, , S e c o n d , - - - - - - - - - - - - - - - - ______. S t u d e n t s - - - - - - - - - - - - - - - - - - - - - - When students do this;they _ _ _ _ _ _ _ _ _ _ _ _ _ _ _ _ _ _ _ _ _ __

~Step

Read the response you wrote above out loud. Try to read slowly and clearly. Practice saying
the whole response several times. Then close your book and say the response without looking
at the words.
Independent Speaking: Organizing Speech

319 - - .

Practice 2 - Personal Preference


~Step

Read the prompt below. Write down your answer and related key points in the blanks.
t

--o;;-;~~;;~fer.to ~~;;;dyo~;f~ee~;;;; outdoors-~ ind~ors?i~~iude~;~;~;ft~ r:a~;;;,d--.. -J

examples to support your answer.

"<~=~=""'""""'"'....,.v'~""""'L'"''"'""'"-'"~~...,....,..~,-.~"'6_,,_...,.,.,...,

..d__.,,.==-='1",,_,,>...,._

_,..,.._...,.~...,...,_-,.,,,_,,,...,,..._ .,.,~,,._..,,,_,.,_...._

... ,......,,,.~..,~L__..="""""'m',>

I prefer spending my free t i m e - - - - - - - - - - - - - - - - - - - - - Reason1: ____________________~---------~

;,

'

Reason2: ______________________________

Read the related ideas and expressions below. Add at least two of your own.
Related Ideas and Expressions

outdoors:
fresh air, nature, scenery, ____________

outdoor activities:
hiking, jogging, playing sports, ______ , - - - - - -

allergies:
sneeze, itch, runny nose, ______ , ______

get better:
relief, cure, treat, ______ , ______

fresh air:
the air outside
hike:
to walk in the mountains
allergy:
a sensitivity to certain
substances
sneeze:
a renex that suddenly
expels air from your nose
and mouth
relief:
a relaxed feeling that is
experienced when
something u~pleasant ends

._,7

~Step

n Listen to two sample responses. Write down any useful expressions.

t@ifij

---..--~--""----~'"~"--------------,&iiliil

~Step

Now create your own response using words and expressions from Steps 1 and 2. Use the
prompts below to help you.
'!'

'

'

I prefer spending my free t i m e - - - - - - - - - - - - - - - - - - - Onereasonisbecausel ________~----------------

This ______________________________ .
I c a n - - - - - - - - - - - - - - - - - - - - - - - - - A second reason is
because ______________________ . I like to _ _ __
- - - - - - - - - - - - I enjoy __________________

~Step

Read the response you wrote above out loud. Try to read slowly and clearly. Practice saying
the whole response several times. Then close your book and say the response without looking
at the words.
lnd~pendent Speaking: Organizing Speech 321 -

\,'

Practice 3 - Personal Preference


~Step

Read the prompt below. Write down your answer and related key points in the blanks.
-~-- ..-~~-_._.._,,.. ...,,.....,..,,_.,.,,. ......,.., ~

.r

'

__.._,_..,........ ~- .....-~........-

Do you prefer classes with a lot of discussion or classes without discussion? Include
specific reasons and examples to support your answer.

-..,...,..,,~<~~<n"'"-"'"""".,_,,,.,...,,._~...,~~._~.,,..,...,.-_,...,=z-~-,.....,...,.~,--,.---~=,,.~~==~'"'~....,.,,.,...,~=""-H~~.-~,,_,.,.,.../

I prefer classes ____________________________


Reason1: ______________________________

Reason2: ______________________________

Read the related ideas and expressions below. Add at least two of your own.
Related Ideas and Expressions

class discussion:
talk, debate, dialogue, _ _ _ _ _ _ _ _ _ __

interesting:
fascinating, stimulating, ____________

organized:
planned, arranged, - - - - - - - - - - - importance:
central, main, chief, ____________

stimulating:
making you feel interested
fascinating:
inspiring great interest or
attraction
constructive:
helpful or useful
organized:
planned or arranged
carefully
central:
main: being at the center

~Step

n Listen to two sample responses. Write down any useful expressions.


--,,.-=.,--.~,.,,..,--. . . ~. . ---'""'-""--=.......--

_ ...

l@j"fl

..._"'"""'"--~-~---..."""'-~=..----=-~. ~'_&h1 &.ll~. ._

---------------------------------------------------------- -- 1

-------

-------------------- - - - - -- I
l

~Step

Now create your own response using words and expressions from Steps 1 and 2. Use the
prompts below to help you.
I prefer classes--------------------~ The fir-st reason is that they a r e - - - - - - - - - - - - - - - - - When you talk about a topic,
--------------------------~This _____

- - - - - - - - - - - - - - - - - - - - - - - - S e c o n d , discussion classes
---------~-----------I like classes ________

- - - - - - - - - - - - - - - - W i t h discussions,-----------

__________.so ______________________
~Step

Read the response you wrote above out loud. Try to read slowly and clearly. Practice saying
the whole response several times. Then close your book and say the response without looking
at the words.
I

I
Independent Speaking: Organizing Speech _323 -----'.

lI.~
~!

':.

l\lecessary Skills
!,\M.f.'l.'.K!!!l~~Wl'f.1~11~~;:,1r.v:".''!'!">D<11t;"'.''l-\e<:.o;".}:r<.\''''

.-,:.,.,

Understanding information in reading and listening passages


Taking notes on important information and using this information in your spoken
response
Synthesizing background information with more specific information
Synthesizing the information given in the reading and listening; using the points in
the listening to highlight principles or differences in the reading
Recognizing a speaker's purpose and attitude
Paraphrasing information

Strategies
An organizational process for responding to a prompt based on integrated material is
detailed below. In each step, there are certain things that you need to keep in mind.

Process

=i ,_[______-'--_s_t_.ra_te_g_y~--~-"",~_::~'"-',ii

l::ead and listen.

Read the prom pt and


understand the task.

Take notes on important information in both the reading


and listening passages.
Identify relationships between information in the
listening passage and the reading passage. What
aspects of each does the prompt want you to discuss?

---------11-----------

Arrange the ideas from the listening and reading


Organize the ideas.
1; passages. Think of a topic sentence that reflects the
11 information.
------------------------.! L ------.. .------------

.
I

Integrated Speaking: Synthesizing Information

325

--~.-

Practice 1 - Reading and Conversation.


~ St~p 1
Read the following announcement. Write five keywords or key phrases that would be useful
in explaining the information to someone else. While reading, try to guess what the conversation
will be about1

Announcement from the Athletics Department


We regret to announce that the university will no longe~ be selling
basketball tickets at a reduced price for students. This policy takes
effect immediately. This is necessary primarily because we are no longer
making a profit on the games. In the past, it was possible to sell reducedprice tickets and still make money, but this is no longer possible.
Moreover, ending reduced pricing will help us raise the additional money
necessary to improve the team's facilities, which are currently in very
poor condition.

regret:
to feel unhappy that
something happened
reduced:
made smaller In size or
amount
policy:

a rule or set of rules


moreover:
additionally; furthermore

condition:
the physical state of
something

:r :.
'- -.

,...

KeyViords/
' Key Phrases
..

_,-,

Cover the passage and look at the keywords and key phrases only. Restate the passage in your
own words.

~Step

n Now listen to a conversation related to the passage in Step 1. As you listen, take notes on
important information. Write down five keywords or key phrases that would be useful in
explaining this information to someone else. 'MW*
term:
a semester at a school or
universlt)'
program:
a group of activities that
work toward the same goal
arena:
an enclosed area designed
to hold events for large
crowds
outdated:
very old and no longer
suitable for use
teak:
a hole that allows water to
pass through

I.
I

"l'''

Restate what you heard in the conversation using the notes or keywords you wrote above.
~Step

Read the prompt below. Circle the most important ideas in your notes from both the reading
and the listening. Write down the main points you need to speak about.
The man expresses his opinion of the announcement made by the. university's athletics
department. State his opinion and explain the reasons he gives for holding that opinion.

Opinion:

The man thinks it is _ _ _ _ that the university-------------

Reason1: - - - - - - - - - - - - - - - - - - - - - - - - - - - - - Detail:
Reason2: - - - - - - - - - - - - - - - - - - - - - - - - - - - - - Detail:

Integrated Speaking: Synthesizing Information

327 - - ' .

~Step

n Listen to a sample response. Write down any useful expressions.

iMf!IM

=-==~===-==--====--~- =--=--=-~1

-~=

~Step

=--=

==

-= =-= == === =- =- - =_ I

Now create your own response using words and expressions from Steps 3 and 4. Use the
prompts below to help you.
The man thinks it is good that the u n i v e r s i t y - - - - - - - - - - - - - - - _______ .This is because, f i r s t , - - - - - - - - - - - - - - - - - - _ _ _ _ _ .Hesaysthat _______________________
_ _ _ _ _ when ___-,-___________________ . Second,
it will help t o - - - - - - - - - - - - - - - - - - - - - - - He also thinks
that the team n e e d s - - - - - - - - - - - - - - - - - - - - - - - - since-------------------------------

"-;.r 5+m.,
t.'l;;;r'

i:;

l_;i

Read the response you wrote above out loud. Try to read slowly and clearly. Practice saying
the whole response several times. Then close your book and say the response without looking
at the words.

Practice 2 - Reading and Conversation


~Step

Read the following announcement. Write five keywords or key phrases that would be useful
in explaining the information to someone else. While reading, try to guess what the conversation
will be about.
From the Music Department
The music department has canceled the planned morning section
of Music Theory (MT 102) for the upcoming semester. The department
had intended to add an additional section of this popular class.
However, the department had to cancel its plan after being unable to
find a professor. There are not enough professors available to teach all
of the sections. Second, there is not enough vacant space available in
the music building in the morning for one more section.

section:
a class that meets at a
particular time
upcoming:
happening in the near future
Intend:

to plan to do something
addltlonal:

extra

vacant:
empty

~Keywords/

Key Phrases

---------~

Cover the passage and look at the keywords and key phrases only. Restate the passage in your
own words.

lntegiated Speaking: Synthesizing Information

___
,

~----------

329 _ _, _

.l

~Step

n Now listen to a conversation related to the passage in Step 1. As you listen, take notes ~n
important information. Write down five keywords or key phrases that would be useful in
explaining this information to someone else. iiiffiildol
. hire:
to ask someone to work for
you
repeatedly:
many times; often
hold:.
to organize or arrange
something

consult:
to ask for advice from

someone
express: ,_ -
to articulate a feeling or
opinion

i''

Restate what you heard in the conversation using the notes or keywords you wrote above.
:: i-

~Step

Read the prompt below. Circle the most important ideas in your notes from both the reading
and the conversation. Write down the main points you need to speak about.
The man expresses his opinion of the announcement made by the university's music
department. State his opinion and explain the reasons he gives for holding that opinion.

Opinion:

Detail:

Detail:

;.:

fl
..

[
[

Listen to a sample response. Write down any useful expressions. i@liijl

--------------------~------

---

-----~-------

--------------

~-------------

------------------------------------------

\4,~}:q;/.}:

The man ______________ with the university's plans to _ _ __

_y::;'k

)
J

.,),!;,, ____________________ .He thinks _________


.. ,.: .{'.-,
-

,)\;,' _ _ _ _ _ , first, because he thinks the university needs t o - - - - - - - - - - - _____ . He believes that _ _ _ _ _ _ _ _ _ _ _ _ _ _ _ _ _ _ _ __
:-.

-_ ..

--------Second, he does not understand w h y - - - - - - - - - - - ___________ .He says that ___________~-----does not need to be held in ___________ because--------- - - - - - - - H e knows t h a t - - - - - - - - - - - - - - - - - -

~Step

l3

Read the response you wrote above out loud. Try to read slowly and clearly. Practice saying
the whole response several times. Then close your book and say the response without looking
. at the words.
Integrated Speaking: Synthesizing Information

331

--~:'

Practice 3 - Reading and Conversation


~Step

Read the following announcement. Write five keywords or key phrases that would be useful
in explaining the information to someone else. While reading, try to guess what the conversation
will be about.
English Department Announcement
For the first time, the university will offer a new night class called
Survey of English Literature next semester. It will be the first of our core
classes being offered at night. The change is designed to meet the
needs of students who wish to take classes in the evening. Second, by
adding this new class, it will expand the offerings of the literature
department, allowing the department to offer more classes to attract a
growing student base.

survey:
a general summruy of a
. subject
core:
the main part or the
required part
offer:
to provide a product or
service
expand:
to make something larger
base:
a group of similar people

i<iiyYiiircls7
.Key Phrases

Cover the passage and look at the keywords and key phrases only. Restate the passage in your
own words.

I,

[
~Step

n Now listen to a conversation related to the passage in Step 1. As you listen, take notes on
important information. Write down five keywords or key phrases that would be useful in
explaining this information to someone else. liZf!ifaj
none too soon:
just at the right time

::e

packed:

"'3

very full

especlaily:

"'

particularty

.,,

major:

a student who is stud~n9 a


particular subject at a
university
advanced:
difficult or complex

"'"'

ffi...

'

Restate what you heard in the conversation using the notes or keywords you wrote above.
~step

Read the prompt below. Circle the most important ideas in your notes from both the reading
and the conversation. Write down the main points you need to speak about .
. . ..... ,. .... ,. .. . .........,....- ..... -......., ...........\
f'~'

I'
t

The woman expresses her opinion of the announcement made by the university's English
department. State her opinion and explain the reasons she gives for holding that opinion.

............................................................... " ' '

.... .,..........................................,.)

Opinion: The woman thinks it i s - - - - - - - - - - - - - - - - - - - - - - Reason1: _____________________________~


Detail:

Reason2: __~---------------------------

Detail:

I
Integrated Speaking: Synthesizing Information

'

333 --.-'

~.Step

Listen to a sample response. Write down any useful expressions.

lii!f!S@+

--------------------------------- ------------------

~Step

Now create your own response using words and expressions from Steps 3 and 4. Use the
prompts below to help you.

The woman thinks it i s - - - - - - - - - - - - - - - - - - - - - - - - - - - - - - - - - - - - - - - First, she says there are not enough _ _ _ __
- - - - - - - - - - - - - - - - - - - - - She mentions that her night class
\---

- - - - - - - - - - - - - - - - - - - - - - S e c o n d , she likes the idea of a


- - - - - - - - - - - - - - - - - - - - - - - S h e says she struggled to

>Step 6
'

Read the response you wrote above out loud. Try to read slowly and clearly. Practice saying
the whole response several times. Then close your book and say the response without looking
at the words.

Practice 1 - Reading and lecture


~Step

c:

~z

Read the following passage. Write five keywords or key phrases that would be useful in
explaining the information to someone else. While reading, try to guess what the lecture will
be about.
Product Classification

classify:

Companies must carefully think about the price and appearance of


their products when marketing them. This is because consumers use
price and appearance to classify products. First, higher priced products
tend to be seen as a luxury. Conversely, the lower the price, the more
likely consumers will see the product as a necessity. Second, consumers
will purchase well-designed and attractive products for decoration and
eniertainment. On the other hand, consumers see less attractive products
as simply utilitarian.

,,:

zGl

to place things into groups


or categories
luxury:
an indulgence rather than a
necessity
conversely:
on the other hand
necessity:

something that is needed


utllltarlan:

having a useful function

': ..\

'

, KeYv/~rds/ .
Key Phrases
'.
!'.'- -,. '' ;,-._, _,
\'--,

Cover the passage and look at the keywords and key phrases only. Restate the passage ih your
own words.

Integrated Speaking: Synthesizing Information

335 .._"

1
,

~Step

(I Now listen to a lecture related to the passage in Step 1. As you listen, take notes on important
information. Write down five keywords or key phrases that would be useful in explaining
this information to someone else. @!@
commodity:
a product that can be
bought and sold
extraordinarily:
extremely; very
appealing:
attractwe or pleasing
bulky:
very large and heavy
steak:
smooth and glossy

Restate what you heard in the lecture using the notes or keywords you wrote above.
~Step

Read the prompt below. Circle the most important ideas in your notes from both the reading
and the lecture. Write down the main points you need to speak about.
Explain how the example of the television supports the theory of product classification.

Classify products by. _ _ _ _ _ _ _ _ _ _ _ _ and _ _ _ _ _ _ _ _ _ _ __


Example:
First way:

price:---------------------------

Example:

television was ___________ ,then _____________

Second way: a p p e a r a n c e : - - - - - - - - - - - - - - - - - - - - - - - - Example:

television was ___________, n o w - - - - - - - - - - - -

- ,>:.

"'0~
z

.
I

"'

~Step

(I Listen to a sample response. Write down any useful expressions. i!!SM

----------------------------

.
I.
I

I~-

---~-----------

------------------ -

.'C"'----------~----------------------__,,

~ Step.5

Now create your own response using words and expressions from Steps 3 and 4. Use the
prompts below to help you.
The lecture is about how consumers will __________________
and the speaker uses - - - - - - - - as an example. The professor shows that _____
_ _ _ _ _ , but w h e n - - - - - - - - - - - - - - - - This supports the idea
that - - - - - - - - - - - - - - - - - - Second, the lecturer says that the first
-----------------~Now

_____________

_____ and people _ _ _ _ _ _ _ _ _ _ _ _ _ _ _ _ _ . This supports the


author's claim t h a t - - - - - - - - - - - - - - - - - - - - - - - - - -

~Step

Read the response you wrote above out loud. Try to read slowly and clearly. Practice saying
the whole response several times. Then close your book and say the response without looking
at the words.
Integrated Speaking: Synthesizing Information

337 - . i

_ _ _ _ _ _ _ _ _ .:..:L.:

Practice 2 - Reading and lecture


~Step

Read the following passage. Write five keywords or key phrases that would be useful in
explaining the information to someone else. While reading, try to guess what the lecture will

,,

~a~~

Echoic Memory
Echoic memory is an auditory form of sensory memory. It consists
of a brief echo that continues to be heard in the mind after an actual sound
has been heard. It has two important qualities. First, it is very short in
duration. In fact, echoic memory is one of the briefest forms of memory,
lasting for only about three or four seconds. Second, because it is an
exact copy of the sound that was heard, echoic memory is very accurate
for that limited period. It allows for direct and accurate reproduction of
information.

auditory:
relating to sound and
hearing
sensory:
reating to the senses
echo:
a sound that Is repeated
durationi
_,.
the period of time that
, something lasts
reproduction:
a copy of something

'

------,.------------------------------,

Keywords/
, Key Phrases

{;:!-;'..
;l'f~/-,

.,

! '
k

f,-

,,

-----'----------------------------------'

'

Cover the passage and look at the keywords and key phrases only. Restate the passage in your
own words.

~Step

n Now listen to a lecture related to the passage in Step 1. As you listen, take notes on important
information. Write down five keywords or key phrases that would be useful in explaining
this information to someone else. @iMii
illustration:

an example
recite:
to say something out loud
last:

to happen for a period of


time

Keywords/
Ke~ Phrases'

accuracy:
the ability to do something
correctly
latter:

the second of two things


mentioned

,_J;.>

Restate what you heard in the lecture using the notes or keywords you wrote above.
~Step

3.

Read the prompt below. Circle the most important ideas in your notes from both the reading
and the lecture. Write down the main points you need to speak about.

I
I

The professor gives an example of recalling a new telephone number. Explain how it
illustrates the features of echoic memory.

Echoic memory is
Example:
First feature:
Example:
Second feature:
Example:

1nteg~~ted Speaking: Synthesizing inforrnation

--,-_---.,----- ------,----,-;

339

--1

~Step

n Listen to a sample response. Write down any useful expressions.

~Step

l@jiiJ

Now create your own response using words and expressions from Steps 3 and 4. Use the
prompts below to help you.
The lecture uses the example of _ _ _ _ _ _ _ _ _ _ _ _ _ _ _ _ _ _ _:._
to d e s c r i b e - - - - - - - - - - - - - - - - - - - First, the professor says that
- - - - - - - - - - - - - - - - - - - T h a t is because, when you hear it, you
- - - - - - - - - - - - - - - - - - - - T h i s supports the author's claim that
- - - - - - - - - - - - - - - - - - - - A l s o , the professor says that you can
- - - - - - - - - - - - - T h i s is, again, because of ___________
This also backs up the claim from the reading t h a t - - - - - - - - - - - - - - - -

~Step

Read the response you wrote above out loud. Try to read slowly and clearly. Practice saying
the whole response several times. Then close your book and say the response without looking
at the words.

I
'

'

Practice 3 - Reading and Lecture

'

'

'

~Step

'
''

Read the following passage. Write five keywords or key phrases that would be useful in
explaining the information to someone else. While reading, try to guess what the lecture will
be about.

unconscio:uS: -'::
- lacking awareness

Unconscious Competence

The concept of unconscious competence refers to the final stage of


learning when a learner has practiced a new skill so extensively that it
becomes natural to them. One feature of unconscious competence is
that the learner can use the skill without thinking about it. Thus, the skill
becomes "second nature" in that it begins to feel natural to perform this
skill. Moreover, with unconscious competence, the learner can use the
skill while doing other things. His or her ability to use the skill becomes
automatic and does not require significant concentration.

competence:
' the ability to do sornething
correctly
extensively! . . ... . .:,
In a way that 1svery.large, lo,.
amount or degree' ' " "Y'

.
'

I
[

',-.,;-i~>',l

. automatic:.
,
done without thinking
significant: . , . , , .
very largo or irnportant .

--~------------------~------------

'<'

Cover the passage and look at the keywords and key phrases only. Restate the passage in your
own words.

Integrated Speaking: Synthesizing Information 341 - . ' ' -

,L.'

~Step

n Now listen to a lecture related to the passage in Step 1. As you listen, take notes on important
information. Write down five keywords or key phrases that would be useful in explaining
this information to someone else. @i!d:i

\.-': '--!

i<eiJ~td~j>,:,:
KeY. Phrases

achieve:
to reach a certain result or
succeed In a task
accomplishment:
the completion of a difficult
task
struggle:
to have problems doing
something
simultaneously:
at the same time
perfection:
a state of being without
error or defect

'------'-----------Restate what you heard in the lecture using the notes or keywords you wrote above.

n- ' '

Read the prompt below. Circle the most important ideas in your notes from both the reading
and the lecture. Write down the main points you need to speak about.

fi' (

L-:,.
-1.;

The professor discusses the process of learning to play an instrument. Explain how it
illustrates the concept of unconscious competence.

Unconscious competence i s - - - - - - - - - - - - - - - - - - - - - - - - Example:


Feature 1:
Example:
Feature 2:
Example:

"'
~

zG'I

(I Listen to a sample response. Write down any useful expressions. +@jiajl


- - - .. '.... ~.... "-----~-------....----~------.....~ ....- ...............- ~----~..--....-~JlM.Vll ....

G'I

~Step

Now create your own response using words and expressions from Steps 3 and 4. Use the
prompts below to help you.
The professor talks a b o u t - - - - - - - - - - - - which happens when _ __
- - - - - - - - - - - - - H e uses the example of __________~-- - - - - - - - - - - A t first, you have to ______________
But when you become skilled, y o u - - - - - - - - - - - - - - - - - - - This
demonstrates the concept introduced by the reading passage that says - - - - - - - - - - - - - - Next. the professor says t h a t - - - - - - - - - - - - - - - when you h a v e - - - - - - - - - - - - This supports the claim from the passage that

~Step

Read the response you wrote above out loud. Try to read slowly and clearly. Practice saying
the whole response several times. Then close your book and say the response without looking
at the words.
Integrated Speaking: Synthesizing lnfonnat1on

343 --.'

Necessary Skills
~r.raM1.~m~~<'<'"..r,;;oi;<E{;~n~11''-'N':-;;:-;--,_~;

,- :;;"

Understanding the key information in listening passages


Taking notes on important information and using this information in your spoken
response
Paraphrasing information
Expressing an opinion or preference
Supporting an opinion with reasons or examples

Strategies
An organizational process for preparing your speech is detailed below. In each step,
there are certain things that you need to keep in mind.
_P_ro_c_es_s__-"'"'1

,_J_ _ _

,. < .
J

'. Strat~gy

Listen to a conversation
or lecture.

Take notes on the key points and important details.

Read the prompt and


understand the task.

Identify what you will need to discuss.

Organize the id-e:-1

Decide on your topic sentence and the


details. Be sure to include reasons and examples for
any personal opinions expressed.

--------=~-_]

supporting~

integrated Speaking: Stating Op'1nions and Summar'1z'1ng

---'- -

345 - -

Practice 1 - Conversation
~Step

n Listen to a conversation. As you listen, take notes on the. problem and the solutions
suggested. @MH*
literary:
relating to writing and
literature
budget:
the amount of money
available to a business
.shut down:
to stop doing business
advertisement:
an announcement that tells
people about a product or
sel\lice
let go:
to tell someone to stop
working for you

Solution 1: - - - - - - - - - - - - - - - - - -

Solution 2: _ _ _ _ _ _ _ _ _ _ _ _ _ _ _ _ __

~Step

Read the prompt below. Write down your opinion .

..

\"'-'"

.,,~:~~i:::i:~:~~:~~~~:~~l~~~~i~~~~;~~-:~;:::l~~~i;.;::~~~:t~~~~:~I~~:. 1

"'

~- ~

'

u<

.-~ ''~''""~--- _,. .. , .. ,, ~~'-"'-"'-~"'"'"'~-"-'"'~-,,, ,_,,,,.~_.,., -~<c''''-'"-' '-"v"vo_,.,-,,,..,,,

'<'"" '"'"""'"' " " "

, "'-'

"'''"''"~~"-""'""'h~~><>-~~~ .. ~,,'"'''

Problem:
Best solution:-------~-------------------Reason 1:
Reason 2:

[
~Step

() listen to two sample responses. Write down any useful expressions. i@jrii
.. - "

_,,_

. . . . . . . . . . . . . . . . . h . . . . . . . . - . . . . . . . . . . . . . . . . . . . . . . .-

. . . . . . . .%

. . . . . . . . . . . . . . . . . . . . . . . - " . . . . . . . . . . . . , , _ _ . . . . . . . . . . . .

JB ...

~Step

Now create your own response using words and expressions from Steps 1, 2, and 3. Use the
prompts below to help you.
I think _ _ _ _ _ _ _ _ _ _ _ _ _ _ _ _ _ _ is the better solution. First, it is
the easiest way to----~--------- If _ _ _ _ _ _ _ _ _ __
- - - - - - - - - - - - - - - - - - - - I t would be easy _ _ _ _ __
________________ . Second, ______________
_ _ _ _ _ . If they _ _ _ _ _ _ _ _ _ _ _ _ _ _ _ _ _ _ _ _ _ __
_ _ _ _ .Then ___________________________

~Step

Read the response you wrote above out loud. Try to read slowly and dearly. Practice saying
the whole response several times. Then close your book and say the response without looking
at the words.

Integrated. Spedkmg Stating Opinions and Summarizing

347 ...:______,

Practice 2 - Conversation
~Step

n Listen to a conversation. As you listen, take notes on the problem and the solutions
suggested.

iiijijltii

preparation:
the process of getting ready
for something
sponsor:
to support someone while
.. they do something
independent:
done alone without the help
of others
devote:.
..
to give all of your attention
to something
teaching assistant:
a student that teaches
part of a class along with a
professor

Problem:-------------------

Solution 1: - - - - - - - - - - - - - - - - - -

Solution 2: - - - - - - - - - - - - - - - - - -

~Step

Read the prompt below. Write down your opinion.


i,............... ,...............,_..................... - ...........-~--~~---..--~---.......- ~-... ~------~-----""'''"''J

The speakers discuss two possible solutions to the man's problem. Describe the problem.
Then state which of the two solutions you prefer and explain why.

,, __ < .,_,..,,,, <'

,_,._,

o - -~-- ,_.,.,

>' ._ - , ,.,, " " ' , , . ,

.N-"o,~.~'4~>~C~<='>"~>LO~~~->"'-"'""~'~"'"~'""'~-~-O,,,.,,,_ "'"-~--~-~

<"

~-L~U>L-~~~7~> -~~~><~>" r-

, . ._

-~ -~

Problem:
Best s o l u t i o n : - - - - - - - - - - - - - - - - - - - - - - - - - - - -
Reason 1:
Reason 2:

n Listen to two sample responses. Write down any useful expressions.

i@IUI
- - - - - ___
1~kti111n.:._

wc1;;:;,.

-------~----------- ---------------------~--------

~z

--------

G'>

---

---

..

--~---------

_______________________

.. _

- - - - ------------

---------------

-----------------------------------------------~

----------------

'"------------------~Step

Now create your own response using words and expressions from Steps 1, 2, and 3. Use the
prompts below to help you.
The man's problem is that _ _ _ _ _ _ _ _ _ _ _ _ _ _ _ _ _ _ _ _ __

~-------I t h i n k - t h a t _ - - - - - - - - - - - - - - - - - - - is better: he s h o u l d - - - - - - - - - - - - - - - - - - - - - - - - First,


__________________ right now. Teaching is ________
______________.Second, ________________

~Step

Read the response you wrote above out loud. Try to read slowly and clearly. Practice saying
the whole response several times. Then close your book and say the response without looking
at the words.

Integrated Speaking: Stating Opinions and Summarizing

349 - ' .

Practice 3 - Conversation
~Step

\) Listen to a conversation. As you listen, take notes on the problem and the solutions
suggested. @if&+
debate:
a discussion In which
people offer different views
on a topic

conference:
a large meeting about a
particular subject

Solution 1: - - - - - - - - - - - - - - - - - -

letdown:
to disappoint

eager:
excited

coach:
to train someone in a new
skill

Solution 2: - - - - - - - - - - - - - - - - - -

~Step

Read the prompt below. Write down your opinion.


The speakers discuss two possible solutions to the man's problem. Describe the problem.
Then state which of the two solutions you prefer and explain why.
,,

,""

Problem:
Best s o l u t i o n : - - - - - - - - - - - - - - - - - - - - - - - - - - - Reason 1:
Reason 2:

~Step 3

n Listen to two sample responses. Write down any useful expressions.

~
l@iM

----------~--. . . . . .~----,,_~------~,..,,------~. 1thtli~

~
'------I

----------------

----------------------------------~----~-

---

------------------------------------- --------

-----------------------------------

~Step

Now create your own response using words and expressions from Steps 1, 2, and 3. Use the
prompts below to help you.
The man has to choose ________________________

I ,,

------------:l think he shbuld ---------------- - - - - - - - - - - - F i r s t of a l l , - - - - - - - - - - - - - - - ____________ .If _ _ _ _ _ _ _ _ _ _ _ _ _ _ _ _ _ __

________ .Second, he _____________________


_ _ . Next t i m e , - - - - - - - - - - - - - - - - - - - - - - - - - - -

~Step

Read the response you wrote above out loud. Try to read slowly and clearly. Practice saying
the whole response several times. Then close your book and say the response without looking
at the words.

Integrated Speaking: Stating Opinions and Summarizing 351 - .

Practice 1 - lecture
~Step

n Listen to a lecture. Fill in the missing information in the notes.

*il!!!IM

Topic: - - - - - - - - - - - - - - - - - - - - -

Thesis:

Key point 1: - - - - - - - - - - - - - - - - -

oral:
spoken .
transmit:
to send or Convey from one
person or place to anothe.r
definitive:
better than au others
embellish: ..
to make a story more
interesting by adding new
details
flexibility:
being able to adapt or
change

Key point 2: - - - - - - - - - - - - - - - - -

~Step

Read the prompt below. Using the information above, write the first sentence you would say
to answer this prompt.

~~;~~~:i~~~~~~~:~:~~;::;r~~;~~:~;~;;::~;~i~~-:~::;~;:diti:::;:;~~:-nci:~~
I'"

;,,

stories.
The lecturer says that in ancient societies,------------------~

> - - - 352 Chapter 3

~Step

n Listen to a sample response. Write down any useful expressions.

--.- - - - - -

~.Step

l@jfii

-------..-------------------------------------

Now ereate your own response using words and expre~sions from Steps 1, 2, ~nd 3. Use the
prompts below to help you.
The lecturer says that in ancient societies, - - - - - - - - - - - - - - - - - - - - - - - - - - - T h e y did not _ _ _ _ _ _ _ _ _ _ _ _ _ _ _ __
and this----------------------~ First, _ _ __
- - - - - - - - - - - - - - - - - - - - F o r instance, _________
__________________ . Second, ____________
____________ like they do today. So many p e o p l e - - - - - - - - -

~Step

Read the response you wrote above out loud. Try to read slowly and clearly. Practice saying
the whole response several times. Then close your book and say the response without looking
at the words.

Integrated Speaking: Stating Opinions and Summarizing

353 - .

Practice 2 - Lecture
~Step

n Listen to a lecture. Fill in the missing information in the notes.

l@i'h

Topic: - - - - - - - - - - - - - - - - - - - - -

Thesis:

animation:
the practice of making
pictures look like they are
moving
differ:
to. have different
characteristics

image:

Key point 1: - - - - - - - - - - - - - - - - - -

a picture of something
time-consuming:
taking a long time to do
reputation:
the opinion that people
have of something

Key point 2: - - - - - - - - - - - - - - - - - -

~Step

Read the prompt below. Using the information above, write the first sentence you would say
to answer this prompt.
f

... ~'="~""'"'=--<,.,~_,

u~vw~""U'-'-"'-",..,.''-"==

... -,,..~.~~""""-~~~~.--......-~woT~..._.<o="'-'-"~~~o,.=+<-~--, ~~"""""=u,o.v.-...,,.~,..,_, ,., ..,,,,,.,._,o,,n,~; .,-~.,

''<.,

Using points and examples from the lecture, explain how traditional animation and
computer animation differ.
J'

There are two

-------------------------------~

[(
~Step

n Listen to a sample response. Write down any useful expressions.

i!t!SM

>Step 4.,
Now create your own response using words and expressions from Steps 1. 2, and 3. Use the
prompts below to help you.
There are two main ways that ______________________
differ. First, they take d i f f e r e n t - - - - - - - - - - - - - - - - - - - - - Traditional a n i m a t i o n - - - - - - - - - - - - - - - - - - - - - - - - - -

Computer a n i m a t i o n - - - - - - - - - - - - - - - - - - - - - - - - - Second, they differ in how _ _ _ _ _ _ _ _ _ _ _ _ _ _ _ _ _ _ _ _ _ __


Many people prefer._________________ because _ _ _ _ _ __
--------------They like--------------less since

-------------------------

>Ste;:i' 5
Read the response you wrote above out loud. Try to read slowly and clearly. Practice saying
the whole response several times. Then close your book and say the response without looking
at the words.
Integrated Speaking: Stuting Opinions and Summarizing

355 - - ;
------

Practice 3 - lecture
~Step

n Listen to a lecture. Fill in the missing information in the notes.

lt!Jt!S!:i I

Topic: - - - - - - - - - - - - - - - - - - - -

Thesis:

brand:
a name given to a group of
related products
draw:
to make someone notice something
blindfold: .
to tie a cloth over
someone's eyes so they
cannot see

attach:

Key point 1: - - - - - - - - - - - - - - - - -

to connect one thing to


another
emergency:
a situation that needs
immediate action

Key point 2: - - - - - - - - - - - - - - - - - -

~Step

Read the prompt below. Using the information above, write the first sentence you would say
to answer this prompt.
Using points and examples from the lecture, explain why a strong brand name is important.

The lecture talks about _ _ _ _ _ _ _ _ _ _ _ _ _ _ _ _ _ _ _ _ _ _ _ __

'

356 Chapter 3

- ---~

~Step

n Listen to a sample response. Write down any useful expressions.

t@ll:!i

------------..------------------------illtid

..
~Step 4.'"
..;

Now create your own response using words and expressions from Steps.1, 2, and 3. Use the
prompts below to help you.

'1.

The lecture talks about _______________________ .


'!

First,-------------------~-- is important because it can

- - - - - - - - - - - - - - - - F o r instance, people can _ _ _ _ _ _ __


_ _ _ _ and t h i n k - - - - - - - - - - - - - - - but they _ _ _ _ _ __
--------Second, _ _ _ _ _ _ _ _ _ _ _ _ _ _ _ can help people
- - - - - - - - - - - - - - - - - - For example, in a health emergency, you will
c h o o s e - - - - - - - - - - - - - - - - - - - - The strong _ _ _ _ _ __

I. .,
I

______ helps you remember __________________ _

>Steri' 5

Read the response you wrote above out loud. Try to read slowly and clearly. Practice saying
the whole response several times. Then close your book and say the response without looking
at the words.
Integrated Speaking: Stating Opinions and Summarizing

357 ____::

~,.
1;.,.
'~
.,

:~

Instructions: Choose the best word or phrase


to complete each sentence.

1. When I yelled in the small room, I heard a(n)


(A)
(B)
(C)
(D)
:,

;\

'i
;j

my brother because he never


gives up.
(A)
(B)
(C)
(D)

express
recite
regret
admire

his or her goals.

.\"-- 358

(A)
(B)
(C)
(D)

achieve
sponsor
attach
intend

5. The university is repairing the building


because it looks
(A)
(B)
(C)
(D)

sensory
packed
outdated
literary

Vocabulary Revie1N 1

advertisement
accuracy
conference
emergency

I'"'i

(A)
(B)
(C)
(D)

conversely
simultaneously
extraordinarily
surprisingly

8. I became a literature ____ because I


love to read.

4. With hard work, anyone can


'i

of

duration
base
luxury
arena

3.

(A)
(B)
(C)
(D)

'

the present.

the ride.
(A)
(B)
(C)
(D)

____ was not very good.

;
Ir:

7. I was _ _ _ excited before I opened

2. Please stay seated for the

11

budget
debate
echo
latter

6. While he could throw the ball very far, his'.

(A)
(B)
(C)
(D)

survey
major
commodity
image

Instructions: Choose the word or phrase


closest in meaning to the underlined part.
9. You shouldn't worry about the things that
you can't control.
(A)
(B)
(C)
(D)

fret
hire
classify
let go

10. Pauly's Pet Shop had to close because it


was not making money.
(A)
(B)
(C)
(D)

hike
embellish
struggle
shut down

'

'~

11. I really disappointed my friend when I forgot


to help him study.
(A) let down
(B) coached
(C) differed
(D) blindfolded
12. I feel very relaxed when I am with my family.
(A) organized
{B) generous
(C) comfortable
(D) talented
13. We wanted to buy a new printer, but we
didn't have enough money left.
(A) illustration
(B) section
(C) funds
(D) program
14. I found today's discussion on marine life
,;very interesting.
;lii(A) stimulating
(B) constructive
(C) courageous
(D) organized
15. I was the first to arrive, so the room was
completely filllQ.!y.
(A) available
(B) sleek
(C) bulky
(D) vacant

Instructions: Write the missing words. Use the


words below to fill in the blanks.
;,), ~~9e[, ..... significant .
animation
.draw
,_ii.'.L
'" . - . brand .i
<~>>,...,-

There are many ways that businesses attempt


to 16.
the eyes of customers
using their advertisements. Since the computer
has made17.
easier, many
advertisers are 18.
to use it
on television. Television has a number of
advantages for advertising. First, it allows a
company to show why its 19. _ _ _ _ __
is the best product available. Second, since
many people spend a 20. _ _ _ _ __
amount of time watching television, companies
know that their messages will reach many people.

3z
Gl

Instructions: Choose one word that does not


belong in each group.

21. reputation
responsibility

opinion
attitude

22. stimulating
interesting

fascinating
upcoming

23. independent
automatic

unconscious
unaware

24. definitive
reproduction

perfection
best

25. concentrate
devote

expand
focus

---.1

'

Vocabulary Review_1 359

;~_: ''
.

Instructions: Choose the best word or phrase


to complete each sentence.

6. The company also

especially
repeatedly
moreover
none too soon

2. I was given a(n)

ten poirits for


answering the bonus question.

(A)
(B)
(C)
(D)

upcoming
additional
core
advanced

3. I learned to ____ the poem from


memory.

(A)
(B)
(C)
(D)

recite
last
struggle
consult

4. The instructions confused me because they


were so _ _ __
(A)
(B)
(C)
(D)

'I
I

utilitarian
patient
available
advanced

5. We consider our dishwasher a(n)


____ because we couldn't live
without it.
(A)
(B)
(C)
(D)

budget
luxury
necessity
accomplishment

free

technical support.
(A)
(B)
(C)
(D)

1. The party is about to begin, so you arrived


(A)
(B)
(C)
(D)

'

regret$
sneezes
offers
transmits

7.

t:
'';

to study tonight,
to a movie.
(A)
(B)
(C)
(D)

intend
encourage
hold
last

'<

;--_;(

'

8. Many people choose to---- e~en>::.


though they don't get paid.
(A)
(B)
(C)
1(0)

:.

concentrate
volunteer
hire
admire

Instructions: Choose the word or phrase


in meaning to the underlined word or ghrase..

"
.i.

9. The store didn't have the shirt Iwanted, sq


I got one that was almost the sarn(;l.
' .
(A)
(B)
(C)
(D)

central
similar
oral
time-consuming
).,

10. This weekend, I'm going with some friends


to walk in the mountains.
(A)
(B)
(C)
(D)

hike
expand
struggle
let go

'.-:.: -

il

r-1---

360 Vocabulary Review 2

'in;:_.-

. .- .'

11. One of the pool's rules is that you can't


play loud music.
(A) echoes
(B) luxuries
(C) surveys
(D) policies
12. I spent last weekend fixing a hole in our
roof.
(A) leak
{B) term
(C) condition
(D) major

,-i

13. I can't carry the TV alone because it is so


large.
(A) definitive
{B) unconscious
(C) bulky
i (D) auditory
14;:The university offers many basic classes
;Y,like Introduction to Chemistry.
(A)
(B)
(C)
(D)

core
sleek
automatic
literary

15. I trained her for three weeks, and now she


is a better volleyball player.
(A) differed
(B) achieved
(C) coached
(D) attached

Instructions: Write the missing words. Use the


words below to fill in the blanks.
. : competence .conver9ely latter
experience ... firefighter

I~-

Many scholars suggest that there are two


different kinds of knowledge: wisdom and
16.
. While the former
requires information gained from books, the
17.
can only be gained by
practicing. In most situations, both kinds of
knowledge are very important. For example,
consider the career of a 18. _ _ _ _ __
The 19.
necessary to
perform his or her duties can be developed by
reading books describing relevant procedures.
20.
, some qualities, such as
bravery, can not be learned in books and must
be developed through practice.

Instructions: Match the words that have


opposite meanings.

21. packed

(A) unimportant

22. advanced

(B) simple

23. reproduction

(C) let go

24. significant

(D) empty

25. hire

(E) original

1.

-1

Vocabulary Review i

361

Independent Speaking: .
Test Questions 1and 2 .
. .Integrated Speaking:
,,- .. ,

Test Questions 3 and 4


1a1.oc:1

Integrated Speaking:
Test Questions 5 and 6

Vocabulary Review 3
, Vocabulary Review 4

A response to the independent speaking question generally


includes the following:
An introduction to the general topic of the essay-usually one sentence
A statement of your opinion-usually one sentence
Your reasons + examples/details to support them-one or two sentences each
Within 60 seconds, the time given for your response, you should be able to say about
eight sentences. These sentences should be similar in length to the following: "I often
take my guitar to parties and play music for my friends there."

Before speaking:
Choose an opinion that is easily supported.
Organize the flow of your response in your mind .
. Make sure that you have adequate reasons and examples.

When speaking:

Make a clear statement of your opinion on the given topic.


State clear reasons for your opinion.
Use concrete examples.
Use transitions to improve the flow of your speech;

Independent Speaking: Test Questions 1 and 2

365 - - , ,

Practice 1 - Personal Experience


~Step

Read and think about the prompt below.

'

~;

(r-.. --.
l]

What is a skill you have always wanted to learn? Why do you want to learn it? Include
specific reasons and examples to support your answer.

\
\,"

-"~,-.-,-;- U

'>> '"'"'"'"

'

"'-"

'>'<

,~o'>J >e~>< m_." <> F~--- r""-'"'""""~'""'""K~--~'"'' ,>.-.'~"''V>-""~~... ~ ,~. ' .,~-~ ''"~""""'~"'~.-"""'.,,..,",'"''''~"' WJ,""" T<oo~-~ <>O"><~-'=?<VO~~'>

The sentences below make up a response to the prompt above. Read the sentences and underline
any transitions you find.
(A) This is because once, when I was little, my grandfather took me

(8)
(C)
(D)
(E)

(F)

sailing.
I also want to learn how to sail because I love the ocean.
However, I cannot do that unless I know how to sail.
I love the ocean so much that someday, I would like to travel the
world in my own sailboat.
Since that day, I have wanted to understand how he steered the
vessel through the wind just by manipulating ropes.
I have always wanted to learn how to sail a boat.

little:
very young
sail:
to control the movement of
a boat through the water

steer:
to control the direction that
something moves
vessel:
a boat

manipulate:
to control something
skillfully

Look at the sentences again. Think about the role of each sentence in the response. Then list the
sentences in the correct order.
A

, ,.J--- 366 Chapte" 4


.t

' t
~-

~Step

Do NOT look at the sentences in Step 1. Answer the following questions in complete sentences.
1. What does this person want to learn?

2. What did she do as a child with her grandfather?

3. What does she want to do in the future?

Using the answers you wrote above, try to speak for 60 seconds explaining this person's
response. Use the words and phrases below while you are speaking. Record your time.
once

h:. Response 1:

~Step

when

because

someday

unless

Speaking time: _ _ _ _ _ seconds

n Now listen to the sample response. How is it different from yours? What parts of the
response can you use in your own? Write down any helpful expressions you hear. MSl:ii

l--

=---------------------=~-------1
I
n

Listen again and repeat after the response. Pay close attention to pronunciation, intonation,
and word stress. lijdjt:@i

Independent Speaking: Test Questions 1 arid 2

367 - - .

~Step

Now write your own answers to the following questions in complete sentences.

1. What is a skill that you have always wanted to learn?


2. What experience have you had that makes you want to learn this skill?
3.

How would you like to use this skill in the future?

Using the answers you wrote above, give a spoken response to the prompt below. Try to
incorporate useful expressions from Step 3, while also paying attention to your pronunciation
and intonation. Record your time.

f"-"'--;~:~~~::~~~~:=-h~~::~ay:~~n~::-;~~:~;-;~;-~:..~:: ::~~~:~:::;;~nclude--~1
i

specific reasons and examples to support your answer. .

\.,~~",'<<="'"',..."'-~~-u'-'",-~'~""'""'v.~~'''"-'~~"'~q"''v;_,.;~.,,..,._,~, "n="'""'-"''''"~''-">c~.__,>n,"~'~---"""'"'<'"-''-"~k-.<--<>,""'-''"'~~- '"""'~-~OoO"~=-,;.<"~~~"~~'""~=~-=-"'"'~

Response 2:

Speaking time: ______ seconds

I
,
I

1---:-. 368 Chapter 4_

. '

j'

Practice 2 - Personal Experience


~Step

Read and think about the prompt below.


--- ..
[

-~::cri~:~:~::;~t you ~:Id lov~ to take, and explain why you want to take it. ~:Jlude
specific reasons and examples to support your answer.

,.,,..,.~......,m......,...,.

__ , _ _ _ _ ............._,_,,_,

_...,.__

....._...._......,

The sentences below make up a response to the question above. Read the sentences and
underline any transitions you find.
(A} So I would like to learn techniques for taking quality photographs.
(8) I recently got one for my birthday, but I do not know how to use all
the features.
(C) Someday, I would love to take a class in photography.
(D} They never come out very well, though.
(E} For one, I would like to take this class because I find cameras
intriguing.
I would also like to take photography because I love taking pictures
of nature and animals.

technique:
.. a w.ay of doing so(Tlethiog;
askill

. le~ture;
an important part of
so(Tlething
. photography:
the process of taking
pictures
come out:
to be revealed

intriguing:
interesting and exciting

Look at the sentences again. Think the role of each sentence in the response. Then list the
sentences in the correct order.
A

Independent Speaking: Test Questions 1 and 2 369

~Step

Do NOT look at the sentences in Step 1. Answer the following questions in complete sentences.
1. What class does this person want to take?

2. What did he receive for his birthday?

3. What does he take that never come out very well?

Using the answers you wrote above, try to speak for 60 seconds explaining this person's
response. Use the words and phrases below while you are speaking. Record your time.
so

for one

Response 1:

~Step

recently

because

but

Speaking time: _ _ _ _ _ seconds

(I Now listen to the sample response. How is it different from yours? What parts of the
response can you use in your own? Write down any helpful expressions you hear. mE'Jll

(I Listen again and repeat after the response. Pay close attention to pronunciation, intonation,
and word stress. @il:fi

~Step

Now write your own answers to the following questions in complete sentences.
1. What class do you want to take?

2. What experience makes you want to take this class?

~
3z

Gl

3. What do you want to learn from this class?

Using the short answers you wrote above, give a spoken response to the prompt below. Try to
incorporate useful expressions .from Step 3, while also paying attention to your pronunciation
and intonation. Record your time.

-........................-...................

- .-"~~"'"'"""----.

.......... ,,,,,,,,,,,... _,,,,,,.,........

_.~_,,,.,,

____ ........
_.

~.]

Describe a class that you would love to take, and explain why you want to take it. Include
specific reasons and examples to support your answer.

,,_...,,:~....,,,i~~~--"'"~~'-""--''"~,~~h~<m"""-''.,_,~~~~.,.-....,. .., ..,,,,,,,,~,.-.,,.,,.,.~.,~"'-~"'<''u"-''"~~"'=~~"'""'~"'"<-'-~i""',,-~_,,_.,.,,_,~.,,.~~'-'~"..w~.~<>==~"~<r~~~~


Response 2:

Speaking time: _ _ _ _ _ seconds

Independent Speaking: Test Questions 1 and 2 '371

-.

Practice 3 - Personal Experience


~Step

Read and think about the prompt below.

---.----"--"-------------

-----

Describe an activity that you like to do in your free time, and explain why you like to do
it. Include specific reasons and examples to support your answer.

.....

~"""""'""

_______

._~,,--,--.....

....,...._ _ _ .,,.,..,.,

...

--]

--~='"'"""""'

A sample outline of a response is given below. Write down transition words or phrases that
can be used to link the ideas.
Activity: Ride bike in mountains

Transition words/phrases:

Reason 1: being in nature

'--'-----

Reason 2: relaxing

Using the above outline, give a spoken response to the prompt above. Record your time.
Response 1:

~Step

Speaking time: _ _ _ _ _ seconds

n Now listen to the sample response. How is is different from yours? What parts of the
response can you use in your own? Write down any helpful expressions you hear.

mm

lengthy:
long
wildlife:
animals in nature

unwind:
to relax

demanding:
requiring a lot of time and effort
hectic:
very busy and often confusing

n listen again and repeat after the response. Pay close attention to pronunciation, intonation,
and word stress. @Ml:tl

f_

~Step

. Without listening to the sample again, give another spoken response to the prompt. Try to
incorporate useful expressions from Step 2, while also paying attention to your pronunciation
and intonation. Record your time.
Speaking time: _ _ _ _ _ seconds

Response 2:
~Step

Make your own outline for the prompt. Try to incorporate transition words and useful phrases
introduced in Steps 1 and 2.
Transitio~ wbrcis/phrases:

Activity: - - - - - - - - - - - - - -

Reason 1: - - - - - - - - - - - - -

I
Reason 2: - - - - - - - - - - - - -

Usirlg your outline, respond to the prompt. Record your time.

--------------------

. ,....

-----~-----------------]

Describe an activity that you like to do in your free. time, and e.xplain why you like to. do
it. Include specific reasons and examples to support your answer.
_.....~~

...

----~=-~...,,,~= ~-~-....-~

Response 3:

------~----...--~--..--"""--

.........- . .

Speaking time: _ _ _ _ _ seconds

I
I

Independent Speaking: Te~t Questions 1 and 2

373 - - .

Practice 4 - Personal Experience


~Step

Read and think about the prompt below.

A sample outline of a response is given below. Write down transition words or phrases that
can be used to link the ideas.
Day: Sister was born

Transition words/phrases:

Reason 1: I met my best friend


Reason 2: My family spent the day together

Using the above outline, give a spoken response to the prompt. Record your time.
Response 1:

Speaking time: _ _ _ _ _ seconds

,,'
I

~Step

n Now listen to the sample response. How is it different from yours? What parts of the

response can you use in your own? Write down any helpful expressions you hear. l!tfl!!l:l:I

lonely:
unhappy because you feel alone

companion:
a friend or partner
wonderful:
very good
delighted:
happy or excited
get along:
to be friendly to someone

n Listen again and repeat after the response. Pay close attention to pronunciation, intonation,
and word stress. llftlhP'
i1

~'i....---

i'I

ii!

374 Chapter 4

~Step

Without listening to the sample again, give another spoken response to the prompt. Try to
incorporate useful expressions from Step 2, while also paying attention to your pronunciation
and intonation. Record your time.
Speaking time: _ _ _ _ _ seconds

Response 2:

~Step

~z

c:>

Make your own outline for the prompt. 1iy to incorporate transition words and useful phrases
introduced in Steps 1 and 2.
Day: ________________

Transition words/phrases:

Reason 1: - - - - - - - - - - - - -

Reason 2: - - - - - - - - - - - - -

U$Jng your outline, respond to the prompt. Record your time .

............... ............ . . -.,...,.

~.--

---"''"''"-~

......,..............,..............,............... ............... . ;.... . . ..,_...............l


~

Describe a day that you believe was the best day of your life, and explain why it was so
memorable. Include specific reasons and examples to support your answer.

.
1
\...,,,.................................. ..... ........................~............................................. ' '"""''""'"""'""''"~~..."'..'''""'""""'' ...../ .

Response 3:

Speaking time: _ _ _ _ _ seconds

Independent Speaking: Test Questions 1 and 2

375

Practice 1 - Personal Preference


~Step'

Read and think about the prompt below.

----.,.----~-------------

---~

..

Should teachers give unannounced tests in class? Why or why not? Include specific
reasons and examples to support your answer.

_ _ _,,,__, _ _
.

..

~~=

--------~-~~~-- ,_,,.~,.,..,_,_,_...,....____

.J

__,,_....,..,

The sentences below make up a response to the prompt above. Read the sentences and underline
any transitions you find.
(A) I think that teachers should give unannounced tests in class.
(B) I know I stay on top of my studying if I suspect there might be a
surprise test at any time.
(C) However, with short, unannounced tests, you can get more points.
(D) Some teachers only give one or two tests, and if you do poorly on
one, your grade suffers severely,
(E) Secondly, unannounced tests are a great way to ,increase your
grade.
(F) For one thing, giving surprise tests makes sure that students keep
up with the material.

unarinounced:
unexpected; surprise
stay on top of:
to be in control of
something
suspect:
to think that something .
might happen
suffer:
to-become Worse
material:
the information studied in a
class

Look at the sentences again. Think about the role of each sentence in the response. Then list the
sentences in the correct order.
B

:i-.":.\",'- 376 Chapter 4

~Step

Do NOT look at the sentences in Step 1. Answer the following questions in complete sentences.
1. What does this person think about unannounced tests?

2. How do unannounced tests affect how he studies?

~
3z

G'l

3. How can unannounced tests affect his grade in a class?

..

Using the short answers you wrote above, try to speak for 60 seconds explaining this person's
response. Use the words and phrases below while you are speaking. Record your time.

[.:-:.
.

J think

:w

Response 1:

for one thing

secondly

however

if

Speaking time: _ _ _ _ _ seconds

~Step 3

n Now listen to the sample response. How is it different from yours? What parts of the
response can you use in your own? Write down any helpful expressions you hear. @SM

. ,,.........................,dl!-it.
1!

I
I n Listen again and repeat after the response. Pay close attention to pronunciation, intonation,
and word stress. @pii
I
I
w~
---1.

!r'!,.!u'~:~'lt!~f!i'! !'l l..l

Independent Speaking: Test Questions I and 2

377

l l.l l .11111.11111.l l .11111111111111111111111111111111111111111111111111111111..llll.. 1111...1111..llll...llllc.lllllll!l.llll... llll.!llll!llllllllllllllllllllllllll!llll!l!!!llllllllllllll---j ..-

lllll._lllll
.. 11111.-11111.11111.llll.111
....

~Step

Now write your own answers to these questions.


1. How do you feel about unannounced tests?

2. How do unannounced tests affect how you study?

3. Do you think unannounced tests affect your grade in a class? Why or why not?

Using the short answers you wrote above, give a spoken response to the prompt. Try to
incorporate useful expressions from Step 3, while also paying attention to your pronunciation
and intonation. Record your time.

(
;

,,,,.,,,_,_,~

---~-----~"-""'''----~- --~--------~--~----"''\

Should teachers give unannounced tests in class? Why or why not? Include specific
reasons and examples to support your answer.

J
\

..

"<.,,,~O<">-~--,>-,,,_,_.,.,_c~i->""'~.<-_,.,,..,..,~,.-,,.,~,"'~~-'~"''""',.. ..,.~~.-. ~k-~~-~-,..._,_._,,_.,"d,. .. ,..~.,._.....,,._..,,,.,.~""',,.. _ _ _ _,......,,_...._,*''''""'""'-"'~"~""""~'N""'_,.~,_-.-,~-

Response 2:

I
j

'

"/

Speaking time: _ _ _ _ _ seconds

)_,

--t

-!

"

~-

!.

~ t;

;.1 ;-----

378

Chapter 4

l~~.::::-,' .-~ : ~e_~~y., -;--c,~-==.~ =~-lC

-;cc,.:-;__-;cc._

-cc-,...--:c'~~"-~--.~-~-~---.._~-

!Practice 2 - Personal Preference


~Step

Read and think about the prompt below.

r. ~. ,. . ~~~~~:~i:fi:~::~~~~~~~:~t~~:~~~;~~~i~i~i~~~~f~~~~~~~:~~:~~~:::i~~~;:~i .
l
examples to support your answer.

~'~"'-'~'"'"'"~,,..,,,~~-""''~'~-"'>.,>...-.~-~-"'""~"'"-''x~"""'~''""'"'"~~~-,,,_,.,,.,.,,,..,,~~-"'~~'-'"'-"=..._,.~.-..m~.,-,,...,_._=~...,.,,_,_,~~c_,,.;,,~,'-'~'"'"-'"'""~~~~-~~~~'M'-

~
3z

__ ,,,

G>

The sentences below make up a response to the prompt above. Read the sentences and underline
any transitions you find.
(A) That way, you can get your degree and find the career that you
really want

r.
{;'

(8) First of all, it is much more difficult to go back to school once you
have entered the work world.
(C) However, they lose the motivation after working for a while.
;i (D) Also, I think it is important to finish your education as soon as you
"'
can.
"{E) I think it is better to go immediately to university rather than work
for a year or two.
(F) Many people may be determined to go back to school.

"

degree:
a document proving the
completion of an academic
program
career:
the type of work a person
does over a lifetime
motivation:
a feeling that makes you
want to do something
immediately:
right now
determined:
very eager to do something

Look at the sentences again. Think about the role of each sentence in the response. Then list the
sentences in the correct order.
E

lndependf.nt Speaking: Test Questions 1 and 2

379

''I
.: :1
~

IfI

~Step

[:r!

Do NOT look at the sentences in Step 1. Answer the following questions in complete sentences.

'illff

1. When would this person prefer to go to university?

'ii'

2. What does she say happens to people who work before going to university?

I
,]i

't:

1-i'
'll1

3. How does she suggest your career is affected by going immediately to university?

"''_\i'I{.
;j'}:
:-i!

'

Using the short answers you wrote above, try to speak for 60 seconds explaining this person's
response. Use the words and phrases below while you are speaking. Record your time.
I think

.;

for one thing

Response 1:

secondly

but

if

Speaking time: _ _ _ _ _ seconds

~Step

n Now listen to the sample response. How is it different from yours? What parts of the

response can you use in your own? Write down any helpful expressions you hear. li!!\!!iii

n Listen again and repeat after the response. Pay close attention to pronunciation, intonation,
and word stress. S@IPi

~Step

c:

zG'l

Now write your own answers to the following questions in complete sentences.
1. When would you prefer to go to university?

2. What do you think happens to people who work before going to university?

~z

G'l

3. How would going straight to university affect your job or career?

Using the short answers you wrote above, give a spoken response to the prompt. Try to
incorporate useful expressions from Step 3, while also paying attention to your pronunciation
and intonation. Record your time.
Some students think it is-b-ett:: work for a ;ar or
~(.'

t:::;:~~;~;::~~ive~~~~~-th-e~~---1

would rather go straight to university. Which do you prefer? Include specific reasons and
examples to support your answer.

-......---

- ""

Response 2:

........ _ _ _ _ _ _ _......_ .. _ _ , ... _. _________.-

.......,,-...---'""'"''"""'-"'"

Speaking time: _ _ _ _ _ seconds

tndepPndent Speaking: Test Questions 1 and 7..

381 - -

Practice 3 - Personal Preference


''

~Step

Read and think about the prompt below.


('"'" ""'"'""' ''"'" '"''''''"''"''"'" '""'"'''''''""'''-'"''""' '''""'''"""''""''""''"'"''"'""'

. I'

~'""'""""""'''" "'''"~"''"'''''M""'''

'"""'"'"" '

"1

Should people be allowed to use cell phones while they drive? Why or why not? Include
specific reasons and examples to support your answer.

\,,,_ __;o-' ""'"'"'"'" . ,.,_,,,,

!
i

'-"'"'W'"-~n>- ""-'"'""-..-=""~"'~'<">~-~~-oc".,._. '~''"~'"' '''""~~==~;,.~>=vro<,...~~'"'"'""''"' ...,.,ku~=.-~~n""'""><-mu;.,,,.,.._,,,,_~-"'-"-<~""'n~-""-'"""'"'-'"'-"' ,,.,,,-_,.)


<

A sample outline of a response is given below.. Write down transition words or phrases that
can be used to link the ideas.
Cell phones while driving: Yes

Transition words/phrases:

Reason 1: It is not distracting.


Reason 2: They are useful in emergencies.

Using the above outline, give a spoken response to the prompt. Record your time.
Response 1:

IP

Speaking time: _ _ _ _ _ seconds

Step 2

(I

Now listen to the sample response. How is it different from yours? What parts of the
response can you use in your own? Write down any helpful expressions you hear. m@m
~Ef~~~
distracting:

stopping you h'om concentrating

attentive:
carefully watching or listening to
something
colllslon:
an accident where a vehicle hits
something
lost:
!:Jnable _t9 figure out where you are
guide:
to show or tell someone where to
go

n Listen again and repeat after the response. Pay close attention to pronunciation, intonation,
and word stress.
~\~.\-----

382 Chapter 4

@iA~i

[
~Step

Without listening to the sample again, give another spoken response to the prompt. Try to
incorporate useful expressions from Step 2, while also paying attention to your pronunciation
and intonation. Record your time.
Response 2:

~Step

Speaking time: _ _ _ _ _ seconds

~
3z

Cl

Make your own outline for the prompt. Try to incorporate transition words and useful phrases
introduced in Steps 1 and 2.
Cell phones while driving: - - - - - - -

Reason 1: _ _ _ _ _ _ _ _ _ _ _ __

I
I
. .
!
I

(:-,

Reason 2: - - - - - - - - - - - - -

Usipg your outline, respond to the prompt. Record your time.

l [
l

Transition words/phrases:

--------.,-----~--~--<".-----~-------------------- ~---1

Should people be allowed to use cell phones while they drive? Why or why not? Include
specific reasons and examples _to support your answer.

..........--......-=,..."<'-'~u~~~~..._.,=_,..,..~,~~,~=~-""-'""""'-'~ '''if'-;>,,m.._"'""''v~~.,.=>=>"~_,,,.=.,._=<""T''u,,-,.,"'H~>~~-..,.,.,~~"'~"~'"-"'~"""-~w~"''~'Q""""-'"'''~~"

Response 3:

Speaking time: _ _ _ _ _ seconds

I
I

I
I
I

Independent Speaking: Test Questions 1 and 2 383

-~-

Practice 4 - Personal Preference


~Step

Read and think about the prompt below.

................. ,., ....................

~'"''----"""'"~'""""'""'''"""''"""''--"'""""''"--~~

..-..

Should music players be banned in schools? Why or why not? Include specific reasons
and examples to support your answer.

\"-. ~-------'"""''''-~--------~--'"--~--- ........ )

r. :

A sample outline of a response is given below. Write down transition words or phrases that
can be used to link the ideas.
Ban music players in schools: No

Transition words/phrases:

Reason 1: They are used outside of class.


Reason 2: They are not dangerous.
;:::,

Using this outline, give a spoken response to the prompt. Record your time.
Response 1:

~Step

Speaking time: _ _ _ _ _ seconds

n Now listen to the sample response. How is it different from yours? What parts of the

response can you use in your own? Write down any helpful expressions you hear. mE

ban:
to prohibit: to forbid
utilize:
to use something
refrain:
to stop yourself from doing
something
hazardous:
dangerous or harmful
mischief:
behavior that causes trouble

n Listen again and repeat after the response. Pay close attention to pronunciation, intonation,
and word stress. @tml@ii

I[

>Step 3
Without listening to the sample again, give another spoken response to the prompt. Try to
incorporate useful expressions from Step 2, while also paying attention to your pronunciation
and intonation. Record your time.
Response 2:

I~-

Speaking time: _ _ _ _ _ seconds

>Step 4
Make your own outline for the prompt. Try to incorporate transition words and useful phrases
introduced in Steps 1 and 2.
Ban music players in schools:

Transition words/phrases:

Reason 1: - - - - - - - - - - - - -

,,,-

__________________

Reason 2:

-------------

Using your outline, respond to the prompt. Record your time.

Response 3:

Speaking time: _ _ _ _ _ seconds

Independent Speaking: Test Questions I and 2

385 - - " ,

A response to the integrated speaking questions generally


includes the following:
Question 3
A statement of the problem or situation, as expressed in the reading
A statement of the speaker's opinion, as introduced in the conversation
His or her reasons + additional information, as taken from the conversation

Question 4
A statement of the main idea or topic of the reading and lecture
Key points that are similar
Key points that contrast

Identify the topic and supporting details.


Organize the flow of your response in your mind.
Make sure that you have adequate reasons and examples.

Begin your response by clearly stating the opinion I main idea of the reading and the
conversation/lecture.
Give reasons or details from the conversation or lecture to support your opinion.
a Make sure statements are clearly connected so that the scorer will more easily
understand your points.

Integrated Speaking: Test Que"stions 3 and 4

387 ---.,

Practice 1 - Reading and Conversation


~Step

Read the passage below and underline important information.

Announcement from the Department of Student Life

We wish to inform you that access to television will no longer be


available in the student union building. Maintenance will remove the
building's current television set by the end of this week. We have
received several complaints from students who wish to use the building
to study and find the television disruptive. In addition, most students
have televisions in their dorm rooms. We, therefore, find it excessive to
provide a television set in this common area.

student union:

a university building used


for student activities
maintenance:
a group of people who
keep a building in good
condition
complaint:

a statement saying that


someone is unhappy with
something
disruptive:

distracting
excessive:
more than is necessary

Write down the main idea and any key points.

'

,_i'

j ._

!'_;

------------------------

...... --

--.------------

Now listen to a related conversation. Take notes on the woman's

opinion. @MW=D

Opinion:--------------------Reason 1:

---------------------~

unreasonable:

unfair
keep from:
to stop someone from
doing something
social:
being with other people
common area:

a place that everyone can


use
constantly:
very often

~Step

-----]

Read and think about the prompt below.

----------------------

[
[

The woman expresses her opinion of the department's decision. State her. opinion and
explain the reasons she gives for holding that opinion.
'""""'"......

~--~..._..

________

......,.,,_~-

WITHOUT looking at the original reading passage, review your notes from the reading and
listening passages. Select the information you think is important. Fill in the blanks of the sample
response below.

- - - - - - - - - - - She says students that want to study could _ _ _ _ _ __


- - - - - - - - - - - S h e also _ _ _ _ _ _ _ _ _ _ _ _ _ _ _ __
"i

with the announcement's statement that

-------------------

She sayi she would have t o - - - - - - - - - - - - - - - - - - - - - - ii she wanted to _ _ _ _ _ _ _ _ _ _ _ _ _ _ _ _ _ _ _ _ _ _ _ _ _ __

After you have filled in the blanks, read the response out loud. Pay attention to your pronunciation,
intonation, and word stress.

Integrated Speaking: Test Questions 3 and 4

389 _ ; ,

~Step

n differences
Now listen to a sample response. How does it differ from your response? Write down any
in information or phrasing. ltl!llQii

n and
Listen again and repeat after the response. Pay close attention to pronunciation, intonation,
word stress.
i@tJm+

~Step

Now give your own spoken response to the prompt.' Try to incorporate useful expressions
from ~teps 2 and 3, while also paying attention to your pronunciation and ie,_1~n,ation. Record
- your time.
Response:

Speaking time: _ _ _ _ _ seconds

I
I

I
I

Practice 2 - Reading and Conversation


~Step

.
I
I
.

Read the passage below and underline important information .

..

Announcement from the Office of Student Housing

If;

This is a notification that students will no longer be allowed to stay


in the dorms over the summer. We require all dorms to have a dorm
supervisor. However, we were unable to recruit enough student volunteers
to supervise the dorms during the summer. In addition, too few students
have applied for on-campus housing this summer to make it cost-effective
for the university. Please visit our office for information on alternatives for
summer housing.

!.-.'

notification:

an announcement
supervisor:
a person who is in charge
recruit

~
3z

to persuade someone to
join you

cost-effective:

GI

gMng profit
alternative:
in place of something else

Write down the main idea and any key points.

r
~~-~'

------

- --

)
(I Now listen to a related conversation. Take notes on the man's
opinion.

@!Jilt

Opinion:-------------------Reason 1: ------------------~

excuse:
a reason tor doing
something
filer:
a paper-based
announcement or
advertisement
insignificant:
not large enough to be
important
fault:
the responsibility for a bad
thing happening

look into:
to research something

Integrated Speaking: Test Questions 3 and 4

391 - - :

. _-,;~

~Step

Read and think about the prompt below.

r'"''"'"'~~e ~:~ ex~~es~:-~:~~~~~;:~co~~er~~~h~anno:~ce~ent~~~~ice ~;~~~de~;"-'''J''

l
-

.......

______

Housing. State his opinion and explain the reasons he gives for holding that opinion.
...
.....
....,___ _,. .... ....

...,,..-~-~

-------

-----..~~--~

.......,".

WITHOUT looking at the original reading passage, review your notes from the reading and
listening passages. Select the information you think is important. Fill in the blanks of the sample
response below.
The man _____________________ with the decision to
- - - - - - - - - - - - - - - - - - - - - - - H e thinks they should have
- - - - - - - - - - - - - - - - - - - - - - - - H e says they could have
- - - - - - - - - - - - - - - - - - - - - - - - - H e also thinks that
_______________________ because the university did not .
- - - - - - - - - - - - - - - - - - - - - Most students do not know that
- - - - - - - - - - - - - - - - - - - - - - - - - - - H e thinks the
university should h a v e - - - - - - - - - - - - - - - - - - - - - - - - After you have filled in the blanks, read the response out loud. Pay attention to your pronunciation,
intonation, and word stress.

,.1

'
l'

[[
~Step

n Now listen to a sample response. How does it differ from your response? Write down any
differences in information or phrasing.

"ilfHj

[
l[

f..
t' .

I.

n and word stress.

Listen again and repeat after the response. Pay close attention to pronunciation, intonation,
iifif11iji

.,.,

rL

~Step

Now give your own spoken response to the prompt. Try to incorporate useful expressions
from Steps 2 and 3, while also paying attention to your pronunciation and intonation. Record
your time.
Response:

Speaking time: _ _ _ _ _ seconds

I
I

I
Integrated Speaking: Test-Questions 3 and 4

393 _ _..,

Practice 1 - Reading and lecture


~Step

Read the passage below and underline important information.

Reward Systems

Many child psychologists recommend that parents use reward systems


because they encourage children to behave properly. Reward systems
are advantageous because they provide positive outcomes for both the
parent and child. The parents witness good behavior in their child, and
the child gets something he or she wants. Reward systems are also
effective because they teach children that good behavior is rewarded.
Children learn to associate good behavior with positive consequences.
As a result, they are more likely to behave well in the future.

reward:
something given or received
in recognition of a job well
done
psychologist:
a person who studies how
the mind works

outcome:
the result of something
witness:
to see something happen

associate:
to connect one thing with
another

).

Write down the main idea and any key points.

n Now listen to a related lecture. Fill in the missing information.


rm11m

Main t o p i c : - - - - - - - - - - - - - - - - - - Key point 1:

~
caregiver:
a person who takes care of
others
preferred:
wanted instead of other
things
liable:
likely
correlate:

to make a connection

Key point 2:

i-J

394 Chapter 5

between two things


bring about:
to make something happen

: {

.:.
I '}.*
.I :,;
..t
I
c

1 .

ff
'

'

'

'rt
r

I
I

I
I

I
!
I

~Step

-'" '. . "'. "" . . . . . . . '. " -. . . . . . . . ., " . . . .,. '" . . . .. . . " ' l

Read and think about the prompt below.

Using details and examples from the lecture, explain the advantages of using reward
systems.

[
,_.,.

.....~-~~~~ _,..,

..

~~-~~"'""~-""' ~~--"---'.~ ~==~

.....-= .... ~

>D W>...,q;< ~""=

~~ '>>.O'<~~r.>~<B

...,,u,

-<>>~

>

'""'' > =~c

,_.,,

> m=~"'"-~L~;."M

,;~

,.,.,.,_,..

WITHOUT looking at the original reading passage, review your notes from the reading and
listening passages. Select the information you think is important. Fill in the blanks of the sample
response below.

-:f-1
"O

I'

"... I

The professor uses the example of ___________________

~ .i:i

t o - - - - - - - - - - - - - - - - - - - He explains that reward systems are

- - - - - - - - - - - - - - - - - - - T h e professor says that _ _ _ __

- - - - - - - - - - - - - - - - is effective because - - - - - - - - - -~------------The

professor also says that _________

-~,..._________

So they are more likely t o - - - - - - - - - - - - - - - - - - - - - - - -

I
I

After you have filled in the blanks, read the response out loud. Pay attention to your pronunciation,
intonation, and word stress.

,-~

'

''

is effective b e c a u s e - - - - - - - - - - - - - - -

tnteg1a:ed $peaking: Test Quest1011S 3 and 4

__

.__

--;~

395

~,-.~,.

I' ~

~Step

Now listen to a sample response. How does it differ from your response? Write down any
differences in information or phrasing. @ilofi

!"i'i',.
."ii.'
; I~ .

'

'(.;-,,

1' :

Listen again and repeat after the response. Pay close attention to pronunciation, intonation,

and word stress. @fl1lf

~Step

Now give your own spoken response to the prompt. Try to incorporate useful expressions
from Steps 2 and 3, while also paying attention to your pronunciation and intonation. Record
your time.
Response:

Speaking time: _ _ _ _ _ seconds

Practice 2 - Reading and Lecture


~Step

Read the passage below and underline important information.

Teaching by Demonstration

Educators employ many different methods for teaching. The best


way to teach a skill, however, is to demonstrate it. Teaching by example
is effective because it provides all the information necessary to learn a
new skill. By demonstrating that skill, an educator conveys every step
students need to know. Teaching by example also shows students how
to apply the information they learn to a real life situation. Students will
not only be able to relay the information but will also understand how to
use it.

employ:
to make use of

demonstrate:
to show how something is
done
convey:
to communicate feelings or
information
apply:
to put into service
relay:
to communicate information

3z

"

Write down the main idea and any key points.

Now listen to a related lecture. Fill in the missing information.

M1ifi

Main topic:

Key point 1:

abstract:
existing in the mind; not of
the physical world
step:
one action in a series
leave out:
to not include something
literally:
used to say that something
is real
series:
a set of similar things

Key point 2:

Integrated Speaking: Test Questions 3 and 4 397 -....,...-

'

------"-~--

- -

.-.-- '".~

~Step

Read and think about the prompt below.


#'~,,,,.,,,,...~_.._,,~.,"-'''''"'''"''''~~=,,>m~=7"'~,-,,.,.,,...,_.."'~=~rn~"'"-~'"=<'~--~~""'~'"''""'""""'~=~~.o->eXo"Y',~,,,,_,,n.-rn~~"~""'"'"'-~,M''""''~~~"-''''""'<~,,._,"_,-'<

..

,,,.,

The professor discusses teaching by demonstration. Using the professor's example,


explain why this teaching method is effective.
,,,_,,,_,.,,,,,_,,.,,,,,.,,.,,,._,.,,,,,,,.,,.,,.,,,

..... ...............................,..,,,..,_,,,,,;,.,,,_.. ....,,........-......,,.,,.,,,,,,,.,,,,,,,,.,.,,.,.,,,.,,,... , ....


~

.)j
,.,.'

WITHOUT looking at the original reading passage, review your notes from the reading and
listening passages. Select the information you think is important. Fill in the blanks cif the sample
response below.
The professor discusses the best way t o - - - - - - - - - - - - - - - - in order to s h o w - - - - - - - - - - - - - - - - - - - - - According to the
professor, you should _ _ _ _ _ _ _ _ _ _ _ _ _ _ _ _ _ _ _ _ _ _ _ __

--~

rather t h a n - - - - - - - - - - - - - - - - - - - - - The professor says that


- - - - - - - - - - - - - - - - - - - - - i s effective because _ _ __
-i

------------~----------The professor says it is also

effective b e c a u s e - - - - - - - - - - - - - - - - - - - - - - - - - After you have filled in the blanks, read the response out loud. Pay attention to your pronunciation,
intonation, and word stress .

. ,._,._'

[
Cl

(I

.:
.
'

Now listen to a sample response. How does it differ from your response? Write down any
differences in information or phrasing. @*H'

--""""'--""-----~---~-------------------Jl.lbii!itll....

~=-~~~-~::.~---~=-:=.==:~=~-~-=~--=~=~-------.----=~- -~~=~~--~=~---~.:. .~. ~.-~:-.:. ~-"J


(I

Listen again and repeat after the response. Pay close attention to pronunciation, intonation,
and word stress. lljftfl1p1

~Step

Nol/\/ give your own spoken response to the prompt. Try to incorporate useful expressions
fron,i Steps 2 and 3, while also paying attention to your pronunciation and intonation. Record
yourtime.
Response:

Speaking time: _ _ _ _ _ seconds

Integrated Speaking: Test Questions 3 and 4

-~

].

399 - - \

A response to the integrated speaking questions generally


includes the following:

A statement of the problem or situation, as expressed in the conversation


A statement of suggested solutions, as mentioned in the conversation
Your opinion of these suggested solutions
Your reasons + examples and details to support them
A summary of the main points of the lecture

Before speaking:
Choose the opinion most easily supported.
Organize the flow of your answer in your mind.
Make sure that you have adequate reasons and examples.

When speaking:

Make a clear statement of your opinion on the given topic.


State clear reasons for your opinion.
Use concrete examples.
Use transitions to improve the flow of your speech.

1io describe problems:

She/He is having a problem with _ _ _ __


The problem is _ _ _ __
She/He needs help with _ _ _ __
She/He is having trouble _____
She/He cannot figure out _____

She/He needs to ~---She/He should _ _ _ __


One (Another) thing she/he can do is _____
The best thing she/he can do is _ _ _ __
If I were her/him, I would _ _ _ __

Integrated Speaking: Test Questions 5 arid 6

401 -

Practice 1 - Conversation
~Step

(I

Listen to a conversation. Take notes on the problem presented and the possible solutions
suggested. iljbfJD+

Problem: - - - - - - - - - - - - - - - - - - r-.

Solution 1: --~---------------Possible benefit: - - - - - - - - - - - - - - - - -

ball:
a formal dance
fund:
to provide money
back out:
to decide not to do
something

costume:
clothes worn to make you
look like someone or
something else
notice:

Solution 2: - - - - - - - - - - - - - - - - - - -

an early warning that


something is going to
happen

Possible benefit: - - - - - - - - - - - - - - - - -

On your own, think of some of the possible benefits to each solution suggested in the
conversation. Write them in the extra spaces provided above.
;:

: ;

il>Step 2
Read and think about the prompt below. Answer the following questions in complete sentences.

........... . . . . . . . , ..................... _...........................................................................". . . . . . . . 1

_,,,-.

The students discuss two possible solutions to the woman's problem. Describe the problem.
Then state which of the two solutions you prefer and explain why.

I
,)

1. What is the problem?

2. What should the woman do?

3. Why?

l,:i; .
iq.~(

\,'):-;<(<'--

402

Chapter G

!:~,-:!

l'o!1

"li''

~11!1!1_111111111111~_1111111-=1111111-_111111-1111111_::-_111111_-llllll_
llifll-_--111111~=1111111==illlrl'~--1111111~_--111111--1111111:=1111111_1111111_ll!llllllll!!!!_!!!!!!!_!!!!!!!---!!!!!!l_!!!I!!!-_~--~-=---~-~=---=~--""'"=-""'""-C"'"""-_--""'_-11111111~11111
"- ___11111~-__

r
~Step

[[

Now create your own response to this topic using words and expressions from Steps 1 and 2.

The woman is o r g a n i z i n g - - - - - - - - - - - - - - - - - - - - - - and _ _ _ _ _ _ _ _ _ _ _ _ _ _ _ _ _ _ _ _ _ _ _ _ . The man says she

lt

should either _ _ _ _ _ _ _ _ _ _ _ _ _ _ _ _ _ _ _ _ _ _ _ _ _ _ __
I think it is best for her to

-------------------------

That way, she won't _ _ _ _ _ _ _ _ _ _ _ _ _ _ _ _ _ _ _ _ _ _ _ __


Also,------------------------------

I.' . n
-;'

Now listen to a sample response. How does it differ from your response? Write down any
differences in information or phrasing. ll'l!flfllt

---- ------ --- I

---,,~<-< r'

,.,_,,_w

.~,._,;,,,.

_.

"---

..

..

""'-''"'"""'"' ,-.-.~ ....-r~w~-~'"_,:.,?'"' ,

----

>-~w~,

,.~:~~ "''~ ,. ,.. .,.a.-,~,_;,m .~,~-~.-~-, ~=~--~.,-~"~;,,,~u~"'""'-'J)

,;

Listen again and repeat after the response. Pay close attention to pronunciation, intonation,
and word stress. llfifjfi

~Step

Now give your own spoken response to the prompt. Try to incorporate useful expressions
from Step 3, while also paying attention to your pronunciation and intonation. Record your
time.
Response:

Speaking time: - - - - - seconds

Jn\egrated Speaking: Test Question5 5 and 6

403 - , '

Practice 2 - Conversation
~Step

(I

listen to a conversation. Take notes on the problem presented and the possible solutions

suggested.

i@fJii

Problem:

Possible benefit: ----~------------

Solution 2: - - - - - - - - - - - - - - - - - - -

interview:
a meeting in which an
employer asks questions to
someone applying for a job
frequently:
often
delay:
to be slowed down
willing:
wanting or agreeing to do
something
lend:
to give something to
someone for a short period
of time

Possible benefit: ---------~-------

'/.

On your own, think of some of the possible benefits to each solution suggested in the
conversation. Write them in the extra spaces provided above.
)

~Step

Read and think about the prompt below. Answer the following questions in complete sentences.

The students discuss two possible solutions to the woman's problem. Describe the problem.
Then state which of the two solutions you prefer and explain why.
1. What is the problem?

2. What should the man do?

3. Why?

ll
~Step

Now create your.own response to this topic using words and expressions from Steps 1 and 2.

[
-"'

The woman s u g g e s t s - - - - - - - - - - - - - - - - - - - - - - - - - I think he s h o u l d - - - - - - - - - - - - - - - - - - - - - - - - I think this is the better option b e c a u s e - - - - - - - - - - - - - - - - - - - -

~-

- - - - - - - - - - - - - - A l s o , the rnan - - - - - - - - - - - - - - - - - - - - - - - - - - - - T h e speakers agree that _ _ _ _ _ _ _ _ __


________________,so ________________

1.:
t

n Now listen to a sample response. How does it differ from your response? Write down any
" differences in information or phrasing. i@fJii
''~'''''"'""=~=-"--~~="-='-~~"~"''~~---"-"-'"''''"'"'"

~t

........ _

=--..

__.._..,_....__ _ _ _

-~-.~

I
I
I
I

I n and word stress.


I

Listen again and repeat after the response. Pay close attention to pronunciation, intonation,

~Step

Now give your own spoken response to the prompt. Try to incorporate useful expressions
from Step 3, while also paying attention to your pronunciation and intonation. Record your
time.
Response:

S@iJLij

J&.f..

Speaking time: _ _ _ _ _ seconds

Integrated Speaking: Test Questions 5 and 6

405 -

Practice 1 ~ lecture
~Step

n Listen to a lecture. Take notes on the information presented.

iijij!Jlii

Main topic: - - - - - - - - - - - - - - - - - - -

Key point 1:
Example:

Key point 2:
Example:

adaptation:
a change that makes a
living thing more able to live
In a place
conducive:
helping something to
happen
blend in:
to be so similar to other
nearby things so you do not
notice them
drastically:
in a way that has a very
large effect
alter:
to change
. disguise:
to change something to
prevent recognition

,-.-:,'

l> Step 2
Read and think about the prompt below. Answer the following questions in complete sentences.
Using points and examples from the lecture, describe the two ways in which animals
camouflage themselves.
1. Why do animals camouflage themselves?

2. How does the Arctic fox camouflage itself in the winter?

3. How is the walking stick bug able to hide from predators?

~Step

[[

Now create your own response to this topic using words and expressions from Steps 1 and 2.

The professor says that animals - - - - - - - - - - - - - - - - - i n order to


- - - - - - - - - - - - - - - - - - - O n e way they do this is _ _ _ _ __
- - - - - - - - - - - T h e Arctic fox _________________
_______,.The Arctic fox is ___________________ and
- - - - - - - - - - - - - - - - - - - - - A n o t h e r way that animals _ __
__________________ is by ______________
_ _ _ _ _ _ .The walking stick, for e x a m p l e , - - - - - - - - - - - - - - ---~-'This

causes ________________________ ,

() Now listen to a sample response. How does it differ from your response? Write down any
>,differences in information or phrasing. IMftfUi

r"

l
I
rI
I

I
I
1

I
I

() Listen again and repeat after the response. Pay close attention to pronunciation, intonation,

and word stress. O@fll:B


~Step

Now give your own spoken response to the prompt. Try to incorporate useful expressions
from Step 3, while also paying attention to your pronunciation and intonation. Record your
time.
Response:

Speaking time: _ _ _ _ _ seconds

Integrated Speaking: Test Questions 5 and 6

407

-:-----!

Practice 2 - Lecture
~Step

n Listen to a lecture. Take notes on the information presented.

ld!ijfliji

role:

Main topic:

purpose
conflict:
a situation w_here two things
cannot exist together
incompatible:

Key point 1:

unable to cooperate or
coexist
diagnosis:
a statement about what is
wrong with a medical
patient
submit:
to accept that someone
else has power over you

Example:

Key point 2:
Example:

~Step

.. .

Read and think about the prompt below. Answer the following questions in complete sentences.

r~ ~ ~~~~ ~::~~s-:~~:~:~~1:~~;~~;~:~;::~:~:.~::~;i~:;::::~~ ;~:; ;:;:~:~;li:t: ~an


"

~,_ ~ ...--,~~""'"<--"-'-='""" ,.,,.,_._,......_-~.,~~-m~~=' ,..,~,~~,,"....--'="- N-~-<> "'r..~

..-<

, -.- - - 'm-'<' '

"~~-<-'>c,.

v>h -,,-,,.,

~-''"'

. '"

"'"~-

.,.,.,. '

1. What are two ways that role conflicts can occur?

2. How can a role conflict arise when a father is also the coach of a baseball team?

3. How can a doctor experience a sudden change in role? How does this create a conflict?

:-~.":\","--

408 Chapter 6
-.-,.-

,--,

~Step
J

Now create your own response to this topic using words and expressions from Steps 1 and 2.

The professor says t h a t - - - - - - - - - - - - - - - - - happen because


- - - - - - - - - - - - - - - - - - - - - O n e way that they occur is
- - - - - - - - - - - - - - - - - - - - F o r example,---------- - - - - - - - - - - - - - - - - A n o t h e r way that _ _ _ _ _ _ _ __

[f

_________ occumis _____________________


For e x a m p l e , - - - - - - - - - - - - - - - - - - - - - - - - In this situation,

Now listen to a sample response. How does it differ from your response? Write down any
differences in information or phrasing. lil!f!fil'

I:..
,v.

I"'

I
I
I

t.. ..

I n and word stress.

Listen again and repeat after the response. Pay close attention to pronunciation, intonation,

I
I
I

Mi*?'

~ Stero 4
"
Now give your own spoken response to the prompt. Try to incorporate useful expressions
from Step 3, while also paying attention to your pronunciation and intonation. Record your
time.
Response:

Speaking time: _ _ _ _ _ seconds

Integrated Spedki119: Test Questions 5 and 6

409 - -

Instructions: Choose the best word or phrase


to complete each sentence.

1. You use a wheel to - - - - - a car.


(A) unwind
(B) suspect
(C) steer
(0) recruit

6. We both agreed that bringing 100


sandwiches to the party was

(A)
(B)
(C)
(0)

abstract
excessive
liable
lengthy

7. My favorite
2. When I was

, I used to get
ice cream whenever I was good.

(A)
(B)
(C)
(0)

social
lost
hectic
little

3. Students who cause - - - - - are


often punished.
(A)
(8).
(C)
(0)

mischief
degree
maintenance
notice

of the music
player is its fantastic sound quality.

(A)
(B)
(C)
(0)

vessel
companion
technique
feature

8. I am
that my painting was
chosen to win the contest.
(A)
(B)
. (C)
(O)

wonderful
delighted
preferred
hazardous

9. Most managers want to make a good


4. I like going into the mountains to see all of
the _ _ _ __
(A)
(B)
(C)
(0)

collision
wildlife
material
motivation

5. We were all very surprised by my


grandmother's
visit.
(A)
(B)
(C)
(0)

unannounced
demanding
lonely
attentive

,'
I. .
I' :
I''
;~.: 1

impression on customers _ _ _ __
after they enter the store.
(A)
(B)
(C)
(0)

drastically
literally
immediately
willing

10. The university decided to _ _ _ __


skateboards on campus.
(A)
(B)
(C)
(O)

manipulate
ban
utilize
refrain

>

'..-'

',

.~

'}

,.

1'
I

11
,,

:r

I'
'

1--I

410 Vocabulary Review 3

11. The group decided to meet at the


_____ to relax for a while.

(A)
(B)
(C)
(D)

diagnosis
career
student union
motivation

12. The student government is _ _ _ __


new sources of funding.
(A)
(B)
(C)
(D)

looking into
keeping from
staying on top of
getting along

13. The university has received many


_____ about noise in the dorms.
(A) conflicts
(B) steps
(C) alternatives
.(D) complaints

14. If you want to

a boat, it is
. best to pick a windy day to do it.
(A) fund
(B) sail
(C) suffer
(D) associate

15. Many universities assist graduating


students looking for a job to begin their
.(A) ball
(8) career
(C) flier
(D) common area

Instructions: Choose the word or phrase


closest in meaning to the underlined part.

[l
"'

:
!

'

'

16. John was involved in a traffic accident


yesterday, but he is OK.
(A)
(B)
(C)
(D)

'

reward
notification
collision
conflict

17. You should not handle unsafe chemicals in


the laboratory.
(A)
(B)
(C)
(D)

hazardous
insignificant
unreasonable
incompatible

18. I believe that we might have a surprise quiz


tomorrow.
(A)
(B)
(C)
(D)

suspect
submit
delay
witness

19. The likely result of turning the paper in late


will be a ten point deduction.
(A)
(B)
(C)
(D)

role
notice
interview
outcome

20. The fall season changes the color of the


leaves to bright red and yellow colors.

(A)
(8)
(C)
(D)

leaves out
lends
alters
conveys

Vocabulary Review 3

411 - -.. .

,I

21. I wore a university shirt so I would !iUn with


the students ihere.
(A) blend in
(B) back out
(C) look into
(D) bring about

26. Many people choose to pass on


information through emails.
(A) relay
(B) delay
(C) disguise
(D) manipulate

22. The $100 they're charging for parking


passes is an unfair price.
(A) insignificant
(B) unreasonable
(C) social
(D) cost-effective

27. My friends enjoyed the speech, but I


thought it was too long.
(A) intriguing
(B) lengthy
(C) demanding
(D) hectic

23. Please give at least a week's warning if you


can't attend the meeting.

28. I often like to relax by listening to music


alone in my room.
(A) submit
(B) associate
(C) employ
(D) unwind

(A)
(B)
(C)
(D)

excuse
notice
interview
series

24. The student's essay omitted too rnany


important details.
(A) stayed on top of
(B) kept from
(C) backed out
(D) left out

29. I'm so tired that I'm likelv to fall asleep right


when I get home.
(A) liable
(B) lonely
(C) lost
(D) willing
>,

25. I am planning to attend the dance this


Saturday.
(A) vessel
(B) feature
(C) ball
(D) step

~"r>.,-iii:

'-i

ttl

412

Vocabulary Review 3

30. Great goalies know many skills that help


them defend the goal.
(A) companions
(B) techniques
(C) degrees
(D) supervisors

Instructions: Write the missing words. Use the


words below to fill in the blanks.
companions
distracting
excessive
utilize
lonely

suffer
frequently
refrain
keep them from
social

There are many students who want to


31.
their music players at
school. However, there are two main reasons
that students should 32. _ _ _ _ __
from doing this. For one, music players can be
extremely 33.
to you and
other people around you. If someone uses a
music player in class, his or her grade will
definitely. 34.
. People
who 35.
use music
players are also annoying to other people
because sometimes they play music so loudly
that everyone can hear it. Secondly, people
who Y.~e music players in school aren't being
36 . .,,,
. When they listen to
music a(n) 37.
amount,
no one is able to talk to them. I think this can
make them 38.
. It can
39.
. interacting with their
40. _ _ _ _ __

Instructions: Choose one wo'rd from each


group that does not belong.

41. notification
notice
42. associate
connect
43. witness
guide
44. intriguing
interesting
45. insignificant
small

announcement
alternative
demonstrate
correlate
steer
manipulate
exciting
demanding
surprising
unimportant

Instructions: Label each pair of words as


similar (S) or opposite (0).

46.
47.
48.
49.
50.

. submit

resist

relay

convey

employ

apply

social

lonely

associate correlate

Vocabulary Review 3

413

-~

Instructions: Choose the best word or


phrase to complete each sentence.

1. The invention of the computer has


changed how we
communicate.
(A)
(B)
(C)
(D)

frequently
constantly
drastically .
literally

6. My schedule next week is so


that I don't know how I'll get anything
done.
(A)
(B)
(C)
(D)

>.

hectic
determined
insignificant
preferred

7. For many people, a dog can be a great and


loyal

2. The conference included a


of speeches by business owners.

(A)
(B)
(C)
(D)

series
costume
caregiver
psychologist

(A)
(B)
(C)
(D)

wildlife
companion
degree
supervisor

8. When someone broke the lamp, everyone


thought it was Tim's

3. He was

in traffic for about

an hour.
(A)
(B)
(C)
(D)

sailed
delayed
suffered
banned

(A)
(B)
(C)
(D)

conftict
excuse
fault
role

9. The student was asked to leave the


classroom because she was being

4. I had planned to go to the movies, but I


had to
(A)
(B)
(C)
(D)

5.

at the last minute.

back out
leave out
keep from
get along

a car accident while I was


driving home from school.
(A)
(B)
(C)
(D)

steered
recruited
funded
witnessed

(A)
(B)
(C)
(D)

unannounced
incompatible
willing
disruptive

1:

..
;

10. I asked a friend for help after I couldn't


understand the difficult
''.<

(A)
(B)
(C)
(D)

material
motivation
mischief
complaint

)
fo

; - - - 414 Voccibulary Review 4

'

. L:' .

t: '.;'<!"~ll1Vt~n;in<~o;:_p~~;sg.\-(_.1,c0.~Nd;c,;:-;,_.,,,~--.._.;;,;,h""..N~~.:;,~<l-~:.'-'"K~\'f(;J.;r,-:,,-Jt,_ .. ,;.~,..,.:,o

..

11. Despit~.all of the problems, my school


project still
. all right.
(A)
(B)
(C)
(D)

Instructions: Choose the word or phrase closest


in meaning to the underlined word or phrase.

I
I
I
I

witness
disguise
apply
lend

I
I

when

hazardous
little
social
lost

14;, Some business

are in

charge of over 100 people.

(A)
(B)
(C)
(D)

supervisors
collisions
common areas
faults

15. Doctors must first reach a _ _ __


before they can treat an illness.
(A) excuse
(8) diagnosis
(C) conflict
(D) series

16. Researchers must verv often verify that their


. results are correct.
(A) immediately
(B) literally
(C) drastically
(D) constantly

(A)
(B)
(C)
(D)

~.i
-

.....

'

41:

'.'.':.t....

guide
refrain
relay
manipulate

18. After much research, the scientists


connected the vase fragments with ancient
Roman civilization.
(A) associated
(B) altered
(C) submitted
(0) conveyed
19. His purpose in the company is to handle
customer complaints.

(A)
(B)
(C)
(0)

companion
feature
role
technique

20. Many students worry that their grades will


worsen if they get sick and miss class.
(A)
(B)
(C)
(0)

],

17. I couldn't find the post office, so I let a


friend lead me there.

13. I realized that we were


the road came to a dead end.

(A)
(8)
i (C)
(D)

Cl

kept from
came out
got along
brought about

12. I joined the painting club so I could finally


_ _ _ _ my knowledge of painting.

(A)
(8)
(C)
(D)

c:

delay
unwind
suffer
suspect

Vocabulary Review 4 415

-1

i .

'

21. Banks often loan money both to individtJals


and new businesses.

opinion of the author.

(A)
(B)
(C)
(0)

(A)
(B)
(C)
(0)

sail
ban
fund
lend

22. When presenting difficult information, it is


!.

26. The new book caused a change in my

helpful to use visual aids.


(A)
(B)
(C)
(0)

correlate
employ
steer
convey

23. His experiment shows that some animals


have very good memories.
(A)
(B)
(C)
(0)

utilizes
suspects
demonstrates
associates

24. While the first half of the play was boring,


the conclusion was verv good.
(A)
(B)
(C)
(0)

wonderful
delighted
lengthy
preferred

25. I bought a new program, but it was


unable to work with my computer.
(A) insignificant
(B) liable
(C) willing
(0) incompatible

~"-416 Vocabulal'yRel.liew4
..1

blended in
brought about
left out
backed out

27. The first stage of a project is planning.


(A)
(B)
(C)
(0)

role
step
notice
adaptation

28. The best way to deal with a problem with a


roommate is to communicate your
problems to him or her.

(A)
(B)
(C)
(0)

submit
suspect
refrain
convey

29: The history of the whale is a fascinating


topic.
(A)
(B)
(C)
(0)

intriguing
demanding
determined
disruptive

30. Eating fish is a good substitute for eating


meat.

(A)
(B)
(C)
(0)

reward
excuse
alternative
flier

'.i:

;~\..r.

. r~tr

Ci~,::'-

"><

,,,.,.,,,1k-=.:;l
., ..... . .
"@'"'''"''"'""''"' '"''"' "

'j\

, i

' '

Instructions: Write the missing words. Use the


words below to fill in the blanks.
technique
adaptations
altering
blend in
disguise

~.
I

""'""''' ,,,.,,,.,,,,,, " ' '.. .

feature
wildlife
literally
drastically
outcome

There are many 31. _ _ _ _ _ _ _ that


allow animals to 32. _ _ _ _ _ __
themselves in the wild. One such
33.
is called camouflage.
Through camouflage, animals can
34.
make themselves look
completely different by 35. _ _ _ _ __
their colors. By changing colors to make
themselves look like their environments, they
are able to 36.
with their
surroundings. Some 37. _ _ _ _ _ _ _ ,
stich as the Arctic fox, changes
38.
in different seasons.
T.h,is important 39.
has a
v~ry'positive 40.
for the
Arctic fox because it is able to hide from
predators that it would otherwise be unable to
defend itself from.

Instructions: Choose one word from each


group that does not belong.

41. flier
reward
42. interview
discussion
43. complaint
reason
44. determined
driven
45. demanding
difficult

notification
notice
meeting
diagnosis
excuse
explanation
motivated
excessive
hectic
challenging

Instructions: Match the words that have


opposite meanings.

46.
47.
48.
49.
50.

insignificant

(A) unwanted

preferred

(B) reluctant

abstract

(C) important
(D) distracted
(E) real

willing
attentive

I
I
I

Vocabulary Review 4 417 -

,-:,._ ___ c.:__ ___

-~--.-----

The tips below can help you to improve both your fluency and
clarity of speech:
During the speech:

Open your mouth while speaking. Try not to mumble.


Pay special attention to the pronunciation of content words and key terms.
Stress each syllable correctly and accurately.
Clearly pronounce both vowels and consonants.
Smoothly link sounds between words within a phrase and in consonant
clusters.
Change pitch between stressed and unstressed syllables.
Speak in sentences or phrases, not word by word.
Speak with appropriate speed, not too quickly.

\/llhen practicing:
Practice speaking by writing down every word you say and marking each place
where you pause or vary intonation.
Examine this transcript of your speech and look for possible mistakes. Practice
these parts again, focusing on correcting the previous mistakes.
Record an.d listen to your speech. Note any areas for improvement.

Pronunciation, Stress and Intonation, Pausing

421 ---.

. \.. ...
..,:---'.

,,-

;.

,-

__ .. c- .. ._.--_'

-,:::.::i.'.""-'"-_ .. __-._,.,:.--'_.-, ' _

_-:-.":<:.:.-:-._ . .''-;,,.,.-.:.',_;

:<::(;; .: J_

.;_:<;;).'-.-,-:

'_>

'

Word stress has a large effect on speech clarity. If stressed on an incorrect syUable, a 11\/ord cannot
be easily recognized by listeners, Therefore, the message may not be delivered successfully.
Thus,
it' ..is essential
for
a speaker
to
stress
the proper syllables
in his or her speech,
.
,_ - ..
_: '.'._.
,- ",_'
- .'. -:
' :
.,_-;.. '._., '.;.<:;. ,
__ ,_,,
. -- _:.;_,._ .. _.. _.. _;
!

,_:'"

:/ . . . ; .. .' ....

'''i,;.;

Stress related to parts of words


.

'\

In many cases, the syllable stressed in a word changes when the form of the word changes. For
example, ,in words that end with the suffixes below, the primary stress usually comes before the
suffix.

J,,
!'

9 Nouns that end with: -let, -ing, -ism, -ent, -ery, -ity, -lion, -sion
9 Adjectives that end with: -ic, -ical, -able, -en
Many two-syllable nouns, when used as verbs, are stressed on the second syllable.
9 record, permit, address, object, contract, suspect, present, convert, project, progress

,,,)
,;

'

!> '.5ttep J

n Listen and mark the primary stress in each word. How does the placement of stress differ
between the words in each pair? IM!!lfiij
1. a.
2. a.
3. a.
4. a.
5. a.

method
economy
academy
luxury
drama

b.
b,
b.
b.
b.

methodology
economic
academic
luxurious
dramatic

6.
7.
8.
9.
10.

a.
a.
a.
a.
a.

recommend
capable
prefer
photograph
negotiate

b.
b.
b.
b.
b.

recommendation
capability
preference
photography
negotiation

n Listen again and repeat the words.


Read the following sentences and circle the syllable in the underlined words that receives the
primary stress.
1. Do you have a campus parking permit for your bike?
2. I hope my professor can advise me on which course to take,
3. She has to present her project to the class tomorrow.
4. My friends and I are going to the protest downtown this afternoon,
5. Did you hear that Jane and her band will record an album this summer?
6. Unfortunately, my parents will not permit me to go skiing this weekend.
7. The police have arrested a suspect in the campus computer lab robbery.

8. In biology, we are studying how plants convert sunlight into energy.

n Listen and repeat the sentences.


-- .,

-----

_,

,- ..

mm

Stress on phrasal verbs


Two-word verbs or phrasal verbs, which are made up of a verb and a preposition or adverb, are
very common in English. In two-word verbs or phrasal verbs, it is normally the preposition or
adverb that receives stress, not the verb.

Ex. drop

m. figure out

The meaning of a phrasal verb is distinct from the meaning of its constituent parts. Do not confuse
these phrasal verbs with other verbs that are followed by a preposition without a changed
meaning.

(<'

1'

,,

-f;i

..

''

~ -{'.,

~Step

Phrasal verbs

Verb + Preposition

look for
build up
hold up
find out
think over
check out
turn off
hold on
stand for

look at
listen to
point at
depend on
think of
talk about
search for
respond to

Read the following sentences. Circle the word that you think receives stress in each underlined
. phrase.

1. The researchers found it out very recently.


2. The robber held up the convenience store.
3. Let's go check out the new restaurant in the student union.

4. Can you help me? I am searching for a journal on anthropology.


5. Do not point at her. That is rude.
6. People often say that I take after my father.

n Listen and repeat the sentences.

@tn

Pronunciation

--

--

--- . -

--

--

423 - -

-,-

;--.: ..':.\. :;

Senten~~ stress andintorlation areveryimportallt ill E~gli~h.ff1~.r6\ithrn i~ ~pok~ll English


alerts listeners to the message presented. Words or phrases impprtant to the content of the
message tend to be stressed in. English, whereas the words or phrases that are not important
are not stressed. In fact, the sounds of these words tend to weaken.

Stress and Intonation


Sentence stress related to content words

:l

Certain words within a sentence are given importance because of the meaning they convey.
These are known as content words. Stressed syllables are pronounced longer, pitched higher,
and spoken slightly louder.

Ex. Tea has less caffeine per cup than coffee.

~Step

Practice saying the following sentences. Be sure to stress the content words.
1. People classify products based on price and appearance.

2. The university should advertise that the dorms are open in the summer.
3. Echoic memory lasts for only a few seconds.
4. The magazine might have to shut down.
5. Giving rewards will teach children to behave well.
6. I was very proud of my brother when he was accepted into medical school.

7. I feel comfortable talking to my sister about my problems.


8. Students should work for a year before going to university.

n Listen and repeat the sentences.

@f!fj

!>Step 2

n Listen to the paragraph. Write only the words you hear most clearly.

o@fildl

Reduction of unstressed words


"~""''-"''""----.-.------'"'""-'"""~ "-"-'"~'"'"'-~"'"""'"'~~--~"~'"'"'''""'"''"'"''"""~"""'\

Function words that have little or no meaning other than the grammatical idea they express are
weakened or reduced. Some sounds in these words are obscured or omitted. For example "can"
becomes /kn/, "have" becomes /hv/, and so on. The groups of words below are commonly
subject to reduction:

c> Articles: a, an, the

c>Simple prepositions: in, to, of, etc.

c> Personal pronouns: I, me, he, him, it, etc.

.:>Possessive adjectives: my, her, your, etc.

.:>Relative pronouns: who, which, that, etc.

.:>Common conjunctions: and, but, that, as if, etc,

.:>State verbs: be, have

c> Auxiliary verbs: can, will, should, etc .

3z

.. ~.....,...........,.,-.~"~=""-""'""'r'<"""'""""''"-'u"'-'v~-~--u==+.,......_,,,.,....,. .,~==-'<>"'""""''~"'-""'-~"-'"''"'''"=,_.,~,=.,-'~'"-U.~""""~"~"~'""'"~~=.>..-=~u~J.LI~'-'-'<>''<'>~.-

~Step

n 'ffii

Listen to the following sentences. Circle any underlined words that are weakened or reduced.

1. The people who moved out to other cities were safe, but those who were in the city were in danger.
2.,~ He is the one in my family who understands my dream.
3. The studentscannot access this section but the teachers can.
4( The government asked him to stop campaigning against the policy.
5. They wanted to create something new and innovative.
6. For homework, you all should have read a bit about wind power.
7. I know I look young, but I am a student at this university.
8. Mathematics is an important aspect of university studies.

Listen again and repeat the sentences.

~Step

Listen to the paragraph. Write only the words you hear most clearly. ll'MlfiW

.. """" .............................~... .,\

Stress and Intonation 425 - - '

--- j -

. .. ':.t....

Intonation
The focus word in a sentence has the most emphasis so that the listener can hear it clearly. At
the beginning of a conversation, the last content word in each sentence is usually the focus of
meaning. Therefore, the primary stress in these sentences usually falls on the last content word,
especially when it is delivering new information. This stands in contrast to less important words.
Ex. He did not include a central HERO.

;.'

The sound of the speaker's voice rises on the focus word and then fC!lls. English listeners pay
attention to this change in pitch.
~

The dog chased a rabbit.

re tep

Read the following sentences. Find the final content word in each sentence, and underline its
stressed syllables.

,: i

1. I had lost an important assignment due to computer problems.


2. That gave me the opportunity to learn about a new culture.
3, I never went on a trip with those friends again.
4. Universities need money for computer labs.
5. Many people think that teaching by example is best.
6. They look more interesting and have more character.
7. A reward system is a good way to teach children to behave well.

n Listen and repeat the sentences, letting your voice rise on the stressed syllable and then
drop afterwards. @iii'

Listen and circle the focus word in each sentence.

Ex. In warm years, they are;bigger;than in cold years.


That will tell us the date at which the house was built.
Mom and Dad want me to attend.
They can cause all kinds of trouble.
I am sure I will land a good job after graduation.
In the past, the television was a luxury.
6. In addition, they are beneficial to mankind.
1.
2.
3.
4.
5.

l'li!lifi""

'

Having appropriate pauses .is also an important part of spoken Englisho Paus~s are gi~en
.after each message unit in order to give listeners time to process the information. If a speaker ..
speaks too rapidly or without thought to the grouping of the information presented, listeners
may have difficulty distinguishing the important content of the message.

>.:.

Pausing

r ;~u:i~g n~::~~::::~~~~~~:n~~ti:~:~~::~;~-~::: t~~h:::~er~~~~:;-:~::~~~~;;~:;~~~~:~;;~


..

>;

I
'1

be aware of pauses and breaks within sentences. Pause after each thought group such as a long
subject, prepositional phrase, or that-clause. Furthermore, it is helpful to pause after a complicated
idea so that the listener has time to understand it. Finally, there should also be a pause after
transitional words.

J "'
3z

''"""-'-'"''""J~"'' >~'-< <dAU,.;,,_.~=~-'''~'-'<'~'>,>.>-;><=.~.<,~>"-''-'~"' ~~~~~ ............. R~~-~-.,_;,.O'<o-~.............- - ~ .. ~p=<-~_.._~~~~=-~~...... ,,._~.,,.,.,,_~,.,.,,..,..._,~Z.~<>~"'-'"""''"~~

~Step

Read the following sentences. Circle any"/" that indicates an appropriate pause.

I
I
I

1. Although we I had not finished I we decided I to. go home.


2. When she stepped I off the boat I she immediately ran I to her car.
3. It I was raining so hard I all day I that they did not I leave the house.
' 4,, If the alarm rings I put down your books I and slowly I leave the building.
" 5. The final test I will be two hours I long I and will count for I twenty-five percent of your final grade.

..

(/ Listen and repeat the sentences. Check your answers. idl!ilfiil

I
I
!

Practice saying the following sentences and write a "/" where you pause.
1. In my opinion, students should not have to pay to go to university.
2. To begin, echoic memory is very short, lasting only three or four seconds.
3. Charities, both local and international, rely on the generosity of individuals to help the less fortunate.

4. In summary, then, I think that teachers should give unannounced tests in class.
5. While a strong brand name can rnake a product more appealing, it can also help customers remember
a company or product.
6. Many people, however, feel that animation done by hand is more artistic than computer animation.
7. It took five years, but he finally achieved his goal.
8. In the case of many oral traditions, it is impossible to identify the stories' authors.

n Listen and repeat the sentences.

i1tl!fif!.1

Pausing

427

-j

----'-~--j _ _.,.~.

,-__._ ., ,, .:.. -.. _-.\ ~ i : ; '. ~.,. ' .. '

~-

- ,' .

""'"-_

The TOEFL iBT Writing Section------------------------------------ 432


Preview --------------..------------ ------------------------------------------ 437

Part 1

integrated Writing

~~WA'1>'.ll.'W.~ll~.i"A11'U.m1:~r~<~J!'.'::, ,_,;-H,_,r,_<J;-~,,1-"-~f"'''"'

';e

Chapter 1

Organizing Information -------------- ----------------------- 447

Chapter 2

Paraphrasing ----------------- --------------------------- 461

Chapter 3

Making Connections------------------------------ 479

Vocabulary Review 'I ------------------------------------------------- 496


Vocabulary Review 2 ------------- ---------------------------------- 498

Part 2
Chapter 4

Brainstorming ---- ----- __ .. ___________ ---------------- 503

Chapter 5

Writing Thesis Statements and Topic Sentences ------------ 521

Chapter 6

Making Ideas Flow --- ----------------------------

529

Vocabulary Review 3 ----------- ---------------- . ----- ---.. ---------------------- 548


Vocabulary Review 4 ---- ------------------------------------------------- 552

Chapter 7

Verb Forms

Chapter 8

Sentence Formation

Answer Key Writing

- - -- - - ------------ -- 559
567

---

-----

777

'

'
The writing section of the test is designed to assess your iiJbility
to organize and support your ideas
in essay format You will have two writing tasks. One task is based on both a reading and on a lecture.
You will be required to summarize the information you have read, and to relate the information
heard in the lecture to the information in the passage. The second task requires you to generate an
essay based on your own experience. In this second task, you will be given no material to work with;
it will be based completely on your own ideas.

!'

Question Types
Questions for the writing section of the TOEFL will appear in the following order:
Question Type
1

Type
Integrated:
250-300 word

Suggested Time

Response Length

20 minutes

150-225 words

reading
250-300 word
lecture

Description
Contrast
information
presented in the
reading passage
with information
presented in the
lecture

Independent

30 minutes

300+ words

---~---

.,

~~.~ . ;

1~~'~,1---.

f(<';li
~ i !l

432 The TOEFL ii iBT Writing Section

.'l1 ; "~--.,-:- - -, ---;_;

-,,,'-1--' _,,

'-.--=:

-:- -:'.'.~ f~:'>'--,~~,-_!,'_:> r '.~;o:~_-;:-f ,.'. ~- --- ~

Present a personal
opinion or
describe an
experience,
including details
and examples

;-.'

rt

Study Tips
Integrated Writing
o Look for magazine or newspaper articles that are about 300 words long. Time yourself as you
read the articles. You should aim to read 300 words in less than three minutes. After reading, try
to outline the article. Then, without looking back at the article, try to write a summary of the
article from your outline.
o Practice listening to short reports given in English. There are many websites where such
reports are available online. While you listen to a report, take notes. Try to summarize the
report from your notes.
o Look for a variety of exercises in writing books you have studied that practice paraphrasing.
Study the methods such books suggest for paraphrasing. Focus especially on exercises that
practice the usage of synonyms and/or changing the grammar of given sentences in order to
paraphrase them.
o Review useful phrases and expressions for citing sources. Pay attention to where these citation
phrases can be placed in sentences and how the phrases should be punctuated.
o Practice your typing skills in English. You must type your essay for the TOEFL.

\>

I.. \
<

);

'

"'
"O

~
2S

G'\

-o

~
J:l

. Independent Writing
1p Practice writing TOEFL essays. Get a list of sample topics at www.ets.org/Media/Tests/TOEFU
pdf/989563wt.pdf. Select a topic at random and write a 30-minute draft essay. Correct the
essay, with the assistance of a teacher if possible, and rewrite it with the suggested corrections.
o When you are studying a group of writing topics, practice sorting the topics into "opinion" or
"experience" topics. This will help you quickly determine the appropriate writing task you will have
when you take the test.
o Practice outlining ideas before you write. You can.do this by taking five or six topics for writing and
making a short outline for each one. Don't write the essays, just write the outlines. You can also
use different techniques for prewriting, such as making simple charts of information, drawing
bubble diagrams, or creating lists of ideas.
o Look for a variety of exercises in writing books you have studied that practice writing introductions
and conclusions. Study the methods that these books suggest for writing introductions and
conclusions. Pay attention to tips for beginning and ending introductions and conclusions.
o Practice your typing skills in English. You must type your essay in the actual TOEFL.

I
I
I
The TOEFL iBT Writing Section

433 - -

Test Management
For the integrated writing task, you will read a passage and listen to a lecture afterwards. The
reading passage disappears during the lecture and reappears afterward, so do not worry about
taking notes on all of the key points in the reading. You will NOT be able to hear the lecture again,
so it is important to take good notes while you listen.
You must type out your answers. You can use icon buttons at the top of the screen for editing. The
editing tools include cut, paste, undo, and redo.
Keep the style of essay writing in English in mind. First select a main idea, explain it clearly, then
support and develop it using details and/or examples. Be sure your essay has a logical flow. There
should be a reason for every sentence in your essay. Such reasons include introducing a new
example or detail to support the main idea, or explaining or supporting an example or detail
mentioned previously. Do not write any sentences that are unrelated to your main idea or that do
not fit into the organizational structure of your essay just to increase your word count.
r

Make every effort to use effective language and appropriate sentence structure and. vocabulary. Try
NOT to use vocabulary or constructions that you are not confident with, as this will increase your
chances of making errors.
Use a variety of language. English has a large number of synonyms and analogous constructions,
so using the same construction repeatedly is considered poor style.
Keep the 50-minute time limit for the entire writing section in mind. Remember that graders are
expecting to read draft essays, not finely polished final products. If you find yourself stuck on a
particular part of your essay, it is best to move on and complete the essay, then go back and fix
the difficult area if time allows.
G>

,\s--

..

'

''''~-- --~

---

Try to leave at least five minutes for revision. When revising, be sure to look for spelling or
grammatical errors (remember, there is no spell checker on the test), as well as ways to improve
the structure and flow of your essay.

434 The TOl:FL iBT Writing Section

--

'"''-",'"--

'..

, ___ ., __,

..

'

How Writing Will Be Scored


ETS graders will score test takers' essays for integrated writing tasks according to the following
scale:
Score

General Description

The essay includes important. information


from both the reading and the lecture and
appropriately explains the information with
regard to the prompt.

The essay is well organized; it may include


minor errors in grammar or word choice, but
the errors do not make sentences difficult to
understand.

The essay includes most of the key points


from the reading and the lecture as they relate
to the prompt. Some points may not be fully
explained or the explanation may be vague.

There are several minor errors with language;


some ideas may not seem connected, but
there are no real problems with clarity.

The essay has one or more of the following


problems: does not include a key point from
the lecture or reading, shows only a limited
understanding of the information, incorrectly
explains a key point, has problems with grammar
or word choice that make some sentences
unclear.

Errors in sentence structure and word choice


may make the meaning of some sentences
unclear; transitions or connections between
ideas are not always easy to follow; overall,
the important ideas in the essay can be
understood.

The essay has one or more of the following


problems: does not include sufficient
information from the reading, lecture, or
both, contains many problems with grammar
or word choice so the reader cannot follow
connections between ideas.

Errors in sentence structure and word choice


make ideas in the essay difficult to understand;
readers unfamiliar with the reading and lecture .
may not be able to follow the essay.

The essay includes few or none of the key points


from the reading, lecture, or both. The essay
is poorly written and difficult to understand.

Frequent and serious errors in grammar and


word choice make some sentences in the
essay impossible to understand.

The essay only copies words from the prompt


or is not related to the topic at all.

There is not enough of the student's writing


available to score.

. ;,
i.(

l
I

'

I
I
I

Key Points

I
The TOEFL0 iBT Writing Section

435 - '

How Writing Will Be Scored


ETS graders will score test takers' essays for independent writing tasks according to the following
scale:
Score

General Description

The response answers the question or prompt


well. The essay is easy to understand and well
organized.

There is good use of language, including


correct choice of words and idioms to express
ideas. Minor errors in grammar and word
choice are acceptable.

The response answers the question or prompt,


but not all of the ideas are fully developed. The
essay can be understood, but there are some
clearly noticeable mistakes in the writing.

There is good use of language, including a


variety of sentence structures and appropriate
range of vocabulary. There are some minor
errors in sentence structure, word form, or the
use of idioms, but these errors do not make
comprehension difficult.

The essay gives a basic answer to the question Little use of connectors to link ideas or show
or prompt, but not many examples or detais are progression of thought. Sentence constructions
provided. Most sentences can be understood, are very simple, or there are frequent errors in
but errors in grammar or word choice could more complex sentence structures. Word
choice and poor grammar may make some
make the meaning of some sentences unclear.
sentences vague or difficult to comprehend.

The essay is very short and not well organized.


The ideas are not connected and examples
are not explained.

Errors in grammar or word choice appear in


almost every sentence. Overall, ideas are difficult
to follow.

The essay is short and confusing. Little or no


detail is given to support ideas, and irrelevant
information is included. Some sentences
cannot be understood by the reader.

There are serious errors in grammar and word


choice.

,;.:. :

:)'_

'

i
'

I
.--~;

Key Points

:, '

. , l

0
.

The essay only copies words from the prompt


or is not related to the topic at all.

Not enough of the student's writing is available


to score .

Integrated Writing Task Preview


'

The following Is a sample Integrated Writing task similar to the one you will see when you take the test.
You will first be asked to read a passage. On the real test, the reading passage will remain on your

Wind power has been suggested as a renewable energy source that could one day be used
to power homes and businesses. By using large fan blades that rotate in the wind, entire towns
may one day be powered without the need for other power sources such as fossil fuels. Indeed,
there are three reasons why wind power is perhaps the most promising source of the world's energy.
First, wind power does not pollute the environment like many other sources of power. Wind
power is what is referred to as a clean energy source. It does not emit harmful greenhouse gasesthe gases that cause global warming-into the atmosphere like many other power sources do.
Thus, wind power is far healthier for the environment than other energy sources.
In addition, wind power is very efficient, meaning that with just a small amount of wind,
enough power can 9e generated to provide electricity to a small town. When many fuels are
burned, a lot of energy is lost in the form of heat. This means that not all of the energy produced
is converted to useable power. With wind power, much more of the energy created can be used
to provide electricity.
!\
Lastly, wind power is sustainable; in essence, wind power will never run out. The world is
-~,'
,,currently facing a shortage of energy sources such as fossil fuels. However, because the wind is
;/always present on the Earth, wind power will always be available. It will never run out.

<

[
m

-z

Cl

screen throughout the task.

'

If
ll

I
~

-<

Now read the professor's lecture below. On the real test, you will listen to this lecture with your headphones.
The lecture will NOT appear on your screen.

--- ---'---------------
There. are plenty of suggestions out there about how we can solve- this energy crisis that the
world is facing. One such solution is wind power. Now, it certainly sounds like an attractive solution.
However, wind power simply does not live up to its potential. Unfortunately, with all of the promise that
wind power has, there are a number of problems with wind power that make it an inadequate solution
to the energy crisis.
The first problem is that wind power does incleed pollute the environment. While it does not
produce the greenhouse gases that fossil fuels do, it emits a different kind of pollution: noise. You see,
when the large fan blades rotate, they create a lot of noise. So much noise that many residents of
towns refuse to live near where they are making wind power. It is that bad. So, really, wind powecdoes.

I'

pollute the environment-with noise.


Second, wind power really is not as efficient as many people believe it is, and it simply will
not be able to create enough power to replace fossil fuels. Most wind turbines today struggle to
operate at thirty"five percent efficiency. Let me explain: thirty-five percent efficiency means that

~---

..----,.---,.------------------------------Preview

;;

-----

437 - . -

only thirty-five percent of the power created by the wind can actually be used as electricity. Now,
thirty-five percent is a very small number. Even with thousands and thousands of turbines worldwide,
we would never be able to provide enough electricity for even one country.
Last, wind power is not particularly sustainable. While it is true that the wind will never stop
blowing altogether, it certainly changes day by day. We all know how weather can change: one
day it can be windy, one day it will not be windy. So what happens if you, say, have a few days in
a row that are not windy? Well, you have no power. So you can only really say that wind power is
sustainable in the few places in the world that get constantly strong winds.
So wind power doesn't really fulfill all of those hopes that many people had for it. It seems
that we'll have to continue searching for a suitable replacement for fossil fuels.
When the lecture concludes, you will be given a prompt that will ask you to give your response for this
task. A prompt for this task might be as follows:
Summarize the points made in the lecture you just heard, explaining how they cast doubt on
points made in the reading.
The reading passage will appear once again on your screen. You will have 20 minutes to plan and write
your response for this task. A sample outline of a response and a sample response can be found below.

Sample Outline
Introduction:
Topic
Main idea of the reading passage
Main idea of the lecture
Body:
Key point 1
Supporting information from the reading passage
Supporting information from the lecture
Key point 2
Supporting information from the reading passage
Supporting information from the lecture
Key point 3 (optional)
Supporting information from the reading passage
Supporting information from the lecture
Conclusion:
Summary of the main ideas from the reading passage and the lecture

ii .
f,.

~~(~".'~?'.--- 438
'I '
\;t{i r;,l
I

" .

Preview

[[
Sample Essay
/Topic
The reading and the lecture both discuss whether wind power is a good replacement for fossil fuels.
The reading says that wind power is a clean and efficient way to provide electricity. The lecture, however,
disagrees by stating that there are a number of problems with wind power.
Main idea of lecture

Main idea of reading

Key point 1 ~
The reading and the lecture first discuss the pollution made by wind power. The reading states that
wind power is good for the environment because it does not pollute the air. Conversely, the lecture says
that the turbines used to make wind power pollute the
Supporting info from lecture

I
I. .
I:
I

I
I
I

Supporting info from reading

Keypoint2 ~
The reading and the lecture next discuss the amount of energy created by wind power. The reading
says that wind power can create enoug~:./.rgy to power a small town. The lecture, on the/other hand,
clalfns that wind power will not be able create enough power to replace fossil fuels.

;y

environme~th noise.

Supporting info from reading

Supporting info from lecture

Keypoint3 ~
Lastly, the reading and lecture discuss the sustainability of wind power. The reading says that wind
power is sustainable because wind will never run out. The lecture, though, argues that wind power is
only sustainable in places that receive a lot of wind~.
/

Supporting info from reading

Supporting info from lecture


In conclusion, the speaker says that wind power is not as good a replacement for fossil fuel as the
reading states. While the reading claims that wind power may one day replace the need for other power
sources, the speaker argues that wind power alone cannot replace fossil fuel.
~, Summary of reading and lecture main points

Preview

. .!,

'1

439 -

Independent Writing Task Preview


The following is a sample Independent Writing task similar to the one you will see when you take the
test. You will first see a prompt on your screen describing the task.

II

Some students think that it is better to attend a university near where they live. Other students

. ~.

would rather attend a university far away from home. Which would you prefer? Give specific
reasons and examples to support your answer.
You

will then have 30 minutes to plan and write your response for this task.

A sample outline of a

response and a sample response .can be found below.

Sample Outline

~:

Introduction:
Topic
Thesis statement
Body:
Key point 1
Topic sentence
Reasons, examples, and details
Key point 2
Topic sentence
Reasons, examples, and details
Key point 3 (optional)
Topic sentence
Reasons, examples, and details
Conclusion:
Restatement of thesis and concluding sentence

]_~

v;_ _ 440 Preview

"

;U

1.,1
''

Sample Essay
/

Topic

Many students believe that it is better to go to universities close to where they live. However, I do

"'
"O

not agree with this. In my opinion, it is better to go to a university in a place that is far away from where
you live. There are three main reasons why I believe this.

25
z

~ Thesis statement

C'l

,._.

i ~.;.

Topic sentence

To begin, I think it is important to explore a new place when you go to university. Many people want
to stay in their own towns to go to university because they already know the city. In my opinion, it is
better to go to a new place because you get to have new experiences. For example, I want to go to
university in Seattle bec0use I know it is very different from my own town.

~ Reasons, examples, and details


Topic sentence ~
/..

\'

In addition, I think students should go to university far from their parents so that they can learn to
be.independent. Some students rely too much on their parents even after they leave for university.
However, students will learn quite a bit more about how to live by themselves if they go to university far
away. That way, parents will not be able to interfere in the students' lives.

~ Reasons, examples, and details


Topic sentence

Lastly, I think that students should go to university in a new place so that they can meet new friends.

It is better to make new friends when you go to university so that you can learn about people from other
places. You might not do this if you go to university in your own city.

~ Reasons, examples, and details


/

Restatement of thesis

Therefore, I think it is better to go to university far away from your own city. I think this makes the
whole university experience better.

I
I
Preview

441

I
I
I

ll

Integrated Writing Process

;
I

Read a Passage &


Listen to a Lecture
Note-taking

i
'

r1

I.

'
:~
-~'

:-f
'I'

'

Understanding the
Question (Prompt)

Final Copy

JJ
/

Planning & Writing


Summarizing
Citing information
Paraphrasing

Revising & Editing

)
.;f.

;i
'"..'~'~ 442
-,,

[
Preview

------ ---~-,.,.---_----- -----~,-/,;.~.:.

--::;,;.-_

..;

------.-~-~---,

,_,_

,Independent Writing Process


~z

zG'>

L.
r:_-.,_

"',,

Understanding the
Question & Brainstorming

~z

. G'>

JJ
Outlining &
Planning

Final Copy
I

Writing a Draft

~'-----

Revising & Editing

__ /

.. ---~---

1. Read the question carefully and be sure that you understand the task.
2. Make a mind map or a traditional outline.
3. Using your mind map or outline, decide on a thesis statement.
4. Make sure that the topic sentences support the thesis statement.
5. Make sure that all supporting ideas relate to the topic.
6. Read through your draft again while editing and making revisions.

Preview

443 - - .

..

---~---,_,--_-

-'

t;~l;i~~i1~i1~~~~~\:~;~1,:

' ,.

chap1~r1;'~;'c::orgal1izlng 1nf'lrrr1::itinn}!t?\:,;tW~F~

~J~1~;~~~'.~~;~~~raph;~;i~g i

.....

fefChapter:~.i:+!:,Making. Collne~tiohs
.:~.... ~ >.: , ;,''.~~'.;i,~~~;.:r;:,~,f.~7~\~:~>.. .

;;KVocaijgli\fyJteview
t. :.
.. '
..

'r;::

')--:~.;:::)., :..-;:.,~;~'!

.votaHl'.iiafy ll~vieW 2
. '

... :?;.;!i:,:S;\'. . .
"" '

. -~ ';

\\):_:,,.,

,:.';,

..

'. ~

:1',i:

:>!

'

'

>,;

Necessary Skills

Understanding information from both the reading and the listening passages
Taking notes on the reading and listening passages
Using information from your notes in your writing
Synthesizing the information taken from both the reading and listening passages

t
!'

Strategies
Though preparation time is limited in the writing portion of the test, it is nevertheless
important to use this time to plan the organization of your response. This way, your
response will be more relevant and coherent. An organizational process for preparing
your response is detailed below. In each step, there are certain things to keep in mind.

~--P_.r_oc_e_s_s_~jj~--------~p_tr_al_e_gY~.~~i~:~~_''._.~-="--_~>!

You will not see the prompt until after you finish reading
and listening, so taking notes is essential. Take notes on
major points from both the reading and the lecture.

Read, listen, and


take notes.

~Read the prompt and

Identify what kind of relationship between the reading and


the lecture the prompt asks you to discuss.

l_~nderstand the ta.sk.

Select "'oo
your notes.

~-1

Choose the points you need to discuss. Think about how


points in the lecture relate to points in the reading. The
listening passage will present details that challenge
information presented in the reading, present a counter
example, or describe the consequences of an attempt to
solve a problem presented in the reading.

I --------------- - - - - - - - - -

-------------~------~
-------~~----~-,

: Organize the ideas.

L ....-----------

---~

i-1nc~~~i1~;~~ati~-~-from both the reading and the lecture.


Clearly show the relationship between information presented
'. in the lecture and that presented in the reading. Limit the
time for organizing to less than two minutes in order to give
: yourself more time for writing and editing.

,,_J I----~---------------------------'
Organizing Information

447

-.'.~-, ;.

Urban Planning
l!.XH~~E.:i-~:.;o.i~1;;J!l.(',Xf.TI";;;;..;,.iK1''~''"'"'''~:F.c;,;C-f''"

~Step

1<!' :;'.;-;.;-;;

'<' ,.., . .-

Read the following passage. Then look at the note diagram and fill in the missing information.

,..-----------------------

1,
'i

Many metropolises today face a severe problem in terms of air pollution


and overcrowded streets, roadways, and bridges. One powerful public
policy tool that can remedy this problem is called congestion pricing. In
short, congestion pricing is a system by which motorists pay a fee to use
certain roads, bridges, and tunnels during peak times of the day. In other
words, a motorist who wants to use a busy street during rush hour will
now have to pay a fee to do so. There are three advantages to this policy.
First, congestion pricing gets more people to use public transportation.
By raising the cost of using the roadways, individuals are encouraged
not to drive. Instead, they are encouraged to find alternative means of
transportation such as mass transit (trains, buses, subways), biking, and
walking. Getting more individuals to give up driving and start using public
transportation will thereby reduce air pollution from vehicle emissions.
Second, congestion pricing, by taking more cars off the roadways,
means faster commuting times for everyone. Drivers that decide to pay
for the right to prive during peak periods will face less traffic and can
reduce the time spent in their daily commute. Similarly, with less overall
congestion, buses will also be able to reach their destinations more quickly.
Third, congestion pricing is essentially an equitable solution that will
benefit the majority. This policy will only punish those who choose to
pollute the air. By increasing the cost to pollute, more people will choose
not to; therefore, everyone will benefit with cleaner air.

metropolis:
city

overcrowded:
a larger quantity of people
than is wanted
remedy:
to solve

congestion:
the state of being crowded
with people or vehicles
motorist:
the driver of a motor vehicle

thereby:
because of; by means of

emission:
a substance that is sent out
into the air
equitable:
fair for all people involved

----------
Main idea:

There are three advantages of using - - - - - - - - - - - - - - - to control pollution and congestion.

Key point 1: It will make more people use - - - - - - - - - - - - - - - - because


they will not want to drive.
Key point 2: It will make _ _ _ _ _ _ _ _ _ _ _ _ _ _ _ _ faster for everyone by
taking cars off the road.

l<ey point 3: It will benefit _ _ _ _ _ _ _ _ _ _ _ _ _ _ _ _ _ because it only


punishes people w h o - - - - - - - - - - - - - - ' - - - - - -

''"'. - - 448
~ ~-~

Chapter 1

'.,

1~

-'
'I'.

~Step

Now listen to a lecture related to the topic in Step 1. Fill in the blanks of the note diagram
below with keywords or key phrases used in the lecture, l@fi!t

Main idea:

Congestion pricing is not the _ _ _ _ _ _ __


_ _ _ _ _ _ _ _ _ _ _ for reducing
congestion and pollution.

Key point 1:

advantage:
a benefit of something

extensive:
large In size or scope

It will not result in faster _ _ _ _ _ _ _ _~-

causes pollution

--------~because buses will be too

proponent:
a person that supports a
set of ideas

crowded.
Key point 3:

policy:
a plan or set of rules made
by an organization or
company

It may not m a k e - - - - - - - - - - - public transportation because many public transportation


systems are not good enough.

Key point 2:

It will not benefit


many people do not have any other choice.

because

I[

polluter:
a person or thing that

implement:
to begin to use the ideas in
e plan
penalize:
to punish someone

affordable:
inexpensive_

-----~Step

Review your notes from both the reading and the lecture. Pay attention to the main ideas and
supporting details. Using the information from your notes, complete the following outline;

__ .,.,-!\

Introduction:
Main topic:
Main idea of the reading p a s s a g e : - - - - - - - - - - - - - - - - - - - - - Main idea of the lecture: - - - - - - - - - - - - - - - - - - - - - - - - -

I
I
I
I;

Organizing Ir.formation

----

449 _ ,

Body:
Key point 1:
,_:.

Supporting information from the reading passage: - - - - - - - - - - - - - - - -

Supporting information from the lecture: - - - - - - - - - - - - - - - - - -

Key point 2: - - - - - - - - - - - - - - - - - - - - - - - - - - - Supporting information from the reading passage:

Supporting information from the lecture: - - - - - - - - - - - - - - - - - -

'.

r:

[,
I'

!:

Key point 3: - - - - - - - - - - - - - - - - - - - - - - - - - - - Supporting information from the reading passage:

Supporting information from the lecture:

Conclusion:
Summary: - - - - - - - - - - - - - - - - - - - - - - - - - - - - - ;

:'

.I

~Step

Use the main ideas and details from Steps 1, 2, and 3to complete the passage. Include information
from both the reading and the lecture.
The lecture and the passage both discuss congestion pricing, a policy that _ _ _ __
people for driving during busy times of the day. While the passage says that congestion pricing is
_ _ _ _ _ , the lecturer argues that it is not a good idea.
According to the passage, the policy
people to use public
transportation instead of cars, which will lead to less
. The lecturer
points out that in some cities, buses do not go to all neighborhoods. As a result, many people will not
be able to get around. Next, the passage claims that it makes travel
because it
___________ traffic. However, the lecturer
that commute times will
increase. Lastly, the passage states that congestion pricing is _ _ _ _ _ and beneficial to
everybody. The lecturer shows that the policy puts additional
stress on
drivers who do not have a lot of extra money.
There are many issues surrounding congestion pricing. The lecture and passage both
______ some of the issues.

::,'/-- 450 Chapter 1

" .
i,i

"'
)

lil
1,

History
",w.'Ja"1.>M<ilYMllli<!!>.~wn;,:nw1.~;,_,,,.,~--"';";.>:,,,._,_,,_,.:.

~Step

G\

-:..- .. -

1
------~----------~-~------

I
I
I

In 1952, a team of archaeologists found a copper scroll that had


been hidden in a cave for nearly 2,000 years. It is now called the Copper
. Scroll. Once the researchers were able to translate the text inscribed on
it, they understood that it was a list of buried treasures. However, there are
many reasons to believe that it does not describe a real treasure.
The scholar assigned to translate the copper scroll concluded that
the list is a work of fiction meant to entertain people. This would not be
surprising as there were many traditional stories in ancient Israel, the
land where the scroll originated. These stories spoke of hidden treasure
from Solomon's Temple.
Another reason the information on the scroll is believed to be false is
that it mentions a treasure of up to 174 tons of gold and silver, an
amount that seems too large for biblical times. In today's terms, that
amount of precious metals would be worth around one billion dollars. It
is very unlikely that anyone in ancient Israel could have amassed that
Jli
amount of wealth.
Finally, even if the scroll did point to real treasure, it has yet to be
found. This is because the sixty-four locations the scroll describes are
referred to vaguely or have ancient place names whose meanings and
locations are no longer known.

Main idea:

[
[
;;j

Read the following passage. Then look at the note diagram and fill in the missing information.

l '

archaeologist:
a scientist who studies past
civilizat!ons and cultures

translate:
to change words from one
language to another
inscribe:
to write or cut words into
something
assign:
to give someone a task or
job

Imaginary:
a thought from the
imagination; not from reality

---1

precious:
valuable; worth a lot of
money
amass:
to gather or collect
something

vaguely:
not clearly; not precisely

The Copper Scroll did not describe a - - - - - - - - - - - - - - -

Key point 1: The Copper Scroll was likely a _______ because there were many similar
stories in Israel.

Key point 2: The amount of treasure described in the scroll seems ___________
to be real since it would be worth one billion dollars today.
Key point 3: The _______ has never been found because the descriptions in the scroll
are vague.

Organizing Information

451 ----->

~Step

n Now listen to a lecture related to the topic in Step 1. Fill in the blanks of the note diagram
below with keywords or key phrases used in the lecture. ti!!lfili
"

I!

Main idea:

There is evidence to suggest that the _ _ _ __


in the Copper Scroll is _ _ _ __

scholar:
a person who has studied a
particular field in great depth

Key point 1:

Since the Copper Scroll uses _ _ _ _ _ _ __


________ , it is probably real.

technical: difficult to understand


without special training or
knowledge

Key point 2:

The amount of treasure described in the scroll might


not be
as some people think because
money was worth different amounts in different places.

generation:
a group of people born at
about the same time

It may be possible t o - - - - - - - - - because another scroll might exist.

region:
a particular area in the
world

Key point 3:

,:

ancient:
veiy old

modest:

somewhat small or
inexpensive
upheaval:
a time of significant change
detailed:
including many facts or
parts; complete

"------------,-------~Step

Review your notes from both the reading and the lecture. Pay attention to the main ideas and
supporting details. Using the information from your notes, complete the following outline.
Introduction:
Main topic:
Main idea of the reading passage: - - - - - - - - - - - - - - - - - - - - - l

Main idea of the lecture: ------------------------~

i:!;;,_._

452 Chapter 1

~l-~;; :-;:-~-,cc-;-.:--~J_;,i;;;.:2'2:;:;:-c:;;;.--"ro;;;-;eJc;;cc:o-t.;--"ccc;~.;:-~;:C:-

-----

--~-

'

Body: ,

....

Key point 1: - - - - - - - - - - - - - - - - - - - - - - - - - Supporting information from the reading passage: -----~---------

"

Supporting information from the lecture: - - - - - - - - - - - - - - - - - -

Key point 2: - - - - - - - - - - - - - - - - - - - - - - - - - - - Supporting information from the reading passage:


I

Supporting information from the lecture: - - - - - - - - - - - - - - - - - -

Key point 3: - - - - - - - - - - - - - - - - - - - - - - - - - - - Supporting information from the reading passage: - - - - - - - - - - - - - - . I

Supporting information from the lecture: - - - - - - - - - - - - - - - - - -

Conclusion:
_ _ _ _ _ _ _ _ _ _ _ _ _ _ _ _ _ _ _ _ _ _ _ _ _ _ _ __
Summa~:

~Step

Use the main ideas and details from Steps 1, 2, and 3 to complete the passage. Include information
from both the reading and the lecture.

Both the lecture and the passage discuss the Copper Scroll. While the passage says that the
______ described in the scroll does not actually
, the lecturer argues the
treasure is real.
According to the passage, the scroll is an old sto~ that was passed down over generations. The
lecturer
this claim by explaining that the scroll used - - - - - - - - - that is not used in folk tales. The passage also argues that the treasure is much too large to be real.
In response, the lecturer
that the amount in the scroll may be _ _ _ _ __
than it seems; therefore, it is probably real. Last, the passage claims that the treasure is
- - - - - - - - - - t o find because the directions on the scroll are not _ _ __
The lecturer says that a different map mentioned in the copper scroll probably has better
_ _ _ _ _ _ _ _ _ _ to the treasure.
The myste~ of the copper scroll has inspired many arguments. Some people still think it is
_ _ _ _ _ _ _ _ _ _ , while others think it is only a sto~.

Organiziiig Information 453 - :

--~

---'"""

-~--

..

---'

._,~---

_,

Environmental Science
~Step

Read the following passage. Then look at the note diagram and fill in the missing information.
derive:
Biofuels are a type of energy source that is derived wholly from plant
to make something out of
or animal sources. Supporters of biofuels claim that these alternative
something else
energy sources will help us meet our energy needs while overcoming the
associated:
connected to
disadvantages associated with fossil fuels. However, biofuels cannot
power:
replace fossil fuels.
to give something energy
First of all, biofuels are not more environmentally friendly than fossil
agricultural:
related to farming
fuels. Many biofuels supporters would suggest that using biofuels is a
crop:
cleaner way to power automobiles than gasoline. However, these biofuels
a plant grown for food
are actually worse for the environment than gasoline. Some pollute the
consume:
to
use something that goes
environment even more than fossil fuels.
away after you use it
Second, biofuels do not have an unlimited supply. Fossil fuels were
vast:
very large in slze or scope
created over millions of years, and today they are running out. Supporters
supplement:
of biofuels suggest that plant-based biofuels can be grown anywhere
to add to something else
and therefore can provide us with an unlimited supply. The problem is,
most of the world's agricultural land is already used. Jo grow biofuel
crops means we would have to grow less food or cut down forests, both
of which would have negative consequences.
Finally, biofuels. cannot meet the energy demands of large nations
like the United States or China. These large countries consume vast
amounts of fossil fuels to power their economies. It is not possible to
grow enough plant-based biofuel crops to meet such large-scale demand.
The best that biofuels can do is supplement that demand .
.,___...... ----------------------------------.... ---------..--------.. ---~-. -----------'"'. _J

Main idea:

_______ cannot replace fossil fuels.

Key point 1: Biofuels are not more __________________ than fossil


fuels because they also pollute the environment.

Key point 2: Biofuels do not have ___________________ because


most agricultural land is already being used.

Key point 3: Biofuels cannot meet the world's


because we need too much fuel.

t.~~--

454

1'::

'1:

--~-..

Chapter 1

"

1.

Step 2

c:

Now listen to a lecture related to the topic in Step 1. Fill in the blanks of the note diagram
below with .keywords or key phrases used in the lecture. @f!fj

Main idea:

Ethanol is not a substance that we can use to

substance:
a particular type of liquid,
solid, or gas

Key point 1:

Ethanol is not _ _ _ _ _ _ _ _ _ _ _ __

replace:
to use one thing instead of
another

gasoline; in fact, it produces more pollution.


Keypoint2:

Ethanol requires more

than we have

available because there is not enough for both food


and fuel crops.
Key point 3:

Ethanol cannot meet the _ _ _ _ _for fuel in the


world because we do not have enough land.

efficient:
working well without
wasting time or resources

massive:
very large
demand:
the desire for a particular
thing
stark:
very clear and often
unpleasant
production:
the act of making
something
hardly:
used to say that something
almost does not happen at
all

----------

Step 3
Review your notes from both the reading and the lecture. Pay attention to the main ideas and
supporting details. Using the information from your notes, complete the following outline.
Introduction:
Main topic:
Main idea of the reading p a s s a g e : - - - - - - - - - - - - - - - - - - - - - Main idea of the lecture: - - - - - - - - - - - - - - - - - - - - - - - -

zGl

Body:
Key point 1: - - - - - - - - - - - - - - - - - - - - - - - - - Supporting information from the reading passage: - - - - - - - - - - - - - - - -

Supporting information from the lecture: - - - - - - - - - - - - - - - - - -

Key point 2: - - - - - - - - - - - - - - - - - - - - - - - - - - - Supporting information from the reading passage:

Supporting information from the lecture: - - - - - - - - - - - - - - - - - -

Key point 3: - - - - - - - - - - - - - - - - - - - - - - - - - - - Supporting information from the reading passage: - - - - - - - - - - - - - - - -

i
'

Supporting information from the lecture: - - - - - - - - - - - - - - - - - - -

Conclusion:

Summa~: - - - - - - - - - - - - - - - - - - - - - - - - - - - - - -

~Step

Use the main ideas and details from Steps 1, 2, and 3 to complete the passage. Include information
from both the reading and the lecture.
Both the lecture and the passage discuss biofuels as a possible - - - - - - - - - - to fossil fuels. The passage states that they cannot replace current fuel sources. The lecturer supports the
_ _ _ _ _ with a discussion about ethanol, a biofuel made from corn.
To begin, the passage says that biofuels are no better for the _ _ _ _ _ _ _ _ __
than fossil fuels. The lecturer
the claim by showing that ethanol creates more pollution
than
. Then the passage states that biofuels place
on
land resources. The lecturer agrees, adding that the
of ethanol makes
corn more expensive. Lastly, the passage says that biofuels cannot meet the world
for
fuel. Again, the lecturer agrees. He says that there is no way that enough ethanol could be produced to
_ _ _ _ _ the needs of the world.
Both sources discuss biofuels. They give three reasons why biofuels cannot _ _ _ _ _ fossil
fuels .
.,

'\',1,-- 456 Chapter 1


li I

ii.l:':.

- - - - - - ------------- -

----------------~

--~---"'~

---~---

- - - --

--

~-

---

Environmental Science
~Step

Read the following passage. Then look at the note diagram and fill in the missing information.

-----

,,;

Polylactic acid (PLA) is a natural substance derived from corn that


can be used to create a biodegradable plastic. Today, containers made
of PLA, called corntainers, are beginning to replace disposable petroleumbased plastic containers. These corntainers are much better for the
environment for three distinct reasons.
The principal advantage of PLA containers over petroleum-based
plastic containers is that the former can be composted, while the latter
cannot. Thus, after being used, a corntainer can be returned to the Earth
as fertilizer. Corntainers biodegrade into two harmless byproducts:
carbon dioxide and water. Plastic, on the other hand, releases toxic
chemicals into the environment.
Because PLA containers are biodegradable, they will take up less
space in landfills. Today, disposable plastics account for one quarter of
the space in landfills. It can take over 1,000 years for plastics to break
down.
Corntainers, on the other hand, break down in around 90 days.
,ll>i
Using corntainers and composting them will save valuable landfill space.
Finally, switching from petroleum-based plastics to PLA plastics is
the socially responsible thing to do. It is better for the environment and
therefore benefits everyone. It is our responsibility as citizens to protect
the environment and minimize the harm that we cause to it. Corntainers
help us do just that.

---~

biodegradable:

able to be broken down


Into natural substances

IL
[

disposable:

able to be thrown away


petroleum:
oil found underground
distinct:
clear
principal:
most fmportant; main
byproduct:

something produced in the


making of something else
landfill:
a place.where trash is
stored
minimize:
to make something as small
as possible

--~4-------~-------------

Main idea:

Corntainers are better for - - - - - - - - - - - for three main reasons.

Key point 1: Corntainers can be-----~- which returns them to the Earth as fertilizer.
Key point 2: Corntainers take up ___________ in landfills because they will
biodegrade.

Key point 3: Using corntainers is t h e - - - - - - - - - - - - - - - - - - - - thing to do because it is better for the environment.

Organizing lnfornldtion

457 --1

l!!!=~m!!!!!!!l!!l!!!!!!!~!!!!l!!!!!!'!!!!l!!!!!!J!!!!!!!!!!!!!ll!li!~~------"------jllliiiiii-----..._.~----..-----i----;
:;-_=----

-----=""-c~cc'--=. ---~'-''~ 0=~:=:::~--

-------

=-

.=-=:,o.::-:c-::-=::.~---~

~3'=-~~==-

~Step

() Now listen to a lecture related to the topic in Step 1. Fill in the blanks of the note diagram
below with keywords or key phrases used in the lecture. &Mf!fii
Main idea:

Corntainers do not solve _ _ _ _ _ _ _ _ __

Key point 1:

Corntainers r e q u i r e - - - - - - - - - - - _ _ _ _ _ _ _ _ _ _ in order to break down.

Key point 2:

Many corntainers end up in _________


because people do not understand how to recycle
them.

Key point 3:

There is n o t h i n g - - - - - - - - - - - about using corntainers because people need corn to


eat.

derivative:
something that comes from
something else
facility:
a building or room designed
.for a specific purpose

subject:
to cause something to
expe_rlence an event
microbe:
a tiny living thing that can
only be seen with a
microscope
sustained:
continuous; not changing
proper:
correct; appropriate
rapidly:
quickly
source:
the place where something
comes from

-~----~-------------

~Step

Review your notes from both the reading and the lecture. Pay attention to the main ideas and
supporting details. Using the information from your notes, complete the following outline.
Introduction:
Main topic:
Main idea of the reading p a s s a g e : - - - - - - - - - - - - - - - - - - - - - -

Main idea of the lecture: - - - - - - - - - - - - - - - - - - - - - - - - -

~;~:,____

458 Chapter 1

i,i ' !'-J =~-:::---==:-:::-:-------y-~~-.--.-_--, -';---~--- --... -~--,-,~--.-t-=~~,,.--,~~---:

\ i

l!_

Body:
Key point 1: - - - - - - - - - - - - - - - - - - - - - - - - - Supporting information from the reading passage: - ' - - - - - - - - - - - - - - - - - - -

'

~~
Supporting information from the lecture: - - - - - - - - - - - - - - - - - -

l.

Key point 2: - - - - - - - - - - - - - - - - - - - - - - - - - Supporting information from the reading passage:

Supporting information from the lecture:

Key point 3: --------------------~------


Supporting information from the reading passage: - - - - - - - - - - - - - Supporting information from the lecture: - - - - - - - - - - - - - - - - - ' - - - - -

: Conclusion:

'"summ'~ry: - - - - - - - - - - - - - - - - - - - - - - - - - - - - - ~Step

Use the main ideas and details from Steps 1, 2,and 3 to complete the passage. Include information
from both the reading and the lecture.
Both the lecture and the passage discuss PLA containers (corntainers). Though the passage
______ corntainers as an eco ..friendly
to other plastic
packages, the lecturer says they do not benefit the environment.
According to the passage, corntainers are good because they ___________
easily and turn into fertilizer. The lecturer
the claim. She shows that the
----process for corntainers is long and complicated. The passage also
states that corntainers are beneficial because they
tl1e amount of waste that ends up
in landfills. However, the lecturer disagrees, saying that rnany corntainers end up in ______,
anyway. Finally, the passage says that using corntainers is
responsible. They help the
environment, and thus,
everyone. In contrast, the !ecturer argues that it is actually
___________ to produce plastic with corn that could be used to feed the hungry.

,,

Many people are interested in doing what is best for the environment. The passage claims that
corntainers are eco-frienclly, but the lecturer disagrees.

. .
I''
.,,

Or"ganizi119 lnforn1a1ion

459 - - . ,

Necessary Skills
~~~~;lNX!.S."1Lce~1.;;,~=t~eft.'l'-",'('';,,J,'"T_:,,-,-

Understanding the original text accurately


Using your own words to convey essential information and ideas from the reading
and the lecture
Being al:lle to express the same information using different vocabulary and sentence
structure

The Process of Paraphrasing


~f!~J!~~Nmm:w-,;f:''l'>r~'''~;,";-1;

',''''

'

Understand the full meaning of the original text.


Take notes on the passage. Write down key information including a few phrases,
major points, and important details.
WITHOUT looking at the original passage, paraphrase the information in your own
words, just by looking at your notes.
Check the original passage for any missed key information.

Strategies
Use related words and phrases, including synonyms and antonyms of words and
concepts in the original passage.
Example: The average daytime temperature in the Gobi desert does not often' go
below 38C. -+The average daytime temperature in the Gobi desert is usually at or
above 38C.
" Change word forms, and rephrase to make things simpler.
Example: for organization -+ in order to organize
people at the age of thirty -+ thirty-year-old people
" Use different sentence structure.
Example: Many Asian countries export rice to North America. -+ Rice is exported
to North America by many Asian countries.
Change the order of presentation of the information.
Cite information from the original source by using signal words.
Example: According to the professor/passage ...
The professor says/mentions/states/argues/believes/found that ...

Paraphrasing

461 - -

Climatology
~'l:Il'[tJW.'ll'ifl",~~>;,'>Jf"~i<'!.~_{~,;:>;f~;,:;>~J:.ii:'',;~,~,;".''."C:l-.>~>-<"

~Step

'r,',~::'.'.'~:-!-'!\~-,'.--\'''-'."'1~:.

-;,

Read the following passage. Predict how the listening passage may contrast with the reading.
Then look at the note diagram and fill in the missing information.
11

I:

l
r:

;:I

Global warming is the theory that temperatures on the Earth's


surface are increasing. Scientists who study the weather over time,
called climatologists, have three reasons to believe that global warming
is occurring.
First, scientists recently learned that there is more carbon dioxide in
our atmosphere than ever before. By studying very old pieces of ice,
they determined that the levels remained the same for nearly 10,000
years. However, the levels have since increased by thirty-one percent,
and experts predict that they will continue to rise. The high amounts of
carbon dioxide is significant because the gas keeps the Earth's heat
from escaping. Therefore, the more carbon dioxide in the atmosphere,
the hotter the temperatures.
Second, the Earth's ice caps, the massive sheets of ice found at the
North and South Poles, are quickly melting as the Earth gets hotter.
According to scientists, over the past century, the ice caps have started
to melt at unbelievably rapid rates. For example, between 1997 and
2002, about nineteen cubic miles of ice melted each year in Greenland's
icecap. More recently, however, the figure has jumped to fifty-seven
cubic miles per year.
Finally, records show that global temperatures have steadily increased
over the past hundred years. The temperatures are measured and
recorded by two separate facilities, the Hadley Center and NASA Both
use data collected with state-of-the-art equipment to calculate the
average global temperature of the planet. Time after time, the results
stay the same: temperatures on Earth are, in fact, increasing.

IVlain idea:

atmosphere:
the mixture of gases around
the Earth
determine:
to find out the truth about
something

significant:
important or noticeable
rapid:
fast or sudden
figure:
an amount expressed in

numbers
facility:
a building where a particular
activity takes place
calculate:
to solve by using math
average:
formulated by adding two
or more amounts together
then dividing the total by
the number of amounts

Climatologists believe t h a t - - - - - - - - - - - - - - - - - - - -

- - - - - - - - - - - - for three reasons.


Key point 1: There is more-------~-~-------------
than ever before, which raises the Earth's temperature.
Key point 2: The Earth's ice caps are ____________ as the Earth gets hotter
Key point 3: The - - - - - - - - - - - - - - - - - h a s been steadily increasing.

~--~'.1---

462 Chapter 2
- : ~
: ~~ -_c:-. '.:, -;. ~.:~-

i
,._<

.,.;

~Step

','
IL

,:

Below is important information from the reading passage above. After each sentence are two
possible paraphrases of it. Choose the best paraphrase for each sentence.

,"

If_'
[

'
'

;==---<::

A. Scientists learned that carbon dioxide levels stayed the same for almost 10,000 years by examining

t..

changes in very old pieces of ice.


1. Ten-thousand-year-old ice can help scientists learn more about temperatures long ago.
2. Scientists know from studying ice that carbon dioxide levels did not change much for 10,000
years.

B. The Hadley Center and NASA use the best technology available to figure out the average global
temperature.
1. Two facilities calculate the average global temperatures using high-tech tools.
2. Advanced technology is being developed by two facilities to find the average global temperature.

C. In the space below, write a paraphrase of the main idea of the reading passage.

~.~ Step 3 '.'.

in Now listen to a lecture related to the topic in Step 1. Fill in the blanks of the note diagram

.f

below with keywords or key phrases used in the lecture. ltl!tlifi

:1.

Main idea:

KeY point 1:

Key point 2:

Global warming is not ___________


on Earth.

telltale:
clearly showing something
secret or hidden

Carbon dioxide levels' have not increased

compelling:
attracting strong interest

- - - - - - - - - - - i n the past quarter


century.

dramatically:
suddenly and noticeab~

There are many different factors that


- - - - - - - - - - t o the melting of ice
caps.

Key point 3:

Reported average global temperatures are not


___________ because the facilities
do not inciude all temperatures in the average.

factors:
something that influences
the result of something else
recede:
to become less; to withdraw
inaccurate:
not completely correct or
exact

misleading:
confusing or deceptive
publish:
to make Information
available lo the pubic

Paraphrasing

463 ---.:'"

~Step
:

Choose the sentences that best combine the main idea and key points from the reading passage
and the lecture.

1. a. The reading says that high levels of carbon dioxide demonstrate that global warming is occurring.
b. While the reading says that global warming is happening now, the lecture says it is not.

2. a. The reading and the lecture both discuss the effects of carbon dioxide on global temperatures.

"'I

.i~.I

b. The reading states that high carbon dioxide levels do not indicate mat temperatures are rising,
though the lecture disagrees with this statement.

ti
,,f
r!it
I~'

3. a. The passage suggests that ice caps are melting because of rising global temperatures. The lecture,
however, says that ice caps melt because of many other reasons.
b. The passage says that global warming is causing the world's ice caps to melt. The lecture discusses
the effects of volcanoes on global warming.

j'

4. a. The author of the passage and the speaker agree that average global temperatures have increased
in recent years.
b. Though the author of the passage says that temperature increases have been recorded, the
speaker says that the records are inaccurate.

~Step

A. Changing Keywords
Below are four incomplete paraphrases of key information from the reading passage and the
lecture. Fill in the missing parts with words or phrases from the box. These words and phrases
are synonyms or are similar in meaning to the actual words used in the reading passage and
lecture.
1. currently I slowly I unlikely I accurate I changing I supportive
2. solved I worsened I maintained I policy I strategy I view
3. confirm I understand I indicate I factors I chemicals I ways
4. amenity I facilities I building I unmonitored I inaccurate I biased

__J

1. While the reading says that global warming is _ _ _ _ _ taking place, the lecture says tl1at
temperatures are not - - - - ' - 2. The reading states that carbon dioxide levels have _ _ _ _ _ _ _ _ _ _ global warming,

though the lecturer disagrees with this _ _ _ __


3. The passage suggests that melting ice caps _ _ _ _ _ rising temperatures. The lecture,

however, says that ice cap loss is due to many other


4. The author of the passage says two
have recorded temperature increases over the
past hundred years, the lecturer points out that the records are _ _ _ _ _ _ _ _ __
ii---

464

Chapter 2

B. Changing Sentence Structure


Complete the following paraphrases of the sentences from part A above.

1. The lecture says that temperatures are not _ _ _ _ _ , while the reading says global warming
is _ _ _ _ _ taking place.

2. The lecturer disagrees with the _ _ _ _ _ presented in the reading, which claims that carbon
dioxide levels have

,,
~z

Ill

global warming.

C\

3. It is suggested in the passage that melting ice caps _ _ _ _ _ global warming; however, the
lecture says other _ _ _ _ _ might be responsible.
4. The lecturer shows that average global temperature records are _ _ _ _ _ _ _ _ __
though the passage says that two _ _ _ _ _ _ _ _ _ _ provide proof that average

temperatures have increased.

~Step

Read the following sentences taken from the reading and the lecture. Create new sentences by
combining the ideas in each pair of sentences.

1. ; a. However, the levels have since increased by thirty-one percent, and experts predict that they will
continue to rise.
b. The fact is that over the past quarter century, carbon dioxide levels have increased by only two
percent.

2.

a. The temperatures are measured and recorded by two separate facilities, the Hadley Center and
NASA.
b. It turns out that even with the most advanced technology, their figures are still inaccurate.

Paraphrasing

465 - -

Psychology

l
f

t'

;;:m;x.:;i.1;;,_;-.:._'"""'"'""?*"'-\0'":'"~'';

i,j

r~{'.1
_iili

~~

:-',

'

~Step

"

,,,-,.,

', _,,

Read the following passage. Predict how the listening passage may contrast with the reading.
Then look at the note diagram and fill in the missing information.

iJ

:dil

'<i\::
.i'
f:j

i::;

Many adults want to protect children from all negative influences.


However, this natural desire to protect children often interferes with the
freedom of play. Adult interference in children's playing is problematic for
three reasons.
First, adult interference in children's playing can hurt children's creativity.
Children who have the freedom of unstructured playtime often create
rules for themselves and their playmates. They imagine complex worlds
and share colorful fantasies. This helps them develop their creativity and
imagination. When an adult interferes with a child's play, it can have the
detrimental effect of stunting their creativity.
Second, children need to develop positive peer role models and
relationships. Part of childhood is learning how to relate to one's peers.
This complex process involves imitating peers, learning socially acceptable
communication, and negotiating to get one's needs and wants met.
Developing positive peer relationships, particularly among young children,
is something learned during playtime. When adults interfere in children's
playing, they can delay or hamper the development of these important
social skills.
Finally, children need to learn how to stand up for themselves. Just
like adults, children must learn how to deal with difficult peers. One of
the main reasons adults, especially parents, interfere in children's playing
is to protect children from bullies or other difficult children. The problem
is that children often suffer more when an adult inteNenes on their
behalf. Children will never learn to resolve these situations themselves
when an adult interferes.

ili _,1

'!
I~

r ll

~
:~I

id

I':1'

I:
1:

r'r

I
!

desire:
a want of something
interference:
an unwanted action meant
to change a situation
detrimental:

harmful; damaging
stunt:
to lessen or stop the
growth of something

peer:
someone of the same age
acceptable:

considered good in a
particular situation
hamper:
to stop something from
happening normally

intervene:
to become involved in a
situation in order to change
it

'----------------------------
Main idea: Adult interference in - - - - - - - - - - - - - - - - - - is problematic.
Key point 1: Adult interference can hurt-----------------~
Key point 2: Children need to develop p o s i t i v e - - - - - - - - - - - - - - - - which is done through playtime.
Key point 3: Children need to learn t o - - - - - - - - - - - - - - - - - - - -

\"~'7'\!----"--

1'.,'

.!:. ~j
;f,,

466

Chapter?

..;

r-...
[
,

Cl

~Step

Below is important information from the reading passage above. After each sentence are two
possible paraphrases of it. Choose the best paraphrase for each sentence.

A. Unstructured playtime allows children to make their own rules and share inventive fantasies, which
helps them develop their creativity and imagination.

[[

f'

1. Children who have unstructured playtime have a chance to develop their fantasies.
2. Unstructured playtime gives children freedom that helps them develop many skills.

B. Adult interference can keep a child from forming skills thafi\Nill


help him or her relate to others.
,,.
1. Children might not learn social skills because of adulfinterference.
2. Forming skills as a child depends on how much he or she relates to others.

C. In the space below, write a paraphrase of the main idea of the reading passage.

I
I
) I
I I
I

~Step

n l\lpw listen to a lecture related to the topic in Step 1. Fill in the blanks of the note diagram
b,fi!low with keywords or key phrases used in the lecture. i\j\flfU
)';
~--;-------

M~in idea:

Adults need to _ _ _ _ _ _ _ children's


playtime.

Key point 1:

Adults need to monitor playtime to make sure it is


___________ because many
children will get into trouble otherwise.

Key point 2:

Children need adultl not peers, as


------+-----because
children
.\
learn bad behavior from other children.

Key point 3:

Adults need to monitor children to _ _ _ _ _ __


them.

consequence:
a result of something
monitor.
to watch something to make
sure it. happens correctly
unstructured:
nqt organized

stimulate:
to cause-interest in
something
. prlmarily:
mainly
engage:
to participate in an activity
cruel:

very mean
deal with:
to take action to solve a
problem

~-----------------~--------~----------

Paraphrasing

467 ------.....

"

~.

11I. I,i
I:

1li'

i
:

1J{

i:a

l~

b
ii:

i1Ji: l
f.!1

il:lii

~Step

Choose the sentences that best combine the main idea and key points from the reading passage
and the lecture.

L_,

i<'.
,,.
,,

1. a. Though the reading discusses adult interference in children's playtime, the lecture disagrees by
stating that it is good.

Ii

,i
I

l!l

fI:.'1
J
.kl

l,

1.:,1

;i
;;

..

L;_,
!i
ii

b. While the reading states that adult interference in children's playtime is problematic, the lecture
says that adults need to interfere in playtime.
2. a. While the author says that adult interference can hurt creativity, the speaker thinks that adults

should monitor playtime to make sure it is positive.


b. Adult interference can hurt children's creativity, so adults need to monitor playtime to make sure
it is positive.
3. a. According to the author, children need to develop positive peer role models. However, the speaker

says that children need adults, not peers, as role models.


b. The author states that children need positive role models, while the speaker does not believe that
this is true.
4. a. The author of the passage discusses whether children should learn to stand up for themselves.

The lecture, though, discusses whether adults should monitor playtime.


b. The author states that children need to learn to stand up for themselves, while the lecture states
that adults need to monitor children to protect them.

~Step

A. Changing Keywords
Below are four incomplete paraphrases of key information from the reading passage and the
lecture. Fill in the missing parts with words or phrases from the box. These words and phrases
are synonyms or are similar in meaning to the actual words used in the reading passage and
lecture.
1. states I describes I implies I challenging I difficult I troublesome
2. change I harm I stop I look I see I watch
3. good I helpful I useful I require I want I ask
4. shelter I defend I raise I mothers I people I parents
1. While the reading

that adult interference in children's playtime is _ _ _ __


the lecture says that adults need to interfere in playtime.

2. While the author says that adult interference can _ _ _ _ _ creativity, the speaker thinks that
adults should _ _ _ _ _ playtime to make sure it is positive.

. --,.,,_,_,

._.,.. i-'.' ___ ,,__ - -

':

3. According to the author, children need to develop


peer role models. However, the
speaker says that children
adults, not peers, as role models.

4. The author states that children need to learn to


themselves, while the lectwe states
that
need to monitor children to protect them.
B. Changing Sentence Structure
Complete the following paraphrases of the sentences from part A above.

1. - - - - - - - - - - - that adult interference in children's playtime is troublesome,


while _ _ _ _ _ _ _ _ _ _ _ that adults need to interfere in playtime.

2. While the author says that creativity can be


the speaker thinks that playtime should be
positive.

to make sure it is
}

.,,

~,..,
m

3. Children need to develop


though, says that children require

""

according to the author. The speaker,


as role models.

....

4. The author states that children need to learn t o - - - - - - - - - - - _ _ __


____________ that parents need to monitor children to protect them.

~Step

kead the following sentences taken from the reading and the lecture. Create new sentences by
combining the ideas in each pair of sentences.

1.

a. When adults interfere with children's playing, it can have the detrimental effect of stunting their
creativity.
b. They need adults to guide them toward positive activities that help them develop their creativity'
during playtime.

2.

a. Children will never learn to resolve these situations themselves when an adult interferes.
b. Sometimes children need their parents to help them deal with their peers.

Paraphrasing 469

&

--1

-]~

Biology
~ ~;:'i'i,'.~;.o..<T.X'~-l~>1jeJtWl\l,}<:11>U1o.%kHGU.<.'~-"'';~__.,,,, w ... ;:.-,-,,,, --'

~Step

Read the following passage. Predict how the listening passage may contrast with the reading.
Then look at the note diagram and fill in the missing information.

::

The dodo was a considerably large and plump bird, standing three
feet tall with a grey body, a hooked beak, and a plume of white feathers
on its rump. If once inhabited an island in the Indian Ocean, and in the
late 17th century, it suddenly went extinct. There is little doubt that the
disappearance of the dodo is a direct result of over hunting by humans.
One reason the dodo fell victim to over hunting was that, unlike most
birds, it was flightless. Despite its large body, the dodo had small, weak
wings, which could not propel it into the air. Therefore, it constituted easy
prey for the human settlers, who would swiftly kill the bird as it walked
through their camps. It was not long before the dodo's population
severely dwindled.
The dodo's unusual friendliness also contributed to its extinction.
lsola\ed from human contact for the entirety of its evolutionary existence, the
dodo greeted the new visitors with a complete lack of fEiar. Rather than run
or hide, the birds approached hunters with child-like curiosity and innocence,
a habit that meant sudden death for much of the dodo population.
In addition, researchers have found evidence in caves of humans killing
dodos in mass quantities. In one particular cave in Europe, researchers
found the bones of many dodos. There are many such caves in Europe
where dodos were taken from their island and killed.

plump:

large and round; slightly fat


plume:

a group of feathers
extinct:

no longer existing
propel:

to push something in a
particular direction
swiftly:
quickly
dwindle:

to slowly become smaller

existence:
lhe state of being real
curiosity:

the quality of wanting to


learn more about things

~---.-----~-~----------~-

Main idea: T h e - - - - - - - - - - - - - - - - - - - is a result of over hunting.


Key point 1: It became extinct b e c a u s e - - - - - - - - - - - - - - - - - - - Key point 2: I t s - - - - - - - - - - - - - - - - - - - - also led to its extinction.
Key point 3: Researchers f o u n d - - - - - - - - - - - - - - - - - - - - in caves.

-'------~---

- -------

----

------------------

- --

--------- ...

~--

~Step

Below is important information from the reading passage above. After each sentence are two
possible paraphrases of it. Choose the best paraphrase for each sentence.
A. Human hunters found the flightless dodo an easy target, which quickly led to a significant decrease
in the dodo population.
1. Humans hunted the dodo often because the bird would refuse to fly.
2: The dodo population declined in part because humans found it easy to hunt them.
B. Having never encountered humans before, dodo birds did not know to protect themselves and
would often curiously approach hunters.
1. When dodos came into contact with humans, they did not defend themselves.
2. Dodos wanted to know more about humans because they had never seen them before.
C. In the space below, write a paraphrase of the main idea of the reading passage.

i!. ~Step 3
. f; (;) Now listen to a lecture related to the topic in Step 1. Fill in the blanks of the note diagram

.t

below with keywords or key phrases used in the lecture. i@fij+

. Y.
-~

Main idea:

It was not over hunting but _________


_ _ _ _ _ _ that led to the dodo's disappearance.

Key point 1:

Humans did not even like _ _ _ _ _ _ _ __


_ _ _ _ _ _ _ _ because it tasted very bad.

Key point 2:

Humans were not the main hunters; ______


_________ were.

coincidence:
an event where two related
things happen by chance
praise:

to talk in a posttive way


about something

abandon:

Key point 3:

There is little evidence of _ _ _ _ _ _ _ _ __


_ _ _ _ _ _ _ _ _, but plenty of evidence of
predators killing dodos.

to stop doing something


pursue:

lo follow In order to capture


something
vulnerable:

easy to attack; weak


repopulate:
to increase the numbers of

a species in an area

scant:
very liltle
predator:

an animal that hunts


another animal for food

Paraphrasing 471

----1

~Step

Choose the sentences that best combine the main idea and key points from the reading passage
and the lecture.

1. a. The author thinks that the disappearance of the dodo was a result of over hunting. However, the
speaker says that it was not over hunting, but other factors that led to its disappearance.
b. The author thinks that the disappearance of the dodo was a result of over hunting. The speaker
agrees by stating that the remains of many dodos were found in caves.

2. a. The author says the dodo became extinct because it was flightless. The speaker says that
humans did not even like dodo meat.
b. The author states that the dodo became extinct because it was flightless, but the speaker thinks
that it was because humans fed their meat to their dogs.
3. a. Though the reading says that its unusual friendliness also led to the dodo's extinction, the lecture
argues that humans were not the main hunters; other animals were.

b. Though the reading says that humans were not the main hunters of the dodo; other animals were,
the lecture argues that its unusual friendliness also led to its extinction.
4. a. The reading and the .lecture both discuss whether there was evidence of humans killing dodos.
They disagree on this point.

b. The reading says that researchers found evidence of humans killing dodos in caves, but the lecture
says that there is little evidence of humans killing dodos.

~Step

A. Changing Keywords
. Below are four incomplete paraphrases of key information from the reading passage and the
lecture. Fill in the missing parts with words or phrases from the box. These words and phrases
are synonyms or are similar in meaning to the actual words used in the reading passage and
lecture.
1. surprise I cause I consequence I reasons I movements I changes
2. while I when I since I look I enjoy I burn
3. strange I bad I useless I key I large I important

~-4_.f_a_ct_l_s_u_b_st_an_c_e_l_p_ro__o_f_l_m_o_d_e_st_l_s_o_m_e_l_s_ca_n_t_ _ _ _ _ _ _ _ _ _ _ _

,_J

1. The author thinks that the disappearance of the dodo is a _ _ _ _ _ of over hunting.
However, the speaker says that it was not over hunting, but other
, that led to its
disappearance.

2. The author says it became extinct _ _ _ _ _ it was flightless. The speaker says that humans
did not even

dodo meat.

3. Though the reading states that the dodo's


lecture argues that humans were not its

friendliness also led to its extinction, the


hunters; other animals were.

4. The reading says that researchers found


of humans killing dodos in caves, but the
lecture says that there is
evidence of humans killing dodos.

[
I[

B. Changing Sentence Structure


Complete the following paraphrases of the sentences from part A above.
1. The author thinks that over hunting caused - - - - - - - - - - - - - - - - - - - - - However, the speaker says that it was other factors, not _ _ _ _ _ _ __
that led to its disappearance.
2. The author says the dodo became extinct because it was flightless, while the
humans did not _ _ _ _ _ _ _ _ _ _ _ __

>> .

I
I

says that

3. The reading says that the dodo's strange friendliness also led to its extinction, _ _ _ _ __

- - - - - - - - - - - - - - - - that humans were not the key hunters; other


animals were.

I
I

sp~aker

,
"

4. The reading says that


was
*;;found in caves by researchers, but the lecture says that there is scant evidence of humans killing
dodos.

;~

+ ~Step 6
Read the following sentences taken from the reading and the lecture. Create new sentences by
combining the ideas in each pair of sentences.
,

1.

a. Isolated from human contact for the entirety of its evolutionary existence; the dodo greeted the
new visitors with a complete lack of fear.
b. Rather, its fearlessness made it vulnerable to the animals humans brought with them.

2.

a. In one particular cave in Europe, researchers found the bones of many dodos.
b. Although a few dodos did make it back to Europe on Portuguese ships, archaeologists have
turned up scant evidence of human dodo hunts.

i
'

. !

I.

Paraphrasing 473 - -

.i~f~~~.~,. ,~,~-c..............~...........-r

~ St.ep 1
Read the following passage. Predict how the listening passage may contrast with the reading.
Then look at the note diagram and fill in the missing information.
pest:
DDT is a human-manufactured chemical once used to control the
a small animal or insect that
spread of unwanted insects. It was used during World War II to kill
annoys you
mosquitoes carrying dangerous human diseases, and afterwards to get
hazard:
something 1hat is
rid of agricultural pests. Over time, however, scientists discovered that
dangerous
DDT did much more harm than good.
accumulate:
to become larger over time
DDT, although designed to protect humans and plants from diseaseIngest:
carrying insects, is itself a hazard to the environment. It accumulates in
to eat or drink something
the soil and in small bodies of water and is then ingested by animals such
produce:
fruit and vegetables
as fish and birds. Because DDT is not easily digested, it gets stored in
resistance:
the body and builds up over time. Too much DDT can poison and even
the ability not to get sick
kill wildlife. It is, for example, a major factor in the decline of the bald eagle.
from something
DDT also endangers the health of humans who consume produce
combat:
to fight or oppose
sprayed with DDT. Studies have shown that exposure to DDT increases
something
the risk of diabetes, birth defects, and various forms of cancer. The effects , transmit:
to send something from
of DDT consumption were found to be so severe that the United States
one place to another
eventually banned the chemical.
Finally, DDT has failed to effectively combat disease-carrying
mosquitoes, and thus diseases like malaria continue to spread. Not long
after DDT was first introduced on the market, mosquitoes began
developing a resistance to the chemical. It was not long before
mosquitoes could once again easily transmit deadly diseases.
----
--------------

Main idea: Scientists discovered that DDT

than _____

l<ey point 1: DDT is a ________ to the ________ because it can poison


and kill wildlife.
Key point 2: DDT can endanger t h e - - - - - - - - - - - - - - - - - - - by causing a number of diseases.
Key point 3: DDT has failed to c o m b a t - - - - - - - - - - - - - - - - - - because insects became resistant to the DDT.

- -- 474

Chaptc1 Z

; ~ Ste!J 2
Below is important information from the reading passage above. After each sentence are two
possible paraphrases of it. Choose the best paraphrase for each sentence.

A. DDT is a chemical that was originally used during World War II in order to combat disease-carrying
mosquitoes and other pests.
1. A chemical, DDT, was first used as weapon of war during World War II.
2. World War II era scientists used DDT to get rid of mosquitoes and other bugs.

[
[

B. Animals often end up eating pockets of the chemical that has collected in the soil and various water
sources.
1. DDT collects in the soil and in the water, where it is often consumed by animals.
2. Soil and water are the most important resources to protect from DDT.

C. In the space below, write a paraphrase of the main idea of the reading passage.

~Step

3
~\

(T Now listen to a lecture related to the topic in Step 1. Fill in the blanks of the note di.~gram
}"

''below with keywords or key phrases used in the lecture. fil'il!!f&!


Main idea:

DDT is not as bad as some people believe, and it has

Key point 1:

DDT can, in some cases, __________

- - - - - - - - - - - because it kills insects


that cause disease.

Key point 2:

i
~,

notorious:
well-known for something
bad
insecticide:
a chemical used to ~II
insects

toxic:
poisonous

DDT can also b~nefit _ _ _ _ _ _ _ _ __


by stopping diseases that kill humans.

Impair:
to make something worse
or less effective

escalate:

Key point 3:

DDT h a s - - - - - - - - - - - - - ______ in the past, and millions of lives have


been saved.

to increase in intensity
inconclusive:
not certain
devastating:
very damaging
intensive:
involving a lot of energy or
a lot of parts

~------------------------------------------------

-----------------

Paraphrosing

475 - - .

~Step

Choose the sentences that best combine the main idea and key points from the reading passage
and the lecture.

1. a. The reading states that even though scientists discovered that DDT did more harm than good,
DDT is not as bad as some people believe, and it has many benefits.
b. Though the reading says that scientists discovered that DDT did more harm than good, the lecture
argues that DDT is not as bad as some people believe, and it has many benefits.
2. a. The reading says that DDT is a hazard to the environment. On the other hand, the lecture says

that DDT can, in some cases, benefit the environment.


b. While the reading says that DDT is a hazard to the environment, DDT can, in some cases, benefit
the environment.
3. a. Both the reading and the lecture state that DDT can endanger the health of humans, and the

lecture argues that it also kills insects.


b. While the reading states that DDT can endanger the health of humans, the lecture argues that
DDT can actually benefit human health.

,,-;!

.--:1:,,

>!

-~ -I .
' l '

l;

4. a. According to the reading, DDT has failed to combat disease-carrying mosquitoes. However, the
lecture states that DDT has effectively controlled diseases in the past.

b. According to the lecture, DDT has failed to combat disease-canying mosquitoes. However, the
reading states that DDT has effectively combated diseases in the past.

~Step

A. Changing Keywords
Below are four incomplete paraphrases of key information from the reading passage and the
lecture. Fill in the missing parts with words or phrases from the box. These words and phrases
are synonyms or are similar in meaning to the actual words used in the reading passage and
lecture.
1. found I studied I denied I advantages I benefits
2. damage I danger I harm I is I with I may
3. allows I responds I says I help I suppose I advance
4. fight I enable I breed I quickly I efficiently I successfully
1. Though the reading says that scientists _ _ _ _ _ that DDT did more harm than good,

the lecture argues that DDT is not as bad as s6me people believe, and it has many
2. The reading says that DDT is a

says that DDT

to the environment. On the other hand, the lecture


, in some cases, benefit the environment.

[(
3. While the reading _ _ _ _ _ that DDT can endanger the health of humans, the lecture argues
that DDT can

human health.

4. According to the reading, DDT has failed to _ _ _ _ _ disease-carrying mosquitoes. However,


the lecture states that DDT has
combated diseases in
the past.

If

B. Changing Sentence Structure


Complete the following paraphrases of the sentences from part A above.

1. According to the reading, scientists _ _ _ _ _ _ _ _ _ _ _ did more harm than good.


However, the lecture
not as bad as some people believe.

and is

1.

'

G'l

2. While _ _ _ _ _ _ _ _ _ _ _ that DDTisadangertotheenvironment, _ _ _ _ __


_ _ _ _ _ that DDT may, in some cases, benefit the environment.

3. While the reading says that _ _ _ _ _ _ _ _ _ _ _ can be endangered by DDT, the


lecture argues that DDT can really benefit human health.

4.

says that DDT has failed to fight disease-carrying mosquitoes.


ijjowever, according to the lecture,
combated diseases in'!the past.

st~p 6
Read the following sentences taken from the reading and the lecture. Create new sentences by
combining the ideas in each pair of sentences.

1.

a. DDT is, for example, a major factor in the decline of the bald eagle.
b. Insect-born diseases have been known to wipe out huge populations of species.

2.

a. The effects of DDT consumption were found to be so severe that the United States eventually
banned the chemical.
b. Such diseases have far more devastating effects on human health than DDT ever had.

Paraphrasing 477 -~.j

--~--

-,;:- __

Strategies
.'<"o'>,,<-r~";;'.:;

.-,,... ,,,.,

1-;, _,; _;

-~.-

'"'

c.~

After determining the framework of your essay, further consideration must be given
to the organization within the paragraphs themselves.
First paragraph contains:
the main idea of the whole response
one key point
examples and/or connection to the
reading

Supporting paragraphs contain:


additional key points
examples and/or connection to the
reading

When developing your points, make sure that the statements are well connected so
that the relationships between ideas can be seen clearly.
Use transitional words and phrases to indicate the relationships among ideas.
Use appropriate expressions to indicate when citing the source.

According to the lecture/passage ...


The reading stated that ...
In the reading, the author states that/discusses how ...
In the author's/professor;s opinion ...
According to the theory in the reading/lecture ...
111 The professor made the point that ...
The lecture supports/illustrates the idea that .. .
111 The lecture contradicts/refutes the idea that .. .

111

similarly, likewise, also, just as, both, by comparison, compared to, but, yet, although,
in contrast, on the contrary, contrary to, on the other hand, however, conversely, is
the opposite of, while, whereas, nevertheless, although, meanwhile, after all, although
this may be true, in spite of, despite

because, since, for, thus, therefore, hence, as a result, accordingly, for the same
reason

-,

::_____- __

Political Science

it

il
i1

fl

I;
1:

~Step

.I

Read the following passage. Then look at the note diagram and fill in the missing information .

f.

I'I
,.

I'

Fair elections depend on voting systems that best show which


candidate is favored among the public. For this reason, many people
have proposed that a computerized voting system which could be
completed over the Internet be implemented. There are several advantages
to using computerized voting systems rather than traditional voting
systems.
First, computerized voting systems are much safer than using paper
ballots. At every election, rnuch care has to be taken in order to properly
handle paper ballots. Unfortunately, the current system allows many
opportunities for dishonesty. For example, someone could easily add
extra ballots to an election when dealing with paper voting systems.
Basically, computerized voting systems eliminate most of the chances
for human errors, making voting safe, accurate; and reliable.
Second, the proposed system is much more affordable than the use
of paper ballots. The current voting system requires thousands of ballots
to be printed out on paper. The sheer volume of ballots needed makes
this a very expensive task. However, a computerized voting system does
not use any paper at all. Therefore, governments do not end up spending
a fortune to produce the paper ballots, which translates into huge savings.
Finally, computerized voting systems result in fewer lost votes.
Obviously, lost votes pose a big problem for elections. Unfortunately, it
is very easy for a paper ballot to get lost during the craziness of an election.
Additionally, some votes have to be thrown out if it is not clear which
candidate the voter meant to specify. With computerized voting systems,
however, all ballots are registered on an electronic database. It is impossible
for computerized voting systems to lose votes.

candidate:
a person competing for an
elected position
propose:

to suggest
ballot:
a piece of paper on which a
vote is written
reliable:

able to be counted on

sheer:
complete; absolute
specify:
to show something clearly
register:

to put information on an
official record

database:
a large amount of
information stored on a
computer system

'-----------Main idea:

There are three advantages to using a ___________ voting system.

Key point 1: They make elections _ _ _ _ _ _ _ _ _ _ because they eliminate the pos-

sibility of human error.


Key point 2: They are much less - - - - - - - - - - t h a n paper ballot systems.
Key point 3: They

:1---

\~
l

480

Chapter 3

the number of lost votes in elections.

~Step

(I

[[

Now listen to a lecture related to the topic in Step 1. Fill in the blanks of the note diagram

below. Mf4'* .

~----------

Main idea:

Key point 1:
Key point 2:

Key point 3:

mishandle:
to deal with badly or
ineffectively

Computerized voting systems have a number of

partiality:
a bias or preference

It is no safer than using paper ballots because it is


open to
attacks.

invest:
to put money into

it.

'

C\

implement:
to put into effect

It is actually more expensive to buy the equipment and


continue to operate and

expense:
money spent on something

It can lose more votes than paper systems in the event


of a
failure.

maintain:
to keep something working
on-going:

continuous
traditional:
customary; used in the past

--------~Step

----------------

,'.~

Retd the sample response below. Then answer the questions. Finally, revise the sample response.

-------------------------------
The professor disagrees with the passage by saying that computerized voting systems have
many drawbacks.
First, the passage says that computerized voting systems are safer than paper ballots. The
systems remove the possibility of human error, making voting safe, reliable, and accurate. Paper
ballots, on the other hand, can be easily lost or mishandled.
The professor does not agree with this point. According to him, it is more expensive to use
computerized systems because the equipment costs more to purchase. The system also requires
continuous spending in order to maintain and operate it.
Finally, while the passage says that computerized voting systems reduce the amount of lost
votes, the professor claims that they actually result in more lost votes. This is because a mechanical
failure could lead to the loss of thousands of votes all at once.
The professor concludes that computerized voting systems are not better than paper-based
systems.
- ... -- .,, _. ......... --- ............................. ---'
_

Making Connections 481 - :

Evaluate the sample response by answering the following guide questions.

'

1. Does the introduction mention the main idea from both the reading passage and the lecture?

D Yes

No

'
2. Does each body paragraph compare the key points of the reading and the lecture?

D Yes

D No

3. Does the conclusion summarize the main points of both the reading passage and the lecture?

D Yes

No

,.:

;,

'

(: '\
!-/'

'.:i l

Revise the sample response by correcting the problems you noted in Step 3. You do NOT need to
paraphrase the sample response.

,.;.,

-----------------------------------------~-----------------~-------~------

-----~-__-

'

~Step

Now write your own response to the reading passage and the lecture. Then answer the questions.
Remember to use what you have learned about paraphrasing. Do NOT use the same wording
as the sample response.
-------~------~---~------------------------------------------------------

('
[[
[[

Usll your own response to answer the following guide questions.


4''

1. Does the introduction mention the main idea from both the reading passage and the lecture?

~D Yes

No

2. Does each body paragraph compare the key points of the reading and the lecture?
0Yes

0No
------~

-.

3. Does the conclusion summarize the main points of both tnE:freadirig passage and the lecture?

D Yes

D No

Making Connections

483 _____.'. .

i "

..

Read the following passage. Then look at the note diagram and fill in the missing information .

di

Forest rangers often use prescribed burning to avoid large, destructive


wildfires by starting smaller, controlled fires. Controlled fire, when applied
in the right way and at the right time, prevents uncontrolled fires from
igniting or spreading.
Prescribed burning prevents wildfires from starting by reducing the
amount of fuel available. Over time, leaves, branches, and dead trees
build up on the forest floor. During the summer months, such underbrush
can be extremely dry; it can ignite instantly at the drop of a match. or the
strike of a lightning bolt. Burning away that underbrush in a controlled
manner thus decreases the risk of naturally occurring fires.
Prescribed burning can also stop an existing wildfire from spreading.
The more underbrush it has to consume, the faster it spreads. If it moves
into an area where the underbrush has already been cleared, however,
it slows down. Thus, by burning high fuel areas, forest rangers can slow
the spread of fires.
Another advantage of prescribed burning is that forest rangers can
set the time and place for a fire to ensure that it will not get out of control.
High winds, for example, might push a fire beyond its intended limits,
endangering surrounding homes or lands. Controlling when and where
a fire is started gives rangers the ability to start small fires when there is
no wind and, thus, no risk of uncontrolled spreading.

Main idea:

ranger:
an official who is responsible
for protecting a forest or
park
prescribed:
decided by a rule or a law
destructive:
causing damage
Ignite:
to begin to burn
underbrush:
the smell bushes that grow
on the floor of a forest
manner:
way
extinguish:
to make a fire stop burning
intended:
in the way that something
was meant to happen

Prescribed burning prevents uncontrolled fires from ___________

l<ey point 1: Prescribed burning _______________ by reducing the

amount of available fuel.


Key point 2:

Prescribed burning can also stop _______________ from


spreading by removing underbrush.

Key point 3:

Forest rangers can control _______________ of prescribed


fires by choosing when and where to start them.

11;
~.~?--c
.... . . 484

1_,1~ _-

:-

chapter 3

~Step

Now listen to a lecture related to the topic in Step 1. Fill in the blanks of the note diagram
below. @!&*'

~-------------------------~~------------------

Main idea:

Key point 1:

Key point 2:

A wildfire that started in the Croatan National Forest is


a good example of ____________

Indispensable:
difficult to do something
without; necessary

________ can be beneficial.


The wildfire may never have started if _ _ _ _ __

execute:
to do something that you
have pl~nned to do

in the area.

kindling:
small pieces of wood used
to make a fire

Prescribed burning did stop the _ _ _ _ _ __


_ _ _ _ _ _ _ by reducing the fuel available.

Key point 3:

The prescribed fires never posed _ _ _ _ _ __


to communities because they were controlled.

outermost:
nearest to the outside of
something

surround:
to occupy the space all
around something
pose:

to create or present
humidity:
the amount of waler in the air -,.
raging:
out of control

step 3
Read the sample response below. Then answer the questions. Finally, revise the sample response.
The reading and lecture discuss how prescribed burning prevents wildfires from starting and
spreading. The reading discusses the benefits of prescribed burning. The lecture also discusses
prescribed burning.
The reading says that prescribed burning prevents wildfires. The professor also discusses this
idea. But the professor discusses Croatan National Forest, too. He says that the Croatan fire may
never have started.
The reading also says that prescribed burning stops wildfires from spreading. The author
explains how this is true. It does this by preventing fires from spreading. The professor illustrates
this. The professor says that this happened too in the forest. So prescribed burning is good.
The reading then talks about kirest rangers. Forest rangers also help to prevent wildfires. The
professor supports this by saying that rangers started fires in Croatan National Forest, but they
were safe.
I

--1

I agree with the information in the reading and the lecture. I think that prescribed burning is a
good thing. Therefore, forest rangers should use prescribed burning.

-----~---- ------~---~

..

-------~---~----

--- ""'

"'"---"----- ... ---

--

,.

- -- --

--- - ....... .
_

.__.,

- '

Milking Connections

. "'L--.

485 - -

Ii

Jli

11n

i~m

'i::,1;

i.~!'.li
1\i:

Evaluate the sample response by answering the follow ng guide questions.


.-''

1. Does the introduction clearly explain both the author's and the speaker's points of view?

Yes

No

2. Does the response clearly explain the information presented in the reading and the lecture?

0Yes

0No

3. Does the conclusion leave out all unnecessary information or personal opinion?

D Yes

No

Revise the sample response by correcting the problems you noted in Step 3. You do NOT need to
paraphrase the sample response.

----~---------------------------~--~--------------------------------------

~Step

Now write your own response to the reading passage and the lecture. Then answer the questions.
Remember to use what you have learned about paraphrasing. Do NOT use the same wording
as the sample response.

[
[:

I
I
'

Use your own response to answer the following guide questions.


1. ,Does the introduction clearly explain both the author's and the speaker's points of view?

"'0

Yes

No

2. Does the response clearly explain the information presented in the reading and the lecture?

D Yes

D No

3. Does the conclusion leave out all unnecessary information or personal opinion?

D Yes

D No

Making Connections

r-- - -

487 _ ,

Business
~Step

Read the following passage. Then look at the note diagram and fill in the missing information.

:;,.

I'

To the middle-aged employee who works a demanding job, early


retirement might seem very appealing. Most people who do retire early,
however, find the experience to be less than satisfying for several reasons.
Employees who opt for early retirement often face financial problems.
More often than not, retirement comes with a significant loss of income.
It means that the retiree is no longer paid each week and can no longer
generate more income to save. For most people, generating enough
savings to support themselves for another thirty or even forty years is not
possible. Early retirees often find themselves in debt or face a lower
standard of living.
People who retire early often struggle with psychological problems.
Many people identify themselves with the jobs they perform and the
amount of money they earn. Their values are often the values of the
company they work for. Therefore, it is not uncommon for early retirees
to feel useless, powerless, or depressed. Without the work that has
brought meaning to them for so many years, they feel as if their lives lack
purpose.
Finally, early retirees often face social problems. They find it difficult
to maintain friendships they developed at the office. No longer employees
of a company, they have trouble relating to their previous coworkers and
often find themselves excluded from company social events. Thus, by
retiring early, they lose their social networks.

demand:
to require a lot of effort and
hard work
appealing:
attractive or Interesting
opt:
to choose
invest:
to spend money on
something that can make
you more money
savings:
money you have saved to
use later

psychological:
relating to the health of the
mind

maintain:
to keep something going
exclude:
to fail to include son1ething

or someone

L-----------..---------------------------------------------. -Main idea:

Most people who retire early find the experience to be ___________

Key point i: People who retire early often face ______________ because
they no longer get regular paychecks.
Key point 2: People who retire early also struggle with _____________
because they feel their lives lack purpose.
Key point 3: Early retirees often face ______________ and feel excluded
from events at the company.

~':.~l-- 488 Chapter 3

I'
(,j\
1'

-. .-------;----=-----,---..- : --:-.,------------,- '.--7-----:--.... -,----c-;--_-'--,--,>;:---,--,-,----,---,-',..,,-------'-,.--:--,-.-:------------'- - - - - - - - _ - - - - - - -

~Step

n Now listen to a lecture related to the topic in Step 1. Fill in the blanks of the note diagram
,

below.

tmi4''

Main idea:

Early retirement can b e - - - - - - - - - - - - - - - - - - - - a person ever makes.

Key point 1:

Many retirees live very agreeable lifestyles in terms of

Key point 2:

Early retirement can be a very _________

burden:
something difficult that you
are forced to deal with

overcome:
to successfully deal with a
problem
agreeable:
pleasant

sustain:
to keep something going

for people because they can enjoy hobbies.

Key point 3:

With regards to one's social life, early retirement is a


because it allows one to meet new people.

hindrance:
something that keeps you
from doing something else;
an obstacle
casual:
relaxed and without strong
feelings

diverse:
very different from one
another
network:
a group of similar things
that are somehow linked
together

. !;

~Step

Read the sample response below. The'n answer the questions. Finally, revise the sample respon~e.

' '.

I.

Bot~-th~-reading and lectur~-~;about::;~ re;~~m:~~;~~rea~;~~;~;l~-=~~::~:~:;;;:~~:~~-1


is a negative experience, while the lecture discusses why it is a positive one.
The reading says that early retirees face financial problems, such as debt and a lower standard
of living. This is because they cannot save enough money to support themselves in retirement.
Thus, they often find themselves wishing that they had not retired.
The reading also says that early retirement can bring about psychological problems. People
who retire early often feel useless and depressed because they no longer worl<. Some even begin
to feel like their lives do not have a purpose.
Last, the reading says that people who retire early face social problems because they lose
many of the friends they had at wort<. They find tr,emselves unable to relate to their former coworkers.
In conclusion, i11e reading and the lecture disagree about whether early retirement is a good thing.
- - _j

1 ..

1oking Connections 489 -~-

,, __ ,_
)_,_:.

Evaluate the sample response by answering the following guide questions.

1. Does the introduction mention the main idea from both the reading passage and the lecture?

D Yes

No

2. Does the response contain a proper balance of ideas from the reading and the lecture?

D Yes

No

3. Does the conclusion summarize the main points of both the reading passage and the lecture?

D Yes

No

Revise the sample response by correcting the problems you noted in Step 3. You do NOT need to
paraphrase the sample response.
<'''

--------------~------------~----------------------------------------------

..:. .:...A-- 490

Chapter 3

~-

'

Now write your own response to the reading passage and the lecture. Then answer the questions.
Remember to use what you have learned about paraphrasing. Do NOT use the same wording
as the sample response.

..,"'
~z
en

---------------------------------~----------------------------------------------------------------------------------------------~------------------

I.
Us~.your

own response to answer the following guide questions.

1. Does the introduction mention the main idea from both the reading passage and the lecture?.

~o Yes

D No

2. Does the response contain a proper balance of ideas from the reading and the lecture?

D Yes

No

3. Does the conclusion summarize the mairi points bf both the reading passage and the lecture?

D Yes

No

lvloking Connections

491

~-~

--~~

law
~Step

Read the following passage. Then look at the note diagram and fill in the missing information.

I;! I'

'I

Cl,

ii'. '
r. i

\~, '

Police departments in many states have set up speed cameras


at street intersections to catch drivers exceeding the speed limit. The
cameras measure the speed of a vehicle over a short distance and
photograph its license plate, so that the owner can be identified and
ticketed. Despite some public objection, there are many benefits to
setting up speed cameras on city streets.
First, speed cameras reduce the number of road accidents by
penalizing drivers who speed. Speeding causes accidents because
it limits the amount of time a driver has to react to changing road
conditions and the actions of other drivers. Excessive speed also makes
it harder to keep a car under control. By catching and penalizing drivers
who speed, speed cameras deter drivers from driving dangerously.
Second, speed cameras reduce the number of deaths on the road.
Speed is. not the only factor in causing accidents, but it is a critical factor
in how severe the accidents are. Many countries that have introduced
speed cameras have seen the number of road deaths drop. France, for
example, reduced road deaths by ten percent in just one year with only
100 speed cameras.
Third, speed cameras allow police officers to focus on more important
tasks than monitoring traffic. The money they bring into the government
through fines also provides additional financial support for police work.

L---------------------------------------..-----------------------~---- Main idea:

There are many benefits of ___________________

Key point 1: Speed cameras _ _ _ _ _ _ _ _ _ _ _ _ _ _ _ _ _ _ by penalizing


speeding drivers.

Key point 2: Speed cameras also r e d u c e - - - - - - - - - - - - - - - - - - because they make accidents less severe.
Key point 3: Speed cameras allow police officers t o - - - - - - - - - - - - - - -

~Step

~z

Now listen to a lecture related to the topic in Step 1. Fill in the blanks of the note diagram

below. @tJi

Main idea:

Cl

Traffic cameras are not _ _ _ _ _ _ _ _ _ as


many believe.

Key point 1:

Speed cameras do not _ _ _ _ _ _ _ _ __


_________ traffic accidents since speed
does not always cause accidents.

Key point 2:

Speed cameras also do not _ _ _ _ _ _ __


because speeding only makes a difference on suburban
roads.

Key point 3:

Speed c a m e r a s - - - - - - - - - - - - _ _ _ _ _ _ _ _ _ ,which is not a good thing.

"'

~z

rarely:
not often

Cl

inadequate:
not good enough
induce:
to cause something to
happen
tackle:
to try to solve a problem
suburban:
relating to a residential area
on the outskirts of a city

pOsitlon:
to put something in a
particular place
detect:
to notice or find something
abide:
to follow a rule

..]

-------,,

~step 3
Read the sample response below. Then answer the questions. Finally, revise the sample response.
The reading and lecture talk about speed cameras that are set up to catch and punish drivers
who speed. While the reading says that speed cameras are valuable, the professor argues that
they are not beneficial.
The reading says that speed cameras reduce the number of road accidents by deterring people
from driving dangerously. The professor argues that speed cameras, in fact, do not reduce the number
of road accidents because there are many more factors that cause such accidents other than
speeding drivers. The same has often been said of red light cameras, which perform a similar service.
The reading also claims that speed cameras reduce the number of road deaths since speeding
can lead to more serious accidents. The professor argues that speed cameras actually do not
reduce the number of road deaths because they are set up on roads where high speed limits
cause deadly accidents, anyway.
Additionally, the reading says that speed cameras give police officers the time and money to
focus on more important jobs. The professor argues that removing police officers from traffic duty
prevents them from catching drivers who drive dangerously in ways other than speeding.

'

Thus, the author's opinion is that speed cameras are good, while the speaker says that the
benefits are exaggerated. There are no speed cameras in my city, so I cannot offer my own view
on the topic.
----............................................... --------...................----------..-..

I
I

---------""'""'''"'"'""""''"''"'"''""'"-~

Making Connections

493

1'

--1

_\'

Evaluate the sample response by answering the following guide questions.


1. Does the introduction clearly explain the author's point of view? Does it explain the speaker's?

'.;

Yes

No

2. Does the response include information that is not related to the topic?

l
jlj

'f~

0Yes

0No

3. Does the conclusion include any unnecessary information or personal opinion?


0

Yes

No

Revise the sample response by correcting the problems you noted in Step 3. You do NOT need to
paraphrase the sample response.

----------------------------------------------------------~---------------

------------------------------------~-------------------------------------

--------------------------------------------------------------------------

J - - - 494 Chapter 3

'. ....~
!

I"

f'
;

J,
I
'

Now write your own response to the reading passage and the lecture. Then answer the questions.
Remember to use what you have learned about paraphrasing. Do NOT use the same wording
as the sample response.

'

i;
'

!
I'

--------------------------------------------------------~-----------------

Use yg.ur own response to answer the following guide questions.


1.

D~es

the introduction clearly explain the author's point of view? Does it explain the speaker's?

[jYes

D No

2. Does the response include information that is not related to the topic?

D Yes

D No

3. Does the conclusion include any unnecessary information or personal opinion?

D Yes

No

Making Connections

!--'---'--'----

-"------""--'--'------""-

''L'.'.

495 --.;.

Instructions: Choose the best word or phrase


to complete each sentence.
1. Living in a busy
can be very
exciting because there is always something
to do.

(A)
(B)
(C)
(0)

substance
metropolis
landfill
proponent

2. Some people believe that it is better to use


natural
to cure illnesses.
(A)
(B)
(C)
(0)

!.---

remedies
sources
byproducts
crops

3. The school wanted to raise money to build


a new sports _____.
(A)
(B)
(C)
(0)

region
demand
facility
emission

4. Scientists hope to create a car


----by the sun.
(A) impaired
(B) assigned
(C) derived
(0) powered
5. Plastic should be recycled rather than
thrown away because it is not easily
(A)
(B)
(C)
(0)

,;--- 496

biodegradable
hampered
engaged
detailed

Vocabulary ileviev,r 1

6. The teacher
a project that
would amount to a big part of our grade ..
(A)
(B)
(C)
(0)

consumed
assigned
escalated
accumulated

7. The lion is one of the most dangerous


_____ in the world.
(A)
(B)
(C)
(0)

predators
pests
peers
polluters

8. Her parents warned that being


_____ with the wrong people could
get her into trouble.
(A)
(B)
(C)
(0)

associated
sustained
inscribed
pursued

Instructions: Choose the word closest in meaning


to the underlined word.
9. Franklin had to change his clothes quickb'
because he was late for practice.
(A)
(B)
(C)
(0)

rapidly
distinctly
extensively
properly

1O. Every Sunday, her father cooked a huge


amount of food for the entire family.
(A)
(B)
(C)
(0)

massive
disposable
intensive
scant

~:

'I

r,I

.l
'

'

11. Most medicine needs to be swallowed in


order for it to get into the body.
(A) ingested
(B) replaced
(C) translated
(0) stunted

12. Ever since she was young, she has had a


wish to become a truck driver.
(A) desire
(B) hazard
(C) existence
(0) demand
13. Before email, people used the telegraph to
send important messages.
(A) combat
(B) transmit
(Q) impair
(0) engage

14. Boats that do not have motors use the


wind to lllifill them forward in the water.
(A) pursue
(B) propel
(C) escalate
(0) derive

Instructions: Write the missing words. Use the


words below to fill in the blanks.
ancient

archaeologists
translated
inscribed
precious

In the 19th century, people were very interested


in finding out more about 16. _ _ _ _ __
Egypt. 17.
went to Egypt to
find the 18.
and beautiful
items that were buried with the bodies of old
kings. They also found stone tablets that were
19.
with writing that they did
not understand. Now, much of this writing has
been 20.
. It gives important
information about how people in Egypt then
lived and what they believed.

I,

g~

"'ffi

-I

Instructions: Match the words that are


opposites.

21.
22.
23.
24.
25.

overcrowded

(A) thin

praise

(B) kind

plump

(C) empty

cruel

(D) expensive

affordable

(E) criticize

15. Eating too much fat and sugar can have a


damaging effect on your health.
(A) detrimental
(B) intensive
(C) inconclusive
(0) stimulating

I
I
!-'-'-"'

Vocabulary Review 1

--~-

--

-------.---

"----~---'--

497 - -

Instructions: Choose the best word or phrase


to complete each sentence.

6. Last night's storm was very _ _ _ __


the roofs on two houses were blown away.

(A)
(B)
(C}
(D)

1. Out of all the cars in the lot, he


_ _ _ _ for the red station wagon.
(A)
(B)
(C)
(D)

posed
maintained
opted
deterred

7. It is
to study your notes the
night before a big exam.
(A)
(B)
(C)
(D)

2. Since Jimmy liked history, he found movies


about other time periods very
(A)
(B)
(C)
(D)

appealing
psychological
inadequate
prescribed

(A)
(B)
(C)
(D)

can be very _____,


(A)
(B)
(C)
(D)

research in the

extensive
equitable
agreeable
affordable

4. After babysitting several children all day,


Deena had a
headache.

(A)
(B)
(C)
(D)

raging
prescribed
demanding
ancient

5. Many people have to _ _ _ __


serious issues before they are successful.
(A)
(B)
(C)
(D)

;:------ 498

'.~1;f
!'.:!

overcome
sustain
pose
induce

Vocabulary Review 2

critical
affordable
equitable
psychological

8. The practice required to play an instrument

3. He felt confident about his project because


he had done
library.

destructive
excluded
inadequate
intended

deterred
demanding
posed
excluded

Instructions: Choose the word closest in meaning


to the underlined word or phrase.
9. The student was punished for cheating
Eiuring a math test.
(A)
(B)
(C)
(D)
1

excluded
penalized
limited
exceeded

o. After walking into the old house, the man


noticed a strange smell.
(A)
(B)
(C)
(D)

detected
posed
executed
deterred

Ir:
'

,......

-,,.. __.,_

., 11. Everyone in our class likes Kline because


he is always so nicR
(A) agreeable
(B) diverse
(C) intended
(D) maintained
12. They had to call the fire department to Qill
out the fire before it spread.
(A) sustain
(B) extinguish
(C) execute
(D) induce
13. After the crash, both of the cars involved
were completely destroyed.
(A) accident
(B) savings
(ff) hindrance
(6) objection
14. A sharp knife is a necessary tool to have in
the kitchen.
(A) indispensible
(B) prescribed
(C) equitable
(D) casual
15. His sister tried to discourage him from
spending all of his money on video games.
(A) deter
(B) limit
(C) induce
(D) maintain

Instructions: Write the missing words. Use the


words below to fill in the blanks.

rarely

casual
tackle

invest
maintain

Inventing the newest item that everyone rushes


out to buy is a popular dream. Some p~ople
have a 16.
approach to
inventing. They draw examples of how their
creation would 17.
common
problems and ~ow it would look in their spare
time. Other people are more extreme. They
18.
money to buy the
things they need to build their inventions.
People 19.
succeed at
becoming rich from their inventions. However,
all inventors should 20.
their
excitement about creating something the world
has never seen but desperately needs.

I
:

ignite

burn

dim

light

equitable

fair

equal

challenge

hindrance ability

obstacle problem

diverse

different

varied

change

exceed

surpass

fail

excel

Vocabi1lary Review 2 499 _;.';

,._-_)_'

,, . I

Instructions: Choose one word in each group


that does not belong.

21.
22.
23.
24.
25.

Necessary Skills
llJIPll::.R:in:Hti>&~ml"....t>:m.-r.'J~llllWI!-l'.llr<-V.':l.lUlt.,_lhX.YJ%"'111rcri CJc,\/,,,r,u~_->..c--:

, ; ~'' .,-. -,-.,

Describing a personal experience


Expressing an opinion on an issue and supporting it with concrete examples and
details
Organizing ideas in an effective way

Read the prompt


and understand the
task.

,t.r,_...a.1_.e_'_._g"''
, .

'._.:,_..__.s
..

y:_-._... .'_'. ... ..'.'...'.

,..,.,_ ..:.;~:_~:_/;;y;(:
'""''1:<-.,
. .'.. .'!
.

-.'.'"::--(_.f<.-

.., ......

..

..

Be sure that you understand the prompt and what it


requires you to do.

~--------------------------~

Brainstorm.

Organize the ideas.

Try to take less than five minutes to brainstorm.


Write down all the ideas you can think of to support your
opinion.
Think of ways to express those ideas in English.
Do not try to organize these points. You will select major
ideas and organize them in the next step.

Select major ideas that can be developed into topics to


support the thesis.
Do NOT include ideas that are unconnected to the task or
topics.
Organize so that minor ideas act to support the major
ideas.

------ . . . ,_,_. J l.__~el~~t-~:.:_~~les !~~~-~early s~pport_the ~opi_~~~------ Your organization may look like this:
Introduction

Body

Restatement of the question Supporting idea 1 + examples


Thesis statement

Supporting idea 2 + examples


Supporting idea 3 + examples

Conclusion

Restatement of
the thesis

Brainstorming

503 _ __;_:

Opinion
.

~Step

W.f~"l:\:l.>;;l.:)-'AW>'!>l%W..<".C<'i'Af,!;cl'<<c~lll'.'<~-V.rn:.t:~<'~P.;,.~--'-~::c ."'<:~-'\

"-i,>. . : .. <

Read the prompt and think about your own opinion. List some ideas about your opinions in
the blanks. They do not have to agree with the examples given.
Do you agree or disagree with the following statement? Universities should pay their teachers
more money instead of spending money on athletics. Give specific reasons and examples to
support your answer.

,.

Paying teachers more:

Spending money on athletics:

- teachers deserve to get paid a lot


- encourages people to be teachers
- shows that teaching is important

- makes people interested in university


- helps bring, money to university
- helps people afford university

II

'
Read the sample response below. Identify the thesis by drawing a box around it, and then
underline the key supporting ideas for the thesis.

--------------I believe that universities should pay their teachers more instead of
spending money on athletics. Doing so attracts excellent teachers and
keeps them. Also, it sends the right signal to students.
First of all, teachers need money to pay for all the education they
needed. It takes a long time and a lot of money to complete your education
so that you can become a university teacher. After that, you do not
always get paid well, and many people who would be excellent teachers
do not enter the field of education. So universities should pay teachers
more so more people will become teachers:
Second, by paying their teachers well, universities can ensure that
people who start out in a teaching career stay in that career. A problem
many universities face is keeping their best teachers after hiring them.
Many people with PhDs want to teach and start out doing so. However,
they find out later they can earn more money by working for a company
or the government This means that such people, who are excellent
teachers, choose to leave teaching because of money.
Third, education needs to be the most important part of a university.
Universities exist to educate people. When they spend more money on
athletics, it sends the wrong message to students. It says to students
that education is notreally that important. Conversely, when universities

signal:
a message that has a
special meaning

field:
a particular area of study or
work
ensure:
to. make sure that something
happens
face:

to deal with; to confront

conversely:
on the other hand

adequately:
in a way that is good
enough
retain:
to keep something

valued:
considered to be important

~~
...

attract good faculty and pay them adequately, it creates a better


environment for students and shows that their education is important.
In conclusion, by paying teachers more money instead of spending
so much on athletics, universities can become better institutions that
attract and retain the best faculty. They will communicate to students
that their education is important and valued, which benefits everyone.

.
'

In complete sentences, answer the following questions about the thesis and topic of the response
in Step 2.
1. What is the thesis statement of the essay?

2. What is the topic sentence of body paragraph 1?


':'.

~------------------------------

/:

-,~

.3. What is the topic sentence of body paragraph 2?

4. What is the topic sentence of body paragraph 3?

Brainstorming

cc::__

___ ._-. __

505 - -

~Step

In complete sentences, answer the following questions about the organization of the response
in Step 2.
:.

1. Which "side" of the prompt does this essay take?

2. How does the writer support the thesis statement?

3. Does the writer present a comment or idea from the opposing argument in the conclusion? If so,
what is the comment or idea?

4. What is the main idea of the conclusion?

I;
: i

~':.'~:..----~-

Read the sample response presenting another possible answer to the prompt from Step 1.
I do not agree that universities should pay teachers more rather than
spend money on athletics. This is because athletes play an important
role in the life of a university. Their contributions are as important as
those of teachers.
First of all, universities want to have well-rounded students.
Well-rounded students are successful. They give the university a good
reputation. Athletics provide students with the opportunity to learn the
importance of personal health, competition, and being part of a team.
Universities that support athletics and academics equally have more
well-rounded graduates.
Second, university athletics are often the only way for some students
to enter college. Many students from disadvantaged backgrounds do
not have the opportunities for an excellent education to prepare them for
college. What they do have is a chance to prove themselves as athletes.
506

Chapter 4

contribution:
an action taken in order to
help in the success of
something

well-rounded:
having knowledge of many
things
disadvantaged:

having fewer opportunities


than other people
dedicated:

focused on completing
something
alumni:
people who have graduated
from a post-secondary educational program
finance:
to pay for something

Universities that support college athletics can include these hard-working


and dedicated individuals in the promise of a college education.
Finally, athletics can help a university make money. Many universities
rely on their alumni for money. They use this money to build new buildings,
finance scholarships, and improve life for current students. Many of these
alumni support their university sports programs. Watching their university's
team win an important game gives them pride. The more they feel pride,
the more likely they are to give money to make their university even better.
Therefore, it is important to support athletics. It brings alumni closer to
the university. It may also encourage them to give money.
In conclusion, athletics and academics are both important for
universities. For that reason, teachers should be paid well, but not at the
expense of university athletic programs.That would be bad for students,
alumni, and the university in general.

~Step
; ':

pride:
a feeling of satisfaction
because of an achievement
at the expense of:
done even though it hurts
someone or something else .

['
[.
-

'':

G'\

,'

''.

"ti

G'\

After,studying the two sample responses, give your own opinion on the prompt. Brainstorm
you~<ideas below. Then, write or type your essay.

..----------------~------------~-------.---~-------------------------------------------~-

_____________________ ___
_;_

,_

__ ...:. .__________________________________
.

._

________ _

Brainstorming

507 - -.. :.

ii
',,

1:~

I;,:.!

tf

"I'

Experience

1111'

t"1:::l

ll.t.:
~"

"''!

3l:n

iiL I
i:i t

rk[

]: I
:

;:re ,-,

i;:'i

i~ ,!"

ml"~~ll"'.:iMC;t;;:2ll!-~'!1V~~"1\1"~''~-o-~,o;;<_~,,:,;.:;,..;,;::-"1:- .. ~X,':>oO.'

1,, I

I, i

'1'lrii

1
'

;~a1;t!
1

.1l.li.,
.:~l:11"

!1JW1

'.::1
'

~Step

Read the prompt and think about your own experience. List some ideas about your experiences
in the blanks. They do not have to agree with the examples given.

'
Some people prefer to work on only one task at a time. Others think multitasking is a better
way to get more work done. Which do you prefer and why? Give specific reasons and examples
to support your answer.
Working on one task at a time:

Multitasking:

- allows you to focus on one thing


- prevents you from getting confused
- helps you work harder

- allows you to get more done


- prevents you from getting bored
- helps you develop more than one skill

..!.1
'-i

,,[.: It
i! !

':I
' I

; :' i
I
I

~Step

Read the sample response below. Identify the thsis by drawing a box around it, and then
underline the key supporting ideas for the thesis . .
While some people might like multitasking, I prefer to work on one
task at a time. When I do that, my work is of higher quality, I feel calmer,
and I am more productive.
First, I can pay better attention to my task when I work on only one
thing at a time. For example, I have a computer programming class,
Programming requires me to pay careful attention to details, Because of
this, if I try to do more than one thing at a time, I am more likely to make
mistakes. By focusing on one task at a time, my work is of the highest
quality possible,
Second, when I only have one thing to do at a time, I feel calm and
relaxed, That way, I enjoy my class work more, and I am willing to take
on new challenges, When I have too many things to do at once, like
writing a program., checking email, and responding to my classmates, it
breaks my concentration, and I get frustrated and upset with my work
situation.
Third, working on one task at a time increases my productivity,
Because I can focus on just one task, I put all my mental energy into
figuring out how to get it done most effectively. Similarly, I am not distracted
by other problems like I would be if I were multitasking. For that reason,
I am more productive and efficient when working on only one.task.

;_..,, - - - -508

multitask:
to work on more than one
thing at the same time
productive:
able to get a lot done
programming:
the designing or wrtting of
computer programs

take on:
to try to do something that
is difficult
challenge:
something that is difficult to
do

concentration:
the ability to pay close
attention to something
distracted:
unable to focus on
something
preferred:
wanted or liked more than
another thing

Ch<ipter 4

f(1't

'~--!

l;

-~~____;___..---'---~~~-.---~_.,.:_+..--.,~.;.,..___,,,_.:._~~....:-.r-:___;:._,__;__;.---'-'-.-i;....._;-.- - .___, _ . ___--_

~-__;~;:.O~~;..;,_,.-'-;~~~'.;_~.:.:-~:;_;:;,_,;.;:;::..~,:,;~-:..-.,.".;:~;..:~~~-"-

"'

''..

(';
[
Cl

In conclusion, I prefer working on one problem at a time rather than


spreading myself thin by multitasking. With my preferred approach, I
produce better quality work, feel better about doing so, and am more
productive.

~Step

'

'

In complete sentences, answer the following questions about the thesis and topic of the
response in Step 2.

1. What is the thesis statement of the essay?

2. What is the topic sentence of body paragraph 1?

.,

'

3. What is t11e topic sentence of body paragraph 2?

., 4. What is the topic sentence of body paragraph 3?

Brainstorming

509 - . -..

~Step

In complete sentences, answer the following questions about the organization of the response
in Step 2.
1. Which "side" of the prompt does this essay take?

2. How does the writer support the thesis statement?

3. Does the writer present a comment or idea from the opposing argument in the conclusion? If so,
what is the comment or idea?

...,

4. What is the main idea of the conclusion?

,:

Read the sample response presenting another possible answer to the prompt from Step 1.

[~~~~as~~ople had. adeq~ate t~~e to ;:cu~-o-~-on~~ing at a


II

time. With modern technology, we can now multitask, which I think is


the better way to get work done.
First, when I multitask, I am a more efficient student. If I run into a
problem and need to wait for my teacher to help me, I do not have to
waste time. By multitasking, I can use that time to pick up another
assignment and start it right away. That lets me be more productive and
efficient with my time.
Another reason I prefer multitasking is that it allows me to be more
creative. I enjoy designing web pages for my school newspaper. It
means I need to be creative, and to do that, I cannot work in a linear
manner. Instead, I do lots of things at once, and that helps me generate
new ideas. So as soon as I have an idea, I work on that idea, even if it
means dropping what I was already doing.

-----------------.-

-------- - --

-------

-----":_---;-

adequate:
enough

run into:
to encounter
design:
to plan and/or build something
linear:
following one after another;
directly connected

generate:

to create
drop:
to stop doing something

dull:
uninteresting; boring

engaged:
interested in or focused on
something

- .-.--.--.-- ___, ________ ,_,

--------

~--

----=---------

Finally, multitasking keeps me interested in what I am doing. Doing


one thing is boring; some tasks, like doing math homework, are just dull.
However, I have to do them. I make it interesting by doing my math
homework while I am chatting with my friends online. I do the homework
while I wait for them to respond to me. That way it is not as boring.
Modern technology makes our world more interesting, and it also
makes multitasking my favored means of working. When I multitask I am
more productive, creative, and engaged, and that means that my work
is of better quality in the end.

~Step

After studying the two sample responses, give your own opinion on the prompt. Brainstorm
your id!!as below. Then, write or type your essay.

~------~-----------------------------------------------------------------~-------------------------------------------------------------------------

'
--------------------------------------------------------------------------

____________

..._

I. .

.:":;-

Br au<storm1ng

511

--~~, .

Opinion
!Mll.>W;;;~H,'.<,\l~;><><;;:;;;.;.,::,:f.;tV:';,;-ccc.vfec-1

~Step

-;', >~< ''' - ',

'

Read the prompt and think about your own opinion. List some ideas about your opinions in the
blanks. They do not have to agree with the examples given.
Do you agree or disagree with the following statement? In twenty years, students will do all of
their schoolwork on a computer. Give specific reasons and examples to support your answer.

j!

1:,,

jl

'

Reasons students will do all schoolwork


on computer:

Reasons students will not do all schoolwork


on computer:

- amount of information shared online is


growing

- too much information online may cause systems


to crash

- much work done in businesses is already


done on computers
- people want to use less paper to protect
environment

- schools are generally reluctant to change how


things have traditionally been done
- computers and software are more expensive
than paper and pencils

"
v r,;.;,,,,,
~'S;";iJ\d .f.

lo..

'--!-

,.j)

Read the sample response below. Identify the thesis by drawing a box around it, and then
underline the key supporting ideas for the thesis.

Every year, more teachers and professors are encouraging students


to make the most of computer resources. Therefore, I agree that in
twenty years, students will do all of their schoolwork on a computer,
One reason I think this is because there is a growing trend toward
sharing information online. Many businesses, for example, have already
stopped handing out paper memos and instead post them online or
send them via email. In addition, many universities are already setting up
online discussion forums for classes. On these forums, students can
. discuss class lectures as well as post assignments. The professor can
also upload reading materials or lecture notes for the students to study,
Therefore, it is highly likely that in twenty years, all schoolwork will be
done online.
Another reason I think all schoolwork will be done on computers in
twenty years is that society in general is moving away irom paper products.
~,--- 512 Chapter 4

resource:

something that helps you


with a task

growing:

-1

increasing

memo:
a short note or message
post:
to put information on a
website

forum:
a webpage that people use
to discuss things

waste:
the trash that people throw
away

refuse:
to say that you will not do
-something

,,

;
I1~.
---~

~t

[
(

People are getting more and more concerned about. environmental


issues like pollution and waste. Already, many professors refuse to print
out paper assignments. They encourage their students to send their
work via email, and they insist on sending back corrections the same
way. Because using computers saves paper, more and more people are
seeing their use as good for the environment. In twenty years, probably
no one will be using paper to do work.
In conclusion, Iagree that students will do all their work on computers
twenty years from now. Perhaps the rest of society will catch on, too,
and start doing all of its work on computers as well.

catch on:
to understand

.
'

In complete sentences, answer the following questions about the thesis and topic of the response
. in Step 2.

1. What is the thesis statement of the essay?


~

2. What is the topic sentence of body paragraph 1?

..., . ' :

3. What is the topic seriterice of body paragraph 2?

Brainstorming

513 - . :.

~Step

In complete sentences, answer the following questions about the organization of the response
in Step 2.
1. Which "side" of the prompt does this essay take?

2. How does the writer support the thesis statement?


.~

'

,)

"
3. Does the writer present a comment
or idea from the opposing argument in the conclusion? If so,

what is the comment or idea?

I
I

,I
I

;,,,,,:,:}--- 514

4. What is the main idea of the conclusion?

Read the sample response presenting another possible answer to the prompt from Step 1.
Today, many classrooms provide computers for students to use, and
many students choose to do homework assignments on their home
computers. Despite the growing popularity of computers, I do not think
it is likely that students will do all of their work on computers in twenty
years. I think this for several reasons.
First, while computers are a useful tool, they cannot do everything.
There are many instances in which a computer is not practical for
completing assignments. Traditional art or music classes, for example,
require students to use materials or instruments that cannot be replicated
digitally. Even biology classes assign tasks, such as dissections or field
observations, that cannot be done on a computer.
Second, computers use up a lot of energy in the form of electricity.
Even today, energy resources are getting scarcer and more expensive,
Twenty years from now, it may not be possible to run computers for
Chapter 4

provide:
to give

popularity:
the quality of being liked by
many people
replicate:
to copy

digitally:
by using a computer

scarce:
rare; in small quantities

access:
the ability to use or have
something

abundant:
available in large quantities

!'
I.

>.

I""

every student since it would require more electricity than society may be
able to generate.
Third, many students still will not have enough access to computers
twenty years from now to do all their work on them. Computers are
expensive. Even in America, there are many schools that cannot afford
supplies like pens and paper, let alone computers. In twenty years, basic
computers may get cheaper and more abundant, but there will still be
families and classrooms that cannot afford them.
For these reasons, I .do not agree that students will do all of their
work on computers in twenty years. Although computers will become
more and more useful for completing tasks, they will never be the only
tool used for doing work.

~Step

,'

[[

After studying the two sample responses, give your own opinion on the prompt. Brainstorm
your ideas below. Then, write or type your essay.

-r;------------------------------------------------------------------------

.
. .
--------------------------------------------------------------,--,----,.....,..--_--...,..
----------------------------------------------------------~---------------

Brainstorining

515 --:\:.

Opinion
"11-11.-<;"m;:;v.,,.m~'"""K.wn~>E!.._..>::"-<'>""'''''h-'>'>-V:

1t
I.'

r
i

~Step

,- ' -- - -

Read the prompt and think about your own opinion. List some ideas about your opinions in the
blanks. They do not have to agree with the examples given.
Do you agree or disagree with the following statement? It is important to learn about events
in other countries even if they do not directly affect you. Give specific reasons and examples
to support your answer.
Pros of learning about other countries:

Cons of learning about other countries:

- makes you aware of world events


- helps you if you travel to a new country
- can help you understand your own country

- can give you a false impression


- will not teach you anything useful
- stops you from learning important things

ti>

Step :?.

Read the sample response below. Identify the thesis by drawing a box around it, and then
underline the key supporting ideas for the thesis.

Every day, events happen in countries all over the world that only
directly affect a relatively small number of people. Even though those
events may not have immediate significance, I agree that it is important
to learn about them even if they do not directly affect you.
One reason is that being educated about crises in other countries
can help you deal with similar crises in your own country. When smallpox
broke out in Europe in the 18'" century, for example, doctors figured out
how to eradicate the disease by vaccinating people. Then, when. the
disease broke out in other countries across the world, they knew how
to deal with the crisis.
You should also learn about events in other countries because you
may one day visit them. It is important to know what is happening in a
country you travel to so that you can be safe. If you know that the
political situation is unstable or there is a civil war going on, you would
be wise to postpone your visit Being informed of a country's current
events is essential to traveling safely within that country.
Learning about events in other countries also gives you a more
accurate understanding of the world. If you focus only on events in your
own country, you may not realize, for example, that things like disease
\"..(r---- 516
! ,'

Chapter 4

relatively:

compared to other similar


things
significance:
the meaning or importance
of something
crisis:
a sudderi and severe
problem
eradicate:
to get rtd of something
completely

vaccinate:
to protect someone from a
disease with a medicine
unstable:
changing olten
postpone:

to delay doing something


fellow:
referring to similar people

I[
[
[

and poverty are all major issues in the world. It is important to understand
global issues in order to protect yourself and to help out your fellow
humans.
In conclusion, I think that it is very important to learn about major
events in other countries. Even if those events do not affect you directly,
learning about them is of great value.

~Step

In complete sentences, answer the following questions about the thesis and topic of the response
in Step 2.

-a

~
n

1. What is the thesis statement of the essay?

2. What is the topic sentence of body paragraph 1?

3. What is the topic sentence of body paragraph 2?

4. What is the topic sentence of body paragraph 3?

Brainstorming

517 ----:.

In complete sentences, answer the following questions about the organization of the response
in Step 2.

1. Which "side" of the prompt does this essay take?

2. How does the writer support the thesis statement?

3. Does the writer present a comment or idea from the opposing argument in the conclusion? If so,
what is the comment or idea?

4. What is the main idea of the conclusion?

Read the sample response presenting another possible answer to the prompt from Step 1.
If events that happen in other countries do not affect you directly, I
do not agree that it is important to learn about them.
First of all, people have far too much to worry about in their own
countries to be worrying about events that happen across the globe.
The United Nations, for example, spends a great deal of money and
manpower trying to pacify conflicts in many different countries. Meanwhile,
the countries who are part of the UN still struggle with things like poverty,
unemployment, and P()Or health care. With so many things to worry
about, it is best to just focus on your own country.
Second, learning about major current events in other countries can
give you the wrong impression about people who come from that country.
Some events are portrayed in the news in bad ways, especially when it
comes to the way governments do things. Because we often associate
citizens with the actions of their government, it often gives us a bad
:.~~~:1~ 518 Chapter 4

i'"'
;;J.I

('>;ti __ "'t'~--~-

globe:
the world

manpower:
the people available to do a
job
pacify:
to establish peace
struggle:
to attempt to do something
difficult
poverty:
a situation where people
are very poor
impression:
an opinion about a person
or thing
portray:
to describe something in a
partiCular way

I
I

~'

impression of those people. We would be better off if we just did not


learn about those events at all.
Finally, it is not important to learn about events in other countries
because they often will not teach you anything about your own situation.
Many worldly events are so different from the events you may experience
in your day-to-day life that learning about them will not show you how
to live your life better. It is therefore more important to spend your time
and energy learning about the lives of people like you.
Therefore, I do not agree that learning about events in other countries
is important, especially if they do not directly affect your own life.

associate:
10 connect two things or
ideas together

[
-

'

After studying the two sample responses, give your own opinion on the prompt. Brainstorm
your ideas below. Then, write or type your essay.

,,

__ ,_..,... ___________________________________________________________________ _

--------------~----------------------------------------------------------

....,._.,...

--\

Brainstorming

519 ---.'.

Necessary Skills
l"J'ilMH<m~~IBU<Z<fil"i',"11,_>:'::.<V,;,;>;;~;-i,,-,"-'~-'f,l"_.\~;;1..;,\;

';;;\~-'.-,,, .- '

Stating your opinion or thesis clearly


Stating clear and strong topic sentences that support the thesis

Strategies
Make your thesis statement clear and concise.
For your thesis, do not write, "I agree with this opinion." Restate the question when
giving your opinion, such as "I agree that when people reach a certain age, they
should no longer be able to drive."
Make your topic sentence a summary of all the points you will cover in the paragraph.
Write clear topic sentences that will naturally lead into the rest of the information in
the paragraph.
Example:
Weak topic sentence -+ I think cows are more useful than horses.
Strong topic sentence -+ Cows are more useful than horses for two main reasons.

~:

1Nnting

Thesis Statements ilnd Topic Sentences

521 _ _:....

Thesis Statements
~Step

Read the following prompts and sample thesis statements. Underline the words in the thesis
statements that are also used in the prompts.
Prompt 1:
r.. ,,.,,,,.,, ........,,,,.. ,., .............,--------..,--,-----------,---------------------..-----...,...... ,...... ,]

Describe a time that you were encouraged to try something new. Explain why you decided
to try or not to try it. Give specific reasons and examples to support your answer.

:
;

~u -"-""""~"

--

-~- ~~--------- ~---- ~~---------------~--------- -~-~-~-"----.~----~ "~


'"'

>. :

Thesis Statement 1:

;f !'

When I was twelve, my brother encouraged me to try out for a football team, but I decided not to try
out for two reasons.

i;):ii
,. J,

,_- J

:- ~;:t" :
Prompt2:
Do you agree or disagree with the following statement? Children should be taught to
use computers at a very early age. Give specific' reasons and examples to support your
answer.

" 'I

','

Thesis Statement 2:

There are three main reasons why I believe that young children must be taught to use computers.

Prompt3:

' -1
,

Describe a place that you have traveled and state whether it changed your opinion of
the place that you live. Give specific reasons and examples to support your answer.

.J

Thesis Statement 3:

When I traveled to a larger city in my country, it made me realize how much I love my own town, and
there are three reasons why I feel this way.

>--- 522

Chapter 5

II

Some people like movies that are fast-paced and entertaining. Others like movies that
have deep meaning and make them think. Which type of movie do you prefer? Give
specific reasons and examples to support your answer.

'. Thesis Statement 4:

In my opinion, the most enjoyable movies to watch are ones that make you think, and I believe this for
two reasons.

Read the following prompts. Decide if each prompt asks you for your experience or your opinion.
Then write a thesis statement in response to each prompt.
Prompt 1:

l~~:~~i~:t~e ~::t imp~~a~~ ~er:~~;~ -~~~~-~;e:~~ :~~;~i~-~~~ ~~:~ ~~r:~~-i: i~~~-~~~~~--1

L~i'.e specific ~eas~_ns and examples to support your answer.

Does this question ask you to explain your opinion or your experience? Select one.
opinion

experience

Prompt 2:
Do you agree or disagree with the following statement? People should never do things
that harm the environment, even if they are beneficial in other ways. Give specific reasons
and examples to support your answer.

Writing Thesis Statements and Topic Sentences

523 - ' - . .

r.. . .

'
.!

"
:L
,,

. I

Does this question ask you to explain your opinion or your experience? Select one.
opinion

experience

D
. t:

Thesis statement:~-------------------------

Prompt3:
r---------------------------------------------------------------------------------------------

t Some people prefer to spend all of their money after they earn it. Others would rather save
most of their money to use later. Which do you think is better? Give specific reasons and
examples to support your answer.

.1

Does this question ask you to explain your opinion or your experience? Select one.
opinion

experience

Thesis statement: - - - - - - - - - - - - - - - - - - - - - - - - - - - -

Prompt 4:
Describe one of your skills that you believe has helped you to be successful. Give
specific reasons and examples to support your answer.
Does this question ask you to explain your opinion or your experience? Select one.
opinion

experience

Thesis statement: --~---'-----------------------

\"...\--- 524 Chapter 5

*-

Topic Sentences
~M~=.'.ll*=-{i'l,.v~<:.X!<'.>i>OF;;,'t~<<J;;..'i,iO.<'M.>t-<:'i'-<'~':>:~;>,tc~.,F.:.:"";;-.''"'>:~,_-,

'.

)Read the prompts and three sentences that could be used in a response to each prompt. One

!sentence is a thesis statement. Another sentence is the topic sentence of the body paragraph.

Thesis statement
Topic sentence
Reason, example, or detail

( 1 )
( 2 )
( 3 )

you agree or disagree with the following statement? Having fun is just as important for
ad1Jlts as it is for children. Give specific reasons and examples to support your answer.

irhe last sentence is a reason, example, or detail used in a body paragraph. Number the
sentences as follows:

There are three reasons why I feel that having fun is just as important for adults as it is for children.
For example, my dad likes to play baseball with me on weekends.
For one, I think that adults need to be able to relax and have fun because they work all the time.

~::~r;~: ~-ti~~ ~~:;-~o~-~~re ~;;~~t~~:~ -~; :~~:t~i~g.---~~~. ~i~-~~~-~~:r~~-~: Y~~~

fear? Give specific reasons and examples to support your answer.


---- -- --- - .
- . - ...... -- ------ - . - - -- -- .
,.
---~

... -

~-

I asked for help from a really nice woman that I met, and she helped me figure out where I
was going.
Another way that I overcame my fear was by telling myself that I could do anything that I
wanted to.
I once had an experience where I was scared because I went to a new place alone, but I
overcame my fear in two important ways.
Prompt 3:
Some people like to try new things as much as they can. Others prefer to do things that
they know very well. Which do you prefer and why? Give specific reasons and examples to
support your answer.

I like to try new things because I get bored if I do the same thing all the time.
I definitely like to try new things as much as I can.
When I tried joining my school's diving team, I found that I really enjoyed it.

'Nriting Thesis Stat~ments and Topic Senten(es

'

525 ----'.

Prompt4:

!. . -~: ~:: agree

or

dis:~;:: ~it~ ~~: ~:11:~i;g:~:t:~:nt; -~~i::::~i:: :~::1~ ::~:i~:

students to live on campus during their first year of university. Give specific reasons and 1

L.~~~~~l~s t~-~~~~~~ ~.~~- a~ ~:e~.

~Step

I
,:-1

.- . . . .. .. . . . . - - - ---- --- . . J

In my opinion, universities should not require students to live in the dormitories for their
freshman year.
Living in an apartment is more enjoyable than living in the dorms.
My friend got stuck with a really annoying roommate in the dorms.

(.
(

;,.\.

Read the following prompts. Complete the thesis statements. Then provide three reasons or
examples that you would use to support each thesis in a response.
Prompt 1:
Some people do not like buying products made by machines because they might be of
lower quality. Do you think products made by hand are better than products made by
machines? Use specific details and examples to support your answer.

Thesis statement: In my opinion, products made by hand - - - - - - - - - - - - - products made by machines.


Reason/Example 1: - - - - - - - - - - - - - - - - - - - - - - - - - Reason/Example 2: - - - - - - - - - - - - - - - - - - - - - - - - - Reason/Example3: _ _ _ _ _ _ _ _ _ _ _ _ _ _ _ _ _ _ _ _ _ _ _ _ __
Choose one of the ideas you listed above. Rewrite the idea as a full sentence that could be
used as the topic sentence of a body paragraph.

Topic sentence: - - - - - - - - - - - - - - - - - - - - - - - - - - - -

--, __, _______ _

__ , .____ ,, ___ "-''-;--'"

"'"- - _: __

.-:--,-- ..

.(

'

Do you agree or disagree with the following statement? You can tell a lot about a person
by the clothing that he or she wears. Give specific reasons and examples to support
your answer.

~--------------------------~------------~-~-~---~---~-'-----~--,---,~--~---------

.. Thesis statement: I believe that y o u - - - - - - - - - - - - - - - - - - - by the clothing he or she wears .


.Reason/Example 1: _ _ _ _ _ _ _ _ _ _ _ _ _ _ _ _ _ _ _ _ _ _ _ _ __
Reason/Example 2: _ _ _ _ _ _ _ _ _ _ _ _ _ _ _ _ _ _ _ _ _ _ _ _~-a

Reason/Example3: _ _ _ _ _ _ _ _ _ _ _ _ _ _ _ _ _ _ _ _ _ _ _ _ __

g
I"\

. Choose one of the ideas you listed above. Rewrite the idea as a full sentence that could be
used as the topic sentence of a body paragraph.

Topic sentence:
'

'

:4:
Prompt 3:

-- .. - .............. ---------------------------------------- ......................... --- ..........................1

~any schools take field trips in which they take students out of school for a day to visit a
place like a zoo. Describe a trip that you think would teach students something valuable.

~-i-~~-~p-e~ifi, r~~~~~-s--~n-~-~~~:~~~~-~~-~~~~~~~~~r-~-~~~e~ ........

...... - .......... __

I
I

;,'

'!

Thesis statement: I think that a trip to ______________ would be a valuable


learning experience.
Reason/Example 1: - - - - - - - - - - - - - - - - - - - - - - - - , - - - Reason/Example 2: _ _ _ _ _ _ _ _ _ _ _ _ _ _ _ _ _ _ _ _ _ _ _ _ __
Reason/Example3: _ _ _ _ _ _ _ _ _ _ _ _ _ _ _ _ _ _ _ _ _ _ _ _ __
Choose one of the ideas you listed above. Rewrite the idea as a full sentence that could be
used as the topic sentence of a body paragraph.

Topic sentence: - - - - - - - - - - - - - - - - - - - - - - - - - - - -

~Vriting Thesis Statements and Tdpic Sentences

527 - -.....-:

Prompt4:
~--- .. ~--O'

- """'"'"""

-<--"> -~"---.--------~---

'

-,-----'>~<-----~---o;---~-"""-''""""'""'~'"'"

Most people listen to music every day, and many people lear~ to play an instrument
when they are young. Why do you think music is important to people? Give specific
reasons and examples to support your answer.

Thesis statement: I think that music is important to people mainly because _ _ _ _ _ _ __

Reason/Example 1: - - - - - - - - - - - - - - - - - - - - - - - - - Reason/Example2: _ _ _ _ _ _ _ _ _ _ _ _ _ _ _ _ _ _ _ _ _ _ _ _ __
Reason/Example3: _ _ _ _ _ _ _ _ _ _ _ _ _ _ _ _ _ _ _ _ _ _ _ _ __
Choose one of the ideas you listed above. Rewrite the idea as a full sentence that could be
used as the topic sentence of a body paragraph.

Topic sentence: - - - - - - - - - - - - - - - - - - - - - - - - - - - -

i:

Strategies
<;J.1>J1'J\'i'.nV.lt~U-'!O<WJ;F.ZlfdCi'.'Ri'.:<N:'ff,.</::;..,-_f;,l1

:~.;;~-fcH.'~MC.l.l'1U.'.l:;.'<

'"'''' ,<. "''"' ..in.,.

Do NOT try to say everything in the introduction; save details and examples for the
body of your essay.
Do NOT start with a statement that is too general; a more specific statement better
sets up the information to follow.
Characteristics of a good introduction:
one paragraph (three to five sentences)
an introduction to the general topic of the essay
includes the thesis statement and a restatement of the question
includes points that will be discussed or elaborated on in the body

Write an accurate and clear topic sentence for each body paragraph.
Make sure there are logical connections between statements.
Characteristics of a good body paragraph:
one to three paragraphs
a topic sentence that states the main idea of that paragraph
has specific examples, reasons, or other details
includes other sentences that link ideas or show transitions between ideas

Do NOT use the same words and expressions in your conclusion that you used in
your introduction.
Do NOT introduce new ideas or concepts that belong in a new body paragraph.
,-----~--------------~--------"~-------

I Characteristics of a good conclusion:


I one paragraph (three to five sentences)
a restatement of your thesis in different words
has a summary of your main points
includes one or both of the following: a consideration of the opposite opinion
I and a recommendation
L..._ .......................- ..... ________ .._ .. ____..

Makin(_) Ideas Flow

529 - -...

- - 530

Opinion
~-i..-'i'Z\)::W.-,;.z-!l;~!;(tl',ro,.>;.,,ii' ..,.,.~.,.,,,_.,,_,,,,.,,~;0,\.'''"-"-''''~':

~Step

..

.0,-.,,,,.,-1,,~-,

-.:_-,_-,

Read the prompt and think of ideas to list in the blanks.


Sorne people watch the news in order to get their information. Others like to look for information
themselves in newspapers or on the Internet. Which do you think is better and why? Give
specific reasons and examples to support your answer.
Watching the news:

Newspapers and the Internet:

- easier than looking for news


- lets you see and hear
- gives just the information you need

- let you search for the news you want


- do not have to wait for a story
- more depth to the stories

~Step

Read the sample response. Think about the role of the sentences in each part of the essay.
Look for any transitions that link the ideas and underline them. Then put the sentences in the
right order.

accessible:
easy to use and understand

Introduction:

(A) Even though other people might like to watch the news on 1V to
get information about the world, I prefer to use newspapers and
the Internet to get news.
(B) I feel this way because newspapers and the Internet are more
accessible, informative, and easier to share with others than 1V.

specialized:
focusing on one particular
thing or set of things
tend:
to usually do something
in depth:
in a lot of detail

Body paragraph 1:

sensational:
ma~ng something seem
more exciting than it is

(A) With 1V, I have to wait through other news stories and
commercials before the weather comes on.
(B) First of all, newspapers and the Internet are more accessible
because I can find exactly what I want when I want it.
(C) If I want to know the weather, I can get that information in a few
seconds from a newspaper or the Internet.

Chapter 6

--- ------- ---

informative:
giving a lot of information
clearly

-:<:~

devote:
to give time or effort to
something
celebrity: .
a famous person

r.

Body paragraph 2:

'

'

(A) They tend to feature stories that discuss issues in depth.


(B) 1V news, on the other hand, tends to be sensational, devoting a
lot of time to celebrities and crime stories.
(C) Second, newspapers and the Internet are more informative.
(D) They also make it easier to learn about international events and
specialized issues like the environment, which might get only minor
coverage on 1V.

Body paragraph 3:

(A) You cannot send a story like that with 1V news.


(B) If I am reading an article on the Internet that I think my friend might
like, I can simply send it to her by email.
(C) Finally, I prefer newspaper and the Internet for my news because
they make it easier for me to share the information that I find.
'Conclusion:

(A) That may not be for everyone, but I think these sources are better
than 1V news because I can access them when I want, I can stay
informed, and I can pass on what I learn to others.
(B) In conclusion, I prefer to get my news from newspapers and the
Internet.

tv1,>king ldras How

531 -----.:'

~Step

Look back at the ideas you wrote in the blanks for Step 1. Write your own response to the
prompt using one of your own ideas or another idea from Step 1.
if.

i'
:''

i1

i!

""

.'I

..

.-''

'.'[.:

J,,.___
'

532

Chapter 6

- - - _ -::-:-~..- .---,-'-,----;- -- - - - - - - --- ---,------:--.----,----:-"---;--_-----,-------------,--,----_-----,--__ ---,----,-_-----.------- ------------------------- - - - - - - - - - - - - -

[[

Opinion
~'WUiill>:l;'-'l!I':';i'iiilllX'.-V.'\f\llJ\->'&J'.l,>ll'.!>~~-O'a">-'0-~;<~H,o'.'_/',"''" <'':-~-,~

,-,,._,_,_- -,

~~~;;

~'Read the prompt and think of ideas to list in the blanks.


;i<!:
iDo you agree or disagree with the following statement? All university students should be
'
!required
to take literature classes. Give specific reasons and examples to support your answer.

Pros of literature classes:

Cons of literature classes:

- teach you important life lessons


- teach you how to read better
- gives just the information you need

- cannot apply to many careers


- can be boring
- make you do a lot of writing

.. ~Step 2
Read the sample response. Think about the role of the sentences in each part of the essay.
Look .for any transitions that link the ideas and underline them. Then put the sentences in the
right order.

Introduction:

(A) All university students should be required to take literature classes.


(B) This is because literature is valuable for what it can teach students
and how it can change them.

bridge:
to overcome; to bypass

connected:
able to communicate
successfully
conflict:
a disagreement between
people or ideas

wise:

Body paragraph 1:

(A) It is important to learn about these things because we live in such


a diverse world.
(B) Literature is one thing that can bring us together and bridge these
differences.
(C) First of all, literature is valuable because it can help students to
learn about other people and cultures.
(D) It can also help us understand people that are different from us.
(E) This means that people are more connected with each other
than ever before, including people from very different cultures and
backgrounds.

smart in terms of making


good decisions
dilemma:
a problem that requires you
to make a difficult decision
attitude:
a set of opinions or beliefs
about something
passion:

a very strong interest

l'vlak1ng Ideas Flow

.533

--1

Body paragraph 2:

(A) These are often problems that all people face, such as love, conflict,
anger, loss, and sadness.
(B) Second, literature can help students become wiser and more
mature.
(C) Also, when you read literature, you learn about different individuals'
experiences with these issues.
(0) These experiences can help you when you go through similar
experiences.
(E) When you read and study literature, you learn about many dilemmas
in life.
Body paragraph 3:

Finally, all students should take literature classes because they will
help them develop a love of reading.
This is unfortunate because they are missing a great pleasure in
life.
Today, fewer young people read than before.
For example, I was very lucky that my parents encouraged me to
read when I was young.
That is not the case for many of my peers.
I think that they would find enjoyment in reading.
If they had been required to take a literature course, I think they
would change their attitudes.
Conclusion:

(A) In conclusion, university students would benefit by being required


to take literature courses.
(B) This would help students learn about other cultures, become more
mature, and develop a passion for reading.

'

'

\"..\'':--1,,Ii';
f; ;I

1.

11

534
-

Chapter 6

~Step

fl

Look back at the ideas you wrote in the blanks for Step 1. Write your own response to the
prompt using one of your own ideas or another idea from Step 1.

----------------------------------------------~--------------------------.

--------------~-----------------------------------------------------------

--------------------------------------------------------------~-----------

,,

.,,

,.

Making ideas Flow

535

-~-

,.j'

Opinion
"-l'Xm~<Ri:<!<!IW?l!iV>~~i.~F.VR-'nmw.:~&<:~.'i"~-o:i:~'-'-'-'''"'"'

~Step

~!':-1--.

, ..,'"

.-,

--~

Read the prompt and think of ideas to list in the blanks.


Some people like to plan everything carefully in their lives. Others would rather live each day
without a strict plan. Which do you prefer and why? Give specific reasons and examples to
support your answer.

''

.,i

Careful planning:

Living without a plan:

- allows you to set goals


- makes you feel more secure
- reduces the chance of having a problem

- lets you adapt to changes


- lets you try new things
- makes life more interesting

'------------------~ Step~2

''

Read the sample response. Think about the role of the sentences in each part of the essay.
Look for any transitions that link the ideas and underline them. Then put the sentences in the
right order.

,\,

--------------Introduction:

(A) This is because I think it is better to have a schedule that is flexible


to change and open to new opportunities.
(B) Some people like to plan everything carefully in their lives, but I
prefer to live each day without a strict plan.
Body paragraph 1:

(A) The main reason I like to live without a strict plan is that it allows
me to be more flexible to change.
(B) For instance, sometimes I intend to go out with my friends after
school, but then I get stuck staying late at school working with a
tutor.
(C) I have to change to deal with circumstances like that.
(D) I, on the other hand, do not let things like that bother me.

fti:-.- f'll

li cl

536

Chapter 6

flexible:
easily changed

open:
willing to accept changes

strict:
rigid; setting a clear
expectation
tutor:
a person that helps you
understand schoolwork
circumstances:
factors that inftuence an
event
hectic:
very busy and unorganized
unforeseen:
unexpected
editor:
a person who determines
the final draft of a text

(E) People today live hectic lives and face all kinds of changes.
(F) People that carefully plan too much get stressed out by these
unforeseen circumstances.

.
:..

.,,
~z
VI

Body paragraph 2:

C\

(A) If I carefully planned all my days, I would miss out on interesting


things.
(8) If I had had a carefully planned schedule, I would have never been
open to that opportunity.
(C) Another reason I do not plan too much is that it allows me to take
advantage of unanticipated opportunities.
(0) I helped her out, and she gave me a job as the photographer for
my school paper.
(E) Then, I ran into the editor of our school paper who needed some
photographs right away.
(F) For example, I stayed late after school one day with nothing to do.
Conclusion:
,------

"

(A) Not planning too much has allowed me to deal with change in a
positive way and to remain open to new opportunities.
(8) To conclude, there are people in this world who like to live by a
strict schedule, but I am not one of them.

IVld~1ng Ideas Flow

537 ----'"

i I

..

;
''

1;

~Step

Look back at the ideas you wrote in the blanks for Step 1. Write your own response to the
prompt using one of your own ideas or another idea from Step 1.
i<

-----------~----------------------------------------~---------------------

...__ 538

Chapter 6

- - - - - - - - - - - - - - - - - - - - - - - ---
' - ---~--~-~--~~0--~-~--~----- - - - ------~-~--~---'- - - - - - - - - -

OpirBion

ltlU!!i."l<Yr:<.r~."'-'i'~-<-~.k<;.,,.;.:.!';';t."iP-'':i'~;,;,~~'-'--"'-"''\\"'-"-h,,'J_, _,,,.: _,,,_<J,,'>

;\-;: ;.-'':: ,--

Read the following prompt.


Some people prefer to be the leader in a group. Others would rather follow the lead of another
person. Which do you prefer and why? Give specific reasons and examples to support your
answer.

r
"',,
~z
Gl

. Now look at the outline of a possible response to the prompt. First, identify which side of the
argument the responses support. Then write one more idea of your own that supports that
side of the argument.

1. Leading gives me the opportunity to make the decisions I think are best
I find it good practice for leading in the future.

3.
~Step

Read the sample introduction below. Then, in the space provided, write body paragraphs for
two of the ideas above. Try to write three to five sentences for each body paragraph. Then
read the sample conclusion paragraph.
Introduction:

For a group to get things done in an efficient and organized manner, it is often essential that they
have a leader. In these situations, I prefer to act as the leader rather than one of the many followers in
the group.
Body 1:

Making Ideas Flow

539 ---

[llllllllll!ll!lli!!!!!f!!!!!!!!!!l!llll!!!!!!!!!!!!!!!!l!lllllllllll!!lllllll!!lllllll!!lllllll!!!!!ll!!lllllll!!l!!ll!!!ll!!!!!ll!!!llllJlll!lll!llll!lll
...............................
-.-~iiilll:
~

._;_~

--

Body2:

Conclusion:

For these reasons, I prefer to assume a leadership position whenever I am given the opportunity.

*'
Write your own response to the prompt in Step 1 from the opposite side of the argument. First,
think of two or three ideas for body paragraphs. Then write a response using your ideas.

rj""f~--j' l

540

Chapter 6

_I

Aj____

- - - - -- - - - - -

l
I

[z

"'

Now read the sample response below. What similarities and differences do you see with the
paragraphs you wrote?

'~

'

~Sample Response
For a group to get things done iri an efficient and organized manner,
it is often essential that they have a leader. In these situations, I prefer to
act as the leader rather than one of the many followers in the group.
One reason I like to lead is that leading gives me the opportunity to
make the decisions I think are the best. For example, in the summers I
often go on backpacking trips with large groups of my friends. I prefer
to lead because that way, I get to make the final decision as to where
we go, which peaks we climb, and how we get there.
Another reason I prefer to be the leader of a group is that I find it
good practice for leading in the future. Almost every career requires
some kind of leadership skills. Even if you start out at the bottom of a
corporation, you eventually work your way up the business hierarchy
dnto some kind of management position. I think it is important to develop
'"the skills it takes to be a good leader as soon as possible.
Lastly, I prefer to lead because as a leader, I get more respect than
as a follower. Because I am responsible for the safety and success of an
entire group, the members of that group tend to look up to me and listen
to what I say. When one of my backpacking groups ran into a huge
snowstorm one year, nobody questioned my decision to turn back. In
fact, niany of them thanked me for keeping them out of danger.
For these reasons, I prefer to assume a leadership position whenever
I am given the opportunity.

[
[

essential:
necessary
follower:

a person who lets other


people lead a group
hierarchy:

a system of organizing
people
respect:

to regard someone else's


choices or actions highly
look up to:

to admire someone
question:
to have doubts about
something
turn back:

to return the way that you

came
assume:
to start to control
something

ivlar:in9 :ueas Flow

541 - -

Opinion
=1;:;~''.';ll~;,-,,,\\t,;1-(,'J>?i.'(;hi<l>.<-

~Step

,; ; '"'-'

~,,,

,. ;.,

Read the following prompt.


Is it better to get advice from a parent or from a friend? Give specific reasons and examples
to support your answer.
Now look at the outline of a possible response to the prompt. Then write the thesis and the
conclusion.

Key point 1: Parents can be more knowledgeable than friends.


Key point 2: Parents do not give bad advice.
Key point 3: Parents will recommend what is the best for you.
Conclusion:----------------------------~Step

:2

Now write an introduction and a conclusion for the prompt in Step 1. Use your thesis and
conclusion from Step 1. Try to write three to five sentences for each paragraph.

Introduction:

~.. ,.---

rr1
':

542

Chapter G

(: , I

'4~---------~---

:
r :'
l
-.:

'

C\

.
'

~Step

Write your own response to the prompt in Step 1. First, think of two or three ideas for body
paragraphs. Then write a response using your ideas.
Body Paragraph 1: _ _ _ _ _ _ _ _ _ _ _ _ _ _ _ _ _ _ _ _ _ __

~----------------------~--------------------------------------------------

Body Paragraph 2: _ _ _ _ _ _ _ _ _ _ _ _ _ _ _ _ _ _ _ _ _ __
--------~-------------------------------------~-----------------------~---

------------------------------------------------~-------------------------

Body Paragraph 3=~-----------------------

---------------------------------------------------'

Making 1c1eas Flow

543

I.

,:
,.-------- 544
... ;;
.

~Step

Now read the sample response below. What similarities and differences do you see with the
paragraphs you wrote?

When you are stuck in a tough situation and you do not know what
to do about it, it is only natural to seek advice. In most situations, I think
it is better to go to a parent for advice than to a friend.
One reason why I hold this belief is because your parents are often
more knowledgeable than your friends. Your parents have already gone
through many of the situations you are going through. In hindsight, they
often have figured out which solutions work and which do not. Therefore,
they may be able to answer many of the questions and concerns your
friends will not be able to answer.
Also, I think it is better to ask your parents for guidance because they
are less likely to give you bad advice. Your friends may be preoccupied
with things like acting cool or wanting to be accepted. They may advise
you to do things like spend your money on clothes when it may not
actually be in your best interest to do so. Your parents, on the other
hand, are more removed from the immediate situation and thus are
better able to see clearly which solutions are best.
Lastly, I think you should ask your parents for advice rather than your
friends because your parents will always support you. While your friends
may turn their backs on you or even desert you, your parents will always
be there to help you. Most of the time, your parents will spend as much
time with you as is needed to solve your problem.
In conclusion, I think that you should almost always go to your parents
for advice rather than seeking advice from a friend.
'----

Chi-lpter 6

stuck:
unable to solve a problem

natural:
reasonable in a given
situation

seek:
to look for something
hindsight:
the ability to look back on
past events and judge them
preoccupied:
thinking about other things

removed:
to be separate from
something
turn one's back:
to stop helping somecne
desert:
to leave someone in a
situation where they need
help

-----------

,,

Opinion
IM>Mn'Wc;w.i:u>J':,.T,.l:;\:l"itlW<<.':>~<'i'.~~=-""~">llJ!il~:;)H<\~Y.~t:,~P~~"'-'"\<

,,,,,., _, >

Read the following prompt.


Do you agree or disagree with the following statement? It is important for people to learn
the customs of a country when they visit. Give specific reasons and examples to support
your answer.
Now look at the outline of a possible response to the prompt. First, identify which side of
the argument the responses support. Then write one more idea of your own that supports
that side of the argument.

I agree that it is important to learn the customs of a country when people visit.
Key point 1: You will learn about behavior to avoid that may be offensive.
Key point 2: You will be better able to communicate with people.
Key point3:

~Step

Read the sample introduction below. Then, in the space provided, write body paragraphs for
two of the ideas above. Try to write three to five sentences for each body paragraph. Then
read the sample conclusion paragraph.
Introduction:

When you visit another country, you often witness a variety of customs much different from your
own. I agree that it is important for people to learn these customs when they travel to a foreign country.
Body 1:

Making Ideas F!ow

545 - -

' .

Body 2:

. 1\:

Conclusion:

In conclusion, I think that it is essential to learn the customs of a country you are planning to visit.

~Step

Write your own response to the prompt in Step 1 from the opposite side of the argument. First,
think of two or three ideas for body paragraphs. Then write a response using your ideas.

,,
<

'

------------~--------------------------~-----------------------------------

--------------------------------------------------------------------------

:\."

------------------

{ - - 546 Chapter 6
i --.--'-~--~'o-

----

_, ___ ,~---~- - --' '--

I""

Now read the sample response below. What similarities and differences do you see with the
paragraphs you wrote?

~z

"'

Sample Response
When you visit another country, you often witness a variety of customs
much different from your own. I agree that it is important for people to
learn theses customs when they travel to a foreign country.
First of all, learning a country's customs helps you avoid embarrassing
situations or miscommunications. In many African countries, for example,
it is considered very rude to touch a person or eat with your left hand.
Ignorance of such a custom would certainly cause you to offend many
of the people you meet. On the other hand, if you learn a country's customs,
you will be able to get along with its people quite well.
Secondly, knowing the customs of a country you are visiting shows
locals that you are genuinely interested in their country and that you
(\respect its people. In the days of colonialism, many colonists blatantly
, disregarded the customs of the locals and therefore angered many of
~,them. When you visit another country, it is important to remember that
, you are a guest of the people who live there. As such, you should
respect their customs, and in order to do so, you must learn them.
Finally, it is important to learn a country's customs because it allows
you to communicate with people. Often, people use certain gestures or
greet each other in a unique way. Knowing what these gestures and
greetings mean and what they are will make it much easier to meet
people and get any information you might need.
In conclusion, I think that it is essential to learn the customs of a
country you are planning to visit.

witness:.
to see something happen

custom:
something that people in a
particular culture usually do
embarrassing:
making you feel
self-conscious or silly
miscommunication:
a misunderstanding
offend:
to make a person upset
because of something that
you do or say
local:
a person who lives in a
particular place
blatantly:
in an obvious way

gesture:
a movement of one's hands
or head to communicate a

message

Making Ideas Flow

-----

547 - - -

Instructions: Choose the best word or phrase


to complete each sentence.

1. Some people are more _ _ _ __


when it's quiet, while others work better
with noise in the background.
lik,'

f''II",

I'

'i
11' ..

'"'
nn
'i~

''

(A)
(B)
(C)
(D)

abundant
productive
linear
traditional

2. Emily had to

the fact that


she didn't study enough for the test and
got a poor grade.

(A)
(B)
(C)
(D)

face
eradicate
post
refuse

3. I found the movie extremely


_ _ _ _ _ . I can't believe that it got
such good reviews.
(A)
(8)
(C)
{D)

adequate
significant
engaging
dull

4. The company did not have enough


_ _ _ _ _ to get the job finished, so
it hired more people.
(A)
(8)
(C)
(D)

programming
access
manpower
signal

5. The committee has been busy planning the


_ _ _ _ _ picnic for former_students
of the university.
(A)
(8)
(C)
(D)

resources
alumni
challenges
posters

';V--- 548 .Vocabulary Review 3

''i

6. Stuart is a very
young
man, so he should have no trouble finding
a job.
(A)
(B)
(C)
(D)

unstable
scarce
well-rounded
distracted

7. Some people do not understand the


importance of recycling
efforts to save the environment.
(A)
(B)
(C)
(D)

in

waste
signals
pride
globes

-~.

8. The company's

of a large
sum of money allowed the new park to be
built.

(A)
(B)
(C)
(D)

impression
contribution
tradition
association

'i;,

9. I would like to go to the movies.


_ _ _ _ _ , I could just watch
television at home and save money.
{A)
(B)
(C)
(D)

Hardly
Primarily
Swiftly
Conversely

'',~
'

10. I decided I needed a


, so I
signed up to run a 15-kilometer marathon
next month.
(A)
(B)
(C)
(D)

crisis
challenge .
forum
program

,,
''i

~
~:

]
..

11. The library has many


to help
students who are researching topics for
reports.
(A) finances
(B) impressions
(C) portrayals
(D) resources

12. George is

to building a
treehouse for his nephews in his sister's
backyard.

(A)
(B)
(C)
(D)

ensured
distracted
dedicated
associated

13. The new Italian restaurant is going to


_____ its menu on its website so
. people can see it.
(A)
, (B)
' ' (C)
(D)

vaccinate
pacify
refuse
post

14. I have never been good at math, so I


_____ with it the whole time I was
at school.
(A)
(B)
(C)
(D)

struggled
dropped
concentrated
retained

15. Everyone who works with Charlotte is


amazed at how good she is at
(A)
(B)
(C)
(D)

poverty
popularity
multitasking
access

Instructions: Choose the word or phrase closest


in meaning to the underlined word.

. 16. I was so interested in reading my new book


that I didn't realize how late it was.
(A) valued
(B) engaged
(C) unstable
(D) linear
17. In order for antiques to keep their value,
they must be treated very carefully.
(A) retain
(B) ensure
(C) refuse
(D) portray

''
'
.

'I

I'
.
'

'

18. It is typical in some countries to put out


milk and cookies for Santa Claus on
Christmas Eve.
(A) productive
(B) dedicated
(C) scarce
(D) traditional
19. Cleopatra is usually described as very
beautiful, though some historians doubt this
is true.
(A) portrayed
(B) associated
(C) eradicated
(D) replicated
20. She felt a lot of satisfaction when she found
out that she had achieved a perfect score
on her exam.
(A)
(B)
(C)
(0)

cns1s
value
pride
signal

Vocabulary.Review 3

549 --.'

'

21. They were pleased to discover they could

l'

l'I

26. My first thought upon meeting him was that

QW for a trip to Europe this summer for the

he wasn't very friendly at all.

entire family.
(A) pacify
(B) finance
(C) refuse
(D) postpone

(A)
(B)
(C)
(D)

impression
signal
concentration
significance

27. It took my mother a while to understand

\i"

22. The flight had been overbooked, so there


was not enough seating on the plane for
everyone.

ij

(A)
(B)
(C)
(D)

'I

.I
:1

sc<irce
growing
disadvantaged
adequate

(A)
(B)
(C)
(D)

catch on to
take on
run into
drop in on

28. In baseball, the pitcher and catcher have

I
II

23. He decided to stop his piano lessons


because he didn't like them as much as he
thought he would.

(A)
(8)
(C)
(D)

I
lI

'I

'

drop
vaccinate
replicate
ensure

signs for what kind of pitch should be


thrown.
(A)
(B)
(C)
(D)

disadvantages
fields
signals
providers

29. The parade had to be delayed because it

"'

24. Several generations of her family have


worked in the area of medicine, so she
wants to be a lawyer instead.
(A)
(B)
(C)
(D)

forum
signal
field
program

25. The boss left a note on Shannon's desk,


but she didn't see it so she missed the
meeting.
(A)
(B)
(C)
(D)

:~---

how to use the Internet, but now she loves


it.

550

production
crisis
resource
memo

Vocabulary Revie>iv 3

'

:{'

was raining so hard on Saturday afternoon.


(A)
(B)
(C)
(D)

pacified
postponed
refused
produced

30. In past centuries, many people kept cats in


order to get rid of mice <ind rats in their
homes.

(A)
(8)
(C)
(D)

associate
ensure
replicate
eradicate

-~

I
I

f
[

;'.~;nstructions: Write the missing words. Use the


words below to fill in the blanks.
ensure
run into
access
abundance
scarce

generate
preferred
crisis
significance .:
globe

There is a lot of 31. _ _ _ _ _ __


to the rising costs of power around the
32.
. It shows that
_______ as our
non-renewable sources of energy such
as coal and oil are becoming more and
more 34.
. The Earth
simply cannot 35.
enough
resources to meet the world's energy needs.
However, using renewable power sources like
wind and solar can 36. - - - - - - that we do not 37.
energy
problems. More countries are gaining
38.
to renewable power
sources in the form of wind farms and
solar power plants. Unlike non-renewable
sources, there will always be an
39..
of sunlight and wind.
In fact, renewable energy is quickly becoming
the 40.
method of power
for many people and governments.

Instructions: Choose one word in each group


that does not belong.

41. give

provide

design
42. pacify

supply

value

43. unbalanced
uneven
44. focus
disadvantage
45. copy
portray

calm
peace

..

unproductive
attention
concentration
replicate
repeat

46.

poverty

wealthy

47.
48.

refuse

reject

associate
distracted
design

connect
focused
plan

50.

.i

unstable

.Instructions: Label each pair of words as similar


(S) or opposite (0).

49.

[
[

..

Vocabulary Review 3

;.

551 _

..

"""'!!!!!!!!!!!!!!!!!!!!!1!!!1!1!!!!!!!!!!!11111!1111111111111111)11!11~1111111~-, __ ,_ :.~ ... __ ,,, ... ....................llllllll!I~.....~.-.;~

_!!!!!_.. - - - - - - - - - - --- ------ -~~"'---------

~-~'U'itl<W<J'>lli!';t..-<"m!O',::---

.-

Instructions: Choose the best word or phrase


to complete each sentence.

1. I'm

on this chemistry
exercise and I just can't figure it out.
(A) natural
(B) strict
(C) wise
(D) stuck

2. He says he has always

6. Many explorers have gone out to


the Fountain of Youth, but
none have found it.
(A) seek
(B) desert
(C) connect
(D) devote

'
g.,
)
-,}

"
i

'J

.,j

r~

his

older brother and grandfather as role


models.
(A) offended
(B) tended
(C) looked up to
(D) turned back

7. Cais and dogs


to fight,
but sometimes they can also become
friends.
(A) offend
(B) assume
(C) tend
(D) refuse

. (

'
. 'i
'

~
j

l
.1'
f>t

;j'ji

8. My best friend is always reading about her

3. She was late because she


a car accident and had to fill out a report
about what happened.
(A) witnesses
(B) deserted
(C) connected
(D) devoted

favorite
magazines.

in entertainment

'!

*,r,

.~

(A) celebrity
(B) alumni
(C) conflict
(D) gesture

fl
;~
(,

9. It is often said that younger people should

i~

their elders and speak to

4. The academic support center has


that can help you out in
any subject.
(A) locals
(B) tutors
(C) editors
(D) celebrities

(A)
(B)
(C)
(D)

l
t'i
~i ~--:

552

for a successful future.


flexible
removed
essential
hectic

Vocabulary Review 4

(A) ensure
(B) desert
(C) respect
(D) pacify

;p;

f
~
:~

~
}(',

5. My parents both believe a good education


is

them politely.

1O. They aren't very


when it
comes to making financial decisions, which
is why they have no money.
(A) abundant
(B) wise
(C) accessible
(D) passionate

' >"il'

~.

~
~

'.<\

l~
!'i!,.,,

~{;

~-i.
z
~

Cl

'

11. Everyone says that you should always be


honest, but sometimes being too honest
_ _ _ _ _ people.
(A)
(B)
(C)
(D)

vaccinates
drops
respects
offends

12. The professor is very _ _ _ __


about students attending class every week.
(A) flexible
(B) strict
(C) specialized
(D) unstable
13. She has a very strong
for
helping animals, so she joined an animal
rights group.
(A)
(B)
(C)
(D)

forum
dilemma
passion
field

14. Personal computers have become a lot


more
since they were first
marketed.

(A)
(B)
(C)
(D)

natural
open
embarrassing
accessible

15. The media seem to create more

- - - - - news stories instead of


just reporting the facts.
(A)
(B)
(C)
(D)

financial
sensational
devotional
digital

Instructions: Choose the word or phrase closest


in meaning to the underlined word.

VI

Cl

"O

16. I really have to doubt whether he really


wants to be a part of this team due to his
attitude.
(A)
(B)
(C)
(D)

!'

~z

'

'

Cl

associate
struggle
assume
question

q
;:;

17. Karen is facing the problem of not knowing

"'-I

which university she wants to attend.


(A) dilemma
(B) gesture
(C) hindsight
(D) resource

'
'

.I

18. Everyone has had uncomfortable moments


in life when they just want to disappear.
(A)
(B)
(C)
(D)

flexible
embarrassing
essential
informative

19. The wild strawberries at my uncle's farm


were so plentiful last summer that we ate
them for dessert nearly every night.
(A)
(B)
(C)
(D)

suburban
linear
abundant
sensational

20. His opinion toward life is really positive,


which makes him really fun to be around.
(A) attitude
(B) passion
(C) popularity
(D) pride

Vocabulary Review 4

553

~-----;

21. Between work and school, my schedule


has gotten pretty busy lately.
(A) unforeseen
(B) hectic
(Cl removed
(D) specialized
22. Sometimes events happen that keep us
from doing the things we really want to do.
(A) conflicts
(B) connections
(C) customs
(D) circumstances
23. He made a movement that meant he
wanted me to follow him into the sporting
goods store.
(A) hierarchy
(B) miscommunication
(C) gesture
(D) follower
24. Due to an unexpected snowstorm, school
was cancelled for the day which made the
students happy.
(A) unforeseen
(B) accessible
(C) informative
(D) adequate
25. I have a lot of admiration for people who
have overcome difficult obstacles in their
lives.
(A) flexibility
(B) respect
(C) preoccupation
(D) dedication

i!---- 554
~;., .;., :.; .,o<:, ...;j"'"';,;.;,~.,;, ;;,;c:;,_.,;;,,;,;
Vocabulary Review 4

c.;;,;,::;:;,; c .:.;,,:::c;;;;,,,:;:;c;;;,;;;,;:c;_,;;;;;;;;,,,,, ,;;.

26. They had a disagreement about how the


project should be completed, but they
worked it out.
(A) preference .
(B) memo
(C) passion
(D) conflict

27. My schedule is pretty changeable, so just


let me know when you want to get together
for lunch.
(A)
(B)
(C)
(D)

removed
open
flexible
informative

28. In medical school, students take a lot of


focused courses relating to the field of
medicine they are interested in.
(A) traditional
(B) resourceful
(C) scarce
(D) specialized
29. It is reasonable to be frightened when
watching scary movies on television late at
night.
(A) stuck
(B) natural
(C) strict
(D) hectic
30. I don't know why she bothered even to
come with us when she obviously didn't
want to.
(A) adequately
.(B) conversely
(C) blatantly
(D) digitally

!(

~' Instructions:

Write the missing words. Use the


words below to fill in the blanks.
connected
turn back
hectic
devote
hindsight

desert
open
removed
attitude
preoccupied

'Ji'\'': In today's busy society, it is easy to become


'!:f,':'~;,:_t:

~i\i'Nt 31..

with our own lives and


from the lives
. j!iib of our friends. However, it is very important to
..
.. ~
~i~F'. stay 33.
with them and be
' :,;!fj 34.
to making changes in
.?~r;;~::.
, order to do so. Although our lives may be
3' extremely 35.
, it is not
<>
.. acceptable to 36.
one's
,-::;-,
friends. If you have the 37. _ _ _ _ _ __
that you are too busy for them, there will be a
da~ when they will be too busy for you when
you need help. In 38. _ _ _ _ _ __
you will wish you had not ignored them, but
it will be too late to 39. _ _ _ _ _ __
and change things. So make sure you
40.
time to your friends.
if;{.~-~:;

, :t~!;,!'{
;,w,,- become 32.

Instructions: Write the missing word. Use the


words below to fill in the blanks.

off

out

off

so

up

41. The game was called


and
rescheduled because of the bad weather.
42. I called to ask if you want to go
_ _ _ _ _ tonight. We could see a
movie.
43. She was _ _ _ _ _ upset that she
failed her midterm exam.
44. We did not buy the sofa because we
thought it would take
too
much room.
45. People often think everyone else is better
_ _ _ _ _ than they are.

11

/,j

Instructions: Match the words that are


opposites.

46.
47.
48.
49.
50.

removed

(A) relaxed

strict

(B) leader

unforeseen

(C) attaohed

follower

(0) agreement

conflict

(E) expected

Vocabulary Review 4

'

555 _ _:1
I

Tips
A clear and understandable essay should include the following:
Coherent organization
A broad range of grammar and sentence structures to avoid monotony
Transitional expressions to clarify organization and flow
Appropriate and precise vocabulary

Strategies
~!iS.illi>-ViW'iffiR{h'i".:>~\W:<\~\ oc.;~,,

c,;"JYi,"Ul;-1'1'.0<'~>~"-n";;,,

;,-_.;,.;;,;1. : ,.

:,, . - .

When writing an essay:


Check for errors in tense in all clauses of a sentence
Use appropriate modal verbs
Use the present participle (-ing) and the past participle (-ed) correctly
Avoid sentence fragments and run-ans
Use noun phrases and noun clauses correctly
Ensure sentences are connected by appropriate conjunctions and adverbs

Verb Forms

559

-~--,

Verb Tense
When writing a TOEFL essay, test takers should be aware of the following:
The summary of a lecture or reading must be written in present tense.
Example: The .author suggests that congestion pricing helps control traffic.
The past perfect tense is used to describe an action that occurred before another specific moment
in the past.
Example: I had completed the assignment when the professor announced the extension.
The subjunctive mood is used in that-clauses following these verbs and expressions:
Verbs - suggest, recommend, insist, propose, advise, etc.
Expressions - it is important/essential/necessary/vital/critical, etc.
Example: The professor suggested that the student revise his paper.
It is essential that a student uses precise vocabulary when writing essays.

In adverbial clauses and first conditionals, the present tense is used to signify probable future
action.
Example: Before she asks the librarian, she will ask the man to lend her the book.
If it rains, I will bring an umbrella.

In second conditionals, the past tense is used to signify improbable future action.
Example: If I won the lottery, I would donate some of the money.

.
-"'

:------- 560

Clrnpter I,

[
~Each of the following paragraphs has eight errors in verb forms. Find the errors and correct
them.
I believing that universities should pay their teachers more money rather than spend so much on
athletics because teachers should are encouraged to do a really good job of teaching. In my opinion,

c.:
I

Cl

people feeling like they are more valuable in their jobs when they get paying more money. However,
many teachers are not paid enough money. This can makes them feel like their hard work be not
valued by the university, so they might not do as good a job. If they are paid more, they will doing a
better job of teaching, and students will learning more.

2.

Both the reading and the lecture discussing whether parents should interfering in children's
playtime. The author stated that parents should have interfered in children's playtime because
children have learned to stand up for themselves and need positive adult role models. The lecture,
on the other hand, disagree by stating that parents should not to interfere when children played.

~Exercise

,,

Write the correct form of each verb.


To (1) ____ (begin), the author of the passage states that congestion pricing can cause

1.

more people to use public transportation. However, the speaker in the lecture (2} _ _ _ _ _ __
(disagree), stating that congestion pricing will not necessarily encourage more people to use public
transportation. Next, the author says that (3)

(implement) congestion pricing

will reduce commuting times for everyone. On the other hand, the speaker suggests that congesting
pricing might (4)

(make) commuting times longer for a lot of people. Last, the author

(state) that congestion pricing will benefit everyone, but the lecture states that not

(5)

everyone will benefit.

2.

Both the reading and the lecture (1)


According to the lecture, corntainers (2)
they can be (3)

(discuss) whether corntainers are beneficial.


(be) much better for the environment because

(compost), they will not take up space in landfills, and they

are more socially responsible than regular plastics. However, the speaker in the lecture disagrees.
The speaker states that corntainers do not (4)
The author (5)

(solve) these environmental concerns.

(refute) the claims of the reading in three ways.

Verb Forms

561 - - . '

Modal Verbs
Modal verbs are used to add specific nuances of meaning to the verbs that follow them. The
modal verbs that test takers often struggle with are as follows:
can

1--t_o_e_x_p_re__ss_th_a_ts_o_m_e_th_in_g_is_p_o_ss_ib_le_o_r_im_po_s_s_ib_le___________-J_
Corntainers can be beneficial for the environment.

may

__]

to express something that could happen or be true


Congestion pricing may cause more people to use buses.

[:m
could
might

must

to express certainty and to predict (more certain than may)


Students will soon do all of their schoolwork on computers.

--------

less positive versions of can and may

- j
---------._-=3----------------------Teachers might/could look for new jobs if they are not paid enough.
to conclude something from logical thinking
She has an umbrella. It JI!!Jfil be raining.

------------~

should

J
-fillill!JQ-u-ld..:gc;.o_t_o_~h~e~r-e~g_i-st_!_a_t_i-o-~--~-ffi-ce_f_o_r-i_n-f_o~rm~"-a_t-i_o-n~.~~~=====~ J

to give advice or sugg~;t-th-at-itis-helPl~t--t_o_d_o_s_o_m_e_th-in_g____


1---Th_e_s_t-ud_e_n_t

-~-au-v:_\_0-~-;o_m_:x-;-~~-:-:e_n_::_:_:-::-~-~;=-r~=-~~-i~;-~:~-:-~-r~-~~-et:t:~::ua_a~-:-:c_e_____-_'---~--~~1

---------------------
---

EG

--------------

to express preference in conditional sentences


.

I would rather go to the concert than watch TV.


----
.
--------~-~-

-----~--=-.:

1
l

------------'"~...;

In order to avoid using the same modal verb repeatedly, it is a good idea to use other equivalent
expressions.
Example:

; - - 562 Chapter 7

The environment may be polluted.


The environment will probably be polluted.
It is likely that the environment will be polluted.

;.,-..:;

~-

-: .

~ Exerdse 1
e

Change each of the following sentences using one of the modal verbs below. Each modal verb
may be used more than once.
would

can

may

could

will

1. To many adults who work very hard, early retirement seems appealing.
2. Many adults find that early retirement is not a satisfying experience.

3. Many people who retire early face financial problems in the future.
4. Early retirees find themselves in debt or face a lower standard of living.

5.. .People who retire early also struggle with psychological problems.

' s: Early retirees feel that their lives no longer have a clear purpose.
7. Early retirees often face social problems.

8. They have trouble relating to the pegple that they used to work with.

~ E}{en:i:se

Fill in the blanks with the most appropriate modal verb.

1.
t

Although I think that the amount that we use computers (1)


the next twenty years, I do not agree with the statement that students (2)

(musVmay) increase in
(have to/will)

do all of their schoolwork on computers in twenty years. In my opinion, computers (3) _ _ __


(could/must) become much more inexpensive before that idea (4)

(would/may) be

possible. There are three reasons why I feel that we (5) _____ (should/would) not believe that
students will do all schoolwork on computers in twenty years.

Verb_Form~

563

---j
!

-- - _;;c

,;

'~"

&
,~.

Present Participle vs. Past Participle

,,%

The present participle (-ing) and the past participle (-ed) are used basically in three ways: as
verbs to indicate tense, as verbs to indicate voice, and as adjectives. Look at the table to see
how they are used.

,ij
h.,

"-'

j;

"'~!~

Past participle

Present participle

;i-,,

';,

tense

"

'

voice

Used in continuous tenses after the verb be Used in perfect tenses after the verb have
She has done nice things for me.
He is using my laptop.
She had left before I arrived.
He has been using my laptop.
Used in the passive after the verb be
The problem was solved.
The problem can be solved.

Used with be to indicate tense in both


active and passive
They are building a school. (Active)
The school is being built. (Passive)

~-~
-~

1;
!'.

~
~~

~~
];

'J

,I

adjective

Used to describe something or


someone that receives feelings
I was annoyed.
He is interested in architecture.

Used to describe something/


someone that produces feelings
His attitude was annoying.
He was interesting.

'

'~

Common mistakes related to the participles are as follows:


He was studied English. (X) -> He studied English.
The book will published. (X) -> The book will be published.
The game was excited. (X) -> The game was exciting.

Write the correct form of each verb.


finish

multitask

work

turn

be

I believe that I am able to work quite a bit faster when I am (1) _ _ _ _ _ _ __

1.

For example, when I was (2)

on my homework last weekend,

I made sure to work on a few different assignments at a time. That way, my work was
(3) _ _ _ _ _ _ _ _ in just a short amount of time. However, if I had

:.:;,.----- 564

"

(4)

working on only one homework assignment, I would not have

(5)

in all of my work by the time I had to have it finished.

Chapter 7

-:~

/):
-~:

._..

" .

motivate

s
e

. give . . tell

be

solve

[
It

In my opinion, a person's problems can best be (1) - - - - - - - - - if he or she


goes to a parent for advice. My parents have been (2)
me good
advice for my entire life. I think that my friends are really nice people, but they do not always give me
great advice. My parents' advice, though, is always helpful and (3) _ _ _ _ _ _ _ __
In addition, my parents have never (4)
of my friends did. Last, since my parents have always (5)
my problems, I know they are a reliable source of help.

~Exercise

,'..->

other people about my problems, while one


there to help me with

Write either the present participle (-ing) or the past participle (-ed) in the blanks.

1. Supporters have been (praise) _________ the benefits of biofuels for many years.

2f However, many of the benefits of biofuels are (exaggerate) _ _ _ _ _ _ _ __

3t Many of the problems associated with biofuels cannot be (solve) _ _ _ _ _ _ __


4. Even more pollution is (generate) _ _ _ _ _ _ _ _ _ _ by biofuels than by fossil fuels.
5. Some scientists are (develop) - - - - - - - - - - n e w ways to make biofuels.
6. Scientists have (suggest) _ _ _ _ _ _ _ _ _ _ that biofuels do not produce enough
energy.
7. Studies have (find) ________ that we do not have enough land to use only biofuels.
8. Most of the world's agricultural land must be (use) _ _ _ _ _ _ _ _ for growing food.
9. Hydrogen power has been (discuss) - - - - - - - - - - a s an alternative to biofuels.
10. Hydrogen fuel may be ( u s e ) - - - - - - - - - in the future, but it is not yet available.

Verb Forms

565 --~----:-"':

-- -- -- ---~-!!!!!!!!!!I!!!.!!!!!!___- - - -- -----11111-_11111111_.
___ lllllllll
_____ 1111111
___ _ ........iiiiiiiiiiiiiiiiiiiiiiiiiiiiiiiiiiiiiiiiiiiiiiiiiiiiiiiiiiiill~

Tips
A clear and understandable essay should include the following:
Transitional expressions to clarify the flow of ideas
Appropriate vocabulary to express ideas precisely
A range of grammar and structures

Strategies
When writing an essay:
Avoid sentence fragments and run-ans
Check that noun phrases and noun clauses are used correctly
Make sure sentences are connected using the appropriate conjunctions and
adverbs

.l

I
'
'

']
1

Sentence Formation

567 - - : ,
I
I

--=-=1-~

Noun Clauses '


A noun clause includes a subject, object, and/or a complement. Noun clauses are connected to a
sentence by conjunctions: question words (who, what, how, etc.), whatever, whoever, whether, if, that,
the fact/idea/belief that, etc.

~,,ii

1iH'.\~.I '

These are mistakes commonly made in relation to noun clauses:

Ill.I!':I. '

1) Subject and verb not in agreement

!6li1!

,..-,:;

Example: The fact that the lake was polluted by the chemicals are not widely known. (.... is)

:, !,

':\\:."--. 568

2) Tenses
Example: Most paleontologists now believe that dinosaurs are warm blooded. (.... were)
I suggest that the woman finds a new roommate. (.... find)
3) Subject-verb inversion
Example: I asked who were they.( .... they were)

iExen:be i

Underline the noun clause. Then write Cfor correct sentences and IC for incorrect sentences.

_ _ _ 1. It is important that children begin to use computers for their schoolwork.


_ _ _ 2. I believe that children will soon use computers for all of their schoolwork.
_ _ _ 3. The fact that children will do all their work on computers are obvious to me.
_ _ _ 4. I do not see how computers will be able to replace a pen and paper.
_ _ _ 5. Many people believe that everyone should learns to use a computer.
6. I explained how computers have help me do my schoolwork.
_ _ _ 7. Some people disagree about which ways do computers help people.

Chapter 8

[[
I[

Combine each pair of sentences into one sentence that includes a noun clause.

1. Congestion pricing is not as beneficial as it seems. This is suggested by the speaker.


It _ _ _ _ _ _ _ _ _ _ _ _ _ _ _ _~-----------~

2. Congestion pricing does not encourage people to use public transportation. It is true.
It _ _ _ _ _ _ _ _ _ _ _ _ _ _ _ _ _ _ _ _ _ _ _ _ _ _ _ _

3. Experts argue this: congestion pricing also punishes the people who are forced to drive.
What

is _ _ _ _ _ _ _ _ _ _ _ _ _ __

4. Congestion pricing will actually increase commute .times. This is the theory.

The

is ~-------------~

5, More people use the bus. Commute times may decrease.


<The fact that

may _ _ _ _ _ _ _ _ _ _ _ __

Sentence Formation

569

~--~
I

__liil_iiiiii_iii_iii_iii_iii
___iiliiil
___L --"
!!!!!!!!!!!!!!!!!!!!!!!!!!!!!!!!!!!!1111. .llll!lmlllll!ll....-lllllllllllJlll!lllll!lll!!lll!--...-illlll-llifli_llifli

'- --- -- _,________

----O--'--------- -

Commonly used subordinating conjunctions are as follows:


Time: before, after, when, while, as soon as, whenever
Cause: because, since, as
Contrast: although, even though, even if, while, whereas
Condition: if, unless
When a subordinating conjunction is used:
1) Avoid separating the dependent clause from the independent clause. Otherwise, it becomes a
sentence fragment.

Example: I was often late. Because I had to help my mother.


(independent)
(dependent)

<> I was often late because I had to help my mother.


2) Use a comma after the dependent clause when it begins the sentence.

Example: Because I had to help my mother I was often late.

\1

Because I had to help my mother, I was often late.

Fix the incorrect sentences by adding or removing a comma or by combining two sentences.

1. The Copper Scroll was discovered in a cave. In 1952.


2. Since this discovery scientists have analyzed and translated the scroll.

3. Now, scientists are trying to figure out if the treasure, described in the scroll is real.
4. After we find the Silver Scroll. We may be able to find the treasure.

5. Even though the scroll gave detailed directions the treasure could not be found.
6. We will not be able to say that the treasure is real, unless we find the treasure.

7. The treasure might exist. Although it would be much smaller than some believe.
8. We know that the treasure is not just a story. Because it uses very technical language.

.,

:f~~- 570 chapter 8

.:;,
;l:! ----_-,7-~_-----

-,---.---,-----------'--

_,-.-_;

[
Rewrite or combine the sentences using subordinating conjunctions.

'1.

It is better to get advice from parents. They will always give good advice. (because)

[ G'I

2. I always go to my parents. I need advice about a very important issue. (when)

3. My parents will give me advice about what is best for me. My friends might give me advice that is
not very good for me. (whereas)

4. ,Many people do not think friends give good advice. I have always received good advice from my

friends. (although)

5. Parents will give you better advice. You might think your friends know you better. (even

i~

6. Parents will be able to help you through bad experiences. They have gone through many of those
same experiences. (since)

7. Your friends might not always be there for you. Your parents will always support you. (while)

Sentence Formation

.---

_________ _:__.;_ __ _

571 -

.I

II
!

Parallel Structure
In order to make a sentence coherent and clear, it is important to use parallel structures in an
parts of the sentence. When words or phrases are connected, those words or phrases should
be parallel in terms of their form, tense, and parts of speech.

Forms: I prefer to watch lV to going to a concert. _. I prefer watching lV to going to a concert.


Tense: We clean the house and are cooking dinner together. _. We clean the house and cook
dinner together.
Parts of speech: She writes well and brilliant. _. She writes well and brilliantly.

~ Exerdse 1

Indicate whether the sentence parts display parallel structure {P) or not {NP).
_ _ _ 1.

To carefully plan is good for some people, but I prefer living without a strict plan.

_ _ _ 2.

I feel organized and relaxed when I have astrict plan.

_ _ _ 3.

Living without a plan is confusing and stress.

_ _ _ 4.

I like to plan when I do my homework, go to bed, and hang out with friends.

_ _ _ 5.

If I do not plan everything, I can change my schedule and go to the movies or hanging
out with friends.

_ _ _ 6.

It is good to plan so you can avoid scheduling problems and their consequences.

_ _ _ 7.

When I know I am going to be busy, I can tell my friends and avoids a scheduling problem.

_ _ _ 8.

When I do not plan everything, I am free to do new things and I did not have to cancel
plans.

_ _ _ 9.

I find that carefully planning everything makes you feel too busy, stressed, and frustrated.

_ _ _ 10. I do not carefully plan everything, yet I never feel like I do not know what I should be
doing.

: . .:.] . s12
'17

~ '!

chapters

'

~
c:

t.-:

II
d

, Underline each phrase that is not parallel to the rest of the sentence. Then change the phrase
to make it parallel.

1. Planning carefully is better than to live without a plan.

Cl

'.

''

Cl

2. It is important for me to plan when I work on homework and hanging out with friends.

3. Planning free time is as important as to plan time to work on homework.

4. To plan some things is good, but I prefer living without a strict plan.

5. If you do not plan your life, you can become stressed and frustrating.

6. If I do not plan everything, I am free to change my plans and I did not have to cancel plans.

7. When I carefully plan everything, I know what I need to do and when I am working on it.
I.

8. When I do not plan everything, I have more opportunities to have fun and getting more done.

,,

I,

Sentence -Formation

573 .

l!!!l!!!!!111!!!!!!!!!!!!1!!!!!!!!!!!!1!!!!!!!!!________...........llllllllil.~--~......................... ~

j ,,...

;i '

,,;;;;Mf44.

~i~.~:1~:'.:~:;} "'.~:~-~ :~.:s,e;:f~;:f~ ~J~ :!;,J, (~;,~i~.~,:~:fl.~i:f;;i~~-1


Reading Section
Directions
In this section, you will read five passages and then answer reading comprehension
questions about each passage. Most questions are worth one point, but the last question in
each set is worth more than one point. The directions indicate how many points you may
receive.
You will have 100 minutes to read all of the passages and answer the questions. Some
passagesinclude a word or phrase that is underlined in blue. For those words, you will see
a definition or an explanation below the passage.
You can skip questions and go back to them later as long as there is time remaining.
When you are ready to continue, press Continue to go to the next page.

Practice Test

577 _ _:-_.

1 Psychology

'

r'

!'

"

;~I~.~J'.~
- w,

l'...._,,.,(-

,im

Wk~

Moral Behavior

The dictionary defines morality as "being in accord with standards of right or good
conduct." The argument over whether our moral behavior is innate or whether it is developed
by our environment and culture has been raging for ages. Many people feel morality is
based on reason, while others feel it comes from religion or one's own spirituality.
Biologists believe that humans' tendency to obey the Golden Rule-"Do unto others as
you would have them do unto you" -is a product of evolution.
Ill A) At first, moral behavior seems to oppose the rules of Charles Darwin's theory of
survival of the fittest and natural selection. II B) However, researchers in the field of
biology feel that as animals evolved to live in groups, the propensity to look only after
one's own needs had to fade in order for a group mentality to emerge. Ill C) To ensure
the group's success as a whole, every member had to look out for the interests of the
majority, a concept known as utilitarianism. Ill D)This is a system of beliefs based on what
does the greatest good for the greatest number of people.
A researcher named Jonathan Haidt at the University of Virginia believes that morality
is driven by two separate mindsets-one ancient and one modern. Dr. Haidt declares that
the human mind is unaware of the distinction between the two. The ancient mental system
is based on the emotion behind moral behaviors, which is a type of intuitive sense of what
is right and wrong that evolved before language developed. These are the "gut reactions"
people experience in tough situations that call for quick action. The more modern system
of thought came with the development of language, as people became able to express
verbally why something was right or wrong. The two work together when we are put in
morally compromising situations. When confronted with a moral dilemma, one's intuition
immediately decides what is right or wrong. Rational thought and judgment about the
morality of an issue follow the decision that one's emotional reaction already made.
Dr. Haid! identified five areas of moral conduct that are common in most countries and
systems throughout the world, and he describes these as the foundation for all moral
behavior. These moral components conceptualize how people treat others and what is
important in being part of a group. Regardless of their background, religion, _socioeconomic
;>tatus, or educ_ational level, Dr. Haidt found that the majority of people hold to these moral
concepts. The first moral concept is the prevention of harm. Generally, people believe that
it is wrong to harm another human being or animal for cruel and needless reasons. The
, second moral concept is fairness. which holds that all people should be treated fairly. For

I
t:

x
'l

i1

1'

".i
i

:
1

i~----- --------------==----=-=-"'='-'=.:.---=.,.-_.=,"-'~-=-~=-..o;;.-.:.=;-;.~=:~---------- -- ----- ----

~;{--

578

.I
l

\-

Practice Test

-------- ------- "o--

0.--~ __

;;;:,-_-,,_,__o;:-

_co---'-'--' --

[f

instance, people should treat the poor in the same manner as the wealthy, and the weak
the same as the strong. The third moral concept is loyalty to one's group. This entails a
strong devotion to the values of the group as a whole. People with strong loyalties believe
that adherence to the laws of society is important because it upholds the integrity of that
society. The idea of loyalty is closely interwoven with the fourth moral foundation: respect
for authority. People who value authority believe in the strength of a governing body and
a strong hierarchy with established roles and rules. The fifth concept involves upholding
high standards of purity. This deals with the way that members of a group view their bodies.
The idea of purity comes into play in the standards of cleanliness in society. Daily hygiene
routines, eating food that has not been contaminated in some way, and burial rules and
rituals fall into this category.
Dr. Haidt's research concludes that these moral concepts are inherent in our physical
makeup and are learned behaviors, reinforced by our environments from a very early age.
The five moral foundations are interpreted differently from society to society, and people
rate them differently in order of importance. While morality may take different forms across
the many different cultures of the world, it remains true that the basic task of morality,
restrainiri~ selfishness, is a part of all humanity's moral behavior.

I[<
['
I

i'.

Cl

'

''

[f

~ intuitive
knowing or perceiving things instinctively
~

moral dilemma
a situation in which one has to choose between right and wrong

socioeconomic status
determined by one's education, occupation, lifestyle, and income

Practice Test 579 --,

ii.

!\11

,,

ji
! 1
11~

jl1j

I~

i'I:::t
I'I''
1fiJ

Ii

'1.

.,,~
1i!

!JI

iii

r;j

l!11

;If

J;

tb

'~ !

11
;._i

1. The word innate in the passage is closest


in meaning to
(A) expected
(B) instinctive
(C) sincere
(D) predictable

5. The word propensity in the passage can

2. Why does the author mention the Golden

6. Which of the following best expresses the

Rule in paragraph 1?
(A) To contrast moral behavior with
immoral behavior
(B) To prove that people generally know
right from wrong
(C) To suggest that evolution shaped
morality
(D) To define the idea of moral behavior

1:,;

li;i
l)' ~

<"t'
!~J

',!j

~\
!'

"

3. Look at the four squares (II] that indicate


where the following sentence could be
added to the passage.
In other words, evolution appears to
favor individuals who have learned
how to get what they need in order to
survive.
Where would the sentence best fit?
(A)
(B)
(C)
(D)

First square
Second square
Third square
Fourth square

4. According to the passage, morality


developed because
(A) people learned to communicate using
spoken language
(B) people had to learn to survive in
groups
(C) people were born with the ability to
know right and wrong
(D) people learned moral behavior from
their ancestors

best be defined as

(-A) sincerity
(B) weakness
(C) tendency
(D) helplessness

essential information in the highlighted


sentence? 1ncorrect answer choices
change the meaning in important ways or
leave out essential information.
(A) The earlier mindset based morality on
an inborn, emotional understanding of
what is proper and acceptable.
(B) Old-fashioned ways of thinking based
goodness on the way a person felt
about a situation.
. (C) Theories about ancient attitudes
claimed language was not necessary
to determine moral behavior.
(D) Emotions governed the morality of
people in ancient times before they
developed a way to communicate.

'f

'

7. Which of the following is NOT correct


about the five moral concepts?

(A) They typically develop in sequential


order.
(B) They serve as the core of all moral
behaviors.
(C) They vary in importance from country
to country.
(D) They explain the various ideas that
drive moral behavior.

. -~

_,,

Which of the following can be inferred about


Dr. Haidt's five areas of moral conduct?
(A) They are disputed in various cultures.
(B) Aspects of them appear in the laws of
many countries.
(C) Many leaders would likely disagree
with their loyalty principles.
(D) They are. based on innate human
tendencies.

9. What can be inferred about humanity as a


whole based on Dr. Haidt's moral concepts?
(A) Morality is a universal characteristic
that applies to the whole world.
(B) The natural world plays a major role
in the development of morality.
(C) Morality is an instinctive characteristic
that humans have from birth.
t (D) People from different cultures will not
:"' . value the same principles.

'
13. Directions: An introductory
sentence for
a brief summary of the passage is provided
below. Complete the summary by selecting
the THREE answer choices that express
the most important ideas in the passage.
Some answer choices do not belong in
the summary because they express ideas
that are not presented in the passage or
are minor ideas in the passage. This
question is worth 2 points.
Moral behavior has certain
characteristics that are common to
most countries throughout the world.

1& According to the passage, morality


(A) conflicts with Darwin's theory of natural
selection
(B) goes against the dominant authority
(C) seeks to restrict human selfishness
(D) includes generosity as an important
moral
11. The word entails in the passage is closest
in meaning to
(A) follows
(B) produces
(C) causes
(D) involves
12. The word This in paragraph 4 ref~rs to
(A) A strong hierarchy
(8) The fifth concept
(C) A governing body
(D) Upholding high standards

Answer Choices
(A) Utilitarianism is a belief based on
what does the greatest good for the
greatest number of people.
(B) Morality is ari outcome of evolution
that made living in groups possible
for humans.
(C) Many people believe that morality is
based mostly on reason.
(D) Dr. Haidt has identified five moral
principles that most countries share:
freedom from harm, fairness, loyalty,
respect for authority, and purity.
(E) Those who value loyalty believe that
rules should be followed regardless.
(F) Morality is a combination of innate
qualities and learned behaviors.
(G) Freedom from harm is the belief that no
creature should be hurt needlessly.

Practice Test

581 _

. ._

,02 History

Ancient Angkor

.. ,.,
,,\

1
1

In the regions of Southeast Asia dwell the remains of an era that far exceeded its time
in developments and industrialization. This ancient city, which was mysteriously deserted
in the 15'" century, is known as Angkor. Located in Cambodia, Angkor was established in
802 CE as the seat of the Khmer Empire. Khmer was the largest continuous empire in
Southeast Asia. Its main city of Angkor grew and developed until it was abandoned in the
year 1431. Many historians theorize as to why it was abandoned, but the mystery remains.
Angkor was a city of power, industry, architecture, and cultural unity, which is why
speculation surrounds its decline. The ancient Khmer city stretched over an area of nearly
120 square miles, comparable to present-day Los Angeles. Each successive ruler to the
, throne brought significant additions that diversified the territory. One ruler is known for
constructing a baray, a massive water reservoir. Another built the imposing Angkor Wat,
a temple of great proportions that survived the city's demise and exists today as a
Buddhist temple. Along with over seventy other temples in the region, Angkor was home
to an expansive waterworks of marked ingenuity when nothing of its kind existed in the
world. The civilization was structured around the Mekong River. Intricate and sophisticated
irrigation sy.stems were fashioned to transport water to people and fields in all parts of the
city, including those removed from the central water source. For this, the city became
known as the "Hydraulic City." The people of Angkor were led by an extensive court system,
made up of religious and secular nobles as well as artisans, fishermen, rice farmers,
soldiers, and elephant keepers. The civilization was guarded by an army transported by
elephants and ruled by shrewd and powerful kings. Yet after 600 years of existence, an
abandoned shell was all that remained.
The land, buildings, and architecture were reclaimed by the surrounding forest
regions until the 19'" century, when French archaeologists discovered the remains and
began restoring sites in tl1e great city of Angkor. Since then, theories have evolved over
time relating to the death of Angkor's civilization. The first theory states that the city fell
because of war. The last two centuries of Angkor's existence showed a decline in the
Khmer Empire's population and power. Ongoing wars with neighboring Thailand had
devastated the nation. In 1431, attackers from Thai nations invaded and looted Angkor,
leaving it desolate and vacant. Continuous war with Thailand culminaffng in a final attack
on the city could have weakened the empire and led to the city's demise.
Another theory states that a change in religion led to the country's downfall. The Khmer

J
-':;":\'..',;- 582

Practice Test

Empire had predominately been a Hindu nation, and the people were unified in their
religion. Jayavarman VII, acclaimed as the greatest of Angkor's kings, took the throne in
1181 CE. He instituted a change in religion from Hinduism to Mahayana Buddhism. This
action subsequently could have destroyed the unity of the people and the overall
foundation of the empire.
A) Natural disaster is another feasible possibility for the scattering of people from the
Angkor region. B) Historians say earthquakes, floods, and drastic climate changes would
have been capable of stripping Angkor of its people. C) One researcher hypothesized
that the city suffered from a lack of water due to the transition from the medieval warm
period to the little ice age. Others dismiss this idea. D)
However, a recently developed theory built on the work of French archaeologist
Bernard-Philippe Groslier may have shed the most light on Angkor's demise. The theory
suggests that the Angkorian civilization was "defined, sustained, and ultimately
overwhelmed by over-exploitation and the environmental impacts of a complex watermanagement network." Its vast waterworks proved too great for the city to manage. Also,
supplying such a massive empire with water had adverse effects on the environment.
EC'ological problems included deforestation, topsoil degradation, and erosion due in part
to clearing vegetation for cropland. Thus, the city inadvertently brought about its own
"
environmental
collapse.
With the use of aerial photography and high-resolution, ground-sensing radar,
researchers were able to support Groslier's theory with images that complete existing
topographical maps. The radar detected surface structures as well as subtle variances in
surface vegetation and soil moisture. This proved that environmental erosion had
occurred. The combined images and ground-based investigations further revealed that
Angkor was a victim of its own industrial ingenuity, a city ahead of its time and vulnerable
to its own power.
'-'j\''

waterworks

a water system, including reservoirs, pipes, buildings, and pumps that supply water to
a community
~

medieval

of the Middle Ages time period

>topographical
pertaining to the features of the land

'

----------=--....=..:- ---------

--- -., ..;:.-..;;;..-=-~-==---==--"'~-~-,--=-..

------------------------------------

-~- - --~-]

Practice Test

583 --'. .

11Jiiilr/L\;l~~f-~\!\lll;!.~l!li>Jl.IW.;i:;."'lf:l-1.lf=.(Jltlil$!W.'/Zl!~~'r;'~~~~~,~~~21'.!>'IMl!i.il"l=i>~W--.u'i<11rio'.li!OiWJ.~~'=''"' '.-"('Ul

1. The author mentions the Khmer Empire in


paragraph 1 in order to
(A) establish the size and importance of
the civilization
(B) explain the downfall of the main city
in the empire
(C) compare the nation's size to a
present-day location
(0) demonstrate why people were not
loyal to the city

2. The word speculation in the passage is

5. The word its in the passage refers to


(A)
(B)
(C)
(0)

Angkor's
baray's
waterworks'
home's

_-:>

,,
:e

6. The word sophisticated in the passage is

h,_,_,

closest in meaning to
(A)
(B)
(C)
(0)

advanced
fantastic
educated
thorough

closest in meaning to
(A)
(B)
(C)
(0)

'.\

\I

evidence
mystery
question
growth

3. .According to paragraph 2, which of the


following is true about the waterworks
built within Angkor?
(A) They transported drinking water to
Angkor Wat.
(B) They were a money-making venture
for the city.
(C) They were built to extend the water
supply.
(0) They irrigated fields along the sides
of the river.

4. All of the following are true about the city


of Angkor EXCEPT:
(A)
(B)
(C)
(0)

It was built around a water source.


It had an advanced road system.
It surpassed other cities of its time.
It is home to a Buddhist shrine.

7. Which of the sentences below best


expresses the essential information in the
highlighted sentence? Incorrect choices
may change the meaning in important
ways or leave out essential information.

(A) Archaeologists built a replica of what


Angkor looked like.
(B) Archaeologists uncovered the
overgrown city and rebuilt its sites.
(C) Finding the city, workers cleared the
forest and studied the architecture.
(0) The city's architecture was inspired
by the forest regions nearby.

.,

,,

'

~;

~z
:{\

J'
"~

~(1
"
-~'

;>!1'
~

:~
',(
,'l
;_<-;

8. What can be inferred from paragraph 4


about the people who inhabited Angkor?

(A) They worshipped ruler Jayavarman


VII.
(B) Hinduism was central to their way of
life.
(C) Religion led to more violence among
them.
(D) They were unified regardless of
national religion.

J
"jfi

%
V
%
b

>

'k;,1
:;:)
,':;
fj')

'\;
j
;}

'

r~

;'~',;-

584 Practice Test

~.

[
[

".'.'.

Ill

Look at the four s'quares [] that indicate


where the following sentence could be
added to the passage.

These natural catastrophes would have


likely resulted in destroyed buildings,
ruined crops, and a decreased water
supply that would have forced citizens
to leave.

12. Directions: Complete the table below to


categorize information about each of the
theories discussed in the passage. Match
the appropriate statements to the theory
with which they are associated. TWO of
the answer choices will NOT be used.
This question is worth 4 points.
Theory

Where would the sentence best fit?


(A)
(B)
(C)
(D)

First square
Second square
Third square
Fourth square

War
Theory

Religion
Theory

Groslier's
Theory

1O. The word inadvertently in the passage is


closest in meaning to

. (A) purposely
(B). freely
(Cj'foadly
(Dfaccidentally

11. According to paragraph 7, which of the


following did researchers prove about
Groslier's theory with the use of aerial
photography and advanced radar?
(A) The surface soil showed evidence of
dirt washing away.
(B) The waterworks were filled with topsoil.
(C) Vegetation was thriving where soil
was deeper.
(D) Soil damage was stable throughout
the changes.

Statements

Cl

. _,,,

Ill

"

Cl

-,,,,,

.,::;:
...Cl

'

"

Answer Choices
(A) The predominant religion was originally
Hindu.
(B) The developments were too vast to
manage.
(C) Ongoing attacks weakened the city.
(D) Powerful rulers made Angkor a
prosperous city.
(E) The irrigation system sapped the city
of its resources.
(F) Thai soldiers invaded and looted the
city.
(G) The city's land was eroded and
overused.
(H) Architectural projects divided the
people.
(I) The change to Buddhism destroyed
the people's unity.

Prnctice Test

585 - - .;''. ,.

D3 Astronomy

l
.,

Magnetars
In 1979, scientists detected a sudden spike in gamma rays that had never before been
. experienced. Over the next several years, scientists detected that the same unidentified
celestial body sent out powerful bursts intermittently. Scientists struggled to determine
what was causing these large bursts. They theorized that whatever it was, it had to have
a super-strong magnetic field in order to release the amount of energy that it did. By the
time another burst was detected in 1998, scientists had come up with an explanation to
explain this strange anomaly. The source of these large bursts was termed a magnetar.
A magnetar is defined as a type of neutron star that possesses the strongest magnetic
field of anything in space. A refrigerator magnet measures 100 Gauss while the Earth's
magnetic field is .5 Gauss. A magnetar measures an amazing 10" Gauss, making it at
least one hundred trillion times (100,000,000,000,000) as powerful as Earth's magnetic
.field. Occasionally, these magnetars will experience sudden "bursts" and hurl bright flashes
of gamma rays into space. Not much is known about the mechanisms that cause these
magnetar "bursts," except that they result from the instability of the star's powerful magnetic
field. One theory speculates that the unstable magnetic field causes the star's crust to fold
and crack, further disrupting the star's magnetic field and generating massive waves that
release large amounts of gamma rays into space.
Although the actual theory was proposed in 1992, magnetars have not yet been
categorized as a recognized major type of star. A magnetar is, however, classified as a
subtype of neutron star. A neutron star is formed when a massive star dies by collapsing
into a supernova or stellar explosion. The mass of a neutron star is greater than the mass
of the sun, but is contained in a star with a diameter of only ten to fifteen miles, about the
size of Manhattan Island in New York. This extremely dense star is composed mostly of
neutrons and has a very strong magnetic field. A magnetar has a magnetic field one
thousand times stronger than any other type of neutron star. This gives it an extraordinary
amount of power for such a tiny star.
Normal magnetars can generate the amount of energy in one second that it takes the sun
a year to produce. Ill t\) However, the magnetar bursts in 1979 and 1998 were abnormal
even for magnetars because they were approximately one thousand times brighter than
a normal magnetar. In fact, during the brief period of these outbursts, the magnetarswere
brighter than supernovas. till 3) The energy that radiates from supernovas in the short
period between when they explode and when they fade is equal to the amount of energy

:~r---

586

Practice Test

'

.>

Ill

-z

Gl

the sun will radiate over ten billion years. C) Yet the two abnormally strong magnetars
of 1979 and 1998 surpassed supernovas in both brightness and energy. D) Magnetars
are also unique because they produce multiple bursts of light that last only a second,
much like a pulsar. A pulsar is a periodic flash of light from a star that is sending a
constant beam of energy or light away from itself. Due to the pulsar's rotation, it appears
to be blinking on and off as its light sweeps over Earth.
Scientists predict that there are likely millions of magnetars in the galaxy, but only five
magnetars have been discovered since the first one was observed in 1979. A recent
magnetar was observed in late December 2004. This burst is considered the brightest
burst ever. It was only .25 seconds long, but it was powerful enough to overwhelm
energy detectors on the many satellites in space so that no energy measurement could
be taken. It also interfered with radio communication on Earth. The magnetar was an
estimated thirty to fifty thousand light years from Earth. Scientists say that the magnetic
force is so strong on a magnetar that it could wipe a credit card clean, or make it
ineffective, from a distance of one hundred thousand miles, or about half the distance to
the moon.
Althougffscientists have only been able to detect a few young magnetars, they expect
to detect many more in the future. Much about magnetars remains a mystery, and they
are a challenge for scientists to study since they appear once every few years and are
observable only for a second. Scientists hope to determine more about what causes the
instability within a magnetar's magnetic field so that they can better understand these
powerful bursts.
~

.,,

=.=.~.

!/:

""z
Gl

-:,-;-;::=

"'
I

grnnmo my
a type of radiation with a short wavelength that can pass through solid objects

celes!ial

in or relating to the sky, heaven, or space


>crust
the thick outer surface of a planet

l.~.-

.-- ..._- ..---~~~ "

Practice Test

i"c__fr

587 - .

~~:~~~'l"'..b:~~

l>'lr~~l<Zl_~l>'>';'~V"frJ1

. ,._,;;

1. The word intermittently in the passage is


closest in meaning to .
(A}
(B}
(C)
(D)

on a frequent basis
occurring repeatedly
not occurring quickly
not happening regularly

2. The word it in paragraph 1 refers to


(A)
(B}
(C)
(D)

spike in gamma rays


unidentified celestial object
energy
magnetic field

3. The word anomaly in the passage is


closest in meaning to
(A)
(B)
(C)
(D}

something out of the ordinary


an irregular occurrence
something never seen before
a natural event

4. Why did the author discuss a refrigerator

,.'

magnet in paragraph 2?
(A) To provide a point of reference for a
magnetar's strength
(B} To illustrate that Earth's magnetic field
is not as strong as most magnets
(C} To demonstrate that a magnet has an
unstable magnetic field
(D} To contrast the strength of magnets
in space and on Earth

5. According to paragraph 2, magnetar


bursts are a product of
(A)
(B)
(C)
(D)

combined magnetic forces


a weakening of a star's magnetic field
stars folding into each other
an unstable magnetic field

'

essential information in the highlighted


sentence? Incorrect answer choices
change the meaning in important ways
or leave out essential information.
(A) The disruption in the magnetic field
leads to the release of gamma-ray
bursts.
(B} Folds in the star's crust create
massive waves that make the
magnetic field unstable.
(C) The star's magnetic field becomes
too unstable to be contained within
the magnetar.
(D) The magnetic force causes the star's
surface to break into gamma-ray
bursts.

7. According to paragraph 3, a magnetar


(A)
(B}
(C)
(D}

has a tiny mass


is a dying star
was once a small star
is larger than Manhattan

~~

1'.;

6. Which of the following best expresses the

{}
-l

,y;
\'.:i

.-:~1

~~
;;?

/,/
i'.'i

~$

;;--h
,,,_

.'J--J

"

'>

I'"

'

f'

'i}

:e

.~'li

n,
v

,'f_

"

-:-.1

~.'-~H
>;'!
;1

~t'

.'','

c'l

.-'
:f

,f,

;'.~
:;_
;--:

8. The author's description of neutron stars


mentions all of the following EXCEPT:
(A} They are formed when a supernova
collapses.
(B} They are the brightest stars that have
been discovered.
(C) They have a huge mass in a small
body.
(D} They have extremely powerful
magnetic fields.

J
,.
';:'

;-;

J
'

;'.

'
.~

-1

';;-~--

588 Practice rest

t
;

t.:

Look at the four squares [II) that indicate


where the following sentence could be
added to the passage.

These supernovas have been some of


the most luminous events ever
observed in the universe.
Where would the sentence best fit?
(A)
(B)
(C)
(D)

First square
Second square
Third square
Fourth square

. 10. Which of the following can be inferred


from paragraph 4 about magnetars?
(A) Most magnetar bursts are abnormal.
(B) Pulsars share many similarities with
supernovas.
, (C) Pulsars and magnetars do not rotate.
'f'.: (D) Normal magnetar bursts are not as
bright as supernovas.
:(:.

11. Which of the following is NOT true about


magnetars in paragraph 57
(A) The brightest magnetar burst was
only .25 seconds long.
(B) The universe is thought to have at
least one million magneiars.
(C) The brightest magnetar is thirty to fifty
thousand light years away.
(D) Scientists have identified a total of
five magnetars since 1978.
12. In paragraph 5, the word it refers to
(A) magnetic force
(B) credit card
(C) magnetar
(D) radio communication

'

13. Directions: An introductory sentence for


a brief summary of the passage is provided
below. Complete the summary by selecting
the THREE answer choices that express
the most important ideas in the passage.
Some answer choices do not belong in
the summary because they express ideas
that are not presented in the passage or
are minor ideas in the passage. This
question is worth 2 points.
This passage discusses a new subtype
of neutron star called a magnetar.

'i.

'

Answer Choices
(A) Characterized by their powerful
magnetic fields, magnetars are
subtypes of neutron stars.
(B) Neutron stars come into being when a
massive star dies in a stellar explosion
and sends charged matter into space.
(C) Pulsars are similar to magnetars in
that their energy beam seems to flash
periodically.
(D) The magnetic force on a magnetar is
stronger than any other magnetic field
in the universe.
(E) Magnetars break brightness records
previously set by supernovas and are
many times brighter than the sun.
(F) The Earth's magnetic field seems
weak in comparison with the magnetic
field of a magnetar.

I
Practice Test

589 ----,

i;p MtlmiMJil

m!!!mlll : ifilB\l'j
Readin9
,'

, 1 of 13
;

.--

00:20:00
-.

;.

Leitmotif
The chords struck in Beethoven's composition for string quartets and the ominous
music that fills moviegoers with dread when the villain Darth Vader enters the scene in the
Star Wars movies are examples of leitmotifs. First coined as a musical term in 1871, it is
defined as the leading theme in a piece of music. Taken from the German words leiten,
which means "to lead," and motiv, which means "motive," the word is literally translated
as "leading motive." A more modern definition is a dominant recurring theme associated with
' a person, subject, or idea in a work of art. Leitmotifs are seen across many genres, including
music, visual arts, and literature.
A leitmotif is usually a short melody that is repeated throughout the body of a piece of
music, though some composers choose to use recurrent chords or percussion rhythms. The
use of leitmotifs helps to show congruency within the piece of music and tie the different
movements together as a unified whole. The leitmotif usually plays on the emotions of the
audience, even if subconsciously, and is a way to convey meaning without words or to
take the composition to a different level. Classical music most often comes to mind when
leitmotifs are mentioned. They are seen in operas and other dramatic compositions.
Bil A) A leitmotif cannot be seen in a single song. Ill B) For instance, the rhythmic
background of hip-hop music is not considered a leitmotif because it does not express
emotion or work to change the mood of the song. II C) A true leitmotif is more than just a
iI style of music; rather, it occurs several times throughout a composition, expressing
i certain emotions and binding the entire work together. II D)
The idea of leitmotif can be seen in the music of modern movies, television shows, and
even video games. The example of the music played when Darth Vader comes on screen
1
in Star Wars is a more classical example of a leitmotif. Likewise, the eerie sound that is
played in the Friday the 13" horror movie series when the murderer is about to take
another victim is a leitmotif as well, though not true music. Both signify the presence of a
lead character and evoke intense emotion. Television shows usually use leitmotifs from
episode to episode. For example, the television series Lost has several leitmotifs that are
played at different points in the show. If the actors are climbing or hiking through the thick
jungle of the island, the audience hears music that allows it to experience the hard work
and time it takes to fight through the dense vegetation. The popular 1990s comedy series
Seinfeld uses a rhythmic melody for scene changes during each episode. Video game
fans can easily recognize the theme music from the Halo series played at the beginning
1.

I
1.

"'''~--~c~~.-~,~~ ~-~--'"-'~''"."'c.c~~~~-.~.- -

590

Practice Test

I
.J

l.

-z
[_
G'\

,..

~z
zG'\
of each game.
Leitmotif is not confined only to music, but can be seen in visual works of art. For
example, Michelangelo is known for incorporating images of the nude male body in many
of his most famous works. His sculpture of David and his painting of the Sistine Chapel
both exemplify this image. A more modern artist, Thomas Kinkade, is known as the painter
of light. Each of his works includes pastel colors and scenes that seem to reflect and even
produce light within the painting. He also paints with themes of simplicity in most of his
. works. Much of his art features cottage-style homes and simplistic gardens and streams
that evoke emotions of comfort and nostalgia
Literature is another art form in which leitmotifs are used. When employed in literature,
a leitmotif is seen as a recurring event, object, or character referred to throughout the
book. Author James Joyce's book A Portrait of the Artist as a Young Man includes themes
of the Virgin Mary and Daedalus from Greek mythology to promote unity as the main
character matures. John Grisham, a modern-day writer of fiction, includes themes of
ethical dilemma and justice throughout his suspenseful thrillers that deal with the US legal
system.
'~'Classical music introduced the idea of leitmotifs to the world, but artists tOok this
'
ccmcept and applied it to other genres as well. The idea of a recurring theme appealed
to'.'artists and audiences alike because it conveyed intense emotion and drew the
audience into the drama and beauty of the artistic piece. Leitmotifs have influenced how
people participate in art, whether through the emotion of the rnusic or through identifying
themes within visual art and literature.

Ill
"'O

~z

G'\

~
3
z

G'\

:~\'

~nostalgia

a mixed feeling of happiness, sadness, and longing when recalling a person, place, or
event from the past

I
Prnctice Test

591 _ _[
I

;.->

!;!,J,~UJW.i:m='M'A"ErUl!l'!illl~~~Rll-llllllil'Allral:!1Ut;<\lV:m11~MWlf>l'~"'$!"',_.;ru~=-'\Zl.~WO'.."'<l~.CCn;}l'Z)';.."3..1'1TI'J!~UWlJ~'"'!;<(:r<;;;cll.'>l~'"''<'~'-'t-_ -~-'C-.,-,,,

1. The word coined in the passage is best


defined as
(A)
(B)
(C)
(0)

named
enjoyed
displayed
invented

2. The word congruency in the passage is

the 13" horror movies?

(A)
(B)
(C)
(0)

(A) To provide an example of a leitmotif


that is not actual music
(B) To demonstrate the use of a recurring
theme within a series
(C) To illustrate how music can frighten
audiences
(0) To compare the emotions displayed
with those in Star Wars

symmetry
regularity
unity
similarity

about the use of a letimotif in music?


(A) It is a melody that repeats throughout
the work.
(B) It can be a specific recurring rhythm.
(C) It ties the various movements together.
(0) It is used in all forms of music.

4. The word They in paragraph 2 refers to


(A)
(B)
(C)
(0)

Classical music
Leitmotifs
Emotions of the audience
Cornpositions

5. Look at the four squares [Ill] that indicate


where the following sentence could be
added to the passage.
Similarly, a leitmotif does not flow
through all of an artist's songs in the
same genre, such as country or roclc.
Where would the sentence best fit?
(A)
(B)
(C)
(0)

"

First square
Second square
Third square
Fourth square

Practice Test

\'

',

7. Why does the author mention the Friday

closest in meaning to

3. Which of the following is NOT correct

...:;;/-- 592

6. The word composition in the passage is


closest in meaning to
(A) harmony
(B) book
(C) make-up
(0) piece

8. What can be inferred from paragraph 4


about leitmotifs in movies?

(A) They affect viewers more than plot


development in movies.
(B) They could be effectively
incorporated into romance movies.
(C) Those used in movies differ from the
original definition of leitmotif.
(0) They ensure an emotional
performance on the part of the actor.

9. The word it in paragraph 4 refers to the


(A)
(B)
(C)
(0)

island
audience
music
thick jungle

10. In terms of leitmotifs, the painting on the


Sistine Chapel and the sculpture are
similar because
(A)
(B)
(C)
(0)

they both feature recurring images


they are both Michelangelo's work
they are from the same time period
their inclusion of nude males is
controversial

'

11. According to paragraph 5, which of the


following is true of Thomas Kinkade's
work?
(A) He paints scenes from his life in a
. cottage-style home.
(B) His art causes viewers to think back
to their childhood.
(C) His work looks best in pastel light.
(D) He paints cottages where he lived as
a child.

12. Which of the following best expresses the


essential information in the highlighted
sentence? Incorrect answer choices
change the meaning in important ways or
leave out essential information.
(A) Leitmotif is popular for its ability to
articulate feelings that pull the listeners
. . into the dramatic storyline.
@(B) The sentiment of the art piece
depends on leitmotif to ensure the
"t"
audience understands what the
composer is trying to convey.
(C) Audiences like leitmotif because of
the many feelings that are evoked by
the story.
(D) Leitmotif is a good idea because
audiences and composers can agree
on its usefulness.
13. Directions: An introductory sentence for
a brief summary of the passage is provided
below. Complete the summary by selecting
the THREE answer choices that express
the most important ideas in the passage.
Some answer choices do not belong in
the summary because they express ideas
that are not presented in the passage or
are minor ideas in the passage. This
question is worth 2 points.

This passage discusses the extensive


use of leitmotifs, or recurring themes,
throughout different types of art.

Answer Choices
(A) Thomas Kinkade uses leitmotifs in his
paintings through light and through
simple, cozy scenes as subjects.
(B) Though leitmotifs are seen mostly in
music, they are often used in literature
and visual art.
(C) A leitmotif is usually a link between
separate movements of music,
chapters in a book, scenes of movies
or TV shows, or an artist's works.
(D) Theme music in movies and TV is an
example of the use of leitmotifs in
modern expressions of art.
(E) A 1.eitmotif.cannotbe seen in a single
.song because it is more than a
foundational rhythm or music in a
chorus.
(F) Leitmotifs are a way to communicate
intense emotion and bring about a
different scene without explicitly
explaining what the artist has in mind.

Practice Test

593 ..,_-'.

LJS Biology
1i1ff!i&iji

. - - - 594

Neurons and Human Memory


Writing a paper, recognizing a smell, remembering a phone number, and singing a
song are all examples of different functions that require some measure of human memory.
The processes of the brain have puzzled and intrigued scientists for years. Many have
sought to understand how the memory works. After years of study, research shows that
memory is recorded and processed by neurons found in the brain.
Neurons, or nerve cells, are the basic units of the nervous system. These cells
comprise the core components of the brain and act as the processing and transmission
centers for the body. They are the way the rest of the body communicates with the brain
and are highly specialized in nature. Some neurons in the brain are capable of signaling
and connecting with thousands of other cells. Others receive many signals, but only send
out one or two. A synapse is essentially a gap between neurons that serves as the link
between them. These neurons transmit information across the synapse in the form of an
electrical impulse. It is within the action of these neurons that memories are created.
Memory is the ability to store, retain, and retrieve information. For the brain to record
memories, it requires a simultaneous and coordinated activation of neuron receptors at
the synapses. For example, a child knows not to touch hot objects from a previous
experience of being burned. When he first touched a hot stovetop, the pain signal from
the skin on his hand along with the view from his retina reached the brain at the same time,
forming a memory. He refers to that memory each time he sees a hot object, remembering
the consequences of touching something hot. Thus, the signals sentto the neurons in the
brain formed a specific arid useful memory.
One researcher has studied the basic functions of memory, finding that individual
memories are "burned onto" receptors that are constantly in motion around nerve synapses.
Iii ;'\) The synapses allow signals to travel through the brain, and often the receptors
containing memories are lost or escape from the synapses. !ill B) When this occurs, a
specific set of molecules catches the loose receptors and takes them to a recycling plant
of sorts, where they are reprocessed and then returned intact to the synapse. Iii G) When the
receptors are not recycled, there is a gradual loss of synaptic function and, subsequently,
a reduced "'QgL11\;.'ic~ ability. These findings may prove useful in understanding memory
loss as well as neurological disorders such as Alzlieimer's and learning disorders like
autism. II -1'
A different group of scientists has performed a study on the specific connection
:~oo~-u'

c_-,-__

Practice Test

-"--c_

~------------------~---

----O~:- ____

o_

__;-~~-'-

~
.:

,,

__

~
between memory and neurons. The study showed that when a person learns something
and then recalls what was learned, the same neurons used in the original experience are
triggered. It actually reintroduces the same emotions felt when the memory was formed.
Additionally, memories are most likely stored in neuron subgroups. Those neurons are
activated in response to various sensory experiences that prompt a memory. This
discovery shows precisely which circuits are active during formation of a specific memory.
Whereas researchers previously knew that neurons existed, they now understand more
comprehensively how they work.
This study was done on a set of mice that contained a specific gene that had been
altered for study. In essence, the scientists genetically tagged, or marked, individual
neurons within each mouse's brain and noted when the neurons were activated within a
given time frame. They reasoned that fear was a natural and necessary emotion in survival
and thus was a valid emotion that mice experience. The technology allowed scientists to
record and measure neuron activity along with certain memories. The mice showed that
the same neurons activated during fear conditioning are reactivated during memory
, retrieval.
f;L Now that researchers have found a link between neurons and memory formation, the
.,. technique can be applied in other settings. The procedure could help physicians discover
!l\how medications work in the brain. Until now, physicians have had trouble evaluating the .
effects of antidepressants on patients because each patient can react differently to a
medication. Antidepressants that work for one individual may not work for another. Often,
physicians can measure the effects of antidepressants on a patient only after months of
observation. This new genetic tagging technology would allow physicians to evaluate
treatment by comparing how a patient's brain works at two different times, during,
treatment, noting how and where the drug affects specific neurons. This would allow
physicians to evaluate treatment options more quickly and accurately.

:'
'

I
''

G'\

~retina

the part at the back of the eye that sends light signals to the brain

>r,ognitiwi
related to memory, reasoning and judgment
>sensory
relating to the senses or transmitting sensation

'

-------..~-=::->;o.~,==--ro=~~==-.:.:;. ~- ~.---

Practice Test

595 -

'

!!;:ig;l,?f:~:<>r.,~.liJJ!l.'>.\i.'IW<-WV:f'-"'"";~<c)l,'Ut:Ol"io'lJ>0m1'A"<:i<:.<k1:1'."t,1lU:W.umTil.~.m!"~t'l..q.1o;11~.!~~~"'"!n~.n';~WY-..m:::i;\l!.'ff.~.1'iJ"~.111;;>?~~/~NAi'R.\g;,'j;}~~{J,~l:oG<.<-.'lli.'J:IV,\<,;:\'(:0:/,1..:,;;c,-

1. The word intrigued in the passage is


closest in meaning to
(A) busied
(C) informed

(8) fascinated
(D) taught

2. The word They in paragraph 2 refers to


(A)
(8)
(C)
(D)

Components of the brain


Basic units
Neurons
Processing and transmission centers

3. According to paragraph 2, what makes .


nerve cells within the brain unique?

(A) They form the body's nervous system.


(8) They receive signals from the rest of
the body.
(C) They are dependent on synapses to
give them information.
(D) Their only function is to store
memories.
4. The word simultaneous in the passage is
closest in meaning to

(A) singular
(C) clear

(8) painful
(D) instant

5. Why does the author mention the child


who touched a hot object?
(A) To argue that the action was done
purposely to form a useful memory
(8) To illustrate a memory that can
prevent repeated action
(C) To compare touching hot objects with
the formation of negative memories
(D) To note a symbolic way that the brain
records memories
6. According to paragraph 3, all of the
following are true of memory formation
EXCEPT:
(A) It results in the storage and retention
of events.

..., - - - 596- Practice Test

(8) Consequences of an action are not


recorded as memory.
(C) Simultaneous and coordinated activity
is required.
(D) The process involves information
moving across synapses.
7. Look at the four squares [l that indicate
where the following sentence could be
added to the passage.

. ,t

'

'

This Is an ongoing process occurring


over minutes or hours, so the brain
continually receives recycled receptors.

Where would the sentence best fit?


(A)
(8)
(C)
(D)

First square .
Second square
Third square
Fourth square

8. Which of the following can be inferred


about neurological illnesses?

(A) The recycling of receptors leads to


Alzheimer's and other diseases.
(8) The cause of these illnesses is not
treatable because it involves neurons.
(C) "Burning" neurons results in a mass
of memories that move as one group.
(D) Loss of receptors is El likely cause of
these brain disorders.

. -~:,

9. According to paragraph 5, how does


emotion from an experience factor into
memory?
(A) Feelings are reintroduced when
recalling the specific memory.
(8) The brain records th.e emotion that
becomes the memory.
(C) Memory-related feelings are only
recalled in certain circuits.
(D) Neurons activate the appropriate
emotion depending on the type of
memory.

'

"'~

~I

i
1O. The word prompt in the passage is
closest in meaning to
I

(A)
(B)
(C)
(D)

happen quickly
arrive on time
come together
bring around

11. The word they in paragraph 5 refers to.


(A)
(B)
(C)
(D)

memories
researchers
neurons
circuits

12. The author discusses fear conditioning in


paragraph 6 in order to
(A) illustrate one of the multiple techniques
used to test the mice
(B) demonstrate a logical emotion to
:1
which mice respond
(C) explain the only feeling recorded by
active neurons
(D) question the researchers' techniques
and findings
13. Which of the sentences below best
expresses the essential information in the
highlighted sentence? Incorrect choices
may change the meaning in important
ways or leave out essential information.
(A) Physicians can see how medication
affects the brain and which drug is
most effective by using this technology.
(B) The new technology allows doctors to
diagnose certain illnesses and then
treat them with a variety of drugs.
(C) By monitoring the patient's brain,
doctors can experiment with different
drugs and note their effect on each
patient.
(0) Patients subject to tagging technology
show notable brain activity and are
more resistant to medication.

14. Directions: An introductory sentence for


a brief summary of the passage is provided
below. Complete the summary by selecting
the THREE answer choices that express
the most important ideas in the passage.
Some answer choices do not belong in
the summary because they express ideas
that are not presented in the passage or
are minor ideas in the passage. This
question is worth 2 points.

[
~'

Studies show that the link between


memory formation and neurons is the
recording of an experience and related
emotions through the transmission of
information from the body to the brain.

Answer Choices
(A) Electrical impulses are the way in .
which information travels from neuron
to neuron.
(B) New tagging technology offers
information about how and when a
neuron is activated.
(C) The body's nervous system involves
neurons, the brain, and nerves
throughout the body.
(0) Researchers found that when a memory
is recalled, the associated feeling is
experienc;ed.
(E) Memories are stored in neuron
subgroups once they are recorded.
(F) Neurons transfer information, but they
also record events to be stored in the
brain.

Practice lf!st

597

--1
I

Listening Section

[
[
[
[

_,
'
'

G'I

Directions
In this section, you will listen to four lectures and two conversations. You will hear each lecture
and conversation one time.
After each listening passage, you will answer some questions about it. Most questions are
worth one point, but some questions are worth more than one point. The directions indicate
how many points you may receive.
You will have 30 to 40 minutes to both listen and answer the questions. The questions ask
about the main idea and supporting details. Some questions ask about a speaker's purpose
or attitude.
You may take notes while you listen. You may use your notes to help you answer the questions.
Your notes will not be scored.
In some questions, you will see this icon: ().This means you will hear part of the lecture or
conversation again.
When you are ready, press Continue.

Practice Test

599 _ _:

~,H
:~-.~

r'l

History

'T
11,i

!I

'.'I'

. ~
..,~.,_.
. .'~~M..'&"..'Ji&
.... -.. ';~...-;. \ .:.,.,,\;{
~1= MNMm
.. ~.x:,'._~.
?X"j.c.."1:dfu-.~..~..'\;.CUc.,.'.
i'r.t~M;'&
~:.ma
!~} uSt~~inQ_; - ;..:i:ot 34<-:; ~ oO: ~O';oo '~futl:.

:i.'I'.:'

1r~~k.'l- {_;~~(':~~{;~,i;~;j~3~~fu~b~:;,~~~'.t~;~;tii;-

'r'
"

'

i>.

~;"''
,-,

-.~

t;"
t

tListen to a lecture in a history class.

Ji
::~

>
;:

'
j

if'.)

,
}"

,,
i!:

~
t

',Ir

'
Jit
'J

'

.~,'---

,,

:~

>

~;

)'l;

'~
R
'it

'2'

ij

i~

_,._,

:t
'i-1
:~,

"

&

',:':;

-~

"'.,,_:"?
""')}'
)_
.~.;

600

Practice Test

What is the main topic of the lecture?

I
i

(A) Two men that promoted yellow


journalism
(B) The effects of yellow journalism on
political campaigns
(C) The birth and early days of yellow
journalism
(D) The reasons people are drawn to
sensational topics in newspapers

2. Listen again to part of the lecture. Then


answer the question.
What does the professor imply when she
says this:
(A) She knows the students associate
yellow journalism with modernity.
(B) She wants the students to consider
. ~j) . the history of yellow journalism.
'(C) She thinks the students lack historical
d perspective about journalism.
(D) She wants to emphasize that yellow
journalism is not a new concept.

3. According to the professor, why did


Pulitzer target the immigrant community?
(A) He worried about the lack of
resources for them.
(B) He realized he could increase his
readership.
(C) He was concerned about the loss of
native identity.
(D) He did not approve of patriotic
newspapers.

4. What does the professor imply when she


says this:
(A) Pulitzer felt his paper was not popular
enough.
(B) Yellow journalism encouraged
immigrants to learn to read English.
(C) Pulitzer used simple methods to
increase sales.
(D) Few people read newspapers in
Pulitzer's time.

5. What is the professor's attitude toward


Hearst's role in the Spanish-American
War?
(A) She thinks that it was wrong of hirn to
start the war.
(B) She agrees that his late coverage
helped inform people about the issue.
(C) She believes it was unnecessary and
harmful to society.
(D) She thinks he was directly responsible
for the outbreak of the war.

6. How does the professor organize the


information about yellow journalism that
she. presents to the class?
(A) She gives modern and historical
examples of yellow journalism.
(B) She describes the effects of yellow
journalism in history.
(C) She speaks about why yellow
journalism lost its popularity.
(D) She demonstrates the negative
effects of yellow journalism.

Practice

re~t 601 -----~I

02 Chemistry

Listen to a lecture in a chemistry class. @*fii

.j
c

---:
_-.:;

:,-._ 602 Practice Test

What is the main topic of the lecture?


(A) The process of crystallization
(8). How supersaturated solutions are
made
(C) The factors that lead to crystal growth
(0) Why temperature is important in
crystallization

11. How is the discussion organized?

(A) The professor lists key terms and how


they relate to crystallization.
(8) The professor describes each step in
the creation of crystals.
(C) The professor gives examples of
various crystal structures.
(0) The professor demonstrates crystal
growth for the class.

.,:e
3

GI

Listen again to part of the lecture. Then


answer the question.

Why does the professor say this:

. (A) He is doubtful that the students will


understand all the concepts.
(8) He wants to encourage the students
to ask many questions.
(C) He feels that the students cannot relate
,, the process to real-world applications.
~Wl He wants .to make a joke that relates
to the topic of the lecture.

1
,,

Yes

No

A stable cluster in solution


forms the nucleus of the crystal.
A supersaturated solution
eliminates growth potential.

What is the professor's attitude toward


crystals?
(A)
(8)
(C)
(0)

12. Based on information from the lecture,


indicate which of the following statements
reflect the process of crystallization. Place
a checkmark in the correct box.

He does not understand their appeal.


He thinks they are appealing to look at.
He thinks it is unethical to sell crystals.
He feels that they are impure
specimens.

A crystal structure is formed


by connected atoms.
The solution becomes
supersaturated and the
crystal stops growing.
-----M-------

10. What happens if particle clusters do not


stabilize?
(A) They form a nucleus.
(8) They dissolve into the solution.
(C) Supersaturation is exhausted.
(0) The solution cannot support the
addition of another substance.

--1'
.I

Practice Test

603

~-~"~----................................................................ ~..

fJ3 Service Encounter

Listen to a conversation between a student and a university employee. @t+t

,,

Ii

,I

II

!,
11

i't.-:-;;_-_

----- 604

Pr<ictice Test

~
Why does the student visit the English
Department?
(A) To find out about the creative writing
program
(B) To sign up for a creative writing
course
(C) To learn about what courses are
being offered
(D) To talk to a professor about a writing
course

14. Why does the student mention the course


announcement booklet?
(A) To tell the receptionist which class
she wants
(B) To tell the receptionist how he can
help her
(C) To a,{?k for more information about
clas"~es
(D) To iqform the receptionist of why she
cam'e

'
16. What does tlie
man imply when he says
this:
(A) The writing classes are for low-level
students.
(B) Many students think the classes are
big, but that is not true.
(C) All of the writing classes are relatively
small.
(D) Many students are not good writers.

I[

[_
[[

17. Listen again to part of the conversation. Then


answer the question.
What does the man imply when he says
this:
(A) He thinks the students are doing
something unnecessary.
(B) He thinks the students are dedicated
to their studies.
(C) He wants the students to wait in
another place.
(D) He believes the students will most
likely be accepted.

15. What is the first step that a student hoping


to take a creative writing course has to
take?
(A) Attend the first day of class
(B) Sign up for the class at the department
(C) Get the professor's permission to take
the course
(D) Submit a writing sample to the professor

'

Practice Test

605 ______ __.

Psychology
~
:,liStCri!fi~':
,,r_..

Listen to a discussion in a psychology class.

+f!!ffi+

'

!--=~=~~=-~~-=-----~~-~-~~~~---~-~-~~ ------~-=~-~-~----,-~-~-=-~~--=~~~---~~- __________!

'
606 Practice Test

18. What is the lecture mainly about?

21. Why does the student discuss his dream?

(A) Processes that disguise the meanings


of dreams
(B) The reasons that people misinterpret
their dreams
(C) The ways that dreams can help people
learn
(D) How Freud thought that dreams
affected daily life

(A) To give an example of condensation


in dreams
(B) To explain how he learned about his
respect_for laws
(C) To demonstrate the symbolic meaning
of his dream
(D) To illustrate why the theory of
symbolization is false

19. What does the professor imply when she


says this:

[[
[i

22. What is the professor's attitude toward

examining dream symbols?

(A) Freud is no longer referenced by


psychologists.
(B) Freud remains an important figure in
the discipline.
(C) Freud's theories have been proven
..
wrong.
~~(D) Freud created a new kind of
-,,
psychology.

20. Listen again to part of the discussion. Then


answer the question.
Why does the student say this:
(A) He thinks that there is value to the
literal meanings of dreams.
(B) He wants to clarify his understanding
of the process.
(C) He feels it is unfair to discount the
face value of a dream.
(D) He knows that some dreams must be
taken at face value.

(A) She believes that it is not useful due


to its subjective nature.
(B) She knows the students do not
understand their dream symbols.
(C) She thinks that it cannot teach people
about their emotions.
(D) She believes that it is not as revealing
as understanding displacement.

23. What happens during the process of


secondary elaboration?

(A) Extra details are added to the dream.


(B) The dreamer redirects an emotion.
(C) Characters represent a variety of
people or events.
(D) Dream images take on a more
symbolic meaning.

Practice Test

!!!!!!

~
l[

---,------ --,---_--_7_~--------~---------~--

------------- -----,---------~ ---

-------------------------~-----~ -- --,----,

- ---- -~------,--

607

--~-

5 Physh:s

Listen to a lecture in a physics class. i@l!f*il

.I
:.

608 Practice Test

24. What is the lecture mainly about?


(A) How pulleys can be used to gain a
mechanical advantage
(B) Why compound pulleys are more
effective than others
(C) The parts of different types of pulley
machines
(D) The kinds of simple machines used
for redirecting force

28. What is a mechanical advantage?


(A) The extra force it takes to operate a
machine
(B) The combination of fixed base and
movable pulleys
(C) The extra force which makes it easier
to perform a function
(D) The amount of friction exerted on the
rope in a pulley

25. How is the lecture organized?

29. Based on information from the lecture,


indicate whether the statements describe
a fixed based pulley, a movable base
pulley, or a compound pulley. Place a
checkmark in the correct box.

(A) The professor gives examples from


the students' lab experiment.
(B) The professor describes how each
simple machine works.
(C) The professor explains pulleys he has
personally used.
.:(D) The professor quizzes the students'
)
knowledge of pulleys.

Fixed
base
pulley

~i

t~;:

26. listen again to pail of the lecture. Then


answer the question.
Why does the professor say this: (J
(A) He wants students to draw a picture.
(B) He wants students to imagine a scene.
(C) He wants students to look at an
illustration.
(D) He wants students to pay closer
attention.

27. What is the student's attitude toward fixed


base pulleys?

(A) She thinks they only apply in a few


situations.
(B) She thinks they are too. complicated
to be useful.
(C) She is unsure of how they are
constructed.
(D) She believes they should not be used
very often.

.,,"'

~z

In

Movable
Compound
base
pulley
pulley

1-----Has an
axis that is
not
stationary
..._

'

Has the
most
potential
for
mechanical
advantage

---------1-------r------..--1 ----------
The most
basic kind
of pulley

J
J

-~----

--------

Practice Test

609 - -

'

D6 Qffice Hours

Listen to a conversation between a student and a professor. l@ff}j

'.1

-~

,,

,J

- - 610

Practice Test

I
-z

G'l

ao: Why does the student visit the professor?


(A) To ask to leave class early on Friday
(B) To find out why he is being punished
(C) To request an extension on a class
essay
(D) To ask for permission to retake a
chemistry exam

31. Why does the student mention his .


chemistry class?
(A) To explain why he cannot
complete his paper
(B) To provide an example of how to
solve the problem
(C) To show the professor that he will
work hard
(D) To suggest that the professor change
her
,,,, policy
df
'

33. What is the professor's attitude toward


the student's problem?
(A) She is sympathetic and wants to help.
(B) She does not think it is legitimate.
(C) She is frustrated with his lack of
motivation.
(D) She believes that he will try his best
anyway.

~
[I

34. What does the student imply when he


says this:

(A) He might not submit the paper on


time.
(B) He is happy with the professor's
solution.
(C) He will focus all of his time on the
paper.
(D) He will write the paper before he
studies for his exam.

;~;{,

Lister)i'i.gain to part of the conversation. Then


answef the question.

Why does the professor say this:

(A) She thinks he had the resources to


plan better.
(B) She knows that he has not studied
enough.
(C) She is irritated that he did not pay
attention to him.
(D) She wants him to get a high score on
the exam.

Practice Test 611 _ _

[
:.am. .Mdi~'tt s
;,.~~.~~~!n.~:,_; . ;r':ff:.u.~.~:z;:.;!;,~~!.i~~1;~.

Speaking Section
Directions
In this section of the test, you will demonstrate your ability to speak about a variety of topics.
You will answer six questions by speaking into the microphone. Answer each question as
completely as possible.

_j

In questions one and two, you will speak about familiar topics. Your response will be scored
on your ability to speak clearly and coherently about the topics.
In questions three and four, you will first read a short text. The text will disappear, and you
will then listen to a talk on the same topic. You will be asked a question about what you
have read and heard. You will need to combine appropriate information from the text and
the talk to provide a complete answer to the question. Your response is scored on your
ability to speak clearly and coherently and on your ability to accurately convey information
about what you have read and heard.
In questions five and six, you will listen to part of a conversation or a lecture. You will be
asked a question about what you have heard. Your response is scored on your ability to
speak clearly and coherently and on your ability to accurately convey information about
what you have heard.
You may take notes while you read and while you 11isten to the conversations and lectures.
You may use your notes to help prepare your responses.
Listen carefully to the directions for each question. The directions are not shown on the
screen.
For each question, you will be given a short time to prepare your response. A clock will
show how much preparation time is remaining. When the preparation time is up, you will
be told to begin your response. A clock will show how much time is remaining. A message
will appear on the screen when the response time has ended.
,i

If you finish before the allotted time, press Continue to go to the next question.

L
I

-~=~~"~"=~~=-=----------

~
--"~-~~-~"~~-~~"=~~~====~~~~"'""!

Practice Test 613

Question 1 @t+t

11~- ~
f:

:_Spe,a~;'.,~ .- , f ,-J:~~~~~/.-:.

[g!j,~);ili<~ . ~1m;

m<i:ti

Describe the most memorable teacher that you have ever had and why that teacher was
memorable. Include specific details and examples to support your answer.

00:00:00

_J

Preparation time: 30 seconds


Response time: 60 seconds

614 Practice Test

l-

,,Jj
I

Question 2 Mffi:+

Ill
"'O

z
en

Is it better to research on the Internet or in books? Include specific reasons and examples
to support your answer.

~z

en

Preparation time :

~.

Preparation time: 30 seconds


Response time: 60 seconds

Pructice Test

615

~~-

Question 3 Wt+ii

The university's Student Athletics department has announced its plan to update the
exercise facilities. Read the announcement about the plan. You will have 45 seconds to
read the announcement. Begin reading now.

l,.

'

Reading time: 45 seconds

'J'I
Student Athletics Announcement
Student Athletics is proud to announce that we will be updating the Scaife Hall
exercise facility over the course of the coming months. Much of the existing exercise
equipment has become old and overused. It has become clear that the old equipment
needs to be replaced with new equipment as soon as possible. Second, our plan calls
for an expansion of the facility, making it more suitable to meet the needs of growing
student demand.

i.
~

;!

i.'

;<---- 616
~-I

'

Pr<Ktice Test .

n Now listen to two students as they discuss the announcement.

~
;I
.1

Prnctice Test

-.- .. -.v=

--

---~ -~---.

--

617 ...........--

. ~.1
The man expresses his opinion regarding the announcement. State his opinion and
explain the reasons he gives for holding that opinion.

:l ' p'reparation
time
"
'

'

'

'

Preparation time: 30 seconds


Response time: 60 seconds

---~

..'

,,

'

;~,,.:r~- 618 PractkeTest

'

,., I

rm11m+

Now read a passage about impression management. You will have 45 seconds .to read .
the passage. Begin reading now.
Reading time: 45 seconds

Impression Management
Impression management refers to .the behavioral str~tegies individuals use to control
how other people see them and how they want to be seen. First, an individual can present
himself in a manner that adjusts to how people think he should act. This adjustment can
often occur when the individual experiences a new situation or associates with certain
people. Second, an individual can present himself in a manner that creates an impression
. i~ the minds of a specific audience. Therefore, he alters his behavior depending on the
,;effect he wishes to have on each group.

Practice Test 619 -~,.

n Now listen to part of a lecture on this topic in a sociology class.


\L;li

i:iilJ

v, .

"'

~z

G>

Using points and examples from the lecture, explain how the. behavior of a computer
store manager demonstrates impression management.

)
I.- ". .<t// oo

. oo oo' ;.,,: .I.

Preparation time: 30 seconds


Response time: 60 seconds

i.

'

Pra(tice Test

621

---1

Question 5 t1111+

.'!;4J:;~,, ~ ~~!.
:-'.,,.: :.-o,

' . "". t

::.m ,.'

n Now listen to a conversation between two students.

'

l!

Ii

'
i"!

Zl:il
'"11

I
/i

,.H'

J
!

j
lI

'
'~.'

:~

'i''

_(?'

1"

'

5
t

622

Practice Test

.."'
~

G>

The students discuss two possible solutions to the woman's problem. Describe the
problem. Then state which of the two solutions you prefer and explain why.

3z

G>

Ol'!ll

iliii::I

Preparati.on time

Preparation time: 30 seconds


Response time: 60 seconds

;;;:;,1

ilifil

F.

Practice Test 623 -

Question 6
~.
. s~~~~ihg_:~ ~ .

n Now listen to part of a lecture in an environmental science class.


il

.c.211

i
1

I
II

~~"-"~~=-=======~:II

-- 624

f>ractice l'est

' . .. ............

t--~

"'

Cl

I
Ij

Using points and exampies from the lecture, explain why the wolf is a keystone species,

'1.

00:00:00

Preparation time: 30 seconds


Response time: 60 seconds

J
~

Practice Test

625 - - -

'--~--

''_-1L':"4t

Speaking Section
Directions
In this section of the test, you will demonstrate your ability to speak about a variety of topics.
You will answer six questions by speaking into the microphone. Answer each question as
completely as possible.

ii/fl

Jill

In questions one and two, you will speak about familiar topics. Your response will be
scored on your ability to speak clearly and coherently about the topics.
In questions three and four, you will first read a short text The text will disappear, and you
will then listen to a talk on the same topic. You will be asked a question about what you
have read and heard. You will need to combine appropriate information from the text and
the talk to provide a complete answer to the question. Your response is scored on your
.: ability to speak clearly and coherently and cin your ability to accurately convey information
about what you have read and heard.
In questions five and six, you will listen to part of a conversation or a lecture. You will be
asked a question about what you have heard. Your response is scored on your ability to
speak clearly and coherently and on your ability to accurately convey information about
what you have heard.
You may take notes while you read and while you listen to the conversations and lectures.
You may use your notes to help prepare your responses.
Listen carefully to the directions for each question. The directions are not shown on the
screen.
For each question, you will be given a short time to prepare your r!'lsponse. A clock will
show how much preparation time is remaining. When the preparation time is up, you will
be told to begin your response. A clock will show how much time is remaining. A message
will appear on the screen when the response time has ended.
::

~~""" '.f_~-ufi:is:~~e:~re the allotted _ti:~~~-~s--C~:~i~ue to go to the next question.


Practice Test

627 --
'

- ------------------- -----------

---

- -- -

----~

Question 1

i?!bf'*''

'

< ~' .:::,::~~..:~-II:

~j.~.~.~)lli
..:11

Where is your favorite place to go and relax? Why do you like to go to this place? Include
reasons and examples to support your answer.

..Ji
[

o0 :o?._:_o_o_~

I
Preparation time: 30 seconds
Response time: 60 seconds

l-~="c~~--~~~,~~===-----

. - 628

llrcictice Test

.-~J

f.:

[
[

i@flJj

~Is it better to live with a roommate or live alone at university? Why? Include specific
reasons and examples to suppcrt your answer.

00:00:00...1
Preparation time: 30 seconds
Response time: 60seconds

Practice Test

629

--.i

Question 3 tmi'di

!.
'

The university has decided to install more lights on campus. Read the announcement
about the decision from the maintenance department. You will have 45 seconds to read
the announcement. Begin reading now.

'

Reading time: 45 seconds

"
Announcement from the Maintenance Department

Iii
I

l
i1

'l

ii

!
I

We wish to notify you that maintenance personnel will be installing additional


lighting around campus during the next couple of months. We have had several
incidences of theft and vandalism on campus recently. We feel that by installing more
lights, we can improve campus security by discouraging such criminal acts. The lights
will also allow students to take part in outdoor activities at night, such as basketball and
jogging.

I
1

--------~~~-~~"~~~===,~--~"-~------- ~~-~-~---~c~~-=-==--""'"~~-~-=-~--.-~--"'= -.......;o:Ji

;~

'

,<

,'--.--.,,

n Now listen to two students as they discuss the announcement.

Practice Test

631

~-~:>;>'g_~~

"'<

~~~,

!i ::s_p~aki.~9-::;

'.?> ;':

3.of 6

admx'"'i~,' ': : ._ ~- -.1~v.,IN:;.;:


The woman expresses her opinion regarding the announcement. State her opinion and
explain the reasons she gives for holding that opinion.

L_

..Ji
00:00:00

SI

Preparation time: 30 seconds


Response time: 60 seconds

i
I:

ill

_;,

,:.,r,__ ___
'

il.i..'---

632

Practice Test

"'
~

GI

@fJii

z~

GI

.!

'

Ill

'

GI

Now read the passage about point of view. You will have 45 seconds to read the passage.
Begin reading now.
Reading time: 45 seconds

Point of View

In the visual arts, point of view refers to the orientation of the viewer when looking
upon a work of art. Artists use point of view for two distinct purposes. All artists use
point of view to direct the viewer's gaze by choosing the position from which their subject
is observed. Possible points of view include from below, from above, from inside, or from
outside. Artists also use point of view to convey a particular meaning or attitude to the
viewer by emphasizing specific details or emotions.

Practice Test

633 ---~l

n Now listen to part of a lecture on this topic in an art class.

l'

'

;)

;/~~i.____

'

634

Practice Test

..

' 4of 6--_:,


d-3./)'-<)

Using the information you have just heard, explain how the portrait by Diego Velazquez uses
point of view.

Ii '

oo: oo :.oa .

'I

Preparation time: 30 seconds


Response time: 60 seconds

Practice Test

635 - --"

Question 5 rmti:
Bil!mtllmf

"'s~ta<~

.._.~:~;:.,

:;:},;

n Now listen to a conversation between two students.


i

IJJI

=~---

"'c~

G>

c:

G>

1-i#li'!ti

.,,"'
~

"'.i5-Qf'6-!)1
{ ~'.' -/.~:{,_,.,),~

.tf:'

2S

G>

The students discuss two possible solutions to the man's problem. Describe the problem.
Then state which of the two solutions you prefer and explain why.

~
3z
G>

El
1i >00:00:00 . I
Preparation time: 30 seconds
Response time: 60 seconds

Practice Test

637

---i

Question 6 @fl@

~
' ._6 Pt.s--':.,

~;;:;; ;:,-5,.:.:'(;i

n Now listen to part of a lecture in an advertising class.


I

.JI

I
I

, - - - 638
'

----------

----~--

Practice Test
------------~------

-,---------~--~~~~--,,__=-~~

[
~i
60#6 /'l

,\,:>'._-;,..-<i.:~

Using points and examples from the lecture, explain how advertisers make products
seem more appealing.

'oo: oo: oo

'I

Preparation time: 30 seconds


Response time: 60 seconds

Practice Test

639 -

'

t~L

Tr-

[
[
[
[

o)l11ons

Writing Section

'

Directions
Be sure your headset is on.

G)

This section measures your ability to write in an academic environment. .


There are two tasks in this section. For the first task, there is a reading passage and a
lecture. You will write a response to a prompt based on what you read and hear.
For the second task, you will write a response to a prompt based on your own knowledge
and experience.
Now, listen to the directions for the first writing task.

Practice Test

641 - . (.
i

r.
I

}!-;IDW~W::<;~').~-~-.\l,\'U.~~f,:~"m<F=~~';f~~.B.fil'oU!l.~li~~~~'ilwm<";,

ldtfilhi

lliEl!D

;,,_Wi)-ti~'u_:\

' , ' : Optrons\ ~; '111 '

'

Te_strngTools , ' )d!J\

Pause ,,. 'Section_~


' -. ,Exit -

.f

.'(,

?i

. -:>

Integrated Writing Directions

',: J

.'

'

!
ji.

1:

For this task, you will have three minutes to read a passage about an academic topic. You
may take notes while reading if you wish. The passage will then disappear, and you will hear
a lecture about the same topic. While listening, you may also take notes.
You will then have 20 minutes to write a response to a prompt related to the relationship
between the lecture and the reading passage. Answer the prompt as completely as
possible using information from both the reading passage and the lecture. The prompt will
not ask you to express a personal opinion. The reading passage will appear again when it is
time for yo~ to start writing. You may use your notes from the lecture and the reading to help
you answer the prompt.
Typically, an effective response for this task will be 150 to 225 words long. Your response
Will be graded on the quality of your writing and on the completeness and accuracy of the
information you include in your response. If you finish your response before your time has run
out, you may click Next to go to the second writing task.
,.
~"

_. '.

Now, you will see a readihg passage for three minutes. Remember that the passage will
be available to you again while you are writing. Immediately after the reading time ends, the
lecture will begin. Be sure to keep your headset on until the lecture has ended.

"''

,.

.,,\---- 642

Practice Test

:...:...:----~~-~--~-.- - ..
'

'

",.

"\

"' >

. ,.

. ._:,.

i -

;=

n:=:-r.~'"''-r-''_,_,_,_:J,.-.-;hJ'~-n:1:r;;z=..t:>-:.>m;;.,n1

~;li~u~!A:
.!~, ,.,

p,;fZ;,:,

,~, ",, ':roo!~'

'!,,, , <

''
.,.,

-.>

--------'-Buzz marketing is a type of advertising


used to generate a high level of interest in a
-product. It gets people to spread the word about
a product just by talking about it. The goal is to.
encourage regular people to generate excitement
or "buzz" about a product and spread that
positive word of mouth throughout their social
networks.
. . A main advantage of buzz marketing is that
'fhe message spreads quickly and easily. A
rparketing team just has to design the message
f~ey want to send and get that message o'ut
fo' the public. A well-crafted buzz marketing
campaign will then take on a life of its own.
Members of the target audience will discuss
the product and generate excitement among
theirfamilies, friends, and acquaintances.
A second advantage of buzz marketing
is that people trust information gained through
buzz marketing more than through traditional
marketing. This is because buzz marketing is
seen simply as sharing information between
friends; therefore, it is far more comfortable and
believable than other marketing methods. With
buzz marketing, people 'learn about products
from someone they trust. As a result, that
information is taken very seriously.
Finally, buzz marketing is an inexpensive
way to make any traditional marketing campaign
more effective. Companies that have added a
buzz marketing component to their traditional
marketing strategy have reached more
consumers at a lower cost than those that rely
on traditional marketing alone.

643 --~-.j

'

Pr<1ctice Test

~~~~Jfu.~;i:;-B{<-;.w;.~~>'R."'.tU4~"'ll;~~~~~,,.,;.~w:;m::~:;mrnaw~~"""m!''*il6l'~'~"'"'';

-;ct

~""""""

llilmlm
W_rltlng_)

,'

, .1 of_7

() Now listen to part ofa lecture on the topic you just read about

,_,

tmf!ij

.. _,

::
,,
,.,
,_,
''

:,

I
I .

;, ,You have 20 minutes to plan and write your response. Your response will be judged on the basis of
\; the quality of your.writing and on how well your response presents the points in the le~t,ureand their
~ relationship to the reading passage. Typically, an effective re$ponse will be 150 225 Words J6hg:' .

to

~
3
.z

Prompt: Summarize the points made in the lecture you just heard, explaining how they
cast doubt on points made in the reading.

..

_,

Buzz marketing is a type of advertising


used to generate a high level of interest in a
product. It gets people to spread the word about
a product just by talking about it. The goal is to
E311COurage regular pecple to generate excitement
or "buzz" about a product and spread that
positive word of mouth throughout their social
networks.
..
A main advantage of buzz marketing is that
the message spreads quickly and easily. A
'njarketing team just has to design the message
they want to send and get that message out
to the public. A well-crafted buzz marketing
campaign will then take on a life of its own.
Members of the target audience will discuss
the product and generate excitement among
their families, friends, and acquaintances.
A second advantage of buzz marketing
is that people trust information gained through
buzz marketing more than through traditional
marketing. This is because buzz marketing is
seen simply as sharing information between
friends; therefore, it is far more comfortable and .
believable than other marketing methods. With
buzz marketing, people learn about products
from someone they trust. As a result, that
information is taken very seriously.
Finally, buzz marketing is an inexpensive ,
way to make any traditional marketing campaign .i
more effective. Companies that have added a
buzz marketing component to their traditional
marketing strategy have reached more
consumers at a lower cost than those that
rely on traditional marketing alone.
I

GI

~:._..o=.=,;.;-.._,;ov~"-"'-'-'--~~:'-';,_.-.;.,.

-- v --

------~~.--......~~

j'

_~
"'------:... -

-~-

- - -

- - - .--;: _ _ _ - - " - - - -

Practice Test

645 - .

---- - - - - - - -
...-...... .!!!!!!!!!!!!!!!!!!!!!!!!!!!!!!......llJll!I
.........lllllll!
.........11111111..1111111!
__..llllllll
____lllllllllm.......~

'Z:.J;n''.3<?;~.'L'I.'.tUil~~-~ ....?.'.V:~...."7\'i. ~"'8<'l'-"Yo"'lf'J-3't."<;~.':"'::!'l:~.C:t.W<1.'-i'!O>L'.~:'l.. ,.!'.l.';W',t-~'fJC~!Jl:>b11.-1.~Vt-:J:l\'.'."i:-\'-'.lfl.0CC:>'!:'l ':'.c~-,-"<>--~<Y~fH->~ 'i.v,:,~-q,~'->SJo'>::~>:-i.V!'> ,.

---

- -0 .-c <\ '. '

'

Independent Writing Directions


For this task, you will write a response to a question that asks you to present, explain, and
support your opinion on an issue. You will have 30 minutes to write your response to the
prompt.
Typically, an effective response for this task will be about 300 words long. Your response
. will be graded on the quality of your writing. Graders will consider various aspects of the
response such as the development of your ideas, the organization of the content, and the
quality and accuracy of the Jang uage used to express ideas.
If you finish your response before your time has run out, you may click Next to end this
section.
When you are ready to begin, click on the Dismiss Directions icon.

~<f.c<f~l?I

~~~~

]'t-~.;

. ~\\:Read the prompt below. You have 30 minutes to. plan,


write,
and revise your essay. Typically, an
:
.
.
: ~;!effective response will contain a minimum of 300 words.
~:\

("'

''

,.

-_

:,.

Do you agree or disagree with the


following statement? All university
. students should be required to
participate in exercise classes or sports.
Use specific reasons and examples to
support your answer.

\'<

~-

I
I
.1

---=o:..-===.o:.o;.;:;o.. :=_-~--'-

;-. __ .o:;.

.. l

1
=~=

Practice Test 647 -----.. j

'

[
[
[

:.

Cl

,-,-pa~S~ 'i se~~ff1i'":!


-n.:~

..-

-'"',-.--

,,:, .

Writing Section

i
'

'

.
I
'

Directions
Be sure your headset is on.

This section measures your ability to write in an academic environment.


There are two tasks in this section. For the first task, there is a reading passage and a
lecture. You will write a response to a prompt based on what you read and hear.
For the second task, you will write a response to a prompt based on your own knowledge
and experience.
Now, listen to the directions for the first writing task.
-

Practice Test

649 - -
i

_I-_
--- :____

____ _:__: __..

:___-

=-~fi.~~Vl<;;i~~>!:C"~JS'A~~-l<W~.fil'al\1~~..l'!~""="'""---=~~"'W"!'?

T>ll'lt/WJJmi~>IWL~~"M&.l<-J.:.,\oi~.',~!

'dM''''

Integrated Writing Directions


For this task, you will have three minutes to read a passage about an academic topic. You
may take notes while reading if you wish. The passage will then disappear, and you will hear
a lecture about the same topic. While listening, you may also take notes.
You will then have 20 minutes to write a response to a prompt related to the relationship
between the lecture and the reading passage. Answer the prompt as completely as possible
using information from both the reading passage and the lecture. The prompt will not ask you
to express a personal opinion. The reading passage will appear again when it is time for you
to start writing. You may use your notes from the lecture and the reading to help you answer
the prompt.
Typically, an effective response for this task will be 150 to 225 words long. Your response
will be graded on the quality of your writing and on the completeness and accuracy of the
information you include in your response. If you finish your response before your time has run
out, you may click Next to go to the second writing iask.
Now, you will see a reading passage for three minutes. Remember that the passage will
.be available to you again while you are writing. Immediately after the reading time ends, the
!i lecture will begin. Be sure to keep your headset on until the lecture has ended.
ji
J~wn>=;c....~-..=>=---..;;:;;_=-,=.n:=.;o~;;~~-,""'"''"'--'~~-,,,, .-,;A,---,;;_.;;;~---''~--r;'.,:;;-,-,"'''"'~'"""-'~'=u~--"''-'C-''"'"~-..::o.-,.,-;:;.<-~--- ..-

- - 650

Practice Test

- ----------------- ---- ------ --

- ---- - - - - - - - -

-1

I
1

=~,"~- ---~~-J
..

"'!;:
c

G\

r.
....

-z

G\

l[
[

--------'-----------~~--=-~=
f-'i.2;~:~;~::::~::J~~~il~\ll

Fluoride is a naturally occurring chemical


found in rocks, soil, streams, and often in the
water people drink. In. the 1930s, scientists
discovered that people who drank water
containing fluoride had better dental health
than those who drank water without it. As a
result, many cities in the United States began
fluoridating, or adding fluoride to, their water.
Since then, the practice has been largely
tieneficial to communities that implement it.
;W The largest benefit of fluoridating water
'ls it reduces incidences of tooth decay. Fluoride
li>oth prevents and reverses tooth decay by
:enhancing remineralization. Remineralization is
the process by which the tooth's enamel, or outer
coating, rebuilds Itself after it has begun to decay.
Several studies conducted since the 1940s have
shown that fluoridation reduces tooth decay
by eighteen to forty percent among children and
nearly thirty-five percent among adults.
Not only does ftuoridation improve dental
hygiene, but it is also safe. Fluoride in large
quantities can be toxic. However, these levels
must be very high for any harm to be done to
humans. Fluoridating water at optimal levels
does not harm people or the environment. The
United States Public Health Service supports
ftuoridation and considers it a safe and effective
method for maintaining dental health inAmerican
communities.
Fluoridation is also cost-effective. The
average cost for a community to fluoridate its
water each year is estimated to _range from
approximately $0.62 per person to about $3.90
per person. Over a lifetime, this comes out to
less than the average cost of one dental filling
to repair one decayed tooth. For most cities,
every $1 invested in community water ftuoridation
saves its citizens $38 dollars in dental treatment '
costs.

Prnctice Test

651 --:

--- - - .

=~~

llEm!l!l!I

'.;~ritin{f'{.

r$1\m:

c.':i of z' .:;

n Now listen to part of a lecture on the topic you just read about.

1fli&j

i
d

iJ

652

Practice Test

: - . lelllml
~f.~J~ing -: :, ;. }:~-f2~ '.roo_~-2~.~.o_~,-~,~::~

,'

;/ tools

> ,.,

:~~:~J:il~~filf~J ;
.

'

i You have 20 minutes to plan and write your response. Your response will be judged on the basis of
:.... the quality of your writing and on how well your response presents the points in the lecture and their
relationship to the reading passage. Typically, an effective response will be 150 to 225 words long.
Prompt: Summarize the points made in the lecture you just heard, explaining how they
cast doubt on points made in the reading.
Fluoride is a naturally occurring chemical
found in rocks, soil, streams, and often in the
water people drink. In the 1930s, scientists
discovered that people who drank water
containing fluoride had better dental health
than those who drank water without it. As a
result, many cities in the United States began
fluoridating, or adding fluoride to, their water.
Since then, the practice has been largely
beneficial to communities that implement it.
; The largest benefit of fluoridating water
is if reduces incidences of tooth decay. Fluoride
both prevents and reverses tooth decay by
enhancing remineralization. Remineralization is
the process by which the tooth's enamel, or outer
coating, rebuilds itself after it has begun to decay.
Several studies conducted since the 1940s have
shown that fluoridation reduces tooth decay
by eighteen to forty percent among children and
nearly thirty-five percent among adults.
Not only does fluoridation improve dental
hygiene, but it is also safe. Fluoride in large
quantities can be toxic. However, these levels
must be very high for any harm to be done to
humans. Fluoridating water at optimal levels
does not harm people or the environment. The
United States Public Health Service supports
fluoridation and considers It a safe and effective
method for maintaining dental health in American
communities.
Fluoridation is also cost-effective. The
average cost for a community to fluoridate its
water each year is estimated to range from
approximately $0.62 per person to about $3.90
per person. Over a lifetime, this comes out to
less than the average cost of one dental filling
to repair one decayed tooth. For most cities,
every $1 invested in community water fluoridation
saves its citizens $38 dollars in dental treatment
costs.
Practice Test

653 - - , ;

~'iia
-:!ofl
mg

!: l!i&lid11!
"

\v~!">;:,.~ii,~ -;~> ~~.-,~s;..~;0:-yJ!' \'hl~Vi',,~M<fl "il.lr.>11..

Independent Writing Directions


For this task, you will write a response to a question that asks you to present, explain, and
support your opinion on an issue. You will have 30 minutes to write your response to the
proropt.
Typically, an effective response for this task will be about 300 words long. Your response
will be graded on the quality of y0ur writing. Graders will consider various aspects of the
response such as the development of your ideas, the organization of the content, and the
quality and accuracy of the language used to express ideas.
If you finish your response before your time has run out, you may click Next to end this
section.
When you are ready to begin, click on the Dismiss Directions icon.

L~- -.................. ------------.-. . .-.. . . -.-. . . - - --. -

654

fl;'iiO.kt: ; .:st

:
l
.

'

:m mrmm ,- -,,,,rnw
';,;;

.__

~!;_.Writing

~ .

2of2

Ii,

_00:30_:00 l-lidEil _

Read the prompt below. You have 30 minutes to plan, write, and revise youressay. Typically, an
effective response will contain a minimum of 300 words.

Cl

Do you agree or disagree with the


following statement? All the knowledge
that people need can be learned from
books. Use specific reasons and
examples to support your answer.

Pra(tice 1est

655 ----,,

-- _. ,.,,, .;-,

, }oe ,.,__ .,, __ _

Note: Highlighting' indicates a repeated listening


sample.

01 Anatomy
W: Yesterday, we only started to talk about
chemoreceptors. Does everybody remember
what chemoreceptors are? Let me give you a
quick definition just to make sure that we're all
clear on this. OK, so chemoreceptors are the
group of cells that act kind of as translators for
chemical stimuli. So, when your body is exposed
to certain chemicals, the chemoreceptors
prompt your body to react, respond, or both.
Does that make sense? What I want to talk about
specifically today is the role of chemoreceptors
in two human sensory systems: the olfactory
and gustatory systems.
OK, then. First, the olfactory system is what
allows you to perceive smells. Smelling is
important. I mean, it gives us a way to tell if
there's food close by, or if food has gone bad.
Anyway, the chemoreceptors used for smelling
are called distance chemoreceptors. Humans
have millions and millions of chemoreceptors
that are located in the nose. If you want to get
really technical, the receptors are all actually
located in a certain part of the nose called the
epithelium. In humans, it's about two inches
long. It's actually by studying the epithelium
that scientists can tell if an animal has a good
sense of smell or not. Let's consider two
examples: humans and dogs. Humans have
about sixteen square centimeters of the
epithelium covered with chemoreceptors.
Dogs, meanwhile, have 150 square centimeters.
So what can we guess by that? That's right,
that dogs have a much better sense of smell than
humans. Anyway, when odor molecules pass
through the nose, they bind to the receptors.
The receptors transduce ... ah, right transduce.
When you transduce something, you change it
from one form to another. So, yeah, the receptors
transduce the chemical stimuli into electrical
signals that can be understood by the brain.
The brain can then interpret what the odor is,
tell how concentrated the odor is, or even
match up the odor with a memory of an odor
smelled in the past.

So that's how chemoreceptors work in the


olfactory system. In the gustatory system,
which allows us to taste, it's a little different.
Again, being able to taste is very important to
humans. Before we all were able to get our food
at the supermarket, humans used to have to go
out and hunt for food. The sense of taste
allowed humans to tell if a food was nutritious or
bad for them. Moving on, I'm sure you all know
that our tongues are covered with thousands of
taste buds. There are about 10,000 taste buds
in the human mouth. Scientists have learned
that dttferent buds are able to discern, or uh, to
figure out different types of flavors. Well, that's
actually where the chemoreceptors are found.
See, there are two kinds of cells that make up
each taste bud. The supporting cell usually
forms the outside of the bud. Then the gustatory
cell, which is the chemoreceptor, is found in the
center of the bud. Unlike the chemoreceptors
in the nose, the chemoreceptors found on the
tongue are called direct chemoreceptors.
There are five different kinds of receptors on the
tongue. Each one is able to perceive salty, sweet,
bitter, sour or savory flavors. Interestingly, each
receptor has a distinct way of communicating
with the brain.

()2 Art History


W: We've been talking a lot about the Spanish
Renaissance in art. You probably remember
that this movement dates back to the 1500s.
Today I want to give you a, um, a profile, I
guess, of one of the most well-known painters
of this period: El Greco.
M: Wait a minute, but wasn't El Greco kind of an
exception in terms of his ideology and style? I
thought that he wasn't even popular at the time.
W: That's absolutely right. But I bring him up
for two reasons: one, because he was an
extraordinarily gifted artist, and two, because
his work exhibits features that popped up_ in art_
later. So consider this a preview of sorts. Now.
you said that ideologically, he was very different
from his contemporaries. Why don't we talk
about that? What were some of the beliefs tr.at
informed El Greco's work?

658 Transcripts
"-

~.-.-c--,-

____ ,_ ___

-,-~.-.~--~

n,
It.
to
Id
10
le

:o
's
1..
p

ly
y

e
s

e
),

e
t,
h

1r
i.

s
1

I
)

1
3

't

Well, first of all, his work was mostly informed by


his imagination and intuition. And he thought it
was important to depict subjects in ways other
than what was expected.
W: Great point. I think that element is probably
best exemplified in the way he portrayed his
subjects as stretched out.
You know, I read something interesting about
that once, about the elongation of El Greco's
subjects. It was an article by this art historian
... he proposed that the reason that El Greco's
subjects looked all stretched out like that is
probably because he had astigmatism. Is that
true, or were the elongated figures really part of
El Greco's style?
W: Well, uh, for those of you who don't know,
astigmatism is a condition that affects the eyes.
Very simply put, it can cause objects to appear
more stretched out than they actually are.
Anyway, that's an interesting theory. I know it
seems like astigmatism could account for El
.. Greco's style, but if you think about it for a
.:;3minute, you'll see that it doesn't really make any
";sense. See, the astigmatism theory recognizes
,,,that the subject will be seen differently, butit fails
~to acknowledge that the artist with the astigmatism
would also see the canvas and the room and
really everything with the same distortion. Does
that make sense?
M: Um, yeah, I guess I can see that. But still, to me
it made a lot of sense actually looking at El
Greco's paintings. I mean, even if everything is
distorted it would still produce that effect.
wouldn't it?

W: Not exactly. Here, let me give you an example.


According to the astigmatism theory, the reason
that the subjects in El Greco's work are stretched
out is becil.use that is how he actually perceived
them due to astigmatism. But consider a work
where the subject is lying down horizontally. If
he were, in fact, just painting what he saw, the
subject would appear to be quite thick.
Instead, his subjects are never really elongated
horizontally; it's more like they're stretched out
vertically. That effect in his work is really just a
result of his desire to get away from the tradition
of having perfect measurements and proportions.

03 Service Encounter
W: Hi, I'm looking for someone who can help me
with my identification card.
M: I can help you. What's the problem? Is it not
letting you into buildings anymore? You know,
sometimes if you leave the card close to a magnet,
the card readers can't read them.
W: No, actually, I, um, seem to have misplaced my
card somewhere. But I'm almost sure that it's in
my room somewhere. I just don't have the time
to look for it now because I have a laboratory
session all afternoon. So I was wondering if I
could have a temporary card or something until
I find my own.
M: Oh, I'm so sorry, but we can't issue temporary
cards. It's just too risky to have that many ID
cards circulating around campus. Especially
since we use them for so many things. I mean,
think about it. You use it to get into buildings, to
eat at the cafeteria. Anyway, we'll just have to
get you a new one.
W: Oh, but I really don't want to have to pay for a
new one if I know that I haven't really lost mine.
M: I'm really sorry, but you'll have to purchase a
replacement. The bright side is that if this is the
first time you've had to replace your card, it will
only cost you five dollars. After that, it's ten dollars
if you have to replace it again.
W: See, the thing is, I've already lost it twice. So it's
ten bucks. It just kills me. And I know it's not
lost this time. It has to be somewhere in_ my. .
room.


M: To be honest, you don't have much of a choice.
Once a card. is. reported missing, we are
required by university policy to deactivate it.
Really, it's just what we have to do to protect
the students.
W: OK, I guess I can see that. What information do
you need to deactivate it?
M: Just your name.
W: It's Olivia Bentham.
M: Let me just put that in the system here. OK, Ms.
Bentham, your old card is now deactivated.
Now, let me just make you a new one and you
can be on your way.
W: Thanks. Um, so, it's ten dollars.
M: That's right. You can either pay with cash or put
it on your student account.
W: I'll just pay for it now. My parents will freak if
they find out I've lost it again.
Transcripts 659

- - --

-----------------

01 Linguistics
M: I wanted to talk a little bit about language
acquisition. As you all probably know, learning
a new language is a process that is not yet
completely understood. Linguists examining
language acquisition have formulated several
theories about it, including one called the
transactional model. This model will be our
topic for today. Let's start off by defining it.
OK, so first, the transactional model emphasizes
the effect of the language environment on the
learner. Right. This is a huge part-let me
explain. It means that a learner of a language
will be affected by things that he or she is
exposed to-different sounds, words, and so
on. The most obvious example might be children
who learn the same language in different parts
of a country. Depending on the environment-in
this case, geographical location-they might
develop different accents.
The transactional model can also work in the
opposite direction, which is my second point. It
also states that the learner can manipulate his
or her environment. For example; um. one child's
habit of oh, using a certain word or phrase,
might be transferred to the other students in the
same class, which is the children's shared
eQvironment. Then, soon, all of the students .in
the class are using this word or phrase, and it
becomes habitual behavior in that environment..

02 History
W: Today I want to continue our lecture on
Alexander the Great. As you already know,
Alexander was a Greek military leader who
conquered much of the known world in the
mid-4lli century BCE. An interesting thing is,
aside from all the territory he conquered,
Alexander the Great also contributed significantly
to the spread of Greek culture.
Alexander was legendary for his policy of
fusion. What I mean by that is that he promoted
the assimilation of foreigners in the army. Hmm.
I think I'm still confusing you all. Basically, he
was more welcoming of people from other
i

\!__

66ff Transcripts

countries than many other rulers in history tended


to be. For example, he would have his Greek
generals marry noblewomen from Persia. He
allowed his own culture to combine with cultures
from around the world.
Another way that Alexander helped spread
Greek culture was by his, uh, his sheer notoriety.
I mean, the guy was practically a superstar, as
famous in his day as some of the musicians you
might listen to today. When you consider that he
was undefeated in battle, it's easy to understand
why he had fans around the world. One good
example is the Romans. It's said that the Romans
really esteemed Alexander and sought to
associate themselves with him. One way we
can see that is that the Romans all prided
themselves on speaking Greek. In fact, it was
the unofficial language of the academics.

:~

03 Office Hours
W: Excuse me, professor? I was hoping you had a
second \o talk about class.
M: Hi Gracie, I'm happy that you came. I've been
curious about the students' sentiments toward
the class. So what's on your mind?
W: Um, well, honestly, I guess I'm just having a
problem with the setup of the class. I feel like
you don't give us the information we need during
class, and that we're, well, uh kind of wasting
time with the group discussions.
M: What do you mean, exactly?
W: Well, it just seems like every time I'm in a group
discussion, nobody's ever really sure what
we're supposed to talk about.
M: Butthe purpose of the group discussion is to
see what ideas you all have. I figured nobody
wanted to hear me go on and on by myself.
W: No, and I can apprectate that, but ... I don'.t
know. I mean. you are the authority.
M: Huh, and here I was thinking that I was probably
boring everyone to death with the lectures.
W: No, definitely not. I mean, I'm pretty interested
in hearing what you have to say.
M: That certainly gives me something to think
about. Thanks for your input.
W: No problem. And I want you to know I reaily do
enjoy your class. See you later!

.:.

<'-'

04 Service Encounter

W: Hi, did you call about a problem with your


Internet connection?
M: Wow, that was fast. Anyway, yeah, I just can't
seem to connect to the campus network. I've
tried just about everything I can think of, but
nothing seems to work.
W: Let me do some quick troubleshooting. Let's
see, well, it looks like your hardware is all in
working order. Did you get your network password
already?
M: My network password? Oops, I didn't even
know about that.
W: A lot of incoming students don't know about it.
Basically, it's just a way for the Campus
Computing Department to track who's on the
network. You know, to make sure that only students
can <recess the network.
M: Gotcha. Well, no, I definitely don't have one of
those. How do I get one?
W: Normally, you'd have to go to the computing
, 1; building, but since I'm here, I can walk you
ttc through it. First, open up your browser and type
"University Log-in dot e-d-u" in the search bar.
1t: Then click on the link that says "Get network
password."
M: Um, I don't see it anywhere.
W: It's on the lower left-hand side. There you go!
Now all you have to do is follow the directives
on the screen.
M: Wait, and after I finish up filling in this information,
it will give me the password?
W: No, this is just going to set up your campus
profile. It's their way of verifying that you're
actually a student. After this is another step.
M: Oh, I get it. So now that I have my campusprofile
approved I have to ...
W: You have to check your campus email address.
M: OK. Oh look, there it is. Thanks a lot for your
help.

05 Literature
W: So, last night I had you read some poems by
Emily Dicl<inson. I imagine that by now you've
gotten a good sense of her style and how different
it is from other writers' of that period. Today, I
want to talk about those differences and how
Dickinson sort of blazed the trail for other writers

after her. Can anybody start us off with how,


stylistically, Dickinson was different from her
contemporaries?
M: I guess I would say that her style is characterized
by the use of dashes everywhere and um, like
weird capitalization in places you don't really
expect it. After reading some other female
poets of the period, I found it to be quite a
departure from the standard punctuation
practices of the time.
W: You're right. One big difference is that Dickinson
didn't follow the accepted rules of punctuation.
Dickinson used dashes in her poems to connect
ideas that weren't always complete thoughts.
Oh, and speaking of complete thoughts, uh, I
guess I should mention grammar, too. See,
she-Dickinson-didn't always use complete
sentences, either. She used a lot of sentence
fragments and sometimes even used sentences
that only consisted of one word. Other poets
focused on language that was very clearly
grammatical and flowed much better than
Dickinson's.
Another thing you'll notice that's pretty different
are the themes that Dickinson covered. If you
remember reading the other poets of the period
a couple of weeks ago, they were nothing like
Dickinson. All the other poets from that period
seemed to talk about religion all the time. But
Dickinson would talk about the importance of
the individual. Most of her poems were very
personal, very emotional. She definitely focused
on the importance of the self. And it is refreshing
to see topics outside of the standard'-someone
writing about different subjects than other writers
of that pt;iriod.

06 Medical Science
M: Today, I want to talk about infectious diseases.
Just to be clear, an infectious disease is a disease
that is caused by any number of pathogens-er
I'm talking about biological agents-you
know, things like viruses, bacteria, parasites.
One question I've heard a lot over the years I've
been teaching this class is, "How did treatabl.e
diseases such as smallpox or even the flu krll
so many people in the past?" Well, let's answer
that.

Transcripts 66~

First, let's remember that we've come a Jong


way in understanding how diseases are
transmitted. Now we know that some diseases
can be transmitted through the air we breathe
or the water we drink. We now know that our
environment can affect our health. It's obvious
to us. That knowledge has helped us up our
standards as far as hygiene is concerned. So,
for example, it's widely accepted that the Black
Death that wiped out about two-thirds of Europe
was pretty much eradicated when people started
being more careful about cleanliness. Remember,
the Black Death is thought to have been a
bubonic plague, which was spread by rats and
the fleas that they carried. When people learned
about this, they were able to make changes in
their environment to avoid contracting the disease.
Another important factor in the mortality rates of
infectious diseases is the lack of immunity within
certain groups of people. First of all, do you all
get what I mean when I say mortality rates? OK,
all that means is the number of people that die
from disease. Anyway, the lack of immunity can
occur for many reasons. Take the American
Indians, for example .. who simply had never
been exposed to diseases like smallpox, which
was, of course, very common in Europe at the
time of the first settlers.

07 Office Hours
M: Excuse me, professor. Do you mind if I ask you
a question about the course outline?
W: Hi, um ... I'm sorry, I'm still trying to remember
everyone's name. Is it Paul?
M: Right. It must be hard trying to keep track of
everybody, huh? Anyway, I was wondering
about the course outline ... see, um, here it
says that we'll be working from the third edition
of Classical Mechanics. Is that right?
W: Um, yes, that's correct. If you're wondering why
I didn't assign the most recent edition, it's a
purely economic .decision. The editions are
virtually identical, and I don't think it's right to
have students pay fifty percent more for the
newest edition if it's all the same anyway.
M: I appreciate that. But the reason I was concerned
is because that's the same book that Professor
Maggart uses in his introductory course. So I
guess I just wasn't sure about how much material
will be repeated from that course .
.,r,y- 662

Transcripts

W: Really? I don't understand. Why would he use


that book? There are much more comprehensive
basic textbooks for an intro course on electricity
and magnetism.
M: I think that's a different course. I'm talking
about Foundations of Mechanics, or something
like that. It's a prerequisite for just about all of the
upper-level courses in the physics department.
That's why I know about it.
W: I guess that makes sense, then. You know, this
is the first time I've ever heard about us using
the same book. Do you remember what chapter
you got up to?
M: If I remember correctly, it was up to chapter six,
which would be, um ...
W: Calculus of variations. Well, I'm kind of surprised
that that's as far as he took you all in a survey
course on classical mechanics. Especially since
it's a prerequisite for the department. But anyway,
as for your question, unfortunately, there will be
some overlap. I can tell you right now that we'll
be blowing through the first six chapters within
the first couple weeks of class. I'll mostly just
.review that material, especially now that I know
that everyone in the course is supposed to
have taken an entire semester on it.
M: OK, and if that's the case, would it be safe for
me to assume that we'll be covering more
advanced topics then?
W: Most definitely. My course will give you deeper
knowledge of those subjects. Is there anything
else?
M: Nope, that answers my questions. Thank you
so much. See you in class.
W: Bye, Paul!

08 Service Encounter
M: Hi, is this where I can change my meal plan?
W: Yes, this is the Dining Services office, so I can
help you with that. Why don't you take a seat?
M: OK, well, here's the thing. I'm a freshman, so I
don't really know what my options are. But I did
hear from some people that there's a deadline
coming up, so I figured I'd come in and try to
see what's going on.
W: Yes, there is a deadline. But why don't we siart
from the beginning so we make sure you're
clear on everything?

.,

M: That sounds good. I'm sorry for trying to charge


through stuff. I clearly don't have any idea what
I'm talking about.
W: Don't worry about it. I know this stuff gets really
bewildering. Anyway, you said you're a freshman,
right?
M: Right.
W: OK, so what that means is that, as per university
policy, you have to be enrolled in one of the
three meal plan options. All freshmen must be
on one of the meal plans. The one that all students
are automatically enrolled in is twenty meals
per week, and that comes with 200 dining
points. This is just the default plan for idl new
students.
M: Sorry, what are dining points for?
W: Oh, no problem. The dining points can be used
at all the campus eateries, excluding the
cafeterias, of course, since you use one of your
20 weekly meal vouchers in the cafeterias.
M: Oh, that's cool. But that sounds excessive for
me. I never even wake up early enough for
b($akfast.
W: Well then, you can try the second meal plan. It's
foyrteen meals per week, plus 150 dining points.
Aijd the last meal plan is seven meals, plus 70
dining points per week.
M: You know, I really think that seven meals per
week is probably enough for me. I don't mind
cooking for myself, anyway. Is there a big
difference in the cost of the plan?
W: There'.s certainly a_ difference. Many students
prefer to downgrade a little bit. And as for the
cost, each plan is $500 less than the next step
up. So that means if you go down to the seven
meals per week, you'll be saving $1,000 per
semester.
M: Oh, that would be great. I know my parents will
like that, anyway.

09 Geology
W: Good afternoon, class. Today we're going to
talk about landslide processes and types. I
know that for most people a landslide is a
landslide, but there are actually different kinds.
What they have in common is that they all
involve rocks and soil and other debris moving
down a slope. So, if there aren't any questions,
I'll talk a little bit about these three types.

The first type I'm going to talk about is creep.


Just in case you're wondering, um, I think your
textbook calls it downhill creep. It's the same
thing, Anyway, creep is when rocks and soil
move slowly down the side of a slope. This mostly
occurs because of gravity. This phenomenon is
responsible for the gentle, um, rounded slopes
you often see on hillsides. Creep is affected by
several factors, including the grade of the
slope and vegetation. Another important factor
in creep movement is water. Some water will
act like glue and help keep the slope fixed.
However, excessive water causes the creep to
move faster. Creep happens so slowly that you
can't really see it; however, if you take look at
the vegetation in an area, you can tell if it is
happening. For example, in regions where
creep has taken place, you'll find curved tree
trunks or bent fences.
OK, let's move on, then. The second type is
most commonly known as a mudslide, but
geologists prefer the term "debris flow." You all
probably think I'm being nitpicky about the
term, but they actually do refer to different
processes. We'll use the scientific terms.
Debris flow, unlike downhill creep, moves pretty
rapidly. It's pretty much comprised of the same
rock and soil materials; however, a debris flow
can be recognized by the rocky front, which is
called the head, and the slushy, um, concretelike tail end. As with creep, there are different
factors that cause debris flows. But excessive
water is usually to blame. What happens is that
the loose soil on the slope becomes saturated
and causes it to, well, just slip off.
Finally, let me talk a little bit about a debris
avalanche. It's pretty much the same process
as creep except that it moves extremelyfast. Just
like an avalanche of snow. So, yeah, this is a very
dangerous kind of flow. A debris avalanche
can travel several kilometers before losing
speed. It can also have devastating effects on
landscapes. For example, many areas affected
by debris avalanches are permanently altered.

Transcripts 663 :.._____:-,

'

10 Zoology
M: All right, now we've all witnessed animals playing
before. Seems like they're just having fun,
right? But there's a lot of evidence telling us
that animals play for explicit reasons. So let's all
take a moment to think of the possible reasons
why animals might play-uh, outside of having
fun, of course. We'll come up with some theories.
Any ideas?
W: Well, it's a learning tool, right? And considering
they're playing at fighting, wouldn't it be, in a
way, practicing to hunt other animals?
M: Great idea. Let me use wolves to support that
idea. OK, so, young wolves always engage in a sort
of play-fighting. In fact, mothers will encourage
this conduct. The mother will do this because
she knows that the cubs will need those skills
once they go off on their own. They will need to
be able to defend themselves against attackers,
and they will need to be strong enough to hunt
other animals.
W: Yeah, but I've studied wolves before, and I
thought that their fighting was real. I mean, they
would actually fight, not just play. Isn't that
true?
M: You're right. Wolves do fight each other. And
they do this for a few different reasons, but I just
want to focus on one, and actually, we're going
to bring the discussion back to play-fighting.
But I promise, I'll answer your question in more
detail.
W: Hold on. I think I might know what you mean.
Are you saying that when animals really fight,
they are just doing it for the same reason as
when they play-fight?
M: Not exactly. What I mean is that play-fighting
actually establishes a sense of l1ierarchy in the
pack. I realize that was a lot of information. Let
me slow it down. What I mean is that-and
make sure you write this down-animals use
play,fighting to develop a sense of place in the
group. You know the term "alpha male," when
one wolf is the leader of the group? Well, as you
know, wolves can't exactly vote for a leader like
we do. So instead, wolves use play-fighting.
When wolves fight with each other, they learn
which one is the strongest. That wolf will become
the head of the pack.
W: So, I still don't understand why wolves would
really fight then.

- - 664 Transcripts

-----

------- - --

- --

M: Well, sometimes there might be a dispute over


which one is the leader. Maybe another wolf
thinks it sl)ould be the alpha male. So it's the
same idea of developing the social structure,
but the fighting becomes real.

. Chapter 2

01 Sociology
W: In today's discussion of group dynamics, we're
going to focus on the very smallest group
possible-a group of two, formally known as a
dyad. Love affairs, marriages, and even really
close friendships are dyads. And t.here are two
characteristics that make these relationships
very special, um, unique. In fact, they're unlike
any other relationship.
The first thing that sets off dyads from all other
social relationships is that they are typically much
more intense than the others. Yeah, there's no
one to focus on, other than the other member, so
naturally, the relationships can be very intensedeep, strong, and often very passionate. What
does this lead to? Well, put simply, these
relationships tend to be the most consequential
relationships of our lives.
The next thing that makes these relationships
unique is that they are highly unstable. If both
members don't actively work on maintaining
the relationship, then the thing ceases to exist.
. I mean, think about it. What happens if a member,
of, oh, say the photography club quits the club?
What happens to the club? Well, probably not
too much. The group survives. This is not the
case in a dyad, right? If one person quits, the
group is finished.

02 Psychology
M: Let's go over some theories about the development
of panic disorder. Panic disorder is a disorderdisorder being a physical or mental health
problem, basically-and with panic disorder
you have inexplicable feelings of really intense
anxiety. Researchers believe that there's a link
between panic disorders and an unusual

'

e
p

a
y
D

s
e
1
)
)

1t
11

awareness of one's own body. Confused?


Allow me to explain.
During a study on the development of anxiety and
panic symptoms, a group of randomly selected
children were asked to guess what their heart-rate
was, and to complete questionnaires about
their anxiety. It was found that children who
were better at approximating their heart-ratesyou know, guessing whether their. hearts were
beating too fast or not-they w,ere more hypersensitive to physical manifestations of bodily
malfunctions-um, let me put that another way. It
seems as though being very aware of your
body, such as being able to accurately guess
your heart-rate, sets you up to be more prone
to anxiety. If you're constantly worried about
how your body is working ... well, I think you
can understand what would happen.
So, OK, what if that's true for heart rate? What
about when experiencing pain? Some people
have pain and just bear it and brush it off.
Some people, however, feel like their bodies
41: are being attacked and start to experience
"' anxiety. These people tend to lead very
psychologically unstable lives-um, I mean
ii that
since they are worried that their bodies are
not working right, they start to develop panic
disorder.

M: Well, I thought I did that, and I was pretty sure


that I took everything in. It didn't quite hit me
until I received my test grade.
W: Well, come see me before the final exam so I
can clear up the main points. I certainly don't
expect you to remember everything I say. But I
absolutely don't mind going over what to
expect.
M: That sounds great. Thank you.
W: Sure. I'm usually available in the afternoons, so
come see me any time you're uncertain about
something.

04 Service Encounter
M: Excuse me, ma'am, I'm looking for the National
Journal of Environmental Science, but I just
can't seem to find it.
W: OK, well, have you looked on the shelves?
M: Yeah, I've looked up and down. I didn't see it at
all.
W: Hmm, maybe it's checked out. Let me look it
up. Ah, no, it's not in our system. We must not
subscribe to that particular journal. Let's see,
the National Journal of Environmental Science,
right? No, we don't have it.
M: Really? I need an article from it to use in a
research paper. Is there any other way to get it?
Hmm. Have you tried the interlibrary loan system?
M: No, I've never heard of it. What is it?
W: It's a system that we have that allows us to borrow
materials from other libraries.
M: Really? But how much does it cost?
W: We do it for free. I think most other libraries do,
too.
M: That's great. OK, so how can I get this journal?
W: Well, we can either get it from a university
library or a public library. It'll be faster from a
university library, but there's usually more
demand for things from university libraries. As
for public libraries, you'll be more likely to find
it in stock, but they can take up to a month to
get it to us.
M: Yeah, that's not good. My paper's due in three
weeks. How about the other university libraries?
W: Let me search through them. OK, I see that,
actually, one of our university libraries does
have it, except it's on reserve for the next couple
of days. So let's make a request right now. That
way, you can make sure that you're given first
priority as soon as it's released from reserve.

w:

03 Office Hours

W: Hey John, how are you today?


M: I'm good. Actually, I need to ask you a question.
W: Sure, what's on your mind?
M: Well, um, I'm really concerned about my
performance in class. I feel as though my
grades haven't been as good as I'd like them to
be, and well, I was hoping to ask you what I can
do to bring them up.
W: Sure. We still have the final exam, so doing well
on that is your best bet. Well, let's see, let me
get the grade book. Hmm ... OK, so I see that
you got a seventy-eight on your midterm exam

M: Yeah, even though I studied and studied for it.


W: I always suggest studying smarter rather than
just studying more. Study only the large, main
points of the book and my lectures. Oh, and of
course on top of that, make sure you're taking
the suggestions I make in class about what to
study.

Transcripts 665 - -

M: Definitely. Thank you so much. I really don't


want to miss the opportunity to get it.
W: Sure. Just fill out this form with your name and
the name of the journal.

05 Botany

:I1

:I

,,1!
.,I.i
[,

'

f:

.j--

W: Today we will be discussing two widespread


diseases that roses are vulnerable to. Gardeners
know how difficult roses are to take care of
since, well, they can be susceptible to va.rious
fungi and bacteria. We'll talk about these rose
diseases today.
The first disease we will discuss is black spot,
caused by a fungus. It's one of the most severe
diseases that affects roses.
M: Let me guess, it causes black spots?
W: How did you guess? It can be detected by, uh,
the characteristic development of round black
spots .which spread out throughout the leaf,
changing the rest of the leaf into a ... yellowy
color. It causes the affected leaves to fall off
prematurely. If nothing is done to control black
spot, well, then the entire plant becomes weak
and won't blossom as well. It's worse after wet
and warm times of the year. So, the best thing
to do is keep it, keep the plant as dry as possible.
Preventative maintenance, basically.
Now then, onto to the next disease-rose rust is
also brought on by a fungus. Any guesses as to
what this one does?
M: Well. I don't think it actually causes rust, since
roses aren't made of metal. So, um ...
W: Actually, rose rust causes orange spots to
show up on the leaves and stems of the plant.
So it looks like rust. II the rose is badly affected,
then an orange dust will appear not only on the
plant but also on the ground below it. Usually
brown growths appear underneath the leaves
and the leaves become twisted. After that, they
die and drop oft. Like black spot, wetness is the
rose's worst enemy. As for preventative measures,
it's best to make sure the plant stays dry. So,
watering in the evening is out, and uh, well; just
try to control the humidity around the plant as
much as you can. So, with that said, are there
any questions?

666

Transcripts

06 Anthropology

W: The environments where we build our cities are


not always the best of places. You know, the
environment may be~ too cold, too hot, too dry
or whatever. But humans have the incredible
ability to adapt to their environments, and uh,
more to the point, to make these environments
adapt to them. That's my claim, and to uh, to
back it up-yes, I'd like to illustrate this key concept
by talking about a place called Tenochtitlan.
Tenochtitlan was the capital city of the Aztecs
in what is now Mexico.
'
To begin with,_Tenochtitlan was originally built
on an island. Solid ground. But with the expansion
of the population to around 200,000, they could
no longer lit on the island. There were just too
many people. Now, they were actually surrounded
by a shallow lake, along with swampland, and
it's not so easy to build in such places. But
these ancient people adapted. They built what
we call floating gardens, which were made by,
well, just by piling tree branches and mud
together. Then they planted trees, and the tree
roots actually kept the islands in place. The
islands were anchored by the roots that grew
and secured themselves to the lake floor. They
basic'ally made solid land where there wasn't
any. Innovative idea, huh?
Another problem they had was, um, well there
wasn't enough fresh water in the city. Again,
the people adapted the environment to their
needs and overcame the problem. Now. th~re
were sources of fresh water away from the city,
but retrieving the water was difficult. So they
built aqueducts. Aqueducts are structures that
are built to carry water to a place-manmade
channels for water, basically. With so many
people to provide water to, the Aztecs built two
aqueducts that allowed fresh water to travel
from springs, past the salty lake water, and into
the city. These aqueducts helped the Aztecs
have fresh water in a place where it was hard to
find naturally.

.,

;
!

.)

07 Office Hours
ire
1e
lry
1le

M: Hi Kate, how's it going?

W: That's exactly what I wanted to hear. What a


relief; I really, really wanted that class.

W: Pretty well, thanks. I hope I'm not bothering

lh,
its
to
pt
.n.
:s,

in
Id
JO

Id.
Id
ut
at
y,
id
1e
1e

you.
M: No. I have time right now. What's up?
W: Well, I took your class on public policy last
semester ...
M: Yes, I remember.
W: And I see that you're teaching two other courses
in the political science department this semester.
I went to both classes, and I'm very interested
in both of them. However, my schedule is so full
that it won't permit me to take both, so I have to
choose one of them.
M: I see.
W: I was hoping that you could help me decide. So
one of them is "Introduction to International
Relations" and the other one is "Representations
of Race and Politics." Both of them look greatright up my alley.
M: OK. Now, what year are you in, Kate?
W: l!fn actually only a freshman; this is my second
semester.
M: Well, the second course is somewhat advanced,
I usually recommend that for either juniors or
seniors.
W: I thought about that, but I also know that courses
in political science are always changing, and
I'd like to take this course before it's no longer
offered.
M: -OK, fair enough. The International Relations course
is fcir those who are more business-oriented.
Are you interested in working for an international
corporation after you graduate?
W: Maybe. I'm not sure.
M: Well, tell me, what is it about political science
that got you interested in the first place?
W: I've always been interested in the weys that
various policies affect minorities, and I'm just
realizing how much they affect me. I think that
this field opens up an entirely new world for me.
M: Interesting. Well, like I said, "Representations
of Race and Politics" is a bit advanced and I
usually wouldn't recommend it to a freshman.
But in your case, I think you can probably handle
it. You're just beginning to develop a strong
awareness of your identity in relation to society
and its constructions of race, and perhaps the
best way to dig into it is by studying it. You
could also bring your personal experiences to
class and allow others to learn from you.
;J'>.)

sp

w
1y
1't

e
n,
1ir

e .
y, .
1y
~t

le
1y
'O

91
:o

:s
:o
-'.i

08 Service Encounter
M: Hello, can I help you?
W: Yes. You see, I'm all set to start school as a
freshman this fall. I got accepted on the WOW
Scholarship.
M: Congratulations!
W: Thanks. But I just received a bill in the mail. It's
for $200. And I have no idea what the bill is for.
M: Hmm ... Let me just type in your information
here. What's your name?
W: Melanie Woods.
M: And what is your student number?
W: Oh, hang on. I've got it written down. It's down
her.e in my purse somewhere. Here it is. My
number is 33569762.
M: OK. Yes, your account says that you owe $200.
W: But my scholarship is supposed to cover my
tuition costs and my books. That's what the letter
said.
M: What about your student fees? Does your
scholarship pay for those as well?
W: What student fees?
M: Well, many classes have an additional fee. For
instance, if you're taking a lab class, there's
usually a lab fee in addition to the tuition cost.
In fact now that I look at it, your account does
- say th~t this $200 is for student fees.
W: Why didn't they tell us that in orientation? This
bill is due at the end of the week, and I don't
have enough money.
M: I'm sorry, but you'll have to pay in order to take
your classes, or you'll be disenrolled.
W: What am I going to do? My parents don't have
the money to loan me $200. Is there any way
that I could get an extension until I get paid?
M: I'm very sorry. We aren't allowed to grant
extensions. You'll have to pay the bill by this
Friday.
W: Gosh. Well, I could ask my grandparents for the
$200 this time, but I don't know how I'm going
to pay for class fees in the future. I can't afford
to pay a couple hundred dollars for class fees
every new semester.
M: Have you ever considered applying for another
scholarship? That could certainly solve your
problem.

Transcripts 667 ~

I
-:-_--o-~="'=----~-

--~~

-------------------------------------.
W: You mean I can apply for another scholars_hip?
I thought that students could only apply for one.
M: Of course not. I know many students who
have used more than one scholarship to get
through schooL There's no limit on the number
of scholarships you can apply for.
W: Wow. I didn't know that. I only need a couple
hundred dollars each semester to pay for class
fees. Do they give out scholarships for that
small an amount?
M: I think so. You'll have to research them to know
for sure. I'd suggest looking for scholarships
online. You should be able to find what you
need on the Internet.
W: I'll do that. Thanks for your help.

_ _ 668

Another benefit is that absolute monarchies


don't require elections. Now, it may seem like a
bad thing that the citizens wouldn't get to choose
who governs them. But that's not necessarily
the case, especially in the past. You see, much
of the population used to be illiterate-they
couldn't read or write-and this would clearly
make elections difficult. What's more, such
people would have a hard time learning about
government in the first place. With an absolute
monarchy, the population doesn't have to worry
about elections, voting, or any government
issues. The new ruler is generally decided by
succession-the son of the current king, for
example-and laws are made by the monarch.

09 Political Science

10 Music History

W: A monarchy is a form of government in which a


single person has sovereign power over the
entire state or nation. Now, absolute monarchies
are monarchies in which the person in power
can rule completely as he or she wants, without
facing any opposition from the people, or even
from legal forces. This deal sounds great if
you're the person in power-not so great if
you're one of the people being ruled over. And,
well, I can read your minds ... you're probably
thinking that this method of governing is bad,
really bad. Always really bad. But you know
what? You'd be wrong. Actually, absolute
monarchies have some distinct advantages.
One good thing about an absolute monarchy is
that, well, with a good leader, a strong leader,
that leader can do great things for the country
because that person has the power to do anything
they can think of. For example, Catherine the
Great, the Empress of Russia between, let's
see ... 1762 and 1796, pushed for absolute
power. And regardless of what you think of her
personally, she actually did some great things
for Russia. Like, uh well, for one, she strengthened
the middle class and tried to get rid of taxes for
the people. What else ... oh, she wrote a
document that helped abolish cruel practices
in the justice system. So, she's proof that if the
monarch is educated and good-hearted as
was Catherine the Great, then many good
things can come about through an absolute
monarchy.

M: What we will talk about today are bards-uh, as


understood in the Celtic tradition. There are
actually a few different definitions of a bard, but
we're going to focus on one.
W: Sorry for the interruption, but what does Celtic
meari?
M: Oh, Celtic just refers to a culture in-uh, along
Western Europe. Think Ireland, Scotland, but
before and during the Middle Ages. OK, now,
to define a bard. In the Celtic tradition, a bard
was essentially a musician, though they had a
very different role than the musicians we know
today. I want to talk more about the role of the
bard in a minute. But basically what the bard
did was compose and perform music-alone,
almost always-and the bard would play an
instrument and sing. But it's what the bards
sang that was really interesting. In fact, the
music had more in common with our modern
literature than it does with our music. Bards
generally sang songs that told long, epic stories,
sometimes of real events that happened, or
maybe just folk stories that had been passed
on from previous generations.
W: So, a bard was like a storyteller. But with music,
too.
M: That's right. Now, I'm sure you're aware from
your history classes of how important storytelling
can be in the, uh, the understanding of a culture's
history. Regardless of whether a story is true or
not, it can give insight into how a culture thought,
uh, that sort of thing ... you know, rather than

Transcripts

--....-..i;.,.-,

the actual events that happened. So, as I think


you can see-uh, yes?
W: I get that the bards are also a solJrce of history,
like other storytellers, but how can history be
passed on through a story if it isn't written
down?
M: That's a good question. See, the bards were so
influential to the people that they actually
helped them-the people-commit these histories
to memory.
This is likely to show up on a quiz next week, by
the way. Back to the question, the effectiveness
of a bard comes in-well, let me give a modern
example. Have you ever heard a melody that is
just so catchy that you can't get it out of your
head? Well, that helps you remember the
words of the song too, right? That's just what
the bards did-they sang songs that people
could remember.
W: I get it, so that quality-the way songs can get
stuck in our heads, actually helped pass on
history because it would help people remember
ii" the stories.
'Ki-.

01 Anthropology
M: For last night's homework, I asked you to read
a couple of myths from both the Mayans and
the Aztecs. You'll remember.thP.t. the Mayan and
the Aztec civilizations were two of the largest
empires in what we now call Latin America. You
probably noticed how many similarities there
were between the two different mythologies.
They both have a great reverence for corn, they
both place a lot of importance on sacrifice, and
they both have similar creation myths. But why
do you all think this is? Why are there so many
similarities in the mythologies between two
cultures that were so distinct?
One of the first theories about why Mayan and
Aztec mythology are so similar is that they must
have descended from the same group of people.
That is, anthropologists believe that since they
share all this mythology, they must have once
been part of the same tribe that just diverged.
uh, split up, at different points. But, you know.
there's a lot of evidence that refutes this theory.
/,.

First, anthropologists have been able to


reconstruct hundreds of years of history for the
Mayans by studying the architecture and tools
they left behind. In fact, they estimate that the
Mayan civilization got started somewhere
around 1800 BCE. Aztec civilization, meanwhile,
dates back only seven hundred years. The
exact origin of either culture has still not been
determined, but one thing is sure: these two
civilizations probably came from two distinct
times, though they did exist at the same time for
awhile.
So, if that's not it, then we're still left with the
question about why their mythologies are so
similar. Here's something I want you to consider:
the Mayans and the Aztecs inhabited completely
different territories. The Mayans were found in
the western and southern parts of modern
Mexico, as well as Belize, Guatemala and El
Salvador. The Aztecs, on the other hand, lived
in central Mexico. But the thing is that they did
come into contact quite frequently. There was
some trade between the Mayans and Aztecs,
and it's believed that in the course of these
negotiations, some of their beliefs were transferred.
This could easily account for why sometimes
archeologists will find statuettes of Mayan gods
in Aztec territory or vice versa.
Something else that I think is significant is the
fact that even if they inhabited different territories,
both cultures moved around over a large
expanse of land. What does this m.ean? Well, it.
means that they probably ended up seeing a
lot of the same landscapes while they wandered
around. Remember that the mythology of any
culture is heavily influenced by the world
around them. So, for example, if both the
Mayans and the Aztecs saw that sometimes the
land produced and at other times it didn't, they
were bound to incorporate the importance of
those conditions into their mythologies. Does
that make any sense to you all? Or, just to give
another example, if they understood that corn
was a staple of their diets-which it was for both
cultures, by the way:--corn would inevitably
have some significance in their mythology.

__

Transcripts 669

02 Business
M: Hi, I'm glad that so many of you made it to this
review session. I want to make sure that we go
over the material that you guys feel you need
reinforcement on. I decided that the best way
to do this is to just let you all ask questions, and
we'll just go from there. So, um, as of now, the
floor is open.
W: Um, OK, here's something that I definitely
should have asked earlier in the semester, but
here goes: what exactly is a low-cost carrier? I
know that it's a pretty recent type of company,
but I don't quite understand how they make
money.
M: That's definitely going to be on the test, so I'm
glad you got around to asking that. To answer
the first part of your question, a low-cost carrier
is an airline that cuts out all the extra services.
That means no delicious mystery meat for dinner,
and definitely no first-class service. But, this
allows them to provide extremely low fares to
customers, which is always popular, as you
can imagine.
W: Ah, so they make money by bringing people in
with low prices. But that's really only part of it,
isn't it? I mean, there's more to the low-cost carrier
business model that helps them make money.
M: Do you remember some of the features of the
business model?
W: Um, well ... I think they're able to cut costs by
only operating one kind of plane.
M: That's a great point. Let me explain. See, by
only using one kind of plane, the company is
able to save on training costs. Plus, they'll also
spend less on maintenance and servicing. And
by buying many of the same type of aircraft at
once, the airline can negotiate discounts with
the manufacturer. Great point. What else?
W: Don't they also use less busy airports?
M: Yes, they do. These airports are sometimes
referred to as secondary airports. And since
secondary airports are usually less busy and
not so affected by delays at the major airports,
the airline can avoid delays.
W: Wouldn't they also pay less for landing fees at
secondary airports?
M: Yes, that's also true. Because remember; every
time a plane lands at an airport, that airline has
to pay a fee to the airport. Now, what else do
you think is a significant cost for all airlines in
general?
.,,___ 670

Transcripts

W: I would say fuel charges.


M: Good. All airlines, even low-cost carriers, have
to worry about the cost of fuel because it can
make operating that much more expensive. But
low-cost carriers avoid fluctuations in the price
of fuel by incorporating fuel hedging programs.
W: I know we covered this recently, but what is a
fuel hedging program again? Doesn't it mean
that they buy fuel in bulk?
M: Good try, but that's not quite it. An example of
a fuel hedging program would be when a
company buys fuel in advance at a fixed price
so that they won't be subject to changes in the
future.

03 Service Encounter
W: Good afternoon. How are you today?
M: I'm good, thank you. But, uh, well, I have a
weird question.
W: What is it?
M: L officially graduated last spring, but I'm still
kind of working on an independent study project
that I started in my senior year. I'm working
under a professor, but I guess by the university's
standards, I'm not really a student here anymore.
W: Ah, so you can't use the university libraries.
Don't worry, there's a way around that.
M: What a relief! I was really starting to worry
about how I was going to get the rest of my
research done.
W: We actually have a lot of people in. similar situations,
so we have to find a way to accommodate people.
I mean, it doesn't make sense to have a facility
like this and not make it available to people like
you. Anyway, what you have to do is apply for
a non-student pass.
1\11: But like I said, I'm kind of a student. Sort of.
W: Officially, you're not. But it's really just a label.
You'd get all the same access as a student,
and you'll even get to keep your campus email
address, since all that stuff is actually administered
by the library.
M: That's cool. Well, I'd like to get this processed
as soon as possible, so I'd be glad to fill out
whatever you need me to.
W: OK, then. Here's the application. At the bottom
it asks you about your relation to the universitymake sure to note that you're a recent graduate
and the name of the professor you're working

M:

W:

M:

W:

with. And if you can put a contact number for


the professor, that would be even better
because then we can confirm the project
faster.
Um, I'm sorry, but I just saw that there's a fee
for this. What's that all about?
That's right. I thought I mentioned that. It's
basically like a membership fee and if you think
about it, it's a pretty good deal. I mean, access
to all campus libraries and the computer network
for only $50 a semester. It's really not that bad.
Does that apply even for people in situations
like mine? I mean, I'm actually working on this
at my professor's request. It just doesn't seem
fair that I should have to pay extra at this point.
I'm sorry, but everybody who wants non-student
library access has to pay.

01 Litefature

W: We're going to start today by talking about genres,


and We'll discuss genres in the history of literature,
all the way back to ancient Greece. OK, so, in
the first place, what's a genre? Basically, it's a
defining category of art, literature or music, that
sort of thing. Let's talk about a few genres.
One of the earliest genres of literature was
tragedy: There a:re a lot of different defining
qualities of a tragedy, but in general there's a
heroic character with a tragic flaw, uh, something
in the character's personality that makes him or
her meet with bad fortune. Like uh. well, like
Mfil!fill. Medea is a play by Euripides, where
the main character, Medea, meets with bad
fortune because of her jealousy. So yeah, her
tragic flaw was jealousy.
OK, comedy is another genre. Now, comedy
today usually means something is really funny,
but comedies earlier in history were more
lighthearted than funny. Generally, strange
events happen because of some sort of
misunderstanding. Can I go on? OK. So, perhaps
the most famous comedies come from
Shakespeare, whom I'm sure you all know.
Shakespeare's comedies usually involve people
in love who are tricked or confused through

some clever ruse. A Midsummer Night's Dream


is a good example ... people fall asleep in a
forest-it's kind of silly-where a magical flower
makes them fall in love with anyone they see.

02 History
W: OK everyone, today we're going to be discussing
the Sea Dogs, who were, well, basically, they
were pirates, around the 1580s. They were
actually English pirates-and, like other pirates,
they stole stuff from ships they captured. But
they were actually employed by the Queen of
England in secret, and, uh, their goal was to
rob and steal from Spanish ships and ports.
Let's get more into why they did this.
First, I want to talk about why they stole gold
from Spanish ships. See, Queen Elizabeth of
England didn't really like King Philip of Spain.
They didn't have a good political relationship,
but Elizabeth wanted to make it seem like they
did. So, she hired these Sea Dogs in secret as
a way to attack King Philip without officially
declaring war. What I mean is ... by stealing
gold from Spanish ships, she was stealing gold
from Spain, and thus, from King Philip. And uh,
the Spanish didn't know it was really Elizabeth
behind it.
Also, the Sea Dogs actually went as far as to,
uh, to capture land, ports mostly. One of the
pirates, Sir Francis Drake, started taking land
from the Spanish in the name of England. Over
time, Drake became more, um, daring in his
exploits, and eventually war broke out between
England and Spain. Some would say that
Drake caused the war, but not me.

03 Office Hours
W: Hi, Professor Burke. Do you have a couple of
minutes to talk?
. M: Of course I do. What can I help you with?
W: I was hoping that you could give me some
advice. See, yours is the first class I've taken in
psychology, and I've really enjoyed it so, far. So
much so, that I'm starting to wish it was my
major. Do you think that I could do that? Switch
my major?

Transcripts

671 __.,...,,,..

''
M: Really? What a great compliment! Let me tell
you about the major requirements and then we
can talk about what you do next.
W: Actually, I read about the requirements. I was
at the registrar's office earlier, and I found the
department handbook. I guess my biggest
concern at this point is if I'll have the time to fulfill
the requirements. Especially the lab requirements.
M: I see. Yes, the lab requirements are pretty
rigorous. What year are vow in, again?
W: This is my fourth semester.
M: I see. Well, that certainly is late to change
majors, but you might think instead about making
psychology your minor since it takes less time.
You won't have that same level of specialization
that a major would grant you, but at this point, I
think it's probably your best bet. But a major
isn't out of the question. I'd advocate some
deep thought at this point. Take some time to
weigh your options.
W: Hmm, you're right. Thank you so much.

04 Service Encounter
W: Hi, welcome to the Academic Support Center.
How can I help you?
M: Hi, this is kind of embarrassing, but I really
need someone to help me with my writing. I
have this crucial essay for my history of
mathematics class and I have to do really well
on it, or I'II fail. Do you have anything for me
here, or is it all tutors and that kind of thing?
W: Well, first of all, yes, we do have someone to
help you with essays. They're called writing
fellows, and what they do is walk you through
the essay-writing process. Like, they'll help you
put together your research and organize your
paper. Then they'll help you revise your drafts.
M: That sounds perfect for me. It's exactly what I
need. How can I get hooked up with one of
these writing fellows?
W: Well, first you need to fill out an application.
Don't worry. It's mostly so that the center can
keep track of what kind of demand there is for
tutors and that kind of thing.
M: OK, and will it lake long for me to get in touch
with a writing fellow?
W: It can take between two and three days. What
we'll do is use the information on your application
to try to find a good match for you. We'll be
i'.
h

~1-.
>fr

'

672 Transcripts

looking for someone who might possibly know


a thing or two about, what was it? The history of
mathematics.
M: Oh wonderful. And then will this person get in
touch with me, or will I have to contact them
somehow?
W: He or she will contact you via email and set up
a time for the two of you to meet and go over
the topic and such.
M: Thanks. Let me get that application filled out,
then.

05 Environmental Science
W: Good morning class, I noticed on the last pop
quiz that you all didn't do so well on the part
that had to do with the rain cycle. After grading
the quizzes, I realized that most of you didn't
really understand how each step is related to
the others, so that's what we're going to work
on today. Would anybody care to start us off by
naming one step in the cycle?
M: I guess I'd start with the evaporation of water.
W: Great. That's step one. And what does water
turn into when it evaporates?
M: Um well, it goes from a liquid state to a
gaseous state. Is that what you mean? The
water droplets turn into water vapor.
W: That's right! And then the vapor gets sucked
into the air currents in the atmosphere. Since
the air is cooler there, it condenses and turns
into a liquid. So condensation is the second
step.
I want to expand upon the third step, since
most of you forgot that on the quiz. It's called
advection. Keep in mind that once the water hits
an air current, it's moved along by the current. That
is called advection. That's why water that
evaporates over oceans will eventually fall over
land. Water moves in air currents. That's all
you need to remember. If you have trouble
remembering those terms, remember that
condensation is_ the .opposite process of
evaporation. That is, condensation occurs
when a gas is cooled. And evaporation happens
when a liquid is heafed. OK, with that said, let's
move on. Of course, the next and most obvious
step is precipitation, and that's just rain, snow,
hail. Water falling back to Earth.

.1
. il
1i
( .
. ;.
,~

06 Psychology

07 Office Hours

M: Good afternoon, everybody. At the end of


yesterday's lecture, I mentioned Bruce
Tuckerman;who developed a model of group
development. Today I want to expand on
Tuckerman's theory, which is characterized by
four steps: forming, storming, norming, and
performing.
The first stage is called "forming." So, as you
can probably_ tell from the name, this is the time
when the group actually comes together and
forms. This is also going to be the stage where
the members of the group get to know each
other and try to establish some familiarity. Are
we all clear on this? To recap, it's just when the
group convenes and the members get to know
each other.
The second stage is called "storming." This is a
period of constant conflict within the group as
different members struggle for leadership.
Some could see this as I.he period of "growing
pains" for;;the group as the members become
accustomed to the processes of operating as
a group ...1 want to point out that sometimes
groups d~m't really make it out of this stage.
Tuckerman believed that advancement past this
level was dependant on the maturity level of the
members of the group.
The third step is called "norming," which is
when some calm comes over the group as they
resolve some of the initial conflicts. The
assumption is that the group reaches a sort of
consensus about operating. You should also
remember that at this point, team members
start trusting each other. It's also been noted
that at this stage, members of the group seem
to share a set of values about rules, professional
behavior, and methods.
Finally, we've got the "performing" stage. At
this point, the group is able to maximize the
potential of its varied member's strengths. A
group in the performing stage will demonstrate
efficacy, organization, and competence.

W: Hi, Professor Albright? I hope this isn't a bad


time. If you don't mind, I just wanted to ask you
a quick question.
M: Hi, Caroline. I've got a couple of minutes to
spare. What's the problem?
W: I just don't know what to do. I was reviewing the
finals schedule on the website yesterday, and I
realized for the first time that the final for your
class is Thursday at nine a.m.
M: No, that can't be right, My schedule says
Tuesday morning. Maybe there was a mistake
in the schedule.
W: ihat's what I thought, too, so I asked my other
professors. And it turns out that according to
the official university calendar, this final is on
Thursday, not Tuesday.
M: Oh, no. I must have been looking at last year's
academic calendar. This is a disaster. Then
again, if I'm the only one that was going by that
calendar, I guess all that really means for my
students is that they get two extra days to
study. I can't see anybody objecting to that.
W: Um, well, I sort of have a problem. See, this
semester I'm taking a theater class, and for our
final we have to do a solo performance for the
class. Since I didn't think I had any commitments
for Thursday, that's when I signed up to do my
performance.
M: Oh, I'm so sorry. Would you be able to switch
places with someone else in the class?
W: I already asked some people in my class, but
the fact is that there are so few slots and so
many people that there's really no way to
change. I really hate to ask, but I wanted to
know if I could take the final on a different day.
M: Oh, Caroline, I don't know. The university has
strict policies about that kind of stuff. I'd have to
request another lecture hall and find a proctor
to administer the test.
W: See, if I can't do my performance, I'll fail the
course. It's such a big component that it's
worth forty-five percent of our grade for the
semester.
M: 01<, I definitely wouldn't want that. Listen,
here's what I'm going to do. Tomorrow during
the lecture, I'll tell the class, and then see how
many other students will be affected. And I
won't forget about you .. Then we can all try to
figure out an alternative time that will work for
everyone.

_.,...fi

Transcripts 673

08 Service Encounter
M: Hi, are you the Housing Director?
W: Yes, that's me. Can I help you with something?
M: Yeah, I was actually sent up here from the
Student Services Department. I'll have a
strange housing requirement next year, and
they told me I'd have to get special permission
to see if they can accommodate me.
W: OK, why don't we start at the beginning?
What's your special need?
M: See, I just got an internship to work abroad with
a language institute. The thing is that the dates
don't exactly match up with the school calendar.
Since it's part of my independent study, my
academic calendar is pretty flexible. But
practically speaking, I won't be returning to
Boston until mid-March, and I'll need housing
at that point.
W: That's impossible. I'm sorry, it's just that with
the housing shortage. there's no way we can
guarantee that space. We'll probably be short
on rooms as it is.
M: Right, I understand that. But what if I could live
off-campus? I mean, if I can arrange my own
housing, then there would be a way to
accommodate everyone.
W: That's a fine idea. I don't see any problems with
that. I don't even need to give you any special
permission. All you have to do is file for off-campus
housing for that semester and then submit
some proof of residence when you get back.
M: That's what I wanted to do, but the folks at
Student Services told me that since I'm
technically still a second-year student, I can't
move off-campus. So I wasn't able to file for
off-campus housing.
W: Oh, I didn't realize that you weren't an
upperclassman. We don't usually have
underclassmen getting internships.
M: Yeah, I saw this one and I knew it was something
I had to go for. I'm really excited and I'm just
hoping that I can get everything settled over
here so that it actually happens.
W: .1 see, so you need my special permission fo file -for off-campus housing because you're an
underclassman. I understand now. Well to be
honest I don't see any moblem with writing a
letter for you.
M: Really? I'd really appreciate that. Would I need
to provide you with any forms or anything? It

_ _ 674 Transcripts

seems like everybody needs some form.


W: Um, why don't you just bring me the letter of
acceptance for the internship. That should suffice.

09 Astronomy
M: Today I wanted to talk about black holes, but I
have to say, I was surprised at the amount of
emails I received in response to the last lecture.
The confusion seemed to surround the mai11
sequence stage jn a star's life. In fact, one
question in particular that kept on coming up 1n
the emails was why stars send off more energy
as they get older, rather than when they first
form. So, let's cover that.
The first thing I want you to understand is the
basic idea of how stars are created.
Remember, when stars are being formed, energy
is mostly created when the atoms from the
gaseous materials that make up the star collide.
Do you all follow what I'm talking about? Here,
let me clarify. Basically, all the different atoms
at the star's core crash into each other, creating
heat energy. At this stage, the star is called a
protostar. The protostar is not very bright and
doesn't really send out too much energy at all,
at least not when compared with the output it
will have as a main sequence star-uh, that's
just the stage where stars spend most of their
lives. The most important thing to remember
about this stage is that it's when the star begins
to shrink.
However, when the star reaches a certain age,
hydrogen fusion begins in the star's core. It has
now started undergoing the main sequence
stage. One by-product of hydrogen fusion is
helium. This is important, now, so please make
sure you're paying attention: the fusion of
hydrogen particles at the core causes s.ome
high-temperature and high-pressure reactions.
This is where all that extra energy you all were
asking me about in the emails comes from. QQ
you all get that? Let me reiterate that: during the
main sequence stage in the lifespan of the sta.r,
hydrogen fusion occurs in the star's core. This
causes high temperature reactions that.result
in the steady increase of the star's lum1nos1ty
and temperature.
Also remember that the star does eventually run

out of, uh, fuel, energy, which is when the star


begins to die. It'll probably make sense to you
that bigger stars run out of fuel more quickly, and
therefore die out fairly quickly.

10 Biology
M: Yesterday we wrapped up our discussion on
the Sirenia order. If there are no questions
about yesterday's lecture, I'd like to move on to
discuss Order Cetacea. So, are there any
questions? No? Good.
So, there are about ninety different species of
the Order Cetacea. Can anyone think of any
cetaceans?
W: Dolphins and manatees.
M: You're right about dolphins, but not manatees.
Those are of the Order Sirenia, remember?
W: Um, OK ... what about whales and porpoises?
M: That's right, on both counts. And if you ever
have trouble remembering that consider that
c&tus actually means "whale" in Latin. Well
anYway, cetaceans are pretty well known as
the most aquatically adapted mammals. So
the'n, what other qualities do you think that
cetaceans might share?
W: Don't they all live in the ocean? Like they're all
marine species, right?
M: You're almost right. Of all the species, there are
only five freshwater varieties. And those would
be the five species-:the freshwater dolphins.
W: OK and I think I remember reading somewhere
that cetaceans actually have hind legs, don't

m?

M: Right! But obviously, they aren't really used at


all. In tact, they can't be because one, they are
so tiny and two, they aren't even attached to the
bacl<bone. But anyway, unlike other mammals,
cetaceans also have no body hair.
W: How do they keep up their body temperature
then? I mean, I Jmow that most other mammals
have fur to keep them warm.
M: Cetaceans have a thick layer of fat called blubber
that l1elps keep them warm.
W: Oh, OK. That makes sense.
M: All right, moving on then, cetaceans are
thought to have evolved from land mammals
into primarily marine animals about fifty million
years ago. Signs of their evolution are evident

to marine biologists today. The most obvious


example is that they need to breathe air from
the surface. But other signs of cetacean evolution
include the bones of their fins, which, it you've
ever examined them, actually look like jointed
hands.
W: That's really interesting. What other adaptations
did they undergo?
M: Well, there are probably too many to name during
one lecture, but I can give you some examples.
Scientists believe that the first adaptation that
occurred was the loss of the hind limbs.
Interestingly enough, they also lost external
ears, since they didn't really have a use tor
them anymore.

01 Literature

W: Let's move on to Henry James, and the uh, the


interesting and groundbreaking characters he
created. James was one of the first novel writers
to really capture-to uh, create characters that
seemed human. Before James, uh, we're talking
late nineteenth century, early twentieth century,
here ... before this, literary characters were
often overly heroic, totally evil, completely
pure-and well, a bit implausible.
OK, so let's get talking about his characters,
starting with something called an unreliable
narrator. So ... to explain. We sometimes tell
lies, right? We lie to one another, whether we
mean to or not. Well, what if a narrator lies to
you? This is one of the ways that a narrator can
be unreliable. We can't trust the narrator. And
Henry James used these kinds of narrators,
which gave his stories, er, I mean his characters
more realistic personalities.
James also used a "stream of consciousness"
style, especially toward the end of his writing
career. This style involves getting onto paper
whatever comes to your head, without censoring
what you're writing. He did this to explore the
inner thoughts of his characters, trying to
understand everything they thought, even if it
was difficult. I'm personally not wild about this
style, but many feel it adds to the overall realism.

Transcripts 675 -

.... ~ ~- .............................. __!_ ___

02 Agriculture
M: Moving on, let's talk about bio fuel, which is um,

it's basically any sort of biological materialdead or alive-that can be used as a source of
energy. As tor biological material, anything
goes, such as plants, wood, wastes, you name
it. Today, I'm going to go through the advantages
and disadvantages of bio fuel.
OK ... the pros: it's a renewable fuel, meaning
it won't run out like fossil fuels will. We can
always grow more plants, right? Right now,
scientists are experimenting with a fuel made
from corn-called ethanol. This kind of fuel
produces fifty percent fewer emissions, so it's
better for the environment. Bio fuels like ethanol
will help keep smog out of cities. So these are
some things that are pretty attractive about bio
fuels, pretty exciting, actually.
Now, for the cons: just because it's renewable
doesn't mean that the Earth goes unharmed
from its use. In fact, it still could contribute to
global warming. For example, while they're
being made, bio fuels still put out gases that
contribute to the greenhouse .effect when
they're burned. In addition, since many bio
fuels are extracted from plants, there would
have to be more land used for agriculture. And
if you ask me, for this reason we really need to
do more research on this subject.

03 Office Hours
W: Hey, Harold. Anything I can do for you?
IVI: Actually, yeah, Professor Steele. I had a dentist's
appointment on the day that we went over the
material for the upcoming exam. Even though
I've borrowed notes from another student, I feel
like I still have a shal1y grasp on what to expect.
W: OK, sure. But shaky ... how?
M: Well, uh, I know that you'll be asking about
society's perceptions of mental illness. I'm not
sure, though, in how much detail.
W: Well, let's put it this way: the exam has twenty
multiple choice questions, but those are just
basic concepts. So you should have no problems
with those. The rest of the exam consists of
essay questions.
M: That's the part I'm worried about.
W: OK, well, the two essays will mainly go over the
topics we've covered more than once, since

those are the, well, the most important things


we've covered. What I said in the review session
was that I don't want to be reading essays
about what people think about those topics in
general. I mean, they should reflect a deep
understanding of the material, not something
that a first-year student could answer. I know I
keep restating that, but it's happened a lot in
the past.
M: OK, then, I'll remember that. Anything else?
W: Not really. If you've listened to lectures, you
should do just fine on the exam.

04 Service Encounter
M: Hi, can I help you?

W: Yes, I'm looking for a textbook.


M: OK, but just to let you know, we don't carry
many textbooks here at the campus bookstore
at this point of the semester. So I can only let
you know about textbooks for next semester.
W: Actually, I'm looking for a textbook for this
semester.
M: Huh? This semester? Did you lose your old
textbook? In that case, you might have to talk to
your professor. We can't order replacements.
W: No, actually, up until now I'd been sharing the
textbook with my roommate, but she transferred
to anoU1er dorm, so ...
M: I see. Well, the thing is, we only order the books
right before the semester begins and we usually
order just enough for the students enrolled in
each course. If, for whatever reason, there's a
surplus, we send them back to the manufacturer
a few weeks after classes have started.
W: You've sent them back already?
M: I'm afraid so.
W: Lovely. So I can't get the book here.
M: No, but you could try the library.
W: OK ...
M: Actually, I believe the library is required to have
a copy of every textbook for each course, so
that students who can't afford to h.ave the
books will still have access. But what a dragyou'll have to do all your reading in the library.
And another student might be reading it when
you want to. Not a good option.
W: Yeah, I don't want to go back and forth to the
library. Do you know of any other bookstores
around here that might have it?

\...-- 676 Transcripts

~ ....,___,......., ... _ "-'"'------

-- - ___ ::.::~:.:=.:.:::.:.:::::..._

..::::- ::.':.::.::.:._::':'.:'.:'.=:.'.:.~: -

:.:..:.:::.:::.::~:::::::;::-_::.::-.. ::".:.~.::.:~-:::::::.:."'::..:::::::::::'.::::::::-:::::'.::'.-.:-.:: :_'~:':_""":_' :::_-::::::':::.."':::--::::..-:.'t.':~:-::.::~:::::.'.:-.:::::'_'.'.::'''~.:'::::'!:._::::=-:=-.:.:~-:.'::~

----

----

-----

----

=-=..::= '

M: Well, not around here, but ... Oh, I've got it ...
right, right. I almost forgot. A friend told me
about a website called unibooksunlimited.com.
My friend said that it has every title you can
think of-new ones, old ones, whatever. That'd
be your best bet.
W: Thanks! I'll try that.

OS Environmental Science

!','

M: OK, everyone, today we're going to be talking


about invasive species, which, um, which are
any species of plant or animal that is not native
to an area, but is somehow introduced. After
being introduced to the new area, they cause
serious damage-they can really cause problems
for other species. Sometimes, the naturally
occurring species become extinct. Yes, invasive
species are very strong competitors. Let me
give you an example.
.. ; Interestingly enough, farmed salmon is causing
"[: huge problems in the northwestern US and uh,
: the west coast of Canada. You see, these fish,
the farrned ones, are raised iri huge pens, um,
! you know, areas of the sea that are surrounded
by nets. But the problem is that many of the
farmed salrnon are escaping from the netted
areas and heading out into the ocean.
Let's talk about the effects on the ecosystern. To
begin, these farmed salmon introduce new
parasites and diseases to the native fish of
these regions. Se?. the local salmon haven't
had any exposure to these diseases until now
so they lack resistance to disease. Therefore:
when the farmed salmon invaded, they spread
a lot of disease among the local salmon. And
it's this point, this first point, that I'm the rnost
worried about. I'm afraid we may see entire
populations of native salmon disappearing.
Second, lil<e I said, they're really competitive,
t11ese invasive salmon. The environment-I'm
talking Canada and Alaska now-these places
had a balance of resources, and the environment
was able to support the existing life there.
However, when the fanned salmon invaded,
suddenly there were more fish competing for
food. It's a big ocean. buc a1 e-1ent like this can
change things for a lot oi spec,es. In this case,
other fish started dying because they couldn"t
find enough food.

06 History
W: The Gettysburg Address is the great speech
delivered by President Abraham Lincoln at
Gettysburg, Pennsylvania that touched many
Americans' hearts. But did you know it was only
about two minutes in length? Amazing, huh?
OK, so first, let me give you some background info.
The president made this speech during the
American Civil War, back in um, in 1863. And
he gave it at Gettysburg, the site of a huge battle.
But here's my question ... why was the speech
so effective? Simple. It touched people's emotions,
and that's what I want to focus on now.
Well, first of all, Lincoln evoked strong emotions
by talking about the importance of human
equality, and um, many people thought that
was really what the Civil War was being fought
over. In his speech, he was referring to the
American Revolution, about how people made
great efforts to achieve freedom, reminding the
American people what they are about: about
liberty and equality among a//. Not among a
select few. But among all. So he made an
emotional appeal-a very strong demand that
all people be equal.
Next, and despite the split within Americaremember, the nation was divided at this timeLincoln made sure to constantly use the word
"nation," to emphasize that America would not
remain divided, but would come together as
one to devote itself to its main causes, which,
again, were liberty and equality. This made
Americans feel as if they were part of a whole
again, creating a feeling of great unity. They
needed to hear this, and it made people react
very emotionally.
Oh, and one more key point. Lincoln did not
introduce any new concepts to the people-in
fact, he reminded them of what they knew,
which is probably why it touched them so
deeply.

Tra.nscrlpts

677

- i

07 Office Hours

'

M: Professor Jones?
W: Hey, Zach. Come in. I haven't seen you in a
while.
M: Yeah, in fact, that's why I'm here~1ve been
sick, and I'm pretty worried about my grades.
W: I understand, for sure. So what's on your mind?
M: Well, I've been meeting with one of the deans,
and she's advised me to go directly to my
professors to discuss what policies each one
has-about, well, students that get sick and
then suffer academically. I have to be honest
with you. I um, I just don't think I can finish
classes this semester.
W: I don't really have any specific policy. But in my
experience, students who do get sick tend to
go on medical leave, which is when students
get permission to take time off from school in
order to recuperate.
M: Right. I heard from the dean that it takes quite
a while to process a medical leave, and since
the end of the semester i.s drawing near, I'd end
up with an incomplete for my classes this
semester.
W: Well, that wouldn't be the end of the world.
M: Thing is, I'm on a scholarship, and one incomplete
semester, then, well, the scholarship will be
withdrawn.
W: Ah, I see. This is a bit complicated. So there's
just no way you can finish up on time ... hmm.
In that case, let me see. Well, how about I give
you an O.H.?
M: Sorry; an O.H.?
W: Yeah, it stands for "On Hold," and it wouldn't
register as an incomplete. It's nice because it
takes into account all of the work that you've
done up to now. You can just continue when
you're ready. I mean, you can take your time
and complete it over the winter break if you'd
like. It's all up to you and your health.
M: That sounds wonderful.
W: Hmm, the thing is, participation counts for ten
percent of your grade, and you've missed a lot
of classes due to your illness. Well, in place of
that, how about writing a three-to-four page
paper to demonstrate what you've learned. I'd
be satisfied with that.
M: Thank you. Thank you for being willing to work
with me on th is.

... 678 Transcripts

W: Well, you can't help being sick. It's important


that you focus on getting better instead of
stressing out over grades. I really do hope your
health improves soon.
M: Thanks. So do I.

08 Service Encounter
W: Hi, there.
M: Hi. Is this the residential life office? For student
housing?
W: You've come to the right place. Anything I can
help you with?
M: Yeah, how can I get housing for next semester
if I'm changing rooms?
.
W: Well, we have a system called the housing lottery,
which randomly assigns dormitory rooms to
students. You have to show up next Monday at
six p.m. at this building for the lottery drawing.
M: I know, I went through it last year, and it was
hectic. I'm not too fond of the system.
W: I'm afraid that's all we have for the moment.
M: Well, I'd actually rather not deal with the hassle
of on-campus housing.'
W: Have you considered off-campus housing?
M: I have, but it seems to be quite a hassle as well.
W: Actually, it's not that bad.
M: Really? I've always thought that it would require
special permission.
W: You do have to obtain permission, but anyone
can get it. And it doesn't take too long to apply
for it or to receive it.
M: OK, so how does it work?
W: You can get it right here, right now. Just fill out
a "permission-to-relocate" form and I can guide
you through it, or if you're in a rush, you can
take it with you and fill it out later. Just make
sure to bring it in by next Friday, by five p.m.
That's the deadline.
M: Actually, I'm in no hurry.
W: OK, then. First of all, you're not a freshman right
now, are you?
M: No, I'm going to be a junior.
W: Oh, that's right; you mentioned that you had
already gone through the lottery process. So
you can apply. Here's the app.
M: Thanks.
W: It'll take about three weeks before we let you
know if you've been approved or not.

,,

M: And once I know that I've been approved, do I


ever have to apply again?
W: Well, it's good for one year. After one year is
up, you have to reapply.
M: OK ... This looks kind of long. On second
thought, I think I'll fill it out later and bring it
back.
W: Sure thing. It's pretty straightforward, so you
shouldn't have any problems with it.
M: Great. I'll give it a try. I have to admit, I was really
worried that this would be a long, complex
process, but it looks like it'll be a piece of cake.
Thanks for your help.

09 Biology
W: We all know animals communicate, but how?
Well, some communicate through vocalizations
such as songs, or by performing certain kinds
of movements. Now, I'm sure you're already
ja,(Tliliar with these, but today's topic might not
b;e,. so widely known-and that's the use of the
olfactory sense-which is, simply, smell.
Animals use smell to communicate a message.
M you'll see, the sense of smell is keenly
developed in many animals. Yes, while we
humans have a rather limited capacity for
smell, in most animals it is highly developed-a
very primal sense. Whether they communicate
through the smell of waste products like feces
and urine, or through a glandular process like
sweating, smell is an important means of animal
communication.
Let's start with cats, shall we? Cats rely on
scents for many purposes. Cats have scents
glands on the sides of their bodies, on their
foreheads, along their tails, underneath their
front paws, and on their lips and chin. Have you
ever noticed that a cat might rub its forehead
against you, or its tail, or any of these areas I
just mentioned? And uh, and I know several of
you must be cat lovers, but frankly, guys, I find
this behavior really, well, tiresome. It really gets
on my nerves. Ah, sorry, I digress. Anyway,
why does a cat do this? Well, it's simple. It may
be trying to communicate something, lil<e
affection, or it may be marking you! Yes, cats
do indeed use scents to mark their property.
Yes, you belong to the cat, at least in the cat's
mind. Oh, and they also mark territory, since

cats are so territorial.


Another example of communicating with smell
can be seen in ants, who use scent for a variety
of purposes. They use pheromones, kinds of
chemicals that can influence animal behavior,
or signal that they're looking for a mate. Oh,
and ants can also release a chemical that acts
as an alarm. As, uh, a warning to other ants. So
if a particular ant is in trouble in some way, ants
release this chemical as a crying-out signal for
help, which other ants respond to by coming to
its aid. So ants use pheromones not only for
mating, but also to relay important information
for survival. Luckily, we can't smell all of these
different chemicals. Imagine what that would
be like.

10 Environmental Science
M: We'll be discussing why the oceans are saltycauses and effects, well, really just causes. But
first, a question. Tell me, what happens when
the water evaporates from the ocean? What
happens to the salt?
W: Well, the salt doesn't evaporate, it stays in the
ocean. That's obvious.
M: Right, and so we need to look at the hydrological
cycle which you've read about, so I'll move on

W: Sorry, Professor McGuiness, I've done most of


the reading, but I'm not familiar with that term.
M: Ah, the uh, the hydrological cycle is simply the
process by which water evaporates, makes
clouds, rains down to Earth, and so on and so on.
And the first step in the process is evaporation.
But salt, of course, is left behind. It doesn't
evaporate.
W: OK, of course, I get it. But how does the salt get
there in the first place?
M: Good question. And I'll ask you another question
to get the answer. Where do oceans get their
water from? I mean mostly?
W: Rivers, right? Freshwater rivers dump billions of
liters of water into the ocean every day. Well,
I'm guessing it's billions.
M: Oh, it is ... and then some. So first of all, when
the rain falls to the ground, and begins to run
over the soil and rocks. it begins to dissolve
them. That's how it picks up mineral salts. Next,
rivers collect these and then head for the ocean.
Transcripts 679 __._1~

W: OK, but isn't there another way the oceans get


salty?
M: Absolutely. And uh, you've hit upon a very
fascinating point. The answer is through
hydrothermal vents. The connection between
the saltiness of the ocean and these vents is
intriguing. Now, you may not be familiar with
these, so let me explain. They are basically,
um, they're cracks in the Earth's crust that hot
water pas~es through. OK, so this super-hot
water dissolves minerals in the Earth's crust.
Now guess where this salt-rich water goes
back to?
W: Back into the ocean.
M: Exactly. Now, we've talked about all these ways
that salt is brought to the ocean. Does this
mean that the ocean is getting saltier and saltier
over time?
W: I guess so, if the salt remains in the ocean and
has nowhere to go. Seems like common sense.
M: The thing is, and really, this is kind of shocking.
The salt does have somewhere to go. See, the
ocean actually doesn't get saltier over time. It's
because these dissolved salts go back to create
new minerals on the oceanic floor. I mean,
everything's a cycle, right? So while the rivers
and the other processes are bringing in salt, a
lot of the ocean's salt makes formations at the
bottom of the sea-at the same rate, more or
less. So the salinity of the ocean is at a steady
state. It's about 3.5 percent, and that's where it
stays.

01 Astronomy
M: All week, we've been talking about the different
characteristics of the inner planets. Just to
refresh your memory, the inner planets are
Mercury, Venus, Earth and Mars. Today,
however, I want to focus on another group of
planets: Jupiter, Saturn, Uranus, and Neptune.
These planets are known as the Jovian planets.
The most immediately obvious characteristic
that the Jovian planets share is that they are the
most distant from the sun. This is a significant
point, and we'll discuss later in class how their
position in relation to the sun affects their makeup.
But before we get to that, I want to touch upon

- - 680 Transcrlpts

some of the other characteristics that the


Jovian planets have in common.
The feature that really, by definition, all the
Jovian planets share is that they're mostly
made up of gas. In fact, another term for Jovian
planets is gas giants, because they are made
up of gas, and they are huge! Anyway, if you
think about what that means, you can probably
guess that for these planets, there is no crust or
outer surface, like, say, on Mars. You couldn't
land on Jupiter, or any of the Jovian planets.
The atmospheres are mostly made up of helium
and hydrogen. Does that sound familiar to you
a!!? Maybe because it's very similar to the
chemical makeup of the sun. See, we don't
completely understand the origins of the Jovian
planets, but one theory is that they, the uh
Jovian planets may have come from the same
stuff that formed our sun .. Interesting theory. If
you read what's out there on the subject, you'll
probably find it quite convincing. I mean, I
haven't read anything that explains it better
OO!ly.
What else do Jovian planets have in common?
Well, in spite of their significant distance from
the sun, they still manage to have very hot
cores. I think your textbook calls them "rocky
cores," but I think that's a little confusing,
since, the temperatures at the center of the
planets melt the heavier elements found there.
What I mean is that sometimes they are called
rocky cores because that's where heavy elements
such as nickel or iron can be found. But you
shouldn't imagine that there's a solid ball of
rock at the center of these planets because at
those temperatures, they are molten liquid
rock.
Um, what else? Oh right. OK, so the last thing I
wanted to mention is that all of the Jovian planets
rotate very rapidly. Consider this, it only takes
Jupiter ten hours to rotate. And we all know it
takes the Earth about twenty-four hours to
rotate. Anyway, the fast rotations affect the
wind patterns on Jovian planets. What happens
is that the wind patterns are broken up into
latitudinal, or east-west, bands. If you've ever
seen pictures of Jupiter, you've probably seen
the stripes that go across them. And on a more
personal note, well, I find these to be the most
beautiful planets in our solar system. The
.bands are pret)y noticeable on all of the Jovian

If
!,

planets with the exception of Uranus.

02 Philosophy
W: This week we read excerpts from Hobbes's
Leviathan. Earlier in the week, I gave you some
topics to think about for this discussion section.
Why don't we just work off the questions on that
sheet? Can somebody read the first one out
loud for the class, please?
M: OK here goes: Hobbes explores themes of
human nature in his works. What are his ideas
about human nature and how do they relate to
his famous social contract theory?
W: Thank you, Carl. So, what's your response to
that?
M: Um, well, I guess the main point of his social
contract theory is that it's everybody's duty, I
guess, to act in ways that will maintain order
and peace. Sometimes this means giving up
some rights to the institution that helps maintain
oi(j~r, namely the government. And that's why
Hobbes was in support of a strong government.
W: Rig,\'JI. Great summary. OK, so the way you all
Seil, it, how does the social contract theory
relate to Hobbes's views on human nature?
That is, how did Hobbes construct his social
contract theory according to his ideas about
human nature?
.
M: He believed that the need for a social contract
arose from the fact that human nature is disruptive
and selfish. I interpret that to mean that since
he thought that humans are more inclined to be
motivated by self-interest, there needs to be a
system, the social contract, to ensure that we
can promote peace and not war, which is not in
anyone's best interest. So, like the book said,
his idea is kind of like a tarnished Golden Rule.
Sort of like, don't do unto others what you
wouldn'twant them to do to you. And that's how
we can maintain peace in the world.
W: Yes, that's a nice, simple way of putting it. You
all clearly have a pretty good grasp on the
general concepts of Hobbesian pllilosophy.
So, having reviewed that material, what's your
general reaction to Hobbes's ideas on human
nature?

M: I guess I'm still confused about that. I mean,


doesn't participating in the government count.
as a selfless thing to do? I don't know how he
can say that people are intrinsically selfish and
then promote a strong government.
W: That's true, but the idea of having a strong
government js also more prescriptive. Do you
understand what I mean? So, basically, he
thinks that one way to counter the selfishness of
humans is to have a strong government. And
that way, there's a central i.nstitution that looks
out for everyone; not just for the indivi.dual.
M: OK, so he's not necessarily saying that any and
all acts that bring about a beneficial outcome
originate with selfishness? I mean, doesn't
morality sort of factor into the equation at some
point, as well?
W: Exactly. You know, later in the semester, we'll
be revisiting the question about altruism and
morality. Anyway, great discussion today. You
guys really seem to have a good grasp of this
stuff.

03 Service Encounter
W: Hi, how are you today? It's pretty busy, huh?
M: Yeah, it's always packed around dinnertime.
Can I swipe your card, please?
W: Oh right. Sorry, here you go. I just don't know
where my head is today.
M: No problem. It is Monday, after all. Um, I'm
sorry, but it looks like you don't have enough
points to pay for this meal.
W: Huh? That can't be. I barely even eat on campus.
It must be some mistake. Do you mind swiping
my card again?
M: Sure, let me give it a try. Sometimes the system
is a little slow. I guess even our card reader is
having one of those days. Hm. Nope, I'm sorry.
According to the system, you're all out of meal
points.
W: I don't understand. I know you must hear this a
lot, but really, this has got to be a mistake. !'.'&
seriously eaten on campus, like, five times this
semester.
M: It could be that the system is misreading your
card. It happens sometimes. But I honestly
bave no way of telling that from here. I couldn't
override the system even if I wanted to.

Transcripts 681 - -

-----,----~---

-;-;_...:__--::;.=_-_-_____ - -

W: OK, I understand. But what do I do now? I

W:

mean, I have to eat.


Well, when situations like this come up, I can
only offer two options. You can either pay cash
for this meal and then go by Dining Services
later to see if you can clear it up and get a
refund, or you can just get your dinner somewhere
else.
Those are my options? How do I even know that
they'll reimburse me?
Here's the thing ... I'm sure you're being honest.
but believe it or not. there are plenty of people
that come by and try to get free meals from us.
It wouldn't be fair to the other students who
pay.
I can see that. OK, I just want to eat. How much
is it again? And you said I'd have to fill out a
form.
It's $7.50. And here's the form. This is kind of
like a receipt. Then you have to go to Dining
Services and file a complaint. I'm sure that
they'll be able to find out what happened with
your account. They're really good about fixing
those problems fast. Because as you said, you
have to eat.
I'll give it a try. Thanks.

01

Linguistics

M:

W:
M:

W:
M:

.,
'

W: Yesterday, we started talking about how linguists


study etymology, which, uh, is the study of the,
uh, the history of words. Today I want to continue
with that d.i.scussion by describing some of the
ways that etymologists think thatnew words are
formed.
One of the most common ways that new words are
added to a language is by the borrowing of words
from other languages. Now, these borrowed
words are called "loanwords" in linguistics.
Loanwords are almost never translated and are
pretty much taken directly from a foreign language.
Another way that new words are created is
through compounding or derivation-uh, these
are two different concepts. but we'll discuss
them together. Compounding is when two
separate words are put together. One example

J~

of a compound word is $kyscraper. So, "sky"


plus "scraper" equals skyscraper. Derivation,
on the other hand, is when prefixes or suffixes
are added to a word. An example of a word that
. was created through derivation is enrich. The
prefix en- has been added to the word "rich" in
thi.s case.
Another interesting case is when the origin of a
word comes from an invention-like when
speakers attempt to imitate sounds with language.
If you think back to English class, you'll remember
that a word, uh, that sort of imitates the sound it
is supposed to represent is called onomatopoeia.
Click, buzz and boom are examples of imitative
words.

02 Biology
W: This week, we've been talking about the different
qualities and functions of blood. Let's see, we
talked about the makeup of blood early in the
week, then, for the last lecture, we focused
specifically on the blood of mammals.
Everyone OK with that? Well then, for today's
lecture I wanted to talk a little bit about blood
production and degradation and the process of
it all.
The term for the process of generating new
blood is called hematopoiesis. That is actually
spelled pretty much like it sounds, hee-ma-toepoe-ee-sis. OK, so what hematopoiesis entails
is the creation of new blood cells. And where
might you guess that this takes place? OK. this
is imperative, so make sure to write this down:
new blood cells are created in the bone marrow,
specifically red bone marrow. The bone marrow.
I'm sure you remember. is the tissue located on
the inside of bone.
Are we all still on the same page so far? OK. As
for the degradation of blood, the next step,
most of it takes place in the spleen. Oh, I mean
breaking down when I say degradation. There
are also special cells in the liver that degrade
blood cells. In a healthy person, blood cells are
typically replaced in 120-day cycles. Cells are
replaced, by the way, to keE)p the body healthy.
We'll go into more depth about that next.

682 Transcripts
- -- --- -- --- -

-- - - ----- -- - ---- - - - --- -- - - --- ---- -- --- - ------- '

,,, ...

03 Zoology
M: Well, I hadn't really planned on.going over this,
but one of the students asked me about it, and,
well, I think it'.s pertinent to a discussion on
anthrozoology. Anyway, today's lecture is
going to be about the sport called falconry.
Since we're talking about the relationships
between humans and animals, we'll contrast
this discussion with our previous topic of people
and dogs.
Falconry is a sport that began in central Asia
about, oh, 4,000 years ago. In essence, a person
would raise and train a falcon and then use the
falcon to hunt. The falcon will hunt for food
when its trainer decrees, and it will bring back
whatever prey it finds. This correlates closely
with how we said that dogs and people hunt
together. However, the motivations are where
things start to diverge. We discussed how dogs
have a bond with people-they, uh, they want to
please their owners. However, falcons don't really
b6nd with owners. In fact, they really only hunt to
get a meal-not to please anyone but themselves.
We-uh, people-are actually the enemy, naturally.
That's probably why falconry isn't really a pgpular
sport, especially for hunting. But some people
still do it today, but mostly just because they,
uh, like falcons I suppose.

'

how its learning would be affected by different


intensities of motivation. Uh, in this case, the
motivation was an electric shock. Nothing that
would harm the mouse, mind you ... just a little
shock when it made a mistake, like if you pinch
yourself.
So, the researchers used a few different levels
of motivation-of electric shock-to see how the
mouse would react to it ... to see how it affected
the speed with which the mouse learned to pick
the correct exit. Any ideas on how the level of
motivation, the uh, level of shock, related to the
speed of learning?
M: I'd guess that the highest level of motivation
caused the mouse to learn the fastest. Seems
obvious.
W: It all depends, actually. They found that given
an undemanding task, higher levels of motivation
generally caused the mouse to learn faster. If
given a slightly more difficult task, a medium
level of shock actually helped the mouse learn
the fastest. The higher levels actually started to
affect the mouse negatively. With the highest
difficulty-th~ hardest task-the mouse actually
learned most easily with lower levels of motivation
-electric shock. And to the mice, choosing the
right door was well, kind of hard. Whef] you
consider it, it actually makes a lot of sense. If
you're $tres.sed out about taking a test, the
stress can cloud your mind, and you might not
perform very' well.

04 Psychology ;
W: Now, moving on, there's another experiment
that was done in 1908 that is also related to
learning, but it was trying to prove a slightly
different theory. The basic idea here was
motivation-how to, um, how motivation affects
performance, or in other words, how ... well,
let me just tell you about the experiment and
how it showed that excessive motivation can
actually hurt performance rather than help it.
M: I think I remember hearing about something
like this. It was a study done by, um, Yerkes
and ...
W: Yerkes and Dodson, yes. So, what they did was
put a mouse in, well. basically in a maze, where
there would De 'iillerent colored exits, either
white or black, ir; r2ndom places. And Yerkes
wanted to see 11cw !ong it would take the
mouse to learn to use only one of the exits, and

05

Economic~

M: Today's lec\ure will spark a lot of discussion,


and I just want to let you know that I welcome it,
so please f~eHree to jump in. I'm going to talk
about why .Marxism failed. Many economists
today agre~ that Marxism is basically just a
bunch of trite "maxims" that don't reflect reality.
But, uh, is ii? Well, why don't we get started?
Um, OK, SQ I guess one big shortcoming of
Marxism is. that it mainly emphasizes class
struggle an~, therefore, doesn't include at.her
social force.s that may divide a society, such as
race. Does. everybody understand what that
means?
.,.....,.
W: .Sure, you j[lst mean that class isn't the only
divicler in a soGiety, righr?

Transcripts 683

~,-

M: Exactly: But that's minor compared to the next


point. The uh, the next reason for the failure of
Marxism is so obvious, you're going to wonder
why Marx never thought of it himself.
Marx said that the value of a product, and
indeed the value of a business itself is a function
of two things: workers' efforts plus the value of
the machinery that workers use to make whatever
product it is they're making. Oh, and when
Marx said "workers," he meant specifically the
people who do the manufacturing of products.
OK? Seems like a, well, an erroneous assumption
if you ask me. Who can tell me why?
W: Umm ... I'll take a stab at it. So you said value
equals workers' efforts plus machinery. But,
um, but what about the value of management,
for one? I mean, the skill of management is really
what holds a business together and uh, and
advances a business.
M: Right! Anything else?
W: Well, sure. What about the value of marketing?
Like, what would some of these big, highly
succeooful companies be without marketing,
and uh, and salespeople. And accountants
and strategic planners and ...
M: OK, OK! You've got it, for sure. To say that the
yalue of a business is only a function of
manufacturing effort and the machinery is,
well, pretty na'ive.

in Impressionist works were too trivial. ni~y felt


like, well, that there wasn't any emotion. So they
stressed emotion in their paintings, and therefore
focused more on people. Should I sum up?
Couldn't hurt. When you think Impressionist,
think landscapes without much emotion, and
with the Post-Impressionists, ihink human subjects
and more emotion.
Now, moving on, there were also some artistic
methods-some techniques-that differed between
the two. With Impressionism, it's suggested
that Claude Monet one day just-uh, 'he just
decided to walk outside and start painting. He
felt like it was important to capture something at
that very moment, uh, in the state it was in. So,
what this translates to is very quick brushstrokes
and a focus on the way that the lighting-the
light at that time of the day affects a piece. But
with the Post-Impressionists, they generally took
a few different, uh, people like Van Gogh and
Seurat took more time with their brushstrokes.
Van Gogh used a lot of swirling patterns, while
Seurat made tiny dots of color to form a picture.
And instead of being concerned with lighting,
they cared more about capturing the emotion
of the subject than capturing the light of a
landscape.

07 Business
06 Art History
W: I noticed that many of you still aren't sure about
the differences between Impressionism and
Post-Impressionism. You got the similarities:
vivid colors, thick application of paint. Let me
make some further points to help you understand
the distinctions,.
The Impressionists started off as a group of
artists who just wanted to do something
different than .what was currently being
prescribed by the Academy of Fine Arts in
19'"-century France. The Impressionists departed
from tradition by painting contemporary
scenes. Additionally, they painted open-air
scen(ls and landscapes This is significant
because before the Impressionists, nobody
painted landscapes. On the other hand, the
Post-Impressionists felt like the scenes painted

- - . - 684

Transcripts

1\11: All right, we've been talking about advertising


theory for a long time. We'll be wrapping up our
advertising unit soon. Anyway, today we're
going to go over another popular method of
advertising anp we'll cover the, uh, the pros
and cons of it. Oh, before we move on. are
there any questions about attractiveness?
W: Um, excuse me.We never went over attractiveness.
M: Oh, you know ~hat? I used the term likeability.
But it's the sar:ne concept. That's my mistake.
Anyway, we'll probably return to that in today's
discussion about spokespeople. Let me get
right to it.
Well then, at the moment I want to focus on
owners who become the spokesperson for il1e
company. This has happened wiih some freque1~cy
throughout recent advertising. CaF dealers EJ8
the most li11ely to do this, I'd say. Anyway, asorry, loot my train of t11ought.
.

W: Owner spokespersons, uh, spokespeople.


M: That's right, thank you. I'll start with the advantages.
The first is really just a monetary advantage.
Some advertisers hire an actor-maybe
somebody famous. Sure, people pay attention
when they see a famous person on the screenwe 've already talked about that-but this
comes at a price, as you know. When it comes
to a company's owners, well, you don't have to
pay an owner. No money has to change hands,
and therefore, the advertising becomes less
expensive. Another advantage is that it can
give an advertisement a sense of authority. If a
message is coming straight from the owner of
the company, it can give an authoritative vibe,
uh, feeling, to the message. People might be
more likely to believe what the speaker has to
say.
OK, well, an owner-spokesperson doesn't
always work. Back to likeability. First of all, well,
some people just don't belong in front of a camera.
I'm re you've seen a commercial where some
guy,J~ standing there, stiff as a board, reading
lines off of a card in a monotone voice. Not an
effeptive way to advertise. This is because
some people just aren't good at delivery. And
no one wants to watch someone like that.
Second, sometimes the opposite happens. If
you have someone who is too excited, too
egotistical, or whatever the case may be, it will
give people a bad impression of the company.
It just sends the wrong message when someone
gets 'up there and, I don't know, thinks they are
better than everyone else. Of course, the point
of advertising is to say you're better than the
competition, but you have to do it right or it just
alienates potential customers.

M:
W:

M:
W:

M:

W:

IVI:

08 Zoology
W: Good morning, class.
Students: Good morning, professor!
W: Wow, you're all in a good mood today. That's
good, because we're going to be going over a .
pretty, uh, involved and difficult topic. So, I
recommend a lot of participation today
because it is really going to help you understand
these concepts.

W:

IVI:

W:

Anyway, we're going to be going over the theory


of memory in animals, and I'm going to support
the theory with a specific example ... but I'll
save that for a little later. First, to introduce it all,
what do you think the relationship is between
animal size. and memory function?.
Um, I'd have to guess that the bigger the animal,
the better the memory.
Actually, there's not a concrete correlation to
be seen here. So, this leads us to the topic for
today. I want you all to understand that some
animals have much more developed memories
than you would have ever anticipated. One
such type of animal is, well, an insect.
An insect? I can't imagine a tiny insect having a
great memory.
Well, there's a study that substantiates this fact.
The study involved an experiment where a
group of researchers placed a circle of pine
cones around a female wasp's nest. The wasp
was inside the nest when this happened. So,
what the wasp did-the wasp left the nest and
started checking out the pine cones, getting
acquainted with their positions, making sure
they weren't a threat ... that kind of thing. So,
when the wasp left the area, a researcher
moved the circle of pine cones a little distance
away from the nest. It was the same pattern,
but it was few feet away from the nest.
What, and the wasp came back and thought
the nest was still in the middle of the pine
cones?
That's exactly what happened. The wasp flew
straight to the middle of the pine cone circle
and buzzed around for a little while, seeming
confused. So, what is it that we can deduce
from this experiment?
Well, it seems to rne that wasps have extremely
developed memories.
Perhaps. I mean, it certainly proves that wasps
have some sort of memories, probably more
than we would have imagined.
Well, doesn't it suggest something about the
nature of a wasp's memory? I mean, how it
works?

That's what I was getting at. I mean, in a way


t11at's a whole different topic, but I did want to
put that out there.

Transcripts

685

---!

09 Psychology
W: Today's lecture is going to be very interesting.
During the course of the semester, the nature
versus nurture debate has come up a couple of
times, and today I'm going to present both
sides of the argument.
Before we start, let me just reiterate the basics
. of the debate involving how we develop
personalities. The idea is this: some people
believe the intrinsic qualities of each individualthe nature of a person, so to speak-are more
influential than the qualities that come from the
individual's experience-the parts that are
assumed to have been nurtured. Then there're
people that believe the opposite. Are we all
more or less clear about that? OK, let's move
on then.
Anyway, let me start with the nature theory,
which would assume that people's personalities
arise as a result of their innate traits, traits they
were born with. The strongest argument for the
nature theory is heredity. In order to test this
theory, scientists usually test twins. For example, a
lot of twin studies go like this: they'll observe twins,
who obviously share the same genetic traits.
The thing is they'll be in different environments.
And time and time again, the results have
shown that the twins exhibit striking similarities
in terms of personality. Nature supporters
argue that this wouldn't be possible if the twin's
genes didn't influence their persooality. So, to
recap: the nature theory says that people are born
with certain traits that affect their personalities.
I hope that's clear.
Now, let's not forget that there's some pretty
compelling evidence on the other side of the
argument as well. The nurture argument is
based on the idea of tabula rc1sa. Tabula rasa
is Latin for "blank slate," and the concept is
basically that humans are blank slates when
they are born, uh ... fresh tablets waiting to be
written on, so to speak. As we grow up, we
learn. from the world around us. I want to bring
up B. F. Skinner, who will sound familiar to you
because we studied his work on conditioning,
responses to conditions. What Skinner did is
actually very important to this debate because
it is completely based on the hypothesis that
behavior is learned and not innate. He suggested
that we learn everything after we're born.
Genes aren't involved in personality. It's all

.,----- -686 Transcripts


'~~-----

about environments. Now, keep in mind, we


can combine these two theories, and they can
both be partially right. They aren't mutually
exclusive.

10 Art History
M: Today we're going to talk about two important
art movements of the 20 century: cubism and
surrealism. I want our focus to be on how they
are constructed, rather than any sort of historical
stuff. So, we'll start off by talking about how the
cubists thought of their art. Let's open up the
discussion. Any thoughts? Yes?
W: Well, I seem to remember reading about how
cubism began. It was, uh, Picasso and ... uh,
Georges Braque. They thought of art in terms of
little cubes.
M: That's true. Let me expand on that. What
cubism-cubist art-tries to do is take apart a
scene or a subject, and tries to put .it back
together using geometric shapes, like cubes,
hence the name cubism. Is that clear?
W: Well, the thing I wonder is ... I mean, everything
is made of shapes anyway, right? So how is
cubism really different?
M: Good question. putting the shapes back together
really isn't all of it. The most important part of rt
is that the shapes have to be reassembled in
an abstract way. So, maybe a cube goes at the
end of an arm where a hand usually would be.
Therefore, it's more abstract. Does that answer
your question?
W: Yeah, and actually, can I say something about
surrealism? I mean, if we're done with cubism.
IVI: Sure, go ahead.
W: 01\, so, from what I recall, surrealism had the
same idea of distorting reality, right? But the
surrealists did It in a different way. I remember
someone saying that the best way to describe
surrealism was to think of it as putting real
things in a dream world.
.
M: You know, I like that idea. I'd say surrealism rs
very much like that. But, in artistic terms, they
took actual things and merged them in strange
ways, rather than re-assemble them in different
shapes. So, like in Dali's works, you might see
clocks on a table out in the desert, melting
clocks, and a tree growing out of the table You
wouldn't usually see those things in real life.

'1

right? At least not together. So, it's almost like


being in a dream world, even just looking at the
painting.

01 Biology.
M: Recently, everyone's been talking about the

'<

~?.

)',

sun as if it's the enemy-some even suggesting


that we avoid it altogether. However, I want to
go over some of the benefits of sunlight today,
and, um, how it can be good for your health.
First of all, the sun provides us with vitamin D
that our bodies can use. Vitamin D is essential
for our health-we need it to absorb calcium.
While we can get uh, a small amount from
foods, such as liver and egg yolk, the sun is
actually our major source of vitamin D. Just 20
minute,s of sunlight during the summer causes
your bpdy to produce 20,000 units of this vitamin,
when the minimum you need is 2,000 units.
Secondly, research suggests that getting out in
the sun' can actually help you be, well, happy.
Sun exposure-and, uh, this can be hard to
grasp, so let me give an example. Some people
start to feel down during times when they don't
get much exposure to the sun, you know, like in
the winter. Evidence suggests that exposure to
sunlight can actually reverse this trend of
unhappiness. There are many theories that try
to explain why this happens, but they all agree
that it is related to sunlight.

02 History
M: A symbol, as you all know, is a thing that stands
for, um, represents something else. A tiger, for
instance, may symbolize power. An interesting
thing is that t11e meaning of a symbol might
change over time. For example, the Bastille
was a prison in Paris, and yet somehow, it has
come to symbolize freedom. Let's talk about
how that happened.
First, let's locus en the onginal-uh, what the
Bastille used to be, used ro stand for. The
Bastille was a p1ison-tl1is is cluring the rule of
King Louis the sixteenth. We're talking about

the late 1700s. At that time, it was for those who


acted, or wrote, or spoke rebelliously against
the French government. It represented a place
of political oppression, um, and the absolute
monarchy that controlled France. Anyone who
said anything that the government thought was
rebellious was put in the Bastille.
But this all changed when the people of Paris
got fed up with the government and decided to
free the prisoners. In what's referred to as the
storming of the Bastille, an angry crowd
attacked the prison and released everyone in it.
This event actually triggered the French
Revolution, which brought great change to
France. The people were finally free of the
absolute monarchy. So, as you can see, the
Bastille actually became a step toward France's
freedom, and I think you can understand why
it's now a symbol of freedom, too.

03 Office Hours
M: Hi, Professor Peters? I was hoping you had a
moment to talk .about the upcoming test. I didn't
do very well on the last one.
W: Sure, Paul. Would you like to talk about how you
can prepare for the next one? I guess I'm
notorious for being extremely tough.
M: Yeah, well, it is Organic Chemistry, after all.
W: True, it's a very difficult subject.
M: I know, but it's really frustrating ...
W: Well, you know what? Since the department
acknowledges that my class is hard, they've
required that I post all of my older tests online.
!VI: Oh, right! I had almost forgotten about that!
W: Well, you'd be surprised at what a great study
aid those tests are-they'll help give you an
idea of what to expect. Just go to the course
webs'rte.
l\il: OK, I will.
W: And also, the department has created online
tutorials for this course. I'm sure I've mentioned
that in class.
lVI: Again, I've been so caught up in trying to read
the textbook that I'd forgotten about the
supplementary materials. I'll definitely check
them out.
W: Yes, read the text-certainly do that, but these
extra things will help reinforce what you're
learning. '
Transcripts

687. -

04 Service Encounter

05 Environmental Science

M: May I help you?


W: Hi. I'm looking for some information about getting
a job here on campus.
M: Are you looking for a specific type of job?
W: Hmm, well, I'm particularly interested in working
in food services. Are there any positions that
are open?
M: Indeed there are! They are always hiring.
There's a rather lengthy hiring process, though.
Now, there are different areas that you could
work in---the cafeteria kitchen, or at one of the
delis on campus, or in the bakeshop. And
there's also catering, which is affiliated with the
university. You'll have to decide which one of
those you want.
W: Actually, one of my friends works with the catering
service, and she said that she really likes it. I'd
like to work there.
M: OK. We have all the applications for Food
Services here at the employment center. Give
me a sec. They should be around here ... here
we go. You can fill it out now if you want.
You should also be aware that you're going to
have to attend an orientation session with Food
Services if you get hired. They always hold
those on Mondays, actually. All new staff need
to attend one. of those before they can start
working.
W: Monday shouldn't be a problem.
M: OK, then, I'll go ahead and pencil you in for an
official interview. One of the managers from Food
Services will meet you here in the employment
office. Can you come here at eight a.m. tomorrow?
W: Um, yeah, that should be OK. Is there any training
I have to go through before I can start working?
Because I'd like to start working as soon as
possible.
M: Yes, there is some training. I'm glad you asked
that, because that's the last step. Food
Services does on-the-job training. If you're
hired, you'll actually start working right away,
but technically you'll be training.
W: Wow, that's great to hear. So, just come bacl<
tomorrow morning for the interview?
M: Yep, at eight a.m.
W: Great. Thanks for your heip.

M: OK, so we'll be talking about mountains,


specifically, why they get taller. Can anyone tell
me what the tallest mountain is on Earth?
W: Mount Everest.
M: Correct, and it's getting taller, too. Well, I'll be
talking about some mountain ranges today-the
tall ones, that is, and what causes them to be
so tall, and even become, over the years, even
taller. OK?
So, some mountains are affected by plate collisions.
You'll remember that the Earth's crust is made
of plates, and those plates can move. First of
all, the Andes were created when the Nazca
plates began to slide under the South American
plate, which you'll remember is called subduction.
Experts created a computerized model of this
range and discovered that the reason why the
Andes are especially high is that the South
American continental plate is particularly longuh, 7,400 kilometers, and this length has
allowed more room for the plates to push
together against each other at different places,
resulting in a huge "w" shape. That's a huge
subduction zone! OK, so we've got subduction
as one process for pushing up mountains.
Now, let me pick on someone-Mary, can you
give another reason that might cause mountains
to grow-uh, I mean old mountains to become
taller?
W: Um ... well, I can think of a lot of reasons why
mountains would get shorter, like erosion ...
but, um, no, I can't think of why one would get
taller.
M: I'll help you out, then. Actually one reason for
taller mountains is global warming. The
European Alps, especially the French Alps,
have actually been growing over time. So
here's what's been happening. Where glaciers
exist, there is some pressure on the Earth's
crust, causing it to push downward .
because they're so heavy, that is. So when the
ice begins to melt, the Earth's crust, then,
begins to spring in the opposite direction-up.
It's just like a spring, actually, so imagine that.
This pushes these mountains toward the sky.
Interesting, huh? Who would ever guess that
global warming could make mountains grow
taller?

688 Transcripts
--

-----

--

--

------

06 History

W: Good afternoon, everyone. Today we'll be

,-:

;-;;:;:;

'

exploring how Confucianism has affected


northeastern Asian countries, such as China,
South Korea, and Japan. Confucianism is an early
collection of ethics and social and philosophical
ideas that are based upon the teachings of the
ancient Chinese philosopher Confucius. The
basic idea of Confucianism is pretty much how
each person can participate in creating a
peaceful, well-organized society. Each of the
nations I mentioned took Confucianism and
modified it to suit its particular tastes.
First of all, Confucian ideals shaped the social
relations of many people in northeastern Asia.
For example, its influence in Japan can be
seen in the way that order and relationships are
understood. Confucianism stresses the importance
of individual relationships, basically, how people
relate to each other in terms of their place in
society. It's like in a family ... parents and children
know their. places. So everyone should know
their place in society. Many Asian countries
have taken this very seriously, and they enjoy a
lot of, uh ,'.social harmony because of it.
Secondly.people have found ways to apply the
idea of Confucian social harmony to business.
Um, Confucianism actually discourages seeking
profit-Confucius said that profit led people to,
uh, do wrong to other people. Well, that idea
wouldn't be a good thing to apply to a business.
However, the idea of harmony can really help a
business run smoothly.
Confucianism's ideals include an aversion to
conflict ... so without conflict in the workplace,
things run more smoothly. Imagine a workplace
where everyone worked together in harmony.
In addition, along the same lines as social
relationships, if everyone knows their place in
the workplace, there are no power struggles,
and businesses can operate without any of those
complications. If workers aren't just trying to,
you know, climb the corporate ladder, they can
instead focus on sharing ideas and cooperating.

:q Office HOU rs
W: Hi, Mike. Come in.

M: Hi, Professor Oliver. Do you have a moment?


W: I sure do. What can I help you with?
M: Well, I really want to take some courses over
.
the summer here, but the problem is, I'll be living
at home, which is nearly four hours away. I
couldn't even consider commuting that distance.
W: No, certainly not! That'd be eight hours out of
your day just for traveling.
M: Exactly. But I don't know what to do. I can't live
away from home because one, I can't afford to
get an apartment, and two, my mom really
wants me home.
W: Well, have you considered taking online courses?
M: You mean the ones offered by the university?
I've considered them, but really, I don't think I
could afford them.
W: No, no, actually, they're less expensive than
conventional courses you would take here.
M: Really? I had always thought that they were
pricey.
W: No actually, in my experience, they don't cost a
lot, and they're well worth it. You know, I actually
received some of my continuing education
credits while taking online courses, so I have
an idea of what to expect. They're not particularly
expensive at all, especially compared to what
you'd have to pay for another semester of college.
I assume that the reason you want to take summer
classes is to graduate earlier.
M: Yeah, I'm actually a bit behind at the moment.
So it'll help me graduate on time if I can take
some summer courses. I haven't come across'
anyone who has taken on line courses until now.
So I won't miss out on the vital parts of the
classroom experience? Like lectures and real
communication?
W: Well, you're a sophomore. I mean, you have two
more full years of classroom experience ahead
of you. So you should get plenty of experience
that way. Is it the motivation you're worried
about, like that by being at home you wouldn't
be as motivated to complete assignments?
M: No, I mean, I just worry that there'll be no
interaction with tl1e professor ... misunderstandings
W: Sure, lectures can be a great way to learn, but
t11ey aren't the only way. You know, when I took
one of my courses online, we did all of our
discussions on an Internet message board,
and I thinl< it allowed people to participate more
than they usually would in a classroom.

Transcripts

689 _ ;

---~- ~

M: That's a good point.

W: And some of the lectures will be text-based, so


you can reread what you don't get, and maybe
even learn better that way.
M: Hmm, this is something to ponder.
M:

08 Service Encounter

W: Hi! Can I help you?


M: Yes, please. I'm here to ask about the student
activity fee.
W: Sure, any specific questions?
M: Yeah, first of all, what exactly am I paying per
semester?
W: Well, it all depends on your enrollment status.
How many credits are you signed up for?
M: Fifteen. I'm a full-time student.
W: OK, meaning you're paying the full-time fee.
Let's see. Well, looking at this chart, you ... are
... paying $560 per semester.
M: Whoa! That's steep! Do I really have to pay
that? I mean, there's no way to waive that fee?
W: I'm sorry, but it's a mandatory fee for all
undergraduate and graduate students.
M: It just seems like an awful lot. Well, my second
question is, what exactly is it for? If it's going to
be so inuch, I might as well find out if I can get
any benefits from it.
W: Certainly. The student activity fee 1s mainly
used to fund two major things: campus events
and campus services.
M: Campus events? Like what? I haven't been to
any this year, not that I'm aware of.
W: Do you remember Spring Weekend? There
were tons of concerts, some big-name bands
that came here to perform, and lots of events
like comedy shows that ran pretty much all
weekend.
M: Oh, yeah. When was that? In March? OK, but
that's once a year.
W: Well, that's one of our bigger events during the
school year, but we always have other, smaller
events going on all the time. For example, we
have free barbecues every month-l1ave you
been to one?
M: No ...
W: Really? But they're always being advertised ...
Anyway; there are also movie viewings every
Thursday night. And special guest lecturers

- - 690 Transcripts

W:

M:
W:
M:
W:

from time to time. If you keep your eyes and ears


open, you'll find that there's. always something
that's being arranged by the student government
and by various student organizations. They
work really hard to keep up a lively atmosphere
on campus.
Didn't you also mention services?
Yes, we have a wide array of available services,
such as health care, student-produced publications
like the weekly newspaper, and free on-campus
transportation. There are also computer centers
located all around campus, special library
facilities, and most recently, the night walk
program.
What's that?
Well, we have groups of people that escort
students around campus after dark.
Hmm, I didn't .know that.
Yes, you see, though the student activity fee
may seem like a lot, if you consider all that it
includes, it's really not at all.

''\
'.:"

,.'.~
f

~~

,.

t
f,

~}_

"

~:
~

f-'

:{-

'

,,-,

k
"

J
\:_
"~-{
);

09 Astronomy
W: Continuing with our progression through the
planets, today we're going to talk about Jupiter.
Um, specifically I want to focus on how weather
patterns relate to how it looks. Jupiter has an
amazing look to it ... striking bands of color,
bright spots. Today we'll go over what makes
Jupiter look that way.
Jupiter's weather can be attributed in large part
to the rotational speed of the planet. Let me talk
about that for a second before I get into
Jupiter's appearance. So, Jupiter rotates really
fast, much faster than Earth. And as we'll see in
a second, this causes some interesting things
to happen.
OK, first I'll start by talking about the atmospheric
bands of color on Jupiter. The bands look like,
well, like strips of either a dark red or a lighter
red, almost sandy color. Well, these colored
bands are caused by convection. Uh, clo you
all remember convection from our previous
classes? Well, let me remind you, just in case
.. Convection refers to warm air rising and cool
air falling. As warm air rises, it takes with it
atmospheric gases. On Jupiter, the rising air
makes lighter bands, and the cooler air sinks

"'

;'

'

and creates darker colors. Does everyone follow


so far? And here's where the rotational speed
comes in. Because the planet is rotating so
fast, these bands wrap all the way around the
planet. Let's compare that now with what's
going on in the Great Red Spot. Now, first think
of hurricanes as they occur on Earth. They travel
over water, build strength, and then dissipate
once they reach land. OK, wi_th that said, the
Great Red Spot is a hurricane, though
researchers estimate that it has been going on
for the past three hundred years. It's also very
big ... actually, it's about the size of Earth, it
not bigger! Now, this sounds very different from
Earth's hurricanes, right? Well, there are a couple
of reasons tor this. First, Jupiter's rotational
speed comes into play again.
Scientists believe that the rotational speed
helps to sustain the storm, though they aren't
exactly sure how yet. In addition, there are no
continents, uh, land masses on Jupiter like we
have here on Earth. Hurricanes lose their
11strength on Earth when they hit land, right?
't:iJVell, there's no land on Jupiter to do that, so it
-just keeps going and going and going ...

10 Biology
M: All right, so we'll be discussing alleles and
hopefully, by the end of this session, we'll be
able_ not only to define them, but understand
them as well. Here's what .I think I'll do ... I'm _
going to go ahead and give you the basic
definition, then I'll show you how alleles workshow them to you in action, and hopefully then
you'll have a very clear idea of what they are
and what they do.
Alleles are a pair or sequence of genes that exist
at a certain location on a specific chromosome,
a chromosome being a strand of DNA that
holds our genetic information. So, what you
need to remember about alleles is that they are
the part of DNA that determines a genotypegenotype referring to cl1aracteristics-genetic
characteristics of an individual. OK, that was a
lot of information. I see you all frantically taking
notes. Are there any questions at this time?
W: Well, yeah, I'm not sure I understand yet what
alleles do. They're part of DNA, but what do
they do in DNA? I mean, what's their function?

M: That will be my next point, so if there aren't any


other questions, I'll get right into that.
OK, so on to the role of alleles. In essence, alleles
determine what traits people inherit as part of
their genetic makeup. Inherit from their-from the
individual's parents.
W: I've always wondered how that happens. I
mean, J look almost exactly like my mom, but
not my dad. Does that have something to do
with dominant genes-or I guess, alleles?
M: You're exactly right. To talk about the role of
alleles, we need to talk about the types. So put
types in your notes. There are actually tour, but
we will go over two of them now and two of
them tomorrow. The first two are dominant and
recessive. Luckily, these are the two that are
the easiest to understand.
W: Because they're opposites, right? Dominant
means they dominate, and recessive means ...
M: Well, yes, but let's put it in clearer terms than
that. Traits need at least two alleles. A dominant
allele will cause the trait to materialize if just one
allele of that trait is present. So, let's use the
brown eyes trait and say it is a dominant trait. It
a child receives one brown eye allele and one
blue eye allele, what color will the child's eyes
be?
W: Brown.
M: Right. And a recessive allele means that two of
that same allele are required to make that trait
occur. So, if we say that blue eyes is a recessive
trait, a child would need to have two blue eye
alleles to be born with blue eyes.

!1 Environmental Science
W: Now, I lrnow everybody knows what a sea is,
but I'm sure that if I asked around right now,
we'd hear a lot of different answers about what
features make up a sea. Another thing that a lot
of people don't know about is that there are
different types of seas. So, uh, that's what
we're going to go over.
OK, the first type of sea is called a mediterranean
sea. And just so we're clear, although the
Mediterranean Sea is an example of a
rnediterranean sea, in oceanography we use
Transcripts 691

--j
i

the term to describe a sea that has limited


exchange of deep water with the oceans. So
how doi:is the water circulate? Well, it mostly
depends on the temperature of the water,
which moves the water from place to place.
The density of water, due to salt dissolved in it,
also changes as its temperature changes. The
denser water sinks, pushing less dense water
toward the surface. Some other examples of
mediterranean seas are the Caribbean Sea
and the Red Sea.
Next, uh, second, there are marginal seas.
These are really areas of the ocean that are
somewhat enclosed by land. I mean, they may
have some islands around that help define their
boundaries. The big distinction to. remember
with marginal seas is that their currents are
mainly caused by ocean winds. Some examples
of marginal seas are the Bering Sea and the
North Sea.

02 Biology
M: We've been talking about marine life lately.
Today I want to talk about respiration. I thought
it would be interesting to talk not only about
what mechanisms allow organisms to breathe
underwater, but how it's done on land as well.
So, to get started, let's talk about breathing
underwater, or aquatic respiration. Here's a
thing to remember: there is relatively little oxygen
in water. So animals that live underwater need
a system that allows them to process oxygen
differently from land animals. To do this, fish
have developed gills. Gills allow organisms to
extract oxygen from water.
Later, we'll talk some more about how gills
work, but right now, I want to contrast that
information with a common system of respiration
for animals that live on land. Since respiration is
really just an exchange of gases-you know,
where carbon dioxide is swapped for oxygenfor simple, single-celled organisms, breathing
is as simple as constantly being exposed to the
air. For those organisms that have more than
one cell, it's a little different. An example is
vertebrates, which have lungs that allow them
to breathe. Special cells in the lungs saturate
blood with oxygen. OK, let's contrast this with a

692 Transcripts

detailed analysis of what happens in fish.

03 Office Hours
W: Hi, Professor Cope. I know your office hours are
almost over, but I really wanted to talk to you
about something.
M: All right, but let's make it quick.
W: Sure. All I wanted to do was ask if it would be
possible for me to switch my group discussion
section to another one.
M: Which of the two are you currently in?
W: I'm in the Friday section that starts at 1O a.m.
It's led by ... oh, what's the teaching assistant's
name? Oh, Laura.
M: Would you mind if I ask you what the problem
is? Obviously, it's completely confidential, but if
it could help me coordinate my teaching
assistants better, I'd really appreciate it. I want
to make sure they're getting the job done.
W: No, no, no. It's not Laura at all. In fact, she's
great. It's just that I have another class that
starts at nine a.m. It normally wouldn't be a
problem, since the class officially ends at 9:50,
but the professor just keeps going on and on. I
just feel bad always showing up to the discussion
group at least ten minutes late.
M: Oh, I see. Let me just make a note of it in my
roster. So you'll be changing to the Thursday
night section with Bill.

04 Service Encounter
W: Hi, I'm Marilyn, the housing coordinator. Can I
help you with anything?
M: I was told to come here about a problem I was
having. It's not really a problem ... it's just
something I have to take care of ... soon.
W: OK, so what exactly do you have to take care
of?
M: I was assigned a great room. It's really nice and
all, like, I really like the private bathroom, and
the closet space is great.
W: You know, usually we don't have students coming
in to talk to us about how great their rooms are.
I have a feeling that there's a complaint
somewhere in there.

M: Well, like I said, Barbour Hall is really great,


butW: Wait, did you say Barbour Hali? But that's a
girls' dormitory.
M: Exactly, which is a shame, because it's probably
the best housing on campus. But I'm assuming
that there was just some mistake in the computers
and I was assigned to the wrong dorm.
W: OK, let's fill out a "Change of Residence" form.
I know I have them here somewhere. f:jold on
just a sec. Here it is. All you have to fill out is the
top part. And please make sure to put your
Student Identification Number on there.
Otherwise, the only way we'll have to track you
down is by looking for the only man in Barbour
Hall.
M: OK, this looks pretty straightforward. Oh, wait,
do I really need a copy of my social security
card? If I do, I'll have to go back to my room to
get it. I don't usually carry it with me.
W: Oh, since you've already been assigned a
room, you're already in the system. That's fine.
You can leave that line blank.

and online resources to push them forward. In


contrast, a new business justwouldn't have the
supply network or the customer following to
make a bricks and clicks model work. So, what
does make the model work?
Well, I kind of just mentioned this before, but let
me highlight the advantage of having supply
and distribution worked out already. Retailers
don't have to worry about logistical stuff, and
they get more choices about their delivery
policies or even price discounts. Does that
make sense?
W: What other kinds of advantages does it have?
M: Well, how about stability? Unlike purely
Internet-based businesses, bricks and clicks
businesses usually have years of experience
under their belts. That perceived stability helps
bring people back, because remember, people
like to go with businesses they trust.
If there are no questions, let's move on to the
disadvantages of this model.

"r.~

,.

'-

'

;'.t <'

06 Anatomy

05 BusinesS:M: Good morning, everybody. Last time, we talked


about various online business models, but
today's model is a little different because it
incorporates both online as well as offline
elements. It's called the bricks and clicks.
model. The name sounds funny, but it works.
Let me give you an example. Think about the
last time you wanted a new, um ... let's say
radio. Maybe you went online and checked
prices at different retailers. And then you find
that you can even order the radio online. If you
want it right away, you could actually go to the
store and pick it up rather than wait for it to be
delivered. That's one example of how the
bricks and clicks model works.
Now, right off the bat, can anyone think of the
kind of business that would be best-suited to
follow this kind of model?
W: I would imagine that companies that have a
strong brand or some sort of presence in the
market already.
M: You're absolutely right. It's just more feasible
for those types of businesses-and we'ri;i talking
about retailers really-um, to use both otfline

W: Last week, we left off talking about the major


human organ systems. We learned that there
are eleven different systems, all of which serve
very important functions. Today, I'm going to
take some time to expand some more on the
integumentary system. In case you don't
remember from the last lecture, the integumentary
system comprises the external covering of the
body. So, we're basically going to be talking
about skin, hair, and nails. To start with, the
integumentary system is the largest organ system
that humans have, at least in terms of surface
area. Anyway, the system has different functions.
First, it provides protection for internal organs.
I'm sure you can all imagine how terribly vulnerable
we'd all be if we didn't have any skin to cover
up all our organs. But also consider that skin
helps keep out infectious elements as well. So,
we can consider our skin a shield for. us, and
we'd be-well, just be happy you have it.
Another function of the integumentary system is
that it helps humans control their temperature.
How does it do that? Well, it acts as both an
insulator and a regulator. between our internal
organs and the external environment. The skin

Transcripts 693

_-_;

J_cc:.c~.:_

---- _-'.___:_.--.. --'-::_-,

-i

'

' mechanisms that help regulate


also has certain
body temperature. Perspiration is an example of
this. See, when we produce sweat, the evaporation
of the sweat has a cooling effect. and it helps
keep us from getting too hot. Because of our
skin, our bodies avoid the damage that can
occur from abrupt changes in temperature.
Finally, the integumentary system is also important
for sensory perception. Um, I mean, for the
ability to feel things. If you think about it, I'm
sure it doesn't take much to appreciate the
importance of the skin as a receptor for touch,
pain, or temperature changes.
07 Office Hours
W: Hi, Professor Daniels. I hope I'm not bothering
you. I had some questions about the paper you
assigned today in class.
M: Hi, Isabel. It's been so long since you've come
to visit me. What exactly can .1 help you with
today? Didn't you like the topics I came up
with?
W: Oh, that isn't it at all. The one on the comparisons
of theories of change was especially fascinating.
I think that topic would be so interesting to
explore.
M: But you seem to have something else in mind.
Care to share?
. W: Ha, I guess you know me pretty well. I suppose
you would by now; I've lost count of how many
classes I've taken with you.
M: I have as well. Anyway, about this paper topic.
You're keeping me on the edge of my seat
here.
W: Oh, of course. Well, there's really nothing special
about my question. What I was thinking is that
I'd like to work on the paper I wrote earlier this
semester. The one on Plato's theory of forms.
Do you remember which one I'm talking about?
M: Of course. Um well, why don't you tell me some
more about what you would be writing about
exactly?
W: The last time, I really only discussed the main
points of his theory. Since then, I've read much
more, and I think I want to explore different
aspects of the theory.

- - 694 Transcripts

M: Can you give me something more tangible?


Just so I have a good idea of what you're aiming
for here.
W: Well, in concrete terms, I want to talk mostly
about the philosophical concerns that motivated
Plato's theory of forms. It would probably
arnount to about ten more pages-at least the
materiall'm planning on covering.
M: That sounds really interesting. I'm having a
hard time deciding if I should let you do this,
though. Somehow it seems kind of unfair to the
rest of the class. Everybody else will be writing
on the newer material, you know?
W: Well, if you'll allow me to make another point on
my behalf-the reason I want to expand on this
paper in particular is because I'd like to submit
it to the department for publication in the
department magazine.
M: Oh, that would be great. I think so many people
would really enjoy that. OK, I'm sold. You can
do it, but you'll have all the same due date and
guidelines as the others.

08 Service Encounter
W: Next, please.
M: Hi, I need to talk to someone about my student
health insurance. I received my policy information
and my card in the mail a couple of days ago,
but I'm still confused about some things.
W: OK, well, I can help clarify it'for you, if you'd
like. Most people need an explanation.
M: That would be wonderful. I don't know, this
insurance stuff is just strange. I mean, I'm glad
that the university provides insurance for all of
the students, but so far I don't have any idea of
what's covered and what's not covered, or
whatever.
W: I know. The worst part is that insurance policies
don't ever really get easier. I think people just
get used to them. But anyway, you already
know that you're covered under the university
health insurance. And just so you know, it'.s not
exactly free. If you look at your student bill at
the beginning of the semester, you'll see a
Health Services Fee. That's what the university
charges you to be covered in the plan.
M: I didn't know that. I was wondering what that
was all about. I'm glad I didn't decide to take it

"'

up with the bursar's office after all. Anyway, I


guess my biggest question about this is about
what coverage I get.
W: The plan is pretty comprehensive. You have
free doctor's visits, which would include routine
check-ups or physicals. Now, if you're going to
have any lab work done, it's um, let me check
on this, that's right, a $15 fee that is not covered
by your policy. So that's out-of-pocket. You'll
also have to pay for X-rays. Oh, and since we're
talking about X-rays, you should know that a lot
of X-rays have to be taken at facilities off-campus
because we don't actually have the equipment
to do all types of X-rays.
M: Oh, OK. And how about dental?
W: Unfortunately, the basic plan does not cover
any dental work. What the university suggests
for students who want dental coverage is to
either seek out another insurance provider, or
check to see if you are covered under your
parents' plan.
M: .Gotcha. Well, I think that about answers my
._;;questions. I guess I should take a look through
''the policy book as well. Thanks a lot!

09 History
W: Continuing our discussion of the Spanish conquest
in Mexico, today, I want to briefly discuss an
indigenous group, called the Purepechawhich still (JXists today, by the way-who.
offered some of the strongest resistance
against the Spanish. So I'll just start by defining
the tribe for you-saying a little about them-but
the main thing I want to focus on is on, um, a
couple of theories that try to explain how they,
uh, how they were able to survive as a culture.
OK, let's go. Um, compared to other tribes, very
little is known about the history of the
Purepecha. Anthropologists are still pondering
their exact origins, but is it, I mean, it's widely
believed that they came from somewhere in
South America. Anyway, what I want to segue
into is that the ter:itory that they inhabited is
Central America-modern Mexico. This is of
particular significance because of the tribe's
relation to the Aztec empire. Are you all following
so far? Let me explain a !ittle further, then. Of
the little bit of Purepecha history that has been

reconstructed, it is known that they were


involved in numerous battles with the Aztecs. In
fact, it was one of the few tribes that were able
to successfully resist the Aztecs. Historians
believe that two factors played a big role in
being able to do so.
First off, unlike any other tribes in the region,
the Purepecha had knowledge of metallurgy.
This is one of the reasons that anthropologists
think that they came from South America.
Anyway, their skill for working metal gave them
the capacity to create advanced weaponry.
Additionally, Purepecha warriors have been
described in many different accounts as having
worn armor. It's assumed that the advanced
weapons and body armor enabled the
Purepecha to defend themselves and their
lands more effectively.
Next, the Purepecha apparently had a military
organization that was pretty much unmatched
by any other indigenous group in the region. In
size, I mean, and in organization, training, and
such. Let me share something I saw in my
readings recently. Not surprisingly, the Aztecs
attempted to invade Purepecha lands on
numerous occasions. In 1478, the Aztecs led a
battle against the Purepecha. However, they
were badly defeated when more than 40,000
Purepecha soldiers met the Aztec forces,
comprised of 24,000 soldiers.

10 Ecology
W: Now, both of the relationships we'll talk about
involve two species that are in direct contact.
To begin, who can tell us what parasitism is?
IVI: It's when one organism literally lives off of
another one. Oh, and the animal that the parasite
lives off of is harmed, while the parasite benefits.
W: Good. You just covered the two points that I
would have used to describe a parasite.
M: I almost want to compare a parasite to a regular
predator. I mean, I guess it's different because
the parasite is usually smaller than the host,
whereas a predator is usually larger than its
prey. But I guess that's mostly off topic. Sorry.
W: No, don't be sorry. That's a very interesting
observation. And along with that, I'd also like to
point out that some parasites work from the

Transcripts 695

inside, while others do their damage from outside.


So, can anybody give an example of a parasitic
relationship in nature?
M: How about ticks? They attach to animals and
feed on their blood for sustenance.
W: That's a great example. I know that most people
think that the world would be better off without
them, but even parasites have a role in nature.
M: Really? So, what are ticks good for?
W: Well, like they help control populations of
species that could do a lot of damage to an
ecosystem if their numbers weren't kept low.
Uh oh, we're kind of running short on time.
Maybe we should move on to mutualism.
Definition, anyone?.
M: Isn't it just like the opposite of parasitism?
W: Not quite. I mean, you're not wrong. But I'm
looking for an answer that goes a step further than
tbfil, Why don't you describe the characteristics
of a mutual interaction?
M: It involves two species interacting, except that
instead of one bringing harm to another, they
both sort of mutually benefit. Right?
W: Good. In which ways can a species benefit
from this kind of interaction?
M: They can use each other for protection. I
remember reading about birds that would eat
the flies and insects off of rhinoceroses.
Obviously, going around with a rhino offers the
bird some protection. But the bird also makes
loud noises when there's a predator around, so
it helps the rhino protect itself, too.
W: Great example!

01 Computer Science
M: Good afternoon, class. Last time, we barely got
started in our discussion about computer
networks. Oh, and just to clarify: the last tirne,
someone asked if there was some rnagic number
of computers that have to be connected in
order to officially have a network-I looked it up
and there's not really a single answer. But for
this class, let's just say a network is two or more
connected computers. Is that clear? Good.
Anyway, today we'll be delving farther into this

696 lranscripts

subject-we'll talk about two different networking


methods, the local area network and wide area
network.
Most of you are probably familiar with local
area networks, also called LANs. You'll find
these in homes and small offices, and you
probably use one in the campus library or even
in your dorms. By definition, they basically only
cover a small area. Local area networks are set
up in different schemes, with the most basic
involving two or more computers connected to
a single server through an Ethernet or wireless
connection. I'm seeing some confused faces.
Remember, a server is the device that administers
different services or resources to Users on a
network. For example, a server makes it possible
for computers to transfer files to each other
over a network, or it makes files accessible to
anyone on the network. Are things a bit clearer,
now? Let's move on.
Now, in the scheme I just described, all users
are connected to the server. There are many
other configurations which we'll encounter
throughout the semester, but overall, LANs can
be characterized by their relatively small scale,
especially when compared to a wide area network.
But the small scale of LANs also offers some
benefits. For example, they are often faster,
especially in terms of file transfer, and they are
built with a leased line, which we'll discuss a little
later in this lecture.
What about wide area networks? In computer
science, we usually refer to them as WANs.
These make resources available on a much
wider scale. What I mean by that is that a WAN
can be used by people in other parts of the
country, or even the world. The best example of
a WAN that I can think of is the Internet, which
is easily the largest network in the world. WANs
are built for many different purposes. They can
be used by private organizations to link up
smaller LANs to each other, or even by Internet
service providers, who use them to provide
Internet access to users. The most common
way to build a WAN is by using a leased line. A
leased line is a connection between two sources
that is used to provide not only networking
services, but telephone and Internet services
as well.
Since a WAN built with a leased line basically

just connects two LANs, it is probably the most,


secure. But the thing is that leased !in.es are
pretty expensive. To avoid this, some WANs
are built using different methods, such as circuit
switching or cell relay.
M:

02 Literature

W:

W: I'd like to talk today about the short story that

M:

W:

M:

W:
M:

W:

M:

W:
M:
W:

you read for homework over the weekend, The


Sandman. Before we get started .. '. and, uh,
rather than giving me a plot summary, could
anyone tell me what it's about? Not a plot
summary, please, but just, well, in general,
what's it about? What's its theme?
It's a tale about obsession, really. And well, I
guess how obsession can lead to one's
demise, you know, to a person's downfall,
defeat, death, whatever.
Well said, Tom. And who might the Sandman
be.x .. traditionally?
The
-,... uh, the sandman is a character in Western
.
myth who helps children fall asleep. But, Hoffman,
the;author, changed the character into a terrible
on.ei-a character who steals children's eyes!
That's it! Pretty gruesome, huh? Now, with this
in mind, let's look at Hoffman's use of motifs in
the story.
Pardon me, but uh, I'm not sure that I fully
understand what motif means, and looking
around the class, I don't think I'm the only one.
I think you may be right about that. I can certainly
clarify. In literature, a motif is an, um, an element
of symbolic significance that is repeated
throughout the story. Are you getting what I
mean? It can be a structural element, such as
when a certain kind of sentence or narrative
voice is used again and again. Or it can also be
a repeated literary device, like a metaphor.
Does that make sense?
Oh yeah, now I remember. Thanks. Um, so,
tiack to your question, one motif that I think
would be pretty hard to miss is that of the eyes.
I'm glad you brour;ht that up. Can you give me
an example of where the eye motif came up?
It was everywhere. ! mean, the Sandman would
supposedly steal Hie eyes of children.
Good. And here's something for you all to think
about too-the names oi the men that the main

M:

W:

character, um, Nathanial believed were the,


um, I guess incarnations of the Sandman also
refer to the eyes. One guy's name was
Coppelius, which comes from Coppola-this is
related to the Italian word coppo, which means
eye socket.
That makes a lot of sense. I wondered if that
name had another meaning.
Uh-huh. Now, earlier I said that a motif has a
symbolic significance. What do you think the
significa~ce of the eyes is in this story?
I think it has to do with the connection to the
soul. You know, since the eyes are supposed to
be the window to the soul. That explains why
another character-a robot, actually-seemed
so vacant to everyone, despite her beauty ...
she had no eyes. So no eyes, no soul.
That's a great interpretation. That would also
explain why Nathanial was so afraid of the
Sandman, or those he associated with the
Sandman. Perhaps he was afraid the Sandman
would steal his soul.

,,

03 Service Encounter
W: Hi, is this the study abroad officf)?
M: Sure, how can I help you?
W: I just got back from a semester abroad. I took
four courses while I was there, but now I don't
know what I'm supposed to do to get university
credit for them.
M: Have you had the department heads sign your
credit request forms yet?
W: Huh? I'm sorry, but I have no idea what that is.
M: Here are the forms. Basically, you fill one out for
each course you want to get credit for. Then
you take them to the head of the department
you want credit from. They usually ask you for
the syllabus and the final exam or paper you
completed for the course.
W: OK, so I take the final and this form and they
have to approve it or something?
M: Yes, exactly. After they look through your material,
they'll decide whether or not it's on par with a
course taught here at the university. Then they
will either reject or approve your request for
credit. It's really a lot easier than it sounds.
W: It sounds like a lot of legwork. Is there a deadline
for when these forms need to be turned in?

Transcripts 697 _ .

M: Yes, in order for the credits to appear on your


transcript, we need the credit request forms
submitted by March 20. If you don't get it done
by then, we won't have time to process the
credits before next semester. If you're graduating
this year, you need to be extra careful to turn
the forms in by the deadline, or you may not
have all the credits you need to graduate.
W: Uh-oh. I guess I should try to get this done
then. I don't want to have any problems at
graduation time. Come May, I want to get my
degree and go.
M: I hear you. Well, then just make sure to turn in
your forms on .time. And if you're worried about
making the time to do this, I advise you to email
the heads before you meet with them to find out
exactly what they want you to bring. That way
you won't have any surprises and you can just
get your forms signed on the spot
W: Thanks a bunch. I really appreciate it

01 History
W: I'm sure you all have heard of \he term "yellow
journalism." In case you haven't, it refers to a
type of journalism that focuses on sensational
topics that will get people's attention. It shouldn't
be such a problem except that it's also associated
with um, uneth1cal practices and sometimes
even outright lying. There are plenty of modern
examples of yellow journalism, but you might
be surprised to learn that it first appeared more
than a century ago.
In spite of all the criticism it garnered then, it
actually played an important role in many
historical events of the era. It brought attention to
the hardships of city life for immigrants, and later,
it drew public attention to the Spanish-American
War. You're probably wondering how this was
possible. Why don't we talk a little bit about how
that happened?
Let me begin by saying that yellow journalism was
born, in an official sense, as a marketing strategy
by a young newspaperman named Joseph
Pulitzer. Very simply, Pulitzer wanted to find a
way to get more people io read his paper. So

i
I

rL
11

698

Tra~scripts

he tried to make his paper. interesting by including


games and contests. Additionally, he realized
that he could increase his readership significantly
by tapping into the immigrant market, which, as
you can imagine, was considerable in the early
19 century. So that's what he did, and his
papers sold like crazy. At this point, I want to
note that, although Pulitzer wanted to find a
way to make more money, he also felt that it
was his duty to improve society. So, um,
although he ran sensational headlines like
"Lines of Little Hearses" or uh, well "How
Babies are Baked" to catch people's attention,
many of the articles were relevant to current
events and society, especially to the immigrant
community of New York City.
Pulitzer's enterprise became quite lucrative and
it inspired another young man named William
Hearst to follow in his footsteps. Hearst used
many of the same techniques as Pulitzer: his
paper featured attention-grabbing headlines,
and its stories usually involved crime or celebrity
scandals. Furthermore, at only one cent. his
newspaper cost less than Pulitzer's. As readership
grew, Hearst had to search high and low to
keep finding scandalous material to draw readers.
Many times, Hearst was criticized for embellishing
stories. And, when there was no story, Hearst
was happy to stir one up. This was sort of the
case with the Spanish-American War. See, prior
to the outbreak of the war, Hearst supposedly
told one of his reporters that he would furnish a
war for him to cover. And when the war finally
did break out in 1895, it was on the front page
nearly every day.
Political cartoons depicted the Spanish as brutal
and cruel. The articles were not always accurate
. .. that is, until Hearst himself traveled to Cuba
to cover the events. Interestingly, although
historians all pretty much agree that Hearst
played a very minor role, if any, in starting the
war, many say that his coverage later in the war
helped promote awareness about the realities that
the Cubans faced under Spanish rule.

02 Chemistry
Today I want to talk about crystallization, which
is the process by which a liquid solution turns
into a crystal. This is the process that is
responsible for creating the beautiful gemstones
that we see in jewelry stores and stalact'1tes that
hang from caves. Just.so yo, know, this)~cture
is going to be pretty term-heavy, So ifvou all
need some more explanation about anything,
please let me know, and I'll try to explaih until
everything is, uh crystal clear Ha ha:
OK, so there are actually a couple of steps in
the process that create these pretty crystals.
The first is called nucleation. During nucleation,
small particles in solution start to accumulate in
little clusters. The solution, of course, is a mixture
of two or more substances. The clusters can
stabilize in the solution and will eventually form
the nucleus of the crystal. If they don't stabilize,
however, the particles in the solution just dissolve
once again. And I want you to understand that the
stability of the clusters is completely dependant
oi1'factors such as saturation levels and even
temperature. When a solution is supersaturated,
il'ineans that it contains more particles than
can be dissolved. Do you all get that? Typically,
supersaturated solutions are more likely to support
a stable cluster that can form crystals. Meanwhile,
temperature can affect saturation levels by
triggering evaporation or vaporization of the
solution. So both of these conditions are very
important in the .growth of the crystal.
Anyway, once the nucleus is formed, the atoms
of the solution begin to form around it. The patterns
in which the atoms form is called the crystal
structure. Just so we're all on the same page, I
want to emphasize that the crystal structure I
refer to here does not reflect the shape of the
crystal once it has fully developed. Instead, the
crystal structure really just describes the pattern
in which the atoms are initially formed. And
that's the nucleation stage.
The next stage is when crystal growth occurs,
and, as you can guess, it mostly entails the
maturation of the crystal. I want to point out that
nucleation and crystallization often occur at the
same time. That's why, if you've ev0r seen a
crystal, you'll note that there are usLaliy different
shapes and sizes of crystals.

So, does the crystal growth just go on and on,


or does it stop eventually? l'rn sure you can
guess that there is an end to crystal growth. It
sort of has to do with the same conditions that
either promote or stop growth: temperature and
saturation levels. See, when the solution is no
longer supersaturated, either because it has been
exhausted by the growth of the crystal or because
other changes have occurred, the crystal pretty
much stops growing. What's interesting, though,
is that you can supersaturate the solution to
start the crystal growing again.

03 Service Encounter
W: Hi, is this where I sign up for the creative writing
courses?
M: Actually, registration doesn't open until next
week.
W: I'm sorry, I must have gotten confused. It says
in the course announcement booklet to sign up
in the English Department.
M: That's right. But we only open class registration
tor one day. Since there are a limited number of
spots in each class, we only have sign-ups on
the Wednesday before the next semester
begins.
W: How many spots are there available in each
class?
M: It depends on which one you want to sign up
tor. For example, the Introduction to Travel Writing
course only allows ten students. The. more
general courses allow slightly more students,
but to tell you the truth, we need to keep it pretty
low. Otherwise, there are too many people for it
to be good for anyone, you know?
W: Yeah, that makes sense. I don't think people
have the opportunity to learn if the class is too
full. But anyway, how does signing-up work,
exactly? Is there a line around the building or
something? I'll bet it gets pretty crowded.
M: Oh yeah, definitely. Some students start showing
up really early, like at five a.m. I don't really
understand. why they do that, since it's not like
signing up means you automatically get into
the class.
W: It doesn't? Then why even l1ave sign up sessions?
M: I know it seems pretty convoluted. The sign up

--1

Transcripts 699

-.-

'

W:
M:

W:

M:

is for the professors to get an idea of how big


the student pool will be in the fall. After you've
signed up for the course, you'll be on the
professor's list. But if you don't show up to the
first class, you'll get kicked out of the course.
So, all I really have to do is sign-up and then
show up to the first day of class next semester?
Nope, that would be too easy. The fact is, there
will definitely be more students than spots. So
the professor will have everyone write and submit
a sample.
Really? So after putting us through all this, we
still have to prove ourselves again later? There
must be a lot of people trying to get into these
classes.
You're right. But don't worry about it. Just do
your best. And even if you don't get in this
semester, you can try next semester. Just be
sure to sign up on Wednesday, or you won't
even have a shot.

04 Psychology .

':I
: ;;
'
.

W: This week, we've been reading about dreams.


For the discussion section, I asked you to
consider the following question: how can
humans interpret their dreams to promote
understanding about the self? What did you all
come up with?
M: I found that Freud actually had a set of processes
that he said are responsible for making dreams
so incoherent sometimes. But by understanding
the processes, it would supposedly be easier
for people to understand what their dreams
mean.
W: You know, I don't find that surprising at all. Freud
was one of the first to study sleep and dreams,
and to this day he remains a big influence in
the field. Anyway, can you describe the
processes that Freud came up with?
M: If I remember correctly, the first is, um,
condensation. The idea is that sometimes in
dreams, ideas or people or events will be
condensed into a single dream image.
W: Good. Do you all understand that? An example
of condensation is when you dream of a character
that looks like your dog, but has the voice of a
childhood friend. How about the second
process?

--__----- - - -- - ----- -

M: The second process is called displacement.


It's when the dreamer redirects emotions or
actions to a seemingly unrelated image. Like if
a person was angry at their friend, he might
dream about breaking a possession dear to the
friend, instead of actually hurting the friend.
W: .That's a great example, If you all are clear on
that, I'd 1.ike to move on to the third process:
symbolization. This process is tricky because it
pretty much r,equires that people interpret their
dreams on a symbolic level and not a literal
level.


M: I'm not sure I understand. Does it mean that
nothing in dreams can be taken at face value?
W: More or less. It's pretty, um, well, it's hard to
apply this idea, considering that so many people
will have so many different subjective views on
symbolism in their dreams. So I'm not so sure
that it's realistic to expect people to have any
real use for symbolism. But, well, maybe you all
disagree. Would someone care to share the
symbolic meaning of a dream they once had?
M:, One time, I had.a dream that I was waiting at a
stop light. And it seemed Iike forever that I was
waiting there, so finally, I just got fed up and
started driving without waiting for the light to
turn green. I just went. It didn't even matter if
there were police around.
W: Interesting. So the literal meaning of that dream
is that you have no respect for laws. But what
do you think the symbolic meaning is?
M: I realized I was just feeling restless at the time.
It was right before the semester was about to
start and I just wanted to go already.
W: Good. So that should help you understand
Freud's idea of symbolization now. OK, so on to
the last process: secondary elaboration.
M: Isn't that when the dreamer adds logic and
details to a dream they've had?
W: Yup, and Freud suggested that if you want to
capture a dream without elaboration, try to
record the dream as soon as you wake up,
before you embellish it with logical details.

-= - - - ---- - -----

-------~---~

,----------------------:-=--=-----

..
I
.

,':.

&~

1:

m~:.. .
\:

~.
~ _:.,_

' <'-\ff ....

..~~.

05 Physics
M: Let's move on to simple machines. You probably
remember that in physics, a simple machine is
any mechanism or device that requires force
from only one source in order to work. In class,
we only had a chance to get sort of a broad
overview of these machines. But today, I want
to go into more detail about the types of simple
machines there are and the kind of work they
perform. Why don't we start with the pulley?
In a nutshell, a pulley is kind of like a wheel that
has a groove in it. Then there's a rope that runs
through the grove in the wheel. The pulley is
used to change the direction of the force that is
applied to the rope. So, in the most basic pulley
systems, you pull the rope to help lift something
that is attached to the other end of the rope. I'm
sure you've all seen pulleys at work before.
Now, there are three types of pulley systems that
I'd like to discuss. The first type is called a
fixed-base pulley. For this type, the axle of the
wheel is.'..secured in one place. And when you

direction of the force currently on the rope.


Does thafmake sense to you? So when you pull
on the rope it makes it easier to lift whatever is
attached to the other end of the rope. Let me
give you an example. OK, um, right..~
!hi.. Imagine a fixed base pulley as the type
used on flag poles to help people raise flags.
W: It doesn't seem like that kind of pulley would .
really be useful for anything but raising something
light ... like a flag. There's a kind of pulley that
can lift heavier things, right?
M: That's right It's called a movable base pulley. You
probably guessed that the main characteristic
for this type of pulley is that the pulley-the
wheel's axis moves. This is pretty helpful,
especially if you want to multiply the force
being exerted on the rope. When you multiply
the forces on the rope, you're increasing the
mechanical advantage of this simple machine.
This means that when you pull on the rope, you
will have double the force on the object
attached on the other end.
OK, let's move on io the last type of pulley: the
compound pulley. A compound pulley mixes
both fixed base ancl movable base pulleys.

Compound pulleys are a good way to give


yourself an even better mechanical advantage
over the pulleys involved in your simple
machine. In theory, the more pulleys you add,
the more mechanical advantage you create,
but in reality, every additional pulley adds more
friction. And if you have too much friction as a
result of too many pulleys, yoLi eliminate any
advantage that you might have gained.

06 Office Hours

W: Hi, Martin, good to see you. What can I help


M:
W:

W:
M:

W:

M:

you with this afternoon?


Hi, Professor Richtor, I wanted to talk to you
about the paper that's due this Friday.
What's the problem? Are you having trouble
writing it? If you want, sometimes it helps students
to talk through a topic. In fact, I've done it with
three other students for this paper already. I'd
love to hear what kinds of ideas you've come
up with.
at st et mg ... uh, I haven't really had any
time to put any ideas together. See, on Friday
I'll also be taking a midterm for my chemistry
class. As a prerequisite for all upper level biology
courses, I really need to do well on it I've even
been meeting with a tutor.
But when do you plan on working on your
paper for my class?
.
Dr. Richtor, I would never ask for an extension
under any other circumstances, but I think this
situation is dffferent. I just didn't foresee that
studying for my chemistry test would take up so
much time.
Actually, I don't know about how valid an
excuse that is. This paper is on the syllabus,
plus I assigned it over a week ago. If you knew
there was going to be some overlap with your
chemistry test, you could have approached me
about it earlier, not two days before the paper
is due.
I'm sorry. I really just don't know what to do. I
think I could do a good job on the paper if I only
had the time to really explore my idea.s. But a~
long as I'm cramming for my e:<arn. I 1ust don t
know how I'll do that.

Transcripts 701
__,,
- ---~--~""''

W: Martin, I don't really know that there's anything


I can do for you. I think that even with two days
you could write a very good paper.
M: What happens if it's late? I mean, what if I just
can't have it to you by Friday?
W: My policy for late papers is that I take off half of
a letter grade for each day that it is late. So if
it's due on Monday, but you don't turn it in until
Wednesday, the best you can get on your
paper is a "B." And that's assuming that the
paper is worth an "A" to beg in with.
M: That's pretty strict. I guess I'll just have to,try mY
best to get it in by Friday.

I
i
i

702

Transcripts

t?
~

-----------

- - - - - - - - - - --------

-------------~-----------------------

-------

-- - - - - - - - - ----

---

------------------------ - - - - - - - - - - - - - - - -

their education. When students do this, they learn to


be more responsible adults.
Chapte~

Q 1 Practice 1
~

Step 2 - Sample Response


One person who has helped me become who I am
today is my father. First of all, he taught me dedication.
I joined the soccer team when I was younger, and
wanted to quit because I was not talented. But he
encouraged me to keep practicing, and I got better.
Second, he taught me to be generous: Because of
my father, I spend time each week volunteering to
help kids learn to read.

Opinion 2
I think that students should not have to pay to go to
university for two reasons. First, education should be
available to all. Since universities charge for education,
some people that are very smart but do not have a lot
of money cannot go to university. If universities did not
charge, these people could attend university. Second,
students study less if they fret about money. Students
often worry about their money. When students do this,
they cannot concentrate on school.

Q2 Practice 2
Q1 Practice 2
~

Step 2 - Sample Response


My uncle Ron, a firefighter, has a job that I admire. I
admire this job for two reasons. First, everywhere he
goes, peopre;respect him. They say that he is courageous
for the work that he does. They admire him. Second,
he is paid to do what he loves most. He says he
always wanted to aid people. Being a firefighter is how
he helps people. Plus, he gets a very good salary.

Step 2 - Sample Responses


Opinion 1
I prefer spending my free time outdoors rather than
indoors. One reason is because I like getting fresh air.
This gives me energy and makes me feel healthy and
content. I feel better when I am outside. A second
reason is because I like doing things in nature. I like to
hike in the mountains or go fishing. I enjoy doing these
things with my family.
Opinion2

Q1 Practice 3
~

Step 2 - Sample Response


I feel comfortable talking with my older sister about rny
problems. First, she is a good listener. She is patient
and pays attention when I tell her my problems. She
often has useful advice or good solutions. Second, she
knows what to say because she has lots of experience.
She is older than I am and once had similar problems,
so she knows what to do.

Q2 Practice 1
Step 2 - Sample Responses
Opinion 1
I think that students should have to pay to go to
university for two reasons. First. universities need a lot
of money. Since universities charge for education,
they have the funds to provide a good education to
students. If universities did not charge, they would not
have enough money. Second, it teaches students
responsibility. Students must save money to pay for
.'

I prefer spending my free time indoors rather than


outdoors. One reason is because I enjoy playing
games. I really like playing video games, chess; and
sometimes board games. I think that these games are .
not appropriate tor the outdoors. A second reason is
because I have allergies. When I go outside, I sneeze
a lot, and sometimes it is hard to breathe. I like to stay
indoors where I can feel relief from my allergies.

r;.2 Practice 3
~ Step 2 - Sample Responses
Opinion 1
I prefer classes with a lot of discussion for two main
reasons. The first reason is that they are more engaging.
A topic is more stimulating when the ideas are discussed
in detail. This is more interesting than just listening to
the teacher. Second, discussion classes let you hear
other opinions. I like classes where you do not only
hear the teacher's opinion. With cliscussions, other
people ask questions and say fascinating things.
Everyone learns more this way.
Transcripts 703

---.~+

Opinion 2
I prefer classes without a lot of discussion for two
reasons. The first reason is that they are boring. When
you talk about a topic, many students like to talk a lot
but do not say constructive things. This wastes a lot
of time and makes me tired. Second, discussion
classes are not organized. I like classes that are clear
and have steps. With discussions, the class can lack
a central topic. So it can be confusing at times.

Q3 Practice 1
~

Step 2 - Conversation

M: Did you hear that they are finally raising baseball


ticket prices?
W: .No. You mean they're raising ticket prices for
students?
M: Yeah, the announcement says that there'll be no
reduoed tickets this term for students.
W: Oh. You go to games all the time. How do you
feel about that?
M: I guess it's not that bad, really. I mean, for one,
they're right in saying that they're not making
enough money. I go to the games all the time,
and most of the people there are students, which
means that most people aren't paying full ticket
price. How are they supposed to make any money
like that? It just makes no sense for the university.
They have to make money on these games, or
they won't be able to have a basketball program.
W: That's right. And no one wants that.
M: And I also agree with the idea that the school
needs more money to improve the facilities. I
mean, have you seen the arena lately?
W: No. Why?
M: Well, it looks horrible, like something from when my
grandfather went to school here. It's old, outdated,
and falling apart. I hear that when it rains, they
have leaks in the locker rooms.
W: I can understand why they would want to fix it up.
~

Step 4 - Sample Response

The man thinks it is good that the university will no


longer provide reduced-price basketball tickets to
students. This is because, first, they are not making a
profit with reduced tickets. He says that it makes
sense to charge students regular price when they are
the majority of the fans at the games. Second, it will
-

-704

help to raise more money to improve the facilities. He


also thinks that the team needs a new arena since the
one they have is old, ugly, and has leaks in the locker
rooms.

Q3 Practice 2
~

Step 2 - Conversation

M: I can't believe they are canceling the extra section


of Music Theory. I was so excited when I heard
they were finally opening up a morning section. I
was planning to take that class.
W: Yeah, I've heard it's great.
M: Right, but I don't get it. I mean, why can't they
just hire another professor to teach it? Tht0 music
department has been growing, and it's about
time the university hires another professor.
Otherwise, they're going to have problems like
this repeatedly. If students can't take the classes
they want, they might not want to attend this
university.
W: I know. They have to start hiring new professors
sometime.
M: The other thing I don't understand is why they
can't hold the class in another building. It's a
music theory class. You can study theory in any
classroom so long as it has a sound system, and
most large classrooms do. It's not like you need
to be in the music building since you don't need
any instruments for the class. I know for a fact
that there are a few empty classrooms in the
Humanities building in the mornings.
W: Right. Do you think there's anything we can do
about it?
M: Well, it sounds like the university has its mind made
up, but maybe we could consult the head of the
music department and express our concerns.
~

Step 4 - Sample Response

The man does not agree with the university's plans to


cancel the morning section of the music theory class.
He thinks it is a bad idea, first, because he thinks the
university needs to hire more professors. He believes
that if the university does not hire more professors,
students may not want to attend the university.
Second, he does not understand why the class cannot
be held in another building. He says that a music
theory class does not need to be held in the crowded
music building because it does not require instruments.
He knows that there are classrooms available in another
building in the mornings.

"O'anscripts

- - - - - - - - - - - - - - -- - - - - - - - - - - - - - - -

Q3 Practice 3
Step 2 - Conversation

M: Did you hear they're offering a new literature


class at night?
W: Yeah, I think it's an excellent idea, and none too
soon.
M: Right.
W: I mean every semester it seems there are more
and more working students coming to. this universily.
I had one night class and it was packed because
it was one of the few night classes the working
students could take. So this is a really good idea
to offer more night classes. Especially if we can
expect more part-time students on campus.
M: I know what you mean.
W: But it's also a good idea to offer more literature
classes in general. You know, for those of us
whose major isn't literature, we don't have a lot
of choices if we want to take a literature class. A
lot of them are only for literature majors. But this
one sounds really good because it's a survey
'~S'.: class. So, it's not too advanced for people like me
W who want an introduction first.
NJ: That's true. Most of the classes are pretty
advanced.
Step 4 - Sample Response

The woman thinks it is a good idea for the university


to expand the English department's offerings. First,
she says there are not enough classes for working
students who .can only study at night. She mentions that
her night class was crowded with working students.
Second, she likes the idea of a new introductory
class. She says she struggled to find a class that was
not too advanced.

Q4 Practice 1
Step 2 - Lecture

M: We've been talking about product marketing, and


now I'd like to discuss the television as an example.
The way consumers classified the television
changed when its price changed. When the
television first appeared, it was seen as a luxury
item. This commodity was something t11at only
the richest people could afford and display in
their homes. It was not, as it is today, a feature of
every household. The first televisions were
extraordinarily expensive. Even if the average

family wanted one, they couldn't afford one.


However, over time, the price came down.
Televisions became less expensive, so cheap
even that any college student can afford one
today. Not only can anyone afford a television,
but who doesn't have a television? Most people
are convinced they need a television these days. So
it isn't just a luxury ... it has become a necessity.
All because of the lower price.
Second, consumers classified the television in
different ways after their design became more
appealing. When the first televisions were released,
they were, well, not the most attractive things.
They were plain looking and bulky-quite ugly, in
fact. They took up a lot of space in the living
room. But, that was acceptable because people
bought them, not for decoration, but rather to
access the new world of television programming.
They wanted to know what was going on in the
television world. They didn't buy them to decorate
their homes. Today, however, with newer, sleeker
models-we now have televisions that are designed
to hang on our walls, much like paintings. Today,
we use televisions to decorate our homes, so
they have taken on a new function.
Step 4 - Sample Response

The lecture talks about how consumers will classify


products based on their price and appearance, and
the speaker uses the television as an example. The
professor shows that the first TVs were very expensive
luxuries, but when they were cheaper, everyone had
to have one. This supports the idea that consumers
classify products based on their prices. Second, the
lecturer says that the first TVs were plain and heavy,
but people liked to watch them. Now TVs are more
appealing and people use them to decorate their
homes, as if they were wall paintings. This supports
the author's claim that consumers classify products
based on appearance.

Q4 Practice 2
~ Step 2 Lec.ture

W: Let me give you an illustration to show how one


form of sensory memory, echoic memory, works.
Let's take the example of hearing a telephone
number for the first time.
First, echoic memory explains why, if someone
recites a phone number to you, you're likely to
Transcripts

705

forget it after a few seconds. When you hear a


new phone number, you can remember it just long
enough to dial it, which takes a second or two.
That's because you are using echoic memory,
which only lasts for a few seconds. If you wait too
long before dialing that number, and you didn't
write it down, then you will forget the number.
The echo is gone from your memory.
Second, echoic memory also explains why you
are almost certain to dial a new phone number with
perfect accuracy. Without thinking about it, you
can dial the number you just heard, again, as
long as you do it right away. And you will dial it
exactly as you heard it because you are hearing
its echo in your mind. Try to dial a phone number
you heard, say, a few minutes ago, and chances
are you are going to dial it incorrectly. In the latter
case, the echo in your mind is going to be gone,
so you're trying to access the number in your
short-term memory, which isn't as accurate.
~

Step 4 - Sample Response

The lecture uses the example of a telephone number


to describe two main features of echoic memory. First,
the professor says that you are only able to remember
a phone number for a short period. That is because,
when you hear it, you use your echoic memory to
remember it, which does not last long. This supports
the author's claim that echoic memory is very brief.
Also, the professor says that you can remember the
number very well for the first few seconds. This is,
again, because of echoic memory. This also backs up
the claim from the reading that echoic memory is very
accurate.

learn where to put your fingers and how to move


them to make music. At first you struggle and
need to watch your hands. But as you practice,
you develop the skill to play the guitar without
looking at your fingers as much. If you continue
pr<1cticing, you'll be able to play your first song
without looking at your fingers at all because
you've developed unconscious competence.
But there is more: when you've developed
unconscious competence with the guitar, you
can begin to do other things simultaneously. Let's
say you have learned a song to near perfection
but you also want to sing while playing. So once
you can play that song without thinking about it,
you can now put your conscious mind into learning
how to sing the lyrics that go along With that song.
Thus, the guitar playing becomes automatic, while
you concentrate on the singing.
~ Step 4 - Sample Response

The professor talks about unconscious competence,


which happens when a skill becomes natl]ral. He .
uses the example of learning to play guitar. At first you
have to look at your fingers. But when you become
skilled, you do not have to look anymore, you can just
play. This demonstrates the concept introduced by
the reading passage that says you can do the skill
without thinking. Next, the professor says that you can do
other things, like sing, when you have unconscious
competence. This supports the claim from the passage
that unconscious competence allows you to concentrate
on other things.

Q4 Practice 3
~

Step 2 - Lecture

M: Now, I'd like to talk about the development of

unconscious competence, the end result of


learning a new skill so that it becomes natural to
you. It becomes so normal that you don't have to
think about using it. Let me explain using the
example of learning a new instrument-the guitar.
When you can play a musical instrument without
thinking about it, you have achieved unconscious
competence. And that's an accomplishment. If
any of you have tried to learn to play a guitar,
you'll know that it can be difficult. You have to

706 Transcripts

QS Practice 1
~ Step 1 - Conversation

M: Hey, Susan, how's it going?


W: Not so good, George.
M: Why, what's wrong?
W: Well, you know our school literary rnagazine, the
Gryphon? Well, we're running out of money. The
business manager, she wasn't really doing such
a good job rnanaging our budget. So we will
probably run out of money by the end of the term
unless we do something. We might have to shut
down.

e
d
,,'

Jt

e
g

e
d
u

s
n

e
I,

J
1.

t..

M: That's terrible, I love reading that magazine.


W: So do a lot of other people.
Have you thought about increasing the number
.~{ M: of
advertisements? Right now, you get good
.tH:
money from advertisers, right? But I bet it might
~:
~?Y
be possible to find more advertisers and make
~4
enough money that way.
W: The question is-are there any other businesses
interested in advertising?
" M: It is worth it to check,,0tt1eiWise, you might want
to think about letting some of your writers go. I
~.~
meen, you probably pay a lot of money to those
f?.;
writers, and while it might mean that you have to
~i
have a smaller magazine, it would certainly help
you cut costs.
('':
W: True, we have about 15 people now, but we
:~\'
might be able to do with only five.
~F M: That's a big jump .
~:,'.
,, W: Still, I'd hate to have to let anyone go.
F M: Well, reducing your staff might help. But so could
p,.
finding more advertisers.
i(
r.:
W: Yeah, I just can't decide which I think is better.
M: ll.eah, that's a tough decision.
i

i~

'
':.<;'<

~:'.

.'-1'

3
3

t
I
II

..

Step 3 - Sample Responses

Opi~fon 1
I think increasing advertisements is the better solution.
First, it is the easiest way to raise money quickly. If lots
of people read the magazine, then businesses certainly
would want to advertise there. It would be easy tor the
magazine to find good advertisers. Second, there is no
guarantee that they would save money if they employ
fewer writers. If they let their writers go but the quality
of the magazine suffers, then no one will want to read
the magazine anymore. Then they will not be able to
find any advertisers.
Opinion 2
I think cutting some staff is the better solution. First, it
is the easiest way to cut costs of the magazine
quickly. If the woman thinks they only need five writers,
then they definitely do not need fifteen. It would be
easy to let at least a few of those writers go. Second,
there is no guarantee they can raise enough money
through advertising. If they look for advertisers but do
not find any, they might have to close the magazine
before they can explore other options. Then the entire
staff will be without jobs rather than just some of
them.

QS Practice 2
~ Step 1 - Conversation
M: Hi, Linda. How's it going?
W: Pretty good. How about you, Alan?
M: Good, except graduation has been on my mind
lately. I only have one year to make my preparations
to become a teacher.
W: I know what you mean.
M: The thing is I that want to get experience as a
teacher. That way I can be prepared when I go
out and look for my first job. But I'm not sure how
to get that experience since I'm so busy.
W: Having experience is really important. Hey, why
not ask Professor Schwartz to sponsor you for an
independent teaching project. She mentioned to
me once that some past students did this. They
put together mini-lessons that they taught in
after-school programs.
M: That seems like a good idea. I am just worried
that I'm too busy to devote a bunch of time to
something like that.
W: Fair enough. You certainly don't want to fall behind
in your schoolwork.
M: Exactly.
W: Well, hey, how about this? Why not just wait until
you get to grad school? You're planning to go
anyway. And they always let you be a teaching
assistant in grad school, right? That way, you'll
get plenty of practice being a teacher before you
actually go out and get a teaching job.
M: That's a good idea, but they don't always let you
become a teaching assistant. .Sometimes they
just don't have enough classes for all"the grad
students to take. And I worry about waiting until
grad school. If I don't get a teaching position . . .
then I really won't have any other chance to get
experience.
W: That's very true. Well, I think you should consider
both options.
M: I definitely will.
~

Step 3 - Sample Responses

Opinion 1
The man's problem is that he wants to get teaching
experience before he becomes a teacher. I think that
the first choice is better: he should work with his professor
to do an indepencent project. First, it will help him get
experience right now. Teaching is something I think you
need a lot of practice to get good at. Second, it will give

Transcripts

707 _ _,

~~

fu,

~r
r_ift,

--------------~--------~---------him a chance to change his mind. If he gets experience


now and finds that he does not like to teach, then he
could do something else.
Opinion 2
The man's problem is that he wants to get teaching
experience before he becomes a teacher. I think the
second 'choice is better: he should wait until he gets
to grad school. First, he is really busy right now.
Teachi~g is something that you need to devote a lot
of time to, and he does not have enough time.
Second, he will get more valuable experience in grad
school. He can teach real classes, which will give him
more genuine teaching experience.

Q5 Practice 3
~ Step 1 - Conversation
M: Hey, Mary, do you have a minute?
W: Sure, Michael, what's up?
M: I have a problem and I need to talk it over. I was
hoping I could get your advice.
W: Oh, OK.
M: So, I'm supposed to be in a team debate this
weekend, with my debate club. But the thing is, I
got a last-minute invitation to attend a national
debate conference, and I'd really love to go.
W: Hmm. That's tough. So, I guess your first option
is to just go to your team debate. I mean, you
don't want to let your team down, right? And I
know you've talked about how you're friends with
all of your team members. They might be mad if
you don't go to the debate.
M: Right ... but I'm just so eager to go to this
conference. I mean, it is a really good opportunity
to learn more about debating.
W: Well . . . in that case, perhaps you should go to
the conference. Look at it this way: if you're going
to learn a lot, then you could go to the conference,
come back, and coach your team members about
all of the new things you learned. Then your team
might be even better.
M: Good point, but I really do feel like I would be letting
my team down. They are really counting on me.
W: Right.
l\11: See, I just feel like either option is going to disappoint
someone.
W: Well, you'll just have to decide whether you want
to be the one disappointed or whether you want
to risk disappointing your team.
l\11: I know. I'll think about it. Thanks for the advice.

- - - 705 Transcripts

Step 3 - Sample Responses

Opinion 1
The man has to choose between participating in a
team debate or attending a national debate conference.
I think he should go to the conference. First of all, it is
an excellent opportunity. If he goes, he could learn a
lot and become a better debater for his team. Second
he says his teammates are friends, so they should
understand that this is important to him. Next time, he
will be there and the team will be even better, so they
should understand that it would help the team.

Opinion 2
The man has to choose between participating in a team
debate or attending a national debate conference. I
think he should stay and attend the team debate. First
of all, his team is counting on him. If they expect him
to be there and they are his friends, then it is important
for him to honor his commitments. Second, he can
always go to another conference. Next time, he can
plan ahead to attend the conference, so that he does
not create a conflict in his schedule like this time.

QG Practice 1
~ Step 1 - Lecture

W: Today, I'd like to talk about oral traditions in ancient


societies. In the ancient world, most people did
not rely on writing for communication, especially
not for storytelling, a fact that has two important
consequences for ancient stories.
First, ancient stories that were transmitted by
word of mouth were not finished, or completed,
like written stories are today. In other words, there
wasn't one definitive version of a story. Today, an
author writes a story and it is complete; it doesn't
change. Ancient stories, on the other hand, were
always told differently. If you were telling a story
you heard, you could change things about the
story: add some details to entertain your audience.
Say, for instance, you wanted to tell your story to
a group of soldiers. You might embellish it by
adding battle scenes. Later, another person
might add a bit of romance to the story to make it
more entertaining for the audience. So there was a
lot of flexibility in how the story could be told.
Different people could tell it different ways.
Second, the stories often didn't have an
identifiable author. We don't know who first told

them. So, when people finally wrote these stories


down on paper, they were not the actual authors,
even though they often put their names on these
stories. In some cases, these stories remain
anonymous. So you have ancient stories that are
said to be the work of some author, but in truth, the
stories were told and retold by many people, so
that we don't really know who first started telling
it. Really, an entire culture might be the most valid
author.
~

Step 3 - Sample Response

The lecturer says that in ancient societies, people told


stories to each other. They did not write them down,
and this had two consequences for ancient stories.
First, it meant that stories often changed each time
someone told them. For instance, the professor talks
about how you could add battle scenes or romance
to make a story more interesting for your audience.
Second, stories never had an author like they do
today. So many people who we think created the
ancientsstories might have just been the ones who
wrote tl\em down.

Q6 Practice 2
~

Step 1 - Lecture

M: Good afternoon, today we're going to talk about

"

animation; specifically, I'd like to discuss the


differences between traditional animationanimation done by hand-and computer animation..
Now, to some the two types of 11nimation might
look very similar, but they are actually very different.
First, they differ in how long it takes to produce
them. You see, traditional animation means that
each image must be individually drawn, painted,
or photographed-one by one. Then these images
are put together to make a film or video. It's a
very time-consuming process. Even a very short
movie could take weeks. Computer animation,
on the other hand, can be created far more
quickly because the computer does most of the
work. With a good program, you could make a
short movie in a few hours.
Second, they differ in reputation. See, many
people prefer traditional animation to computer
animation. This is because they feel it is a more
artistic form of animation-it is like a moving
painting by a talented artist. Moreover, people

respect the hard work needed to make traditional


animation. Conversely, fans think of computer
animation as less artistic and easier to produce.
It is not as creative, they argue, since anyone with
knowledge of a computer program can make
animation-you don't necessarily have to have
artistic talent.
~ Step 3 - Sample Response

There are two main ways that traditional and computer


animation differ. First, they take different amounts of
time to produce. Traditional animation is done by hand,
so it takes a long time to do. Computer animation,
though, does not take nearly as long. Second, they
differ in how people feel about them. Many people
prefer traditional animation because they think it is
more artistic and the product of hard work. Some
people like computer animation less since anyone with
a computer program can do it.

Q6 Practice 3
~

Step 1 - Lecture

W: I'd like to discuss the importance of a strong


brand name. Establishing a strong brand name is
an important part of marketing a product.
First, a strong brand name can make a product
more appealing. Soda is a perfect example.
There are several popular brands of soda, and
everyone has his or her favorite. But, the interesting
thing is that consumers are drawn more t the
brand than to the taste of the soda. In fact, when
people are blindfolded and taste several brands
of soda-when they don't know what brand they
are drinking . . . well, guess what? They are as
likely to enjoy the taste of their favorite brand as
an unknown brand of soda. It is not the taste alone
that appeals to them, it is also the brand. It seems
many people are drawn more to the name
attached to the soda than the soda itse~.
Second, a strong brand name can help people
remember a company or product. Emergency
situations are a good example. When you have
an emergency, you are probably in a hurry to
make a decision. A good brand name can help
you make that decision. Say you are sick with a
cold and you need to pick up some medicine. So
you head to the store and go to the medicine
aisle ... but there are so many brands! Now, in

Transcripts 709 --/.

this case, you are more likely to buy medicine


from a well-known brand just because you already
know the company.

Step 3 - Sample Response


The lecture talks about the importance of a strong
brand name. First, a strong brand name is important
because it can make a product more appealing. For
instance, people can taste different sodas and think
they are both OK, but they are going to buy the one
with the more popular brand name. Second, brands
can help people remember your company. For example,
in a health emergency, you will choose the brand of
medicine you already know. The strong brand name
gives you confidence in that product.

Q 1 Practice 1

Step 3 Sample Response

W: I have always wanted to learn how to sail a boat.


This is because once, when I was little, my
grandfather took me sailing. Since that day, I
have wanted to understand how he steered the
vessel through the wind just by manipulating
ropes. I also want to learn how to sail because I
love the ocean. I love the ocean so much that
someday, I would like to travel the world in my
own sailboat. However, I cannot do that unless I
know how to sail.

Q 1 Practice 2
~

.i

I
i

j,~-

Step 3 Sample Response

M: Someday, I would love to take a class in


photography. For one, I would like to take this
class because I find cameras fascinating. I recently
got one for my birthday, but I do not know how
to use all the features. I would also like to take
photography because I love taking pictures of
nature and animals. They never come out very
well, though. So I would like to learn techniques
for taking quality photographs.

710

W: 1 really like to go for lengthy bike rides in the


mountains when I have free time. First, I like doing
this because I love being out in nature. Sometimes
I even see wildlife, like birds and squirrels, and
occasionally a fox. I also like mountain biking
because it helps me unwind. I have a very
demanding and hectic school schedule. Mountain
biking lets me forget all the things I have to do for
a little while.

Q1 Practice 4
Step 2 Sample Response
M: One of the best days of my life would have to be
the day my sister was born. It was the best day
because it was the day that I first met my best
friend-my sister. Until then, I was an only child
and I was kind of lonely. After she was born, I
always had a companion nearby. It was also a
wonderful day because we got to spend all day
together as a family. Everyone was so delighted
because Tara was born and we all got along
really well that day.

Part 2

Q1 Practice 3
Step 2 Sample Response

Q2 Practice 1
Step 3 Sample Response
M: I think that teachers should give unannounced
tests in class. For one thing, giving surprise tests
makes sure that students keep up with the material
I know I stay on top of my studying if I suspect
there might be a surprise test at any time.
Secondly, unannounced tests are a great way to
increase your grade. Some teachers only give
one or two tests, and if you do poorly on one, your
grade suffers severely. But with short, unannounced
tests, you can get more points.

>:i2 Practice 2
~

Step 3 Sample Response

W: I think it is better to go immediately to university


rather than work for a year or two. First of all, it is
much more difficult to go bacl< to school once
you have entered the work world. Many people
are determined to go back to school. However,

'

Transcripts

__-: __'C

oo__co_

they lose the motivation after working for a while.


Also, I think it is important to finish your education
as soon as you can. That way, you can get your
degree and find the career that you really want.

Q2 Practice 3

>Step 2 - Sample Response


M: I think that people should be allowed to use cell
phones while they drive. First of all, I do not think
cell phones are distracting. When I drive and talk
on the cell phone, I de not feel any less attentive
to what is happening on the road. I have never
been in a collision due to using my phone. Second
of all, cell phones can be useful in emergency
situations. For example, if you get lost while you
. are driving, it would be helpful to use your cell
phone to call someone to guide you.

Q2 Practice 4

>Step;2
,. - Sample Response
W: I 'do not think that music players should be
b$nned in school. I do not agree because music
pl~yers are only used outside of class. Students
should not be utilizing music players in the
classroom, but most students refrain from doing
so. There's no reason why they should not be
able to listen to them during lunch or between
classes. I also do not agree because I do not
think music players are hazardous items. If students
carry them in their backpacks and use them in
the halls, they are not causing any mischief.

Q3 Practice 1
~

Step 1 - Conversation

W: I can't believe they're taking away the TV from the


student union building!
M: I know. It Seems pretty unreasonable. But I
guess the university has its reasons. Like the
announcement said, it's keeping students from
studying, which is never a good thing.
W: Yeah, I mean sure. some students might want to
study there. But I know lots of students who
come there to relax or hang out when they need

a break from studying. It's supposed to be a social


area, not a study hall. Students who need a quiet
area to study could just go to the library. There's
no reason that they need to use the common area.
M: True. But like the announcement said, most
students have TVs in their rooms. Do you think
students actually come to the student union to
watch television?
W: Sure! I watch TV there all the time. Especially in
between classes when I don't have time to go
back to my dorm room. I mean, it is a twenty-minute
walk back to my dorm room, and I'm not about
to do that when I can just go to the student union
to relax for a while. I know a lot of students do the
same thing.
M: Hmm. You're probably right.

>Step 3 - Sample Response


M: The woman is disappointed that the television will
no longer be available to students. She thinks
that area should not be used for studying. She
says students that want to study could go to the
library She also disagrees with the announcement's
statement that the TV is not necessary because
many students have TVs in their dorm room. She
says she would have to walk twenty minutes to
her dorm room if she wanted to watch TV between
classes.

Q3 Practice 2

>Step 1 ~ Conversation
M: Great, now where am I going to stay when summer
classes start?
W: I don't know. Are they really closing the dorms for
the summer?
M: Yeah, that's what it says here. But that's not fair. I
mean, they say they weren't able to get enough
supervisors. But that seems like such a bad
excuse. I mean, they should have asked more
people. It's not like I saw fliers or announcements
or anything asking people to be supervisors.
W: Maybe they did, but no one wanted to be a
supervisor.
M: I'm sure they didn't. I would've done it, but no
one asked me!
W: Maybe you're right. But that wouldn't really matter
since such an insignificant number of students
applied for summer housing anyway.
M: I feel like that's the university's fault. They should've

Transcripts

711 - .

II--

advertised that the dorms are available in the


summer. They should've put up fliers or something.
Most students don't even know they're open for
the summer. If they knew they were open, then
more people would apply. I didn't even find out
myself until a couple of weeks ago. One of my
professors mentioned that she thought they were
open, but she wasn't sure. So I looked into it. But
I would never have known if she didn't say
something.
Step 3 - Sample Response
W: The man does not agree with the decision to
close the dorms in the summer. He thinks they
should have asked more people to supervise the
dorms if they could not get enough volunteers.
He says they could have found enough supervisors
if they had asked. He also thinks that too few
students applied for summer housing because
the university did not advertise the fact that the
dorms were open. Most students do not know
that the dorms are open during the summer. He
thinks the university should have posted fliers.

Q4 Practice 1
~

Step 1 - lecture

M: A common way to encourage good behavior in

children is to use a reward system. So, for example,


a parent or a caregiver might request a certain
behavior -like picking up toys or playing nicely
with other kids. Then if the child does what is
asked of him, his behavior is rewarded with
something as simple as a preferred snack.
This method works well, for one, because it
produces a positive outcome for both the parents
and the child. A parent may offer something like,
"You can have an extra fruit snack if you share
your toys with your brother." Of course, fruit snacks
are often something that kids enjoy, while the
parents succeed in making the child behave in a
positive way. By rewarding a child with something
he wants, parents are encouraging him to repeat
tliose actions.
Reward systems are also effective because they
teach kids to correlate good behavior and good
results. Returning to the example of receiving an
extra snack, if the child receives a fruit snack after
doing something good, he will realize that doing
that good thing caused him to receive a reward:

He will soon realize that good behavior brings


about .good things. He is thus more likely to
repeat that behavior.
Step 3 ' Sample Response
M: The professor uses the example of giving an extra
snack to explore how reward systems teach children
positive behavior. He explains that reward systems
are good for parents and children. The professor
says that giving an extra snack is effective
because kids will get something they want, while
parents are able to teach children good behavior.
The professor also says that giving a snack is
effective because it teaches kids that good
behavior will get them good things. So they are
more likely to repeat that behavior later on.

04 Practice 2
Step 1 - lecture

W: So as an educator, one of the most effective


ways of teaching someone how to do something
is by demonstration. Let's say you're trying to
teach people how to use a computer. You could
try to explain to them how to turn it on and open
programs and such, but all that information might
be too abstract if they don't have a computer in
front of them. Your students will learn how to use
a computer much easier if you demonstrate how
to do it.
First of all, demonstrating how to use a computer
is going to give your students all the information
they need to know right there. By watching you
turn the computer on, open programs, check
e-mail, change fonts, or whatever else you do,
your students learn those steps by remembering
how you did them. And as the demonstrator, you
don't have to worry about leaving out a step in an
extensive description. Since you're doing all the
steps, you literally can't forget a step.
Also, demonstrating how to use a computer
shows your students how to use the information
you're trying to teach them in a real life situation.
You're not just telling them how to- mal<e
documents and create files. You're showing them
how they would do it on their own computers. So
it isn't just a list of facts: it is a series of real !ife
skills that they can immediately do themselves.

712 Transcripts
-'-'

Step 3 Sample Response


W: The professor discusses the best way to teach
someone how to use a computer in order to show
why teaching by example is a good method.
According to the professor, you should show
someone how to do something rather than tell
them how to do it. The professor says that teaching
someone to use a computer this way is effective
because it gives them all the information they
need for using a computer. The professor says it
is also effective because it shows your students
how they would use the information in a real life
situation on their own computers.

Step 3 Sample Response


M: The woman is organizing a Halloween dance,
and the club that was funding it backed out at the
last minute. The man says she will either have to
cancel the dance or find a new sponsor. I think it
is best for her to look for a new sponsor. That
way, she will not disappoint all the people-like
her friend-who have already made costumes.
Also, a week and a half is plenty of time to find a
new sponsor and I think her efforts would be
worth it.

QS Practice 2
~

Step 1 Conversation

W: Having car trouble?


M: Yeah, it won't start. And I've got to get to a job

QS Practice 1
~

Step 1 Conversation

M: H\l)(: Martha, how's the planning going for the


Ha:.bween Ball?
W: Terrible! I just got an e-mail from the Martial Arts
Clup. They were going to fund the dance and
even do a live performance. But now they're
backing out at the last minute.
M: Really? That's awful. Do you know why?
W: Yeah, they don't have enough money after all,
which is too bad because I've put all this time
and effort into organizing the dance. So now I'm
not sure. what to do.
M: Well, I guess you could cancel it. Elut I know that
would disappoint a lot of people. My friends and
I already made costumes. We're going to be
pirates.
W: That's awesome! I wish I didn't have to cancel it,
but I don't see any other option. I mean, I'm not
going to spend my own money on a school dance.
M: True. But there's gotta be another way. What if you
searched around to see if some other organization
will sponsor it?
W: I guess I could call around. But it is going to be
really hard to find someone on such short notice
... and the longer I wait to cancel it, the more
people are going to be in your situationcostumes made and everything.
M: Right. Well, I'm sure you'll figure something out.
Let me know if I can help.
W: Thanks. I appreciate that.

interview all the way across town.

W: Oh man, that's too bad. What're you going to do?


M: I don't know.
W: I guess you could take the bus. I'm sure it'll take

M:

W:

M:

W:
M:

you wherever you need to go. The only problem


with that would be the fact that it stops so
frequently. It might take a while for you to get
there.
Well, my interview's in forty minutes. I might be
delayed if I took the bus. At least I know that the
bus can get me there, though.
Well, I'd let you borrow my car if I had one. But
unfortunately, I don't. But, hey, your roommate
has a oar, right? Why don't you ask him if he
would be willing to lend you his oar?
That's a good idea. It would definitely be faster
than the bus. But what if he needs his car this
afternoon? I would feel really bad if I took his car
and he ended up needing it. I mean, what if he
needed it for an emergency?
True. Either way, you had better decide quickly so
you're not late to the interview.
Tell me about it! I don't know what I'm going to
do ...

~ Step 3 Sample Response

W: The man has to get to an interview across town,


but his car will not start. The woman suggests
either taking the bus or asking his roommate if he
could borrow his car. I thinl< he should ask to borrow
his roommate's car. I Think this is the better option
because his roommate may not need it that

Transcripts

713 - -

. l

afternoon and might be happy to lend it. Also, the


man would not have to worry about being late to
his interview. The speakers agree that the bus is
slow, so it is a bad idea to take it to such an
important interview.

animals camouflage themselves is by disguising


themselves. The walking stick, for example, tricks
predators into thinking it is just another twig. This
causes predators to ignore it.

Q6 Practice 2
Q6 Practice 1

~ Step 1 - Lecture

W: As members of society, we often take on many

M: Over the course of evolution, animal species

'I
i
,/
;I

develop special adaptations that help them live in


places that might not be very conducive to their
survival. One common adaptation is camouflage,
which allows animals to blend in to their environment
so that they can hide from both predators and
prey. There are two important kinds of camouflage
that animals use.
One way animals blend into their environment is by
changing color when the environment changes.
This is known as cryptic coloration-changing
color to match the environment. The seasons, for
example, drastically alter the appearance of an
animal's habitat. In spring and summer, the
ground may be full of greens and browns, while
in fall and winter, everything may be covered in
snow. Now, a brown or gray fox may be able to
hide well in a summer setting, but corne winter,
he'll stand out against the white snow like a wine
stain on a wedding dress. Which is why the Arctic
fox turns white. Many mammals and birds change
the color of their fur or feathers in accordance with
the seasons.
Another way that animals camouflage themselves
is through disguise. In essence, animals look like
other objects that predators won't pay attention to.
An interesting example of this kind of camouflage
is the walking stick. The walking stick is an insect
whose body looks almost identical to an ordinary
twig. A predator may see the walking stick, but
chances are it will think it's only a stick and not
food. And so it will ignore it. Other insect species
have evolved to look like tree leaves, or stranger
yet, the eyes of a much larger animal.
~

Step 3 - Sample Response

M: The professor says that animals camouflage


i

:'i

Step 1 - Lecture

themselves in order to hide from prey and predators.


One way they do this is by changing colors. Tl1e
Arctic fox changes Its colors to match the look of the
environment. The Arctic fox is brown in the summer
and turns white in the winter Another way that

different roles; we may be students, daughters,


tutors, competitors, and sisters all at the same
time. Now what often happens, because we
assume so many different roles, is that these
roles will come into conflict with one another. In
sociology, we have a term for this-we call it role
conflict, and it can happen in one of two ways.
One way role conflict occurs is by trying to be two
incompatible things at the same time. For example,
it's common for a father to also be the coach of
a baseball team, but what happens when he is the
coach of his son's baseball team? In this situation,
he must act both the role of father and the role of
coach. So say his son makes a bad play in a
game. As a father, he might want to comfort his
son when he does something wrong-put his
arm around him and tell him its OK-but as a
coach, he would want to tell the boy exactly what
he did wrong. He must choose one role ... and
there's no easy way out of the conflict.
Role conflict also occurs when there is a sudden
change in role. A doctor, for example, must at .
times be a patient, but because he sees himself
as a doctor, he may have trouble assuming the
role of a patient when he is suddenly ill or injured
and cannot care for himself. He may try to make
diagnoses or order tests when he should be
listening to the advice of his own doctors and
submitting to.their care.

~i

,,/,

f
ll'
}.'
r,,

t;

;,

'
r-

'

"
~

"'
,,

:1

'

P Step 3 - Sample Response:


W: The professor says that role conflicts happen
because people take on many different roles at
one time. One way that they occur is when
someone tries to take on two roles that are not
compatible. For example, a man might try to be
a father and a coach to l1is son at the same time.
Another way that role conflict occurs is when
someone suddenly changes his role. For example,
a doctor might suddenly have to be a patient. In
this situation, he will struggle to adjust to being a
patient instead of a doctor.

,,,____ 714 Transcripts


--~"

- ._ -> - __._.o_._,,

,. - _____- __-: - .--: ,,. -

------------------------==---:::-~c::_:.-;.;,_:_~,.:::.:..~.,_::_;..._~_;;;_.;:;;;,,~----------- -.:.:.......:::_:_- ------:..:..:...----=-~

I' Part 3
~~:
If"

Ghapter 7'

'i:,

,,:

Chapters.

i11mW"il~

Stress related to parts of words


~Step 1

1. a. method
2. a. economy
3. a. academy
4. a. luxury
5. a.drama
6. a. recommend
7. a. capable
6. a. prefer
9. a. photograph
10. a. negotiate
~Step

. :

b.
b.
b.
b.
b.
b.
b.
b.
b.
b.

methodology
economic
academic
luxurious
dramatic
recommendation
capability
preference
photography
negotiation

1. Do you have a campus parking permit for your


bike? ..,:
2. I hope ~y professor can advise me on which
course to take.
3. She has.;.to present her project to the class
tomorrow'.
4. My friends and I are going to the protest downtown
this afternoon.
5. Did you hear that Jane and her band will record
an album this summer?
6. Unfortunately, my parents will not permit me to
go skiing this weekend.
.
7. The police have arrested a suspect in the campus
computer lab robbery.
8. In biology, we are studying how plants convert
sunlight into energy.

Stress on phrasal verbs


~Step 1
1. The researchers found it out very recently.
2. The robber held up the convenience store.
. 3. Let's go check out tl1e new restaurant in the
student union.
4. Can you help me? I am searching for a journal on
anthropology.
5. Do not point at lier. 1hat is rude.
6. People often say that I take after my father.

Sentence stress related to content words


~Step 1
1. People classify products based on price and
appearance.
2. The university should advertise that the dorms
are open in the summer.
3. Echoic memory lasts for only a few seconds.
4. The magazine might have to shut down.
5. Giving rewards will teach children to behave well.
6. I was very proud of my brother when he was
accepted into medical school.
7. I feel comfortable talking to my sister about my
problems.
8. Students should work for a year before going to
university.
.~Step2

One person that I really admire is my father. For one,


he has worked very hard in his life. He was very poor
as a child arid did not get a good education, but he
worked very hard to educate himself and get a good
job. Second, he is a very nice person. He is always
giving money and assistance to other people. Just
last weekend, he helped a coworker remodel his
house.

Reduction of unstressed words


>Step 1
1. The people who moved out to other cities were
safe, but those who were in the city were in danger.
2. He is the one in my family who understands my
dream.
3. The students cannot access this section but the
teachers can.
4. The government asked him to stop campaigning
against the policy.
5. They wanted to create something new and
innovative.
6. For homework, you all should have read a bit
about winp power.
7. I know I look young, but I am a student at this
university.
8. Mathematics is an important aspect of university
studies.

Transcripts

715

Step 2
The man supports the decision by the university to
offer literature classes at night. For one, he thinks it is
good because it allows people who work to take
more classes. He says that there are many working
students who take classes at night. He also thinks it
is good because it will allow the university to offer
more types of literature classes. The speaker says he
wanted to take a basic literature class, and now he
can.

Intonation
Step 1
1. I had lost an important assignment due to computer .
problems.
2. That gave me the opportunity to learn about a
new culture.
3. I never went on a trip with those friends again.
4. Universities need money for computer labs.
5. Many people think that teaching by example is
best.
6. They look more interesting and have more
character.
7. A reward system is a good way to teach children
to behave well.

Step 2
1. That will tell us the date at which the house was

2.
3.
4.
5.
6.

built.
Morn and Dad want me to attend.
They can cause all kinds of trouble.
I am sure I will land a good job after graduation.
In the past, television was a luxury.
In addition, they are beneficial to mankind.

~Step

1. Although we had not 1inished, we decided to go


home.
2. When she stepped off the boat, she immediately
ran to her car.
3. It was raining so hard all day that they did not
leave the house.
4. If the alarm rings, put down your books and
slowly leave the building.

o - - - 716 Transcripts

5. The final test will be two hours long and will count
for twenty-fivepercent of your final grade.

Step 2
1. In my opinion, students should not have to pay to
go to university.
2. To begin, echoic memory is very short, lasting
only three to four seconds.
3. Charities, both local and international, rely on the
generosity of individuals to help the less fortunate.
4. In summary, then, I think that teachers should
give unannounced tests in class.
5. While a strong brand name can make a product
more appealing, it can also . help customers
remember a company or product.
6. Many people, however, feel that animation done
by hand is more artistic than computer animation.
7. It took 1ive years, but he finally achieved his goal.
8. In the case of many oral traditions, it is impossible
to identify the stories' authors.

Practice Test 1
Question 3
M: I'm so glad they are finally updating the exercise

W:
M:

W:
M:

W:
M:

W:
M:

facilities at Scaife Hall.

What do you mean?


Have you ever worked out there?
No, why?
Well, if you did, you'd realize that they don't have
a great selection of exercise equipment. The
problem is most of it is old. Some of it's even
broken. I mean, it's a really popular place, so the
equipment gets used a lot. It's just natural that it
wears down over time. Just seems that the
school waited a long time to finally update it. Still,
it's the right move.
Oh, I see.
Yeah, and since it is so popular, it's really crowded
there. If you go during the afternoon or evening,
you have to wait. Sometimes you wait fifteen
minutes to use a machine. With the proposed
expansion, it should be better. You might be able
to just walk in and work out on any machine that
you want.
That certainly sounds like an improvement.
Let's hope so!

Question 4
M: Today I'd like to talk about impression management.
And I think you'll get a sense of how it works by
looking at the behavior of a manager I know who
runs a small computer repair business.
First, customers expect him to be professional, so
he dresses and acts professionally. For example, in
order for the business to be successful, the manager
thinks that he needs to appear professional and
competent to his customers-that he needs to
convince them that he is able to answer their
questions and solve their problems. When he
talks to his customers, he makes sure to speak
really clearly and use big, advanced words that
he wouldn't normally use. In essence, he is Jiving
up to what he thinks customers want him to be.
Second, the manager also wants to create a good
impression with his young employees. He wants
them to like him as a friend. So even though he is
dressed up like a professional, he jokes around
wtth them when customers are not around. He
1i makes sure that he knows a little bit about the
latest video games and music that they like, so
. he can talk about these things with his employees.
i, He wants them to see him as not just a professional
and a boss, but also as a friend.

Question 5

<.

M: Margie, how's it going?


W: . Not so great, Ted ..
M: What's wrorig?
W: Well, remember how I told you I was planning to
take that History of Science class?
M: Yeah.
W: Well, I'm afraid I waited too long to register. The
class is full.
M: Oh no.
W: I'm not sure what to do.
M: Hmm. I guess you could just wait a few days and
see if someone drops it. You know how it is.
Every semester people sign up. Then they change
their mind after they attend the first class or two.
As long as you keep checking the class list, you
could sign up when someone drops. But you'd
have to check the class list everyday and sign up
right away.
W: That's not a bad idea. I think that might work, but
then again, what if no one wants to drop it?
Everyone says it is supposed to be a great class.

M: The other thing you could think about is actually


contacting the professor directly. It's taught by Dr.
Harmon, right? I heard he is really cool. So maybe
you can convince him to let you into the class.
Maybe when.he sees how excited you are to take
his class, he'll let you in.
W: That might work, too. I'm sure he'd let me in if he
could, but there just might not be any seats left.
M: That's possible, but it's still worth a try. Think about
both of my suggestions.

Question 6
W: Today, I want to talk about keystone species.
Now, a keystone species is any plant or animal
that has a huge effect on an environment. Jn
essence, the environment would completely change
if that species were not there. The wolf is one
such species. Its presence has two important
effects on an environment.
First, the wolf controls the populations of animals
in an environment. Recently, wolves were brought
back into Yellowstone National Park. As a result,
there are now changes in the animal population.
For eighty years, there were no wolves in the
park. During that time, the populations of two
species-elk and deer-the populations of these
species increased. The park was overpopulated
by deer and elk; there were too many. But, when
the wolves returned, within a short time, there
were profound changes in the populations of
these two species. What happened? Well, the
population of both elk and deer declined, and
rather quickly to healthier levels. This is because
wolves are predators; they eat elk and deer.
But here is the really interesting part: the wolves'
return has also helped the environment. Specifically,
there are now more cottonwood trees in the area.
Now, you might be asking yourself how a wolf
could possibly help the populations of cottonwood
trees. Well, it all comes back to the elk and deer.
See, the deer and elk had been eating and
destroying all of the young cottonwood trees
before they had a chance to grow. But when the
wolf came back and elk and deer populations fell,
cottonwood trees were able to grow freely again.

Transcripts 717 - .

Practice Test 2

experience the action and confusion.


In addition, Velazquez chose this particular point
of view to convey a particular meaning. He does
not wish to show you the inner character of these
royal people. He didn't want it to be an emotional
piece. If he had, he might have painted close-ups
of faces to better emphasize facial expression.
But in this case, he uses a broader point of view to
emphasize the group rather than any one person.

Question 3
M: Did you get the notice that they're going to be
installing more lights on campus?
W: Yeah, I wish they weren't. I think it's a pretty bad
idea.
M: You do? Why's that? Seems to me like the
maintenance department had some pretty good
reasons for doing it.
W: Well, I think it's a waste of money and electricity.
For one thing, I don't think it'll stop people from
vandalizing and stealing things. If anything, I think
it'll encourage more vandalism and theft.
M: Really? But wouldn't people be afraid they'd get
caught.
W: Maybe. But there's still fewer security guys on
campus at night, and with more lights, it's easier
for thieves to see whatthere is to steal. Personally,
I prefer locking my bike up where it's dark. If they
put lights there, I'm afraid lots of people will start
. noticing it's there.
M: That makes sense.
W: And also, I think there's plenty of light already
to do outdoor activities at night. Plus, some of
those things . . . like jogging . . . can be more
enjoyable at night. It's nice to just go out into the
dark and enjoy the night without it being so bright
everywhere.

Question 4
M: As artists, when we refer to point of view, we
mean the location of the viewer - the location of
the viewer looking in on the scene in a painting.
So, take for example, the famous group portrait
of Spanish royalty painted by the 17th-century
artist Diego Velazquez. In this scene, the artist
himself is painting a portrait of the king and
queen. For this scene, Velazquez uses point of
view to do exactly two things.
First of all, he directs the eyes of the viewer so
that you are looking at the scene from inside the
house, as if you are part of it. He does not choose
to paint a close-up of the king and queen, but
rather reveals the entire scene and everyone
involved . . . including the dog. By painting the
scene using this broad, yet intimate, point of
view, Velazquez positions his viewers so that they

718 Transcripts

Question 5
W: Hey, Dave, have you decided where you're going
M:

W:
M:

W:

M:
W:

M:

to live next semester? Those apartments we looked


at downtown were pretty nice.
Yeah, they were. But I don't know, I'd really like to
stay on campus. It's too bad university policy
says we have to move out of the dorms after
freshman year.
Yeah, well they probably just don't have enough
housing.
True, but it's too bad because I don't have a car
and I don't know how I'd get to class and baseball
practice every day from downtown.
Oh man. That is a problem. Are you sure the policy
says we have to move off campus? I mean,
maybe you can go talk to someone in campus
housing and see if they'll make an exception for
someone in your situation.
That's not a bad idea. It might be worth a try. But
I think they're pretty strict with that policy.
Oh; hey. Why don't you apply to be a resident
assistant? Not only will you get to stay on campus,
but you'll get free housing!
That's possible, but being an RA is a lot of work.
I'm not sure I'd have the time to do the job with
classes and practice and everything else.

Question 6
W: When you watch television or flip through a
magazine, you're bombarded with advertisem.ents
... mostly for common products-cars, shampoos,
shoes, games, food. So how is it that advertisers
can make these everyday products look so
appealing? Well, there are a couple of ways they
do this.
First of all, many advertising campaigns . .
magazine ads, television commercials, and the
like ... many of these campaigns rely heavily on

exaggeration. Exaggeration simply means to


overstate or magnify something out of proportion,
and if it's used correctly, it can be a very effective
advertising technique. So for example, there's a
commercial for fast food in which two people
order a meal at the restaurant's drive thru. And as
soon as they're handed their order, the car tilts on
its side. The point is that these meals really are
big. But the exaggeration is so extreme that it is
obvious and amusing to the viewer.
Advertisers also use humor to make their products
look desirable. For example, one company has an
ad for photo editing software, and on one page is
a family photo that's addressed to grandma. It's a
lovely photo, except that the son is wearing a
leather jacket with chains and a crazy haircut.
The second page shows the same photo, but the
son has been "edited," supposedly using this
software program, to look clean-cut and nicely
dressed. The ad says something to the effect of
how "Grandma is saved from a holiday shock." So,
wrat the advertisers try to do with this is make
p~9ple associate good feelings-like laughingwilh their products. Thus, the product is more
appealing .

. ~-

Transcripts 719 --_-1

'---- -

-~~--

"

Chapter t
01 Urban Planning

more employees to work from home. Those are


more effective long-term solutions that would get
more cars off the streets and reduce pollution.

M: Many people talk about congestion pricing as the

best solution to the problems of overcrowding and


pollution in our cities. Well, while the policy has its
advantages, by itself it is not the best solution and
does not live up to its promises. Let me explain
why.
First of all, congestion pricing will not
necessarily encourage more people to use
public transportation. This is because, in many
cities, the public transportation systems are not
extensive enough to meet increased demand. In
other words, buses and subways don't go to
every neighborhood, and that means that even
if you don't want to drive your car, you have no
choice because there's no public transportation
in your neighborhood. That's a problem.
A second point I would like to make is that
congestion pricing will not automatically result in
faster commutes like proponents say it will. It
doesn't mean that you can get to and from work
in less time. This may be true for some people,
but more than likely, once your city implements
congestion pricing, you will actually spend more
time going to and from work. Why? Well, because,
as more people use public transportation, trains
and buses get more crowded. That means you
wait longer before you can even get on the bus
or train.
Third, congestion pricing will not benefit everyone.
It's not that simple. Sure, some people will choose
to drive to work because it is more convenient.
And congestion pricing will certainly tax those
individuals for driving. But what about the
individuals who have no other choice? Many
workers cannot afford to live in the city or near
public transportation lines because housing in
those areas is too expensive. Those individuals
must drive to work, and many of them are from
poor neighborhoods and have low-paying jobs.
Congestion pricing will penalize them-force
them to pay a much larger part of their budget for
transportation and pollution. That is simply not a
fair policy, and it certainly doesn't benefit everyone.
A better policy would be to expand public
transportation, to build affordable housing closer
to jobs, and to encourage businesses to allow

02 History

W: Now, there are many scholars who doubt that the


treasure of the Copper Scroll is real; however,
there is enough evidence. to suggest that those
scholars are wrong, and the treasure listed in the
scroll may still be out there in the Israeli desert
waiting to be found.
First of all, the difficult language used in the
scroll suggests that it describes a real treasure.
Traditional treasure stories from ancient Israel
used colorful poetic and religious language. The
Copper Scroll, on the other hand, is written with
a highly technical vocabulary, In ]act, the vocabulary
is very difficult for modern scholars to read
because it is unlike anything they have ever seen
before. Using technical language seems a very
strange way to tell a story that is. supposed to
be passed on from generation to generation. So
it probably isn't a story: it's real.
Second, the treasure the scroll describes is not
necessarily as large as some people think and that
means it is more likely to be real. The treasure is
described using an ancient measurement called
a "talent." Now, in ancient times, money was
different in different areas, so the measurement of
one talent could mean a large amount in one
region and a small amount in another region.
Confusing, yes, but that was what it was like in
the ancient world. What it tells us is that the
treasure may be much more modest than we first
thought. In that case, it very well could have been
the treasure of the Temple. And historical records
suggest that the ancient Israelites hid the Temple's
treasure during a time of political upheaval.
Finally, although there are many challenges in
finding ancient treasure, it would not be impossible
to find the treasure listed on the copper scroll. One
reason is that the scroll talks about a second
scroll, a silver scroll, which gives more detailed directions to each of the different treasure locations.
If someone could find that silver scroll, they would
have better directions to the treasure. So, it would
not be impossible to find the treasure just because
the directions contained in the Copper Scroll are
not entirely clear to modern scholars.

- - 720 Transcripts

--- -------- -

------~...:=~=:.=.o..=-=-=

03 Environmental Science

;,

e
r,

M: You hear a lot of talk about biofuels in the news


today, but the promises of biofuels do not match
the facts. Let's take a look at ethanol, one of the
most widely used biofuels. It can be made from
corn, sugar cane, or a few other crops, Ethanol
,'~~can be used to fuel cars and other vehicles, but
not without problems. Ethanol really just isn't a
substance that we can use to replace fossil fuels.
gl
First of all, it is not cleaner than the gasoline that
t,:.:
it replaces. Why? Well, at first it looks cleaner.
~-%~.
Burning one gallon of ethanol in your car produces
less pollution than burning one gallon of gasoline.
So it's cleaner, right? Wrong! It's actually dirtier
because ethanol is a less efficient fuel. In other
words, you get less energy out of a gallon of ethanol
than you get out of a gallon of gasoline. So you
have to burn more to travel the same distance.
And that means more pollution.
Now, ethanol requires a lot of land that we simply
don't have. In recent years, there has been a
ma$'sive push to produce more ethanol from corn
gr6\\in in the US. At first, this sounds good, but
the problem is people need that corn to eat. So
what happens when more corn is used to
proi:luce ethanol, well, um, the price of food goes
up. You can't have both cheap food and cheap
f>
'.;,
fuel. There is not enough land for both.
:~'
One last point: ethanol won't be able to meet the
demand for fuel. Let me give you a stark example. In
order to replace the current demand for petroleum
in the US; you would need to use seventy-five
percent of all the agricultural land in the world to
grow ethanol-producing crops. Seventy-five percent!
That doesn't leave much land for food production;
in fact, it hardly leaves any at all. The demands of
drivers in large nations like the US, China, and
India are simply just too large to be met by ethanol.
Therefore, the wortd needs to keep looking for
more ways to replace gasoline. Perhaps hydrogen
power is the answer. But ethanol certainly is not.
"~

e
rt

e
I.

ll

e
h
y
j

y
)

,_,

t
I
j

correct. Corntainers don't solve these environmental


concerns. Let me explain.
First of all, PLA plastics need special facilities to
make them biodegradable. You can't simply take
a corntainer and bury it in your backyard, and
have it just turn back into dirt. It doesn't work that
way. To break down PLA plastics you need special
facilities. In these facilities, the corntainers are
subjected to highly controlled conditions, including
the presence of microbes and sustained
temperatures of 140 degrees. In such conditions,
PLA breaks down into water and carbon dioxide
in about ninety days. Without those conditions, it
doesn't compost well.
Second, containers are ending up in landfills
when people mistakenly try to recycle them like
regular plastics. People take PLA to regular
recycling facilities where the PLA must be separated
out from the petroleum-based plastics. The two
can't be recycled together; it creates a mess. So
recycling facilities must spend time and money
to separate the PLA from the plastic, and they
just throw it out. And where does it go? It goes
into the landfills, anyway. So without proper
facilities our landfills are filling up with corntainers.
Finally, there is nothing socially responsible about
using corn to make plastic containers. Why? Well,
because people need corn to eat. There is only
so much corn grown in the world. And, as you
are aware, the world's population is growing rapidly
and people need to eat. People need to eat corn.
It is very irresponsible to take corn that could feed
people and turn it into plastic. We already have a
source for plastic, and it is a source that we can't
eat and that source is petroleum.
Rather than waste time investing in corntainers,
we'd be better off using fewer disposable things
in the first place. A generation ago, people
reused cloth bags when they went shopping.
No waste. They reused glass bottles for food
and drink storage. Again, no waste.

04 Environmental Science
W: Today, I'd like to take a closer look at corntainers,
containers made frorn polylactic acid, or PLA, which
is a derivative of corn. Now, you may have heard
that PLA plastics can solve many environmental
concerns. The problem is, those claims are not

:1 Climatology
1\11: Global warming is an impo1tant topic today. For
years, scientists have investigated, keeping an

Transcripts

721 - - .

'

.,

t
;;;,
w

"
\~

;,

'

:v

eye out for telltale signs that global warming is


happening. The things is, as compelling as some
of the arguments may sound, they're false.
Global warming is not taking place on Earth.
Let's begin by talking about carbon dioxide levels.
Many people mistakenly believe that a high level
of carbon dioxide in the atmosphere automatically
means that global wanning is, in fact, going on.
However, let's consider something for a second:
carbon dioxide is a naturally occurring gas, which
means it exists even without humans adding
more. Moreover, it's not as if the levels have
increased as dramatically as some sources would
have us think. The fact is that over the past thirty
years, carbon dioxide levels have increased by
only two percent. That's including the extra pollution
that humans throw into the mix.
Next, the melting of ice caps doesn't necessarily
prove that temperatures are getting hotter around
the world. Scientists agree that ice caps are
influenced by a variety of factors. A combination
of these factors could easily account for the
receding ice caps. For example, recent research
suggests that at least some of the ice cap melting
is caused by volcanoes. When volcanoes located
under the sheets of ice erupt, they destroy the
ice it comes into contact with. And that's just
one of the many different factors that affect ice
caps.
Finally, what about these facilities that have
records of temperature increases? Turns out that
even with the most advanced technology, their
figures are still inaccurate. A number of scientific
agencies have spoken up about the inaccuracies,
pointing out that it's misleading to suggest that
they have actual proof of global warming. The
problem is that the Hadley Center and NASA take
samples from some places and not others. As a
result, the averages published by those facilities
might be off the mark by at least two degrees.

First of all, adults need to carefully monitor children's


unstructured playtime to make sure that it is
positive. Just because children are allowed to
structure their own playing does not mean they
will do something positive. Many kids, left on their
own, will get into trouble and in the worst cases,
even get hurt. Others will waste their playtime sitting
for hours in front of a TV eating junk food. They
need adults to guide them to positive activities
that help them develop their creativity during
playtime. For example, um, adults can provide kids
with stimulating toys and games and limit their
time in front of the TV and playing video games.
They can talk to their children about what they
are playing and check in on them. This is not
interference; it's careful monitoring.
Second, children need adults, not peers, as role
models. Too many children are not learning good
behaviors from their peers.. They learn these bad
behaviors because adults don't monitor their
playtime. In fact, it is dangerous to suggest that
. children should learn primarily from their peers.
They need adults~parents, teachers-to teach
them what is right and what is wrong. One way
adults can do this is by monitoring who their
children play with and what kinds of activities
they engage in.
Third, adults need to monitor their kids' play to
protect them. Children can be very cruel to each
other. They are still learning how to control
themselves and don't always think about the
consequences of their actions. Certainly, a child
needs to learn how to defend him or herself. But
children also need to be able to trust that their
parents will help them if they are being hurt by
another child. This is a very important lesson.
Sometimes children need their parents to help
them deal with their peers.

1)3

02 Psychology
W: It' is unfortunate that there are still people who
believe that adult interference in children's playing
can have harmful consequences. The truth is that
adults need to monitor children's playing because
play is so important to a child's development. Not
doing so can have harmful effects on a child's
development.

l:
,;

:C
I/.

fi

%
_\,

#.
,

:-

'

f:

~
~

i-)

.,

..
.,
l
1;

Biology

M: Many of you have undoubtedly heard tales of


the dodo-a fat, funny-looking bird that lived on
an island off the east coast of Africa. It isn't a
coincidence that the dodo disappeared soon
after humans arrived on their island. Yet it wasn't
over hunting, but rather other factors, that led to
its extinction.
First of all, humans didn't even like eating dodo .
meat. It is (rue that the dodo's. flightlessness

I ~ .'~ ~ns!cr!i~!,!'! '!'1!~ ~ ~ ~!il~i-i 'i iil~ji j1i~ ~!i ~i!i i!li i;i li i;i i j i ji i 1i i!li~ili i i ilili i1 1 1l l l l l ~

made it an easy catch for hunters and trappers.


But its meat was tough and tasted pretty horrible.
In their private journals, sailors gave detailed
descriptions of the terrible-tasting dodos while
praising other local species, such as the Red Rail,
for their good taste. And so it is highly likely that
the sailors abandoned dodo hunting and moved
on to more desirable species. It just doesn't
make sense that hunters would keep pursuing a
bad-tasting animal if better options were available.
. Second of all, humans weren't the dodo's main
hunter; other animals were. The dodo's fearlessness
is also often cited as evidence supporting the
over hunting theory. But the truth of the matter is
that the dodo's lack of fear didn't make it vulnerable
to humans. Rather, its fearlessness made it
vulnerable to the animals humans brought with
them. Dogs and pigs, for example, went wild on
the island, rampaging poorly hidden dodo nests
and eating many of the dodos themselves.
Without nests and offspring, the dodos had a
g\eat deal of trouble repopulating themselves,
an& their numbers quickly dropped.
F'iro/ally, there is little evidence of humans hunting
large numbers of dodos. Although a few dodos
di~ make it back to Europe on Portuguese ships,
archaeologists have turned up scant evidenoe of
human dodo hunts. It's much more likely that the
dodos died because humans introduced predators
that had never before existed oh the island and at
the same time destroyed the forests where the
birds made their homes.
86 we can't really blame human hunting for
the extinction of the dodo. There were many
contributing factors. Their loss of habitat was
certainly a factor, as was the introduction of new
predators into the areas where dodos lived. So,
really, we can't say that one thing caused the
dodo to disappear, and it certainly wasn't over
hunting.

04 Environmental Science
M: So, perhaps the most well-known example of an
insecticide is DDT. Now, DDT is notorious for
being an extremely dangerous chemical. It's been
criticized for its toxic effects on the environment,
on humans, and for its inability to completely
wipe out disease. But let me remind you that,
despite its downsides, DDT wasn't nearly as bad

as we've been made to believe. In fact, it had


several benefits.
First of all, DDT can, in some cases, benefit
plants and animals. Yes, DDT, when consumed in
extremely large quantities, will impair, or even kill,
an animal. But in smaller quantities . . . which
is how it was most often found to be in the
environment ... it mostly just kills insects, which
can spread diseases from plant to plant or animal
to animal. Insect-born diseases have been known
to wipe out huge populations of species. And so,
by preventing insect populations from escalating,
DDT prevents certain animal and plant populations
from shrinking.
DDT can also benefit human health. Again, DDT
can be toxic if you ingest it in large quantities, but
even the World Health Organization considers it
only "moderately hazardous." Which means that
in smaller quantities, DDT doesn't really do much
at all if you happen to ingest it. Sure, there are
studies that show a correlation between DDT and
things like cancer or reproductive disease, but
many of these studies are inconclusive. What
DDT does do, though, is kill the insects that
spread deadly human diseases, like malaria or
typhus. Such diseases have far more devastating
effects on human health than DDT ever had. So
by reducing their spread, DDT in fact improves
public health.
Lastly, DDT has effectively combated diseases,
particularly malaria, in the past. Although some
-mosquitoes have now developed a resistance to
DDT, there are many cases in which it successfully
wiped out the disease. Sri Lanka, for example,
had 1.5 million cases of malaria between 1934
and 1955; 80,000 people died. The year after the
country completed an intensive anti-mosquito
program using DDT, only seventeen cases were
reported. Since then, millions of lives have been
saved because of the widespread use of DDT.

~~lill .

~..

01 Political Science

M: Computerized voting systems l1ave drawn a lot of


attention recently as a possible alternative to the
current paper-based systems. fvlany governments
are currently looking into switching to the new

Transcripts

723 -

1
!
i

system. However, before we change everything, I


think it's important to consider some of the
drawbacks of using such a system. Let me give
you three examples of how computerized voting
systems are bad.
First, supporters of the computerized system
say that it is a lot safer than the current voting
methods. But that's not true at all. Let's be honest:
there are just as many possibilities for the system
to be mishandled over the Internet as on paper. If
we're going to start voting over the Internet, we
have to be aware of the dangers that are present
on the web. For example, hackers could completely
ruin an election if they were to attack the computerbased system. A skilled hacker with a particular
partiality could easily get into the system and add
or subtract votes. Clearly, this is no safer than
paper ballots.
Next, computerized systems aren't actual~ cheaper
for governments, either. In order to implement the
system, a lot of expensive equipment is needed
for counting and managing the electronic votes.
So if a country wants to switch over, they'll have
to be willing to invest in the equipment to begin
with. The expenses don't stop there. See, these
machines cost money to maintain and operate.
Overall, the system would introduce an on-going
expense for governments rather than saving
money.
Finally, I want to clarify that it is possible for
computerized systems to lose votes. In fact, they
result in more lost votes than traditional paper
ballots. Why? The system relies entirely on
machines to sort and count votes. This is all very
nice when it's functioning, but what happens if
the machines break down. A mechanical failure
would result in the loss of all the votes registered
on that computer. In the end, thousands of votes
could be lost from a single failure.

02 Resource Management

. :

W: A wildfire that started in the Croatan National


Forest in North Carolina several years ago is a
good example of how prescribed burning can
be indispensable in preventing and controlling
wildfires. This wildfire was actually controlled by
the prescribed burning that had been done in the
area. In the end, the fire wasn't as bad as it could
have been. Let me give you three examples of
why this is true, and you'll be able to see how
724 Transcripts

beneficial prescribed burning can be.


First of all, the wildfire may never have started
in the first place had prescribed burning been
executed in the area where the fire ignited. See,
the forest caught fire in an area of very thick
underbrush-dead leaves and shrubs and
branches and all that, which is not at all surprising
considering the fact that underbrush acts as a
sort of natural kindling for forest fires. Many forest
rangers and fire experts suggested that, had the
place been previously cleared by prescribed
burning, there may not have been enough fuel to
start a fire.
In this case, what prescribed burning did was
stop the fire from spreading. Forest rangers, of
course, couldn't burn the whole forest, so they
just did prescribed burning on the outermost
areas. That way, if a fire did break out ... which,
unfortunately, happened that year . . . it wouldn't
spread too far. So as the Croatan National Forest
fire moved out of the high fuel areas . . . where
there was a lot of underbrush . . . and onto the
timberlands, where forest rangers had already
done some prescribed burning, it slowed down
enough so that firefighters could do their job and
put out the fire. If those prescribed burns hadn't
been there, it would have been pretty bad news
for the surrounding communities.
And what's also great about the prescribed burns
is that they never posed a threat to the communities
because rangers chose the very best conditions
in which to execute them. They chose a day when
there wasn't a lot of wind, when there was a little
bit of humidity, when temperatures weren't too
high. And in this way, they stopped a wild, raging
fire by starting ones they could control.

i)3

Business

M: All right, we've been talking about retirement


plans, and there's one option we haven't covered,
which is early retirement. Now, there are a lot of
people who will tell you that you should never
retire early-that it's too much of a financial
burden, it will make you feel useless and depressed,
and it will ruin your social life. But the truth is, you
can easily overcome those problems and emiv
retirement may be one of the best choices you
ever make.
First of all, many early retirees live very agreeable
lifestyles in terms of their finances. Retiring early

'f'

;y
,i,'-

doesn't mean your money's going to run out in


ten years and you'll find yourself swimming in
debt. Although it's not always easy, it's very much
possible to save enough money . . . or invest
enough money . . . for early retirement. And you
don't have to be making high figures, either.
There are a lot of very good retirement plans out
there. And there are a lot of knowledgeable
financial consultants, many of whom are hired by
companies for the sole purpose of helping
employees retire early. So really, if you plan right,
you can sustain your current lifestyle throughout
your retirement.
Secondly, early retirement can be an extremely
positive experience psychologically. Most people
have values and interests and hobbies that aren't
work-related at all. Retiring early gives them a
chance to spend time pursuing the things they
never had the time for-whether it's fly-fishing,
traveling, volunteering, or hanging out with their
families. And often, because they retired at a
relatiyely
early age, they've got the energy to do
,
SI
whfll th8.'enjoy.
And" finally; with regards to one's social life, early
retir:e.men\is a benefit rather than a hindrance.
Sure; you may lose a few of the casual friendships
that sprung up around the office. But retiring early
can give you the freedom to branch out and
establish new, more diverse friendships. You might
join a running club and meet friends there. Or you
might volunteer at a hospital and have lunch with
the staff every week. Or you might start. hosting
dinner parties. By retiring early, you acquire the
time to build your social network-time you may
not have had when you were working.

04 Law

W: In theory, speed cameras ... which basically are


replacing police officers ... are supposed to be
a really great thing. But the truth is, they're not as
valuable as some people make them out to be.
And here's why.
First of all, speed cameras don't do a whole lot to
reduce traffic accidents. You see, speed is very
rarely the cause of accidents. There are lots of
other causes: bad ,-;:1ad conditions, inadequate
signaling, distract1c1~s. things like cell phones or
. food. So basicaiiy, speed cameras are just
addressing one of many t11ings that induce

accidents. So really, if the city spent more time


and money on tackling these other factors, we'd
see a much more significant reduction in road
accidents than speed cameras will ever bring
about.
Speed cameras also don't do much to reduce road
deaths. Speeding really only makes a difference
on suburban roads, where speed limits are around
twenty miles per hour. Going thirty-five or forty
miles per hour on these streets makes a huge
difference as to how likely a victim is to survive an
accident. But pretty much all speed cameras are
positioned on fast, main roads where the speed
limit is at least forty-five miles per hour. At these
speeds, accidents are quite often deadly, So a
speeding vehicle really isn't all that much more
dangerous than one going the speed limit.
Preventing drivers from speeding on these streets,
therefore, isn't very effective in reducing the
number of deadly accidents.
And one more thing: speed cameras remove
police officers from duty, which isn't a good thing.
Skilled officers are much more able to detect and
deal with dangerous driving, including offences
other than speeding. Drivers could be driving
aggressively, they could be cutting other drivers
off, or they could be running stop signs. All of
these a speed camera will most certainly miss ~
the driver is abiding by the speed limit. It can
actually be dangerous to replace police officers
with cameras.

'I,I

'I

;!

M: Now, we've been talking about marketing.

Today, I want to discuss one popular marketing


strategy called buzz marketing. Many of the
so-called advantages of buzz marketing are also
its disadvantages.
First of all, buzz marketing is fast and easy, which
means it spreads negative information quickly
and easily, too. If you want to get people interested
in your product, so they buy it, you might choose
to use bu.zz marketing, right? People learn about it
quickly and teil all their friends. Great, right? Not
always. Unless you have good testing and know
that your product can live up to ils claims, you

Transcripts

725 - - l

i'

II=

should be cautious about buzz marketing. Bad


information flows just as quickly and easily as
good information. If there is a problem with your
product and you have relied on social networks to
generate positive buzz, well, um, you'll have to
deal with the negative buzz that results when
people find out about that problem. That can hurt
your company's reputation.
Second, if you have bad buzz, people will take it
more seriously if they hear it from their friends.
And that is harder to counter with additional
marketing. Buzz marketing gains its credibility
from word of mouth. One person tells another
person, and since they know each other, they
trust the information. This works fine if you have
a product that lives up to its claims. But if your
. product has flaws, the buzz can turn negative
and that can damage your company's image for
a long time. When people hear bad news about
a product from a competitor, they might ignore it.
When they hear it from a friend or acquaintance,
they take that news more seriously.
Third, buzz marketing can end up being a lot
more expensive than traditional marketing,
especially if you start to generate bad buzz. Just
think of this: a company starts a buzz campaign,
and their product starts getting bad buzz. How
does a company fix this? Well, honestly, they
have to start a new advertising campaign just to
counter the bad buzz. And this means a lot more
money. So, if this happens, it would have been
cheaper to just go with traditional advertising in
the first place.
So 1 hope you can see that buzz marketing isn't
nearly as good as some people think it is. It can
fail too easily.

For one thing, it doesn't actually 'po a whole lot to


prevent tooth decay. There was a study done
recently by the National Institute of Dental
Research which compared the teeth of children
in fluoridated communities with those in
non-fluoridated communities. What it found was
that their teeth really weren't all that different.
In fact, the improvement that fluoridation
supposedly caused was so small that it really
can't be considered significant. The truth is
that dental decay continues to exist even in
communities with fluoridated water. Therefore, it
is hard for us to conclude that fluoridating water
is really such a good thing.
Another thing-fluoridated water isn't as safe to
drink as many people believe it is. There is some
scientific evidence that fluoride is damaging to
the body. It has been shown to weaken bone
strength, which can increase the chance that you
might break a wrist or a hip. Other studies have
shown that fluoride can cause brain damage, that
it can reduce thyroid function ... your thyroid,
controls your growth ... and fluoride might also
cause bone cancer. These are very serious
consequences, and they can happen due to even
just the small doses of fluoride added to water.
So, in the end, fluoridation really isn't cost-effective,
either. Even though we may be saving a small
amount of money on dental bills by fluoridating
our teeth, we're going to be hit with medical bills
later in life after we've been fluoridating our
bodies. And the medical costs for such things as
cancer treatment or even fixing a broken bone
are going to be much higher than the cost of a
few fillings. When you look at the bigger picture,
water fluoridation is going to cost us a lot more
in terms of finances-and also in terms of our
overall health-than drinking water without added
chemicals.

I~
W: One issue that we have yet to discuss is water
fluoridation. We all know what fluoridation is, right?
Well, it's basically the government's solution to
taking care of public dental health. Add a little
fluoride to city water, and you've taken care of
everyone's tooth decay problems. Well, as it
turns out, water fluoridation really isn't all that
beneficial.

r - - 726 Transcripts
c-c-,-,_ -~--~-:----'--- ., ___ "----'--~ - ------------------

--------~-

'-'---

--------- -

_ _ _ _ :~~-:_~__:

t to
1ne

ital
lren
in
vas

mt.
:ion
ally
is
in
~. it
1ter
I

to

me

to
me
IOU
lV9

hat
Jid,
Jso
lUS

1en r

r.
ve,
1all
ing
1ills
JUr

as
1ne
I a
ire,
1re
JUr

led

'
Note: Graphic Organizer answers are suggestions only
and may not match students' answers exactly.

. Preview: ffest

2.
5.
8.
11.
14.

1. (0)
4. (A)
7. (B)
10. (C)
13. (A)

(8)
(A)
(C)
(0)
(A. C, F)

3. (B)
6. (8)
9. (A)
12. (C)

facilitated the audience's understanding of events. By


singing or speaking in llllisQn, the chorus was also
responsible fiir influencing the way audiences reacted to
a play. It did this by showing embellished reactions and
discussing the play's events and characters with the
audience.

03 Ecology
Changes in Coral Reefs
Elfect

Cause

1. Increased ocean Bleaching of coral occurs when it loses


temperatures
algae

: ,~'.Chaptei.il'i
'
.

Sediment fills water and blocks sunlight,

2. Tropical storms

causing algae to die

01 Linguistics

1.

The Effect of the Gold Rush on the English Language


Theory
Example
Many English
expressions

"Pan out" came to mean "to be

successful"

originated during
"Strike it rich" came to mean "to
California Gold Rush become wealthy"
1.

(C)

2.

3.

(8)

(0)

Summary: The passage discusses English phrases that


can be traced back to the gold rush in California. The
phrase "pan out" came from the pans that miners used.
Gold would settle to the bottom of their pans, and the
sediment would be removed. The expression "strike it
rich" originated fromrniners striking the rocks to find gold
and becoming 't/filillb)l quickly if they found it.

ii

, The Greek Chorus and the Audience

.,,;

Role
<----------~---------------!
Helped audience understand play by
Definition

Person or group of

people that added


to a play in various
ways

p'roviding important plot information

----------------;
Interacted with audience, ellher by
showing emotion or by communicating
with audience members

.ii

ii,.

1.

(0)

2.

(C)

3.

(8)

Summary: The role of the chorus in Greek theater was to


provide a commentaey on a play to the audience in order
to ~ the plot. The chorus explained the stories and

~--

(0)

Summary: Global warming is responsible for fillfiling the


ocean's temperatures, which affects coral reefs. Coral
reefs are vulnerable to temperature changes, and they
become damaged if exposedto higher temperatures for
orolonged periods. One indication of coral damage due
to warm ocean temperatures is bleaching. An increase in
tropical storms is another way coral reefs are being
damaged because of global warming. Sediment from
overflowing rivers clouds the ocean, which decreases the
amount of sunlight reaching the corals.

04 Literature
r-~~~~~~~~~~--~

I _

The Romance vs. The Novel

The Romance

<l

i\

--

3.

(A)

The Novel

Both

1----------1----+-~------ Moral stories of heroic


i> Popular
Stories of everyday
adventures
in 151h
ltte

02 Theater
.\

2.

(8)

Used language
considered lofty

century

o Idealized portrayal of
characters

Used language
considered low
w Realistic portrayal

of characters

-----------~----~------

1.

(A)

2.

(0)

3.

(B)

Summary: The passage discusses the appearance of


the novel as it flourished along with the romance in the
15'" century. The invention of the printing press increased
the distribution of written texts to people. The novel often
parodied the impossible characters and formal style of the
romance, instead presenting more realistic temoerament
and motives of people. Also, the main fQQQ[ point of the
novel was character, as opposed to plot in the romance.

728 Answer Key


'"_; ~-o--

fa_(,.,:-

"~-~~--

-----

- J __ _

-:__ '---

-----~c~-~-

..

;_;

05 Anthropology

Chapter 2

Symbolic Behavior
Theory

O1 Psychology

Support

Superstitions can 1. Rituals can give sense of control over


uncertain situations
have positive
psychological
2. Taboos can relieve anxiety in many
effects
situations

1.

I.

{B)

2.

(A)

3.

{C)

Summary: The passage discusses symbolic behavior as


a means for people to QQlm with uncertainty in their lives.
Two common types of symbolic behavior are rituals and
taboos. Rituals are behaviors done repeatedly in order to
bring about a certain 011tcom0. Although a ritual cannot
~ bring about a desired result, an individual can get
a feeling of control through performing the ritual. Taboos
are forbidden behaviors that can make some people
believe that something terrible will transpire. By avoiding
a certain taboo, a person may feel that he or she can also
avoid misfortune.
~
.,,

06 Sociolli'gy

Conversation: Men vs. Woinen

Men

Both

More likely to make


conversation fun and

make jokes
Do not generally
chat with male
friends often

1.

Women

Conversation More likely to


has similar
discuss problems
goals
and emotions

Otten call friends


just to talk

(C)

2.

3.

(B)

(D)

Summary: According to the passage, men and women


llilill!ll.!l in very different same-sex conversations. For
men, talk is generally not personal, is more likely to
include innocent teasing and joking, .and is not very
common. Men often consider personal feelings a sign of
vulnerability. By contrast, women often lliilllQb personal
topics such as feelings and problems with one another.
Female conversation is thought to be very important to
maintain and llliill.!@ female relationships.

"

Globalization

02 Agriculture
Effect

Cause
1. Dutch East India Company began

Water Shortage

worldwide export aclivity


2. Traded with nations previously closed to

Beginning of
globalization

trade, increasing cultural mixing

2.

(A)

Problem
Agricultural

3.

{D)

Summary: The passage discusses the Dutch East India


Company as an important impetus in the trend of
globalization beginning in the 17'" century. It provided an
l!'&illfil for cultural exchanges through the trade of goods
by sea. Through their maritime trade abilities, the Dutch
grew in power and eliminated competition by expelling
the Portuguese and the British from the East Indies.
Through the trade activities of the Dutch East India
Company, hig~ly prized and exotic items such as spice
and textiles were able to circulate around the world.

waste of water

1.

(D)

Solution
1. Use underground watering to reduce
evaporation and runoff

'

I.

2. Grow ncitive plant species that thrive in


an environment

2.

(C)

3.

(D)

Summary: The passage discusses ways in which farmers


are trying to. =Dlfil shortages of clean, fresh water.
When crops are watered above the surface of the ground,
much of the water evaporates instead of reaching the.
plants. By installing new underground irrigation systems,
a more uniform application of water can reach the root
systems of the plants. Another llQYJl! idea for farmers to
use water more efficiently is to grow more native species
of crops. Native crops can grow without excessive watering,
thus slowing the depletion of precious water resGUrces.

Answer Key

729 _

03 Marketing

OS Anthropology
Product Demonstration

Definition

Giving
customers an

opportunity to
experience

the product in
person

1.

(D)

Advantage
1. Can make a

Example
Car dealers often

product more real to give test drives to


a customer, which
customers
can increase sales

2. Can demonstrate

'

unique benefit or

Sales of a company's

(D)

Judges cultures based on


one culture's standards

Von Daniken did not believe


ancient Egyptians built
pyramids

3.

(8)

Summary: According to the passage, while product


advertising is advantageous in selling goods and services,
a hands-on demonstration is often more effective with
prospective customers. Product demonstration makes an
item more illnlJi!.l!ll to buyers, in that they can actually touch
and use it before they buy it. This shows consumers what
a product's function is, which can lead to higher sales of
the product. For example, a manufacturer of non-stick
cookware made many more sales after demonstrating its
product at an exposition.
.
.

1.

(C)

'

Support

'

Example

Developing nations Developing nations


cannot progress
spend more on
while remaining
imports than on
dependent on
exports
other nations

(C)

(A)

3.

Grenada exports
bananas, nutmeg,
and cocoa but must

import many other


products
(A)

Summary: The passage discusses dependency theory,


which explains why some developing nations cannot
integrate into the world economy. These countries will not
succeed economically until they restructure their import
and export programs. Because the revenue.gained from
exports is much lower than the expenditur~s needed to
import basic goods, these nations cannot' generate a
healthy economy. Dependency theorists suggest that the
only 'lia!:lkl solution for these countries .is to increase
manufacturing in order to decrease dependence upon
developed nations for their welfare.
.

,,____ 730 Answer KeY


,, __ -,

2.

(D)

3.

(D)

l,,!
,,

06 History
The History of the Globe Theatre

Dependency Theory

Egyptians must have built


pyramids

Summary: The passage discusses two ways in which


cultures can be understood. The first is ethnocentrism
which is the interpretation of a culture based on th~
standards of one's own culture. Ethnocentrism is thought
to be a universal occurrence. Another way to interpret
cultures is cultural relativism, which judges cultures
according to their own standards. However, this position
can be difficult to adopt without extensive effort and
study of other cultures. Anthropologists help alleviate this
problem.

04 Political Science

Theory

Judges cultures based only


on that culture's standards

cookware went up

function of a product after demonstrating


its benefits
2.

Cultural Interpretations
Ethnocentrism
Cultural Relativism

Step 1: Built in
1599 but burned
down In 1613

1.

(A)

Step 2: Rebuilt in
1614 but closed
down and
demolished
2.

(C)

Step 3: Replica
built in 1997 to
match original

3.

(D) .

Summary: According to the passage, London's famous


Globe Theatre has a complex and .. illustrious/turbulent
history spanning about 400 years: Constructed as QD
open-air amphitheater, its center was a large pit in which
attendees would stand and watch the performances.
After it was destroyed twice, it existed only in the minds
of its admirers for 350 years. Finally in 1997, the Globe
Theatre was again rebuilt. It is an open-air '&lliill. just lil<e
the original.

03 Biology
The Botanical Career of Carl Linnaeus

01 Literature
Unreliable Narrators
Definition

Role
1. Make readers question story and

Narrator who does


truthfully

'

,_;

1.

2. Add realism to characters

(D)

Step 3: Became
professor and
!continued to gather
plant species from
around the world

Step 2: Went on
expedilions to find
new plant species
and developed
Systema Naturae

narrator

not describe events

'

Step 1: Developed
love for plants
and began to
name and classify
them

2.

(A)

3.

(D)

Summary: According to the passage, an unreliable


narrator is a literary~ that is used to challenge the
reader's trust. Unreliable narrators are also used by
authors to add a sense of llll!lifiln to a character. The
example used is "The Cask of Amontillado" by Edgar Allen
Poe. The unreliable narrator in this case is Montresor,
. whose view is lfilDJfil! witih prejudice and a desire for
revenge. Readers must contemplate whether or not to
believe the unreliable narrator, thereby gaining a more
comprehensive understanding of the character.

1.

(D)

2.

(B)

3.

(D)

Summary: According to the passage, Carl Linnaeus was a


~who dedicated his life to creating a classification

system of plants. The process began with a love of plants


as a child. Linnaeus set out on many expeditions to find
new plants ..As a professor, he gained worldwide Ifillll.l'l!l
and collected many plant specimens from around the
world. His Systema Naturae grew considerably into a
multivolume work that was preserved by other scientists
after his death .

04 Psychology

':~

Fuzzy Trace Theory

02 Business

Definition

Successful Entrepreneurship

,.\

Requirement

1. Innovation

Example
Amazon.com rnade buying books more
convenient

2. Risk-taking Debbi Fields took risks to finance cookie chain

;:, ;

1.

(D)

2.

(B)

3.

False
memories
are created
due to brain

making
traces of
events

1. Gist trace: trace of Soccer player may


basic idea of an
entire event

2. Verbatim trace:
trace of specific
details of an event

(D)
1.

Summary: According to the passage, entrepreneurialism


is the launching of a business 'lfilllJlli! in order to make
money. Two factors that can bring about success in new
business are innovation and risk-laking. Innovation is
crucial in fulfilling a unique public demand. Risk-taking is
another necessary component of a new business because
many entrepreneurs are competing in a new or difficult
territory. For example, the owner of a successful cookie
chain had to convince a bank to !iJIBngji her business
concept in order to become successful.

(A)

Example

Category

2.

(C)

remember how a game

went but forget specific


details
Patient may remember
specific details from an
event but forget event

itself ..
3.

(D)

Summary: According to the passage, the phenomenon


of generating false memories can be explained by the
fuzzy trace theory. This theory states that after an
experience, the brain makes traces of information. A gjfil
trace is a remembrance of the event in a general sense.
Because the brain remembers a general sense of the
~ of an event rather than specific details, false
memories may be created to fill in the
A verbatim
trace is a recollection of events based on specific details
or moments, rather than the experience as a whole.

Answer Key

731 - ....

,.
05 Astronomy

Chapter 4
Lunar Craters

Type
1. Impact craters

01 Literature

Explanation
Form from meteorites and asteroids
crashing into surface of moon

2. Volcanic craters

Identifying Theme

Step 1: Examine title

Step 2: Look for repeated words or

for clues

Magma under surlace breaks through

images

and causes surface to collapse

l.
1.

(C)

2.

(A)

3.

(B)

2.

3.

(C)

(B)

(D)

Summary: The moon has accumulated a number of


craters that form its jagged .lfill:lli!J. The Yl!fil majority of
lunar craters are caused by the impact of meteorites and
asteroids. Because the moon has no atmosphere, there
is no protective barrier to burn up meteorites before they
hit the moon's surface. Therefore, when meteorites explode
upon impact, surface material is~ with great force.
Another way lunar craters are formed is when !JlQ!lfilJ rock
rises through the moon's surface, breaking it. A cavity
forms beneath the surface, which eventually collapses to
form a crater.

Summary: The focus of the passage is on literary themes


and how to interpret/discern themes in poetry. A theme is
the general idea that is expressed in a poem. The author
describes two ways of djscerninglinternretinq themes in
poetry. First, the reader can analyze a title, which often
indicates what the poem may be about. For example,
Lewis Carroll's "A Boat beneath a Sunny Sky" suggests
that the poem is about something joyful and llilD.mJil.
Next, the reader should look for repeated words or
images, as they often help the reader conclude the
theme of the poem.

02 Meteorology
06 Zoology
How Aurora Borealis Is Formed

Wolves and Dogs


Theory

1. Bott"l form hierarchical relationships


many characteristics 2. Both are territorial and can be
of wolves
aggressive

Modern dogs retain

1.

(A)

2.

(B)

3.

atmosphere

atmospheric gases

Description
Plasma is made

Plasma travels

Gas atoms build

up of particles

through space;

up energy; release

eventually comes
into contact with
almosphere

it as light

(D)

Summary: According to the passage, while dogs and


wolves bear little physical resemblance to each other,
they demonstrate common traits. For example, their
social structures consist of hierarchies within packs or
families. They are also both highly territorial and become
aggressive to assert dominance and ownership. Wolves
and many dogs have an inherent belligerence toward
strangers. They react to a threat by lli!rirul their teeth and
displaying other aggressive behavior.

732

Step
3. Plasma particles
1. Sun discharges 2. Plasma
plasma
reaches Earth's
collide with
.

Support

1.

(D)

2.

3.

(A)

(D)

Summary: According to the passage, many of the


spectacular weather phenomena seen from Earth are
caused by powerful storms in space. For example,
Aurora Borealis is an extra-planetary storm that results in
the appearance of impressive lights in the northern sky.
Aurora Borealis is caused when particles are discharged
from the sun. Then, the particles QQ]fu;ill with gases in the
Earth's atmosphere. In the process, an electric charge is
transferred to the gases. The collision causes energy to
build up. When the energy is released, the gases emit
lights with a continuously changing spectrum of colors.

Answer Key

'"' c---

- -- -

----r

03 Environmental Science

OS Oceanography
How Rogue Waves Are Formed
Theory
Support

Electric Cars
Theory
Electric cars

are better
for the
environment
and economy

1.

~ ..

(C)

Support
1. Study showed electric cars reduce
carbon dioxide emissions by 100 percent

1. Wave
interactions with
ocean currents

2. Reduce influence of petroleum prices on

2. Wave

world economy

1. Agulhas Current runs past southern


tip of Africa, a region with a high
incidence of rogue waves
2. When two or more waves join, their

reinforcement

respective heights add together

3. Less expensive to operate

2.

(A)

3.

(B)

Summary: The invention of the automobile has had a


profound effect on modern transportation. The passage
describes the !llMm of the electric car and its benefits to
the environment and the economy. Gasoline engines
produce gases and ~ that contribute to global
warm.ing. Electric cars, meanwhile, can reduce emissions
by one hundred percent. Electric cars are also better
because they reduce reliance on petroleum. Decreased
dependence reduces the impact of fluctuating petroleum
prices on the economy.

1.

(C)

2.

(B)

3.

(A)

Summary: According to many personal accrnmts and


mathematical calculations, rogue waves can reach
incredible heights. Stories of rogue waves have circulated
for centuries, but.scientists still are not sure what causes
them. A high incidence of rogue waves off the southern
tip of Africa seems to be caused by contact between
normal wave patterns and ocean currents. In other areas,
rogue waves may be caused by two or more waves
joining together to form one massive wave. However, the
exact circumstances in which rogue waves appear are
still unknown, and in fact, they may sometimes occur
randomly.

o4' Biology
Theory

Altruism and Evolution


Explanation

06 Anthropology
Example

Animal sacrifices
While some bees do not
chances of reproducing reproduce, this actually
Kin
helps species to survive
Selection or surviving to help
evolutionary success of
species

.--L=

1.

(A)

Native American Pottery

~1--.---':.c::;.::c.;.:._~.:..-

2.

(B)

3.

Type

Technique

Use of extremely hot


temperatures

.2. Northeastern

Many
different

Mix materials with clay,. use of


coil technique, rub with stones

materials

(D)

Summary: The passage discusses altruism in biology.


Altruism, or filllfl= behavior, is considered an evolutionary
mechanism that seems to .!:!fill1 the theory of natural
selection. Some scientists believe that by exercising kin
selection, some animals promote the genetic fitness of
their species by sacrificing themselves. For example,
worker bees forgo their ability to reproduce so that they
can protect t11e hive and their queen.

Materials

1. Southwestern Dry clay

1.

(A)

2.

(C)

3.

(D)

Summary: The passage discusses artifacts recovered in


the southwestern and the northeastern parts of the United
States. A popular and lfil'filfil! art form, the pottery of
southwestern Native Americans has an extensive history.
In the beginning, they created more functional pottery out
of clay, subjecting it to extremely hot temperatures.
Northeastern pottery featured many different materials
due to disparate geography. Native Americans in the
northeast also finished pots with stones to add a polished
look.

Answer Key

733 - -

I Vocabulary Review 1 I

02 Biology

1. (B)
4. (D)
7. (D)

2. (D)
5. (A)
8. (B)

3. (C)
6. (C)

9. (C)
12. (D)
15. (C)

10. (A)
13. (B)

11. (B)
14. (A)

16. phenomenon 17. spectacular 18. discharge


19. gap
20. indication
21. (C)
24. (B)

22. (A)
25. (D)

23. (E)

I Mini Test 1 I

.I

01 Environmental Science

'.!

1. (D)
4. .(B)
7. (B)

3.
6.

(B)
(C)

8.

(D)
(D)
(C)

2.
5.
8.

(D)
(A)
(D)

3.
6.

(A)
(A)

2.
5.

02 Marketing
1.
4.
7.

(C)
(C)
(D)

Angiosperms
100 Million
Years Ago

80 Million
Years Ago

Gigantopterids Earliest

Bees helped

Angiosperms

canlained
chemicals

angiosperm

angiosperms

became most

fossil

spread

common type
of plant

250 Million
Years Ago

130 Million
Years Ago

used by
angiosperms

1.

(A)

2.

3.

(B)

(B)

Summary: The passage discusses angiosperms,


flowering plants that continued to !!Jllill long after
dinosaurs became extinct. The earliest angiosperm fossil
is thought to be 130 million years old. It is also estimated
that angiosperms began to evolve about 250 million
years ago from the gigantopterids. Gigantopterids may
be the earliest ancestor of angiosperms. Bees helped the
plants ~ across the world by spreading the plants'
pollen. Angiosperms are now the most common type of
plant on Earth.

03 Economics
Inflation
An increase in prices of all goods and services

Type

Point

1. Larger than active vocabulary

Vocabulary 2. Expands as a person ages


1. Smaller than passive vocabulary

Vocabulary 2. Limited by words needed to communicate


1.

II!

1 ~llii~l1l
1

Hfi11 -

2.

(B)

3.

(A)

Summary: The passage describes two different types of


vocabulary that linguists recognize. Passive vocabulary
is the set of words that a person knows but avoids using
in speech or writing. A person's ~ vocabulary
expands throughout his or her lifetime. Active vocabulary,
on the other hand, is the set of words that a person can
actually produce when speaking or writing.

11
'.I .i

r11:n

(C)

734

Answer Key

People "bid up" price of

Rising production costs drive


up general prices

1.

Vocabulary

2. Active

Cost-push inflation

goods, causing companies


to charge. more

01 Linguistics

1. Passive

Demand-pull inflation

(C)

2.

(D)

3.

(B)

Summary: The passage discusses two economic theories


explaining inflation. The first is demand-pull inflation. This
is when supplies are unable to meet consumer demand.
By their willingness to pay more for the product, consumers
effectively bid up the price. Factors causing demand-pull
inflation include an increase in currency or exports. The
second type of inflation is cost-push inflation. This occurs
when the production price of something increases
Companies must charge consumers more in order to
l<eep the businesses profitable. Cost-push inflation can also
be caused by an increase in the price of raw materials.

04 Photography

06 Anthropology
Contrast

Step
1. Lighting

~:

~'~i
:wfi

The way light hits


subject affects
contrast

!ii,.
2. Exposure

1.

(C)

Origin of Austronesian Languages

Example

Description

Amount of light
that hits film
affects color
2.

(A)

Theory

Support
1. Nine of ten

To increase contrast,

photographer can use


colored lens filters

Austronesian

languages formed aborigines

Austronesian

from Formosan

To ensure proper

languages

exposure, photographer
can bracket shot

originated in
Migration patterns prove
2. Austronesian
ancient
peoples migrated Austronesian-sPeaking
Taiwan
from Taiwan
peoples are genetically

3.

(B)

languages

and linguistically related

f': .Summary: The passage describes ways that photographers

i:

Explanation
Formosan languages
spoken by Taiwanese

can achieve effective contrast in photographs. Proper


contrast in a photograph will f.!irect the viewer's attention
to its most interesting aspects. One way is to regulate the
amount of time the film is exposed to light. By using lens
!i!IBra, photographers can accentuate certain colors and
reduce others. The other way that photographers control
contrast is by exposing the film correctly. Exposure can be
manipulated by changing the aperture or the shutter speed.
A technique called bracketing can allow a photographer
to ch:\Jose an exposure that wjll mndfil the best contrast
in a photograph.

05 Health

to Taiwanese aborigines

1.

(A)

2.

3.

(C)

(C)

Summary: The passage proposes that the often-ig:r}QraQ


aborigines of Taiwan are the descendants of the original
speakers of Austronesian languages. Anthropologists
have uncovered evidence that confirms the origins of this
language family. The most convincing data stems from
the fact that languages usually originate in areas with the
most linguistic diversity. Studies reveal that Formosan
languages form nine out of the ten branches of Austronesian
language. Scientists also can genetically link peoples from
countries that speak Austronesian languages to the ancient
inhabitants of Taiwan by studying their migration patterns .

Fatigue
A cond_ition of extreme sleepiness or exhaustion

Physical Fatigue

Mental Fatigue

Person cannot
perform at usual

Sleepiness or lack Caused by disease

Pathological Fatigue

--

of cOncentration

01 History

level
1.

(B)

2.

(A)

3.

The Formation of the United Nations


1943
1944

(B)
919

Summary: The passage discusses three types of


fatigue, the feeling of extreme weariness or exhaustion.
Physical fatigue can be described as feeling .d.r.ai!:!fil!,
and is caused by physical exertion. This type of fatigue is
a temporary loss of muscle function, and is usually
pronounced after some type of axmilln. The second type
is mental fatigue. It is characterized by feelings of
drowsiness and difficulty concentrating. The third type is
pathological fatigue, which may be caused by a disease
such as chronic fatigue syndrome.
'

Leagu e of

During talks,

Five natiOns
leaders agree meet to
the sta ge for to create
discuss
, future
international goals of
izations ...organization ...___
organization
Nations sets

l.~ga~- _
1.

(C)

.__

________
2.

(B)

__

3.

1945

United
Nations
convenes for
first time

(A)

Summary: The passage discusses the history of the


United Nations. Another international organization called
the League of Nations preceded its formation, but it had
been ineffective in entorc:na its resolutions. World leaders
wanted to avoid another devastating world war, so they
made an agreement to create a new peacekeeping
organization. After a series of wartime conferences with

Answer Key

735 - -

representatives from around the world, the United Nations


officially convened for the first time on October 24, 1945.

'

04 Biology

02 Sociology

1. Directional reaction to stimulus

Urbanization in Mexico City

Problem
1. Water

shortages

Solution

1 Tropisms

2. Nastic

Seek new sourcestar water and increase

Movements

public awareness about water conservation

2. Heavy traffic Improve public transportation to reduce traffic


1.

Plant Movements
Characteristic

Type

(C)

2.

(C)

3.

(A)

Summary: The passage discusses the problems


associated with urbanization. A large population shift
from !l!rn! communities to large cities can cause great
difficulties, especially in areas where population growth
suroasses a city's ability to meet the needs of its citizens.
For example, in the metropolis of Mexico City, issues such
as water shortages and heavy traffic are a challenge for
city officials. Mexican administrators are now exploring
options that will provide people with a safe and rnllilll!sl
water supply and efficient public transportation.

1.

(C)

2. Irreversible
1. Response to environmental stimulus

2. Reversible
2.

3.

(D)

(A)

Summary: While plants may appear to be stationary


objects, they do in fact move a great deal. For example,
ivy plants can interweave with a pole, and sunflowers
Qillll!l their faces toward the sun. One type of plant
movement is tropism. Tropisms occur when a plant reacts
to a directional stimulus. For instance, phototropism is
when a plant moves in response to a light stimulus.
Another type of plant movement is nastic movement. It
occurs when a plant reacts to environmental changes.
Thigmonasty occurs when the plant moves in response to
touch. This movement is a mechanism that allows plants
suc,h as the Venus flytrap. to feed.

05 Health Sciences

03 Astronomy

Effects of Sugar
Binary Star Systems
Classification

Theory

Explanation

'

Support

2. Spectroscopic

Can be seen with a telescope


------Can only be seen with a spectrometer

1. Sugar is
Surveys reveal that people seek sugary
psychologically food to improve mood and become
addictive
anxious without it

3. Eclipsing

Continuously eclipse each other as they

2. Sugar is
physically

1. Visual

orbit around center of mass

addictive

Studies show that rats become addicted


and experience withdrawal symptoms if
sugar is taken away

~~.~~~~_;;,~~~~--'-~.~~~~-

1.

(8)

2.

(C)

3.

(C)

Summary: Pairs of stars that orbit around the same center


of mass in space are called binary star systems. The
point that the mass of a system is concentrated on is the
binary star system's center of mass. Visual binary stars
can be seen through a telescope with high resolving
power. Spectroscopic binary stars can only be seen with a
spectrometer, which measures unperceivable differences
in the light the stars emit. Eclipsing binary stars travel
along an orbit in which they eclipse each other. Studying
eclipsing binary stars allow scientists to determine tl1e
composition of space.

Answer Key

1.

(B)

2.

(D)

3.

(A)

Summary: The passage discusses the theory that sugar


can be addictive. When a person ~ sugary foods
and experiences a negative reaction in its absence, he or
she may have a sugar addiction. Although sugar cane
has been cultivated for thousands of years, too much
sugar can cause tooth decay and suporess the immune
system. According to surveys, people indulge in sugary
foods when they are feeling angry or sad. This suggests
that sugar is psychologically addictive. Evidence that it
may also be physically addictive was revealed after a
study on rats. The rats that were dependent on sugar
experienced tremors and other withdrawal symptoms
when the sugar was taken away.

06 Psychology
How Pfungst Discovered the Clever Hans Effect
Step
1. Pfungst
removed

2. Pfungst
3. Pfungst
had people
asked
Hans from
aside from
questions
Hans's
audience
without
to ask
trainer ask
Hans
queslions

questions

seein.g him

cooler temperatures cause it to condense. Usually, water


droplets fall back down to the ground as precipitation
after they evaporate. However, if a mountain physically
obstrncts the rain clouds, the rain will fall before it can reach
the other side. Therefore, the water never returns to that area.

4. Pfungst had
people ask
Hans questions

to which they
did not know

02 Business
The Online Auction Business Model
Disadvantage
Advantage

answers

Result
Hans
answered

Hans

Hans

answered

correctly

correctly

pertormed
poorly

1.

'.

2.

(A)

3.

(C)

Hans
performed
poorly

1. No time or geographic 1. Decreased consumer confidence


because of criminal activity

constraints

2. Need for increased Internet


security

2. High level of
consumer loyalty

(B)

Summary: The passage discusses a psychological


phenomenon called the Clever Hans Effect. It was
discovered by German psychologist Oskar Pfungst when
he documented the case of a horse that could allegedly
pertorm arithmetic and keep track of the date. In order to
showcase his.\jmazing skills, Clever Hans and his owner
traveled the cgntinent. Pfungst subjected the horse to a
series of tes!S and discovered that the horse was
actually just~ at interpreting human body language.
By being more receptive to QlJllli from the trainer's
involuntary body language, Clever Hans was able to
produce the correct answers.

1.

(0)

2.

(C)

3.

(8)

Summary: The passage is about the online auction


business model, which allows people to llill on and
purchase items via the Internet. The business model has
both advantages and disadvantages. The model does
not have the time or geographic constraints of other
businesses, and it provides high customer loyalty. However,
the anonymity of patrons allows for such criminal activities
as the sale of stolen or co1mterfeit items.

03 Biology
Species Related to Amphibian Evolution
Species

1. Crossopterygian fish 2. Panderichthys . 3. /chthyostega.


Description
Primitive feet; similar

skull and teeth to

01 Geography

1. Water on

amphibians

The Rain Shadow Effect


Step
2. Moist 3. Cold air
4. Mountains

surface

air rises

evaporates

and
cools

I
I

1.

(C)

2.

(A)

causes
water vapor
to condense
and produce
precipitation

--

3.

block rain
from reaching
leeward side,
creating rain
shadow

(0)

Summary: The passage discusses the rain shadow effect,


which occurs when the hydrologic cycle is interrupted.
When the water in the air evaporates, the moist air rises.
When it reaches the high elevation of a mounlain range, the

1.

(0)

2.

Flat bodies, straight First true


tails, and more
amphibian; had
developed feet
legs and lungs
(A)

3.

(0)

Summary: The passage is about the evolution of


amphibians, which are cold-blooded animals that hatch
in water and are born with gjj)Q. Few fossils of early
amphibians have been found, but scientists have
managed to determine some transitional species among
them. From the earliest ancestor, the crossopterygian
fish, scientists observed the development of feet-like
apparatus. Later, the Panderichthys fish was found to have
amphibian-like features, unlike its earlier counteroarls.
Finally, the fossils of the lchthyostega are believed to be
the first QrllPer amphibian.

Answer Key 737 - . .1


I

06 Anthropology

04 History

Cullural Diffusion
Explanation

Wha.t Caused the Fall of Rome?


Theory

Type

Support
Romans conquered other nations, but as

complex to

empire grew bigger and more complex, it

sustain itself

did not have resources to sustain itself

1.

2.

(C)

(A)

3.

Culture close to another adopts an


aspect of other culture

1. Direct contact

In an attempt to solve food shortages,

Roman society
became too

diffusion

"Middlemen" help spread part of a

2. Intermediate
contact diffusion

culture to other areas

One cullure's technology prompts

3. Stimulus

(D)

similar technology in another culture

diffusion

Summary: The passage is about Joseph Tainter's


theory regarding the fall of the Roman Eomi@. This great
civilization has been studied by many scholars.
According to Tainter, Roman society was ruined because
it became too complex. Complex societies need resources
to fillfiltffi the costs of solutions to their problems. In an
attempt to address their food shortage, the Romans
decided to conquer neighboring lands and take their
resources. Eventually, the Romans only created more
problems for themselves and extended their resources
until they could no longer maintain the society.

1.

(C)

2.

3.

(B)

(D)

Summary: The passage discusses cultural diffusion,


when aspects of a culture are incorporated into another.
There are three different modes of cultural diffusion.
Direct contact diffusion occurs when one culture takes on
aspects of a culture that it has contact with. Intermediate
contact diffusion is when the spread of a culture happens
through a "middleman." Stimulus diffusion is when the
knowledge of a certain trait fillill the invention of a similar
trait in another culture, though it may not be an exact

!llDliilll.
05 Psychology
-

Intelligence

Type
1. Analytical

2. Creative

-Ti. Contextual

Details

Related to
skills taught in

Ability to apply
Ability to make
knowledge to new changes to succeed ln

school

situations

any context

01 Biology
Fungi

Characteristic

1.

(D)

2.

(C)

3.

(A)

Summary: The passage is about Robert Sterberg's


Triarchic Theory of Human Intelligence. His approach
studies three components of intelligence: analytical,
creative, and contextual. Analytical intelligence is
measured by proficiency in academic tasks, and is often
determined through the use of standardized tests.
Creative intelligence is how well a person can apply
known skills to new situations. Often, creatively intelligent
people are quite intuitive. Contextual intelligence is the
ability to behave appropriately in any given context.

Answer Key

Role

~!--~~--~~--~~-~

1. Decompose organic
Allbw nutrients to be returned to
material to gain nutrients soil through decomposition

--------.
--------------2. Reproduce through
Reproduce very quickly, which
spores put into the air

1.

(D)

2.

led to their use in medicine

(B)

3.

(C)

Summary: According to the passage, although fungi can


be a nuisance, they are vital to ecosystems. Despite their
destructive capabilities, fungi help to decompose organic
matter so that nutrients will be returned to the environment.
In addition, fungi filillillil reproductive behavior through
the use of spores. Once the spores find a habitable area,
they reproduce very quickly. This reproductive ability has
led to their use in the development of antibiotics in modern
medicine.

02 Business

Definition

Reducing a
product's

04 History
Product Stewardship
Aspect

I,~
~~

1.

Role

1. Manufacturer Use safe packaging and


participation encourage recycling

environmental 2. Retailer
impact
participation

throughout
life cycle

Olmsted and Public Space

Aid manufacturers in gelling


message to consumers

3. Government Encourage recycling and


participation create statewide programs

2.

(D)

(C)

3.

(A)

Summary: The passage discusses the concept of product


stewardship. It encourages environmental protection
plans that are implemented throughout a product's entire
life cycle. Manufacturers have begun using new packaging
that is recyclable and non-!QlilQ. Retailers are also acting
as mediators between manufacturers and consumers.
They are also executing/instating recycling programs.
Lastly, state governments are fostering recycling through
many programs, such as the container deposit program.

Topic

Point

Explanation

Olmsted 1. Thought public Made Central Park resemble


spaces should New York as little as possible
changed
provide escape while remaining cohesive part
the way
from city life
of city
public
spaces
2. Thought public Ensured Central Park was in
were
spaces should a location that everyone
viewed
be for everyone could get to easily
1.

(C)

2.

(BJ

3.

(C)

Summary: The passage discusses the changes in public


spaces effected by landscape architect Frederick Law
Olmsted. He realized that cities lacked a cohesive balance
of places in which to work, play, and live. Olmsted planned
Central Park in New York around the idea that people
needed a sanctuary in order to escape the bustle of city
life. While previous public spaces had ~ to certain
groups of people, Olmsted wanted Central Park to be
accessible to all members of the public.

'.\'

03 Astronomy

05 Psychology

The Runaway Greenhouse Effect


Cause
Effect
1. Greenhouse gases trap solar

Temperature on Venus
radiation, preventing heat loss becomes very high .

2. Heat causes oceans to


evaporate, .water vapor

escapes atmosphere

1.

(C)

2.

No water remains on
Venus

Select'1ve Attention Theories

Theory
1. Filter

Theories

Brain filters
unnecessary
information

Example
In a crowded room, you will
not hear every conversation,
but will recognize your- name

2. Attentional Attention is fixed In a crowded room, one


resource - amount to divide must divide attention among
theories
among tasks
events

-(DJ

Explanation

3.

(A)

1.

Summary: The passage discusses the !l.lllilllifill


greenhouse effect on Venus. Greenhouse gases llil!Yfl to
heat both Earth and Venus. However, their atmospheres
are much different despite their relative proximity to each
other. Venus's high concentration of carbon dioxide acts
as a barrier. This exacerbates the heating effect by
preventing the sun's infrared radiation from escaping
Venus's atmosphere. The heal caused by the gases
allowed water to evaporate and escape the atmosphere,
which is why oceans never formed on Venus.

(BJ

2.

(C)

3.

(AJ

Summary: The passage discusses selective attention


theories, which try to determine why people focus their
attention in different ways. Filter theories say that the
brain filters out unwanted information. For example, a
person disregards conversations in a room if they have
no relevance to his or her thoughts. One's brain can filter
unfamiliar information and redirect attention to familiar
names or voices. Attentional resource theories describe
attention as a fixed amount to divide according to how
much attention one wants to pay to something. A person
can ~a certain amount of attention to certain stimuli.

Answer Key

739

-~

06 Literature
Allegory
Definition
Use of
extended
metaphor
in a story

1.

01 Botany

Role

Example

Plato's allegory of prisoners


1. To describe
difficult concepts in a cave to describe
philosophical knowledge
2. To tell familiar

George Orwell's allegory of

story in new way animals on a farm to


describe a political situation

(B)

2.

(B)

3.

Plant Stems

Plant structure that


supports buds and
leaves and carries

nutrients to plant

Function

Carries carbohydrates
1. Phloem from leaves to roots
2. Xylem

Water and nutrients pulled


from roots to leaves

(C)
1.

Summary: The passage discusses the use of allegory in


literature. Allegory is used as an extended metaphor for
two reasons. The first is to explain difficult concepts. For
example, Plato uses the bondage of prisoners to explain
philosophical knowledge. The second purpose of allegory
is to tell a familiar story in a new way. For instance, in George
Orwell's Animal Farm, farm animals filiJl).e. a revolution to
fil!ii;l humans from their farm. They soon discover that
they are not Ji! to run the farm themselves. The story's
events figuratively represent the political situation of
Orwell's time.

(A, C, D)

Summary: Stems are the support structure for a plant's


buds and leaves. Concentric rings within the stem transport
nutrients downward from the source to the plant's roots
through the pressure-flow mechanism. Water evaporates
from the leaves, which atl!1!.QIB more water molecules
upward from the roots. This process creates l!lillillm in the
interior of the stem for continuous water movement. More
water enters through the roots from the soil, sustaining
the life of the plant.

02 Archaeology

I Vocabulary Review 2 I

Experimental Archaeology
Example
Role

1. (B)
4. (D)
7. (C)

2. (D)
5. (D)
8. (B)

3. (A)
6. (A)

9. (A)
12. (D)
15. (A)

10. (A)
13. (C)

11. (B)
14. (B)

16. Fossils
19. identifying

17. preserved 18. devote


20. instrumental

1.

21. (B)
24. (A)

22. (E)
25. (D)

23. (C)

2. (C)

3. (D)
6. (A)

Summary: Experimental archaeology attempts to


replicate certain skills of past cultures. By doing this, we
can determine what made ancient artifacts so durable.
Archaeologists use only the primitive materials and
methods available at the time the items were made. One
experiment recreated the process of paint-making using
ores, minerals, and sedimentary deposits. Then, animal
fat was extracted using tools available at the time. Finally,
yucca roots were added to recreate the substance likely
made by ancient paint makers.

Definition
Subfield of
archaeology that
attempts to

processes

01 Zoology
1. (B)
4. (B)
7-. (B)

5. (C)
8. (D)

02 linguistics
1. (B)
4. (C)
7. (A)

Paint~making using
Reveals new
information about primitive methods
ancient cultures and materials

replicate ancient

I Mini Test 2 I

740

Part

Definition

2. (C)
5. (A)
8. (A)

3. (C)
6. (B)

Answer Key

. --"'---'

(A, B, E)

03 History

05 Art History

Entrepreneurship During the California Gold Rush


Cause

Effect

California Gold
Rush

1.

z.
&. '

f"

i."'

Levi Strauss

Increase in
entrepreneurial

capitalized on rieeds

opportunities

of gold miners

(C, D, F)

Summary: The 1849 gold rush in California inadvertently


created opportunities for entrepreneurs to make their
fortunes in ways other than mining. A rapidly growing
population increased the demand for basic necessities
such as food and clothing. While some merchants io!!file.Q
their prices to take advantage of the miners' !lligbj, many
were honest and hardworking. Levi Strauss was a merchant
who sold various types of fillllfilfil, including a type of
work trousers that used metal rivets. This provided a type
of pants that fulfilled miners' needs in a new way and led
to his success.

}
.,

Stieglitz Photography

Example

Definilion

Photography as
art rather than
production of
standard
photographs
1.

Role
Promoted arlistic

side of
photography

Example .
Equivalents-images
of shadow and light
to capture moment of
experience

(B, C, F)

Summary: Alfred Stieglitz was a photographer who


made his work an expressjon of artistic interpretation of.
the world around him. Dada art had a profound jmpaQl on
the way he viewed photography, so he became interested
in the promotion of photography as art. Stieglitz edited and
published a photography periodical and focused much
of his work on urban surroundings. His Equivalents series
was intended to completely capture a moment in time,
thereby affording the viewer an eg11ivalent experience of
the feeling of that moment.

04 Astronomy
Why the Sky Is Blue
Theory
Molecules

Support

The Role of the Media in Socialization


Disadvantage
Advantage

1. Rayleigh discovered sunlight is scattered

in the
by water molecules in atmosphere
atmosphere
scatter color 2. Rayleigh found that light scattered is mostly
in blue part of spectrum, giving sky blue color
in sunlight

1.

06 Sociology

(B, E, F)

Summary: The sky appears to be .blue because of the


scattering of light waves against water molecules in the
atmosphere. The colors of the light spectrum we can see
lilJ1llil from red, which has the longest wavelength, to
violet, which has the shortest. When light rays are broken
or refracted by water in the atmosphere, colors appear
separated because of their wavelengths. The scattering
of light molecules in the sky is Qrn!;lominantly at the blue
end of the light spectrum because of its shorter
wavelength, most easily seen by the human eye. Farther
from t11e sun, the greater saturation of color makes the
sky appear a deeper blue.

1. Educate children about


1. Show v'1olence, which can
academic and social skills
teach the wrong lessons
2. Provide common topics
to discuss

1.

2. Depict stereotypes that are


learned and believed by public

(C,D,E)

Summary: The passage discusses the media's role in


socialization, which has only intens'1fied as television's role
has grown in society. Television is undeniably an important
part of modern culture. The media have educational and
interactional advantages and disadvantages. Many people
believe that the med'1a help educate children and give
people common topics to discuss in social settings.
However, others believe that the media promote violence
and implant often unrealistic stereotyQes.

Answer Key

741

-,:'Y"'~~ .

Chapter 10

1.
Society

Statements

01 Geology

A married woman has a choice of several

Hunter-Gatherer

SeiSmic Waves

Attribute

Classification

1. P Waves
(Primary Waves)

1. The fastest waves


2. Can travel through any medium
3. Vibrate in the direction they travel

Herds often split into groups that move


separately.
About 12,000 years ago, humans
thought to develop this method.
Sons usually live in a group with their
fathers, even after they marry.

Pastoralist

1. Slower than P waves

2. S Waves
(Secondary Waves) 2. Travel through only solid mediums
3. Vibrate at right angles

1.
Statements

Seismic Wave
G

Pwaves

Produce vibrations thar move in the


direction of the waves

Move through underground water


Are the fastest moving waves
Can only move through solids

S waves

o Produce vibrations thatmove up and


down or side to side

Summary: According to the passage, earthquakes


generate shock waves known as seismic waves. Two
kinds of seismic waves travel through the Earth's interior.
Primary waves, or P waves, travel outward from where an
earthquake originates. They can move through solids,
liquids, or gases under the surface of the Earth,
alternately compressing and expanding them. S waves
are secondary waves that can only move through solids.
They vibrate at right angles to their direction. s waves
travel more slowly than P waves and can only travel
through solid mediums. Scientists can compare S waves
and P waves to pinpoint the location of earthquakes,
even thousands of miles away.

Summary: The way in which a pre-industrial society finds


or produces its food affects its social structure.
Hunter-gatherer societies, in which members forage for
food, are egalitarian. Members are usually related
through marriage or lsinfilliu. Everyone gets an equal
shar.e of the meat, and they can easily move from one
band to another. Pastoralist societies are hierarchical.
Usually an older male member heads up the group,
which consists of his sons and their families. Pastoralists
also rely on domesticated animals for food and other
products.

03 Philosophy
The Philosophies of Plato and Aristotle
Problem
What is the
best form of

Both

o Lots of social
mobility among
bands

Even ruler must be subject

to laws

1.
Philosopher

Based on
small groups

Plato

Statements

ai

Pastoralists
&

Hierarchical
e Extended family

II

__ . ]

headed by father
Part or all of group

moves with herd

Rule of law is necessary for good government


in the absence of a philosopher-king.
A world exists that is beyond the human

senses.
o. Man perceives reality through his senses.
Aristotle

o A gove~nment based on the rule of law is the


best method.
o Even rulers must follow the laws of the state.

I
~,

mt--- 742 Answer Key


.:..-

1------+------------1

e The world of Forms is the true reality.

1--~~~~~-+--~

Egalitarian society
Basic unit is band

Explanation

Only philosopher can


1. Philosopher-king
understand ideal of good life

Only a philosopher can understand the ideal


of the good life.

Hunter-Gatherers vs. Pastoralists

Theory

government? 2. Rule of law

02 Anthropology
Hunter-Gatherers

different groups with whom she can live.


Members of the group are treated as
equals.

._, ___ _

Summary: According to the passage, Plato believed


reality existed in a world of Forms that could only be
understood by philosophers. Because human senses
cannot give a perfect rendering of the world, it is
only through the mind that humans can truly know
reality. For Plato, the ideal form of government was a
philosopher-king, who could fillfilllil that the concept
llfilil&d. from the world of Forms was promoted. Without
a philosopher-king, Plato believ.ed that the rule of law was
the s11bsequent form of good government. Aristotle,
however, believed that what people perceived through
their senses was reality. His theory that even rulers
must follow the rule of law formed the underoinning of
constitutional governments today.

05 Sociology
The Changing Ideas of Race
Meaning
1. People who shared 2. Race was
3. Race is cultural
same identity or
characterized
idea that has no
way of life were
by physical
biological basis
called a race
attributes

1.
Period

Original Usage Said that a race of people shared a lifestyle


16111 and 17 111
Centuries

'i
'

,f*

Concept of Race

04 Art History

Divided people into. races according to

physical traits
Stated that parents pass racial
characteristics to their children

Baroque vs. Rococo

Baroque

Both

Very large
buildings

Integration of
painting,
sculpture, and

Bright colors
lrlipressive.Views

architecture

Rococo

Pastel tones

Unified interiors

Statements

Style

Palaces were especially massive and imposing.

Baroque

Different, dramatic views were used in the


design of the interior.
o Paintings using bright colors decorated
church interiors.

Rococo

Swirls and Curves were used in interior

design.
Sculptures tended to be smaller and less

_J__~~--------Summary: There are four key elements of Baroque


architecture. Buildings were massive. Architecture,
painting, and sculpture worked together. The interiors
used bright colors and different materials, and there were
dramatic views off the main visual axis. Baroque
architecture was too robust for the tastes of the time.
However, Rococo architecture was a refinement of these
elements. Its decoration was more delicate and subtle. It
also featured the integration of painting and sculpture
with its architecture.

Proved that races are not determined by

biology

Less imposing,
smaller scale

~.

Architectural

20 Century

Described race as a cultural creation

Summary: According to the passage, it was conventional


wisdom that race simply denoted a group of people who
shared an identity, such as being from the same country.
Race meant something much different from its current
association. Later, people thought race was a biological
fact. It was believed that certain physical and behavioral
traits could be used to distinguish each race. Finally,
scientists showed there was no scientific basis for the
idea of race, and it was actually a cultural construct.

06 Economics

-E

'Keynes

_12eyne~ -

iJ

Money supply does


not affect economy

1J

Key to strong
economy is demand

:. Spending will create


deinand and
slrengthen economy

vS~ Friedman

1--B-0-th--.-----F-ri-ed_m_a_n-----;
~

Based
on
quantity

theory of
money

-(f

"It

Money supply affects


economy
Contraction of money

supply caused Great


Depression
o More money available
for spending increases

health of economy
----~---~--~-------~

Answer Key 7 43

03 Biology

1.
Theory

Statements

1.

Keynesian Governments can change tax policies to


Infectious
Agents

change consumer spending.

The employment rate will fall without sufficient


demand.
The health of an economy depends on three
kinds of spending.

Virions

Monetarist Governments need to control the supply of


money in order to 3.ttect the economy.

how governments can influence the economy. Keynesian


economists believe that three variables affect economic
activity: consumer spending, government spending, and
investment. Keynes's theories became so widely accepted
that economists rejected monetary policy to manage
economic activity. On the other hand, Friedman's
monetarists believe that money supplies have a QQ.lJfilll
effect on the economy. They attribute the fiscal disaster
of the Great Depression to a contraction in the money
supply, causing the market to decline.

Can be prevented before infection


Include a piece of genetic material that is
replicated
Cau_se irreversible damage to tissues

Prions

The economy will be negatively affected by a


contracted supply of money.

Summary: Two economists developed new theories on

Characteristics

Are composed of matter found In the body


Form an abnormal and irregular structure

04 History
1.
Theory

Statements
A piece of a large asteroid may have

broken away in space and hit Earth.


Chicxulub
Theory

An object from outer space left a crater on


the Yucatan Peninsula.
Dinosaurs became extinct because of a
meteor's impact on Earth.
Global warming caused by volcanic
activity contributed to extinction.

I Vocabulary Review 3 I
1. (B)
4. (B)
7. (0)

2. (D)
5. (C)
8. (C)

3. (A)
6. (A)

9. (A)
12. (0)
15. (A)

10. (B)
13. (C)

11. (B)
14. (D)

16. denotes
19. stereotypes

17. construct
20. ranking

18. underpinning

21. (B)
24. (E)

22. (C)
25. (D)

23. (A)

Volcanic eruptions released toxic gases


and caused acid rain.

Keller's Theory Di.nosaur species weakened after years of


stressful living conditions.

The Earth was bombarded by multiple


meteors that diminished dinosaur numbers.

I Practice Test I
01 Psychology
1. (B)

4. (B)

I Mini Test 3 I
01 Health
1.

(B, C, E)

C2 Business
1.

(B, E, F)

744 Answer Key

7. (A)
10. (C)
13. (B, D, E)

2. (C)
5. (C)
8. (B)
11. (0)

3.
6.
9.
12.

(B)
(A)
(A)
(B)

02 History
1. (A)
4. (B)
7. (B)
10. (D)

2. (C)
5. (C)
8. (B)
11. (A)

3. (C)
6. (A)
9. (C)

Theory
War Theory
Religion
Theory

Statements

Ongoing attacks weakened the city.


Thai soldiers invaded and looted the city.
The predominant religion was originally Hindu.
The change to Buddhism destroyed the
people's unity.
The developments were too vast to manage.

Groslier's
Theory

The irrigation system sapped the city of its


resources.
The ci!Ys land was eroded and overused.

03 Astronomy
1. (D)

4. (A)
7. (B)
10. (D)
13. (A, D, E)

v.:1

2. (B)
5. (D)
8. (B)
11. (D)

3. (A)
6. (A)
9. (B)
12. (B)

2.
5.
8.
11.

(C)
(C)
(B)
(B)

3. (D)
6. (D)
9. (B)
12. (A)

2.
5.
8.
11.
14.

(C)
(B)
(D)
(B)
(A, D, F)

3.
6.
9.
12.

04\Music
if''
;-r-'"

f'. (D)
4,_,
7,
10.
13.

(B)
(A)
(A)
(B, C, F)

05 Biology
1.
4.
7.
10.
13.

(B)
(D)
(C)
(D)
(A)

(B)
(B)
(A)
(B)

Answer Key

;-- _______,._-/--.'-.

745 ___;__...,

Note: Graphic Organizer answers are suggestions


only and may not match students' answers
exactly.

01

02 History

1. (B)

2. (D)

4. (A)

5. {B)

3. (A)
6. (A)

Alexander the Great and Greek Culture

Contributions
His notoriety gained many
admirers in other countries

Had a policy of fusion for


foreigners

02
8. (A)
11. (B)

7. (D)

10. (C)
12.

Had his Greek generals


marry Persian noblewomen

9. (B)

YES
Elongated subjects

Summary: The transactional linguistic model helps


linguists understand the process of language acauisition.
The theory says that the learner is influenced by his or
her environment. It also says that the learner can have a
similar effect on other students within the environment.

NO

Highly realistic

Representative of Renaissance art

I
I

Inspired by his imagination

03
13. (B)

14. (D)

16. (B)

17. (D)

1. (C)

2.

(D)

Summary: Alexander the Great spread Greek culture


to many parts of the world. To promote cultural
assimilation, he had his generals marry Persian
. women. The professor also mentions Alexander's
notoriety, giving the example of Romans trying to
associate themselves with Greece by speaking
Greek.

15. (B)

03 Office Hours
Problem
Student is unhappy with
professor's teaching style

01 Linguistics

E:

~t~~

1.

The Transactional Model of Language Acquisition

Points

----------

Examples

anguage environment affects Child might learn an accent


ne' s language acquisition

based on where he/she


learns the language

ear ner can influence his/her

Language learner may use


certain words/phrases that
are adopted by others

ronment
--~~------M

1. (D)

746

Answer Key

2.

(B)

Ancient Romans spoke Greek


in an attempt to associate
themselves with Greece

(C)

2.

Solution
Professor agrees to consider
changing class discussions

--

(A)

Summary: The student goes to see the professor


because she thinks she is not spending time
effectively in the class. In particular, the student
thinks that the group discussions waste time. She
feels that class time could be better spent by listening
to lectures. The professor agrees to think about
changing the way he teaches the class.

''

1, (C)

04 Service Encounter
Problem

1. (D)

2.

Woman helps student


get network password
and connect to network

(C)

Summary: The student called the employee


because he needs help connecting to the campus
network. The woman discovers that the student has
not yet received a network password. The woman
walks the student through the steps of obtaining a
password and logging-on to the network.
05 Literature

Emily Dickinson

Both

Female Contemporaries
Standard grammar
Wrote about
religious themes

Wrote on
importance of
self and emotion

(B)

Problem

Solution

Student is worried that


professor's class will repeat
material from another class

2.

Professor assures student


that material will be more
advanced in her class

(B)

. Summary: The student approaches the professor to


ask about what material will be covered in the class.
He is concerned because the professor assigned
the same book that is used in an introductory
course. The professor reassures the student that
new information will be covered in her course.

Emily Dickinson vs. Her- Female Contemporaries

1.

07 Office Hours

1. (A)

Unique slyle-,used Same


unconventionBI
literary
punctuation i[id
period
grammar

(C)

Summary: The professor discusses why diseases


that are treatable today once killed so many people.
The first reason mentioned is the lack of understanding
of how diseases are transmitted. Another factor is
the lack of immunity to certain types of diseases
within some populations.

Solution

Student is unable to
connect to the campus
network

2.

r!

08 Service Encounter
2. (B)

Summary: The lecture compares the poetry of


Emily Dickinson to that of other poets of her era.
The student says that Dickinson's style is marked
by unusual punctuation. The professor adds that
she used unusual grammatical patterns as well.
The professor also says that Dickinson focused on
very personal themes, while other writers focused
primarily on religion.

06 Medical Science

t=---=~-- Diseases}~.0~E~:.t___-=--=.------~

bk"~use _______. _~----~~-~Effec!_ ____ j


Lavk of understanding of
infectious diseases

Solution

Problem

:
~

r---------~---~

Student wants information


about university meal plans
and wants to to change
plans

1.

(C)

2.

Employee suggests plan with


'
fewer meals

(C)

Summary: The conversation takes place in the


Dining Services administrator's office. The student
asks for more information about the university meal
Qli!illi. The administrator explains that as a freshman,
he must remain enrolled in one of the three campus
meal plans. The student listens to the information
and chooses a plan with fewer meals than the
default plan.

Scread of infectious
!
Lack of immunity due to linte ; diseases ana higher mortality )
to no access to
) rates
'
vaccinations or no exposure
to foreign diseases
!
j.

_J

--~-~------~-

''
'[!
Answer Key

747 ------:

09 Geology

. Chapter 2
.

Types of Landslide Processes


Types

01 Sociology

Characteristics
Dyads

Rocks move slowly down slope

Creep

Definition

Water can affect speed


Faster than creep

Debris Flow

Excess water causes soil to slip

Very fast
Very dangerous to landscapes

Debris Avalanche

Characteristics

Intense because each member's attention


is focused on other member

Groups of
just two
people

Unstable because if one person leaves,


group is finished

1. (B)
2.

1. (A)

2.

Summary: The professor discusses the three different


types of landslides, beginning with lllilflP, which
moves slowly and can be affected by water. The
professor then discusses debris flow, which moves
faster than creep and is caused by soil being
saturated with water. The professor then describes
a debris avalanche, which follows the same
process as creep but moves much faster and causes
more destruction.

02 Psychology
Children and Panic Disorder

Theory

10 Zoology
Play-fighting in Animals
Example

Reason

To learn to fight and hunt

Wolf mothers
encourage fighting as a
learning tool

To establish who will be the


leader

2.

3. (D)

Summary: The professor discusses dyads, which are


groups of two people. This type of group is yery unique
for two reasons. First, it is an intense relationship,
since each member has only the other member to
focus on. Second, it is a very unstable group-if
one person leaves, the group is finished.

1. (C)

(D)

(A)

Wolves fight to
determine who will be alpha
male

(B)

Summary: The professor gives reasons for


play-fighting, which is when animals appear to be
fighting, but are actually just playing with each
other. The first function of play-fighting is to help
animals learn to fight and hunt later in life. The
professor also says that animals fight to establish
who the 1lllidfil is.

Support

There is a link
between awareness
of one's body and
development of panic
disorder

.
1.

(B)

2.

The ability to guess one's heart rate


was linked to development of panic
disorder
Some who react strongly to pain will
develop panic disorder because they
think their bodies are being attacked.

(D)

3.

(A)

. Summary: The professor discusses a theory that


links panic disorder with being aware of one's
blli;!y. The professor describes the study, in which
children were asked to guess their heart rates. The
study showed that the children that guessed their
heart rates accurately were more likely to have anxietv.
The professor also discussed how people who
worry when they feel pain can develop panic disorder.

.,'

748 Answer Key

03 Office Hours

1.

Problem

Do better on next exam


Study main points of book and
lectures
Visit professor to clarify main points

(B)

1.

2. (C)

3.

3. (C)

(A)

(D)

Summary: The student goes to see the professor


because he is worried that he is not doing well in
the class. The professor suggests that he try to do well
on the final exam. In addition, the professor suggests
studying only the main points. The professor also
recommends that the student meet with her to talk
about the material on the clear up any remaining
confusion about the material.

04 Service Encounter
~'
...

2.

Summary: Roses are vulnerable to various djseases


that are caused by fungi, bacteria, and viruses.
One of the diseases is called black spot, which
causes round black spots to appear on the ~.
causing eventual damage to the entire plant.
Another disease called rose rust causes orange
spots to appear on the leaves, as well as the stems.

Solution

Student wants to
improve grades

(A). (B)

06 Anthropology
Tenochlillan
Theory

Support

Some civilizations adapt


environment to their needs

1. (B)

2.

Aztecs built floating gardens


to expand in uninhabitable
swamplands
Aztecs built aqueducts to
provide fresh water to area
surrounded by swamp

(A)

3.

(C)

,~

Problem

Solution

Options
University
library-fast but
high demand

.;
'i

Student needs
a journal but
cannot find it

Librarian
suggests
interlibrary loan

1.

2.

(C)

(A)

Public library-slow
but less demand

3.

(B)

Summary: A student needs a science journal at the


library but cannot find it. The librarian suggests
something called an interlibrary loan, which is a
system that allows the library to borrow books and
journals from other libraries. She can look for the
journal at a public library, where it will probably be
in stock. However, the student asks her to look at a
university library, which will be faster.

Summary: The professor discusses how ancient


civilizations sometimes had to adapt to environments
that were difficult to live in. The Aztecs adapted to
their environment in a couple of different ways. They
had to build floating gardens over the swampland
to make room for people to live. Second, they built
aqueducts to provide fresh water to their city, which
was surrounded by salty water.
07 Office Hours
----------------------~

"Introduction to International Relations" vs.


"Representations of Race and Politics"

-ln-te;;a-tio-n;;IR~i~tio_n_s r~--s~h

Race and Politics

-1---~-----+------1

Easier class

Advanced

Political

science classes
Less interesting
to student

OS Botany

-------------C-------------.

Diseases Affecting Roses

se 1: Black Spot

!.

Student has

More personally

relevant

interest

--~-- --------~------~

Disease 2: Rose Rust

1.

(D)

2.

(8)

3.

(A)

------------------j
-----------L------~--------

Characteristics

Round black spots on leaves 1 Orange spots on leaves,


. orange dust

Ir

--~~ed -p1ant----J-'"Br;:;;gr9wths

---------

-----

Answer Key

-.O~i.-~~=~-='-"

- - - . - ----

749 _ ___,

Summary: The student goes to the professor's


office for advic(;l. The student cannot decide which
class ~- She likes both classes, though one
class is more appropriate for her level of study.
However, the other class is more interesting to her.
The professor recommends the more advanced
class.

Solution

Problem

1. (D)

2.

Worker suggests student .


apply for other scholarships

(B)

Celtic Bards

2.

(A)

Summary: The student did not realize that certain


university classes had fees, and she now has .
$200 fee she can't pay. She doesn't know how she
will pay in the future. The student did not know that
she could apply for more than one scholarship, so
the worker advises her to research additional
scholarships.

Celtic musicians who played


instruments and sang

(A)

2.

Helped pass on history by


helping people remember
stories

(B)

3.

(A)

Summary: The lecture defines bards as musicians


that played instruments and sang. Bards are a part
of Celtic culture and had an important role in its
history. The professor discusses the role of bards,
saying that they helped the people remember the
stories that described their b.ifilllrY. The catchy
qualities of music helped the people ieniember the
stories.

I Vocabulary Review 1 I
1. (C)

4. (B)
7. (0)
9. (D)

09 Political Science

Role

Definition

1.

08 .Service Encounter

Student has class tee that


scholarship will not cover

1O Music History

12. (C)
15. (A)

2. (A)
5. (A)
8. (C)

3. (A)
6. (0)

10. (B)
13. (B)

11 (0)
14. (C)

Absolute Monarchy
Advantage
Creates stronger
government

succession

!l

tiI

il
ii'
11

:u

:r,.
- - -

(C)

3.

-(A)

Summary: The professor discusses the advantages


of absolute monarchies. First, absolute monarchies
can give a good leader the power to make good
changes. The professor uses the example of
Catherine the Great. With an absolute monarchy
there is no need for elections. This made government
easier in the past, since many people could not
read or write.

\l

2.

21. (E)
24. (B)

Many people in the past


were illiterate, absolute
monarchy simplified

No need for elections

1. (C)

16. tuition
17. fees
18. contract
19. extensions 20. disenrolled

Support
Catherine the Great made
many changes because
she could do. whatever stie
wanted

750 Answer Key

22. (A)
25. (0)

23. (C)

2. (B)
5. (C)

3. (0)

8. (C)

9. (A)
12. (0)

I Mini Test 1 I
1. (C)
4. (A), (B)

7. (0)
10. (B)
13. (A)

11. (A), (0)

14. (C)
17. (B)

15. (B)
18. (B)

6. (A)

16. (C)

,(

<l:

. Chapter 3

\\

..

''

1.

01 Literature
Genre

'

Definition: A category of art, literature, or music

'

Example 1: Tragedy

Example 2: Comedy

Definition: A heroic character


encounters bad fortune

Definition: People become


confused through
misunderstandings

Example: Medea with her


tragic flaw - jealousy

Example: A Midsummer
Night's Dream - people fall
in love with anyone they see

i-

1/

't

~
P.i

u
ll
~

~!.<:
l

~
!~

,.;J,

-~
"('

"

ti.

f,
,......('.i

;:

(B)

2.

3.

(A)

Summary: The conversation takes place between


a student and a professor. The student approaches
the professor in order to ask his advice about her
academic plan. She enjoyed his class, so now she
is considering changing her major. The professor
expresses concern that the student considering
doing this so liJre. in her academic career. He thinks
that a minor in psychology might be better.

04 Service Encounter
Problem

1.

2.

(A)

(B)

. 3.

(D)

02 History

'

Solution

Student needs help writing


essay

Summary: The professor defines a Qfillill as a


category of art, music, or literature. She mentions
two different genres. First, tragedy features a hero
who experiences bad things due to a flaw in his or
her character. Euripides was a writer of tragedy. The
professoW,also mentions comedy, which dealt with
misunderstandings. Shakespeare wrote lighthearted
comedies about confusion in love.

The Sea Dogs

(C)

1.

2.

(C)

Employee suggests
finding a writing fellow

{A)

3.

(C)

Summary: The student visits the Academic


Support Center to request help with a paper. The
coordinator suggests that he work with a writing fellow,
who would be able to walk him through the entire
essay-writing process. The student fills out an
application, which the coordinator tells him will be
used to find a writing fellow for him .

05 Environmental Science
Example

Definition

I-------:".---,---;------------

Stole gold from Spanish ships


English pirates that attacked to harm King Philip
Spanish ships
.,fT~~-k ov~78Pa-n-ish_p_o_rts_i_n--l
the name of England

'--------------~----------

1.

(C)

2.

(A)

3.

------

..

(C)

1.
Step 1:

':3 Office Hours

i .

Evaporation

Description: Water turns into water vapor

----------------------!
Step 2:

Condensation

Description: Water vapor turns back to liquid

-------------Step 3:

Summary: The professor discusses a group of


English pirates called the Sea Dogs, who stole from
Spanish ships. The Sea Dogs were hired by QlJllfill
Elizabeth of England. They stole money from
Spanish ships to try to hurt l<ing Philip. They also
took over Spanish ports.

'

The Water Cycle

------1

Advection

1 ~~:_:~~~~~~--~~~8!~-~::~~~ curren~~~~-~an

to land

~--------- ----------------'
Step 4:

Precipitation

Description: Water falls.back to Eartll

..

1.

(A)

2.

(B)

3.

(A)

Summary: The discussion is about the water cycle.


The professor explains the process, beginning with
evaporation, when water becomes water vapor.
Mext, the water condenses and turns into a liquid
again. Advection is the third step, in which water
moves in air currents. Last, precipitation happens
when water falls back to Earth.
Answer Key 751

'------.--.--!--'"''

'

06 Psychology

08 Service Encounter
.

The Tuckerman Model


Step 1:
Forming
Description: Group convenes and attempts to establish
familiarity
Step 2:
Storming
Description: Group members are in conflict-as basic rules
for operation are developed
Norming
Step 3:
Description: Group matures and members are able to agree
upon modes of operation

Step 4:
Performing
Description: Group reaches a point where it can
maximize strengths and be productive

1.

2.

(A)

(A)

3.

(B)

Summary: The professor introduces the


Tuckerman model and describes four stages in
group development. The first is "forming," which is
when the group is first formed. The second is
"storming," which is the time of conflict as the
group struggles within itself. In the third stage,
"norming," the group be.gins to mature and form
shared values. The final stage is "performing,"
which is when a mature group is able to work
together in an effective manner.

07 Office Hours
Problem

Solution

Mistake in final exam

Prolessor will try to find

schedule

alternate time to offer exam

1.

{B)

2.

{A)

3.

(A)

Summary: The conversation is between a student


and a professor The student points out to the
professor that he has made a mistake in scheduling
the final exam. The professor is surprised to learn of
his mistake, but assumes that it will not change a lot
for his students. The student does have a conflict,
though. The professor agrees to ask the class if
they can ~ the exam time.

Problem
Student needs housing for
unusual period of time

Solution
Director agrees lo let
student live off-campus

2. (B)

1. (B)

3.

(C)

Summary: The conversation takes place in the


housing director's office. The student has come
there to request permission to live off-campus
because he will be returning to campus &Jill
unusual time. The director agrees to write a letter
for the student granting permission for him to live
off-campus.

09 Astronomy
Star Development
Step 1:

contract

Step 2:
Star reaches
main
sequence
phase,
begins
hydrogen
fuSion

1. (B)

2. (A)

Protostar
begins to

Step 3:

Step4:
Star's energy
Hydrogen
fusion causes used up,
begins to die
reactions,
star gets
brighter and
hotter

3.

(B)

Summary: The process describes the different


filllgllli in the life of a star. First, the star begins life
as a prcitostar when energy is created by atoms
colliding. In the next stage, fusion in the star's core
begins, and the star becomes a main sequence
star. Next, reactions cause the star to get hotter and
brighter. Lastly, when the star runs out of energy, it
begins to die.

10 Biology

==i

Order Cetecea
Point 1: Dolphins, whales and porpoises

---:--i

:P=o=in=l2=:=E=vo=lv=e=d=ab=o=u=t5=0=m=il=lio=n=y=ea=rs=a=g=o=~~~~~1
Point 3: No body hair-use blubber to keep warm

1.

l'llt--- 752 Answer Key

(A)

2.

(D)

3.

(B)

,,

~.:

Summary: The discussion is about the Order


Cetecea. The professor gives examples of animals
included in the order. Cetaceans evolved from land
animals about fifty million years ago. He describes
some of the sivldence indicating that cetaceans
evolved from mammals. However, he points out
that they have no body hair, unlike other mammals.
They do not have external fl.ill, either.

..::

Summary: The professor presents the advantages


and disadvantages of bio fuels, which are fuels that
are derived from any sort of biological material,
such as corn. Some of the advantages include less
gas emissions and reduced pollution. On the other
hand, they still contribute to global warming and
use up agricultural lallii.
03 Office Hours

IHIEl~lMji

Parts of the Exam

01 Literature

Multiple Choice

Essay Section

20
Basic
Should
questions concepts be

Two essays

easy

Realism and Henry James


Theory

Henry James was one of the


first authors to create truly
realistic characters
'i:--

":~1

. ?1;)~

Support
James used unreliable
narrators who told lies like
real people do
James used a style called
stream of consciousness to
capture thought processes of
characters

1. {C)

2. (C)

3.

(D)

Summary: The professor discusses Henrv James


and his creation of realistic characters. James used
something called an unreliable narrator to make his
characters seem more like real humans. He also
used a method called stream of consciousness,
which. resembles how people actually lhi.oJs.

1.

2.

(C)

Summary: The student has missed a class where


the professor went over material for an exam. He
approaches the professor in order to get a better
idea of what to expect on the exam. The professor
explains that the exam has a simple multiple choice
section and an in-depth ~section.

04 Service Encounter
Solulion

Problem

>-------"-------!

Student needs a textbook.


but it is-too late in semester

-----------------~

-.-.

Bio Fuels

---------!
Disadvantages

Contribute to g-lo_b.::.al_w_a-rm-in_g_

--------t~~~~~
se !and for agriculture

Beller for the environment


than fossil fuels

-----------

1.

(A)

Student can make use-- of


library to re8d material in

book
/student can purchase book
lfrom online bookseller

-------~--~--..i.-......-----------'

--------Advantages
Rene~able

3. (El)

(D)

and bookstore does not


have it

U2 Agriculture

Require deep
understanding
of concepts

2.

(A)

-------'-----'
3.

(A)

1. (D)

2 (A)

3.

(D)

Summary: The student attempts to find a textbook


at the university bookstore. An employee tells her
that textbooks are no longer in stock this late into the
semester. The student then asks him to recommend
some filillfill that would carry the book and is
advised to visit the librarl. Another option for the
student is to make a purchase online.

Answer Key

753 - . , ,

---------------

------

05 Environmental Science

07 Office Hours

Problem (Cause)

Effect

Introduction of new parasites


and diseases could lead to
Farmed salmon are escaping extinction of local salmon
to the ocean and becoming
Increased competition for
invasive
natural resources may cause
serious problems

1. (C)

2.

(C)

Solution

Problem

Invasive Species

3.

(A)

Summary: The professor discusses invasive


species, which he defines as species of animals
that filllfil environments they are not native to. The
professor uses farmed salmon as an example of the
effects that an invasive species can cause. First,
these farmed fish introduce diseases that were not
found in the ecosystem before they came. Second,
the farmed fish increase competition for food and
other resources.

Go on medical leave and take class


at another tim_e

Student is ill and


concerned grades
will suffer

1.

(B)

Putplass on hold and finish when he


is better

2.

(C)

(D)

Summary: The student has been ill and has


missed a lot of classes, so he is worried that his
grades will suffer. The professor suggests medical
!filM!, but that will mean the student will lose his
scholarship. The professor decides to give him an
"O.H.," which means "On Hold" at the end of the
semester. This will allow him to finish his coursework
at some time in the future.

08 Service Encounter
Solution

Problem

--

Participate in Residential Life Office's


lottery system

Student wants to
change rooms nex_t
semester

06 History

3.

--

Move into own apartment off-campus

The Gettysburg Address


Theory

Support

Speech was so
effective because it
touched the
audience emotionally

Reminded ~eople about the


importance of equality and freedom
Used the word "nation" to bring the
country together again
Contained no new ideas, only familiar
ones

1.

(A)

2.

(D)

3.

(B)

2.

(C)

3.

(A)

Summary: The student plans to move and asks


about his options. He states that he would like to live
off-campus. The employee explains how the process
works and provides afQ[m for the student to fill out.
The student decides to~ for off-campus housing,
but he will return the form later.

(C)

Summary: The professor discusses why the


Gettysburg Address, a speech by Abraham Lincoln,
is so memorable. She says that it's largely because
it appealed to people's emotions. Lincoln reminded
people that they fought to secure freedom. He also
said that the country would one day be reunited.
Finally, he did not bring in any new concepts.

!'!t--- 754 AnswerKey

1.

09 Biology

l__

Scent Communication in Animal_s,_ _ _ _


Cats use body scents
Ants use pheromones

To show
affection

To mark
territory, objects,
or people

To look for a
mate

To call for
help

---''--'--------'----'----1. (B)

2.

(D)

3.

(A)

Summary: The professor talks about how animals


can communicate using smells. The professor first
uses the example of Ql and states that these animals
will rub against things in order to show affection or
to !lli!rk territory. The professor also talks about fill!
and says that they release chemicals to transmit
dillerent messages.

10 Environmental Science

I Mini Test 2 I
1. (A)
4. (8)

2. (A)
5. (A)

3. (C)
6. (D)

7. (C)
10. (A)

8. (8)
11. (A)

12. (8)

13. (A)
16. (A)

14. (8)
17. (A)

15. (8)
18. (C)

9. (C)

Why Oceans are Salty


Causes
Hydrothermal vents:

Rivers:

1. Vents superheat water


2. Hot water dissolves salts
3. Salt goes back into ocean

1. Rain water dissolves salt


2. Collects in rivers
3. Rivers dump slightly salty
water into ocean

Chapter 5 J~~
01 Linguistics
The Creation of Words

Effect

1. (C)

2. (A)

3.

(D)

.,.,,'('

..

Explanation

Classification

Ocean is salty

Loanwords

Words borrowed from other


languages

Compounding and Derivation

Combination of words or
parts of words

Onomatopoeia or imitation

Words created to imitate


sounds

<f

Summary: The professor explains why the oceans


are~- The first reason is related to the hydrological
process. Rain falls on land and dissolves mineral
fil!!ill. which accumulate in rivers and eventually
flow into the ocean. Hydrothermal vents are another
reason why the oceans are salty. Very hot water on
the ocean floor di.~ mineral salts found there.
However, the professor notes that the oceans are
not becoming fillllifil over time.

I Vocabulary Review 2 I
1.
4.
7.
10.
13.

(C)
(A)
(D)
(C)
(8)

2. (D)
5. (A)
8. (D)
11. (C)
14. {A)

3. {D)
6. (D)
9. (8)

12. (C)
15. (8)

22. collide
25. expanso

2. (C)

3. (8)

Summary: The professor describes ways that new


words are filklfili to a language. One common way
of creating new words is througll loanwords. These aie
words that are directly taken from another language
without translation. Another way that new words are
added is through compounding or derivation. A
third way that words are created is when speakers
try to imitate sounds.

02 Biology
. ~--liiood Production and Degradation

16. administered
17. territory
18. descended from
19. inevitably
20. maintenance
21. exploit
24. title

1. (C)

Step 1

Step 2

New blood cells produced rn


bone marrow

1.

(B)

2.

(C)

Blood 1s degraded 1n spleen


and liver

3.

(A)

23. surplus

-1

Answer Key

755

Summary: The lecture expands on a previous talk


about the quality and function of blood. Hematopoiesis
is the process of creating red blood cells in the
human body. New red blood cells are created in
red bone marrow, which is the soft tissue found
inside lillllfili. Blood cells usually degrade in the
~or the liver.

03 Zoology

OS Economics
Flaws of Marxism
Support

Argument
Marxism had limited theory
about origins of conflict

Academics believe other


factors such as race are also
responsible for social conflict

Marxism incorrectly
assumed that value
equals workers' effort plus
value of machinery

Marx failed to consider


value of management, sales,
accounting, strategic
planning, etc.

Hunting Dogs vs. Falcons


Hunling Dogs
Hunt to please
owners

1. {C)

Both
Have hunting
relationship with
humans

2.

(A)

1. (B)

Falcons
No bond wilh
humans, only
hunt for food

3.

{A)

Summary: The professor describes the sport of


falconry. He compares the bond between falcons
and humans to that of dogs and humans. Both
relationships involve hunting, but falcons only hunt
because they are hungry, while dogs will hunt in
order to please humans.

2.

(B)

3. (A)

Summary: The discussion is about why Marxism


failed. The first point made by the professor was
that this economic theory did not include other factors
that divide a society, like rag)_. The professor continues
by saying that another key premise of the theory
was completely wrong, Marx said that the value of
a company equals workers, effort plus the value of
the machinery to make the products. Marx failed to
account for the added value of management. sales,
filQ.

06 Art History

04 Psychology
lmpressionisfn vs. Post-Impressionism

Yerkes and Dodson


Experiment
A mouse had to choose the
correct door in a maze. If the
mouse made the wrong
choice, it got an electric
shock.

Resulls
easy task -+ high motivation
best
medium.task-+ medium
motivation best
hard task -+ tow motivation
best

--

{A)

2.

(C)

3.

Painted
landscapes
Fast
brushstrokes

Post-Impressionism

Vivid colors

Painted humans

Expressive
brushstrokes

Swirling patterns,
color dots
Focus On
emotion

Focus on lighting

~--

1.

{B)

2.

(A)

3.

(B)

(A)

Summary: The professor discusses an experiment


that supports the theory that too much motivation can
hurt one's performance. The experiment involved
mice that were given different levels of shock when
they made a mistake trying to choose a door. The
study found that when the. mice did more difficult
tasks, their performance was harmed by too much
motivation.

756 Answer Key

Both

Impressionism

Summary: The professor clarifies the differences


between Impressionism and Post-Impressionism. The
professor first says that thev both had bright color and
thick applications of paint. However, the Impressionists
painted landscapes and focused on fast brushstrokes
and lighting, while the Post-Impressionists painted
llliQJ2ill and focused on more careful brushstrokes
and more emotion.

07 Business

09 Psychology
Owner Spokespeople

Advantages

Nature vs. Nurture

Disadvantages

Less expensive

Some owners aren't good at


delivery

Gives sense of authority to


the message

Owner's personality might


alienate customers

(A)

2.

(C)

3.

Nature
Both
Innate personality Try to describe
traits from birth
how we develop
personalilies
Twin studies to
support

Nurture
Humans acquire
personality and
intelligence
through experience
'

B.F. Skinner's work


on conditioning

(B)

Summary: The talk is about the advantages and


disadvantages of an owner spokesperson in
advertising. The professor gives two pros and two
cons. As for advantages, the professor says that it
helps to save money. and it can make people
believe the message. However, some people are
not good spokespeople, either because they are
too awkward in front of the camera or their personality
sends the wrong message about the company.
" ,,

2. (A)

1. (B)

3.

(B)

Summary: The professor explains the nature versus


nurture debate. She clarifies what the debate is about
and discusses twin studies that suggest personality
traits are intrinsic, passed through the genes. Next,
she talks about tabula rasa, the idea that humans
are born as blank slates. She mentions B.F.
. Skinner's work on conditioning to support that people
are more affected by their enyjronmentor experiences.

~:;'

08 Zoolo'S)y

'

10 Art History
MerTiory in Animals

Experiment

Result

Wasp remembered Wasp flew into

location of nest in
relation to circle of
pine cones it saw
only once

circle seeming to
"remember" nest
was inside it

Cubism vs. Surrealism

Conclusion
Some aniinals
seem to have
some kind of
memory

Cubism
Used geometric
shapes

1. (B)

1. (C)

2.

(B)

Surrealism

Both
Distorted reality

2.

(A)

Mixed objects
that did not
belong together

3.

(C)

3. (D)

Summary: The discussion revolves around the


professor's statement that animals have more
developed memories than !2e.QQLe think. She supports
this idea with the example of a 'i'l.llliQ.. Researchers
used a circle of pine con.ill> to test the memory of
the insect. The insect remembered the location of
her nest in relation to the pine cones. The experiment
indicates that even smail insects have some kind of
memory.

Summary: The class compares the artistic aspects


of two art movements: cubism and surrealism. In
the discussion, it is stated that they both altered
reality, though they did this in different ways.
Cubism focused on changing the shape of the
subject and distorting it using cubes. On the other
hand, surrealists distorted reality by mixing subjects
that would usually not appear together.

Answer Key 757

' Chapter, 6'

Summary: The professor talks about how certain


things can represent greater ideas for people, and
he uses the example of the Bastille. Originally it was
a place that held prisoners. It was a sign of !be.
king's power and oppression. However, after an
event where the people freed the inmates of the
Bastille, it came to represent the French Revolution
and freedom.

01 Biology
The Sun and Your Health
Theory

Support
Sun provides vitamin D, which
helps us absorb calcium

Exposure to some sunlight


is good for you
Exposure to sunlight can help
make people happy

03 Office Hours
Problem

1.
YES
The sun is a better source of
Vitamin D than food is.

Student needs help


preparing for
upcoming test

NO

,/

The human body requires two


hours of sunlight every day.

Make use of online test bank


Make use of online tutorials

,/

1.

The sun can positively affect


mental health.

,/

YES

. Summary: The professor discusses the benefits of


sunlight to humans. He says that it provides Yilfilrlin
.Q, which helps our bodies absorb calcium. The
professor also says that exposure to the sun can
have benejjIB for mental health, helping to make
people feel happy.

Symbolism of the Bastille

Key Points

Support

Prison that held rebels

Symbol of oppression

People revolted, sparked


French Revolution

Symbol of freedom

The student is worried about the


next test.

,/

The professor posts tests


online.

,/

NO

The professor provides online


tutorials.

,/

Summary: The student visits the professor to talk


about the upcoming test. The professor recommends
two ways that the student can prepare. First, the
professor keeps copies of old tests online for students
to review. Second, the professor says that there are
illl!ioo tutorials that can help the student as well.
The professor also mentions that reading the textbook
is important.

02 History

Support

04 Service Encounter
1.

--~~-------Bastine

YES

e events at tile
were
used by tho .revolution.

Ll_~

r:~
-[Th

e Bastille has remained a


mbol of political oppression.

e Bastille was a prison that


jw as attacked;

' ---

---- ---

,/
,/

_/
I

---------'<"""""':-:--.--<::->-----~---'-'-,-____:__:.'-'~

,/

'

758 Answer Key

:":;-T,~~,--,,.,

J_

NO

~_.:__::_

Steps in the_H_iri_ng_P_r_oc..,e_s_s_ _ _ _.--1

1. Have official
2. Attend
3. Go through
Interview with
orientation
training period
manager from
session
after starting
Food Services
work
-----"----~-~-----

06 History

1.
YES
IS

The student is looking for a job


that begins immediately.

ta .
1n
1e

~;,

in

NO
Confucianism

./

The student will work in the


bakeshop.

./

The student hopes to begin


working as soon as possible.

Confucianism in northeast
Asian countries

./

Summary: A student wants to sign up for an oncampus job, so she visits the student employment
office. The employee tells her about the bidng process.
First, she must go through an official interview.
Then she must go to an orientation session. Last,
there is a training period that occurs after she
begins work.
,__
~--

Effect

Topic

05 Environmental Science

1. (C) .

2.

Shaped ideals of family and


social organization
Can lead to liarmony in
workplace

(B)

3.

(A)

Summary: The professor discusses the effects of


Confucianism on northeastern Asia. In particular,
the professor mentions how Confucianism affected
the culture of J.ai;IBn. First of all, its influence can be
seen in how people rel<jte to each other. It emphasizes
living in harmony with others by knowing one's
place in society. The professor then discusses how
. Confucianism applies to businesses.

How Mountains Grow

Cause

. 'j

Effect

Subductio_n;;f; one continental plate


slides under' another
Global war~'jng - glaciers melt, relieve
pressure

Mountains get

Problem

taller over time

Student
wants to take
summer
courses.but
will beat
home

s:

1.
YES

s
<

The Andes are so tall because


of earthquakes.

07 Office Hours

NO

Solution
Take online
courses

./

The process of global warming


is affecting the height of some
mountains.

./

1.

Summary: The professor talks about why some


mountain ranges are taller than others. He says that
this is sometimes caused by Q!.alSl collisions. This
happens when two of the Earth's plates come
together. The professor also discusses gjQQfil
warming as a cause. Glaciers push down on the
Earth. But when they melt, the Earth smings back
lJll. and mountains grow taller.

(A)

Less
expensive
than regular
courses

No classroom Not
experience
necessary for
learning
experience_

./

Mounlains can be formed by


plate collisions.

Refutation

Concern

Too
expensive

2. (C)

3.

(D)

Summary: The student asks for advice from his


professor about summer courses. The professor
mentions ooljne courses. but the student is worried
that such classes are too expensive. The professor
says the student would actually save money and
graduate earlier. The student then worries that he
will miss the classroom experience, but the professor
says that certain aspects of these classes could be
helpful.

Answer Key

759 _

-----~

08 Service Encounter
Questions

Summary: The professor explains the weather


patterns on the surface of J..!JQilfil. She describes its
unique colors, which are caused by the rotational
.~ of the planet along with the process of
convection. These two things together cause
bands of colored gases to wrap around the planet.
The professor also discusses the Great Red Spot,
which is a hurricane maintained by the planet's fast
rotational speed.

Answers

How much student activity


fee is

$560 per semester

What student activity fee


is used for

University events like Spring


Weekend

Student services

The student wants to find out


how much the fee is and what it
is used for.

10 Biology

NO

YES
,/

Alleles
Role

Definition

The student activity fee doesn't


include campus events..

,/

The student activity fee is used


mostly for campus services.

,/

Sequences of

genes on a
specific
chromosome

Summary: The student asks about the student


activity fee. He wants to know how much it is and
what it .is used for. He is told that it helps pay for
both university events, which are arranged by student
organizations, and is also used for various services.
A few services mentioned by the woman are the
weekly newspaper, computer and library facilities,
and free on-campus transportation.

Types

Determine the
appearance of
specific tr8.its

Dominant - only
need one allele for
trait to appear

Recessive - need
two identical alleles
for trait ta appear

1. ,
YES
Alleles determine the appearance
of a character trait.

,/

,/

09 Astronomy
Weather Patterns on Jupiter

Classification

Bands of color

Description

---

j Convection creates color

fbands, rotational speed

wraps them around planet

NO

,/

Alleles are made up of


chromosomes.
There are four different types of
alleles.

~'

Summary: The professor discusses alleles, which


are a pair of ~ that are located on a specific
chromosome. The professor talks about their role,
which is to determine specific traits. The professor
then discusses how this occurs by defining dominant
and recessive alleles.

300-year-old hurricane
caused by rotational speed
and lack of land

Great Re.d Spot

1---~-----+-A_tmoseheric Bands
Similar in n_ature
to a hurricane
1

Caused by

I convection

i Dark and light


ired color

01 Environmental Science

Great Red Spot

r==-----

,/

[!0~ne~n

,/

.
,/

I Marg._in_a_I______ ]

!Water circulates as a result otfwater circulates as a result of !


j changes i.n water temperature Iocean currents

and density
1

__J

li-mitS'd---1--Curren. t s - ] Part of o~ean

l exchange
f ater with
ns
760 Answer Key

Types of Seas

te~p~rature

-Currents"
l
with some
caused by
land around . ocean winds

variation

_ _ _ __J__ _____,

caused by
------

04 Service Encounter

her
; its

1. (B)

Ila!

Summary: The professor discusses the different


types of seas. The first type is called a medjterranean
sea. The water in this type of sea circulates through
changes in temperature and water density. The
second type is called a marginal sea, which is nearly
enclosed by~ or some other kind of land. The
waters within this kind of sea are circulated by currents
caused by wind.

Of
Jse
1et.
IOI,

ast

2.

3.

(B)

(D)

02 Biology
Respiration Underwater vs. Respiration on Land
Underwater

Both

Land

Requires gills to Exchange of


take oxygen from carbon dioxide
and oxygen
water

r
'

:.

~h

1. (B)
u

2. (B)

Requires organs
such as lungs

3.

(A)

~ummary: The professor discusses two types of


respiration: underwater and on land. He explains
:ttle need for different respiration QLQfilIB for animals
that live underwater. Gills are adapted to extract
oxygen from water. For animals that live on land,
there are different ways of breathing. Some animals
can breathe just by having their cells exposed to
the air. Some animals require special organs to
breathe, lil1e !.u.ng,,

fie

Ill.
or
!O!

03 Office Hours

r-

Problem
Student is always late to
discussion section

Switch to another
discussion ser::tion at a
I better time

~--~~ .. J..:: __ ~-------------

i--

1. (A)

---1-- -=-~---=--=

2.

(C)

3.

Problem

Solution

Student was accidentally


assigned to room in
all-girl's dormitory

1. (B)

Housing Coordinator helps


hlm change rooms

2. (A)

3.

(C)

Summary: The conversation takes place between


a housing employee and a student. The student
has accidentally been assigned to a dorm that
houses only girls, and he needs to change dorms.
He says the room is very nice, but he obviously
cannot stay there. The employee provides him with
a form that will allow him to move to a different
dorm. She also tells him that he can leave part of
the form blank.

OS Business
The Bricks and Clicks Business Model

"~'

Definition

Aspects
Has established supply and
distribution networks

Busine~s model that useS


both online and offline
components

1. (B)

2.

Has stability, which helps


attract customers

(D)

3.

(A)

Summary: The discussion is about the bricks and


Clicks business model. It is a model that includes
both offline and QIJ)ioo elements. The professor
explains a couple of advantages for this business
model. First, he says that it allows businesses more
freedom in how they operate. He also says the system
is stable because established businesses already
have fil1Qll.!)! and distribution chains set up.

(B)

06 Anatomy

]
I

-~

)f ':

--i

Summary: The conversation takes place between


a professor and a studeot She 1s approaching the
professor to change the time of her discussion section.
The professor asks her why, and she explains that
it is a problem with her schedule caused by another
professor. She has troub1e gettLog to her group on
time. The professor agrees. to change her to a different
group.

The lntegumentary System


Definition

I -'-'-------Functions
)"Protects internal organs

; Body covering such as skin,

! Helps regulate bo~dy


.

I hair, and nails

jftem~era~_re------~----

L_________, Provides sense of touch

Answer Key

761

1.

(A)

2.

(A)

3.

(C)

Summary: The professor discusses the integumentary


system, which comprises the body's external
coverings, such as skin, hair, and nails. First, the
integumentary system provides protection for our
internal organs. Second, it helps us to regulate our
body temperature. Last, it acts as a receptor for
pain and temperature changes.

Problem
Student wants to expand a
previous paper rather than
write a new one

Solution
Professor agrees, as long as
student follows the same
rules as everyone else

(A)

3.

(D)

Summary: A student approaches her professor to


discuss the paper topics he assigned. The student
wants to expand on a previous paper that she wrote
rather than choose from the new topics the professor
presented. The professor ~ to let her do this
because the student plans to submit the paper to
the department's magazine.

r-~ ~""....
Doctor's visits

~>C-00 ~-"'

X-rays

Lab tests
s

Dental work

--------------~

(A)

2. (D)

3.

(A)

Summary: The conversation takes place between


a student and an employee at health services. The
student approaches the employee with questions
about the university insurance plan. The employee
explains the cost as well as some of the services
that are covered and not covered in the plan. The
student also finds out that dental insurance is not
covered by the universiiy plan at all.

762

Answer Key

Example
Had better ways of defending Knew how to work metal to
themselves than other tribes create weapons and armor
Outnumbered and defealed
Aztec army in numerous
battles

Had well-organized military

(A)

3.

(B)

Summary: The Purepecha are a tribe living today


in Central America. The professor presents two
theories about why the Purepecha peoples were
able to effectively @fil Aztec and Spanish forces.
First, she argues that the advanced Purepecha
knowledge of working metal helped them produce
weapons and armor. She also proposes that the
Purepecha army was relatively 1filg_e and wellorganized.

10 Ecology
Parasitism vs. Mutualism
Parasitism
One species
benefits, other is
harmed

08 Service Encounter

1.

The Success of the Purepecha


' Point

1. (B)

07 Office Hours

1. (B)

09 History

1. (C)

Both

Mulualism
Both species

Two species
coexist

2.

(A)

benefit

3.

(B)

Summary: The discussion begins with a definition


and example of parasitism and ends with a definition
and example of mutualism. In the first type of
relationship, one animal ljves off of another animal,
and by doing so, harms the other animal. By contrast,
in the second relationship, two species are able to
help each other by either providing protection or
nutrition.

I Vocabulary Review 3 I
. 1.:

1. (0)
4. (A)
7. (C)

2. (D)
5. (0)
8. (A)

03 Service Encounter
11. (0)
13.

3. (B)
6. (C)

12. (B)
YES

9. (B)
12 (A)
15. (B)

10. (B)
13. (C)

16. comprises
19: enrich

17. complaint 18. feasible


20. deduce

21. (B)
24. (E)

22. (A)
25. (C)

11. (C)
14. (A)

23. (0)

Receive course approval from the head of the


deparlment

.r

Submit the credit request form by the


deadline

.r

NO

Contact her professors about the courses she


plans to take

.r

Send her professors copies of her final


exam

.r

14. (A)

15. (B)

I Mini Test 3 I

!,

r:

Practice Test

01 Computer Science
1. (C)
4. .!

2. (B)

3.

(B)

01 History

>

Local area network Wide area network

.r

Private organization

1. (C)
4. (A)

2. (0)
5. (B)

3. (B)
6. (B)

8. (0)
11. (B)

9. (B)

Home office

.r

Internet service provider

02 Chemistry

.r

Small office

7. (A)
10. (B)
12.

Dormitory

5.

(B)

YES.

A stable cluster in solution forms the nucleus


of the crystal.

02 Literature
6.

(A)

7.

(A)

A supersaturated solution eliminates


growth potential.

8. (0)

9.

I--'------------------The names of some characters in The


Sandman refer to lhe eyes.

I! YES./

A crystal structure is formed by


connected atoms.

NO

The main character, Nathanial, became blind


as a young boy.

One of the story's !hemes is obs~;;;~-.---~ -7~/


4

Thc Sandman tr;di.tionally

ha~-;;-~;ydi~rent

1-,.-r-,

meaning from that 1n l~"._::~tory~--------t-+-J


The professor discusses vision and op1ical
i

illusions.

i ./

.r
.r

-----.

.r

---

---~--

.r

.r

--

The solution becomes supersaturated and


the crystal stops growing.

1------------------f---{-

NO

f--'-

03 Service Encounter
13. (B)
16. (C)

14. (0)

15. (B)

17. (A)

J!

~----------------------

10. (C)
Answer Key

763

~. . .

04 Psychology
18. (A)
21. (C)

19. (B)
22. (A)

20. (B}
23. (A)

,;:.,:

i.

OS Physics
24. (A)
27. (A)

25. (B)

26. (B)

28. (C)

29.
Fixed base
pulley
Has an axis that
is not stationary

Movable base
pulley
,/

Has the most


potential
for mechanical
advantage

,/

The most basic


kind of pulley

,/

06 Office Hours
30. (C)
33. (8)

764 Answer Key

Compound
pulley

31. (A)
34. (A}

32. (A)

;>+',e'. m;,,~;,.

' . u

f
~.

~.

Part 1

Step 1

Chapter1

r~,.

r
r
l;.

'.

Q2 Practice 3

Q1 Practice 1
Step 1

Related Ideas and Expressions:

chat, conversation, argument


intriguing, attention-grabbing
prepared, in order, structured
key, major, essential, fundamental

Related Ideas and Expressions:

conscientious, passion, commitment, devotion


brilliant, trained, experienced
inspire, excite
benevolent, unselfish, helpful, nice

Q3 Practice 1
Q1 Practice 2

Step 1

Step 1

Suggested Keywords/Key Phrases:

Related Ideas and Expressions:

basketball tickets, reduced price, profit, improve


facilities, poor condition

approve of, think a lot of, regard highly, value


heroic, bold, valiant
support, help out, provide assistance
income, money, payment, wage

Q1 Practice 3

Sample Restatement:

The university is not going to offer basketball tickets at


a reduced price anymore. This is because it has not
made a profit recently. The facilities also need to be
improved because they are in poor condition.

Step 1

'

I'

Related Ideas and Expressions:

~Step

calm, content, happy, at peace


answer, trouble, dilemma, challenge
skill, talent, data
comparable, almost the same, close to

raising ticket prices, full ticket price, arena, outdated,


leaks

Suggested Keywords/Key Phrases:

Sample Restatement:

Q2 Practice 1
Step 1
Related Ideas and Expressions:

cash, wealth, riches, charge, pay


agonize over, be stressed about, be upset about
commit to, consider, contemplate
instruction, academic, tutor

02 Practice 2
~Step

Related Ideas and Expressions:

open air, count1yside, landscape


walking, rock climbing, skiing, swimming
stuffed up, wate1y eyes
medicine, healthy, feeling good

The man thinks that raising ticl\et prices is good. He


says that most people do not pay the full ticket price
since most people at the games are students. He also
says that arena is very outdated. He says he has
heard that the roof leaks.
~Step

Opinion:

The man thinks it is good that the university will no


longer sell reduced-price tickets to students.
Reason 1:

The university is not making money.


Detail:

Most of the people at basketball games are students.


Reason 2:

The university needs money to improve the facilities.


Detail:

The university is old, ugly, and the roof leaks.


Answer Key

,..

765 - . , :

------------------------------------itf!!(~

Q3 Practice 2

Sample Restatement:

Step 1

The announcement says that a new class is going to


be offered in the English department. This is supposed
to meet the needs of working students who need to
take classes in the evening. Also, the university wants
to expand its literature class offerings to meet the
needs of the growing student base.

Suggested Keywords/Key Phrases:

canceled, Music Theory, additional section,


unable to find a professor, not enough space
Sample Restatement:

The university says that it has canceled a section of


music theory. It was supposed to be an additional
section. The announcement says it is because they
were unable to find a professor. Also, there was not
enough space in the music building.
Step 2
Suggested Keywords/Key Phrases:

hire. another professor, growing, another building, do


not need instruments, head of the music department
Sample Restatement:

The man says he does not agree with the


announcement. He says the university should hire
another professor since the music program has been
growing lately. He also says that they could have
the class in another building since you do not need
instruments for the class. The speakers decide to talk
to the head of the music department.
~Step

Opinion:

The university should not cancel the music theory


class.
Reason 1:

The university should hire more professors to attract


students.
Detail:

The music department has been growing quickly.


Reason 2:

Step 2
Suggested KeYwords/Key Phrases:

night classes, working students, packed, majors,


survey class
Sample Restatement:

The woman is happy with the announcement. She


. says the university needs more night classes
for students who work during the day. When she had
a night class, it was packed. Also, she is glad the
university is offering a class that is not just for majors.
Many of these classes are too advanced, and a
survey class is better for her needs.
Step 3
Opinion:

The woman thinks it is a good idea for the university


to expand the English department's offerings.
Reason 1:
More working students are coming to the university.
Detail:

The woman's night class was packed with working


students.
Reason2:

The university needs to offer more basic literature


classes.
Detail:

The woman struggled to find a class that was not too


advanced.

Another building can be used to hold the class.


Detail:

The class can be held in any building because it does


not require instruments.

Q4 Practice 1
~Step 1
Suggested Keywords/Key Phrases:

"'~
.M

'.:3 Practice 3

i~i1

~Step 1

Sample Restatement:

If~

~~;~:::~~:iy::~~~~~:~~~~:::~~ the offerings,

Consumers classify products by price and appearance.


Expensive products are often luxuries, while inexpensive
products sometimes become necessities. Also, a more

l.~J~i

!~Ii

growing student base

f:, - - 766 Answer Key

price, appearance, luxury, necessity, decoration

(-

attractive product is more likely to be classified as


decoration.
~Step

Suggested Keywords/Key Phrases:


television, luxury, . lower price, more appealing,
decoration
Sample Restatement:
The television shows how consumers classify products
by price and appearance. The television was first
considered a luxury because it was so expensive. But
the lower price made it more of a necessity. Also, the
first 1Vs were very big and ugly. But now that they are
more appealing, more people use them for decoration.
~Step

Classify products by price and appearance


Example: television
First way: price: luxury or necessity
Example:.'

,:.,

;:
,

television wis first an expensive luxury, then became


an inexpensjve necessity
Second way:
appearance: better appearance allows for decoration
Example:
television was plain and ugly at first, now they are
attractive and hung like paintings

Q4 Practice 2
~Step 1

Suggested Keywords/Key Phrases:


memory, echo, limited duration, exact copy, accurate
reproduction
Sample Restatement:
Echoic memory is a type of memory. With it, you hear
an echo of sound for a few seconds. It has a limited
duration. But it makes an exact copy of what you hear, so
it is a very accurate reproduction of what was heard.
~Step

Suggested Keywords/Key Phrases:


telephone number, forget, lasts a few seconds,
perfect accuracy, hearing echo

Sample Restatement:
When you hear a phone number for the first time, you
remember it in echoic memory. Because echoic
memory is brief, you will forget the number quickly.
That is because echoic memory only lasts a few seconds.
Also, because echoic memory is accurate, you can .
remember the number for a few seconds with perfect
accuracy. That is because you can hear an echo of
the number in your memory.
~Step

Echoic memory is when information you just heard


stays in your mind for a few seconds.
Example: hearing a new telephone number
First feature: very short
Example:
you will forget the number after a few seconds
Second feature: accurate
Example:
you can dial it accurately if you do so immediately

Q4 Practice 3
~Step

Suggested Keywords/Key Phrases:


unconscious competence, without thinking, second
nature, doing other things, automatic
Sample Restatement:
Unconscious competence is when you can do
something naturally. One feature is that you can do it
without thinking about it; it becomes second nature.
In addition, you can do it while doing other things.
That is because it becomes automatic.
~Step 1

Suggested Keywords/Key Phrases:


learning new instrL1ment, play without thinking,
looking at your fingers, do other things simultaneously,
sing while playing
Sample Restatement:
The process of learning a new instrument demonstrates
unconscious competence. \IV~.en you learn the guitar, it
1s hard at first. but aitsr 8 wl1ile you can play without
thinking. You can swn plEy w1ti~cut locking at your lingers.
In addition, you can do other things simultaneously.
For example, you can sing while playing.
Answer Key 767 -

~Step

Unconscious competence is doing something


automatically.
Example: learning to play guitar
Feature 1: can do it without having to concentrate
Example: do not have to watch fingers any more
Feature 2: can do other things
Example: singing and playing at the same time

QS Practice 2
~Step

Problem:
The man does not know how to get teaching experience.
Solution 1:
Ask a professor to sponsor an independent project
Solution 2:
Become a teaching assistant in grad school
~Step

Opinion 1

QS Practice 1
~Step

Problem:
The literary magazine is running out of money.
Solution 1: Look for more advertisers
Solution 2: Let some cit the writers go
~Step

Opinion 1
Problem:
The literary magazine is running out of money.
Best Solution:
She should look for more advertisers.
Reason 1:
It is the fastest way to find more money.
Reason 2:
There is no guarantee that letting writers go will help.
Opinion 2
Problem:
The litera1y magazine is running out of money.
Best Solution:
She should let some of ihe writers go.
Reason 1:
It is the easiest way to cut costs.
Reason 2:
They might not be able to find more advertisers.

768

Problem:
The man does not know how to get teaching experience.
Best Solution:
Ask a professor to sponsor an independent project
Reason 1:
He will get experience right away.
Reason 2:
It will give him a chance to change his mind.

QS Practice 3

Problem:
The man wants to attend a conference but is scheduled
to go to a team debate.
Solution 1:
Go to the team debate and forget the conference
Solution 2:
Go to the conference and disappoint his team

Answer Key

... -'""-'"

i.I
i

Ii
'

Problem:
The man does not know how to get teaching experience.
Best Solution:
Become a teaching assistant in grad school
Reason 1:
He is too busy to do a project now.
Reason 2:
He will get better experience in grad school.

---"":;.:.,

!
"

Opinion 2

~Step

~-

,,_,

.. ,_.

; Step 2
Opinion 1
Problem:
The man wants to attend a conference but is scheduled
to go to a team debate.
Best Solution:
Go to the team debate and forget the conference
Reason 1: His team is counting on him.
Reason 2:
He can go to other conferences when he is available.
Opinion 2
Problem:
The man wants to attend a conference but is scheduled
to go to a team debate.
Best Solution: Go to the conference
Reason 1: He could learn a lot at the conference.
Rea~pn 2: His team members will understand.

Q6 Practice 3
; Step 1
Topic: Brand names
Thesis:
A popular brand name can help a product in two ways.
Key point 1:
A popular brand name can make a product more
appealing.
Keypoint2:
A popular brand name helps people remember a
product.

I Vocabulary Review 1 I
1. (C)
4. (A)
7. (C)

2. (A)
5. (C)
8. (B)

3. (0)
6. (B)

9. (A)
12. (C)
15. (0)

10. (0)
13. (C)

11. (A)
14. (A)

Q6 Practice 1
; Step 1

16. draw
19. brand

17. animation 18. eager


20. significant

Topic: How ancient stories are told


Thesis:
Because stories were not written down, they were
affected in two ways.
Key point 1:
Stories often changed when they were told aloud.
Key point 2:
Stories did not have only one author.

21. responsibility 22. upcoming 23. independent


24. reproduction 25. expand

Q6 Practice 2
~Step 1

Topic: Animation
Thesis:
There are two differences between traditional
animation and computer animation.
Key point 1:
They differ in how long it takes to do them.
Key point 2: They differ in reputation.

I Vocabulary Review 2 I
1. (0)
4. (0)
7. (A)

2. (B)
5. (C)
8. (B)

3. (A)

9. (B)
12. (A)
15. (C)

10. (A)
13. (C)

11. (0)
14. (A)

6. (C)

16. experience 17. latter


18. firefighter
19. competence 20. Conversely
21. (0)
24. (A)

22. (B)
25. (C)

23. (E)

Answer Key

769 -

'

Part 2

Q2 Practice 1
Step 1

Q1 Practice 1

Transition words and phrases:


if, However, and, Secondly, For one thing
Sentence order: (A) (F) (8) (E) (D) (C)

Step 1
Transition words and phrases:
This is because, once, also, because, however,
someday, since that day
Sentence order: (F) (A) (E) (8) (D) (C)
Step 2

1. The speaker wants to learn how to sail a boat.


2. The speaker went sailing on a boat with .her
grandfather.
3. The speaker wants to sail around the world
someday.
Q1 Practice 2
~Step 1

Transition words and phrases:


so, recently, someday, though, for one, because,
also
Sentence order: (C) (E) (8) (F) (D) (A)
~Step

1. The speaker wants to take a photography class.


2. The speaker received a camera for his birthday.
3. The speaker takes pictures of nature and animals.

Step 2

1. The speaker thinks it is good to give unannounced


tests.
2. The speaker says that he is encouraged to study
the material thoroughly because of unannounced
tests.
3. The speaker says that unannounced tests give
him a chance to improve his grade.

Q2 Practice 2
Step 1
Transition words and phrases:
That way, and, First of all, However, Also
Sentence order: (E) (B) (F) (C) (D) (A)
Step 2

1. The speaker would prefer to go to university


immediately.
2. The speaker says that they lose motivation when
they do not go to university right away.
3. The speaker suggests that you will enter into
your career more quickly if you go immediately to
university.

Q2 Practice 3
Q1 Practice 3
~Step

~Step 1

Possible transition words/phrases:


because, also, first, and, in addition, consequently

Possible transition words/phrases:


first of all, second of all, for example, basically,
in addition, besides that

Cl.1 Practice 4

Q2 Practice 4

~Step

~Step

Possible transition words/phrases:


moreover, because, also, in the first place, later, while

,,,,___ 770 Answer Key

Possible transition words/phrases:


because, but, therefore, undoubtedly, hence,
furthermore

Q3 Practice 1

Key point 2:
It will provide them with the knowledge necessary to
apply the information.

~Step 1

Opinion:
The television should not be removed from the student
union building.
Reason 1:
That area should not be used for studying.
Reason 2:
Many people do not want to go back to the dorms to
watch 1V.

Q3 Practice 2

'

~Step 1

opinion:
He does not agree with the decision to close the
dorms in the summer.
Reason 1:
w:
,. The univers1fy should have searched for supervisors.
,. Reason 2:
j_-) ' '

The universify did not advertise that the dorms were


open.

Q4 Practice 1
~Step 1
Main topic:
Reward systems encourage good behavior in children.
Key point 1:
Reward systems provide a positive outcome for both
parents and child.
Key point2:
Reward systems encourage children to continue to
behave well.

Q4 Practice 2
~Step 1

Main topic:
Teaching by demonstration is the best way to teach
how to use a computer.
l<ey point 1:
It will give students all the information they need to
use a computer.

QS Practice 1
~Step 1
Problem: The school dance has lost funding
Solution 1: Cancel dance right away
Possible benefit:
Lets people know early if dance must be canceled
Solution 2: Look for funding elsewhere
.Possible benefit: Do not have to disappoint e~eryone
~Step

Suggested Answers:
1. The school dance the woman was organizing lost
its funding.
2. The woman should look for another sponsor.
3. This way, she will not disappoint the people who
have already made costumes.

QS Practice 2
~Step 1

Problem:
The man has an interview across town but his car will
not start.
Solution 1: Take the bus
Possible benefit:
He knows he can get there using the bus.
Solution 2: Ask to borrow roommate's car
Possible benefit: He will not be late to the interview.
~Step

1. The man needs to go to an interview but his car

is not working.
2. The man should ask his roommate if he can borrow
his car.
3. He will not have to wait for the bus, and he can
get to his interview on time.

Answer Key

771 ----.

- ----~~--~

Q6 Practice 1
~Step 1

Main topic:
There are two types of camouflage that help animals
hide from predators.
Key point 1:

Animals camouflage themselves by changing to look


like the environment.
Example:
The Arctic fox's coat changes to white to match the
snow in winter.
Key point 2:
Animals disguise themselves to look like other things.
Example:
The walking stick looks like an ordinary twig.
~Step

1. Animals camouflage themselves in order to hide


from predators.
2. The Arctic fox's fur turns white so it can blend in
with the environment.
3. The walking stick is able to hide from predators
because it looks like a twig.

Q6 Practice 2

2. A role conflict happens because the father wants


to be a dad to the child but he also has to act as
a coach.
3. A doctor can experience a role conflict when he
becomes a patient. He wants to still be a doctor,
but he cannot because his role has been reversed.

I Vocabulary Review 3 I
1.
4.
7.
10.
13.

(C)
(B)
(D)
(B)
(D)

2.
5.
8.
11.
14.

(D)
(A)
(B)
(C)
(B)

3.
6.
9.
12.
15.

(A)
(B)
(C)
(A)
(B)

16.
19.
.22.
25.
28.

(C)
(D)
(B)
(C)
(D)

17.
20.
23.
26.
29.

(A)
(C)
(B)
(A)
(A)

18.
21.
24.
27.
30.

(A)
(A)
(D)
(B)
(B)

31.
34.
37.
39.
40.

33. distracting
utilize
. 32. refrain
suffer
35. frequently 36. social
excessive 38. lonely
keep them from
companions

41 . alternative
43. witness

42. demonstrate
44. demanding 45. surprising

46. (0)
49. (0)

47. (S)
50. (S)

~Step 1

Main topic: There are two kinds of role conflicts.


Key point 1:

Role conflict can happen when a person takes on two


incompatible roles.
Example:
A father might have to act as both a father and a
coach.
Key point 2:
Role conflicts can happen when someone has to take
on a new role suddenly.
Example:
A doctor might have a conftict if he suddenly becomes
a patient.
>Step 2

II
I

1--

1. Role conflicts can occur either when someone


has two incompatible roles or when someone is
suddenly given a new role.

48. (S)

I Vocabulary Review 4 I
1.
4.
7.
10.
13.

(C)
(A)
(B)
(A)
(D)

2.
5.
8.
11.
14.

(A)
(D)
(C)
(B)
(A)

3.
6.
9.
12.
15.

(B)
(A)
(D)
(C)
(B)

16.
19.
22.
25.
28.

(D)
(C)
(B)
(D)
(D)

17.
- 20.
23.
26.
29.

(A)
(C)
(C)
(B)
(A)

18.
21.
24.
27.
30.

(A)
(D)
(A)
(B).
(C)

31.
34.
37.
40.

adaptations 32. disguise 33. technique


literally
35. altering
36. blend in
wildlife
38. drastically 39. feature
outcome

772 Answer Key

- - - - -- - - -- - - - -- - - - - - - -- - - - - -- - - -- - - - -------- - - -

41. reward
44. excessive

42. diagnosis 43. complaint


45. hectic

46. (C)

47. (A)
50. (D)

49. (B)

48. (E)

'

2. The robber held up the convenience store.


3. Let's go check out the new restaurant in the
student union.
4. Can you help me? I am searching for a journal
on anthropology.
5. Do not point at her. That is rude.
6. People often say that I take after my father.

~:

t
.~'

Part 3

,.\

Sentence stress related to content words

'i

Stress related to parts of words

~Step 1

~Step

Suggested content words in bold:

1. a. method
2. a. economy
3. a. academy
4. a. luxury
5. a. drama
6.<~. recommend
7. 'i'i. capable
8. a. prefer
9. a. photograph
10. a. negotiate

I.
I
I

~Step

b. methodology
b. economic
b. academic
b. luxurious
b. dramatic
b. recommendation
b. capability
b. preference
b. photography
b. negotiation

1. Do you have a campus parking permit for your


bike?
2. I hope my professor can advise me on which
course to take.
3. She has to present her project to the class
tomorrow.
4. My friends and I are going to the protest downtown
this afternoon.
5. Did you hear that Jane and her band will record
an album this summer?
6. Unfortunately, my parents will not permit me to
go skiing this weekend.
7. The police have arrested a suspect in the campus
computer lab robbery.
8. In biology, we are studying how plants convert
sunlight into energy.

1. People classify products based on price and


appearance.

2. The university should advertise that the dorms


are open in the summer.
3. Echoic memory lasts for only a few seconds.

4. The magazine might have to shut down,


5. Giving rewards will teach children to behave well.
6. I was very proud of my brother when he was
accepted into medical school.
7. I feel comfortable talking to my sister about my
problems.
8. Students should work for a year before going to
university.
~Step

Sample clear words in bold:


One person that I really admire is my father. For
one, he has worked very hard in his life. He was very
poor as a child and did not get a good education, but
he worked very hard to educate himself and get a
good job. Second, he is a very nice person. He is
always giving money and assistance to other people.
Just last weekend, he helped a coworker remodel
his house.

Reduction d unstressed words


~Step

Stress on phrasal 'ferbs


>Step 1
1. The researchers found it out very recently.

Sample weakened and reduced words in bold:


1. The people who moved out to other cities were
safe, but those who were io th.e city were in danger.
Answer Key 773 - - ,

2. He is the one in my family who understands ill


dream.
3. The students cannot access this section but the

5. In the past, the television was a luxury.


6. In addition, they are beneficial to mankind.

teachers~.

4. The government asked him to stop campaigning


against~ policy.
5. J:b.m( wanted to create something new and
innovative.
6. For homework, you all should have read a bit
about wind power.
7. I know I look young, but I am a student at this
university.
8. Mathematics is an important aspect of university
studies.

~Step

Sample clear words in bold:


The man supports the decision by the university to
offer literature classes at night. For one, he thinks it
is good because it allows people who work to take
more classes. He says that there are many working
students who take classes at night. He also thinks
it is good because it will allow the university to offer
more types of literature classes. The speaker says
he wanted to take a basic literature class, and now
he can.

Intonation
~Step 1
1. I had lost an important assignment due to computer
. llli!blems.

2. That gave me the opportunity to learn about a


new Q!,!\ture.
3. I never went on a trip with those friends again.
4. Universities need money for comruiter labs.
5. Many people think that teaching by exrunple is
best.
6. They look more interesting and have more
character.
7. A reward system is a good way to teach children
to behave well.
~Step

1. That will tell us the date at which the house was


built.
2. Mom and Dad want me to attend.
3. They can cause all kinds of trouble.
4. I am sure I will land a good iob after graduation.

77 4

Answer Key

~step1

1. Although we had not finished I we decided to go


home.
2. When she stepped off the boat I she immediate~
ran to her car.
3. It was raining so hard all day I that they did not
leave the house.
4. If the alarm rings I put down your books I and
slowly leave the building.
5. The final test will be two hours long I and will
count for twenty-five percent of your final grade.
~Step

1. In my opinion, I students should not have to pay


to go to university.
2. To begin, I echoic memory is very short, I lasting
only three to four seconds.
3. Charities, I both local and international, I rely on the
generosity of individuals to help the less fortunate.
4. In summary, I then, I I think that teachers should
give unannounced tests in class.
5. While a strong brand narne can make a product
more appealing, I it can also help customers
remember a company or product.
6. Many people, I however, I feel that animation done
by hand is more artistic than computer animation.
7. It took five years, I but he finally achieved his goal.
8. lri the case of many oral traditions, I ii is impossible
to identify the stories' authors.

Question 1
Sample Response
Mr. Daniels was the most memorable teacher I have
ever had. First, I remember him because he was very
strict. You could not mess around in his English class.
He did not tolerate lazy students, either. Second, he is
memorable because he loved to teach. He was

always excited about the stories and poems that he


presented to us. He made everything more interesting
that way.

says that the manager jokes around with his employees


because he wants them to see him as a friend. This
is because he wants to create a specific impression in
the minds of those employees.

Question 2
Sample Response 1
I think that it is better to do research on the Internet
than use books. One reason is that the Internet is
faster. If I want to find information, I can do so quickly
using the Internet. It takes much longer to find that
sarne information in a book. Second, the Internet is
more up-to-date. Many books in the library are old.
.People update the Internet everyday, so the information
is fresh and new.

JO
1ly

ot
id

1y

ig

1e
d
;t
s

<'),:>-..

Sample Response 2
I think it is better to use the books for research than
to use the Internet. For one, I think books are more
accurate than the Internet. There are many people
that post false information on the Internet. But people
check facts that are in books. Second, I think it is
, eailier to find what you are looking for in books. That
is because it is easy to find a book you need using a
library catalog.

}.:

Question 3
Sample Response
The university announcec that it will update an exercise
facility because the exercise machines are old and it
. is not big enough for all the students who use it. The
man thinks this is a good idea because he goes there
and there is not a lot of good exercise equipment.
Sarne is even broken. Moreover, lie feels that it is too
crowded. He goes there and has to wait fifteen minutes
to use a machine. So the place needs to be expanded
to fit all the students who go there.

Question 4

e
y
i.

s
s

Sample Response
The lecturer uses an example of a computer store
manager to explain impression management. First,
the manager tries to act in a really professional way
because he thinks that customers expect it. This
supports the idea from ihe reading that people want
to control how others see them. Tl1e speaker also

Question 5
Sample Response 1
The woman wants to take a class that is full. She can
either wait for a student to drop or talk to the professor.
I believe that she should wait for a student to drop it.
For one, this is the easier solution. If she spoke to the
professor, she would have to make an appointment
and try to convince him to let her into the class.
However, this way, she can just check the class
online. Also, it is the most reliable solution: The
speakers say that there might not be enough seats
left in class. But if someone drops, then the woman
would definitely be able to get in.
Sample Response 2
The woman's problem is that she wants to take a
class but it is already full. So she either has to wait
for an opening when a student drops it or to talk to
the professor to get in. I think she should talk to the
professor. First, it is better because it can save her
time. If he says yes, then she is in the class and does
not have to wait for people to drop it. Second, it will
almost certainly get her into the class. The speakers say
that the professor will let her in as long as there are
seats left.

Question 6
Sample Response
The lecture talks about the keystone species as a
species in the environment that plays a very important
role. The professor shows how the wolf is a good
example. First, they affect animal populations. After
wolves were reintroduced to a park where they did
not exist for eighty years, the large populations of two
animals, deer and elk, went down to healthier levels.
They also affect the environment. The wolves' return
also improved the number of cottonwood trees
because it reduced the number of animals that ate
the young trees.

Answer Key

775

Practice' Test 2

Sample Response
I like to go to the beach to relax, I like relaxing at the
beach because I can listen to the waves, They are
very soothing, The sound takes my mind off of all the
things I have to do, I also like to relax at the beach
because I can lie in the sun, I can take naps or read
a book, I do not worry about anything when I am lying
in the warmth of the sun,

Sample Response
According to the professor, point of view refers to the
position of the viewer when he's looking at a work of
art. The professor says that Velazquez uses point of
view for two purposes. First, he paints the scene so
that the viewer sees it from inside the house but is not
too close to the people. That way, the viewer sees all
the action. He also uses point of view to show a certain
meaning. He uses a broad point of view to emphasize
the group instead of the emotions of one person.

Question 2

Question 5

Sample Response 1
I think it is better to live with a roommate at
university, I think this because having a roommate
gives you a chance to meet other students like
yourself. Your roommate could turn out to be a really
good friend, and he or she can introduce you to other
students. It is also better to have a roommate because
you can do chores together, That way, you can get
the cleaning done much faster and have more fun
doing it.

Sample Response 1
The man wants to live in the dorms the next semester,
but university policy says he has to move out after
freshman year. He could either talk to the campus
housing department or apply to be a resident
assistant. I think it would be better for him to talk with
someone in campus housing; That way, he might be
al)le to get a dorm room without having to be ari RA
Also, even if they do not let him stay in the dorms,
they may be able to help him find housing close to
campus.

Question 1

Sample Response 2
I think it is better to live alone at university, For one, I
think living alone prevents distractions, When you live
with a roommate, you might want to have fun with
him instead of study. When you live alone, you can
focus on studying. Also, I think living with a roommate
can be annoying. Sometimes you have to live with
someone who is loud or messy. You do not have to
worry about that if you live alone.

Question 3
Sample Response
The woman does not think it is a good idea for the
maintenance department to install more lights around
campus. She thinks that additional lights will not stop
people from vandalizing and stealing, She thinks that
having more lights will reveal things that thieves did
not know were there in the first place. She also thinks
that the campus already has enough lights for doing
things outdoors at night. Plus, she likes doing things,
like jogging, when it is somewhat dark outside.

776

Question 4

Answer Key

Sample Response 2
The man's problem is that he is forced to move out of
the dorms but he wants to stay. The speakers say
that he could either talk to campus housing or apply
to be a resident assistant. I think he should become a
resident assistant. For one, he will definitely get housing
this way, whereas he might not be allowed to stay in
the dorms if he asks to have the policy changed. In
addition, he will get free housing. They always pay
resident assistants, so he gets two good things out of
this solution.

Question 6
Sample Response
The professor says that there are two ways that
advertisers make everyday products look appealing.
One way they do this is to use exaggeration. The
professor uses the example of a fast food commercial,
in which two people buy food at a drive thru window
The food is so big it makes their car tip to one side.
Another way, according to the professor, is with
humor. The professor gives the example of a photo
editing ad, in which a bad family portrait is edited to
look more presentable. By using humor, people get a
good impression of the product.

-r

Part 1
01 Urban Planning
~Step 1

Main ide<i: There are three advantages of using


congestion pricing to control pollution and congestion.
Key point 1: It will make more people use public
transportation because they will not want to drive.
Key point 2: It will make commuting times faster for
everyone by taking cars off the roads.
Key point 3: It will benefit many people because it
only punishes people who pollute the air.
~Step

Main idea: Congestion pricing is not the best solution


for reducing congestion and pollution.
Key'pointc 1: It may not make pemle use public
transportation because many public transportation
systems are not good enough.
Key'poinf2: It will not result in faster commute times
because buses will be too crowded.
Key point 3: It will not benefit everybody because
many people do not have any other choice.
.~Step 3

Introduction:
Main topic: lscongestion pricing a good way to
reduce pollution and congestion?
Main idea of the reading passage: There are three
advantages to using congestion pricing.
Main idea of the lecture: Congestion pricing is not
the best solution.
Body:
Key point 1: What is the impact of congestion pricing
on public transportation?
Supporting information from the reading passage:
It will make more people use public transportation
because people will not want to drive.

Supporting information from the lecture: It may


not make more people use public transportation
because many public transportation systems are not
good enough.
Key point 2: What is the impact of congestion pricing
on commuting times?
Supporting information from the reading passage:
It will make commuting faster for everyone by taking
more cars off the road.
Supporting information from the lecture: It will
not result in faster commuting times because buses
will be too crowded.
Key point 3: How many people will benefit from
congestion pricing?
Supporting information from the reading passage:
It will benefit as many people as possible because it
only punishes people who pollute the air.
Supporting information from the lecture: It will
not benefit everyone because many people do not
have any other choice.
Conclusion:
Summary: The reading says congestion pricing .is
good, but the lecture disagrees.
~Step

The lecture and the passage both discuss congestion


pricing, a policy that charges people for driving during
bl.lsy times of the day. While the passage says that
congestion pricing is helpful, the lecturer argues that
it is not a good idea.
Aocording to the passage, the policy encourages people
to use public transportation instead of cars, which will
lead to less oollution. The lecturer points out that in
some cities, buses don't go to all neighborhoods. As a
result, many people will not be able to get around. Next,
the passage claims that it makes travel faster because
it eliminates traffic. However, the lecturer predicts that
commute times will increase. Lastly, the passage
states that congestion pricing is fair and beneficial to
everybody. The lecturer shows that the policy puts
additional financial stress on drivers who do not have
a lot of extra money.
There are many issues surrounding congestion
pricing. The lecture and passage both present some
of the issues.

Answer Key

777

02 History
~Step

Main idea: The Copper Scroll did not describe a real


treasure.
Key point 1: The Copper Scroll was likely a story
because there were many similar stories in Israel.
Key point 2: The amount of treasure described in the
scroll seems too larae to be real since it would be
worth one billion dollars todaY,
Key point 3: The treasure has never been found
because the descriptions in the scroll are vague.
~Step

Main idea: There is evidence to suggest that the


treasure in the Copper Scroll is rfil![.
Key point 1: Since the Copper Scroll uses technical
language, it is probably real.
Key point 2: The amount of treasure described in the
scroll might not be as large as some people think
because money was worth different amounts in different
places.
Key point 3: It may be possible to find the treasure
because another scroll might exist.
~Step

Introduction:
Main topic: Is the treasure described in the Copper
Scroll is real or imaginary?
Main idea of the reading passage: The Copper
Scroll did not describe a real treasure.
Main idea of the lecture: It is possible that the
treasure is real.
Body:
Key point 1: Was the scroll a story?
Supporting information from the reading passage:
The scroll was probably just a story used to entertain
people because there were many similar stories in.
Israel.
Supporting information from the lecture: The
technical language used means it probably was not
just a story.

778

Answer Key

Key point 2: Is the size of the treasure false?


Supporting information from the reading passage:
The scroll describes a treasure that is too large to be
real since it would be worth one billion dollars today.
Supporting information from the lecture: The
treasure might not be as large as some people think
because money was worth different amounts in different
places.
Key point 3: Where is the treasure?
Supporting information from the reading passage:
The treasure has never been found because the
descriptions in the scroll are vague.
Supporting information from the lecture: It might
be possible to find the treasure by using another
scroll.
Conclusion:
Summary: The reading says that the treasure does
not exist, but the speaker thinks that it might.
~Step 4
Both the lecture and the passage discuss the Copper
Scroll. While the passage says that the tieas11fe
described in the scroll does not actually exist, the
lecturer argues the treasure is real.
According to the passage, the scroll is an old story
that was passed down over generations. The lecturer
corrects this claim by explaining that the scroll used
vocabulary that is not used in folk tales. The passage
also argues that the treasure is much too large to be
real. In response, the lecturer reasons that the amount
in the scroll may be smaller than it seems; therefore,
it is probably real. Last, the passage claims that the
treasure is impossible to find because the directions.
on the scroll are not clear. The lecturer says that a
different map mentioned in the copper scroll probably
has better directions to the treasure.
The mystery of the copper scroll has inspired many
arguments. Some people still think it is authentic, while
others think it is only a story.

03 Environmental Science

>Step 1
Main idea: Biofuels cannot replace fossil fuels.
Key point 1: Biofuels are not more environmentally
friendly than fossil fuels because they also pollute the
environment.
Key point 2: Biofuels do not have an unlimited supply
because most agricultural land is already being used.
Key point 3: Biofuels cannot meet the world's demand
for fuel because we need too much fuel.

>Step 2
Main idea: Ethanol is not a substance that we can
use to replace gasoline.
Key point 1: Ethanol is not cleaner than gasoline; in
fact, it produces more pollution.
Key point 2: Ethanol requires more laoQ than we
have available because there is not enough for both
food and fuel crops.
Key'lpoint 3: Ethanol cannot meet the demand for
fuel In the world because we do not have enough land.
,.)_;)'.'

>Step 3
Introduction:
Main topic: Can biofuels can be used as a substitute
for fossil fuels?
Main idea of the reading passage: Biofuels cannot
be used as a substitute for fossil fuels.
Main idea.of the lecture: Ethanol is not something
we can w>e to replace gasoline.

Body:
Key point 1: What is the environmental impact of
biofuels?
Supporting information from the reading passage:
Biofuels are not better for the environment than fossil
fuels because they also pollute the environment.
Supporting information from the lecture: Ethanol
actually pollutes the environment more than fossil
fuels.
Key point 2: Where does the supply come from?
Supporting infonnation from the reading passage:
Biofuels do not have an unlimited supply because most
agriculturat land is already used.

Supporting information from the lecture: Ethanol


requires too much land to make because th ere is not
enough land for both food and fuel crops.
Key point 3: What is the demand for biofuels?
Supporting information from the reading passage:
Biofuels cannot meet the world's demand for fuel
because we need too much fuel.
Supporting information from ttie lecture: Ethanol
cannot meet the demand for fuel in the world because
we do not have enough land.
Conclusion:
Summary: The reading says that biofuels cannot
replace fossil fuels, and the lecture support it with the
example of ethanol.
~Step

4
Both the lecture and the passage discuss biofuels as a
possible alternative to fossil fuels. The passage states
that they cannot replace current fuel sources. The
lecturer supports the ~ with a discussion about
ethanol, a biofuel made from corn.
To begin, the passage says that biofuels are no better
for the environment than fossil fuels. The lecturer
proves the claim by showing that ethanol creates more
pollution than gasoline. Then the passage states that
biofuels place strain on land resources. The lecturer
agrees, adding that the production of ethanol makes
corn more expensive. Lastly, the passage says that
biofuels cannot meet the world demand for. fuel. Again,
the lecturer agrees. He says that there is no way that
enough ethanol could be produced to satisfy the
needs of the world.
Both sources discuss biofuels. They give three
reasons why biofuels cannot replace fossil fuels.

I
'

04 Environmental Science

} Step I
Main idea: Corntainers are better for the environment
for three main reasons.
Key point 1: Corntainers can be composted, which
returns them to the Earth as fertilizer.
Key point 2: Corntainers take up less space in landfills
L'ecause they will biodegrade.
Key point 3: Using corntainers is the socially
resoonsible thing to do because it is better for the
environment.
Answer Key

779 - -

'-~


~Step

Main idea: Corntainers do not solve environmental


concerns.
Key point 1: Corntainers require special facilities in
order to break down.
Key point 2: Many corntainers end up in landfills
because people do not understand how to recycle
them.
Key point 3: There is nothing socially responsible
about using corntainers because people need corn to
eat.
~Step

Introduction:
Main topic; Are corntainers good for the environment?
Main idea of the reading passage: Corntainers are
better for the environment for three main reasons.
Main idea of the lecture: Corntainers do not solve
environmental concerns.

780

~Step

Both the lecture and the passage discuss PLA


containers (corntainers). Though the passage presents
corntainers as an eco-friendly alternative to other plastic
packages, the lecturer says they do not benefit the
environment.
According to the passage, corntainers are good
because they break down easily and turn into fertilizer.
The lecturer challenges the claim. She shows that the
composting process for corntainers is long and
complicated. The passage also states that corntainers
are beneficial because they reduce the amount of waste
that ends up in landfills. However, the lecturer disagrees,
saying that many corntainers end up in landfills, anywey.
Finally, .the passage says that using corntainers is
socially responsible. They help the environment, and
thus, !2De!i! everyone. In contrast, the lecturer argues
it is actually irresponsible to produce plastic with corn
that could be used to feed the hungry.
Many people are interested in doing what is best for
the environment. The passage claims that corntainers
are eco-friendly, but the lecturer disagrees.

Body:
Key point 1: Corntainers can be composted.
Supporting information from the reading passage:
Corntainers can be composted, which returns them
to the Earth as fertilizer.
Supporting informationfromthe lecture: Corntainers
require special facilities in order to break down.
Key point 2: Corntainers do not fill up landfills.
Supporting information from the reading passage:
Corntainers will not take up space in landfills because
they will biodegrade.
Supporting information from the lecture: Many
corntainers will just end up in landfills anyway because
people don't understand how to recycle them.
Key point 3: Using corntainers is socially responsible.
Supporting information from the reading passage:
Using corntainers is socially responsible because it is
better for tl1e environment.
Supporting information from the lecture: Using
corntainers is not socially responsible because people
need corn to eat.

A. 2
B. 1
C. The ongoing process of global warming can be
proved by three different points.

Conclusion:

:>Step 3

Summary: The reading says that corntainers are


good, while the lecture says that they do not help the
environment.

Main idea: Global warming is not taking place on


Earth ..

Answer

Key

01 Climatology
~Step 1
II/lain idea: Climatologists believe that global warming
is occurring for three reasons.
Key point 1: There is more carbon dioxide in the
atmosphere than ever before, which raises the Earth's
temperature.
Key point 2: The Earth's ice caps are melting raoidly
as the Earth gets hotter.
Key point 3: The average global temperature has
been steadily increasing.
~Step

1:
.

'

'..1

Key point 1: Carbon dioxide levels have not increased


dramatically in the past quarter century.
Key point 2: There are many different factors that
contribute to the melting of ice caps.
Key point 3: Reported average global temperatures
are not accurate because the facilities do not include all
temperatures in the average.
Step 4

02 Psychology
2. a

1. b

the lecturer points out that over the past quarter


century, the levels have only increased by two
percent.
2. The Hadley Center and NASA record average
temperatures, but their figures are inaccurate
despite their use of the most advanced technology.

3. a

4.b

Step 5

A.
1. While the reading says that global warming is
currently taking place, the lecture says that
temperatures are not changing.
2. The reading states that carbon dioxide levels
have . worsened global warming, though the
lecturer disagrees with this view.
3. The passage suggests that melting ice caps
iodic~te rising temperatures. The lecture, however,
says that ice cap loss is due to many other factors.
4. The author of the passage says two facilities have
recorded temperature increases over the past
hundred years, Jhe lecturer points out that the
records are inaccurate.

Step 1
Main idea: Adult interference in children's playing is
problematic.
Key point 1: Adult interference can hurt children's
creativitv.
Key point 2: Children need to develop positive Plll[
role models, which is done through playtime.
Key point 3: Children need to learn to stand up for
themselves.
Step 2

A. 2
B.1
C. There are three reasons why adult interference
causes problems.
~Step

Main idea: Adults need to monitor children's playtime.

B.
1. The lecture says that temperatures are not changing,
while the reading says global warming is currently
taking place.
2. The lecturer disagrees with the view presented in
the reading, which claims that carbon dioxide
levels have worsened global warming.
3. It is suggested in the passage that melting ice
caps indicate global warming; however, the
lecture says other factors might be responsible.
4. The lecturer shows that average global temperature
records are inaccurate, though the passage says
that two facilities provide proof that average
temperatures have increased.
~Step

Key point 1: Adults need to monitor playtime to


__make sure it is positive becaus.e many children will get . .
into trouble otherwise.
Key point 2: Children need adults, not peers, as IDll2
models because children learn bad behavior from
other children.
Key point 3: Adults need to monitor children to protect
them.
~ Step 4

1. b

2. a

3.a

4.b

'Step 5
A.
1. states/troublesome
3. good/require

2. harm/watch
4. defend/parents

1. Experts say that carbon dioxide levels have


increased by thirty-one percent in recent years, but

Answer Key

781 - : 1

;:.-.

B.
1. The author states that adult interference in children's
playtime is troublesome, while the lecture says
that adults need to interfere in playtime.
2. While the author says that creativity can be
harmed by adult interference, the speaker thinks
that playtime should be monitored by adults to
make sure it is positive.
3. Children need to develop good peer role models
according to the author. The speaker, though,
says that children require adults, not peers, as
role models.
4. The author states that children need to learn to
defend themselves. The lecturer states that parents
need to monitor children to protect them.
~Step

1. In order to prevent stunting creativity through


interference, adults should guide children toward
positive activities that develop creativity.
2. Children should learn how to deal with difficult
situations on their own; however, at times a
supportive parent is beneficial.

03 Biology
~Step 1

Main idea: The disappearance of the dodo is a result


of over hunting.
Key point 1: It became extinct because it was
flightless.
Key point 2: Its unusual friendliness also led to its
extinction.
Key point 3: Researchers found evidence of humans
killing dodos in caves.
~Step

A. 2
B. 1
C. It is clear that the dodo bird went extinct because
humans hunted it too much.
~Step

Main idea: It was not over hunting but other factors


that led to the dodo's disappearance.
Key point 1: Humans did not even like eating dodo
meat because it tasted very bad.
782

Answer Key

Key point 2: Humans were not the main hunters;


other animals were.
Key point 3: There is little evidence of humans killing
dodos, but plenty of evidence of predators killing
dodos.
~Step

2.a

1. a
~Step

3.a

4.b

A.
1. consequence/reasons 2. since/enjoy
3. strange/key
4. proof/scant

B.
1. The author thinks that over hunting caused the
disappearance of the dodo. However, the speaker
says that it was other factors, not over hunting,
that led to its disappearance.
2. The author says the dodo became extinct because
. it was flightless, while the speaker says that
humans did not even like dodo meat.
3. The reading says that the dodo's strange friendliness
also led to its extinction, though the lecture says
that humans were not the key hunters; other
animals were.
4. The reading says that evidence of humans killing
dodos was found in caves by researchers, but the
lecture says that there is scant evidence of
humans killing dodos.

~Step

1. Not knowing any better, the dodo fearlessly


approached all new creatures, including animals
that it might be vulnerable to.
2. There is little archeological evidence that humans
hunted dodos, though one cave in Europe was
found to have dodo bones.

04 Environmental Science
~

Step 1

Main idea: Scientists discovered that DDT did more


harm than good.
Key point 1: DDT is a hazard to the environment
because it can poison and .kill wildlife.

Key point 2: DDT can endanger the health of humans


by causing a number of diseases.

"

Key point 3: DDT has failed to combat disease-canyiog


mosquitoes because insects became resistant to DDT.
Step 2

A. 2
B.1
C. Scientists eventually realized that DDT was not as
good as they thought.

Step 6

1. Diseases that start with insects can kill entire


populations, although in the case of the bald eagle,
DDT was mostly responsible for its population
decline.
2. The United States eventually banned DDT because
it was too dangerous, though some argue that
diseases spread by mosquitoes are worse.

Step 3
Main idea: DDT is not as bad as some people
believe, and it has many benefits.
Key point 1: DDT can, in some cases, benefit plants
and animals because it kills insects that cause disease.
Key point 2: DDT can also benefit human health by
stopping diseases that kill humans.
Key point 3: DDT has effectively combated diseases
in the past, and millions of lives have been saved.

'.-:~

Step 4

.t1;"-',
-~''

I'\.

1. b

2.a

3.b

4.a

.
>Step 5

j,

1. found/benefits
3. says/help

2. danger/may
4. fight/successfully

B.

Step 1
Main idea: There are three advantages to using a
computerized voting system.
. Key point 1: They make elections sa!fil because they
eliminate the possibility of human error.
Key point 2: They are much less expensive than
paper ballot systems.
Key point 3: They reduce the number of lost votes in
elections.
~Step

A.

01 Political Science

1. According tb the reading, scientists found that


DDT did more harm than good. However, the
lecture argues that DDT has many benefits and is
not as bad as some people believe.
2. While the reading says that DDT is a danger to
the environment, the lecture says that DDT may,
in some cases, benefit the environment.
3. While the reading says that human health can be
endangered by DDT, the lecture argues that DDT
can really benefit human health.
4. The reading says that DDT has failed to fight
disease-carrying rr.osquitoes. However, according
to the lecture, DDT has combated diseases in the
past.

.Main idea: Computerized voting systems have a


number of drawbacks.
Key point 1: It is no safer than using paper ballots
because it is open to hacker attacks.
Key point 2: It is actually more expensive to buy the
equipment and continue to operate and maintain it.
Key point 3: It can lose more votes than paper
systems in the event of a mechanical failure.
~Step

1. No

3
2. No

3. No

rJ2 Resource Management


~ Step 1
Main idea: Prescribed burning prevents uncontrolled
fires from igniting or spreading.
Key point 1: Prescribed burning prevents wildfires by
reducing the amount of available fuel.

Answer Key

~-~--~-

783 - - . \

Key point 2: Prescribed burning can also stop an


existing fire frorn spreading by removing underbrush.
Key point 3: Forest rangers can control the time and
place of prescribed fires by choosing when and where
to start them.
~Step

1. No

2.No

3.No

03 Business
1

~Step

Main idea: Most people who retire early find the


experience to be less than satisfVing.
Key point 1: People who retire early often face
financial problems because they no longer get regular
paychecks.
Key point 2: People who retire early also struggle
with psychological problems because they feel their
lives Jack purpose.
Key point 3: Early retirees often face social problems
and feel excluded from events at the company.
~Step

Main idea: Early retirement can be one of the best


!lhoices a person ever makes.
Key point 1: Many retirees Jive very agreeable lifestyles
in terms of their finances.
Key point 2: Early retirement can be a very positive
psychological experience for people because they
can enjoy hobbies.
Key point 3: With regards to one's social life, early
retirement is a benefit rather than a hindrance because
it allows one to meet new people.

784 Answer Key

2. No

3.No

04 Law
1

Main idea: There are many benefits of setting up


speed cameras.
Key point 1: Speed cameras reduce the number of
traffic accidents by penalizing speeding drivers.
Key point 2: Speed cameras also reduce the number
of deaths on the road because they make accidents
less severe.
Key point 3: Speed cameras allow police officers to
focus on more important tasks.
~Step

1. Yes

~Step

Main idea: A wildfire that started in the Croatan


National.Forest is a good example of bow prescribed
burning can be beneficial.
Key point 1: The wildfire may never have started if
prescribed burning had been done in the area.
Key point 2: Prescribed burning did stop the wildfire
from spreading by reducing the fuel available.
Key point 3: The prescribed fires never posed !!
threat to communities because they were controlled.
~Step

~Step

Main idea: Traffic cameras are not as valuable as


many believe.
Key point 1: Speed cameras do not do very much to
reduce traffic accidents since speed does not always
. cause accidents.
Key point 2: Speed cameras also do not reduce the
number of deaths on the road because speeding only
makes a difference on suburban roads.
Key point 3: Speed cameras remove police officers
from duty, which is not a good thing.
~Step

1. Yes

2. Yes

3. Yes

I Vocabulary Review 1 I
1. (B)
4. (D)
7. (A)

2. (A)
5. (A)
8. (A)

3. (C)
6. (B)

9. (A)
12. (A)
15. (A)

10. (A)
13. (B)

11. (A)
14. (B)

16. ancient
18. precious

17. Archaeologists
19. inscribed 20. translated

21. (C)
24. (B)

22. (E)
25. (D)

23. (A)

that education is not really that important. Conversely,


when universities attract good faculty and pay them
adequately, it creates a better environment for students
and shows that their education is important.
In conclusion, by paying teachers more money
instead of spending so much on athletics, universities
can become better institutions that attract and retain
the bestlaculty. They will communicate to students
that their education is important and valued, which
benefits everyone.

Step 3
1. I believe that universities should pay their teachers
more instead of spending money on athletics.
2. First of all, teachers need money to pay for all the
education they needed.
3. Second, by paying their teachers well, universities
can ensure that people who start out in a teaching
career stay in that career.
4. Third, education needs to be the most important
part of a university.

fl as

Part 2

Step 4

llhe

01 Opinion
.~Step 2
I believe that universities should pay their teachers
more instead of spending money on athletics. Doing
so attracts excellent teachers and keeps them. Also,
it sends the right signal to students.
First of all, teachers need money to pav for all the
education they needed. It takes a long time and a lot
of money to complete your education so that you can
become a university teacher. After that, you do not
always get paid well, and many people who would be
excellent teachers do not enter the field. So universities
should pay teachers more so more people will
become teachers.
Second, by paying their teachers well, universities
can ensure that people who start out in a teaching
career stay in that career. A problem many universities
face is l<eeping their best teachers after hiring them.
Many people with PhDs want to teach and start out
doing so. However, they find out later they can earn more
money by working for a company or the government.
This means that such people, who are excellent teachers,
choose to leave teaching because of money.
Third, education needs to be the most important part
of a university. Universities exist to educate people.
When they spend more money on athletics, it sends
the wrong message to students. It says to students

teachers more instead of spending money on


athletics.
2. The writer supports the thesis by giving three
reasons why teachers should be paid more.
3. The writer does not include a comment from the
opposing argument.
4. The main idea of the conclusion is that universities
will be better if they pay teachers more.

only

1. The essay agrees that universities should pay

02 Experience
>Step 2

While some people might like multitasking, I prefer to


work on one task at a time. When I do that, my work
is of higher quality, I feel calmer, and I am more
productive.
First. I can pay better attention to my task when I
work on only one thing at a time. For example, I have
a computer programming class. Programming requires
me to pay careful attention to details. Because of this,
if I try to do more than one thing at a time, I am more
likely to make mistakes. By focusing on one task at a
time, my work is of the l1ighest quality possible.
Answer Key

785

Second. when I only have one thing to do at a time. I


feel calm and relaxed. That way, I enjoy my class work
more, and I am willing to take on new challenges.
When I have too many things to do at once, like writing
a program, checking email, and responding to my
classmates, it breaks my concentration, and I get
frustrated and upset with my work situation.
Third. working on one task at a time increases my
productivity. Because I can focus on just one task, I
put all my mental energy into figuring out how to get
it done most effectively. Similarly, I am not distracted
by other problems like I would be if I were multitasking.
For that reason, I am more productive and efficient
when working on only one task .
In conclusion, I prefer working on one problem
at a time rather than spreading myself thin by
mulrnasking. With my preferred approach, I produce
better quality work, feel better about doing so, and
am more productive.

Step 3
1: While some people might like multitasking, I prefer
to work on one task at a time.
2. First, I can pay better attention to my task when I
work on only one thing at a time.
3. Second, when I only have one thing to do at a
time, I feel calm and relaxed.
4. Third, working on one task at a time increases my
productivity.
~Step

1. The essay takes the side of the prompt that states


that working on one thing at a time is better.
2. The writer supports the thesis statement by giving
three reasons why working on one thing at a time
is better.
3. No, the writer does not present a comment or idea
from the opposing argument in the conclusion.
4. The main idea of the conclusion is that the writer
feels better working on one task at a time.

,,,.
l'

'

03 Opinion
>Step 2
Every year, more teachers. and professors are
encouraging students to make the most of computer

786

Answer .Key

resources. Therefore, I agree. that in twenty years,


students will do all of their schoolwork on a computer.
One reason I think this is because there is a growing
trend toward sharing information online. Many
businesses, for example, have already stopped
handing out paper memos and instead post them
online or send them via e-mail. In addition, many
universities are already setting up online discussion
forums for classes. On these forums, students can
discuss class lectures as well as post assignments.
The professor can also upload reading materials or
lecture notes for the students to study. Therefore, it is
highly likely that in twenty years, all schoolwork will be
done online.
Another reason I think all schoolwork will be done on
computers in twenty years is that society in general is
moving away from paper products. People are
getting more and more concerned about environmental
issues like pollution and waste. Already, many professors
refuse to print out paper assignments. They encourage
their students to send their work via e-mail, and they
insist on sending back corrections the same way.
8ecause using computers saves paper, more and
more people are seeing their use as good for the
environment. In twenty years, probably no one will be
using paper to do work.
In conclusion, I agree that students will do all their
work on computers twenty years from now. Perhaps
the rest of society will catch on, too, and start doing
all of its work on computers as well.
~Step

1. Therefore, I agree that in twenty years, students


will do all of their schoolwork on a computer.
2. One reason I think this is because there is a growing
trend toward sharing information online.
3. Another reason I think all schoolwork will be done
on computers in twenty years is that society in
general is moving away from paper products.
~Step

1. This essay agrees with the prompt, which says


that students will do all of their schoolwork on a
computer in twenty years.
2. The writer supports the thesis statement by
describing two reasons that students will do all of
their schoolwork on computers in twenty years.

3. The writ~r does not present a comment frorn the


other side of the argument.
4. The main idea of the conclusion is that maybe
everyone will do all of their work on computers in
. twenty years.

3. You should also learn about events in other countries


because you may one day visit them.
4. Learning about events in other countries also gives
you a more accurate understanding of the world.
~Step

04 Opinion
~Step

Every day, events happen in countries all over the


world that only directly affect a relatively small number
of people. Even though those events may nothave
irnmediati;i significance, I agree that ltis impor\antto
learn abo.utthem even ~ they do not directly affect you.
One reason is that being educated about crises in
other countries can help you deal with similar crises in
your own country. When smallpox broke out in Europe
in the 18th century, for example, doctors figured out
how to eradicate the disease by vaccinating people.
Then, when the disease broke out in other countries
--;.,'
across the '1'orld, they knew how to deal with the
crisis.
t
You should also learn abcut events in other countries
.,;., ' because you may one day visit them. It is important
' :
to know wh~t is happening in a country you travel to
"
so that you can be safe. If you know that the political
situation is unstable or there is a civil war going on,
you would be wise to postpone your visit. Being
informed of a country's current events is essential to
traveling safely within that country. .
Learning about events in other countries also gives
you a more accurate understanding of the world. If
you focus only on. events in your own country, you
may not realize, for example, that things like disease
and pover\y are all major issues in the world. It is
important to understand global issues in order to protect
yourself and to help out your fellow humans.
In conclusion, I think that it is very important to learn
about major events in other countries. Even if those
events do not affect you directly, learning about them
is of great value.

~Step

1. This essay agrees with the statement that it is


important to learn about events in other countries.
2. The writer supports the thesis statement by
providing three reasons that learning about
foreign events is important.
3. The writer does not include an opposing argument
in the conclusion.
4. The main idea of the conclusion is that learning
about events in 'other countries is very valuable.

01 Thesis Statements
~Step

Thesis statement 1:
When I was twelve, my brother encouraged me to ffi'
out for a football team, but I decided not to try out for
two reasons.
Thesis statement 2:
There are three main reasons why I believe that young
children mustbe taught to use computers.
Thesis statement 3:
When I traveled to a larger city in my country, it made
me realize how much I love my own town, and there
are three reasons why I feel this way.
Thesis statement 4:
In my opinion, the most enjoyable movies to watch
are ones that make you think, and I believe this for
two reasons.

1. Even though those events may not have immediate


significance, I agree that it is important to learn
about them even if they do not directly affect you.
2. One reason is that being educated about crises in
other countries can help you deal with similar crises
in your own country.

~Step

Prompt 1: experience
Thesis statement: I believe that the most important
person in my life is my father because he has taught
me more than anyone else.

Answer Key

-,

--~_,

_ _ _ .._ ___ ---

-- '".

787 - - ,

---~..;o~

--=='-'=

Prompt 2: opinion
Thesis. statement: I do not agree that people should
never do things that harm the environment because
sometimes doing these things is necessary.
Prompt 3: opinion
Thesis statement: In my opinion, it is better to save
your money until later, and I feel this way for three
reasons.
Prompt 4: experience
Thesis statement: I think the skill that has allowed me
to do well in school is my ability to work well with
other people.

02 Topic Sentences
~Step

Prompt 1:
( 1 ) There are three reasons why I feel that having
fun is just as important for adults as it is for children.
( 2 ) For one, I.think that adults need to be able to
relax and have fun because they work all the time.

- - 788

( 3 ) For example, my dad likes to play baseball with


me on weekends.
Prompt2:
( 1 ) I once had an experience where I was scared
because I went to a new place alone, but I overcame
my tear in two important ways.
( 2 ) Another way that I overcame my fear was by
telling myself that I could do anything that I wanted to.
( 3 ) I asked for help from a really nice woman that I
met, and she helped me figure out where I was going.

( 3 ) My friend got stuck with a really annoying


roommate in the dorms.
~Step

Prompt 1:
Thesis statement: In my opinion, products made by
hand are not as good as products
made by machines.
Reason 1: Machin.es do not make mistakes.
Reason 2: Machines are faster.
Reason 3: Machines can do things that humans
cannot.
Topic sentence: To begin, I feel that machines can
make better products because
machines do not make mistakes.
Prompt 2:
Thesis statement: I believe that you can tell a lot
about a person by the clothing he
or she wears.
Reason 1: You can tell a lot about a person's style.
Reason 2: You can learn about personality.
Reason 3: You can find out what a person likes.
Topic sentence: I also think it is possible to find out
what a person likes by looking at
what he or she wears.

Prompt3:
( 1 ) I definitely like to try new things as much as I can.
( 2 ) I like to try new things because I get bored if I do
the same thing all the time.
( 3) When I tried joining my school's diving team, I
found that I really enjoyed it.

Prompt3:
Thesis statement: I think that a trip to an aquarium
would be a valuable learning
experience.
Reason 1: It would be valuable to learn about fish.
Reason 2: It is a good idea to learn outside of the
classroom.
Reason 3: It gives students a way to learn that is
interesting.
Topic sentence: In addition, I believe that a field trip to
an aquarium would gives students
the opportunity to learn in a fun
environment.

Prompt4:
( 1 ) In my opinion, universities should not require
students to live in the dormitories for their freshman year
( 2 ) Living in an apartment is more enjoyable than
living in the dorms.

Prompt 4:
Thesis statement: I think that music is important to
people mainly because it can affect
people on three different levels.
Reason 1: The lyrics in music can make people think.

Answer Key

f.
i:,:

"1.

Reason 2: The melodies can make people feel a certain


way.
Reason 3: Some songs remind you of things.
Topic sentence: I also think music is important because
the melodies can make you feel a lot
of different emotions.

Body paragraph 3: (A), (C), (B), (0), (E}, (G), (F)


Transitions: Finally, because, Today, For example
Conclusion: (A), (B)
Transitions: In conclusion

03 Opinion
~Step

Introduction: (B), (A)


Transitions: This is because, but

01 Opinion
~Step 2
Introduction: (A), (B)
Transitions: Even though, because
Body paragraph 1: (B}, (C}, (A}
Transitions: First of all, because
Body paragraph 2: (C}, (A), (D), (B)
Tran~itions: on the other hand, Second, also

Bod~paragraph 3: (C}, (B}, (A)

Body paragraph 1: (A), (E}, (B}, (C), (F), (D)


Transitions: The main reason, For instance, but, on
the other hand
Body paragraph 2: (C), (A), (F), (E}, (D), (B)
Transitions: Another reason, Then, For example
Conclusion: (B), (A)
Transitions: To conclude, but

Transitions: Finally

04 Opinion
Conclusion: (B), (A)
Transitions: but, because, In conclusion

~Step 1 (possible

student response}

3. Being. the leader makes other people look up to


me.

02 Opinion
~Step

05 Opinion

Introduction: (A), (B)


Transitions: This is because

~Step

Body paragraph 1: (C), (A), (E), (B), (D)


Transitions: First of all, because, also, before

Thesis: It is better to get advice from a parent than


from a friend.
Conclusion: Parents will be able to give you better
advice than a friend will.

Body paragraph 2: (B), (E}, (A), (C), (D)


Transitions: Second, Also

iJ6 Opinion
;. Step 1 (possible student response)
Key point 3: It is interesting to learn about other
cultures.

Answer Key

---~-

__._,

789 -

I Vocabulary Review 3 I

Part 3

1.
4.
7.
10.
13.

(B)
(C)
(A)
(B)
(D)

2.
5.
8.
11.
14.

(A)
(B)
(B)
(D)
(A)

3.
6.
9.
12.
15.

(D)
(C)
(D)
(C)
(C)

16.
19.
22.
25.
28.

(B)
(A)
(D)
(D)
(C)

17.
20.
23.
26.
29.

(A)
(C)
(A)
(A)
(B)

18.
21.
24.
27.
30.

(D)
(B)
(C)
(A)
(D)

31.
34.
37.
40.

significance 32. globe


scarce
35. generate
38. access
run into
preferred

41. design
42. value
44. disadvantage 45. portray
46. (0)
49. (0)

47. (S)
50. (S)

33. crisis
36. ensure
39. abundance

43. unproductive

48. (S)

I Vocabulary Review 4 I

t---

790

1.
4.
7.
10.
13.

(D)
(B)
(C)
(B)
(C)

2. (C)
5. (C)
8. (A)
11.(D)
14. (D)

3. (A)
6. (A)
9. (C)
12. (B)
15. (B)

16.
19.
22.
25.
28.

(D)
(C)
(D)
(B)
(D)

17.
20.
23.
26.
29.

18.
21.
24.
27.
30.

31.
34.
37.
40.

preoccupied 32. removed


open
35. hectic
attitude
38. hindsight
devote

(A)
(A)
(C)
(D)
(B)

(B)
(B)
(A)
(C)
(C)

33. connected
36. desert
39. turn back

41. off 42.


44. up 45.

out
off

43. so

46. (C)
49. (B)

47. (A)
50. (D)

48. (E)

Answer Key

Chapter 7
Verb Tense
~Exercise 1

1. I believe that universities should pay their teachers


more money rather than spend so much on
athletics because teachers should be encouraged
to do a really good job of teaching. In my opinion,
people ![ like they are more valuable in their
jobs when they get lIBi.d more money. However,
many teachers are not paid enough money. This
can make them feel like their hard work ill not
valued by the university, so they might not do as
good a job. If they are paid more, they will QQ a
better job of teaching, and students will learn
more.
2. Both the reading and the lecture discuss whether
parents should interfere in children's playtime. The
author .states that parents should interfere. in
children's playtime because children have !Q.larn
to stand up for themselves and need positive
adult role models. The lecture, on the other hand,
disagrees by stating that parents should not
interfere when children Q@y.
~Exercise

1. To (1) begin, the author of the passage states that


congestion pricing can cause more people to use
public transportation. However, the speaker in the
lecture (2) disagrees, stating that congestion pricing
will not necessarily encourage more people to use
public transportation. Next, the author says that
(3) implementing congestion pricing will reduce
commuting times for everyone. On the other hand,
the speaker suggests that congesting pricing
might (4) !llilke commuting times longer for a lot
of people. Last, the author (5) states that congestion
pricing will benefit everyone, but the lecture states
that not everyone will benefit.
2. Both the reading and the lecture (1) discuss
whether corntainers are beneficial. According to
the lecture, corntainers (2) are much better for the
environment because they can be (3) composted,
they will not take up space in landfills, and they are

I
more socially responsible than regular plastics.
However, the speaker in the lecture disagrees. The
speaker states that corntainers do not (4) solve
these environmental concerns. The author (5) refutes
the claims of the reading in three ways.

Modal Verbs
~Exercise

1. To many adults who work very hard, early retirement


2.

3.
4.

5.
6.

.,,,.

,~, ~

~\

'"'

'''

\;

"'I

7.
8.

mavJcan seem appealing.


Many adults rr:mi find that early retirement is not
a satisfying experience.
Many people who retire early will/cao/mav face
financial problems in the future.
Early retirees could/may/can find themselves in
debt or face a lower standard of living.
People who retire early can/may also struggle
with psychological problems.
Early re\irees mav/can feel that their lives no longer
have a clear purpose.
Early retirees can/may often face social problems.
They could/may/can have trouble relating to the
people that they used to work with.

at a time. That way, my work was (3) finished in


just a short amount of time. However, if I had (4)
blill working on only one homework assignment, I would not have (5) 1!Jillfili in all of my work
by the time I had to have it finished.

2. In my opinion, a person's problems can best be


(1) solved if he or she goes to a parent for advice.
My parents have been (2) giving me good advice
for my entire life. I think that my friends are really
nice people, but they do not always give me great
advice. My parents' advice, though, is always
helpful and (3) motivating. In addition, my parents
have never (4) told other people about my
problems, while one of my friends did. Last, since
my parents have always (5) OOeJ1 there to help me
with my problems, I know they are a reliable source
of help.
Exercise 2
1. Supporters have been praising the benefits of

.l

2.
3.
4.

Exercise 2
1. Although I think that the amount that we use
computers (1) may increase in the next twenty

5.

years, I do not agree with the statement that


students (2) will do all of their schoolwork on
computers in twenty years. In my opinion, computers
(3) must become much more inexpensive before
that idea (4) would be possible. There are three
reasons why I feel that we (5) should not believe
that students will do all schoolwork on computers
in twenty years.

6.
7.
8.

9.
10.

Present Participle vs. Past Participle


~Exercise 1

biofuels for many years.


However, many of the benefits of biofuels are
exaggerated.
Many of the problems associated with biofuels
cannot be solved.
Even more pollution is generated by biofuels than
by fossil fuels.
Some scientists are developing new ways to make
biofuels.
Scientists have suggested that biofuels do not
produce enough energy.
Studies have found that we do not have enough
land to use only biofuels.
Most of the world's agricultural land must be
used for growing food.
Hydrogen power has been discussed as an
alternative to biofuels.
Hydrogen fuel may be used in the future, but it is
not yet available.

1. I believe that I am able to work quite a bit faster


when I am (1) multitasking. For example, when I
was (2) working on my homework last weekend,

I made sure to work on a few different assignments

Answer Key

791 - -

Chapter 8
Noun Clauses
~

Exercise 1

_Q_ 1. It is important that children begin to use


_Q_ 2.

_JQ_ 3.
_Q_ 4.

_JQ_ 5.
_JQ_ 6.
_JQ_ 7.

computers for their schoolwork.


I believe that children will soon use computers
for all of their schoolwork.
The fact that children will do all their work
on computers are obvious to me.
I do not see how computers will be able to
replace a pen and paper.
Many people believe that evervone should
learns to use a computer.
I explained how computers have help me
do my schoolwork.
Some people disagree about which ways
do computers help people.

I> Exercise 2
1.

2.
3.
4.

5.

It is suggested by the speaker that congestion


pricing is not as beneficial as it seems.
It is true that congestion pricing does not encourage
people to use public transportation.
What experts argue is congestion pricing also
punishes the people who are forced to drive.
The theorv is congestion pricing will actually increase
commute times.
The fact that more people use the bus may
decrease commute times.

Subordinating Conjunctions
~

Eicercise 1

1. The Copper Scroll was discovered in a cave in


1952.
2. Since this discovery, scientists have analyzed and
translated the scroll.
3. Now, scientists are trying to figure out if the treasure
described in the scroll is real.
4. After we find the Silver Scroll, we may be able to
find the treasure.
5. Even though the scroll gave detailed directions,
the treasure could not be found .

...__ 792

Answer Key

6. We will not be able to say that the treasure is real

unless we find the treasure.


7. The treasure might exist, although it would be
much smaller than some believe.
8. We know that the treasure is not just a story
because it uses very technical language.
~

Exercise 2

1. It is better to get advice from parents because


they will always give good advice.
2. I always go to my parents when I need advice
about a very important issue.
3. : My parents will give me advice about what is best
for me, whereas my friends might give me advice
that is not very good for me.
4. Although many people do not think friends give
good advice, I have always received good advice
from my friends.
5. Parents will give you better advice, even if you
might think your friends know you better.
6. Parents will be able to help you through bad
experiences since they have gone through many
of those same experiences.
7. Your friends might not always be there for you,
while your parents will always support you.

Parallel Structure
~

Exercise 1

NP 1. To carefully plan is good for some people,


but I prefer living without a strict plan.
_L 2. I feel organized and relaxed when I have a
strict plan.
NP 3. LJving without a plan is confusing and stress.
_P_ 4. I like to plan when I do my homework, go
to bed, and hang out with friends.
NP 5. If I do not plan everything, I can change my
schedule and go to the movies or hanging
out with friends.
_L 6. It is good to plan so you can avoid scheduling
problems and their consequences.
NP 7. When I know I am going to be busy, I can
tell my friends and avoids a scheduling
problem.

NP 8. When I do not plan everything, I am free to


do new things and I did not have to cancel
'plans.
___ 9. I find that carefully planning everything makes

you feel too busy, stressed, and frustrated.


___ 10.1 do not carefully plan everything, yet I never
feel like I do not know what I should be
doing.

>Exercise 2
1. Planning carefully is better than to live without a
plan.

2.

3.
: l .

.' J

. ~ l
r

4.

5.

6.

7.

8.

Planning carefully is better than living without a


plan.
It is important for me to plan when I work on
homework and hanging out with friends.
It is important for me to plan when I work on
homework and hang out with friends.
Planning free time is as important as to plan time
to work on homework.
Planni~g free time is as important as planning
time to work on homework.
i--'
To plap some things is good, but I prefer living
without'a strict plan.
Planning some things is good, but I prefer living
without a strict plan.
If you do not plan your life, you can become stressed
and trustrating.
If you do not plan your life, you can become
stressed and
frustrated.
,_,_
If I do not plan everything, I am free to change my
plans and I did not have to cancel plans.
If I do not plan everything, I am free to change my
plans and I do not have to cancel plans.
When I carefully plan everything, I know what I
need to do and when I am working on it.
When I carefully plan everything, I know what I
need to do and when I ne.ed to work on it.
When I do not plan everything, I have more
opportunities to have fun and getting more
done.
When I do not plan everything, I have more
opportunities to have fun and get more done.

Pra~tic.e

l;est 1 ..

Sample Responses
>Task 1
Both the reading and the lecture talk about buzz
marketing. Whereas the reading discusses three
advantages of buzz marking, the lecture argues that
these so-called advantages are also the problems
with buzz marketing.
First, the reading says that buzz marketing can get
information about a product out quickly and easily by
using social networks. This is fine, the professor points
out, if it is good information, but buzz marketing also
spreads bad information quickly and easily.
Second, according to the reading, when people learn
information from their friends, like with buzz marketing,
they trust it more. However, as the professor points out,
if you get bad buzz due to problems with a product,
it is hard to change that. Buzz marketing can ruin your
reputation.
Finally, the reading says buzz marketing is an
inexpensive way to make a regular advertising
campaign better. The professor disagrees. He says
that if you have a bad buzz campaign, you will end up
spending more money to fix all of the problems that
the bad buzz started.
In conclusion, the reading points to three advantages
of buzz marl<eting~speed, trust. and effectiveness.
The professor finds flaws with each of these, thus
challenging the advantages of buzz marketing.
~Task

I do not think that university students should be


required to participate in exercise classes or sports. I
feel this way because physical education is not the
job of the university. Moreover, forcing students to
exercise does not respect their independence.
First, it is not the job of universities to make people
exercise. The job of universities is to educate people.
Students go to university to develop their intellect and
to prepare themselves for their future careers. Playing
sports and exercising is something that students can
do outside of their studies if they want. Universities
should focus their resources and energies on educating
young people. not on secondary missions like getting
them to exercise.
Second, requiring university students to exercise
do8s not respect their independence. Mandatory

Answer Key 793 ~

, -)_!

--

--------

physical education is fine when you are in high school.


During those years of your life, you are not an adult,
and you are not expected to make decisions or be as
independent as an adult is. However, when you go to
university, you need to learn to be more independent
and act like an adult. You do not need the university
to dictate to you when to exercise. That is something
that university students can and should decide on
their own. Some people find that they do not have
time to exercise all the time, so they exercise when
they can fit it in their schedule. Making exercis~
mandatory takes away students, freedom to choose
when they can exercise and even if they want to exercise
at all.
In conclusion, universities need to do their jobs, which
is to educate people, not to get them to exercise.
They need to respect students' rights to be
independent adults. For these reasons, universities
should not require students to participate in exercise
classes or sports.

Sample Responses
~Task

The reading and lecture discuss water fluoridation,


the practice of adding fluoride to city drinking water.
The reading supports the practice of fluoridation and
gives several reasons why tt is beneficial. The professor,
on the other hand, argues that fluoridation is not good
for society and provides several disadvantages.
The reading says that fluoridation reduces tooth
decay by helping teeth rebuild themselves. The
professor argues that several recent studies have
shown that drinking fluoridated water seems to have
very little effect on dental health.
The reading claims that fluoridation is safe since
fluoride is added to drinking water in very low doses.
The professor argues that even those small doses
can cause serious health problems, and therefore
fluoridation is not worth the risk.
'
The reading also says that fluoridation is cost. effective since it saves people trom having to pay a lot
of money to fix their teeth later in life. The professor
argues that fluoridation is not cost-effective because
it can cause health problems that are even more
costly to take care of than decaying teeth .

.- - 794 Answer Key

The reading concludes that fluoridation has many


positive outcomes, while the professor concludes
that fluoridation is too damaging to be worthwhile.

~Task

Books offer quite a lot of knowledge. In fact, it would


be impossible for one person to learn all of the
knowledge contained in books. However, I do not
agree that all knowledge can be learned from books.
First of all, many professions require skills that cannot
be learned from reading books. Surgeons, for example,
must learn how to make precise cuts. Although books
may be good reference tools, surgeons can only
perfect these skills by practicing them. It is just not
possible to learn the kind of careful hand skills that
surgeons need from books. Similarly, painters learn
the techniques of painting by practicing them on
canvasses rather than simply reading about them in
books.
Secondly, books can only teach you about past
knowledge. Scholars, particularly scientists, are
learning more about the world every day. They do this
by experimentation and by observation. They discover
things that have never been written about in books
and they discover things that contradict what ha~
already been written. Books, therefore, offer on~ a
piece of the knowledge that can be found in the
world.
Thirdly, books cannot teach us how to interact with
people. Humans are social beings. We need to be
able to communicate and work together in order to
function as a society. Reading books does not teach
us how to do those things. We must learn them
through experience: by watching our parents, by
building relationships, and by simply living our
everyday lives.
In conclusion, 1think it is impossible to learn everything
we need to know as humans from the books that we
read. Books may be valuable to knowledge, but they
do not tell us everything about our world.

You might also like